You are on page 1of 251

FINALS CONSTITUTIONAL LAW I ACJUCO NOV 11, 2017 1

FOREIGN AFFAIRS
G.R. No. 212426 SERENO, J.:

RENE A.V. SAGUISAG, WIGBERTO E. TAÑADA, The petitions1 before this Court question the constitutionality
FRANCISCO "DODONG" NEMENZO, JR., SR. MARY of the Enhanced Defense Cooperation Agreement (EDCA)
JOHN MANANZAN, PACIFICO A. AGABIN, ESTEBAN between the Republic of the Philippines and the United States
"STEVE" SALONGA, H. HARRY L. ROQUE, JR., EVALYN of America (U.S.). Petitioners allege that respondents
G. URSUA, EDRE U. OLALIA, DR. CAROL PAGADUAN- committed grave abuse of discretion amounting to lack or
ARAULLO, DR. ROLAND SIMBULAN, AND TEDDY excess of jurisdiction when they entered into EDCA with the
CASIÑO, Petitioners, U.S.,2 claiming that the instrument violated multiple
vs. constitutional provisions.3 In reply, respondents argue that
EXECUTIVE SECRETARY PAQUITO N. OCHOA, JR., petitioners lack standing to bring the suit. To support the
DEPARTMENT OF NATIONAL DEFENSE SECRETARY legality of their actions, respondents invoke the 1987
VOLTAIRE GAZMIN, DEPARTMENT OF FOREIGN Constitution, treaties, and judicial precedents.4
AFFAIRS SECRETARY ALBERT DEL ROSARIO, JR.,
DEPARTMENT OF BUDGET AND MANAGEMENT A proper analysis of the issues requires this Court to lay down
SECRETARY FLORENCIO ABAD, AND ARMED FORCES at the outset the basic parameters of the constitutional powers
OF THE PHILIPPINES CHIEF OF STAFF GENERAL and roles of the President and the Senate in respect of the
EMMANUEL T. BAUTISTA, Respondents. above issues. A more detailed discussion of these powers and
roles will be made in the latter portions.
x-----------------------x
I. BROAD CONSTITUTIONAL CONTEXT OF THE POWERS
G.R. No. 212444 OF THE PRESIDENT: DEFENSE, FOREIGN RELATIONS,
AND EDCA
BAGONG ALYANSANG MAKABAYAN (BAYAN),
REPRESENTED BY ITS SECRETARY GENERAL RENATO A. The Prime Duty of the State and the Consolidation of
M. REYES, JR., BAYAN MUNA PARTY-LIST Executive Power in the President
REPRESENTATIVES NERI J. COLMENARES AND
CARLOS ZARATE, GABRIELA WOMEN'S PARTY-LIST Mataimtim kong pinanunumpaan (o pinatotohanan) na
REPRESENTATIVES LUZ ILAGAN AND EMERENCIANA tutuparin ko nang buong katapatan at sigasig ang aking mga
DE JESUS, ACT TEACHERS PARTY-LIST tungkulin bilang Pangulo (o Pangalawang Pangulo o
REPRESENTATIVE ANTONIO L. TINIO, ANAKPAWIS Nanunungkulang Pangulo) ng Pilipinas, pangangalagaan at
PARTY-LIST REPRESENTATIVE FERNANDO HICAP, ipagtatanggol ang kanyang Konstitusyon, ipatutupad ang mga
KABATAAN PARTY-LIST REPRESENTATIVE TERRY batas nito, magiging makatarungan sa bawat tao, at itatalaga
RIDON, MAKABAYANG KOALISYON NG MAMAMAYAN ang aking sarili sa paglilingkod sa Bansa. Kasihan nawa aka
(MAKABAYAN), REPRESENTED BY SATURNINO ng Diyos.
OCAMPO AND LIZA MAZA, BIENVENIDO LUMBERA, JOEL
C. LAMANGAN, RAFAEL MARIANO, SALVADOR FRANCE, - Panunumpa sa Katungkulan ng Pangulo ng Pilipinas ayon
ROGELIO M. SOLUTA, AND CLEMENTE G. BAUTISTA, sa Saligang Batas5
Petitioners,
vs. The 1987 Constitution has "vested the executive power in the
DEPARTMENT OF NATIONAL DEFENSE (DND) President of the Republic of the Philippines."6 While the
SECRETARY VOLTAIRE GAZMIN, DEPARTMENT OF vastness of the executive power that has been consolidated
FOREIGN AFFAIRS SECRETARY ALBERT DEL ROSARIO, in the person of the President cannot be expressed fully in one
EXECUTIVE SECRETARY PAQUITO N. OCHOA, JR., provision, the Constitution has stated the prime duty of the
ARMED FORCES OF THE PHILIPPINES CHIEF OF STAFF government, of which the President is the head:
GENERAL EMMANUEL T. BAUTISTA, DEFENSE
UNDERSECRETARY PIO LORENZO BATINO, The prime duty of the Government is to serve and protect the
AMBASSADOR LOURDES YPARRAGUIRRE, people. The Government may call upon the people to defend
AMBASSADOR J. EDUARDO MALAYA, DEPARTMENT OF the State and, in the fulfillment thereof, all citizens may be
JUSTICE UNDERSECRETARY FRANCISCO BARAAN III, required, under conditions provided by law, to render personal
AND DND ASSISTANT SECRETARY FOR STRATEGIC military or civil service.7 (Emphases supplied)
ASSESSMENTS RAYMUND JOSE QUILOP AS
CHAIRPERSON AND MEMBERS, RESPECTIVELY, OF THE B. The duty to protect the territory and the citizens of the
NEGOTIATING PANEL FOR THE PHILIPPINES ON EDCA, Philippines, the power to call upon the people to defend the
Respondents. State, and the President as Commander-in-Chief

x-----------------------x The duty to protect the State and its people must be carried
out earnestly and effectively throughout the whole territory of
KILUSANG MAYO UNO, REPRESENTED BY ITS the Philippines in accordance with the constitutional provision
CHAIRPERSON, ELMER LABOG, CONFEDERATION FOR on national territory. Hence, the President of the Philippines,
UNITY, RECOGNITION AND ADVANCEMENT OF as the sole repository of executive power, is the guardian of
GOVERNMENT EMPLOYEES (COURAGE), the Philippine archipelago, including all the islands and waters
REPRESENTED BY ITS NATIONAL PRESIDENT embraced therein and all other territories over which it has
FERDINAND GAITE, NATIONAL FEDERATION OF LABOR sovereignty or jurisdiction. These territories consist of its
UNIONS-KILUSANG MAYO UNO, REPRESENTED BY ITS terrestrial, fluvial, and aerial domains; including its territorial
NATIONAL PRESIDENT JOSELITO USTAREZ, NENITA sea, the seabed, the subsoil, the insular shelves, and other
GONZAGA, VIOLETA ESPIRITU, VIRGINIA FLORES, AND submarine areas; and the waters around, between, and
ARMANDO TEODORO, JR., Petitioners-in-Intervention, connecting the islands of the archipelago, regardless of their
RENE A.Q. SAGUISAG, JR., Petitioner-in-Intervention. breadth and dimensions.8

DECISION
FINALS CONSTITUTIONAL LAW I ACJUCO NOV 11, 2017 2

To carry out this important duty, the President is equipped with prior to the Constitution and on the presence of foreign military
authority over the Armed Forces of the Philippines (AFP),9 troops, bases, or facilities.
which is the protector of the people and the state. The AFP's
role is to secure the sovereignty of the State and the integrity D. The relationship between the two major presidential
of the national territory.10 In addition, the Executive is functions and the role of the Senate
constitutionally empowered to maintain peace and order;
protect life, liberty, and property; and promote the general Clearly, the power to defend the State and to act as its
welfare.11 representative in the international sphere inheres in the
person of the President. This power, however, does not
In recognition of these powers, Congress has specified that crystallize into absolute discretion to craft whatever instrument
the President must oversee, ensure, and reinforce our the Chief Executive so desires. As previously mentioned, the
defensive capabilities against external and internal threats12 Senate has a role in ensuring that treaties or international
and, in the same vein, ensure that the country is adequately agreements the President enters into, as contemplated in
prepared for all national and local emergencies arising from Section 21 of Article VII of the Constitution, obtain the
natural and man-made disasters.13 approval of two-thirds of its members.
To be sure, this power is limited by the Constitution itself. To
illustrate, the President may call out the AFP to prevent or Previously, treaties under the 1973 Constitution required
suppress instances of lawless violence, invasion or ratification by a majority of the Batasang Pambansa,19 except
rebellion,14 but not suspend the privilege of the writ of habeas in instances wherein the President "may enter into
corpus for a period exceeding 60 days, or place the international treaties or agreements as the national welfare
Philippines or any part thereof under martial law exceeding and interest may require."20 This left a large margin of
that same span. In the exercise of these powers, the President discretion that the President could use to bypass the
is also duty-bound to submit a report to Congress, in person Legislature altogether. This was a departure from the 1935
or in writing, within 48 hours from the proclamation of martial Constitution, which explicitly gave the President the power to
law or the suspension of the privilege of the writ of habeas enter into treaties only with the concurrence of two-thirds of all
corpus; and Congress may in turn revoke the proclamation or the Members of the Senate.21 The 1987 Constitution returned
suspension. The same provision provides for the Supreme the Senate's power22 and, with it, the legislative's traditional
Court's review of the factual basis for the proclamation or role in foreign affairs.23
suspension, as well as the promulgation of the decision within
30 days from filing. The responsibility of the President when it comes to treaties
and international agreements under the present Constitution
C. The power and duty to conduct foreign relations is therefore shared with the Senate. This shared role,
petitioners claim, is bypassed by EDCA.
The President also carries the mandate of being the sole
organ in the conduct of foreign relations.15 Since every state II. HISTORICAL ANTECEDENTS OF EDCA
has the capacity to interact with and engage in relations with
other sovereign states,16 it is but logical that every state must A. U.S. takeover of Spanish colonization and its military
vest in an agent the authority to represent its interests to those bases, and the transition to Philippine independence
other sovereign states.
The presence of the U.S. military forces in the country can be
The conduct of foreign relations is full of complexities and traced to their pivotal victory in the 1898 Battle of Manila Bay
consequences, sometimes with life and death significance to during the Spanish-American War.24 Spain relinquished its
the nation especially in times of war. It can only be entrusted sovereignty over the Philippine Islands in favor of the U.S.
to that department of government which can act on the basis upon its formal surrender a few months later.25 By 1899, the
of the best available information and can decide with Americans had consolidated a military administration in the
decisiveness. x x x It is also the President who possesses the archipelago.26
most comprehensive and the most confidential information
about foreign countries for our diplomatic and consular When it became clear that the American forces intended to
officials regularly brief him on meaningful events all over the impose colonial control over the Philippine Islands, General
world. He has also unlimited access to ultra-sensitive military Emilio Aguinaldo immediately led the Filipinos into an all-out
intelligence data. In fine, the presidential role in foreign affairs war against the U.S.27 The Filipinos were ultimately defeated
is dominant and the President is traditionally accorded a wider in the Philippine-American War, which lasted until 1902 and
degree of discretion in the conduct of foreign affairs. The led to the downfall of the first Philippine Republic.28 The
regularity, nay, validity of his actions are adjudged under less Americans henceforth began to strengthen their foothold in
stringent standards, lest their judicial repudiation lead to the country.29 They took over and expanded the former
breach of an international obligation, rupture of state relations, Spanish Naval Base in Subic Bay, Zambales, and put up a
forfeiture of confidence, national embarrassment and a cavalry post called Fort Stotsenberg in Pampanga, now
plethora of other problems with equally undesirable known as Clark Air Base.30
consequences.17
When talks of the eventual independence of the Philippine
The role of the President in foreign affairs is qualified by the Islands gained ground, the U.S. manifested the desire to
Constitution in that the Chief Executive must give paramount maintain military bases and armed forces in the country.31
importance to the sovereignty of the nation, the integrity of its The U.S. Congress later enacted the Hare-Hawes-Cutting Act
territory, its interest, and the right of the sovereign Filipino of 1933, which required that the proposed constitution of an
people to self-determination.18 In specific provisions, the independent Philippines recognize the right of the U.S. to
President's power is also limited, or at least shared, as in maintain the latter's armed forces and military bases.32 The
Section 2 of Article II on the conduct of war; Sections 20 and Philippine Legislature rejected that law, as it also gave the
21 of Article VII on foreign loans, treaties, and international U.S. the power to unilaterally designate any part of Philippine
agreements; Sections 4(2) and 5(2)(a) of Article VIII on the territory as a permanent military or naval base of the U.S.
judicial review of executive acts; Sections 4 and 25 of Article within two years from complete independence.33
XVIII on treaties and international agreements entered into
FINALS CONSTITUTIONAL LAW I ACJUCO NOV 11, 2017 3

The U.S. Legislature subsequently crafted another law called Military Assistance Agreement55 with the U.S. This executive
the Tydings-McDuffie Act or the Philippine Independence Act agreement established the conditions under which U.S.
of 1934. Compared to the old Hare-Hawes-Cutting Act, the military assistance would be granted to the Philippines,56
new law provided for the surrender to the Commonwealth particularly the provision of military arms, ammunitions,
Government of "all military and other reservations" of the U.S. supplies, equipment, vessels, services, and training for the
government in the Philippines, except "naval reservations and latter's defense forces.57 An exchange of notes in 1953 made
refueling stations."34 Furthermore, the law authorized the it clear that the agreement would remain in force until
U.S. President to enter into negotiations for the adjustment terminated by any of the parties.58
and settlement of all questions relating to naval reservations
and fueling stations within two years after the Philippines To further strengthen their defense and security
would have gained independence.35 Under the Tydings- relationship,59 the Philippines and the U.S. next entered into
McDuffie Act, the U.S. President would proclaim the American the MDT in 1951. Concurred in by both the Philippine60 and
withdrawal and surrender of sovereignty over the islands 10 the U.S.61 Senates, the treaty has two main features: first, it
years after the inauguration of the new government in the allowed for mutual assistance in maintaining and developing
Philippines.36 This law eventually led to the promulgation of their individual and collective capacities to resist an armed
the 1935 Philippine Constitution. attack;62 and second, it provided for their mutual self-defense
in the event of an armed attack against the territory of either
The original plan to surrender the military bases changed.37 party.63 The treaty was premised on their recognition that an
At the height of the Second World War, the Philippine and the armed attack on either of them would equally be a threat to
U.S. Legislatures each passed resolutions authorizing their the security of the other.64
respective Presidents to negotiate the matter of retaining
military bases in the country after the planned withdrawal of C. Current legal regime on the presence of U.S. armed forces
the U.S.38 Subsequently, in 1946, the countries entered into in the country
the Treaty of General Relations, in which the U.S. relinquished
all control and sovereignty over the Philippine Islands, except In view of the impending expiration of the 1947 MBA in 1991,
the areas that would be covered by the American military the Philippines and the U.S. negotiated for a possible renewal
bases in the country.39 This treaty eventually led to the of their defense and security relationship.65 Termed as the
creation of the post-colonial legal regime on which would Treaty of Friendship, Cooperation and Security, the countries
hinge the continued presence of U.S. military forces until sought to recast their military ties by providing a new
1991: the Military Bases Agreement (MBA) of 1947, the framework for their defense cooperation and the use of
Military Assistance Agreement of 1947, and the Mutual Philippine installations.66 One of the proposed provisions
Defense Treaty (MDT) of 1951.40 included an arrangement in which U.S. forces would be
granted the use of certain installations within the Philippine
B. Former legal regime on the presence of U.S. armed forces naval base in Subic.67 On 16 September 1991, the Senate
in the territory of an independent Philippines (1946-1991) rejected the proposed treaty.68

Soon after the Philippines was granted independence, the two The consequent expiration of the 1947 MBA and the resulting
countries entered into their first military arrangement pursuant paucity of any formal agreement dealing with the treatment of
to the Treaty of General Relations - the 1947 MBA.41 The U.S. personnel in the Philippines led to the suspension in 1995
Senate concurred on the premise of "mutuality of security of large-scale joint military exercises.69 In the meantime, the
interest,"42 which provided for the presence and operation of respective governments of the two countries agreed70 to hold
23 U.S. military bases in the Philippines for 99 years or until joint exercises at a substantially reduced level.71 The military
the year 2046.43 The treaty also obliged the Philippines to arrangements between them were revived in 1999 when they
negotiate with the U.S. to allow the latter to expand the concluded the first Visiting Forces Agreement (VFA).72
existing bases or to acquire new ones as military necessity
might require.44 As a "reaffirm[ation] [of the] obligations under the MDT,"73 the
VFA has laid down the regulatory mechanism for the
A number of significant amendments to the 1947 MBA were treatment of U.S. military and civilian personnel visiting the
made.45 With respect to its duration, the parties entered into country.74 It contains provisions on the entry and departure of
the Ramos-Rusk Agreement of 1966, which reduced the term U.S. personnel; the purpose, extent, and limitations of their
of the treaty from 99 years to a total of 44 years or until activities; criminal and disciplinary jurisdiction; the waiver of
1991.46 Concerning the number of U.S. military bases in the certain claims; the importation and exportation of equipment,
country, the Bohlen-Serrano Memorandum of Agreement materials, supplies, and other pieces of property owned by the
provided for the return to the Philippines of 17 U.S. military U.S. government; and the movement of U.S. military vehicles,
bases covering a total area of 117,075 hectares.47 Twelve vessels, and aircraft into and within the country.75 The
years later, the U.S. returned Sangley Point in Cavite City Philippines and the U.S. also entered into a second
through an exchange of notes.48 Then, through the Romulo- counterpart agreement (VFA II), which in turn regulated the
Murphy Exchange of Notes of 1979, the parties agreed to the treatment of Philippine military and civilian personnel visiting
recognition of Philippine sovereignty over Clark and Subic the U.S.76 The Philippine Senate concurred in the first VFA
Bases and the reduction of the areas that could be used by on 27 May 1999.77
the U.S. military.49 The agreement also provided for the
mandatory review of the treaty every five years.50 In 1983, Beginning in January 2002, U.S. military and civilian
the parties revised the 1947 MBA through the Romualdez- personnel started arriving in Mindanao to take part in joint
Armacost Agreement.51 The revision pertained to the military exercises with their Filipino counterparts.78 Called
operational use of the military bases by the U.S. government Balikatan, these exercises involved trainings aimed at
within the context of Philippine sovereignty,52 including the simulating joint military maneuvers pursuant to the MDT.79
need for prior consultation with the Philippine government on
the former' s use of the bases for military combat operations In the same year, the Philippines and the U.S. entered into the
or the establishment of long-range missiles.53 Mutual Logistics Support Agreement to "further the
interoperability, readiness, and effectiveness of their
Pursuant to the legislative authorization granted under respective military forces"80 in accordance with the MDT, the
Republic Act No. 9,54 the President also entered into the 1947 Military Assistance Agreement of 1953, and the VFA.81 The
FINALS CONSTITUTIONAL LAW I ACJUCO NOV 11, 2017 4

new agreement outlined the basic terms, conditions, and begin to analyze the constitutionality or validity of an official
procedures for facilitating the reciprocal provision of logistics act of a coequal branch of government, however, petitioners
support, supplies, and services between the military forces of must show that they have satisfied all the essential requisites
the two countries.82 The phrase "logistics support and for judicial review.93
services" includes billeting, operations support, construction
and use of temporary structures, and storage services during Distinguished from the general notion of judicial power, the
an approved activity under the existing military power of judicial review specially refers to both the authority
arrangements.83 Already extended twice, the agreement will and the duty of this Court to determine whether a branch or
last until 2017.84 an instrumentality of government has acted beyond the scope
of the latter's constitutional powers.94 As articulated in
D. The Enhanced Defense Cooperation Agreement Section 1, Article VIII of the Constitution, the power of judicial
review involves the power to resolve cases in which the
EDCA authorizes the U.S. military forces to have access to questions concern the constitutionality or validity of any treaty,
and conduct activities within certain "Agreed Locations" in the international or executive agreement, law, presidential
country. It was not transmitted to the Senate on the decree, proclamation, order, instruction, ordinance, or
executive's understanding that to do so was no longer regulation.95 In Angara v. Electoral Commission, this Court
necessary.85 Accordingly, in June 2014, the Department of exhaustively discussed this "moderating power" as part of the
Foreign Affairs (DFA) and the U.S. Embassy exchanged system of checks and balances under the Constitution. In our
diplomatic notes confirming the completion of all necessary fundamental law, the role of the Court is to determine whether
internal requirements for the agreement to enter into force in a branch of government has adhered to the specific
the two countries.86 restrictions and limitations of the latter's power:96

According to the Philippine government, the conclusion of The separation of powers is a fundamental principle in our
EDCA was the result of intensive and comprehensive system of government. It obtains not through express
negotiations in the course of almost two years.87 After eight provision but by actual division in our Constitution. Each
rounds of negotiations, the Secretary of National Defense and department of the government has exclusive cognizance of
the U.S. Ambassador to the Philippines signed the agreement matters within its jurisdiction, and is supreme within its own
on 28 April 2014.88 President Benigno S. Aquino III ratified sphere. But it does not follow from the fact that the three
EDCA on 6 June 2014.89 The OSG clarified during the oral powers are to be kept separate and distinct that the
arguments90 that the Philippine and the U.S. governments Constitution intended them to be absolutely unrestrained and
had yet to agree formally on the specific sites of the Agreed independent of each other. The Constitution has provided for
Locations mentioned in the agreement. an elaborate system of checks and balances to secure
coordination in the workings of the various departments of the
Two petitions for certiorari were thereafter filed before us government. x x x. And the judiciary in turn, with the Supreme
assailing the constitutionality of EDCA. They primarily argue Court as the final arbiter, effectively checks the other
that it should have been in the form of a treaty concurred in by departments in the exercise of its power to determine the law,
the Senate, not an executive agreement. and hence to declare executive and legislative acts void if
violative of the Constitution.
On 10 November 2015, months after the oral arguments were
concluded and the parties ordered to file their respective xxxx
memoranda, the Senators adopted Senate Resolution No.
(SR) 105.91 The resolution expresses the "strong sense"92 of As any human production, our Constitution is of course lacking
the Senators that for EDCA to become valid and effective, it perfection and perfectibility, but as much as it was within the
must first be transmitted to the Senate for deliberation and power of our people, acting through their delegates to so
concurrence. provide, that instrument which is the expression of their
sovereignty however limited, has established a republican
III. ISSUES government intended to operate and function as a harmonious
whole, under a system of checks and balances, and subject
Petitioners mainly seek a declaration that the Executive to specific limitations and restrictions provided in the said
Department committed grave abuse of discretion in entering instrument. The Constitution sets forth in no uncertain
into EDCA in the form of an executive agreement. For this language the restrictions and limitations upon governmental
reason, we cull the issues before us: powers and agencies. If these restrictions and limitations are
transcended it would be inconceivable if the Constitution had
A. Whether the essential requisites for judicial review are not provided for a mechanism by which to direct the course of
present government along constitutional channels, for then the
distribution of powers would be mere verbiage, the bill of rights
B. Whether the President may enter into an executive mere expressions of sentiment, and the principles of good
agreement on foreign military bases, troops, or facilities government mere political apothegms. Certainly, the
limitations and restrictions embodied in our Constitution are
C. Whether the provisions under EDCA are consistent with the real as they should be in any living constitution. x x x. In our
Constitution, as well as with existing laws and treaties case, this moderating power is granted, if not expressly, by
clear implication from section 2 of article VIII of [the 1935]
IV. DISCUSSION Constitution.

A. Whether the essential requisites for judicial review have The Constitution is a definition of the powers of government.
been satisfied Who is to determine the nature, scope and extent of such
powers? The Constitution itself has provided for the
Petitioners are hailing this Court's power of judicial review in instrumentality of the judiciary as the rational way. And when
order to strike down EDCA for violating the Constitution. They the judiciary mediates to allocate constitutional boundaries, it
stress that our fundamental law is explicit in prohibiting the does not assert any superiority over the other departments; it
presence of foreign military forces in the country, except under does not in reality nullify or invalidate an act of the legislature,
a treaty concurred in by the Senate. Before this Court may but only asserts the solemn and sacred obligation assigned to
FINALS CONSTITUTIONAL LAW I ACJUCO NOV 11, 2017 5

it by the Constitution to determine conflicting claims of The OSG maintains107 that there is no actual case or
authority under the Constitution and to establish for the parties controversy that exists, since the Senators have not been
in an actual controversy the rights which that instrument deprived of the opportunity to invoke the privileges of the
secures and guarantees to them. This is in truth all that is institution they are representing. It contends that the
involved in what is termed "judicial supremacy" which properly nonparticipation of the Senators in the present petitions only
is the power of judicial review under the Constitution. x x x x. confirms that even they believe that EDCA is a binding
(Emphases supplied) executive agreement that does not require their concurrence.

The power of judicial review has since been strengthened in It must be emphasized that the Senate has already expressed
the 1987 Constitution. The scope of that power has been its position through SR 105.108 Through the Resolution, the
extended to the determination of whether in matters Senate has taken a position contrary to that of the OSG. As
traditionally considered to be within the sphere of appreciation the body tasked to participate in foreign affairs by ratifying
of another branch of government, an exercise of discretion treaties, its belief that EDCA infringes upon its constitutional
has been attended with grave abuse.97 The expansion of this role indicates that an actual controversy - albeit brought to the
power has made the political question doctrine "no longer the Court by non-Senators, exists.
insurmountable obstacle to the exercise of judicial power or
the impenetrable shield that protects executive and legislative Moreover, we cannot consider the sheer abstention of the
actions from judicial inquiry or review."98 Senators from the present proceedings as basis for finding
that there is no actual case or controversy before us. We point
This moderating power, however, must be exercised carefully out that the focus of this requirement is the ripeness for
and only if it cannot be completely avoided. We stress that our adjudication of the matter at hand, as opposed to its being
Constitution is so incisively designed that it identifies the merely conjectural or anticipatory.109 The case must involve
spheres of expertise within which the different branches of a definite and concrete issue involving real parties with
government shall function and the questions of policy that they conflicting legal rights and legal claims admitting of specific
shall resolve.99 Since the power of judicial review involves the relief through a decree conclusive in nature.110 It should not
delicate exercise of examining the validity or constitutionality equate with a mere request for an opinion or advice on what
of an act of a coequal branch of government, this Court must the law would be upon an abstract, hypothetical, or contingent
continually exercise restraint to avoid the risk of supplanting state of facts.111 As explained in Angara v. Electoral
the wisdom of the constitutionally appointed actor with that of Commission:112
its own.100
[The] power of judicial review is limited to actual cases and
Even as we are left with no recourse but to bare our power to controversies to be exercised after full opportunity of
check an act of a coequal branch of government - in this case argument by the parties, and limited further to the
the executive - we must abide by the stringent requirements constitutional question raised or the very lis mota presented.
for the exercise of that power under the Constitution. Demetria Any attempt at abstraction could only lead to dialectics and
v. Alba101 and Francisco v. House of Representatives102 cite barren legal questions and to sterile conclusions of wisdom,
the "pillars" of the limitations on the power of judicial review as justice or expediency of legislation. More than that, courts
enunciated in the concurring opinion of U.S. Supreme Court accord the presumption of constitutionality to legislative
Justice Brandeis in Ashwander v. Tennessee Valley enactments, not only because the legislature is presumed to
Authority.103 Francisco104 redressed these "pillars" under abide by the Constitution but also because the judiciary in the
the following categories: determination of actual cases and controversies must reflect
the wisdom and justice of the people as expressed through
1. That there be absolute necessity of deciding a case their representatives in the executive and legislative
departments of the government. (Emphases supplied)
2. That rules of constitutional law shall be formulated only as
required by the facts of the case We find that the matter before us involves an actual case or
controversy that is already ripe for adjudication. The Executive
3. That judgment may not be sustained on some other ground Department has already sent an official confirmation to the
U.S. Embassy that "all internal requirements of the Philippines
4. That there be actual injury sustained by the party by reason x x x have already been complied with."113 By this exchange
of the operation of the statute of diplomatic notes, the Executive Department effectively
performed the last act required under Article XII(l) of EDCA
5. That the parties are not in estoppel before the agreement entered into force. Section 25, Article
XVIII of the Constitution, is clear that the presence of foreign
6. That the Court upholds the presumption of constitutionality military forces in the country shall only be allowed by virtue of
a treaty concurred in by the Senate. Hence, the performance
(Emphases supplied) of an official act by the Executive Department that led to the
entry into force of an executive agreement was sufficient to
These are the specific safeguards laid down by the Court satisfy the actual case or controversy requirement.
when it exercises its power of judicial review.105 Guided by
these pillars, it may invoke the power only when the following 2. While petitioners Saguisag et. al., do not have legal
four stringent requirements are satisfied: (a) there is an actual standing, they nonetheless raise issues involving matters of
case or controversy; (b) petitioners possess locus standi; (c) transcendental importance.
the question of constitutionality is raised at the earliest
opportunity; and (d) the issue of constitutionality is the lis mota The question of locus standi or legal standing focuses on the
of the case.106 Of these four, the first two conditions will be determination of whether those assailing the governmental act
the focus of our discussion. have the right of appearance to bring the matter to the court
for adjudication.114 They must show that they have a
1. Petitioners have shown the presence of an actual case or personal and substantial interest in the case, such that they
controversy. have sustained or are in immediate danger of sustaining,
some direct injury as a consequence of the enforcement of the
challenged governmental act.115 Here, "interest" in the
FINALS CONSTITUTIONAL LAW I ACJUCO NOV 11, 2017 6

question involved must be material - an interest that is in issue All obligations under this Agreement are subject to the
and will be affected by the official act - as distinguished from availability of appropriated funds authorized for these
being merely incidental or general.116 Clearly, it would be purposes. (Emphases supplied)
insufficient to show that the law or any governmental act is
invalid, and that petitioners stand to suffer in some indefinite This provision means that if the implementation of EDCA
way.117 They must show that they have a particular interest would require the disbursement of public funds, the money
in bringing the suit, and that they have been or are about to must come from appropriated funds that are specifically
be denied some right or privilege to which they are lawfully authorized for this purpose. Under the agreement, before
entitled, or that they are about to be subjected to some burden there can even be a disbursement of public funds, there must
or penalty by reason of the act complained of.118 The reason first be a legislative action. Until and unless the Legislature
why those who challenge the validity of a law or an appropriates funds for EDCA, or unless petitioners can
international agreement are required to allege the existence pinpoint a specific item in the current budget that allows
of a personal stake in the outcome of the controversy is "to expenditure under the agreement, we cannot at this time rule
assure the concrete adverseness which sharpens the that there is in fact an appropriation or a disbursement of funds
presentation of issues upon which the court so largely that would justify the filing of a taxpayers' suit.
depends for illumination of difficult constitutional
questions."119 Petitioners Bayan et al. also claim129 that their co-petitioners
who are party-list representatives have the standing to
The present petitions cannot qualify as citizens', taxpayers', or challenge the act of the Executive Department, especially if it
legislators' suits; the Senate as a body has the requisite impairs the constitutional prerogatives, powers, and privileges
standing, but considering that it has not formally filed a of their office. While they admit that there is no incumbent
pleading to join the suit, as it merely conveyed to the Supreme Senator who has taken part in the present petition, they
Court its sense that EDCA needs the Senate's concurrence to nonetheless assert that they also stand to sustain a derivative
be valid, petitioners continue to suffer from lack of standing. but substantial injury as legislators. They argue that under the
Constitution, legislative power is vested in both the Senate
In assailing the constitutionality of a governmental act, and the House of Representatives; consequently, it is the
petitioners suing as citizens may dodge the requirement of entire Legislative Department that has a voice in determining
having to establish a direct and personal interest if they show whether or not the presence of foreign military should be
that the act affects a public right.120 In arguing that they have allowed. They maintain that as members of the Legislature,
legal standing, they claim121 that the case they have filed is they have the requisite personality to bring a suit, especially
a concerned citizen's suit. But aside from general statements when a constitutional issue is raised.
that the petitions involve the protection of a public right, and
that their constitutional rights as citizens would be violated, The OSG counters130 that petitioners do not have any legal
they fail to make any specific assertion of a particular public standing to file the suits concerning the lack of Senate
right that would be violated by the enforcement of EDCA. For concurrence in EDCA. Respondent emphasizes that the
their failure to do so, the present petitions cannot be power to concur in treaties and international agreements is an
considered by the Court as citizens' suits that would justify a "institutional prerogative" granted by the Constitution to the
disregard of the aforementioned requirements. Senate. Accordingly, the OSG argues that in case of an
allegation of impairment of that power, the injured party would
In claiming that they have legal standing as taxpayers, be the Senate as an institution or any of its incumbent
petitioners122 aver that the implementation of EDCA would members, as it is the Senate's constitutional function that is
result in the unlawful use of public funds. They emphasize that allegedly being violated.
Article X(1) refers to an appropriation of funds; and that the
agreement entails a waiver of the payment of taxes, fees, and The legal standing of an institution of the Legislature or of any
rentals. During the oral arguments, however, they admitted of its Members has already been recognized by this Court in
that the government had not yet appropriated or actually a number of cases.131 What is in question here is the alleged
disbursed public funds for the purpose of implementing the impairment of the constitutional duties and powers granted to,
agreement.123 The OSG, on the other hand, maintains that or the impermissible intrusion upon the domain of, the
petitioners cannot sue as taxpayers.124 Respondent explains Legislature or an institution thereof.132 In the case of suits
that EDCA is neither meant to be a tax measure, nor is it initiated by the legislators themselves, this Court has
directed at the disbursement of public funds. recognized their standing to question the validity of any official
action that they claim infringes the prerogatives, powers, and
A taxpayer's suit concerns a case in which the official act privileges vested by the Constitution in their office.133 As
complained of directly involves the illegal disbursement of aptly explained by Justice Perfecto in Mabanag v. Lopez
public funds derived from taxation.125 Here, those Vito:134
challenging the act must specifically show that they have
sufficient interest in preventing the illegal expenditure of public Being members of Congress, they are even duty bound to see
money, and that they will sustain a direct injury as a result of that the latter act within the bounds of the Constitution which,
the enforcement of the assailed act.126 Applying that principle as representatives of the people, they should uphold, unless
to this case, they must establish that EDCA involves the they are to commit a flagrant betrayal of public trust. They are
exercise by Congress of its taxing or spending powers.127 representatives of the sovereign people and it is their sacred
duty to see to it that the fundamental law embodying the will
We agree with the OSG that the petitions cannot qualify as of the sovereign people is not trampled upon. (Emphases
taxpayers' suits. We emphasize that a taxpayers' suit supplied)
contemplates a situation in which there is already an
appropriation or a disbursement of public funds.128 A reading We emphasize that in a legislators' suit, those Members of
of Article X(l) of EDCA would show that there has been neither Congress who are challenging the official act have standing
an appropriation nor an authorization of disbursement of only to the extent that the alleged violation impinges on their
funds. The cited provision reads: right to participate in the exercise of the powers of the
institution of which they are members.135 Legislators have
the standing "to maintain inviolate the prerogatives, powers,
and privileges vested by the Constitution in their office and are
FINALS CONSTITUTIONAL LAW I ACJUCO NOV 11, 2017 7

allowed to sue to question the validity of any official action, of any other party that has a more direct and specific interest
which they claim infringes their prerogatives as in raising the present questions.141
legislators."136 As legislators, they must clearly show that
there was a direct injury to their persons or the institution to An exhaustive evaluation of the memoranda of the parties,
which they belong.137 together with the oral arguments, shows that petitioners have
presented serious constitutional issues that provide ample
As correctly argued by respondent, the power to concur in a justification for the Court to set aside the rule on standing. The
treaty or an international agreement is an institutional transcendental importance of the issues presented here is
prerogative granted by the Constitution to the Senate, not to rooted in the Constitution itself. Section 25, Article XVIII
the entire Legislature. In Pimentel v. Office of the Executive thereof, cannot be any clearer: there is a much stricter
Secretary, this Court did not recognize the standing of one of mechanism required before foreign military troops, facilities,
the petitioners therein who was a member of the House of or bases may be allowed in the country. The DFA has already
Representatives. The petition in that case sought to compel confirmed to the U.S. Embassy that "all internal requirements
the transmission to the Senate for concurrence of the signed of the Philippines x x x have already been complied with."142
text of the Statute of the International Criminal Court. Since It behooves the Court in this instance to take a liberal stance
that petition invoked the power of the Senate to grant or towards the rule on standing and to determine forthwith
withhold its concurrence in a treaty entered into by the whether there was grave abuse of discretion on the part of the
Executive Department, only then incumbent Senator Pimentel Executive Department.
was allowed to assert that authority of the Senate of which he
was a member. We therefore rule that this case is a proper subject for judicial
review.
Therefore, none of the initial petitioners in the present
controversy has the standing to maintain the suits as B. Whether the President may enter into an executive
legislators. agreement on foreign military bases, troops, or facilities

Nevertheless, this Court finds that there is basis for it to review C. Whether the provisions under EDCA are consistent with the
the act of the Executive for the following reasons. Constitution, as well as with existing laws and treaties

In any case, petitioners raise issues involving matters of Issues B and C shall be discussed together infra.
transcendental importance.
1. The role of the President as the executor of the law includes
Petitioners138 argue that the Court may set aside procedural the duty to defend the State, for which purpose he may use
technicalities, as the present petition tackles issues that are of that power in the conduct of foreign relations
transcendental importance. They point out that the matter
before us is about the proper exercise of the Executive Historically, the Philippines has mirrored the division of
Department's power to enter into international agreements in powers in the U.S. government. When the Philippine
relation to that of the Senate to concur in those agreements. government was still an agency of the Congress of the U.S., it
They also assert that EDCA would cause grave injustice, as was as an agent entrusted with powers categorized as
well as irreparable violation of the Constitution and of the executive, legislative, and judicial, and divided among these
Filipino people's rights. three great branches.143 By this division, the law implied that
the divided powers cannot be exercised except by the
The OSG, on the other hand, insists139 that petitioners department given the power.144
cannot raise the mere fact that the present petitions involve
matters of transcendental importance in order to cure their This divide continued throughout the different versions of the
inability to comply with the constitutional requirement of Philippine Constitution and specifically vested the supreme
standing. Respondent bewails the overuse of "transcendental executive power in the Governor-General of the
importance" as an exception to the traditional requirements of Philippines,145 a position inherited by the President of the
constitutional litigation. It stresses that one of the purposes of Philippines when the country attained independence. One of
these requirements is to protect the Supreme Court from the principal functions of the supreme executive is the
unnecessary litigation of constitutional questions. responsibility for the faithful execution of the laws as
embodied by the oath of office.146 The oath of the President
In a number of cases,140 this Court has indeed taken a liberal prescribed by the 1987 Constitution reads thus:
stance towards the requirement of legal standing, especially
when paramount interest is involved. Indeed, when those who I do solemnly swear (or affirm) that I will faithfully and
challenge the official act are able to craft an issue of conscientiously fulfill my duties as President (or Vice-
transcendental significance to the people, the Court may President or Acting President) of the Philippines, preserve and
exercise its sound discretion and take cognizance of the suit. defend its Constitution, execute its laws, do justice to every
It may do so in spite of the inability of the petitioners to show man, and consecrate myself to the service of the Nation. So
that they have been personally injured by the operation of a help me God. (In case of affirmation, last sentence will be
law or any other government act. omitted.)147 (Emphases supplied)

While this Court has yet to thoroughly delineate the outer limits This Court has interpreted the faithful execution clause as an
of this doctrine, we emphasize that not every other case, obligation imposed on the President, and not a separate grant
however strong public interest may be, can qualify as an issue of power.148 Section 1 7, Article VII of the Constitution,
of transcendental importance. Before it can be impelled to expresses this duty in no uncertain terms and includes it in the
brush aside the essential requisites for exercising its power of provision regarding the President's power of control over the
judicial review, it must at the very least consider a number of executive department, viz:
factors: (1) the character of the funds or other assets involved
in the case; (2) the presence of a clear case of disregard of a The President shall have control of all the executive
constitutional or statutory prohibition by the public respondent departments, bureaus, and offices. He shall ensure that the
agency or instrumentality of the government; and (3) the lack laws be faithfully executed.
FINALS CONSTITUTIONAL LAW I ACJUCO NOV 11, 2017 8

The equivalent provisions in the next preceding Constitution shorn of the power to do so. The interests of the Philippines
did not explicitly require this oath from the President. In the will be best served by strict adherence to the basic principles
1973 Constitution, for instance, the provision simply gives the of constitutional government.
President control over the ministries.149 A similar language,
not in the form of the President's oath, was present in the 1935 In light of this constitutional duty, it is the President's
Constitution, particularly in the enumeration of executive prerogative to do whatever is legal and necessary for
functions.150 By 1987, executive power was codified not only Philippine defense interests. It is no coincidence that the
in the Constitution, but also in the Administrative Code:151 constitutional provision on the faithful execution clause was
followed by that on the President's commander-in-chief
SECTION 1. Power of Control. - The President shall have powers,164 which are specifically granted during
control of all the executive departments, bureaus, and offices. extraordinary events of lawless violence, invasion, or
He shall ensure that the laws be faithfully executed. rebellion. And this duty of defending the country is unceasing,
(Emphasis supplied) even in times when there is no state of lawlesss violence,
invasion, or rebellion. At such times, the President has full
Hence, the duty to faithfully execute the laws of the land is powers to ensure the faithful execution of the laws.
inherent in executive power and is intimately related to the
other executive functions. These functions include the faithful It would therefore be remiss for the President and repugnant
execution of the law in autonomous regions;152 the right to to the faithful-execution clause of the Constitution to do
prosecute crimes;153 the implementation of transportation nothing when the call of the moment requires increasing the
projects;154 the duty to ensure compliance with treaties, military's defensive capabilities, which could include forging
executive agreements and executive orders;155 the authority alliances with states that hold a common interest with the
to deport undesirable aliens;156 the conferment of national Philippines or bringing an international suit against an
awards under the President's jurisdiction;157 and the overall offending state.
administration and control of the executive department.158
The context drawn in the analysis above has been termed by
These obligations are as broad as they sound, for a President Justice Arturo D. Brion's Dissenting Opinion as the beginning
cannot function with crippled hands, but must be capable of of a "patent misconception."165 His dissent argues that this
securing the rule of law within all territories of the Philippine approach taken in analyzing the President's role as executor
Islands and be empowered to do so within constitutional limits. of the laws is preceded by the duty to preserve and defend the
Congress cannot, for instance, limit or take over the Constitution, which was allegedly overlooked.166
President's power to adopt implementing rules and
regulations for a law it has enacted.159 In arguing against the approach, however, the dissent grossly
failed to appreciate the nuances of the analysis, if read
More important, this mandate is self-executory by virtue of its holistically and in context. The concept that the President
being inherently executive in nature.160 As Justice Antonio T. cannot function with crippled hands and therefore can
Carpio previously wrote,161 disregard the need for Senate concurrence in treaties167 was
never expressed or implied. Rather, the appropriate reading
[i]f the rules are issued by the President in implementation or of the preceding analysis shows that the point being
execution of self-executory constitutional powers vested in the elucidated is the reality that the President's duty to execute
President, the rule-making power of the President is not a the laws and protect the Philippines is inextricably interwoven
delegated legislative power. The most important self- with his foreign affairs powers, such that he must resolve
executory constitutional power of the President is the issues imbued with both concerns to the full extent of his
President's constitutional duty and mandate to "ensure that powers, subject only to the limits supplied by law. In other
the laws be faithfully executed." The rule is that the President words, apart from an expressly mandated limit, or an implied
can execute the law without any delegation of power from the limit by virtue of incompatibility, the manner of execution by
legislature. the President must be given utmost deference. This approach
is not different from that taken by the Court in situations with
The import of this characteristic is that the manner of the fairly similar contexts.
President's execution of the law, even if not expressly granted
by the law, is justified by necessity and limited only by law, Thus, the analysis portrayed by the dissent does not give the
since the President must "take necessary and proper steps to President authority to bypass constitutional safeguards and
carry into execution the law."162 Justice George Malcolm limits. In fact, it specifies what these limitations are, how these
states this principle in a grand manner:163 limitations are triggered, how these limitations function, and
what can be done within the sphere of constitutional duties
The executive should be clothed with sufficient power to and limitations of the President.
administer efficiently the affairs of state. He should have
complete control of the instrumentalities through whom his Justice Brion's dissent likewise misinterprets the analysis
responsibility is discharged. It is still true, as said by Hamilton, proffered when it claims that the foreign relations power of the
that "A feeble executive implies a feeble execution of the President should not be interpreted in isolation.168 The
government. A feeble execution is but another phrase for a analysis itself demonstrates how the foreign affairs function,
bad execution; and a government ill executed, whatever it may while mostly the President's, is shared in several instances,
be in theory, must be in practice a bad government." The namely in Section 2 of Article II on the conduct of war;
mistakes of State governments need not be repeated here. Sections 20 and 21 of Article VII on foreign loans, treaties, and
international agreements; Sections 4(2) and 5(2)(a) of Article
xxxx VIII on the judicial review of executive acts; Sections 4 and 25
of Article XVIII on treaties and international agreements
Every other consideration to one side, this remains certain - entered into prior to the Constitution and on the presence of
The Congress of the United States clearly intended that the foreign military troops, bases, or facilities.
Governor-General's power should be commensurate with his
responsibility. The Congress never intended that the In fact, the analysis devotes a whole subheading to the
Governor-General should be saddled with the responsibility of relationship between the two major presidential functions and
administering the government and of executing the laws but the role of the Senate in it.
FINALS CONSTITUTIONAL LAW I ACJUCO NOV 11, 2017 9

Understandably, this Court must view the instant case with the
This approach of giving utmost deference to presidential same perspective and understanding, knowing full well the
initiatives in respect of foreign affairs is not novel to the Court. constitutional and legal repercussions of any judicial
The President's act of treating EDCA as an executive overreach.
agreement is not the principal power being analyzed as the
Dissenting Opinion seems to suggest. Rather, the preliminary 2. The plain meaning of the Constitution prohibits the entry of
analysis is in reference to the expansive power of foreign foreign military bases, troops or facilities, except by way of a
affairs. We have long treated this power as something the treaty concurred in by the Senate - a clear limitation on the
Courts must not unduly restrict. As we stated recently in President's dual role as defender of the State and as sole
Vinuya v. Romulo: authority in foreign relations.

To be sure, not all cases implicating foreign relations present Despite the President's roles as defender of the State and sole
political questions, and courts certainly possess the authority authority in foreign relations, the 1987 Constitution expressly
to construe or invalidate treaties and executive agreements. limits his ability in instances when it involves the entry of
However, the question whether the Philippine government foreign military bases, troops or facilities. The initial limitation
should espouse claims of its nationals against a foreign is found in Section 21 of the provisions on the Executive
government is a foreign relations matter, the authority for Department: "No treaty or international agreement shall be
which is demonstrably committed by our Constitution not to valid and effective unless concurred in by at least two-thirds
the courts but to the political branches. In this case, the of all the Members of the Senate." The specific limitation is
Executive Department has already decided that it is to the best given by Section 25 of the Transitory Provisions, the full text
interest of the country to waive all claims of its nationals for of which reads as follows:
reparations against Japan in the Treaty of Peace of 1951. The
wisdom of such decision is not for the courts to question. SECTION 25. After the expiration in 1991 of the Agreement
Neither could petitioners herein assail the said determination between the Republic of the Philippines and the United States
by the Executive Department via the instant petition for of America concerning Military Bases, foreign military bases,
certiorari. troops, or facilities shall not be allowed in the Philippines
except under a treaty duly concurred in by the Senate and,
In the seminal case of US v. Curtiss-Wright Export Corp., the when the Congress so requires, ratified by a majority of the
US Supreme Court held that "[t]he President is the sole organ votes cast by the people in a national referendum held for that
of the nation in its external relations, and its sole purpose, and recognized as a treaty by the other contracting
representative with foreign relations." State.

It is quite apparent that if, in the maintenance of our It is quite plain that the Transitory Provisions of the 1987
international relations, embarrassment - perhaps serious Constitution intended to add to the basic requirements of a
embarrassment - is to be avoided and success for our aims treaty under Section 21 of Article VII. This means that both
achieved, congressional legislation which is to be made provisions must be read as additional limitations to the
effective through negotiation and inquiry within the President's overarching executive function in matters of
international field must often accord to the President a degree defense and foreign relations.
of discretion and freedom from statutory restriction which
would not be admissible where domestic affairs alone 3. The President, however, may enter into an executive
involved. Moreover, he, not Congress, has the better agreement on foreign military bases, troops, or facilities, if (a)
opportunity of knowing the conditions which prevail in foreign it is not the instrument that allows the presence of foreign
countries, and especially is this true in time of war. He has his military bases, troops, or facilities; or (b) it merely aims to
confidential sources of information. He has his agents in the implement an existing law or treaty.
form of diplomatic, consular and other officials ....
Again we refer to Section 25, Article XVIII of the Constitution:
This ruling has been incorporated in our jurisprudence through
Bavan v. Executive Secretary and Pimentel v. Executive SECTION 25. After the expiration in 1991 of the Agreement
Secretary; its overreaching principle was, perhaps, best between the Republic of the Philippines and the United States
articulated in (now Chief) Justice Puno's dissent in Secretary of America concerning Military Bases, foreign military bases,
of Justice v. Lantion: troops, or facilities shall not be allowed in the Philippines
except under a treaty duly concurred in by the Senate and,
. . . The conduct of foreign relations is full of complexities and when the Congress so requires, ratified by a majority of the
consequences, sometimes with life and death significance to votes cast by the people in a national referendum held for that
the nation especially in times of war. It can only be entrusted purpose, and recognized as a treaty by the other contracting
to that department of government which can act on the basis State. (Emphases supplied)
of the best available information and can decide with
decisiveness .... It is also the President who possesses the In view of this provision, petitioners argue170 that EDCA must
most comprehensive and the most confidential information be in the form of a "treaty" duly concurred in by the Senate.
about foreign countries for our diplomatic and consular They stress that the Constitution is unambigous in mandating
officials regularly brief him on meaningful events all over the the transmission to the Senate of all international agreements
world. He has also unlimited access to ultra-sensitive military concluded after the expiration of the MBA in 1991 -
intelligence data. In fine, the presidential role in foreign affairs agreements that concern the presence of foreign military
is dominant and the President is traditionally accorded a wider bases, troops, or facilities in the country. Accordingly,
degree of discretion in the conduct of foreign affairs. The petitioners maintain that the Executive Department is not
regularity, nay, validity of his actions are adjudged under less given the choice to conclude agreements like EDCA in the
stringent standards, lest their judicial repudiation lead to form of an executive agreement.
breach of an international obligation, rupture of state relations,
forfeiture of confidence, national embarrassment and a This is also the view of the Senate, which, through a majority
plethora of other problems with equally undesirable vote of 15 of its members - with 1 against and 2 abstaining -
consequences.169 (Emphases supplied) says in SR 105171 that EDCA must be submitted to the
FINALS CONSTITUTIONAL LAW I ACJUCO NOV 11, 2017 10

Senate in the form of a treaty for concurrence by at least two- We look to the language of the document itself in our search
thirds of all its members. for its meaning. We do not of course stop there, but that is
where we begin. It is to be assumed that the words in which
The Senate cites two constitutional provisions (Article VI, constitutional provisions are couched express the objective
Section 21 and Article XVIII, Section 25) to support its sought to be attained. They are to be given their ordinary
position. Compared with the lone constitutional provision that meaning except where technical terms are employed in which
the Office of the Solicitor General (OSG) cites, which is Article case the significance thus attached to them prevails. As the
XVIII, Section 4(2), which includes the constitutionality of Constitution is not primarily a lawyer's document, it being
"executive agreement(s)" among the cases subject to the essential for the rule of law to obtain that it should ever be
Supreme Court's power of judicial review, the Constitution present in the people's consciousness, its language as much
clearly requires submission of EDCA to the Senate. Two as possible should be understood in the sense they have in
specific provisions versus one general provision means that common use. What it says according to the text of the
the specific provisions prevail. The term "executive provision to be construed compels acceptance and negates
agreement" is "a term wandering alone in the Constitution, the power of the courts to alter it, based on the postulate that
bereft of provenance and an unidentified constitutional the framers and the people mean what they say. Thus, these
mystery." are the cases where the need for construction is reduced to a
minimum.178 (Emphases supplied)
The author of SR 105, Senator Miriam Defensor Santiago,
upon interpellation even added that the MDT, which the It is only in those instances in which the constitutional
Executive claims to be partly implemented through EDCA, is provision is unclear, ambiguous, or silent that further
already obsolete. construction must be done to elicit its meaning.179 In Ang
Bagong Bayani-OFW v. Commission on Elections,180 we
There are two insurmountable obstacles to this Court's reiterated this guiding principle:
agreement with SR 105, as well as with the comment on
interpellation made by Senator Santiago. it [is] safer to construe the Constitution from what appears
upon its face. The proper interpretation therefore depends
First, the concept of "executive agreement" is so well- more on how it was understood by the people adopting it than
entrenched in this Court's pronouncements on the powers of in the framers' understanding thereof. (Emphases supplied)
the President. When the Court validated the concept of
"executive agreement," it did so with full knowledge of the The effect of this statement is surprisingly profound, for, if
Senate's role in concurring in treaties. It was aware of the taken literally, the phrase "shall not be allowed in the
problematique of distinguishing when an international Philippines" plainly refers to the entry of bases, troops, or
agreement needed Senate concurrence for validity, and when facilities in the country. The Oxford English Dictionary defines
it did not; and the Court continued to validate the existence of the word "allow" as a transitive verb that means "to permit,
"executive agreements" even after the 1987 Constitution.172 enable"; "to give consent to the occurrence of or relax restraint
This follows a long line of similar decisions upholding the on (an action, event, or activity)"; "to consent to the presence
power of the President to enter into an executive or attendance of (a person)"; and, when with an adverbial of
agreement.173 place, "to permit (a person or animal) to go, come, or be in,
out, near, etc."181 Black's Law Dictionary defines the term as
Second, the MDT has not been rendered obsolescent, one that means "[t]o grant, approve, or permit."182
considering that as late as 2009,174 this Court continued to
recognize its validity. The verb "allow" is followed by the word "in," which is a
preposition used to indicate "place or position in space or
Third, to this Court, a plain textual reading of Article XIII, anything having material extension: Within the limits or
Section 25, inevitably leads to the conclusion that it applies bounds of, within (any place or thing)."183 That something is
only to a proposed agreement between our government and the Philippines, which is the noun that follows.
a foreign government, whereby military bases, troops, or
facilities of such foreign government would be "allowed" or It is evident that the constitutional restriction refers solely to
would "gain entry" Philippine territory. the initial entry of the foreign military bases, troops, or
facilities. Once entry is authorized, the subsequent acts are
Note that the provision "shall not be allowed" is a negative thereafter subject only to the limitations provided by the rest
injunction. This wording signifies that the President is not of the Constitution and Philippine law, and not to the Section
authorized by law to allow foreign military bases, troops, or 25 requirement of validity through a treaty.
facilities to enter the Philippines, except under a treaty
concurred in by the Senate. Hence, the constitutionally The VFA has already allowed the entry of troops in the
restricted authority pertains to the entry of the bases, troops, Philippines. This Court stated in Lim v. Executive Secretary:
or facilities, and not to the activities to be done after entry.
After studied reflection, it appeared farfetched that the
Under the principles of constitutional construction, of ambiguity surrounding the meaning of the word "activities"
paramount consideration is the plain meaning of the language arose from accident. In our view, it was deliberately made that
expressed in the Constitution, or the verba legis rule.175 It is way to give both parties a certain leeway in negotiation. In this
presumed that the provisions have been carefully crafted in manner, visiting US forces may sojourn in Philippine territory
order to express the objective it seeks to attain.176 It is for purposes other than military. As conceived, the joint
incumbent upon the Court to refrain from going beyond the exercises may include training on new techniques of patrol
plain meaning of the words used in the Constitution. It is and surveillance to protect the nation's marine resources, sea
presumed that the framers and the people meant what they search-and-rescue operations to assist vessels in distress,
said when they said it, and that this understanding was disaster relief operations, civic action projects such as the
reflected in the Constitution and understood by the people in building of school houses, medical and humanitarian
the way it was meant to be understood when the fundamental missions, and the like.
law was ordained and promulgated.177 As this Court has
often said: Under these auspices, the VFA gives legitimacy to the current
Balikatan exercises. It is only logical to assume that "Balikatan
FINALS CONSTITUTIONAL LAW I ACJUCO NOV 11, 2017 11

02-1," a "mutual anti- terrorism advising, assisting and training allowed in the Philippines," there is no need for such
exercise," falls under the umbrella of sanctioned or allowable reference. The law is clear. No less than the Senate
activities in the context of the agreement. Both the history and understood this when it ratified the VFA.
intent of the Mutual Defense Treaty and the VFA support the
conclusion that combat-related activities -as opposed to 4. The President may generally enter into executive
combat itself-such as the one subject of the instant petition, agreements subject to limitations defined by the Constitution
are indeed authorized.184 (Emphasis supplied) and may be in furtherance of a treaty already concurred in by
the Senate.
Moreover, the Court indicated that the Constitution continues
to govern the conduct of foreign military troops in the We discuss in this section why the President can enter into
Philippines,185 readily implying the legality of their initial entry executive agreements.
into the country.
It would be helpful to put into context the contested language
The OSG emphasizes that EDCA can be in the form of an found in Article XVIII, Section 25. Its more exacting
executive agreement, since it merely involves "adjustments in requirement was introduced because of the previous
detail" in the implementation of the MDT and the VFA.186 It experience of the country when its representatives felt
points out that there are existing treaties between the compelled to consent to the old MBA.191 They felt
Philippines and the U.S. that have already been concurred in constrained to agree to the MBA in fulfilment of one of the
by the Philippine Senate and have thereby met the major conditions for the country to gain independence from
requirements of the Constitution under Section 25. Because the U.S.192 As a result of that experience, a second layer of
of the status of these prior agreements, respondent consent for agreements that allow military bases, troops and
emphasizes that EDCA need not be transmitted to the Senate. facilities in the country is now articulated in Article XVIII of our
present Constitution.
The aforecited Dissenting Opinion of Justice Brion disagrees
with the ponencia's application of verba legis construction to This second layer of consent, however, cannot be interpreted
the words of Article XVIII, Section 25.187 It claims that the in such a way that we completely ignore the intent of our
provision is "neither plain, nor that simple."188 To buttress its constitutional framers when they provided for that additional
disagreement, the dissent states that the provision refers to a layer, nor the vigorous statements of this Court that affirm the
historical incident, which is the expiration of the 1947 continued existence of that class of international agreements
MBA.189 Accordingly, this position requires questioning the called "executive agreements."
circumstances that led to the historical event, and the meaning
of the terms under Article XVIII, Section 25. The power of the President to enter into binding executive
agreements without Senate concurrence is already well-
This objection is quite strange. The construction technique of established in this jurisdiction.193 That power has been
verba legis is not inapplicable just because a provision has a alluded to in our present and past Constitutions,194 in various
specific historical context. In fact, every provision of the statutes,195 in Supreme Court decisions,196 and during the
Constitution has a specific historical context. The purpose of deliberations of the Constitutional Commission.197 They
constitutional and statutory construction is to set tiers of cover a wide array of subjects with varying scopes and
interpretation to guide the Court as to how a particular purposes,198 including those that involve the presence of
provision functions. Verba legis is of paramount consideration, foreign military forces in the country.199
but it is not the only consideration. As this Court has often said:
As the sole organ of our foreign relations200 and the
We look to the language of the document itself in our search constitutionally assigned chief architect of our foreign
for its meaning. We do not of course stop there, but that is policy,201 the President is vested with the exclusive power to
where we begin. It is to be assumed that the words in which conduct and manage the country's interface with other states
constitutional provisions are couched express the objective and governments. Being the principal representative of the
sought to be attained. They are to be given their ordinary Philippines, the Chief Executive speaks and listens for the
meaning except where technical terms are employed in which nation; initiates, maintains, and develops diplomatic relations
case the significance thus attached to them prevails. As the with other states and governments; negotiates and enters into
Constitution is not primarily a lawyer's document, it being international agreements; promotes trade, investments,
essential for the rule of law to obtain that it should ever be tourism and other economic relations; and settles international
present in the people's consciousness, its language as much disputes with other states.202
as possible should be understood in the sense they have in
common use. What it says according to the text of the As previously discussed, this constitutional mandate
provision to be construed compels acceptance and negates emanates from the inherent power of the President to enter
the power of the courts to alter it, based on the postulate that into agreements with other states, including the prerogative to
the framers and the people mean what they say. Thus, these conclude binding executive agreements that do not require
are the cases where the need for construction is reduced to a further Senate concurrence. The existence of this presidential
minimum.190 (Emphases supplied) power203 is so well-entrenched that Section 5(2)(a), Article
VIII of the Constitution, even provides for a check on its
As applied, verba legis aids in construing the ordinary exercise. As expressed below, executive agreements are
meaning of terms. In this case, the phrase being construed is among those official governmental acts that can be the subject
"shall not be allowed in the Philippines" and not the preceding of this Court's power of judicial review:
one referring to "the expiration in 1991 of the Agreement
between the Republic of the Philippines and the United States (2) Review, revise, reverse, modify, or affirm on appeal or
of America concerning Military Bases, foreign military bases, certiorari, as the law or the Rules of Court may provide, final
troops, or facilities." It is explicit in the wording of the provision judgments and orders of lower courts in:
itself that any interpretation goes beyond the text itself and into
the discussion of the framers, the context of the Constitutional (a) All cases in which the constitutionality or validity of any
Commission's time of drafting, and the history of the 1947 treaty, international or executive agreement, law, presidential
MBA. Without reference to these factors, a reader would not decree, proclamation, order, instruction, ordinance, or
understand those terms. However, for the phrase "shall not be regulation is in question. (Emphases supplied)
FINALS CONSTITUTIONAL LAW I ACJUCO NOV 11, 2017 12

constitutionalist Fr. Joaquin Bernas quoted the Court's ruling


In Commissioner of Customs v. Eastern Sea Trading, in Eastern Sea Trading, the Constitutional Commission
executive agreements are defined as "international members ultimately decided that the term "international
agreements embodying adjustments of detail carrying out agreements" as contemplated in Section 21, Article VII, does
well-established national policies and traditions and those not include executive agreements, and that a proviso is no
involving arrangements of a more or less temporary longer needed. Their discussion is reproduced below:207
nature."204 In Bayan Muna v. Romulo, this Court further
clarified that executive agreements can cover a wide array of MS. AQUINO: Madam President, first I would like a
subjects that have various scopes and purposes.205 They are clarification from the Committee. We have retained the words
no longer limited to the traditional subjects that are usually "international agreement" which I think is the correct judgment
covered by executive agreements as identified in Eastern Sea on the matter because an international agreement is different
Trading. The Court thoroughly discussed this matter in the from a treaty. A treaty is a contract between parties which is
following manner: in the nature of international agreement and also a municipal
law in the sense that the people are bound. So there is a
The categorization of subject matters that may be covered by conceptual difference. However, I would like to be clarified if
international agreements mentioned in Eastern Sea Trading the international agreements include executive agreements.
is not cast in stone. x x x.
MR. CONCEPCION: That depends upon the parties. All
As may be noted, almost half a century has elapsed since the parties to these international negotiations stipulate the
Court rendered its decision in Eastern Sea Trading. Since conditions which are necessary for the agreement or whatever
then, the conduct of foreign affairs has become more complex it may be to become valid or effective as regards the parties.
and the domain of international law wider, as to include such
subjects as human rights, the environment, and the sea. In MS. AQUINO: Would that depend on the parties or would that
fact, in the US alone, the executive agreements executed by depend on the nature of the executive agreement? According
its President from 1980 to 2000 covered subjects such as to common usage, there are two types of executive
defense, trade, scientific cooperation, aviation, atomic energy, agreement: one is purely proceeding from an executive act
environmental cooperation, peace corps, arms limitation, and which affects external relations independent of the legislative
nuclear safety, among others. Surely, the enumeration in and the other is an executive act in pursuance of legislative
Eastern Sea Trading cannot circumscribe the option of each authorization. The first kind might take the form of just
state on the matter of which the international agreement conventions or exchanges of notes or protocol while the other,
format would be convenient to serve its best interest. As which would be pursuant to the legislative authorization, may
Francis Sayre said in his work referred to earlier: be in the nature of commercial agreements.

. . . It would be useless to undertake to discuss here the large MR. CONCEPCION: Executive agreements are generally
variety of executive agreements as such concluded from time made to implement a treaty already enforced or to determine
to time. Hundreds of executive agreements, other than those the details for the implementation of the treaty. We are
entered into under the trade-agreement act, have been speaking of executive agreements, not international
negotiated with foreign governments. . . . They cover such agreements.
subjects as the inspection of vessels, navigation dues, income
tax on shipping profits, the admission of civil air craft, custom MS. AQUINO: I am in full agreement with that, except that it
matters and commercial relations generally, international does not cover the first kind of executive agreement which is
claims, postal matters, the registration of trademarks and just protocol or an exchange of notes and this would be in the
copyrights, etc .... (Emphases Supplied) nature of reinforcement of claims of a citizen against a
country, for example.
One of the distinguishing features of executive agreements is
that their validity and effectivity are not affected by a lack of MR. CONCEPCION: The Commissioner is free to require
Senate concurrence.206 This distinctive feature was ratification for validity insofar as the Philippines is concerned.
recognized as early as in Eastern Sea Trading (1961), viz:
MS. AQUINO: It is my humble submission that we should
Treaties are formal documents which require ratification with provide, unless the Committee explains to us otherwise, an
the approval of two-thirds of the Senate. Executive explicit proviso which would except executive agreements
agreements become binding through executive action without from the requirement of concurrence of two-thirds of the
the need of a vote by the Senate or by Congress. Members of the Senate. Unless I am enlightened by the
Committee I propose that tentatively, the sentence should
xxxx read. "No treaty or international agreement EXCEPT
EXECUTIVE AGREEMENTS shall be valid and effective."
[T]he right of the Executive to enter into binding agreements
without the necessity of subsequent Congressional approval FR. BERNAS: I wonder if a quotation from the Supreme Court
has been confirmed by long usage. From the earliest days of decision [in Eastern Sea Trading] might help clarify this:
our history we have entered into executive agreements
covering such subjects as commercial and consular relations, The right of the executive to enter into binding agreements
most-favored-nation rights, patent rights, trademark and without the necessity of subsequent Congressional approval
copyright protection, postal and navigation arrangements and has been confirmed by long usage. From the earliest days of
the settlement of claims. The validity of these has never been our history, we have entered into executive agreements
seriously questioned by our courts. (Emphases Supplied) covering such subjects as commercial and consular relations,
most favored nation rights, patent rights, trademark and
That notion was carried over to the present Constitution. In copyright protection, postal and navigation arrangements and
fact, the framers specifically deliberated on whether the the settlement of claims. The validity of this has never been
general term "international agreement" included executive seriously questioned by our Courts.
agreements, and whether it was necessary to include an
express proviso that would exclude executive agreements Agreements with respect to the registration of trademarks
from the requirement of Senate concurrence. After noted have been concluded by the executive of various countries
FINALS CONSTITUTIONAL LAW I ACJUCO NOV 11, 2017 13

under the Act of Congress of March 3, 1881 (21 Stat. 502) . . xxx
. International agreements involving political issues or
changes of national policy and those involving international MR. ROMULO: I wish to be recognized first. I have only one
agreements of a permanent character usually take the form of question. Do we take it, therefore, that as far as the Committee
treaties. But international agreements embodying is concerned, the term "international agreements" does not
adjustments of detail, carrying out well established national include the term "executive agreements" as read by the
policies and traditions and those involving arrangements of a Commissioner in that text?
more or less temporary nature usually take the form of
executive agreements. FR. BERNAS: Yes. (Emphases Supplied)

MR. ROMULO: Is the Commissioner, therefore, excluding the The inapplicability to executive agreements of the
executive agreements? requirements under Section 21 was again recognized in
Bayan v. Zamora and in Bayan Muna v. Romulo. These
FR. BERNAS: What we are referring to, therefore, when we cases, both decided under the aegis of the present
say international agreements which need concurrence by at Constitution, quoted Eastern Sea Trading in reiterating that
least two-thirds are those which are permanent in nature. executive agreements are valid and binding even without the
concurrence of the Senate.
MS. AQUINO: And it may include commercial agreements
which are executive agreements essentially but which are Executive agreements may dispense with the requirement of
proceeding from the authorization of Congress. If that is our Senate concurrence because of the legal mandate with which
understanding, then I am willing to withdraw that amendment. they are concluded. As culled from the afore-quoted
deliberations of the Constitutional Commission, past Supreme
FR. BERNAS: If it is with prior authorization of Congress, then Court Decisions, and works of noted scholars,208 executive
it does not need subsequent concurrence by Congress. agreements merely involve arrangements on the
implementation of existing policies, rules, laws, or
MS. AQUINO: In that case, I am withdrawing my amendment. agreements. They are concluded (1) to adjust the details of a
treaty;209 (2) pursuant to or upon confirmation by an act of
MR. TINGSON: Madam President. the Legislature;210 or (3) in the exercise of the President's
independent powers under the Constitution.211 The raison
THE PRESIDENT: Is Commissioner Aquino satisfied? d'etre of executive agreements hinges on prior constitutional
or legislative authorizations.
MS. AQUINO: Yes. There is already an agreement among us
on the definition of "executive agreements" and that would The special nature of an executive agreement is not just a
make unnecessary any explicit proviso on the matter. domestic variation in international agreements. International
practice has accepted the use of various forms and
xxx designations of international agreements, ranging from the
traditional notion of a treaty - which connotes a formal, solemn
MR. GUINGONA: I am not clear as to the meaning of instrument - to engagements concluded in modem, simplified
"executive agreements" because I heard that these executive forms that no longer necessitate ratification.212 An
agreements must rely on treaties. In other words, there must international agreement may take different forms: treaty, act,
first be treaties. protocol, agreement, concordat, compromis d'arbitrage,
convention, covenant, declaration, exchange of notes, statute,
MR. CONCEPCION: No, I was speaking about the common pact, charter, agreed minute, memorandum of agreement,
use, as executive agreements being the implementation of modus vivendi, or some other form.213 Consequently, under
treaties, details of which do not affect the sovereignty of the international law, the distinction between a treaty and an
State. international agreement or even an executive agreement is
irrelevant for purposes of determining international rights and
MR. GUINGONA: But what about the matter of permanence, obligations.
Madam President? Would 99 years be considered
permanent? What would be the measure of permanency? I do However, this principle does not mean that the domestic law
not conceive of a treaty that is going to be forever, so there distinguishing treaties, international agreements, and
must be some kind of a time limit. executive agreements is relegated to a mere variation in form,
or that the constitutional requirement of Senate concurrence
MR. CONCEPCION: I suppose the Commissioner's question is demoted to an optional constitutional directive. There
is whether this type of agreement should be included in a remain two very important features that distinguish treaties
provision of the Constitution requiring the concurrence of from executive agreements and translate them into terms of
Congress. art in the domestic setting.

MR. GUINGONA: It depends on the concept of the executive First, executive agreements must remain traceable to an
agreement of which I am not clear. If the executive agreement express or implied authorization under the Constitution,
partakes of the nature of a treaty, then it should also be statutes, or treaties. The absence of these precedents puts
included. the validity and effectivity of executive agreements under
serious question for the main function of the Executive is to
MR. CONCEPCION: Whether it partakes or not of the nature enforce the Constitution and the laws enacted by the
of a treaty, it is within the power of the Constitutional Legislature, not to defeat or interfere in the performance of
Commission to require that. these rules.214 In turn, executive agreements cannot create
new international obligations that are not expressly allowed or
MR. GUINGONA: Yes. That is why I am trying to clarify reasonably implied in the law they purport to implement.
whether the words "international agreements" would include
executive agreements. Second, treaties are, by their very nature, considered superior
to executive agreements. Treaties are products of the acts of
MR. CONCEPCION: No, not necessarily; generally no. the Executive and the Senate215 unlike executive
FINALS CONSTITUTIONAL LAW I ACJUCO NOV 11, 2017 14

agreements, which are solely executive actions.216 Because b. International agreements are similar instruments, the
of legislative participation through the Senate, a treaty is provisions of which may require the ratification of a designated
regarded as being on the same level as a statute.217 If there number of parties thereto. These agreements involving
is an irreconcilable conflict, a later law or treaty takes political issues or changes in national policy, as well as those
precedence over one that is prior.218 An executive agreement involving international agreements of a permanent character,
is treated differently. Executive agreements that are usually take the form of treaties. They may also include
inconsistent with either a law or a treaty are considered commercial agreements, which are executive agreements
ineffective.219 Both types of international agreement are essentially, but which proceed from previous authorization by
nevertheless subject to the supremacy of the Constitution.220 Congress, thus dispensing with the requirement of
concurrence by the Senate.227
This rule does not imply, though, that the President is given
carte blanche to exercise this discretion. Although the Chief c. Executive agreements are generally intended to implement
Executive wields the exclusive authority to conduct our foreign a treaty already enforced or to determine the details of the
relations, this power must still be exercised within the context implementation thereof that do not affect the sovereignty of
and the parameters set by the Constitution, as well as by the State.228
existing domestic and international laws. There are
constitutional provisions that restrict or limit the President's 2. Treaties and international agreements that cannot be mere
prerogative in concluding international agreements, such as executive agreements must, by constitutional decree, be
those that involve the following: concurred in by at least two-thirds of the Senate.

a. The policy of freedom from nuclear weapons within 3. However, an agreement - the subject of which is the entry
Philippine territory221 of foreign military troops, bases, or facilities - is particularly
restricted. The requirements are that it be in the form of a
b. The fixing of tariff rates, import and export quotas, tonnage treaty concurred in by the Senate; that when Congress so
and wharfage dues, and other duties or imposts, which must requires, it be ratified by a majority of the votes cast by the
be pursuant to the authority granted by Congress222 people in a national referendum held for that purpose; and that
it be recognized as a treaty by the other contracting State.
c. The grant of any tax exemption, which must be pursuant to
a law concurred in by a majority of all the Members of 4. Thus, executive agreements can continue to exist as a
Congress223 species of international agreements.

d. The contracting or guaranteeing, on behalf of the That is why our Court has ruled the way it has in several
Philippines, of foreign loans that must be previously concurred cases.
in by the Monetary Board224
In Bayan Muna v. Romulo, we ruled that the President acted
e. The authorization of the presence of foreign military bases, within the scope of her constitutional authority and discretion
troops, or facilities in the country must be in the form of a treaty when she chose to enter into the RP-U.S. Non-Surrender
duly concurred in by the Senate.225 Agreement in the form of an executive agreement, instead of
a treaty, and in ratifying the agreement without Senate
f. For agreements that do not fall under paragraph 5, the concurrence. The Court en banc discussed this intrinsic
concurrence of the Senate is required, should the form of the presidential prerogative as follows:
government chosen be a treaty.
Petitioner parlays the notion that the Agreement is of dubious
5. The President had the choice to enter into EDCA by way of validity, partaking as it does of the nature of a treaty; hence, it
an executive agreement or a treaty. must be duly concurred in by the Senate. x x x x. Pressing its
point, petitioner submits that the subject of the Agreement
No court can tell the President to desist from choosing an does not fall under any of the subject-categories that xx x may
executive agreement over a treaty to embody an international be covered by an executive agreement, such as
agreement, unless the case falls squarely within Article VIII, commercial/consular relations, most-favored nation rights,
Section 25. patent rights, trademark and copyright protection, postal and
navigation arrangements and settlement of claims.
As can be gleaned from the debates among the members of
the Constitutional Commission, they were aware that legally The categorization of subject matters that may be covered by
binding international agreements were being entered into by international agreements mentioned in Eastern Sea Trading
countries in forms other than a treaty. At the same time, it is is not cast in stone. There are no hard and fast rules on the
clear that they were also keen to preserve the concept of propriety of entering, on a given subject, into a treaty or an
"executive agreements" and the right of the President to enter executive agreement as an instrument of international
into such agreements. relations. The primary consideration in the choice of the form
of agreement is the parties' intent and desire to craft an
What we can glean from the discussions of the Constitutional international agreement in the form they so wish to further
Commissioners is that they understood the following realities: their respective interests. Verily, the matter of form takes a
back seat when it comes to effectiveness and binding effect of
1. Treaties, international agreements, and executive the enforcement of a treaty or an executive agreement, as the
agreements are all constitutional manifestations of the parties in either international agreement each labor under the
conduct of foreign affairs with their distinct legal pacta sunt servanda principle.
characteristics.
xxxx
a. Treaties are formal contracts between the Philippines and
other States-parties, which are in the nature of international But over and above the foregoing considerations is the fact
agreements, and also of municipal laws in the sense of their that - save for the situation and matters contemplated in Sec.
binding nature.226 25, Art. XVIII of the Constitution - when a treaty is required,
the Constitution does not classify any subject, like that
FINALS CONSTITUTIONAL LAW I ACJUCO NOV 11, 2017 15

involving political issues, to be in the form of, and ratified as, entered into the Romulo-Kenney Agreement, which undertook
a treaty. What the Constitution merely prescribes is that to clarify the detention of a U.S. Armed Forces member,
treaties need the concurrence of the Senate by a vote defined whose case was pending appeal after his conviction by a trial
therein to complete the ratification process. court for the crime of rape. In testing the validity of the latter
agreement, the Court precisely alluded to one of the inherent
xxxx limitations of an executive agreement: it cannot go beyond the
terms of the treaty it purports to implement. It was eventually
x x x. As the President wields vast powers and influence, her ruled that the Romulo-Kenney Agreement was "not in accord"
conduct in the external affairs of the nation is, as Bayan would with the VFA, since the former was squarely inconsistent with
put it, "executive altogether." The right of the President to a provision in the treaty requiring that the detention be "by
enter into or ratify binding executive agreements has been Philippine authorities." Consequently, the Court ordered the
confirmed by long practice. Secretary of Foreign Affairs to comply with the VFA and
"forthwith negotiate with the United States representatives for
In thus agreeing to conclude the Agreement thru E/N BF0- the appropriate agreement on detention facilities under
028-03, then President Gloria Macapagal-Arroyo, represented Philippine authorities as provided in Art. V, Sec. 10 of the VFA.
by the Secretary of Foreign Affairs, acted within the scope of "233
the authority and discretion vested in her by the Constitution.
At the end of the day, the President - by ratifying, thru her Culling from the foregoing discussions, we reiterate the
deputies, the non-surrender agreement - did nothing more following pronouncements to guide us in resolving the present
than discharge a constitutional duty and exercise a controversy:
prerogative that pertains to her office. (Emphases supplied)
1. Section 25, Article XVIII of the Constitution, contains
Indeed, in the field of external affairs, the President must be stringent requirements that must be fulfilled by the
given a larger measure of authority and wider discretion, international agreement allowing the presence of foreign
subject only to the least amount of checks and restrictions military bases, troops, or facilities in the Philippines: (a) the
under the Constitution.229 The rationale behind this power agreement must be in the form of a treaty, and (b) it must be
and discretion was recognized by the Court in Vinuya v. duly concurred in by the Senate.
Executive Secretary, cited earlier.230
2. If the agreement is not covered by the above situation, then
Section 9 of Executive Order No. 459, or the Guidelines in the the President may choose the form of the agreement (i.e.,
Negotiation of International Agreements and its Ratification, either an executive agreement or a treaty), provided that the
thus, correctly reflected the inherent powers of the President agreement dealing with foreign military bases, troops, or
when it stated that the DFA "shall determine whether an facilities is not the principal agreement that first allows their
agreement is an executive agreement or a treaty." entry or presence in the Philippines.

Accordingly, in the exercise of its power of judicial review, the 3. The executive agreement must not go beyond the
Court does not look into whether an international agreement parameters, limitations, and standards set by the law and/or
should be in the form of a treaty or an executive agreement, treaty that the former purports to implement; and must not
save in cases in which the Constitution or a statute requires unduly expand the international obligation expressly
otherwise. Rather, in view of the vast constitutional powers mentioned or necessarily implied in the law or treaty.
and prerogatives granted to the President in the field of foreign
affairs, the task of the Court is to determine whether the 4. The executive agreement must be consistent with the
international agreement is consistent with the applicable Constitution, as well as with existing laws and treaties.
limitations.
In light of the President's choice to enter into EDCA in the form
6. Executive agreements may cover the matter of foreign of an executive agreement, respondents carry the burden of
military forces if it merely involves detail adjustments. proving that it is a mere implementation of existing laws and
treaties concurred in by the Senate. EDCA must thus be
The practice of resorting to executive agreements in adjusting carefully dissected to ascertain if it remains within the legal
the details of a law or a treaty that already deals with the parameters of a valid executive agreement.
presence of foreign military forces is not at all unusual in this
jurisdiction. In fact, the Court has already implicitly 7. EDCA is consistent with the content, purpose, and
acknowledged this practice in Lim v. Executive Secretary.231 framework of the MDT and the VFA
In that case, the Court was asked to scrutinize the
constitutionality of the Terms of Reference of the Balikatan 02- The starting point of our analysis is the rule that "an executive
1 joint military exercises, which sought to implement the VFA. agreement xx x may not be used to amend a treaty."234 In
Concluded in the form of an executive agreement, the Terms Lim v. Executive Secretary and in Nicolas v. Romulo, the
of Reference detailed the coverage of the term "activities" Court approached the question of the validity of executive
mentioned in the treaty and settled the matters pertaining to agreements by comparing them with the general framework
the construction of temporary structures for the U.S. troops and the specific provisions of the treaties they seek to
during the activities; the duration and location of the exercises; implement.
the number of participants; and the extent of and limitations
on the activities of the U.S. forces. The Court upheld the In Lim, the Terms of Reference of the joint military exercises
Terms of Reference as being consistent with the VFA. It no was scrutinized by studying "the framework of the treaty
longer took issue with the fact that the Balikatan Terms of antecedents to which the Philippines bound itself,"235 i.e., the
Reference was not in the form of a treaty concurred in by the MDT and the VFA. The Court proceeded to examine the
Senate, even if it dealt with the regulation of the activities of extent of the term "activities" as contemplated in Articles 1236
foreign military forces on Philippine territory. and II237 of the VFA. It later on found that the term "activities"
was deliberately left undefined and ambiguous in order to
In Nicolas v. Romulo,232 the Court again impliedly affirmed permit "a wide scope of undertakings subject only to the
the use of an executive agreement in an attempt to adjust the approval of the Philippine government"238 and thereby allow
details of a provision of the VFA. The Philippines and the U.S. the parties "a certain leeway in negotiation."239 The Court
FINALS CONSTITUTIONAL LAW I ACJUCO NOV 11, 2017 16

eventually ruled that the Terms of Reference fell within the as opposed to combat itself such as the one subject of the
sanctioned or allowable activities, especially in the context of instant petition, are indeed authorized.247
the VFA and the MDT.
Hence, even if EDCA was borne of military necessity, it cannot
The Court applied the same approach to Nicolas v. Romulo. It be said to have strayed from the intent of the VFA since
studied the provisions of the VFA on custody and detention to EDCA's combat-related components are allowed under the
ascertain the validity of the Romulo-Kenney Agreement.240 It treaty.
eventually found that the two international agreements were
not in accord, since the Romulo-Kenney Agreement had Moreover, both the VFA and EDCA are silent on what these
stipulated that U.S. military personnel shall be detained at the activities actually are. Both the VFA and EDCA deal with the
U.S. Embassy Compound and guarded by U.S. military presence of U.S. forces within the Philippines, but make no
personnel, instead of by Philippine authorities. According to mention of being platforms for activity beyond Philippine
the Court, the parties "recognized the difference between territory. While it may be that, as applied, military operations
custody during the trial and detention after conviction."241 under either the VFA or EDCA would be carried out in the
Pursuant to Article V(6) of the VFA, the custody of a U.S. future the scope of judicial review does not cover potential
military personnel resides with U.S. military authorities during breaches of discretion but only actual occurrences or blatantly
trial. Once there is a finding of guilt, Article V(l0) requires that illegal provisions. Hence, we cannot invalidate EDCA on the
the confinement or detention be "by Philippine authorities." basis of the potentially abusive use of its provisions.

Justice Marvic M.V.F. Leonen's Dissenting Opinion posits that The fourth difference is that EDCA supposedly introduces a
EDCA "substantially modifies or amends the VFA"242 and new concept not contemplated in the VFA or the MDT: Agreed
follows with an enumeration of the differences between EDCA Locations, Contractors, Pre-positioning, and Operational
and the VFA. While these arguments will be rebutted more Control.248
fully further on, an initial answer can already be given to each
of the concerns raised by his dissent. As previously mentioned, these points shall be addressed fully
and individually in the latter analysis of EDCA's provisions.
The first difference emphasized is that EDCA does not only However, it must already be clarified that the terms and details
regulate visits as the VFA does, but allows temporary used by an implementing agreement need not be found in the
stationing on a rotational basis of U.S. military personnel and mother treaty. They must be sourced from the authority
their contractors in physical locations with permanent facilities derived from the treaty, but are not necessarily expressed
and pre-positioned military materiel. word-for-word in the mother treaty. This concern shall be
further elucidated in this Decision.
This argument does not take into account that these
permanent facilities, while built by U.S. forces, are to be The fifth difference highlighted by the Dissenting Opinion is
owned by the Philippines once constructed.243 Even the VFA that the VFA does not have provisions that may be construed
allowed construction for the benefit of U.S. forces during their as a restriction on or modification of obligations found in
temporary visits. existing statues, including the jurisdiction of courts, local
autonomy, and taxation. Implied in this argument is that EDCA
The second difference stated by the dissent is that EDCA contains such restrictions or modifications.249
allows the prepositioning of military materiel, which can
include various types of warships, fighter planes, bombers, This last argument cannot be accepted in view of the clear
and vessels, as well as land and amphibious vehicles and provisions of EDCA. Both the VFA and EDCA ensure
their corresponding ammunition.244 Philippine jurisdiction in all instances contemplated by both
agreements, with the exception of those outlined by the VFA
However, the VFA clearly allows the same kind of equipment, in Articles III-VI. In the VFA, taxes are clearly waived whereas
vehicles, vessels, and aircraft to be brought into the country. in EDCA, taxes are assumed by the government as will be
Articles VII and VIII of the VFA contemplates that U.S. discussed later on. This fact does not, therefore, produce a
equipment, materials, supplies, and other property are diminution of jurisdiction on the part of the Philippines, but
imported into or acquired in the Philippines by or on behalf of rather a recognition of sovereignty and the rights that attend
the U.S. Armed Forces; as are vehicles, vessels, and aircraft it, some of which may be waived as in the cases under Articles
operated by or for U.S. forces in connection with activities III-VI of the VFA.
under the VFA. These provisions likewise provide for the
waiver of the specific duties, taxes, charges, and fees that Taking off from these concerns, the provisions of EDCA must
correspond to these equipment. be compared with those of the MDT and the VFA, which are
the two treaties from which EDCA allegedly draws its validity.
The third difference adverted to by the Justice Leonen's
dissent is that the VFA contemplates the entry of troops for "Authorized presence" under the VFA versus "authorized
training exercises, whereas EDCA allows the use of territory activities" under EDCA: (1) U.S. personnel and (2) U.S.
for launching military and paramilitary operations conducted in contractors
other states.245 The dissent of Justice Teresita J. Leonardo-
De Castro also notes that VFA was intended for non-combat The OSG argues250 that EDCA merely details existing
activides only, whereas the entry and activities of U.S. forces policies under the MDT and the VFA. It explains that EDCA
into Agreed Locations were borne of military necessity or had articulates the principle of defensive preparation embodied in
a martial character, and were therefore not contemplated by Article II of the MDT; and seeks to enhance the defensive,
the VFA.246 strategic, and technological capabilities of both parties
pursuant to the objective of the treaty to strengthen those
This Court's jurisprudence however established in no capabilities to prevent or resist a possible armed attack.
uncertain terms that combat-related activities, as opposed to Respondent also points out that EDCA simply implements
actual combat, were allowed under the MDT and VFA, viz: Article I of the VFA, which already allows the entry of U.S.
troops and personnel into the country. Respondent stresses
Both the history and intent of the Mutual Defense Treaty and this Court's recognition in Lim v. Executive Secretary that U.S.
the VFA support the conclusion that combat-related activities troops and personnel are authorized to conduct activities that
FINALS CONSTITUTIONAL LAW I ACJUCO NOV 11, 2017 17

promote the goal of maintaining and developing their defense to the VFA in describing U.S. personnel, a term defined under
capability. Article I of the treaty as follows:

Petitioners contest251 the assertion that the provisions of As used in this Agreement, "United States personnel" means
EDCA merely implement the MDT. According to them, the United States military and civilian personnel temporarily in the
treaty does not specifically authorize the entry of U.S. troops Philippines in connection with activities approved by the
in the country in order to maintain and develop the individual Philippine Government. Within this definition:
and collective capacities of both the Philippines and the U.S.
to resist an armed attack. They emphasize that the treaty was 1. The term "military personnel" refers to military members of
concluded at a time when there was as yet no specific the United States Army, Navy, Marine Corps, Air Force, and
constitutional prohibition on the presence of foreign military Coast Guard.
forces in the country.
2. The term "civilian personnel" refers to individuals who are
Petitioners also challenge the argument that EDCA simply neither nationals of nor ordinarily resident in the Philippines
implements the VFA. They assert that the agreement covers and who are employed by the United States armed forces or
only short-term or temporary visits of U.S. troops "from time to who are accompanying the United States armed forces, such
time" for the specific purpose of combined military exercises as employees of the American Red Cross and the United
with their Filipino counterparts. They stress that, in contrast, Services Organization.258
U.S. troops are allowed under EDCA to perform activities
beyond combined military exercises, such as those Article II of EDCA must then be read with Article III of the VFA,
enumerated in Articles 111(1) and IV(4) thereof. Furthermore, which provides for the entry accommodations to be accorded
there is some degree of permanence in the presence of U.S. to U.S. military and civilian personnel:
troops in the country, since the effectivity of EDCA is
continuous until terminated. They proceed to argue that while 1. The Government of the Philippines shall facilitate the
troops have a "rotational" presence, this scheme in fact fosters admission of United States personnel and their departure from
their permanent presence. the Philippines in connection with activities covered by this
agreement.
a. Admission of U.S. military and civilian personnel into
Philippine territory is already allowed under the VFA 2. United States military personnel shall be exempt from
passport and visa regulations upon entering and departing the
We shall first deal with the recognition under EDCA of the Philippines.
presence in the country of three distinct classes of individuals
who will be conducting different types of activities within the 3. The following documents only, which shall be required in
Agreed Locations: (1) U.S. military personnel; (2) U.S. civilian respect of United States military personnel who enter the
personnel; and (3) U.S. contractors. The agreement refers to Philippines; xx xx.
them as follows:
4. United States civilian personnel shall be exempt from visa
"United States personnel" means United States military and requirements but shall present, upon demand, valid passports
civilian personnel temporarily in the territory of the Philippines upon entry and departure of the Philippines. (Emphases
in connection with activities approved by the Philippines, as Supplied)
those terms are defined in the VFA.252
By virtue of Articles I and III of the VFA, the Philippines already
"United States forces" means the entity comprising United allows U.S. military and civilian personnel to be "temporarily
States personnel and all property, equipment, and materiel of in the Philippines," so long as their presence is "in connection
the United States Armed Forces present in the territory of the with activities approved by the Philippine Government." The
Philippines.253 Philippines, through Article III, even guarantees that it shall
facilitate the admission of U.S. personnel into the country and
"United States contractors" means companies and firms, and grant exemptions from passport and visa regulations. The
their employees, under contract or subcontract to or on behalf VFA does not even limit their temporary presence to specific
of the United States Department of Defense. United States locations.
contractors are not included as part of the definition of United
States personnel in this Agreement, including within the Based on the above provisions, the admission and presence
context of the VFA.254 of U.S. military and civilian personnel in Philippine territory are
already allowed under the VFA, the treaty supposedly being
United States forces may contract for any materiel, supplies, implemented by EDCA. What EDCA has effectively done, in
equipment, and services (including construction) to be fact, is merely provide the mechanism to identify the locations
furnished or undertaken in the territory of the Philippines in which U.S. personnel may perform allowed activities
without restriction as to choice of contractor, supplier, or pursuant to the VFA. As the implementing agreement, it
person who provides such materiel, supplies, equipment, or regulates and limits the presence of U.S. personnel in the
services. Such contracts shall be solicited, awarded, and country.
administered in accordance with the laws and regulations of
the United States.255 (Emphases Supplied) b. EDCA does not provide the legal basis for admission of U.S.
contractors into Philippine territory; their entry must be
A thorough evaluation of how EDCA is phrased clarities that sourced from extraneous Philippine statutes and regulations
the agreement does not deal with the entry into the country of for the admission of alien employees or business persons.
U.S. personnel and contractors per se. While Articles
I(l)(b)256 and II(4)257 speak of "the right to access and use" Of the three aforementioned classes of individuals who will be
the Agreed Locations, their wordings indicate the presumption conducting certain activities within the Agreed Locations, we
that these groups have already been allowed entry into note that only U.S. contractors are not explicitly mentioned in
Philippine territory, for which, unlike the VFA, EDCA has no the VFA. This does not mean, though, that the recognition of
specific provision. Instead, Article II of the latter simply alludes their presence under EDCA is ipso facto an amendment of the
FINALS CONSTITUTIONAL LAW I ACJUCO NOV 11, 2017 18

treaty, and that there must be Senate concurrence before they


are allowed to enter the country. Noting that from time to time elements of the United States
armed forces may visit the Republic of the Philippines;
Nowhere in EDCA are U.S. contractors guaranteed immediate
admission into the Philippines. Articles III and IV, in fact, Considering that cooperation between the United States and
merely grant them the right of access to, and the authority to the Republic of the Philippines promotes their common
conduct certain activities within the Agreed Locations. Since security interests;
Article II(3) of EDCA specifically leaves out U.S. contractors
from the coverage of the VFA, they shall not be granted the xxx
same entry accommodations and privileges as those enjoyed
by U.S. military and civilian personnel under the VFA. Article I - Definitions

Consequently, it is neither mandatory nor obligatory on the As used in this Agreement, "United States personnel" means
part of the Philippines to admit U.S. contractors into the United States military and civilian personnel temporarily in the
country.259 We emphasize that the admission of aliens into Philippines in connection with activities approved by the
Philippine territory is "a matter of pure permission and simple Philippine Government. Within this definition: xx x
tolerance which creates no obligation on the part of the
government to permit them to stay."260 Unlike U.S. personnel Article II - Respect for Law
who are accorded entry accommodations, U.S. contractors
are subject to Philippine immigration laws.261 The latter must It is the duty of United States personnel to respect the laws of
comply with our visa and passport regulations262 and prove the Republic of the Philippines and to abstain from any activity
that they are not subject to exclusion under any provision of inconsistent with the spirit of this agreement, and, in particular,
Philippine immigration laws.263 The President may also deny from any political activity in the Philippines. The Government
them entry pursuant to his absolute and unqualified power to of the United States shall take all measures within its authority
prohibit or prevent the admission of aliens whose presence in to ensure that this is done.
the country would be inimical to public interest.264
Article VII - Importation and Exportation
In the same vein, the President may exercise the plenary
power to expel or deport U.S. contractors265 as may be 1. United States Government equipment, materials, supplies,
necessitated by national security, public safety, public health, and other property imported into or acquired in the Philippines
public morals, and national interest.266 They may also be by or on behalf of the United States armed forces in
deported if they are found to be illegal or undesirable aliens connection with activities to which this agreement applies,
pursuant to the Philippine Immigration Act267 and the Data shall be free of all Philippine duties, taxes and other similar
Privacy Act.268 In contrast, Article 111(5) of the VFA requires charges. Title to such property shall remain with the United
a request for removal from the Philippine government before States, which may remove such property from the Philippines
a member of the U.S. personnel may be "dispos[ed] xx x at any time, free from export duties, taxes, and other similar
outside of the Philippines." charges. x x x.

c. Authorized activities of U.S. military and civilian personnel Article VIII - Movement of Vessels and Aircraft
within Philippine territory are in furtherance of the MDT and
the VFA 1. Aircraft operated by or for the United States armed forces
may enter the Philippines upon approval of the Government
We begin our analysis by quoting the relevant sections of the of the Philippines in accordance with procedures stipulated in
MDT and the VFA that pertain to the activities in which U.S. implementing arrangements.
military and civilian personnel may engage:
2. Vessels operated by or for the United States armed forces
MUTUAL DEFENSE TREATY may enter the Philippines upon approval of the Government
of the Philippines. The movement of vessels shall be in
Article II accordance with international custom and practice governing
such vessels, and such agreed implementing arrangements
In order more effectively to achieve the objective of this as necessary. x x x (Emphases Supplied)
Treaty, the Parties separately and jointly by self-help and
mutual aid will maintain and develop their individual and Manifest in these provisions is the abundance of references to
collective capacity to resist armed attack. the creation of further "implementing arrangements" including
the identification of "activities [to be] approved by the
Article III Philippine Government." To determine the parameters of
these implementing arrangements and activities, we referred
The Parties, through their Foreign Ministers or their deputies, to the content, purpose, and framework of the MDT and the
will consult together from time to time regarding the VFA.
implementation of this Treaty and whenever in the opinion of
either of them the territorial integrity, political independence or By its very language, the MDT contemplates a situation in
security of either of the Parties is threatened by external which both countries shall engage in joint activities, so that
armed attack in the Pacific. they can maintain and develop their defense capabilities. The
wording itself evidently invites a reasonable construction that
VISITING FORCES AGREEMENT the joint activities shall involve joint military trainings,
maneuvers, and exercises. Both the interpretation269 and the
Preamble subsequent practice270 of the parties show that the MDT
independently allows joint military exercises in the country.
xxx Lim v. Executive Secretary271 and Nicolas v. Romulo272
recognized that Balikatan exercises, which are activities that
Reaffirming their obligations under the Mutual Defense Treaty seek to enhance and develop the strategic and technological
of August 30, 1951; capabilities of the parties to resist an armed attack, "fall
FINALS CONSTITUTIONAL LAW I ACJUCO NOV 11, 2017 19

squarely under the provisions of the RP-US MDT."273 In Lim, Under these auspices, the VFA gives legitimacy to the current
the Court especially noted that the Philippines and the U.S. Balikatan exercises. It is only logical to assume that "Balikatan
continued to conduct joint military exercises even after the 02-1," a "mutual anti-terrorism advising, assisting and training
expiration of the MBA and even before the conclusion of the exercise," falls under the umbrella of sanctioned or allowable
VFA.274 These activities presumably related to the Status of activities in the context of the agreement. Both the history and
Forces Agreement, in which the parties agreed on the status intent of the Mutual Defense Treaty and the VFA support the
to be accorded to U.S. military and civilian personnel while conclusion that combat-related activities - as opposed to
conducting activities in the Philippines in relation to the combat itself- such as the one subject of the instant petition,
MDT.275 are indeed authorized. (Emphases Supplied)

Further, it can be logically inferred from Article V of the MDT The joint report of the Senate committees on foreign relations
that these joint activities may be conducted on Philippine or and on national defense and security further explains the wide
on U.S. soil. The article expressly provides that the term range and variety of activities contemplated in the VFA, and
armed attack includes "an armed attack on the metropolitan how these activities shall be identified:277
territory of either of the Parties, or on the island territories
under its jurisdiction in the Pacific or on its armed forces, These joint exercises envisioned in the VFA are not limited to
public vessels or aircraft in the Pacific." Surely, in maintaining combat-related activities; they have a wide range and variety.
and developing our defense capabilities, an assessment or They include exercises that will reinforce the AFP's ability to
training will need to be performed, separately and jointly by acquire new techniques of patrol and surveillance to protect
self-help and mutual aid, in the territories of the contracting the country's maritime resources; sea-search and rescue
parties. It is reasonable to conclude that the assessment of operations to assist ships in distress; and disaster-relief
defense capabilities would entail understanding the terrain, operations to aid the civilian victims of natural calamities, such
wind flow patterns, and other environmental factors unique to as earthquakes, typhoons and tidal waves.
the Philippines.
xxxx
It would also be reasonable to conclude that a simulation of
how to respond to attacks in vulnerable areas would be part Joint activities under the VFA will include combat maneuvers;
of the training of the parties to maintain and develop their training in aircraft maintenance and equipment repair; civic-
capacity to resist an actual armed attack and to test and action projects; and consultations and meetings of the
validate the defense plan of the Philippines. It is likewise Philippine-U.S. Mutual Defense Board. It is at the level of the
reasonable to imagine that part of the training would involve Mutual Defense Board-which is headed jointly by the Chief of
an analysis of the effect of the weapons that may be used and Staff of the AFP and the Commander in Chief of the U.S.
how to be prepared for the eventuality. This Court recognizes Pacific Command-that the VFA exercises are planned. Final
that all of this may require training in the area where an armed approval of any activity involving U.S. forces is, however,
attack might be directed at the Philippine territory. invariably given by the Philippine Government.

The provisions of the MDT must then be read in conjunction xxxx


with those of the VFA.
Siazon clarified that it is not the VFA by itself that determines
Article I of the VFA indicates that the presence of U.S. military what activities will be conducted between the armed forces of
and civilian personnel in the Philippines is "in connection with the U.S. and the Philippines. The VFA regulates and provides
activities approved by the Philippine Government." While the the legal framework for the presence, conduct and legal status
treaty does not expressly enumerate or detail the nature of of U.S. personnel while they are in the country for visits, joint
activities of U.S. troops in the country, its Preamble makes exercises and other related activities. (Emphases Supplied)
explicit references to the reaffirmation of the obligations of
both countries under the MDT. These obligations include the What can be gleaned from the provisions of the VFA, the joint
strengthening of international and regional security in the report of the Senate committees on foreign relations and on
Pacific area and the promotion of common security interests. national defense and security, and the ruling of this Court in
Lim is that the "activities" referred to in the treaty are meant to
The Court has already settled in Lim v. Executive Secretary be specified and identified infurther agreements. EDCA is one
that the phrase "activities approved by the Philippine such agreement.
Government" under Article I of the VFA was intended to be
ambiguous in order to afford the parties flexibility to adjust the EDCA seeks to be an instrument that enumerates the
details of the purpose of the visit of U.S. personnel.276 In Philippine-approved activities of U.S. personnel referred to in
ruling that the Terms of Reference for the Balikatan Exercises the VFA. EDCA allows U.S. military and civilian personnel to
in 2002 fell within the context of the treaty, this Court perform "activities approved by the Philippines, as those terms
explained: are defined in the VFA"278 and clarifies that these activities
include those conducted within the Agreed Locations:
After studied reflection, it appeared farfetched that the
ambiguity surrounding the meaning of the word "activities" 1. Security cooperation exercises; joint and combined training
arose from accident. In our view, it was deliberately made that activities; humanitarian assistance and disaster relief
way to give both parties a certain leeway in negotiation. In this activities; and such other activities as may be agreed upon by
manner, visiting US forces may sojourn in Philippine territory the Parties279
for purposes other than military. As conceived, the joint
exercises may include training on new techniques of patrol 2. Training; transit; support and related activities; refueling of
and surveillance to protect the nation's marine resources, sea aircraft; bunkering of vessels; temporary maintenance of
search-and-rescue operations to assist vessels in distress, vehicles, vessels, and aircraft; temporary accommodation of
disaster relief operations, civic action projects such as the personnel; communications; prepositioning of equipment,
building of school houses, medical and humanitarian supplies, and materiel; deployment of forces and materiel; and
missions, and the like. such other activities as the Parties may agree280
FINALS CONSTITUTIONAL LAW I ACJUCO NOV 11, 2017 20

3. Exercise of operational control over the Agreed Locations These terms of Reference are for purposes of this Exercise
for construction activities and other types of activity, including only and do not create additional legal obligations between the
alterations and improvements thereof281 US Government and the Republic of the Philippines.

4. Exercise of all rights and authorities within the Agreed II. EXERCISE LEVEL
Locations that are necessary for their operational control or
defense, including the adoption of apfropriate measures to 1. TRAINING
protect U.S. forces and contractors282
a. The Exercise shall involve the conduct of mutual military
5. Use of water, electricity, and other public utilities283 assisting, advising and training of RP and US Forces with the
primary objective of enhancing the operational capabilities of
6. Operation of their own telecommunication systems, both forces to combat terrorism.
including the utilization of such means and services as are
required to ensure the full ability to operate telecommunication b. At no time shall US Forces operate independently within RP
systems, as well as the use of the necessary radio spectrum territory.
allocated for this purpose284
c. Flight plans of all aircraft involved in the exercise will comply
According to Article I of EDCA, one of the purposes of these with the local air traffic regulations.
activities is to maintain and develop, jointly and by mutual aid,
the individual and collective capacities of both countries to 2. ADMINISTRATION & LOGISTICS
resist an armed attack. It further states that the activities are
in furtherance of the MDT and within the context of the VFA. xxxx

We note that these planned activities are very similar to those a. RP and US participating forces may share, in accordance
under the Terms of Reference285 mentioned in Lim. Both with their respective laws and regulations, in the use of their
EDCA and the Terms of Reference authorize the U.S. to resources, equipment and other assets. They will use their
perform the following: (a) participate in training exercises; (b) respective logistics channels. x x x. (Emphases Supplied)
retain command over their forces; (c) establish temporary
structures in the country; (d) share in the use of their After a thorough examination of the content, purpose, and
respective resources, equipment and other assets; and (e) framework of the MDT and the VFA, we find that EDCA has
exercise their right to self-defense. We quote the relevant remained within the parameters set in these two treaties. Just
portion of the Terms and Conditions as follows:286 like the Terms of Reference mentioned in Lim, mere
adjustments in detail to implement the MDT and the VFA can
I. POLICY LEVEL be in the form of executive agreements.

xxxx Petitioners assert287 that the duration of the activities


mentioned in EDCA is no longer consistent with the temporary
No permanent US basing and support facilities shall be nature of the visits as contemplated in the VFA. They point out
established. Temporary structures such as those for troop that Article XII(4) of EDCA has an initial term of 10 years, a
billeting, classroom instruction and messing may be set up for term automatically renewed unless the Philippines or the U.S.
use by RP and US Forces during the Exercise. terminates the agreement. According to petitioners, such
length of time already has a badge of permanency.
The Exercise shall be implemented jointly by RP and US
Exercise Co-Directors under the authority of the Chief of Staff, In connection with this, Justice Teresita J. Leonardo-De
AFP. In no instance will US Forces operate independently Castro likewise argues in her Concurring and Dissenting
during field training exercises (FTX). AFP and US Unit Opinion that the VFA contemplated mere temporary visits
Commanders will retain command over their respective forces from U.S. forces, whereas EDCA allows an unlimited period
under the overall authority of the Exercise Co-Directors. RP for U.S. forces to stay in the Philippines.288
and US participants shall comply with operational instructions
of the AFP during the FTX. However, the provisions of EDCA directly contradict this
argument by limiting itself to 10 years of effectivity. Although
The exercise shall be conducted and completed within a this term is automatically renewed, the process for terminating
period of not more than six months, with the projected the agreement is unilateral and the right to do so automatically
participation of 660 US personnel and 3,800 RP Forces. The accrues at the end of the 10 year period. Clearly, this method
Chief of Staff, AFP shall direct the Exercise Co-Directors to does not create a permanent obligation.
wind up and terminate the Exercise and other activities within
the six month Exercise period. Drawing on the reasoning in Lim, we also believe that it could
not have been by chance that the VFA does not include a
The Exercise is a mutual counter-terrorism advising, assisting maximum time limit with respect to the presence of U.S.
and training Exercise relative to Philippine efforts against the personnel in the country. We construe this lack of specificity
ASG, and will be conducted on the Island of Basilan. Further as a deliberate effort on the part of the Philippine and the U.S.
advising, assisting and training exercises shall be conducted governments to leave out this aspect and reserve it for the
in Malagutay and the Zamboanga area. Related activities in "adjustment in detail" stage of the implementation of the
Cebu will be for support of the Exercise. treaty. We interpret the subsequent, unconditional
concurrence of the Senate in the entire text of the VFA as an
xx xx. implicit grant to the President of a margin of appreciation in
determining the duration of the "temporary" presence of U.S.
US exercise participants shall not engage in combat, without personnel in the country.
prejudice to their right of self-defense.
Justice Brion's dissent argues that the presence of U.S. forces
under EDCA is "more permanent" in nature.289 However, this
argument has not taken root by virtue of a simple glance at its
FINALS CONSTITUTIONAL LAW I ACJUCO NOV 11, 2017 21

provisions on the effectivity period. EDCA does not grant a geopolitical prognosis of situations affecting the country, a
permanent bases, but rather temporary rotational access to belief is engendered that a much longer period of military
facilities for efficiency. As Professor Aileen S.P. Baviera notes: training is needed, the President must be given ample
discretion to adopt necessary measures including the
The new EDCA would grant American troops, ships and flexibility to set an extended timetable.
planes rotational access to facilities of the Armed Forces of
the Philippines – but not permanent bases which are Due to the sensitivity and often strict confidentiality of these
prohibited under the Philippine Constitution - with the result of concerns, we acknowledge that the President may not always
reducing response time should an external threat from a be able to candidly and openly discuss the complete situation
common adversary crystallize.290 being faced by the nation. The Chief Executive's hands must
not be unduly tied, especially if the situation calls for crafting
EDCA is far from being permanent in nature compared to the programs and setting timelines for approved activities. These
practice of states as shown in other defense cooperation activities may be necessary for maintaining and developing
agreements. For example, Article XIV(l) of the U.S.-Romania our capacity to resist an armed attack, ensuring our national
defense agreement provides the following: sovereignty and territorial integrity, and securing our national
interests. If the Senate decides that the President is in the best
This Agreement is concluded for an indefinite period and shall position to define in operational terms the meaning of
enter into force in accordance with the internal laws of each temporary in relation to the visits, considered individually or in
Party x x x. (emphasis supplied) their totality, the Court must respect that policy decision. If the
Senate feels that there is no need to set a time limit to these
Likewise, Article 36(2) of the US-Poland Status of Forces visits, neither should we.
Agreement reads:
Evidently, the fact that the VFA does not provide specificity in
This Agreement has been concluded for an indefinite period regard to the extent of the "temporary" nature of the visits of
of time. It may be terminated by written notification by either U.S. personnel does not suggest that the duration to which the
Party and in that event it terminates 2 years after the receipt President may agree is unlimited. Instead, the boundaries of
of the notification. (emphasis supplied) the meaning of the term temporary in Article I of the treaty
must be measured depending on the purpose of each visit or
Section VIII of US.-Denmark Mutual Support Agreement activity.295 That purpose must be analyzed on a case-by-
similarly provides: case basis depending on the factual circumstances
surrounding the conclusion of the implementing agreement.
8.1 This Agreement, which consists of a Preamble, While the validity of the President's actions will be judged
SECTIONs I-VIII, and Annexes A and B, shall become under less stringent standards, the power of this Court to
effective on the date of the last signature affixed below and determine whether there was grave abuse of discretion
shall remain in force until terminated by the Parties, provided remains unimpaired.
that it may be terminated by either Party upon 180 days written
notice of its intention to do so to the other Party. (emphasis d. Authorized activities performed by US. contractors within
supplied) Philippine territory - who were legitimately permitted to enter
the country independent of EDCA - are subject to relevant
On the other hand, Article XXI(3) of the US.-Australia Force Philippine statutes and regulations and must be consistent
Posture Agreement provides a longer initial term: with the MDT and the VFA

3. This Agreement shall have an initial term of 25 years and Petitioners also raise296 concerns about the U.S.
thereafter shall continue in force, but may be terminated by government's purported practice of hiring private security
either Party at any time upon one year's written notice to the contractors in other countries. They claim that these
other Party through diplomatic channels. (emphasis supplied) contractors - one of which has already been operating in
Mindanao since 2004 - have been implicated in incidents or
The phrasing in EDCA is similar to that in the U.S.-Australia scandals in other parts of the globe involving rendition, torture
treaty but with a term less than half of that is provided in the and other human rights violations. They also assert that these
latter agreement. This means that EDCA merely follows the contractors employ paramilitary forces in other countries
practice of other states in not specifying a non-extendible where they are operating.
maximum term. This practice, however, does not
automatically grant a badge of permanency to its terms. Article Under Articles III and IV of EDCA, U.S. contractors are
XII(4) of EDCA provides very clearly, in fact, that its effectivity authorized to perform only the following activities:
is for an initial term of 10 years, which is far shorter than the
terms of effectivity between the U.S. and other states. It is 1. Training; transit; support and related activities; refueling of
simply illogical to conclude that the initial, extendible term of aircraft; bunkering of vessels; temporary maintenance of
10 years somehow gives EDCA provisions a permanent vehicles, vessels, and aircraft; temporary accommodation of
character. personnel; communications; prepositioning of equipment,
supplies, and materiel; deployment of forces and materiel; and
The reasoning behind this interpretation is rooted in the such other activities as the Parties may agree297
constitutional role of the President who, as Commander-in-
Chief of our armed forces, is the principal strategist of the 2. Prepositioning and storage of defense equipment, supplies,
nation and, as such, duty-bound to defend our national and materiel, including delivery, management, inspection,
sovereignty and territorial integrity;291 who, as chief architect use, maintenance, and removal of such equipment, supplies
of our foreign relations, is the head policymaker tasked to and materiel298
assess, ensure, and protect our national security and
interests;292 who holds the most comprehensive and most 3. Carrying out of matters in accordance with, and to the
confidential information about foreign countries293 that may extent permissible under, U.S. laws, regulations, and
affect how we conduct our external affairs; and who has policies299
unrestricted access to highly classified military intelligence
data294 that may threaten the life of the nation. Thus, if after
FINALS CONSTITUTIONAL LAW I ACJUCO NOV 11, 2017 22

EDCA requires that all activities within Philippine territory be engage in illegal or undesirable activities. There is nothing that
in accordance with Philippine law. This means that certain prevents them from being detained in the country or being
privileges denied to aliens are likewise denied to foreign subject to the jurisdiction of our courts. Our penal laws,309
military contractors. Relevantly, providing security300 and labor laws,310 and immigrations laws311 apply to them and
carrying, owning, and possessing firearms301 are illegal for therefore limit their activities here. Until and unless there is
foreign civilians. another law or treaty that specifically deals with their entry and
activities, their presence in the country is subject to unqualified
The laws in place already address issues regarding the Philippine jurisdiction.
regulation of contractors. In the 2015 Foreign Investment
Negative list,302 the Executive Department has already EDCA does not allow the presence of U.S.-owned or -
identified corporations that have equity restrictions in controlled military facilities and bases in the Philippines
Philippine jurisdiction. Of note is No. 5 on the list - private
security agencies that cannot have any foreign equity by virtue Petitioners Saguisag et al. claim that EDCA permits the
of Section 4 of Republic Act No. 5487;303 and No. 15, which establishment of U.S. military bases through the
regulates contracts for the construction of defense-related "euphemistically" termed "Agreed Locations. "312 Alluding to
structures based on Commonwealth Act No. 541. the definition of this term in Article II(4) of EDCA, they point
out that these locations are actually military bases, as the
Hence, any other entity brought into the Philippines by virtue definition refers to facilities and areas to which U.S. military
of EDCA must subscribe to corporate and civil requirements forces have access for a variety of purposes. Petitioners claim
imposed by the law, depending on the entity's corporate that there are several badges of exclusivity in the use of the
structure and the nature of its business. Agreed Locations by U.S. forces. First, Article V(2) of EDCA
alludes to a "return" of these areas once they are no longer
That Philippine laws extraneous to EDCA shall govern the needed by U.S. forces, indicating that there would be some
regulation of the activities of U.S. contractors has been clear transfer of use. Second, Article IV(4) ofEDCA talks about
even to some of the present members of the Senate. American forces' unimpeded access to the Agreed Locations
for all matters relating to the prepositioning and storage of
For instance, in 2012, a U.S. Navy contractor, the Glenn U.S. military equipment, supplies, and materiel. Third, Article
Marine, was accused of spilling fuel in the waters off Manila VII of EDCA authorizes U.S. forces to use public utilities and
Bay.304 The Senate Committee on Foreign Relations and the to operate their own telecommunications system.
Senate Committee on Environment and Natural Resources
chairperson claimed environmental and procedural violations a. Preliminary point on badges of exclusivity
by the contractor.305 The U.S. Navy investigated the
contractor and promised stricter guidelines to be imposed As a preliminary observation, petitioners have cherry-picked
upon its contractors.306 The statement attributed to provisions of EDCA by presenting so-called "badges of
Commander Ron Steiner of the public affairs office of the U.S. exclusivity," despite the presence of contrary provisions within
Navy's 7th Fleet - that U.S. Navy contractors are bound by the text of the agreement itself.
Philippine laws - is of particular relevance. The statement
acknowledges not just the presence of the contractors, but First, they clarify the word "return" in Article V(2) of EDCA.
also the U.S. position that these contractors are bound by the However, the use of the word "return" is within the context of
local laws of their host state. This stance was echoed by other a lengthy provision. The provision as a whole reads as follows:
U.S. Navy representatives.307
The United States shall return to the Philippines any Agreed
This incident simply shows that the Senate was well aware of Locations, or any portion thereof, including non-relocatable
the presence of U.S. contractors for the purpose of fulfilling structures and assemblies constructed, modified, or improved
the terms of the VFA. That they are bound by Philippine law is by the United States, once no longer required by United States
clear to all, even to the U.S. forces for activities under this Agreement. The Parties or the
Designated Authorities shall consult regarding the terms of
As applied to EDCA, even when U.S. contractors are granted return of any Agreed Locations, including possible
access to the Agreed Locations, all their activities must be compensation for improvements or construction.
consistent with Philippine laws and regulations and pursuant
to the MDT and the VFA. The context of use is "required by United States forces for
activities under this Agreement." Therefore, the return of an
While we recognize the concerns of petitioners, they do not Agreed Location would be within the parameters of an activity
give the Court enough justification to strike down EDCA. In that the Mutual Defense Board (MDB) and the Security
Lim v. Executive Secretary, we have already explained that Engagement Board (SEB) would authorize. Thus, possession
we cannot take judicial notice of claims aired in news reports, by the U.S. prior to its return of the Agreed Location would be
"not because of any issue as to their truth, accuracy, or based on the authority given to it by a joint body co-chaired by
impartiality, but for the simple reason that facts must be the "AFP Chief of Staff and Commander, U.S. PACOM with
established in accordance with the rules of evidence."308 representatives from the Philippines' Department of National
What is more, we cannot move one step ahead and speculate Defense and Department of Foreign Affairs sitting as
that the alleged illegal activities of these contractors in other members."313 The terms shall be negotiated by both the
countries would take place in the Philippines with certainty. As Philippines and the U.S., or through their Designated
can be seen from the above discussion, making sure that U.S. Authorities. This provision, seen as a whole, contradicts
contractors comply with Philippine laws is a function of law petitioners' interpretation of the return as a "badge of
enforcement. EDCA does not stand in the way of law exclusivity." In fact, it shows the cooperation and partnership
enforcement. aspect of EDCA in full bloom.

Nevertheless, we emphasize that U.S. contractors are Second, the term "unimpeded access" must likewise be
explicitly excluded from the coverage of the VFA. As visiting viewed from a contextual perspective. Article IV(4) states that
aliens, their entry, presence, and activities are subject to all U.S. forces and U.S. contractors shall have "unimpeded
laws and treaties applicable within the Philippine territory. access to Agreed Locations for all matters relating to the
They may be refused entry or expelled from the country if they prepositioning and storage of defense equipment, supplies,
FINALS CONSTITUTIONAL LAW I ACJUCO NOV 11, 2017 23

and materiel, including delivery, management, inspection, the U.S. retains ownership if it paid for the facility.325 Under
use, maintenance, and removal of such equipment, supplies EDCA, an immovable is owned by the Philippines, even if built
and materiel." completely on the back of U.S. funding.326 This is consistent
with the constitutional prohibition on foreign land
At the beginning of Article IV, EDCA states that the Philippines ownership.327
gives the U.S. the authority to bring in these equipment,
supplies, and materiel through the MDB and SEB security Despite the apparent similarity, the ownership of property is
mechanism. These items are owned by the U.S.,314 are but a part of a larger whole that must be considered before the
exclusively for the use of the U.S.315 and, after going through constitutional restriction is violated. Thus, petitioners' points
the joint consent mechanisms of the MDB and the SEB, are on operational control will be given more attention in the
within the control of the U.S.316 More importantly, before discussion below. The arguments on policy are, however,
these items are considered prepositioned, they must have outside the scope of judicial review and will not be discussed
gone through the process of prior authorization by the MDB
and the SEB and given proper notification to the AFP.317 Moreover, a direct comparison of the MBA and EDCA will
result in several important distinctions that would allay
Therefore, this "unimpeded access" to the Agreed Locations suspicion that EDCA is but a disguised version of the MBA.
is a necessary adjunct to the ownership, use, and control of
the U.S. over its own equipment, supplies, and materiel and b. There are substantial matters that the US. cannot do under
must have first been allowed by the joint mechanisms in play EDCA, but which it was authorized to do under the 1947 MBA
between the two states since the time of the MDT and the
VFA. It is not the use of the Agreed Locations that is exclusive The Philippine experience with U.S. military bases under the
per se; it is mere access to items in order to exercise the rights 1947 MBA is simply not possible under EDCA for a number of
of ownership granted by virtue of the Philippine Civil Code.318 important reasons.

As for the view that EDCA authorizes U.S. forces to use public First, in the 1947 MBA, the U.S. retained all rights of
utilities and to operate their own telecommunications system, jurisdiction in and over Philippine territory occupied by
it will be met and answered in part D, infra. American bases. In contrast, the U.S. under EDCA does not
enjoy any such right over any part of the Philippines in which
Petitioners also point out319 that EDCA is strongly its forces or equipment may be found. Below is a comparative
reminiscent of and in fact bears a one-to-one correspondence table between the old treaty and EDCA:
with the provisions of the 1947 MBA. They assert that both
agreements (a) allow similar activities within the area; (b) 1947 MBA/ 1946 Treaty of General Relations EDCA
provide for the same "species of ownership" over facilities; 1947 MBA, Art. I(1):
and (c) grant operational control over the entire area. Finally,
they argue320 that EDCA is in fact an implementation of the The Government of the Republic of the Philippines
new defense policy of the U.S. According to them, this policy (hereinafter referred to as the Philippines) grants to the
was not what was originally intended either by the MDT or by Government of the United States of America (hereinafter
the VFA. referred to as the United States) the right to retain the use of
the bases in the Philippines listed in Annex A attached hereto.
On these points, the Court is not persuaded.
1947 MBA, Art. XVII(2):
The similar activities cited by petitioners321 simply show that
under the MBA, the U.S. had the right to construct, operate, All buildings and structures which are erected by the United
maintain, utilize, occupy, garrison, and control the bases. The States in the bases shall be the property of the United States
so-called parallel provisions of EDCA allow only operational and may be removed by it before the expiration of this
control over the Agreed Locations specifically for construction Agreement or the earlier relinquishment of the base on which
activities. They do not allow the overarching power to operate, the structures are situated. There shall be no obligation on the
maintain, utilize, occupy, garrison, and control a base with full part of the Philippines or of the United States to rebuild or
discretion. EDCA in fact limits the rights of the U.S. in respect repair any destruction or damage inflicted from any cause
of every activity, including construction, by giving the MDB and whatsoever on any of the said buildings or structures owned
the SEB the power to determine the details of all activities or used by the United States in the bases. x x x x.
such as, but not limited to, operation, maintenance, utility,
occupancy, garrisoning, and control.322 1946 Treaty of Gen. Relations, Art. I:

The "species of ownership" on the other hand, is distinguished The United States of America agrees to withdraw and
by the nature of the property. For immovable property surrender, and does hereby withdraw and surrender, all rights
constructed or developed by the U.S., EDCA expresses that of possession, supervision, jurisdiction, control or sovereignty
ownership will automatically be vested to the Philippines.323 existing and exercised by the United States of America in and
On the other hand, for movable properties brought into the over the territory and the people of the Philippine Islands,
Philippines by the U.S., EDCA provides that ownership is except the use of such bases, necessary appurtenances to
retained by the latter. In contrast, the MBA dictates that the such bases, and the rights incident thereto, as the United
U.S. retains ownership over immovable and movable States of America, by agreement with the Republic of the
properties. Philippines may deem necessary to retain for the mutual
protection of the Republic of the Philippines and of the United
To our mind, both EDCA and the MBA simply incorporate what States of America. x x x.
is already the law of the land in the Philippines. The Civil
Code's provisions on ownership, as applied, grant the owner EDCA, preamble:
of a movable property full rights over that property, even if
located in another person's property.324 Affirming that the Parties share an understanding for the
United States not to establish a permanent military presence
The parallelism, however, ends when the situation involves or base in the territory of the Philippines;
facilities that can be considered immovable. Under the MBA,
FINALS CONSTITUTIONAL LAW I ACJUCO NOV 11, 2017 24

xxxx and with full respect for the Philippine Constitution and
Philippine laws;
Recognizing that all United States access to and use of
facilities and areas will be at the invitation of the Philippines xxxx
and with full respect for the Philippine Constitution and
Philippine laws; EDCA. Art. II(4):

xxxx "Agreed Locations" means facilities and areas that are


provided by the Government of the Philippines through the
EDCA, Art. II(4): AFP and that United States forces, United States contractors,
and others as mutually agreed, shall have the right to access
"Agreed Locations" means facilities and areas that are and use pursuant to this Agreement. Such Agreed Locations
provided by the Government of the Philippines through the may be listed in an annex to be appended to this Agreement,
AFP and that United States forces, United States contractors, and may be further described in implementing arrangements.
and others as mutually agreed, shall have the right to access
and use pursuant to this Agreement. Such Agreed Locations Third, in EDCA, the Philippines is- guaranteed access over
may be listed in an annex to be appended to this Agreement, the entire area of the Agreed Locations. On the other hand,
and may be further described in implementing arrangements. given that the U.S. had complete control over its military bases
under the 1947 MBA, the treaty did not provide for any
EDCA, Art. V: express recognition of the right of access of Philippine
authorities. Without that provision and in light of the retention
1. The Philippines shall retain ownership of and title to Agreed of U.S. sovereignty over the old military bases, the U.S. could
Locations. effectively prevent Philippine authorities from entering those
bases.
xxxx
1947 MBA EDCA
4. All buildings, non-relocatable structures, and assemblies No equivalent provision.
affixed to the land in the Agreed Locations, including ones
altered or improved by United States forces, remain the EDCA, Art. III(5):
property of the Philippines. Permanent buildings constructed
by United States forces become the property of the The Philippine Designated Authority and its authorized
Philippines, once constructed, but shall be used by United representative shall have access to the entire area of the
States forces until no longer required by United States forces. Agreed Locations. Such access shall be provided promptly
consistent with operational safety and security requirements
in accordance with agreed procedures developed by the
Parties.
Second, in the bases agreement, the U.S. and the Philippines
were visibly not on equal footing when it came to deciding Fourth, in the bases agreement, the U.S. retained the right,
whether to expand or to increase the number of bases, as the power, and authority over the establishment, use, operation,
Philippines may be compelled to negotiate with the U.S. the defense, and control of military bases, including the limits of
moment the latter requested an expansion of the existing territorial waters and air space adjacent to or in the vicinity of
bases or to acquire additional bases. In EDCA, U.S. access is those bases. The only standard used in determining the extent
purely at the invitation of the Philippines. of its control was military necessity. On the other hand, there
is no such grant of power or authority under EDCA. It merely
1947 MBA/ 1946 Treaty of General Relations EDCA allows the U.S. to exercise operational control over the
1947 MBA, Art.I(3): construction of Philippine-owned structures and facilities:

The Philippines agree to enter into negotiations with the 1947 MBA EDCA
United States at the latter's request, to permit the United 1947 MBA, Art.I(2):
States to expand such bases, to exchange such bases for
other bases, to acquire additional bases, or relinquish rights The Philippines agrees to permit the United States, upon
to bases, as any of such exigencies may be required by notice to the Philippines, to use such of those bases listed in
military necessity. Annex B as the United States determines to be required by
military necessity.
1946 Treaty of Gen. Relations, Art. I:
1947 MBA, Art. III(1):
The United States of America agrees to withdraw and
surrender, and does hereby withdraw and surrender, all rights It is mutually agreed that the United States shall have the
of possession, supervision, jurisdiction, control or sovereignty rights, power and authority within the bases which are
existing and exercised by the United States of America in and necessary for the establishment, use, operation and defense
over the territory and the people of the Philippine Islands, thereof or appropriate for the control thereof and all the rights,
except the use of such bases, necessary appurtenances to power and authority within the limits of territorial waters and
such bases, and the rights incident thereto, as the United air space adjacent to, or in the vicinity of, the bases which are
States of America, by agreement with the Republic of the necessary to provide access to them, or appropriate for their
Philippines may deem necessary to retain for the mutual control.
protection of the Republic of the Philippines and of the United
States of America. x x x. EDCA, Art. III(4):

EDCA, preamble: The Philippines hereby grants to the United States, through
bilateral security mechanisms, such as the MDB and SEB,
Recognizing that all United States access to and use of operational control of Agreed Locations for construction
facilities and areas will be at the invitation of the Philippines activities and authority to undertake such activities on, and
FINALS CONSTITUTIONAL LAW I ACJUCO NOV 11, 2017 25

make alterations and improvements to, Agreed Locations. roads and bridges that would afford it access to its military
United States forces shall consult on issues regarding such bases.
construction, alterations, and improvements based on the
Parties' shared intent that the technical requirements and 1947 MBA EDCA
construction standards of any such projects undertaken by or 1947 MBA, Art. III(2)(b):
on behalf of United States forces should be consistent with the
requirements and standards of both Parties. Such rights, power and authority shall include, inter alia, the
right, power and authority: x x x x to improve and deepen the
Fifth, the U.S. under the bases agreement was given the harbors, channels, entrances and anchorages, and to
authority to use Philippine territory for additional staging construct or maintain necessary roads and bridges affording
areas, bombing and gunnery ranges. No such right is given access to the bases.
under EDCA, as seen below:
EDCA, Art. III(2):
1947 MBA EDCA
1947 MBA, Art. VI: When requested, the Designated Authority of the Philippines
shall assist in facilitating transit or temporary access by United
The United States shall, subject to previous agreement with States forces to public land and facilities (including roads,
the Philippines, have the right to use land and coastal sea ports, and airfields), including those owned or controlled by
areas of appropriate size and location for periodic maneuvers, local governments, and to other land and facilities (including
for additional staging areas, bombing and gunnery ranges, roads, ports, and airfields).
and for such intermediate airfields as may be required for safe
and efficient air operations. Operations in such areas shall be Eighth, in the 1947 MBA, the U.S. was granted the automatic
carried on with due regard and safeguards for the public right to use any and all public utilities, services and facilities,
safety. airfields, ports, harbors, roads, highways, railroads, bridges,
viaducts, canals, lakes, rivers, and streams in the Philippines
1947 MBA, Art.I(2): in the same manner that Philippine military forces enjoyed that
right. No such arrangement appears in EDCA. In fact, it merely
The Philippines agrees to permit the United States, upon extends to U.S. forces temporary access to public land and
notice to the Philippines, to use such of those bases listed in facilities when requested:
Annex B as the United States determines to be required by
military necessity. 1947 MBA EDCA
1947 MBA, Art. VII:
EDCA, Art. III(1):
It is mutually agreed that the United States may employ and
With consideration of the views of the Parties, the Philippines use for United States military forces any and all public utilities,
hereby authorizes and agrees that United States forces, other services and facilities, airfields, ports, harbors, roads,
United States contractors, and vehicles, vessels, and aircraft highways, railroads, bridges, viaducts, canals, lakes, rivers
operated by or for United States forces may conduct the and streams in the Philippines under conditions no less
following activities with respect to Agreed Locations: training; favorable than those that may be applicable from time to time
transit; support and related activities; refueling of aircraft; to the military forces of the Philippines.
bunkering of vessels; temporary maintenance of vehicles,
vessels, and aircraft; temporary accommodation of personnel; EDCA, Art. III(2):
communications; prepositioning of equipment, supplies, and
materiel; deploying forces and materiel; and such other When requested, the Designated Authority of the Philippines
activities as the Parties may agree. shall assist in facilitating transit or temporary access by United
States forces to public land and facilities (including roads,
Sixth, under the MBA, the U.S. was given the right, power, and ports, and airfields), including those owned or controlled by
authority to control and prohibit the movement and operation local governments, and to other land and facilities (including
of all types of vehicles within the vicinity of the bases. The U.S. roads, ports, and airfields).
does not have any right, power, or authority to do so under
EDCA. Ninth, under EDCA, the U.S. no longer has the right, power,
and authority to construct, install, maintain, and employ any
1947 MBA EDCA type of facility, weapon, substance, device, vessel or vehicle,
1947 MBA, Art. 111(2)(c) or system unlike in the old treaty. EDCA merely grants the
U.S., through bilateral security mechanisms, the authority to
Such rights, power and authority shall include, inter alia, the undertake construction, alteration, or improvements on the
right, power and authority: x x x x to control (including the right Philippine-owned Agreed Locations.
to prohibit) in so far as may be required for the efficient
operation and safety of the bases, and within the limits of 1947 MBA EDCA
military necessity, anchorages, moorings, landings, takeoffs, 1947 MBA, Art. III(2)(e):
movements and operation of ships and water-borne craft,
aircraft and other vehicles on water, in the air or on land Such rights, power and authority shall include, inter alia, the
comprising right, power and authority: x x x x to construct, install,
maintain, and employ on any base any type of facilities,
No equivalent provision. weapons, substance, device, vessel or vehicle on or under the
ground, in the air or on or under the water that may be
Seventh, under EDCA, the U.S. is merely given temporary requisite or appropriate, including meteorological systems,
access to land and facilities (including roads, ports, and aerial and water navigation lights, radio and radar apparatus
airfields). On the other hand, the old treaty gave the U.S. the and electronic devices, of any desired power, type of emission
right to improve and deepen the harbors, channels, entrances, and frequency.
and anchorages; and to construct or maintain necessary
EDCA, Art. III(4):
FINALS CONSTITUTIONAL LAW I ACJUCO NOV 11, 2017 26

x xx x
The Philippines hereby grants to the United States, through
bilateral security mechanisms, such as the MDB and SEB, 3. "United States contractors" means companies and firms,
operational control of Agreed Locations for construction and their employees, under contract or subcontract to or on
activities and authority to undertake such activities on, and behalf of the United States Department of Defense. United
make alterations and improvements to, Agreed Locations. States contractors are not included as part of the definition of
United States forces shall consult on issues regarding such United States personnel in this Agreement, including within
construction, alterations, and improvements based on the the context of the VFA.
Parties' shared intent that the technical requirements and
construction standards of any such projects undertaken by or
on behalf of United States forces should be consistent with the
requirements and standards of both Parties. Twelfth, EDCA does not allow the U.S. to exercise jurisdiction
over any offense committed by any person within the Agreed
Tenth, EDCA does not allow the U.S. to acquire, by Locations, unlike in the former military bases:
condemnation or expropriation proceedings, real property
belonging to any private person. The old military bases 1947 MBA EDCA
agreement gave this right to the U.S. as seen below: 1947 MBA, Art. XIII(l)(a):

1947 MBA EDCA The Philippines consents that the United


1947 MBA, Art. XXII(l):
States shall have the right to exercise jurisdiction over the
Whenever it is necessary to acquire by following offenses: (a) Any offense committed by any person
within any base except where the offender and offended
condemnation or expropriation proceedings real property parties are both Philippine citizens (not members of the armed
belonging to any private persons, associations or corporations forces of the United States on active duty) or the offense is
located in bases named in Annex A and Annex B in order to against the security of the Philippines.
carry out the purposes of this Agreement, the Philippines will
institute and prosecute such condemnation or expropriation No equivalent provision.
proceedings in accordance with the laws of the Philippines.
The United States agrees to reimburse the Philippines for all Thirteenth, EDCA does not allow the U.S. to operate military
the reasonable expenses, damages and costs therebv post exchange (PX) facilities, which is free of customs duties
incurred, including the value of the property as determined by and taxes, unlike what the expired MBA expressly allowed.
the Court. In addition, subject to the mutual agreement of the Parenthetically, the PX store has become the cultural icon of
two Governments, the United States will reimburse the U.S. military presence in the country.
Philippines for the reasonable costs of transportation and
removal of any occupants displaced or ejected by reason of 1947 MBA EDCA
the condemnation or expropriation. 1947 MBA, Art. XVIII(l):

No equivalent provision. It is mutually agreed that the United States

Eleventh, EDCA does not allow the U.S. to unilaterally bring shall have the right to establish on bases, free of all licenses;
into the country non-Philippine nationals who are under its fees; sales, excise or other taxes, or imposts; Government
employ, together with their families, in connection with the agencies, including concessions, such as sales commissaries
construction, maintenance, or operation of the bases. EDCA and post exchanges; messes and social clubs, for the
strictly adheres to the limits under the VFA. exclusive use of the United States military forces and
authorized civilian personnel and their families. The
1947 MBA EDCA merchandise or services sold or dispensed by such agencies
1947 MBA, Art. XI(l): shall be free of all taxes, duties and inspection by the
Philippine authorities. Administrative measures shall be taken
It is mutually agreed that the United States shall have the right by the appropriate authorities of the United States to prevent
to bring into the Philippines members of the United States the resale of goods which are sold under the provisions of this
military forces and the United States nationals employed by or Article to persons not entitled to buy goods at such agencies
under a contract with the United States together with their and, generally, to prevent abuse of the privileges granted
families, and technical personnel of other nationalities (not under this Article. There shall be cooperation between such
being persons excluded by the laws of the Philippines) in authorities and the Philippines to this end.
connection with the construction, maintenance, or operation
of the bases. The United States shall make suitable No equivalent provision.
arrangements so that such persons may be readily identified
and their status established when necessary by the Philippine In sum, EDCA is a far cry from a basing agreement as was
authorities. Such persons, other than members of the United understood by the people at the time that the 1987
States armed forces in uniform, shall present their travel Constitution was adopted.
documents to the appropriate Philippine authorities for visas,
it being understood that no objection will be made to their Nevertheless, a comprehensive review of what the
travel to the Philippines as non-immigrants. Constitution means by "foreign military bases" and "facilities"
is required before EDCA can be deemed to have passed
EDCA, Art. II: judicial scrutiny.

1. "United States personnel" means United States military and c. The meaning of military facilities and bases
civilian personnel temporarily in the territory of the Philippines
in connection with activities approved by the Philippines, as An appreciation of what a military base is, as understood by
those terms are defined in the VFA. the Filipino people in 1987, would be vital in determining
whether EDCA breached the constitutional restriction.
FINALS CONSTITUTIONAL LAW I ACJUCO NOV 11, 2017 27

Henceforth, any proposed entry of U.S. forces into the


Prior to the drafting of the 1987 Constitution, the last definition Philippines had to evolve likewise, taking into consideration
of "military base" was provided under Presidential Decree No. the subsisting agreements between both parties, the rejection
(PD) 1227.328 Unlawful entry into a military base is of the 1991 proposal, and a concrete understanding of what
punishable under the decree as supported by Article 281 of was constitutionally restricted. This trend birthed the VFA
the Revised Penal Code, which itself prohibits the act of which, as discussed, has already been upheld by this Court.
trespass.
The latest agreement is EDCA, which proposes a novel
Section 2 of the law defines the term in this manner: "'[M]ilitary concept termed "Agreed Locations."
base' as used in this decree means any military, air, naval, or
coast guard reservation, base, fort, camp, arsenal, yard, By definition, Agreed Locations are
station, or installation in the Philippines."
facilities and areas that are provided by the Government of the
Commissioner Tadeo, in presenting his objections to U.S. Philippines through the AFP and that United States forces,
presence in the Philippines before the 1986 Constitutional United States contractors, and others as mutually agreed,
Commission, listed the areas that he considered as military shall have the right to access and use pursuant to this
bases: Agreement. Such Agreed Locations may be listed in an annex
to be appended to this Agreement, and may be further
1,000 hectares Camp O'Donnel described in implementing arrangements.332

20,000 hectares Crow Valley Weapon's Range Preliminarily, respondent already claims that the proviso that
the Philippines shall retain ownership of and title to the Agreed
55,000 hectares Clark Air Base Locations means that EDCA is "consistent with Article II of the
VFA which recognizes Philippine sovereignty and jurisdiction
150 hectares Wallace Air Station over locations within Philippine territory.333

400 hectares John Hay Air Station By this interpretation, respondent acknowledges that the
contention of petitioners springs from an understanding that
15,000 hectares Subic Naval Base the Agreed Locations merely circumvent the constitutional
restrictions. Framed differently, the bone of contention is
1,000 hectares San Miguel Naval Communication whether the Agreed Locations are, from a legal perspective,
foreign military facilities or bases. This legal framework
750 hectares Radio Transmitter in Capas, Tarlac triggers Section 25, Article XVIII, and makes Senate
concurrence a sine qua non.
900 hectares Radio Bigot Annex at Bamban, Tarlac329
Article III of EDCA provides for Agreed Locations, in which the
The Bases Conversion and Development Act of 1992 U.S. is authorized by the Philippines to "conduct the following
described its coverage in its Declaration of Policies: activities: "training; transit; support and related activities;
refueling of aircraft; bunkering of vessels; temporary
Sec. 2. Declaration of Policies. - It is hereby declared the maintenance of vehicles, vessels and aircraft; temporary
policy of the Government to accelerate the sound and accommodation of personnel; communications;
balanced conversion into alternative productive uses of the prepositioning of equipment, supplies and materiel; deploying
Clark and Subic military reservations and their extensions forces and materiel; and such other activities as the Parties
(John Hay Station, Wallace Air Station, O'Donnell Transmitter may agree."
Station, San Miguel Naval Communications Station and
Capas Relay Station), to raise funds by the sale of portions of This creation of EDCA must then be tested against a proper
Metro Manila military camps, and to apply said funds as interpretation of the Section 25 restriction.
provided herein for the development and conversion to
productive civilian use of the lands covered under the 194 7 d. Reasons for the constitutional requirements and legal
Military Bases Agreement between the Philippines and the standards for constitutionally compatible military bases and
United States of America, as amended.330 facilities

The result of the debates and subsequent voting is Section Section 25 does not define what is meant by a "foreign military
25, Article XVIII of the Constitution, which specifically restricts, facility or base." While it specifically alludes to U.S. military
among others, foreign military facilities or bases. At the time facilities and bases that existed during the framing of the
of its crafting of the Constitution, the 1986 Constitutional Constitution, the provision was clearly meant to apply to those
Commission had a clear idea of what exactly it was restricting. bases existing at the time and to any future facility or base.
While the term "facilities and bases" was left undefined, its The basis for the restriction must first be deduced from the
point of reference was clearly those areas covered by the spirit of the law, in order to set a standard for the application
1947 MBA as amended. of its text, given the particular historical events preceding the
agreement.
Notably, nearly 30 years have passed since then, and the
ever-evolving world of military technology and geopolitics has Once more, we must look to the 1986 Constitutional
surpassed the understanding of the Philippine people in 1986. Commissioners to glean, from their collective wisdom, the
The last direct military action of the U.S. in the region was the intent of Section 25. Their speeches are rich with history and
use of Subic base as the staging ground for Desert Shield and wisdom and present a clear picture of what they considered in
Desert Storm during the Gulf War.331 In 1991, the Philippine the crafting the provision.
Senate rejected the successor treaty of the 1947 MBA that
would have allowed the continuation of U.S. bases in the SPEECH OF COMMISSIONER REGALADO334
Philippines.
xxxx
FINALS CONSTITUTIONAL LAW I ACJUCO NOV 11, 2017 28

We have been regaled here by those who favor the adoption


of the anti-bases provisions with what purports to be an 2. That no lasting peace and no true sovereignty would ever
objective presentation of the historical background of the be achieved so long as there are foreign military forces in our
military bases in the Philippines. Care appears, however, to country;
have been taken to underscore the inequity in their inception
as well as their implementation, as to seriously reflect on the 3. That the presence of foreign military bases deprives us of
supposed objectivity of the report. Pronouncements of military the very substance of national sovereignty and this is a
and civilian officials shortly after World War II are quoted in constant source of national embarrassment and an insult to
support of the proposition on neutrality; regrettably, the our national dignity and selfrespect as a nation;
implication is that the same remains valid today, as if the world
and international activity stood still for the last 40 years. 4. That these foreign military bases unnecessarily expose our
country to devastating nuclear attacks;
We have been given inspired lectures on the effect of the
presence of the military bases on our sovereignty - whether in 5. That these foreign military bases create social problems
its legal or political sense is not clear - and the theory that any and are designed to perpetuate the strangle-hold of United
country with foreign bases in its territory cannot claim to be States interests in our national economy and development;
fully sovereign or completely independent. I was not aware
that the concepts of sovereignty and independence have now 6. That the extraterritorial rights enjoyed by these foreign
assumed the totality principle, such that a willing assumption bases operate to deprive our country of jurisdiction over civil
of some delimitations in the exercise of some aspects thereof and criminal offenses committed within our own national
would put that State in a lower bracket of nationhood. territory and against Filipinos;

xxxx 7. That the bases agreements are colonial impositions and


dictations upon our helpless country; and
We have been receiving a continuous influx of materials on
the pros and cons on the advisability of having military bases 8. That on the legal viewpoint and in the ultimate analysis, all
within our shores. Most of us who, only about three months the bases agreements are null and void ab initio, especially
ago, were just mulling the prospects of these varying because they did not count the sovereign consent and will of
contentions are now expected, like armchair generals, to the Filipino people.338
decide not only on the geopolitical aspects and contingent
implications of the military bases but also on their political, xxxx
social, economic and cultural impact on our national life. We
are asked to answer a plethora of questions, such as: 1) In the real sense, Madam President, if we in the Commission
whether the bases are magnets of nuclear attack or are could accommodate the provisions I have cited, what is our
deterrents to such attack; 2) whether an alliance or mutual objection to include in our Constitution a matter as priceless
defense treaty is a derogation of our national sovereignty; 3) as the nationalist values we cherish? A matter of the gravest
whether criticism of us by Russia, Vietnam and North Korea is concern for the safety and survival of this nation indeed
outweighed by the support for us of the ASEAN countries, the deserves a place in our Constitution.
United States, South Korea, Taiwan, Australia and New
Zealand; and 4) whether the social, moral and legal problems xxxx
spawned by the military bases and their operations can be
compensated by the economic benefits outlined in papers x x x Why should we bargain away our dignity and our self-
which have been furnished recently to all of us.335 respect as a nation and the future of generations to come with
thirty pieces of silver?339
xxxx
SPEECH OF COMMISSIONER BENNAGEN340
Of course, one side of persuasion has submitted categorical,
unequivocal and forceful assertions of their positions. They xxxx
are entitled to the luxury of the absolutes. We are urged now
to adopt the proposed declaration as a "golden," "unique" and The underlying principle of military bases and nuclear
"last" opportunity for Filipinos to assert their sovereign rights. weapons wherever they are found and whoever owns them is
Unfortunately, I have never been enchanted by superlatives, that those are for killing people or for terrorizing humanity. This
much less for the applause of the moment or the ovation of objective by itself at any point in history is morally repugnant.
the hour. Nor do I look forward to any glorious summer after a This alone is reason enough for us to constitutionalize the ban
winter of political discontent. Hence, if I may join on foreign military bases and on nuclear weapons.341
Commissioner Laurel, I also invoke a caveat not only against
the tyranny of labels but also the tyranny of slogans.336 SPEECH OF COMMISSIONER BACANI342

xxxx xxxx

SPEECH OF COMMISSIONER SUAREZ337 x x x Hence, the remedy to prostitution does not seem to be
primarily to remove the bases because even if the bases are
MR. SUAREZ: Thank you, Madam President. removed, the girls mired in poverty will look for their clientele
elsewhere. The remedy to the problem of prostitution lies
I am quite satisfied that the crucial issues involved in the primarily elsewhere - in an alert and concerned citizenry, a
resolution of the problem of the removal of foreign bases from healthy economy and a sound education in values.343
the Philippines have been adequately treated by previous
speakers. Let me, therefore, just recapitulate the arguments SPEECH OF COMMISSIONER JAMIR344
adduced in favor of a foreign bases-free Philippines:
xxxx
1. That every nation should be free to shape its own destiny
without outside interference;
FINALS CONSTITUTIONAL LAW I ACJUCO NOV 11, 2017 29

One of the reasons advanced against the maintenance of of war in some of these areas because of foreign intervention
foreign military bases here is that they impair portions of our which views these conflicts through the narrow prism of the
sovereignty. While I agree that our country's sovereignty East-West conflict. The United States bases have been used
should not be impaired, I also hold the view that there are as springboards for intervention in some of these conflicts. We
times when it is necessary to do so according to the should not allow ourselves to be party to the warlike mentality
imperatives of national interest. There are precedents to this of these foreign interventionists. We must always be on the
effect. Thus, during World War II, England leased its bases in side of peace – this means that we should not always rely on
the West Indies and in Bermuda for 99 years to the United military solution.352
States for its use as naval and air bases. It was done in
consideration of 50 overaged destroyers which the United xxxx
States gave to England for its use in the Battle of the Atlantic.
x x x The United States bases, therefore, are springboards for
A few years ago, England gave the Island of Diego Garcia to intervention in our own internal affairs and in the affairs of
the United States for the latter's use as a naval base in the other nations in this region.
Indian Ocean. About the same time, the United States
obtained bases in Spain, Egypt and Israel. In doing so, these xxxx
countries, in effect, contributed to the launching of a
preventive defense posture against possible trouble in the Thus, I firmly believe that a self-respecting nation should
Middle East and in the Indian Ocean for their own safeguard its fundamental freedoms which should logically be
protection.345 declared in black and white in our fundamental law of the land
- the Constitution. Let us express our desire for national
SPEECH OF COMMISSIONER TINGSON346 sovereignty so we may be able to achieve national self-
determination. Let us express our desire for neutrality so that
xxxx we may be able to follow active nonaligned independent
foreign policies. Let us express our desire for peace and a
In the case of the Philippines and the other Southeast Asian nuclear-free zone so we may be able to pursue a healthy and
nations, the presence of American troops in the country is a tranquil existence, to have peace that is autonomous and not
projection of America's security interest. Enrile said that imposed. 353
nonetheless, they also serve, although in an incidental and
secondary way, the security interest of the Republic of the xxxx
Philippines and the region. Yes, of course, Mr. Enrile also
echoes the sentiments of most of us in this Commission, SPEECH OF COMMISSIONER TADEO354
namely: It is ideal for us as an independent and sovereign
nation to ultimately abrogate the RP-US military treaty and, at Para sa magbubukid, ano ha ang kahulugan ng U.S. military
the right time, build our own air and naval might.347 bases? Para sa magbubukid, ang kahulugan nito ay
pagkaalipin. Para sa magbubukid, ang pananatili ng U.S.
xxxx military bases ay tinik sa dibdib ng sambayanang Pilipinong
patuloy na nakabaon. Para sa sambayanang magbubukid,
Allow me to say in summation that I am for the retention of ang ibig sabihin ng U.S. military bases ay batong pabigat na
American military bases in the Philippines provided that such patuloy na pinapasan ng sambayanang Pilipino. Para sa
an extension from one period to another shall be concluded sambayanang magbubukid, ang pananatili ng U.S. military
upon concurrence of the parties, and such extension shall be bases ay isang nagdudumilat na katotohanan ng patuloy na
based on justice, the historical amity of the people of the paggahasa ng imperyalistang Estados Unidos sa ating Inang
Philippines and the United States and their common defense Bayan - economically, politically and culturally. Para sa
interest.348 sambayanang magbubukid ang U.S. military bases ay
kasingkahulugan ng nuclear weapon - ang kahulugan ay
SPEECH OF COMMISSIONER ALONTO349 magneto ng isang nuclear war. Para sa sambayanang
magbubukid, ang kahulugan ng U.S. military bases ay isang
xxxx salot.355

Madam President, sometime ago after this Commission SPEECH OF COMMISSIONER QUESADA356
started with this task of framing a constitution, I read a
statement of President Aquino to the effect that she is for the xxxx
removal of the U.S. military bases in this country but that the
removal of the U.S. military bases should not be done just to The drift in the voting on issues related to freeing ourselves
give way to other foreign bases. Today, there are two world from the instruments of domination and subservience has
superpowers, both vying to control any and all countries which clearly been defined these past weeks.
have importance to their strategy for world domination. The
Philippines is one such country. xxxx

Madam President, I submit that I am one of those ready to So for the record, Mr. Presiding Officer, I would like to declare
completely remove any vestiges of the days of enslavement, my support for the committee's position to enshrine in the
but not prepared to erase them if to do so would merely leave Constitution a fundamental principle forbidding foreign military
a vacuum to be occupied by a far worse type.350 bases, troops or facilities in any part of the Philippine territory
as a clear and concrete manifestation of our inherent right to
SPEECH OF COMMISSIONER GASCON351 national self-determination, independence and sovereignty.

xxxx Mr. Presiding Officer, I would like to relate now these attributes
of genuine nationhood to the social cost of allowing foreign
Let us consider the situation of peace in our world today. countries to maintain military bases in our country. Previous
Consider our brethren in the Middle East, in Indo-China, speakers have dwelt on this subject, either to highlight its
Central America, in South Africa - there has been escalation
FINALS CONSTITUTIONAL LAW I ACJUCO NOV 11, 2017 30

importance in relation to the other issues or to gloss over its REBUTTAL OF COMMISSIONER NOLLEDO362
significance and !llake this a part of future negotiations.357
xxxx
xxxx
The anachronistic and ephemeral arguments against the
Mr. Presiding Officer, I feel that banning foreign military bases provisions of the committee report to dismantle the American
is one of the solutions and is the response of the Filipino bases after 1991 only show the urgent need to free our
people against this condition and other conditions that have country from the entangling alliance with any power bloc.363
already been clearly and emphatically discussed in past
deliberations. The deletion, therefore, of Section 3 in the xxxx
Constitution we are drafting will have the following
implications: xx x Mr. Presiding Officer, it is not necessary for us to possess
expertise to know that the so-called RP-US Bases Agreement
First, the failure of the Constitutional Commission to decisively will expire in 1991, that it infringes on our sovereignty and
respond to the continuing violation of our territorial integrity via jurisdiction as well as national dignity and honor, that it goes
the military bases agreement which permits the retention of against the UN policy of disarmament and that it constitutes
U.S. facilities within the Philippine soil over which our unjust intervention in our internal affairs.364 (Emphases
authorities have no exclusive jurisdiction contrary to the Supplied)
accepted definition of the exercise of sovereignty.
The Constitutional Commission eventually agreed to allow
Second, consent by this forum, this Constitutional foreign military bases, troops, or facilities, subject to the
Commission, to an exception in the application of a provision provisions of Section 25. It is thus important to read its
in the Bill of Rights that we have just drafted regarding equal discussions carefully. From these discussions, we can deduce
application of the laws of the land to all inhabitants, permanent three legal standards that were articulated by the
or otherwise, within its territorial boundaries. Constitutional Commission Members. These are
characteristics of any agreement that the country, and by
Third, the continued exercise by the United States of extension this Court, must ensure are observed. We can
extraterritoriality despite the condemnations of such practice thereby determine whether a military base or facility in the
by the world community of nations in the light of overwhelming Philippines, which houses or is accessed by foreign military
international approval of eradicating all vestiges of troops, is foreign or remains a Philippine military base or
colonialism.358 facility. The legal standards we find applicable are:
independence from foreign control, sovereignty and
xxxx applicable law, and national security and territorial integrity.

Sixth, the deification of a new concept called pragmatic i. First standard: independence from foreign control
sovereignty, in the hope that such can be wielded to force the
United States government to concede to better terms and Very clearly, much of the opposition to the U.S. bases at the
conditions concerning the military bases agreement, including time of the Constitution's drafting was aimed at asserting
the transfer of complete control to the Philippine government Philippine independence from the U.S., as well as control over
of the U.S. facilities, while in the meantime we have to suffer our country's territory and military.
all existing indignities and disrespect towards our rights as a
sovereign nation. Under the Civil Code, there are several aspects of control
exercised over property.
xxxx
Property is classified as private or public.365 It is public if
Eighth, the utter failure of this forum to view the issue of "intended for public use, such as roads, canals, rivers,
foreign military bases as essentially a question of sovereignty torrents, ports and bridges constructed by the State, banks,
which does not require in-depth studies or analyses and which shores, roadsteads, and others of similar character[,]" or
this forum has, as a constituent assembly drafting a "[t]hose which belong to the State, without being for public
constitution, the expertise and capacity to decide on except use, and are intended for some public service or for the
that it lacks the political will that brought it to existence and development of the national wealth. "366
now engages in an elaborate scheme of buck-passing.
Quite clearly, the Agreed Locations are contained within a
xxxx property for public use, be it within a government military camp
or property that belongs to the Philippines.1avvphi1
Without any doubt we can establish a new social order in our
country, if we reclaim, restore, uphold and defend our national Once ownership is established, then the rights of ownership
sovereignty. National sovereignty is what the military bases flow freely. Article 428 of the Civil Code provides that "[t]he
issue is all about. It is only the sovereign people exercising owner has the right to enjoy and dispose of a thing, without
their national sovereignty who can design an independent other limitations than those established by law." Moreover, the
course and take full control of their national destiny.359 owner "has also a right of action against the holder and
possessor of the thing in order to recover it."
SPEECH OF COMMISSIONER P ADILLA360
Philippine civil law therefore accords very strong rights to the
xxxx owner of property, even against those who hold the property.
Possession, after all, merely raises a disputable presumption
Mr. Presiding Officer, in advocating the majority committee of ownership, which can be contested through normal judicial
report, specifically Sections 3 and 4 on neutrality, nuclear and processes.367
bases-free country, some views stress sovereignty of the
Republic and even invoke survival of the Filipino nation and In this case, EDCA explicitly provides that ownership of the
people.361 Agreed Locations remains with the Philippine govemment.368
FINALS CONSTITUTIONAL LAW I ACJUCO NOV 11, 2017 31

What U.S. personnel have a right to, pending mutual it has never been independent and self-reliant. Facts, data
agreement, is access to and use of these locations.369 and statistics will show that it has been substantially
dependent upon a foreign power. In March 1968,
The right of the owner of the property to allow access and use Congressman Barbero, himself a member of the Armed
is consistent with the Civil Code, since the owner may dispose Forces of the Philippines, revealed top secret documents
of the property in whatever way deemed fit, subject to the showing what he described as U.S. dictation over the affairs
limits of the law. So long as the right of ownership itself is not of the Armed Forces of the Philippines. He showed that under
transferred, then whatever rights are transmitted by existing arrangements, the United States unilaterally
agreement does not completely divest the owner of the rights determines not only the types and quantity of arms and
over the property, but may only limit them in accordance with equipments that our armed forces would have, but also the
law. time when these items are to be made available to us. It is
clear, as he pointed out, that the composition, capability and
Hence, even control over the property is something that an schedule of development of the Armed Forces of the
owner may transmit freely. This act does not translate into the Philippines is under the effective control of the U.S.
full transfer of ownership, but only of certain rights. In Roman government.376 (Emphases supplied)
Catholic Apostolic Administrator of Davao, Inc. v. Land
Registration Commission, we stated that the constitutional Commissioner Sarmiento proposed a motherhood statement
proscription on property ownership is not violated despite the in the 1987 Constitution that would assert "independent" and
foreign national's control over the property.370 "self-reliant" armed forces. This proposal was rejected by the
committee, however. As Commissioner De Castro asserted,
EDCA, in respect of its provisions on Agreed Locations, is the involvement of the Philippine military with the U.S. did not,
essentially a contract of use and access. Under its pertinent by itself, rob the Philippines of its real independence. He made
provisions, it is the Designated Authority of the Philippines that reference to the context of the times: that the limited resources
shall, when requested, assist in facilitating transit or access to of the Philippines and the current insurgency at that time
public land and facilities.371 The activities carried out within necessitated a strong military relationship with the U.S. He
these locations are subject to agreement as authorized by the said that the U.S. would not in any way control the Philippine
Philippine govemment.372 Granting the U.S. operational military despite this relationship and the fact that the former
control over these locations is likewise subject to EDCA' s would furnish military hardware or extend military assistance
security mechanisms, which are bilateral procedures involving and training to our military. Rather, he claimed that the
Philippine consent and cooperation.373 Finally, the Philippine proposal was in compliance with the treaties between the two
Designated Authority or a duly designated representative is states.
given access to the Agreed Locations.374
MR. DE CASTRO: If the Commissioner will take note of my
To our mind, these provisions do not raise the spectre of U.S. speech on U.S. military bases on 12 September 1986, I spoke
control, which was so feared by the Constitutional on the selfreliance policy of the armed forces. However, due
Commission. In fact, they seem to have been the product of to very limited resources, the only thing we could do is
deliberate negotiation from the point of view of the Philippine manufacture small arms ammunition. We cannot blame the
government, which balanced constitutional restrictions on armed forces. We have to blame the whole Republic of the
foreign military bases and facilities against the security needs Philippines for failure to provide the necessary funds to make
of the country. In the 1947 MBA, the U.S. forces had "the right, the Philippine Armed Forces self-reliant. Indeed that is a
power and authority x x x to construct (including dredging and beautiful dream. And I would like it that way. But as of this
filling), operate, maintain, utilize, occupy, garrison and control time, fighting an insurgency case, a rebellion in our country -
the bases."375 No similarly explicit provision is present in insurgency - and with very limited funds and very limited
EDCA. number of men, it will be quite impossible for the Philippines
to appropriate the necessary funds therefor. However, if we
Nevertheless, the threshold for allowing the presence of say that the U.S. government is furnishing us the military
foreign military facilities and bases has been raised by the hardware, it is not control of our armed forces or of our
present Constitution. Section 25 is explicit that foreign military government. It is in compliance with the Mutual Defense
bases, troops, or facilities shall not be allowed in the Treaty. It is under the military assistance program that it
Philippines, except under a treaty duly concurred in by the becomes the responsibility of the United States to furnish us
Senate. Merely stating that the Philippines would retain the necessary hardware in connection with the military bases
ownership would do violence to the constitutional requirement agreement. Please be informed that there are three (3)
if the Agreed Locations were simply to become a less obvious treaties connected with the military bases agreement; namely:
manifestation of the U.S. bases that were rejected in 1991. the RP-US Military Bases Agreement, the Mutual Defense
Treaty and the Military Assistance Program.
When debates took place over the military provisions of the
Constitution, the committee rejected a specific provision My dear Commissioner, when we enter into a treaty and we
proposed by Commissioner Sarmiento. The discussion are furnished the military hardware pursuant to that treaty, it
illuminates and provides context to the 1986 Constitutional is not in control of our armed forces nor control of our
Commission's vision of control and independence from the government. True indeed, we have military officers trained in
U.S., to wit: the U.S. armed forces school. This is part of our Military
Assistance Program, but it does not mean that the minds of
MR. SARMIENTO: Madam President, my proposed our military officers are for the U.S. government, no. I am one
amendment reads as follows: "THE STATE SHALL of those who took four courses in the United States schools,
ESTABLISH AND MAINTAIN AN INDEPENDENT AND but I assure you, my mind is for the Filipino people. Also, while
SELF-RELIANT ARMED FORCES OF THE PHILIPPINES." we are sending military officers to train or to study in U.S.
Allow me to briefly explain, Madam President. The Armed military schools, we are also sending our officers to study in
Forces of the Philippines is a vital component of Philippine other military schools such as in Australia, England and in
society depending upon its training, orientation and support. It Paris. So, it does not mean that when we send military officers
will either be the people's protector or a staunch supporter of to United States schools or to other military schools, we will
a usurper or tyrant, local and foreign interest. The Armed be under the control of that country. We also have foreign
Forces of the Philippines' past and recent experience shows officers in our schools, we in the Command and General Staff
FINALS CONSTITUTIONAL LAW I ACJUCO NOV 11, 2017 32

College in Fort Bonifacio and in our National Defense College, [t]he authority to perform those functions of command over
also in Fort Bonifacio.377 (Emphases supplied) subordinate forces involving organizing and employing
commands and forces, assigning tasks, designating objective,
This logic was accepted in Tañada v. Angara, in which the and giving authoritative direction necessary to accomplish the
Court ruled that independence does not mean the absence of mission.383
foreign participation:
At times, though, operational control can mean something
Furthermore, the constitutional policy of a "self-reliant and slightly different. In JUSMAG Philippines v. National Labor
independent national economy" does not necessarily rule out Relations Commission, the Memorandum of Agreement
the entry of foreign investments, goods and services. It between the AFP and JUSMAG Philippines defined the term
contemplates neither "economic seclusion" nor "mendicancy as follows:384
in the international community." As explained by
Constitutional Commissioner Bernardo Villegas, sponsor of The term "Operational Control" includes, but is not limited to,
this constitutional policy: all personnel administrative actions, such as: hiring
recommendations; firing recommendations; position
Economic self reliance is a primary objective of a developing classification; discipline; nomination and approval of incentive
country that is keenly aware of overdependence on external awards; and payroll computation.
assistance for even its most basic needs. It does not mean
autarky or economic seclusion; rather, it means avoiding Clearly, traditional standards define "operational control" as
mendicancy in the international community. Independence personnel control. Philippine law, for instance, deems
refers to the freedom from undue foreign control of the operational control as one exercised by police officers and
national economy, especially in such strategic industries as in civilian authorities over their subordinates and is distinct from
the development of natural resources and public utilities.378 the administrative control that they also exercise over police
(Emphases supplied) subordinates.385 Similarly, a municipal mayor exercises
operational control over the police within the municipal
The heart of the constitutional restriction on foreign military government,386 just as city mayor possesses the same power
facilities and bases is therefore the assertion of independence over the police within the city government.387
from the U.S. and other foreign powers, as independence is
exhibited by the degree of foreign control exerted over these Thus, the legal concept of operational control involves
areas.1âwphi1 The essence of that independence is self- authority over personnel in a commander-subordinate
governance and self-control.379 Independence itself is "[t]he relationship and does not include control over the Agreed
state or condition of being free from dependence, subjection, Locations in this particular case. Though not necessarily
or control. "380 stated in EDCA provisions, this interpretation is readily implied
by the reference to the taking of "appropriate measures to
Petitioners assert that EDCA provides the U.S. extensive protect United States forces and United States contractors."
control and authority over Philippine facilities and locations,
such that the agreement effectively violates Section 25 of the It is but logical, even necessary, for the U.S. to have
1987 Constitution.381 operational control over its own forces, in much the same way
that the Philippines exercises operational control over its own
Under Article VI(3) of EDCA, U.S. forces are authorized to act units.
as necessary for "operational control and defense." The term
"operational control" has led petitioners to regard U.S. control For actual operations, EDCA is clear that any activity must be
over the Agreed Locations as unqualified and, therefore, planned and pre-approved by the MDB-SEB.388 This
total.382 Petitioners contend that the word "their" refers to the provision evinces the partnership aspect of EDCA, such that
subject "Agreed Locations." both stakeholders have a say on how its provisions should be
put into effect.
This argument misreads the text, which is quoted below:
b. Operational control vis-à-vis effective command and control
United States forces are authorized to exercise all rights and
authorities within Agreed Locations that are necessary for Petitioners assert that beyond the concept of operational
their operational control or defense, including taking control over personnel, qualifying access to the Agreed
appropriate measure to protect United States forces and Locations by the Philippine Designated Authority with the
United States contractors. The United States should phrase "consistent with operational safety and security
coordinate such measures with appropriate authorities of the requirements in accordance with agreed procedures
Philippines. developed by the Parties" leads to the conclusion that the U.S.
exercises effective control over the Agreed Locations.389
A basic textual construction would show that the word "their," They claim that if the Philippines exercises possession of and
as understood above, is a possessive pronoun for the subject control over a given area, its representative should not have
"they," a third-person personal pronoun in plural form. Thus, to be authorized by a special provision.390
"their" cannot be used for a non-personal subject such as
"Agreed Locations." The simple grammatical conclusion is For these reasons, petitioners argue that the "operational
that "their" refers to the previous third-person plural noun, control" in EDCA is the "effective command and control" in the
which is "United States forces." This conclusion is in line with 1947 MBA.391 In their Memorandum, they distinguish
the definition of operational control. effective command and control from operational control in
U.S. parlance.392 Citing the Doctrine for the Armed Forces of
a. U.S. operational control as the exercise of authority over the United States, Joint Publication 1, "command and control
U.S. personnel, and not over the Agreed Locations (C2)" is defined as "the exercise of authority and direction by
a properly designated commander over assigned and
Operational control, as cited by both petitioner and attached forces in the accomplishment of the mission x x
respondents, is a military term referring to x."393 Operational control, on the other hand, refers to
"[t]hose functions of command over assigned forces involving
the composition of subordinate forces, the assignment of
FINALS CONSTITUTIONAL LAW I ACJUCO NOV 11, 2017 33

tasks, the designation of objectives, the overall control of whereas these factors are included in the concept of
assigned resources, and the full authoritative direction command and control.400
necessary to accomplish the mission."394
This distinction, found in the same document cited by
Two things demonstrate the errors in petitioners' line of petitioners, destroys the very foundation of the arguments
argument. they have built: that EDCA is the same as the MBA.

Firstly, the phrase "consistent with operational safety and c. Limited operational control over the Agreed Locations only
security requirements in accordance with agreed procedures for construction activitites
developed by the Parties" does not add any qualification
beyond that which is already imposed by existing treaties. To As petitioners assert, EDCA indeed contains a specific
recall, EDCA is based upon prior treaties, namely the VFA and provision that gives to the U.S. operational control within the
the MDT.395 Treaties are in themselves contracts from which Agreed Locations during construction activities.401 This
rights and obligations may be claimed or waived.396 In this exercise of operational control is premised upon the approval
particular case, the Philippines has already agreed to abide by the MDB and the SEB of the construction activity through
by the security mechanisms that have long been in place consultation and mutual agreement on the requirements and
between the U.S. and the Philippines based on the standards of the construction, alteration, or improvement.402
implementation of their treaty relations.397
Despite this grant of operational control to the U.S., it must be
Secondly, the full document cited by petitioners contradicts emphasized that the grant is only for construction activities.
the equation of "operational control" with "effective command The narrow and limited instance wherein the U.S. is given
and control," since it defines the terms quite differently, operational control within an Agreed Location cannot be
viz:398 equated with foreign military control, which is so abhorred by
the Constitution.
Command and control encompasses the exercise of authority,
responsibility, and direction by a commander over assigned The clear import of the provision is that in the absence of
and attached forces to accomplish the mission. Command at construction activities, operational control over the Agreed
all levels is the art of motivating and directing people and Location is vested in the Philippine authorities. This meaning
organizations into action to accomplish missions. Control is is implicit in the specific grant of operational control only during
inherent in command. To control is to manage and direct construction activities. The principle of constitutional
forces and functions consistent with a commander's command construction, "expressio unius est exclusio alterius," means
authority. Control of forces and functions helps commanders the failure to mention the thing becomes the ground for
and staffs compute requirements, allocate means, and inferring that it was deliberately excluded.403 Following this
integrate efforts. Mission command is the preferred method of construction, since EDCA mentions the existence of U.S.
exercising C2. A complete discussion of tenets, organization, operational control over the Agreed Locations for construction
and processes for effective C2 is provided in Section B, activities, then it is quite logical to conclude that it is not
"Command and Control of Joint Forces," of Chapter V "Joint exercised over other activities.
Command and Control."
Limited control does not violate the Constitution. The fear of
Operational control is defined thus:399 the commissioners was total control, to the point that the
foreign military forces might dictate the terms of their acts
OPCON is able to be delegated from a lesser authority than within the Philippines.404 More important, limited control does
COCOM. It is the authority to perform those functions of not mean an abdication or derogation of Philippine
command over subordinate forces involving organizing and sovereignty and legal jurisdiction over the Agreed Locations.
employing commands and forces, assigning tasks, It is more akin to the extension of diplomatic courtesies and
designating objectives, and giving authoritative direction over rights to diplomatic agents,405 which is a waiver of control on
all aspects of military operations and joint training necessary a limited scale and subject to the terms of the treaty.
to accomplish the mission. It should be delegated to and
exercised by the commanders of subordinate organizations; This point leads us to the second standard envisioned by the
normally, this authority is exercised through subordinate framers of the Constitution: that the Philippines must retain
JFCs, Service, and/or functional component commanders. sovereignty and jurisdiction over its territory.
OPCON provides authority to organize and employ
commands and forces as the commander considers ii. Second standard: Philippine sovereignty and applicable law
necessary to accomplish assigned missions. It does not
include authoritative direction for logistics or matters of EDCA states in its Preamble the "understanding for the United
administration, discipline, internal organization, or unit States not to establish a permanent military presence or base
training. These elements of COCOM must be specifically in the territory of the Philippines." Further on, it likewise states
delegated by the CCDR. OPCON does include the authority the recognition that "all United States access to and use of
to delineate functional responsibilities and operational areas facilities and areas will be at the invitation of the Philippines
of subordinate JFCs. and with full respect for the Philippine Constitution and
Philippine laws."
Operational control is therefore the delegable aspect of
combatant command, while command and control is the The sensitivity of EDCA provisions to the laws of the
overall power and responsibility exercised by the commander Philippines must be seen in light of Philippine sovereignty and
with reference to a mission. Operational control is a narrower jurisdiction over the Agreed Locations.
power and must be given, while command and control is
plenary and vested in a commander. Operational control does Sovereignty is the possession of sovereign power,406 while
not include the planning, programming, budgeting, and jurisdiction is the conferment by law of power and authority to
execution process input; the assignment of subordinate apply the law.407 Article I of the 1987 Constitution states:
commanders; the building of relationships with Department of
Defense agencies; or the directive authority for logistics, The national territory comprises the Philippine archipelago,
with all the islands and waters embraced therein, and all other
FINALS CONSTITUTIONAL LAW I ACJUCO NOV 11, 2017 34

territories over which the Philippines has sovereignty or important, they claim that the Agreed Locations invite
jurisdiction, consisting of its terrestrial, fluvial, and aerial instances of attack on the Philippines from enemies of the
domains, including its territorial sea, the seabed, the subsoil, U.S.417
the insular shelves, and other submarine areas. The waters
around, between, and connecting the islands of the We believe that the raised fear of an attack on the Philippines
archipelago, regardless of their breadth and dimensions, form is not in the realm of law, but of politics and policy. At the very
part of the internal waters of the Philippines. (Emphasis least, we can say that under international law, EDCA does not
supplied) provide a legal basis for a justified attack on the Philippines.

From the text of EDCA itself, Agreed Locations are territories In the first place, international law disallows any attack on the
of the Philippines that the U.S. forces are allowed to access Agreed Locations simply because of the presence of U.S.
and use.408 By withholding ownership of these areas and personnel. Article 2(4) of the United Nations Charter states
retaining unrestricted access to them, the government asserts that "All Members shall refrain in their international relations
sovereignty over its territory. That sovereignty exists so long from the threat or use of force against the territorial integrity or
as the Filipino people exist.409 political independence of any state, or in any other manner
inconsistent with the Purposes of the United Nations."418 Any
Significantly, the Philippines retains primary responsibility for unlawful attack on the Philippines breaches the treaty, and
security with respect to the Agreed Locations.410 Hence, triggers Article 51 of the same charter, which guarantees the
Philippine law remains in force therein, and it cannot be said inherent right of individual or collective self-defence.
that jurisdiction has been transferred to the U.S. Even the
previously discussed necessary measures for operational Moreover, even if the lawfulness of the attack were not in
control and defense over U.S. forces must be coordinated with question, international humanitarian law standards prevent
Philippine authorities.411 participants in an armed conflict from targeting non-
participants. International humanitarian law, which is the
Jurisprudence bears out the fact that even under the former branch of international law applicable to armed conflict,
legal regime of the MBA, Philippine laws continue to be in expressly limits allowable military conduct exhibited by forces
force within the bases.412 The difference between then and of a participant in an armed conflict.419 Under this legal
now is that EDCA retains the primary jurisdiction of the regime, participants to an armed conflict are held to specific
Philippines over the security of the Agreed Locations, an standards of conduct that require them to distinguish between
important provision that gives it actual control over those combatants and non-combatants,420 as embodied by the
locations. Previously, it was the provost marshal of the U.S. Geneva Conventions and their Additional Protocols.421
who kept the peace and enforced Philippine law in the bases.
In this instance, Philippine forces act as peace officers, in Corollary to this point, Professor John Woodcliffe, professor of
stark contrast to the 1947 MBA provisions on jurisdiction.413 international law at the University of Leicester, noted that there
is no legal consensus for what constitutes a base, as opposed
iii. Third standard: must respect national security and territorial to other terms such as "facilities" or "installation."422 In
integrity strategic literature, "base" is defined as an installation "over
which the user State has a right to exclusive control in an
The last standard this Court must set is that the EDCA extraterritorial sense."423 Since this definition would exclude
provisions on the Agreed Locations must not impair or most foreign military installations, a more important distinction
threaten the national security and territorial integrity of the must be made.
Philippines.
For Woodcliffe, a type of installation excluded from the
This Court acknowledged in Bayan v. Zamora that the definition of "base" is one that does not fulfill a combat role.
evolution of technology has essentially rendered the prior He cites an example of the use of the territory of a state for
notion of permanent military bases obsolete. training purposes, such as to obtain experience in local
geography and climactic conditions or to carry out joint
Moreover, military bases established within the territory of exercises.424 Another example given is an advanced
another state is no longer viable because of the alternatives communications technology installation for purposes of
offered by new means and weapons of warfare such as information gathering and communication.425 Unsurprisingly,
nuclear weapons, guided missiles as well as huge sea vessels he deems these non-combat uses as borderline situations that
that can stay afloat in the sea even for months and years would be excluded from the functional understanding of
without returning to their home country. These military military bases and installations.426
warships are actually used as substitutes for a land-home
base not only of military aircraft but also of military personnel By virtue of this ambiguity, the laws of war dictate that the
and facilities. Besides, vessels are mobile as compared to a status of a building or person is presumed to be protected,
land-based military headquarters.414 unless proven otherwise.427 Moreover, the principle of
distinction requires combatants in an armed conflict to
The VFA serves as the basis for the entry of U.S. troops in a distinguish between lawful targets428 and protected
limited scope. It does not allow, for instance, the re- targets.429 In an actual armed conflict between the U.S. and
establishment of the Subic military base or the Clark Air Field a third state, the Agreed Locations cannot be considered U.S.
as U.S. military reservations. In this context, therefore, this territory, since ownership of territory even in times of armed
Court has interpreted the restrictions on foreign bases, troops, conflict does not change.430
or facilities as three independent restrictions. In accord with
this interpretation, each restriction must have its own Hence, any armed attack by forces of a third state against an
qualification. Agreed Location can only be legitimate under international
humanitarian law if it is against a bona fide U.S. military base,
Petitioners quote from the website http://en.wikipedia.org to facility, or installation that directly contributes to the military
define what a military base is.415 While the source is not effort of the U.S. Moreover, the third state's forces must take
authoritative, petitioners make the point that the Agreed all measures to ensure that they have complied with the
Locations, by granting access and use to U.S. forces and principle of distinction (between combatants and non-
contractors, are U.S. bases under a different name.416 More combatants).
FINALS CONSTITUTIONAL LAW I ACJUCO NOV 11, 2017 35

objection before this Court from the Senate or any of its


There is, then, ample legal protection for the Philippines under members.
international law that would ensure its territorial integrity and
national security in the event an Agreed Location is subjected The provisions in EDCA dealing with Agreed Locations are
to attack. As EDCA stands, it does not create the situation so analogous to those in the aforementioned executive
feared by petitioners - one in which the Philippines, while not agreements. Instead of authorizing the building of temporary
participating in an armed conflict, would be legitimately structures as previous agreements have done, EDCA
targeted by an enemy of the U.S.431 authorizes the U.S. to build permanent structures or alter or
improve existing ones for, and to be owned by, the
In the second place, this is a policy question about the wisdom Philippines.445 EDCA is clear that the Philippines retains
of allowing the presence of U.S. personnel within our territory ownership of altered or improved facilities and newly
and is therefore outside the scope of judicial review. constructed permanent or non-relocatable structures.446
Under EDCA, U.S. forces will also be allowed to use facilities
Evidently, the concept of giving foreign troops access to and areas for "training; x x x; support and related activities; x
"agreed" locations, areas, or facilities within the military base x x; temporary accommodation of personnel;
of another sovereign state is nothing new on the international communications" and agreed activities.447
plane. In fact, this arrangement has been used as the
framework for several defense cooperation agreements, such Concerns on national security problems that arise from foreign
as in the following: military equipment being present in the Philippines must
likewise be contextualized. Most significantly, the VFA already
1. 2006 U.S.-Bulgaria Defense Cooperation Agreement432 authorizes the presence of U.S. military equipment in the
country. Article VII of the VFA already authorizes the U.S. to
2. 2009 U.S.-Colombia Defense Cooperation Agreement433 import into or acquire in the Philippines "equipment, materials,
supplies, and other property" that will be used "in connection
3. 2009 U.S.-Poland Status of Forces Agreement434 with activities" contemplated therein. The same section also
recognizes that "[t]itle to such property shall remain" with the
4. 2014 U.S.-Australia Force Posture Agreement435 US and that they have the discretion to "remove such property
from the Philippines at any time."
5. 2014 U.S.-Afghanistan Security and Defense Cooperation
Agreement436 There is nothing novel, either, in the EDCA provision on the
prepositioning and storing of "defense equipment, supplies,
In all of these arrangements, the host state grants U.S. forces and materiel,"448 since these are sanctioned in the VFA. In
access to their military bases.437 That access is without rental fact, the two countries have already entered into various
or similar costs to the U.S.438 Further, U.S. forces are allowed implementing agreements in the past that are comparable to
to undertake construction activities in, and make alterations the present one. The Balikatan 02-1 Terms of Reference
and improvements to, the agreed locations, facilities, or mentioned in Lim v. Executive Secretary specifically
areas.439 As in EDCA, the host states retain ownership and recognizes that Philippine and U.S. forces "may share x x x in
jurisdiction over the said bases.440 the use of their resources, equipment and other assets." Both
the 2002 and 2007 Mutual Logistics Support Agreements
In fact, some of the host states in these agreements give speak of the provision of support and services, including the
specific military-related rights to the U.S. For example, under "construction and use of temporary structures incident to
Article IV(l) of the US.-Bulgaria Defense Cooperation operations support" and "storage services" during approved
Agreement, "the United States forces x x x are authorized activities.449 These logistic supplies, support, and services
access to and may use agreed facilities and areas x x x for include the "temporary use of x x x nonlethal items of military
staging and deploying of forces and materiel, with the purpose equipment which are not designated as significant military
of conducting x x x contingency operations and other equipment on the U.S. Munitions List, during an approved
missions, including those undertaken in the framework of the activity."450 Those activities include "combined exercises and
North Atlantic Treaty." In some of these agreements, host training, operations and other deployments" and "cooperative
countries allow U.S. forces to construct facilities for the latter’s efforts, such as humanitarian assistance, disaster relief and
exclusive use.441 rescue operations, and maritime anti-pollution operations"
within or outside Philippine territory.451 Under EDCA, the
Troop billeting, including construction of temporary structures, equipment, supplies, and materiel that will be prepositioned at
is nothing new. In Lim v. Executive Secretary, the Court Agreed Locations include "humanitarian assistance and
already upheld the Terms of Reference of Balikatan 02-1, disaster relief equipment, supplies, and materiel. "452 Nuclear
which authorized U.S. forces to set up "[t]emporary structures weapons are specifically excluded from the materiel that will
such as those for troop billeting, classroom instruction and be prepositioned.
messing x x x during the Exercise." Similar provisions are also
in the Mutual Logistics Support Agreement of 2002 and 2007, Therefore, there is no basis to invalidate EDCA on fears that
which are essentially executive agreements that implement it increases the threat to our national security. If anything,
the VFA, the MDT, and the 1953 Military Assistance EDCA increases the likelihood that, in an event requiring a
Agreement. These executive agreements similarly tackle the defensive response, the Philippines will be prepared
"reciprocal provision of logistic support, supplies, and alongside the U.S. to defend its islands and insure its territorial
services,"442 which include "[b ]illeting, x x x operations integrity pursuant to a relationship built on the MDT and VFA.
support (and construction and use of temporary structures
incident to operations support), training services, x x x storage 8. Others issues and concerns raised
services, x x x during an approved activity."443 These logistic
supplies, support, and services include temporary use of A point was raised during the oral arguments that the
"nonlethal items of military equipment which are not language of the MDT only refers to mutual help and defense
designated as significant military equipment on the U.S. in the Pacific area.453 We believe that any discussion of the
Munitions List, during an approved activity."444 The first activities to be undertaken under EDCA vis-a-vis the defense
Mutual Logistics Support Agreement has lapsed, while the of areas beyond the Pacific is premature. We note that a
second one has been extended until 2017 without any formal proper petition on that issue must be filed before we rule
FINALS CONSTITUTIONAL LAW I ACJUCO NOV 11, 2017 36

thereon. We also note that none of the petitions or in respect of foreign affairs. They cannot cripple him when he
memoranda has attempted to discuss this issue, except only deems that additional security measures are made necessary
to theorize that the U.S. will not come to our aid in the event by the times. As it stands, the Philippines through the
of an attack outside of the Pacific. This is a matter of policy Department of Foreign Affairs has filed several diplomatic
and is beyond the scope of this judicial review. protests against the actions of the People's Republic of China
in the West Philippine Sea;462 initiated arbitration against that
In reference to the issue on telecommunications, suffice it to country under the United Nations Convention on the Law of
say that the initial impression of the facility adverted to does the Sea;463 is in the process of negotiations with the Moro
appear to be one of those that require a public franchise by Islamic Liberation Front for peace in Southern Philippines,464
way of congressional action under Section 11, Article XII of which is the subject of a current case before this Court; and
the Constitution. As respondents submit, however, the system faces increasing incidents of kidnappings of Filipinos and
referred to in the agreement does not provide foreigners allegedly by the Abu Sayyaf or the New People's
telecommunications services to the public for Army.465 The Philippine military is conducting reforms that
compensation.454 It is clear from Article VIl(2) of EDCA that seek to ensure the security and safety of the nation in the
the telecommunication system is solely for the use of the U.S. years to come.466 In the future, the Philippines must navigate
and not the public in general, and that this system will not a world in which armed forces fight with increasing
interfere with that which local operators use. Consequently, a sophistication in both strategy and technology, while
public franchise is no longer necessary. employing asymmetric warfare and remote weapons.

Additionally, the charge that EDCA allows nuclear weapons Additionally, our country is fighting a most terrifying enemy:
within Philippine territory is entirely speculative. It is the backlash of Mother Nature. The Philippines is one of the
noteworthy that the agreement in fact specifies that the countries most directly affected and damaged by climate
prepositioned materiel shall not include nuclear weapons.455 change. It is no coincidence that the record-setting tropical
Petitioners argue that only prepositioned nuclear weapons are cyclone Yolanda (internationally named Haiyan), one of the
prohibited by EDCA; and that, therefore, the U.S. would most devastating forces of nature the world has ever seen hit
insidiously bring nuclear weapons to Philippine territory.456 the Philippines on 8 November 2013 and killed at least 6,000
The general prohibition on nuclear weapons, whether people.467 This necessitated a massive rehabilitation
prepositioned or not, is already expressed in the 1987 project.468 In the aftermath, the U.S. military was among the
Constitution.457 It would be unnecessary or superfluous to first to extend help and support to the Philippines.
include all prohibitions already in the Constitution or in the law
through a document like EDCA. That calamity brought out the best in the Filipinos as
thousands upon thousands volunteered their help, their
Finally, petitioners allege that EDCA creates a tax exemption, wealth, and their prayers to those affected. It also brought to
which under the law must originate from Congress. This the fore the value of having friends in the international
allegation ignores jurisprudence on the government's community.
assumption of tax liability. EDCA simply states that the taxes
on the use of water, electricity, and public utilities are for the In order to keep the peace in its archipelago in this region of
account of the Philippine Government.458 This provision the world, and to sustain itself at the same time against the
creates a situation in which a contracting party assumes the destructive forces of nature, the Philippines will need friends.
tax liability of the other.459 In National Power Corporation v. Who they are, and what form the friendships will take, are for
Province of Quezon, we distinguished between enforceable the President to decide. The only restriction is what the
and unenforceable stipulations on the assumption of tax Constitution itself expressly prohibits. It appears that this
liability. Afterwards, we concluded that an enforceable overarching concern for balancing constitutional requirements
assumption of tax liability requires the party assuming the against the dictates of necessity was what led to EDCA.
liability to have actual interest in the property taxed.460 This
rule applies to EDCA, since the Philippine Government stands As it is, EDCA is not constitutionally infirm. As an executive
to benefit not only from the structures to be built thereon or agreement, it remains consistent with existing laws and
improved, but also from the joint training with U.S. forces, treaties that it purports to implement.
disaster preparation, and the preferential use of Philippine
suppliers.461 Hence, the provision on the assumption of tax WHEREFORE, we hereby DISMISS the petitions.
liability does not constitute a tax exemption as petitioners have
posited. SO ORDERED.

Additional issues were raised by petitioners, all relating


principally to provisions already sufficiently addressed above.
This Court takes this occasion to emphasize that the
agreement has been construed herein as to absolutely
disauthorize the violation of the Constitution or any applicable
statute. On the contrary, the applicability of Philippine law is
explicit in EDCA.

EPILOGUE

The fear that EDCA is a reincarnation of the U.S. bases so


zealously protested by noted personalities in Philippine history
arises not so much from xenophobia, but from a genuine
desire for self-determination, nationalism, and above all a
commitment to ensure the independence of the Philippine
Republic from any foreign domination.

Mere fears, however, cannot curtail the exercise by the


President of the Philippines of his Constitutional prerogatives
FINALS CONSTITUTIONAL LAW I ACJUCO NOV 11, 2017 37

which is attached hereto and made an integral part hereof as


G.R. No. 175888 February 11, 2009 Annex "A," committed as follows:

SUZETTE NICOLAS y SOMBILON, Petitioner, "That on or about the First (1st) day of November 2005, inside
vs. the Subic Bay Freeport Zone, Olongapo City and within the
ALBERTO ROMULO, in his capacity as Secretary of jurisdiction of this Honorable Court, the above-named
Foreign Affairs; RAUL GONZALEZ, in his capacity as accused’s (sic), being then members of the United States
Secretary of Justice; EDUARDO ERMITA, in his capacity Marine Corps, except Timoteo L. Soriano, Jr., conspiring,
as Executive Secretary; RONALDO PUNO, in his capacity confederating together and mutually helping one another, with
as Secretary of the Interior and Local Government; lewd design and by means of force, threat and intimidation,
SERGIO APOSTOL, in his capacity as Presidential Legal with abuse of superior strength and taking advantage of the
Counsel; and L/CPL. DANIEL SMITH, Respondents. intoxication of the victim, did then and there willfully, unlawfully
and feloniously sexually abuse and have sexual intercourse
x - - - - - - - - - - - - - - - - - - - - - - -x with or carnal knowledge of one Suzette S. Nicolas, a 22-year
old unmarried woman inside a Starex Van with Plate No.
G.R. No. 176051 February 11, 2009 WKF-162, owned by Starways Travel and Tours, with Office
address at 8900 P. Victor St., Guadalupe, Makati City, and
JOVITO R. SALONGA, WIGBERTO E. TAÑADA, JOSE DE driven by accused Timoteo L. Soriano, Jr., against the will and
LA RAMA, EMILIO C. CAPULONG, H. HARRY L. ROQUE, consent of the said Suzette S. Nicolas, to her damage and
JR., FLORIN HILBAY, and BENJAMIN POZON, Petitioners, prejudice.
vs.
DANIEL SMITH, SECRETARY RAUL GONZALEZ, CONTRARY TO LAW."1
PRESIDENTIAL LEGAL COUNSEL SERGIO APOSTOL,
SECRETARY RONALDO PUNO, SECRETARY ALBERTO Pursuant to the Visiting Forces Agreement (VFA) between the
ROMULO, The Special 16th Division of the COURT OF Republic of the Philippines and the United States, entered into
APPEALS, and all persons acting in their capacity, on February 10, 1998, the United States, at its request, was
Respondents. granted custody of defendant Smith pending the proceedings.

x - - - - - - - - - - - - - - - - - - - - - - -x During the trial, which was transferred from the Regional Trial
Court (RTC) of Zambales to the RTC of Makati for security
G.R. No. 176222 February 11, 2009 reasons, the United States Government faithfully complied
with its undertaking to bring defendant Smith to the trial court
BAGONG ALYANSANG MAKABAYAN (BAYAN), every time his presence was required.
represented by Dr. Carol Araullo; GABRIELA, represented by
Emerenciana de Jesus; BAYAN MUNA, represented by Rep. On December 4, 2006, the RTC of Makati, following the end
Satur Ocampo; GABRIELA WOMEN'S PARTY, represented of the trial, rendered its Decision, finding defendant Smith
by Rep. Liza Maza; KILUSANG MAYO UNO (KMU), guilty, thus:
represented by Elmer Labog; KILUSANG MAGBUBUKID NG
PILIPINAS (KMP), represented by Willy Marbella; LEAGUE WHEREFORE, premises considered, for failure of the
OF FILIPINO STUDENTS (LFS), represented by Vencer prosecution to adduce sufficient evidence against accused
Crisostomo; and THE PUBLIC INTEREST LAW CENTER, S/SGT. CHAD BRIAN CARPENTER, L/CPL. KEITH
represented by Atty. Rachel Pastores, Petitioners, SILKWOOD AND L/CPL. DOMINIC DUPLANTIS, all of the US
vs. Marine Corps assigned at the USS Essex, are hereby
PRESIDENT GLORIA MACAPAGAL-ARROYO, in her ACQUITTED to the crime charged.
capacity as concurrent Defense Secretary, EXECUTIVE
SECRETARY EDUARDO ERMITA, FOREIGN AFFAIRS The prosecution having presented sufficient evidence against
SECRETARY ALBERTO ROMULO, JUSTICE SECRETARY accused L/CPL. DANIEL J. SMITH, also of the US Marine
RAUL GONZALEZ, AND INTERIOR AND LOCAL Corps at the USS Essex, this Court hereby finds him GUILTY
GOVERNMENT SECRETARY RONALDO PUNO, BEYOND REASONABLE DOUBT of the crime of RAPE
Respondents. defined under Article 266-A, paragraph 1 (a) of the Revised
Penal Code, as amended by R.A. 8353, and, in accordance
DECISION with Article 266-B, first paragraph thereof, hereby sentences
him to suffer the penalty of reclusion perpetua together with
AZCUNA, J.: the accessory penalties provided for under Article 41 of the
same Code.
These are petitions for certiorari, etc. as special civil actions
and/or for review of the Decision of the Court of Appeals in Pursuant to Article V, paragraph No. 10, of the Visiting Forces
Lance Corporal Daniel J. Smith v. Hon. Benjamin T. Pozon, et Agreement entered into by the Philippines and the United
al., in CA-G.R. SP No. 97212, dated January 2, 2007. States, accused L/CPL. DANIEL J. SMITH shall serve his
sentence in the facilities that shall, thereafter, be agreed upon
The facts are not disputed. by appropriate Philippine and United States authorities.
Pending agreement on such facilities, accused L/CPL.
Respondent Lance Corporal (L/CPL) Daniel Smith is a DANIEL J. SMITH is hereby temporarily committed to the
member of the United States Armed Forces. He was charged Makati City Jail.
with the crime of rape committed against a Filipina, petitioner
herein, sometime on November 1, 2005, as follows: Accused L/CPL. DANIEL J. SMITH is further sentenced to
indemnify complainant SUZETTE S. NICOLAS in the amount
The undersigned accused LCpl. Daniel Smith, Ssgt. Chad of ₱50,000.00 as compensatory damages plus ₱50,000.00 as
Brian Carpentier, Dominic Duplantis, Keith Silkwood and moral damages.
Timoteo L. Soriano, Jr. of the crime of Rape under Article 266-
A of the Revised Penal Code, as amended by Republic Act SO ORDERED.2
8353, upon a complaint under oath filed by Suzette S. Nicolas,
FINALS CONSTITUTIONAL LAW I ACJUCO NOV 11, 2017 38

As a result, the Makati court ordered Smith detained at the


Makati jail until further orders. Sec. 25. After the expiration in 1991 of the Agreement
between the Philippines and the United States of America
On December 29, 2006, however, defendant Smith was taken concerning Military Bases, foreign military bases, troops, or
out of the Makati jail by a contingent of Philippine law facilities shall not be allowed in the Philippines except under a
enforcement agents, purportedly acting under orders of the treaty duly concurred in by the Senate and, when the
Department of the Interior and Local Government, and Congress so requires, ratified by a majority of the votes cast
brought to a facility for detention under the control of the by the people in a national referendum held for that purpose,
United States government, provided for under new and recognized as a treaty by the other contracting State.
agreements between the Philippines and the United States,
referred to as the Romulo-Kenney Agreement of December The reason for this provision lies in history and the Philippine
19, 2006 which states: experience in regard to the United States military bases in the
country.
The Government of the Republic of the Philippines and the
Government of the United States of America agree that, in It will be recalled that under the Philippine Bill of 1902, which
accordance with the Visiting Forces Agreement signed laid the basis for the Philippine Commonwealth and,
between our two nations, Lance Corporal Daniel J. Smith, eventually, for the recognition of independence, the United
United States Marine Corps, be returned to U.S. military States agreed to cede to the Philippines all the territory it
custody at the U.S. Embassy in Manila. acquired from Spain under the Treaty of Paris, plus a few
islands later added to its realm, except certain naval ports
(Sgd.) Kristie A. Kenney and/or military bases and facilities, which the United States
Representative of the United States of America retained for itself.

DATE: 12-19-06 (Sgd.) Alberto G. Romulo This is noteworthy, because what this means is that Clark and
Representative of the Republic of the Philippines Subic and the other places in the Philippines covered by the
RP-US Military Bases Agreement of 1947 were not Philippine
DATE: December 19, 2006 territory, as they were excluded from the cession and retained
and the Romulo-Kenney Agreement of December 22, 2006 by the US.
which states:
Accordingly, the Philippines had no jurisdiction over these
The Department of Foreign Affairs of the Republic of the bases except to the extent allowed by the United States.
Philippines and the Embassy of the United States of America Furthermore, the RP-US Military Bases Agreement was never
agree that, in accordance with the Visiting Forces Agreement advised for ratification by the United States Senate, a disparity
signed between the two nations, upon transfer of Lance in treatment, because the Philippines regarded it as a treaty
Corporal Daniel J. Smith, United States Marine Corps, from and had it concurred in by our Senate.
the Makati City Jail, he will be detained at the first floor, Rowe
(JUSMAG) Building, U.S. Embassy Compound in a room of Subsequently, the United States agreed to turn over these
approximately 10 x 12 square feet. He will be guarded round bases to the Philippines; and with the expiration of the RP-US
the clock by U.S. military personnel. The Philippine police and Military Bases Agreement in 1991, the territory covered by
jail authorities, under the direct supervision of the Philippine these bases were finally ceded to the Philippines.
Department of Interior and Local Government (DILG) will have
access to the place of detention to ensure the United States To prevent a recurrence of this experience, the provision in
is in compliance with the terms of the VFA. question was adopted in the 1987 Constitution.

The matter was brought before the Court of Appeals which The provision is thus designed to ensure that any agreement
decided on January 2, 2007, as follows: allowing the presence of foreign military bases, troops or
facilities in Philippine territory shall be equally binding on the
WHEREFORE, all the foregoing considered, we resolved to Philippines and the foreign sovereign State involved. The idea
DISMISS the petition for having become moot.3 is to prevent a recurrence of the situation in which the terms
and conditions governing the presence of foreign armed
Hence, the present actions. forces in our territory were binding upon us but not upon the
foreign State.
The petitions were heard on oral arguments on September 19,
2008, after which the parties submitted their memoranda. Applying the provision to the situation involved in these cases,
the question is whether or not the presence of US Armed
Petitioners contend that the Philippines should have custody Forces in Philippine territory pursuant to the VFA is allowed
of defendant L/CPL Smith because, first of all, the VFA is void "under a treaty duly concurred in by the Senate xxx and
and unconstitutional. recognized as a treaty by the other contracting State."

This issue had been raised before, and this Court resolved in This Court finds that it is, for two reasons.
favor of the constitutionality of the VFA. This was in Bayan v.
Zamora,4 brought by Bayan, one of petitioners in the present First, as held in Bayan v. Zamora,5 the VFA was duly
cases. concurred in by the Philippine Senate and has been
recognized as a treaty by the United States as attested and
Against the barriers of res judicata vis-à-vis Bayan, and stare certified by the duly authorized representative of the United
decisis vis-à-vis all the parties, the reversal of the previous States government.
ruling is sought on the ground that the issue is of primordial
importance, involving the sovereignty of the Republic, as well The fact that the VFA was not submitted for advice and
as a specific mandate of the Constitution. consent of the United States Senate does not detract from its
status as a binding international agreement or treaty
The provision of the Constitution is Art. XVIII, Sec. 25 which recognized by the said State. For this is a matter of internal
states: United States law. Notice can be taken of the internationally
FINALS CONSTITUTIONAL LAW I ACJUCO NOV 11, 2017 39

known practice by the United States of submitting to its Senate own peace and safety and declares that it would act to meet
for advice and consent agreements that are policymaking in the common dangers in accordance with its constitutional
nature, whereas those that carry out or further implement processes.
these policymaking agreements are merely submitted to
Congress, under the provisions of the so-called Case– Any such armed attack and all measures taken as a result
Zablocki Act, within sixty days from ratification.6 thereof shall be immediately reported to the Security Council
of the United Nations. Such measures shall be terminated
The second reason has to do with the relation between the when the Security Council has taken the measures necessary
VFA and the RP-US Mutual Defense Treaty of August 30, to restore and maintain international peace and security.
1951. This earlier agreement was signed and duly ratified with
the concurrence of both the Philippine Senate and the United Article V. For the purpose of Article IV, an armed attack on
States Senate. either of the Parties is deemed to include an armed attack on
the metropolitan territory of either of the Parties, or on the
The RP-US Mutual Defense Treaty states:7 island territories under its jurisdiction in the Pacific Ocean, its
armed forces, public vessels or aircraft in the Pacific.
MUTUAL DEFENSE TREATY BETWEEN THE REPUBLIC
OF THE PHILIPPINES AND THE UNITED STATES OF Article VI. This Treaty does not affect and shall not be
AMERICA. Signed at Washington, August 30, 1951. interpreted as affecting in any way the rights and obligations
of the Parties under the Charter of the United Nations or the
The Parties of this Treaty responsibility of the United Nations for the maintenance of
international peace and security.
Reaffirming their faith in the purposes and principles of the
Charter of the United Nations and their desire to live in peace Article VII. This Treaty shall be ratified by the Republic of the
with all peoples and all governments, and desiring to Philippines and the United Nations of America in accordance
strengthen the fabric of peace in the Pacific area. with their respective constitutional processes and will come
into force when instruments of ratification thereof have been
Recalling with mutual pride the historic relationship which exchanged by them at Manila.
brought their two peoples together in a common bond of
sympathy and mutual ideals to fight side-by-side against Article VIII. This Treaty shall remain in force indefinitely. Either
imperialist aggression during the last war. Party may terminate it one year after notice has been given to
the other party.
Desiring to declare publicly and formally their sense of unity
and their common determination to defend themselves In withness whereof the undersigned Plenipotentiaries have
against external armed attack, so that no potential aggressor signed this Treaty.
could be under the illusion that either of them stands alone in
the Pacific area. Done in duplicate at Washington this thirtieth day of August,
1951.
Desiring further to strengthen their present efforts for
collective defense for the preservation of peace and security For the Republic of the Philippines:
pending the development of a more comprehensive system of
regional security in the Pacific area. (Sgd.) Carlos P. Romulo

Agreeing that nothing in this present instrument shall be (Sgd.) Joaquin M. Elizalde
considered or interpreted as in any way or sense altering or
diminishing any existing agreements or understandings (Sgd.) Vicente J. Francisco
between the Republic of the Philippines and the United States
of America. (Sgd.) Diosdado Macapagal

Have agreed as follows: For the United States of America:

Article I. The parties undertake, as set forth in the Charter of (Sgd.) Dean Acheson
the United Nations, to settle any international disputes in
which they may be involved by peaceful means in such a (Sgd.) John Foster Dulles
manner that international peace and security and justice are
not endangered and to refrain in their international relation (Sgd.) Tom Connally
from the threat or use of force in any manner inconsistent with
the purposes of the United Nations. (Sgd.) Alexander Wiley8

Article II. In order more effectively to achieve the objective of Clearly, therefore, joint RP-US military exercises for the
this Treaty, the Parties separately and jointly by self-help and purpose of developing the capability to resist an armed attack
mutual aid will maintain and develop their individual and fall squarely under the provisions of the RP-US Mutual
collective capacity to resist armed attack. Defense Treaty. The VFA, which is the instrument agreed
upon to provide for the joint RP-US military exercises, is
Article III. The Parties, through their Foreign Ministers or their simply an implementing agreement to the main RP-US Military
deputies, will consult together from time to time regarding the Defense Treaty. The Preamble of the VFA states:
implementation of this Treaty and whenever in the opinion of
either of them the territorial integrity, political independence or The Government of the United States of America and the
security of either of the Parties is threatened by external Government of the Republic of the Philippines,
armed attack in the Pacific.
Reaffirming their faith in the purposes and principles of the
Article IV. Each Party recognizes that an armed attack in the Charter of the United Nations and their desire to strengthen
Pacific area on either of the parties would be dangerous to its international and regional security in the Pacific area;
FINALS CONSTITUTIONAL LAW I ACJUCO NOV 11, 2017 40

exclusive power of this Court to adopt rules of procedure for


Reaffirming their obligations under the Mutual Defense Treaty all courts in the Philippines (Art. VIII, Sec. 5[5]). They argue
of August 30, 1951; that to allow the transfer of custody of an accused to a foreign
power is to provide for a different rule of procedure for that
Noting that from time to time elements of the United States accused, which also violates the equal protection clause of the
armed forces may visit the Republic of the Philippines; Constitution (Art. III, Sec. 1.).

Considering that cooperation between the United States and Again, this Court finds no violation of the Constitution.
the Republic of the Philippines promotes their common
security interests; The equal protection clause is not violated, because there is
a substantial basis for a different treatment of a member
Recognizing the desirability of defining the treatment of United of a foreign military armed forces allowed to enter our
States personnel visiting the Republic of the Philippines; territory and all other accused.11

Have agreed as follows:9 The rule in international law is that a foreign armed forces
allowed to enter one’s territory is immune from local
Accordingly, as an implementing agreement of the RP-US jurisdiction, except to the extent agreed upon. The Status of
Mutual Defense Treaty, it was not necessary to submit the Forces Agreements involving foreign military units around the
VFA to the US Senate for advice and consent, but merely to world vary in terms and conditions, according to the situation
the US Congress under the Case–Zablocki Act within 60 days of the parties involved, and reflect their bargaining power. But
of its ratification. It is for this reason that the US has certified the principle remains, i.e., the receiving State can exercise
that it recognizes the VFA as a binding international jurisdiction over the forces of the sending State only to the
agreement, i.e., a treaty, and this substantially complies with extent agreed upon by the parties.12
the requirements of Art. XVIII, Sec. 25 of our Constitution.10
As a result, the situation involved is not one in which the power
The provision of Art. XVIII, Sec. 25 of the Constitution, is of this Court to adopt rules of procedure is curtailed or
complied with by virtue of the fact that the presence of the US violated, but rather one in which, as is normally encountered
Armed Forces through the VFA is a presence "allowed under" around the world, the laws (including rules of procedure) of
the RP-US Mutual Defense Treaty. Since the RP-US Mutual one State do not extend or apply – except to the extent agreed
Defense Treaty itself has been ratified and concurred in by upon – to subjects of another State due to the recognition of
both the Philippine Senate and the US Senate, there is no extraterritorial immunity given to such bodies as visiting
violation of the Constitutional provision resulting from such foreign armed forces.
presence.
Nothing in the Constitution prohibits such agreements
The VFA being a valid and binding agreement, the parties are recognizing immunity from jurisdiction or some aspects of
required as a matter of international law to abide by its terms jurisdiction (such as custody), in relation to long-recognized
and provisions. subjects of such immunity like Heads of State, diplomats and
members of the armed forces contingents of a foreign State
The VFA provides that in cases of offenses committed by the allowed to enter another State’s territory. On the contrary, the
members of the US Armed Forces in the Philippines, the Constitution states that the Philippines adopts the generally
following rules apply: accepted principles of international law as part of the law of
the land. (Art. II, Sec. 2).
Article V
Applying, however, the provisions of VFA, the Court finds that
Criminal Jurisdiction there is a different treatment when it comes to detention as
against custody. The moment the accused has to be detained,
xxx e.g., after conviction, the rule that governs is the following
provision of the VFA:
6. The custody of any United States personnel over whom the
Philippines is to exercise jurisdiction shall immediately reside Article V
with United States military authorities, if they so request, from
the commission of the offense until completion of all judicial Criminal Jurisdiction
proceedings. United States military authorities shall, upon
formal notification by the Philippine authorities and without xxx
delay, make such personnel available to those authorities in
time for any investigative or judicial proceedings relating to the Sec. 10. The confinement or detention by Philippine
offense with which the person has been charged. In authorities of United States personnel shall be carried out in
extraordinary cases, the Philippine Government shall present facilities agreed on by appropriate Philippines and United
its position to the United States Government regarding States authorities. United States personnel serving sentences
custody, which the United States Government shall take into in the Philippines shall have the right to visits and material
full account. In the event Philippine judicial proceedings are assistance.
not completed within one year, the United States shall be
relieved of any obligations under this paragraph. The one year It is clear that the parties to the VFA recognized the difference
period will not include the time necessary to appeal. Also, the between custody during the trial and detention after
one year period will not include any time during which conviction, because they provided for a specific arrangement
scheduled trial procedures are delayed because United to cover detention. And this specific arrangement clearly
States authorities, after timely notification by Philippine states not only that the detention shall be carried out in
authorities to arrange for the presence of the accused, fail to facilities agreed on by authorities of both parties, but also that
do so. the detention shall be "by Philippine authorities." Therefore,
the Romulo-Kenney Agreements of December 19 and 22,
Petitioners contend that these undertakings violate another 2006, which are agreements on the detention of the accused
provision of the Constitution, namely, that providing for the
FINALS CONSTITUTIONAL LAW I ACJUCO NOV 11, 2017 41

in the United States Embassy, are not in accord with the VFA registrable under the Case-Zablocki Act, and thus lack
itself because such detention is not "by Philippine authorities." legislative implementing authority.

Respondents should therefore comply with the VFA and Finally, the RP-US Mutual Defense Treaty was advised and
negotiate with representatives of the United States towards an consented to by the US Senate on March 20, 1952, as
agreement on detention facilities under Philippine authorities reflected in the US Congressional Record, 82nd Congress,
as mandated by Art. V, Sec. 10 of the VFA. Second Session, Vol. 98 – Part 2, pp. 2594-2595.

Next, the Court addresses the recent decision of the United The framers of the Constitution were aware that the
States Supreme Court in Medellin v. Texas ( 552 US ___ No. application of international law in domestic courts varies from
06-984, March 25, 2008), which held that treaties entered into country to country.
by the United States are not automatically part of their
domestic law unless these treaties are self-executing or there As Ward N. Ferdinandusse states in his Treatise, DIRECT
is an implementing legislation to make them APPLICATION OF INTERNATIONAL CRIMINAL LAW IN
enforceable.1avvphi1 NATIONAL COURTS, some countries require legislation
whereas others do not.
On February 3, 2009, the Court issued a Resolution, thus:
It was not the intention of the framers of the 1987 Constitution,
"G.R. No. 175888 (Suzette Nicolas y Sombilon v. Alberto in adopting Article XVIII, Sec. 25, to require the other
Romulo, et al.); G.R. No. 176051 (Jovito R. Salonga, et al. v. contracting State to convert their system to achieve alignment
Daniel Smith, et al.); and G.R. No. 176222 (Bagong and parity with ours. It was simply required that the treaty be
Alyansang Makabayan [BAYAN], et al. v. President Gloria recognized as a treaty by the other contracting State. With
Macapagal-Arroyo, et al.). that, it becomes for both parties a binding international
obligation and the enforcement of that obligation is left to the
The parties, including the Solicitor General, are required to normal recourse and processes under international law.
submit within three (3) days a Comment/Manifestation on the
following points: Furthermore, as held by the US Supreme Court in Weinberger
v. Rossi,13 an executive agreement is a "treaty" within the
1. What is the implication on the RP-US Visiting Forces meaning of that word in international law and constitutes
Agreement of the recent US Supreme Court decision in Jose enforceable domestic law vis-à-vis the United States. Thus,
Ernesto Medellin v. Texas, dated March 25, 2008, to the effect the US Supreme Court in Weinberger enforced the provisions
that treaty stipulations that are not self-executory can only be of the executive agreement granting preferential employment
enforced pursuant to legislation to carry them into effect; and to Filipinos in the US Bases here.
that, while treaties may comprise international commitments,
they are not domestic law unless Congress has enacted Accordingly, there are three types of treaties in the American
implementing statutes or the treaty itself conveys an intention system:
that it be "self-executory" and is ratified on these terms?
1. Art. II, Sec. 2 treaties – These are advised and consented
2. Whether the VFA is enforceable in the US as domestic law, to by the US Senate in accordance with Art. II, Sec. 2 of the
either because it is self-executory or because there exists US Constitution.
legislation to implement it.
2. Executive–Congressional Agreements: These are joint
3. Whether the RP-US Mutual Defense Treaty of August 30, agreements of the President and Congress and need not be
1951 was concurred in by the US Senate and, if so, is there submitted to the Senate.
proof of the US Senate advice and consent resolution?
Peralta, J., no part." 3. Sole Executive Agreements. – These are agreements
entered into by the President. They are to be submitted to
After deliberation, the Court holds, on these points, as follows: Congress within sixty (60) days of ratification under the
provisions of the Case-Zablocki Act, after which they are
First, the VFA is a self-executing Agreement, as that term recognized by the Congress and may be implemented.
is defined in Medellin itself, because the parties intend its
provisions to be enforceable, precisely because the As regards the implementation of the RP-US Mutual Defense
Agreement is intended to carry out obligations and Treaty, military aid or assistance has been given under it and
undertakings under the RP-US Mutual Defense Treaty. As this can only be done through implementing legislation. The
a matter of fact, the VFA has been implemented and VFA itself is another form of implementation of its provisions.
executed, with the US faithfully complying with its obligation to
produce L/CPL Smith before the court during the trial. WHEREFORE, the petitions are PARTLY GRANTED, and the
Court of Appeals’ Decision in CA-G.R. SP No. 97212 dated
Secondly, the VFA is covered by implementing legislation, January 2, 2007 is MODIFIED. The Visiting Forces Agreement
namely, the Case-Zablocki Act, USC Sec. 112(b), (VFA) between the Republic of the Philippines and the United
inasmuch as it is the very purpose and intent of the US States, entered into on February 10, 1998, is UPHELD as
Congress that executive agreements registered under constitutional, but the Romulo-Kenney Agreements of
this Act within 60 days from their ratification be December 19 and 22, 2006 are DECLARED not in
immediately implemented. The parties to these present accordance with the VFA, and respondent Secretary of
cases do not question the fact that the VFA has been Foreign Affairs is hereby ordered to forthwith negotiate with
registered under the Case-Zablocki Act.1avvphi1 the United States representatives for the appropriate
agreement on detention facilities under Philippine authorities
In sum, therefore, the VFA differs from the Vienna as provided in Art. V, Sec. 10 of the VFA, pending which the
Convention on Consular Relations and the Avena status quo shall be maintained until further orders by this
decision of the International Court of Justice (ICJ), Court.
subject matter of the Medellin decision. The Convention
and the ICJ decision are not self-executing and are not
FINALS CONSTITUTIONAL LAW I ACJUCO NOV 11, 2017 42

The Court of Appeals is hereby directed to resolve without


delay the related matters pending therein, namely, the petition
for contempt and the appeal of L/CPL Daniel Smith from the
judgment of conviction.

No costs.

SO ORDERED.
FINALS CONSTITUTIONAL LAW I ACJUCO NOV 11, 2017 43

G.R. No. 162230 April 28, 2010 This is an original Petition for Certiorari under Rule 65 of the
Rules of Court with an application for the issuance of a writ of
ISABELITA C. VINUYA, VICTORIA C. DELA PEÑA, preliminary mandatory injunction against the Office of the
HERMINIHILDA MANIMBO, LEONOR H. SUMAWANG, Executive Secretary, the Secretary of the Department of
CANDELARIA L. SOLIMAN, MARIA L. QUILANTANG, Foreign Affairs (DFA), the Secretary of the Department of
MARIA L. MAGISA, NATALIA M. ALONZO, LOURDES M. Justice (DOJ), and the Office of the Solicitor General (OSG).
NAVARO, FRANCISCA M. ATENCIO, ERLINDA
MANALASTAS, TARCILA M. SAMPANG, ESTER M. Petitioners are all members of the MALAYA LOLAS, a non-
PALACIO, MAXIMA R. DELA CRUZ, BELEN A. SAGUM, stock, non-profit organization registered with the Securities
FELICIDAD TURLA, FLORENCIA M. DELA PEÑA, and Exchange Commission, established for the purpose of
EUGENIA M. LALU, JULIANA G. MAGAT, CECILIA providing aid to the victims of rape by Japanese military forces
SANGUYO, ANA ALONZO, RUFINA P. MALLARI, in the Philippines during the Second World War.
ROSARIO M. ALARCON, RUFINA C. GULAPA, ZOILA B.
MANALUS, CORAZON C. CALMA, MARTA A. GULAPA, Petitioners narrate that during the Second World War, the
TEODORA M. HERNANDEZ, FERMIN B. DELA PEÑA, Japanese army attacked villages and systematically raped the
MARIA DELA PAZ B. CULALA, ESPERANZA MANAPOL, women as part of the destruction of the village. Their
JUANITA M. BRIONES, VERGINIA M. GUEVARRA, communities were bombed, houses were looted and burned,
MAXIMA ANGULO, EMILIA SANGIL, TEOFILA R. and civilians were publicly tortured, mutilated, and
PUNZALAN, JANUARIA G. GARCIA, PERLA B. BALINGIT, slaughtered. Japanese soldiers forcibly seized the women
BELEN A. CULALA, PILAR Q. GALANG, ROSARIO C. and held them in houses or cells, where they were repeatedly
BUCO, GAUDENCIA C. DELA PEÑA, RUFINA Q. raped, beaten, and abused by Japanese soldiers. As a result
CATACUTAN, FRANCIA A. BUCO, PASTORA C. of the actions of their Japanese tormentors, the petitioners
GUEVARRA, VICTORIA M. DELA CRUZ, PETRONILA O. have spent their lives in misery, having endured physical
DELA CRUZ, ZENAIDA P. DELA CRUZ, CORAZON M. injuries, pain and disability, and mental and emotional
SUBA, EMERINCIANA A. VINUYA, LYDIA A. SANCHEZ, suffering.2
ROSALINA M. BUCO, PATRICIA A. BERNARDO, LUCILA
H. PAYAWAL, MAGDALENA LIWAG, ESTER C. BALINGIT, Petitioners claim that since 1998, they have approached the
JOVITA A. DAVID, EMILIA C. MANGILIT, VERGINIA M. Executive Department through the DOJ, DFA, and OSG,
BANGIT, GUILLERMA S. BALINGIT, TERECITA requesting assistance in filing a claim against the Japanese
PANGILINAN, MAMERTA C. PUNO, CRISENCIANA C. officials and military officers who ordered the establishment of
GULAPA, SEFERINA S. TURLA, MAXIMA B. TURLA, the "comfort women" stations in the Philippines. However,
LEONICIA G. GUEVARRA, ROSALINA M. CULALA, officials of the Executive Department declined to assist
CATALINA Y. MANIO, MAMERTA T. SAGUM, CARIDAD L. the petitioners, and took the position that the individual
TURLA, et al. In their capacity and as members of the claims of the comfort women for compensation had
"Malaya Lolas Organization", Petitioners, already been fully satisfied by Japan’s compliance with
vs. the Peace Treaty between the Philippines and Japan.
THE HONORABLE EXECUTIVE SECRETARY ALBERTO
G. ROMULO, THE HONORABLE SECRETARY OF Issues
FOREIGN AFFAIRS DELIA DOMINGO-ALBERT, THE
HONORABLE SECRETARY OF JUSTICE MERCEDITAS N. Hence, this petition where petitioners pray for this court to (a)
GUTIERREZ, and THE HONORABLE SOLICITOR declare that respondents committed grave abuse of discretion
GENERAL ALFREDO L. BENIPAYO, Respondents. amounting to lack or excess of discretion in refusing to
espouse their claims for the crimes against humanity and war
DECISION crimes committed against them; and (b) compel the
respondents to espouse their claims for official apology and
DEL CASTILLO, J.: other forms of reparations against Japan before the
International Court of Justice (ICJ) and other international
The Treaty of Peace with Japan, insofar as it barred future tribunals.
claims such as those asserted by plaintiffs in these actions,
exchanged full compensation of plaintiffs for a future peace. Petitioners’ arguments
History has vindicated the wisdom of that bargain. And while
full compensation for plaintiffs' hardships, in the purely Petitioners argue that the general waiver of claims made by
economic sense, has been denied these former prisoners and the Philippine government in the Treaty of Peace with Japan
countless other survivors of the war, the immeasurable bounty is void. They claim that the comfort women system
of life for themselves and their posterity in a free society and established by Japan, and the brutal rape and enslavement of
in a more peaceful world services the debt.1 petitioners constituted a crime against humanity,3 sexual
slavery,4 and torture.5 They allege that the prohibition against
There is a broad range of vitally important areas that must be these international crimes is jus cogens norms from which no
regularly decided by the Executive Department without either derogation is possible; as such, in waiving the claims of
challenge or interference by the Judiciary. One such area Filipina comfort women and failing to espouse their complaints
involves the delicate arena of foreign relations. It would be against Japan, the Philippine government is in breach of its
strange indeed if the courts and the executive spoke with legal obligation not to afford impunity for crimes against
different voices in the realm of foreign policy. Precisely humanity. Finally, petitioners assert that the Philippine
because of the nature of the questions presented, and the government’s acceptance of the "apologies" made by Japan
lapse of more than 60 years since the conduct complained of, as well as funds from the Asian Women’s Fund (AWF) were
we make no attempt to lay down general guidelines covering contrary to international law.
other situations not involved here, and confine the opinion only
to the very questions necessary to reach a decision on this Respondents’ Arguments
matter.
Respondents maintain that all claims of the Philippines and its
Factual Antecedents nationals relative to the war were dealt with in the San
FINALS CONSTITUTIONAL LAW I ACJUCO NOV 11, 2017 44

Francisco Peace Treaty of 1951 and the bilateral Reparations through a series of lawsuits, discussion at the United Nations
Agreement of 1956.6 (UN), resolutions by various nations, and the Women’s
International Criminal Tribunal. The Japanese government, in
Article 14 of the Treaty of Peace7 provides: turn, responded through a series of public apologies and the
creation of the AWF.19
Article 14. Claims and Property
Lawsuits
a) It is recognized that Japan should pay reparations to the
Allied Powers for the damage and suffering caused by it In December 1991, Kim Hak-Sun and two other survivors filed
during the war. Nevertheless it is also recognized that the the first lawsuit in Japan by former comfort women against the
resources of Japan are not presently sufficient, if it is to Japanese government. The Tokyo District Court however
maintain a viable economy, to make complete reparation for dismissed their case.20 Other suits followed,21 but the
all such damage and suffering and at the present time meet Japanese government has, thus far, successfully caused the
its other obligations. dismissal of every case.22

b) Except as otherwise provided in the present Treaty, the Undoubtedly frustrated by the failure of litigation before
Allied Powers waive all reparations claims of the Allied Japanese courts, victims of the comfort women system
Powers, other claims of the Allied Powers and their nationals brought their claims before the United States (US). On
arising out of any actions taken by Japan and its nationals in September 18, 2000, 15 comfort women filed a class action
the course of the prosecution of the war, and claims of the lawsuit in the US District Court for the District of Columbia23
Allied Powers for direct military costs of occupation. "seeking money damages for [allegedly] having been
subjected to sexual slavery and torture before and during
In addition, respondents argue that the apologies made by World War II," in violation of "both positive and customary
Japan8 have been satisfactory, and that Japan had international law." The case was filed pursuant to the Alien
addressed the individual claims of the women through the Tort Claims Act ("ATCA"),24 which allowed the plaintiffs to sue
atonement money paid by the Asian Women’s Fund.1avvphi1 the Japanese government in a US federal district court.25 On
October 4, 2001, the district court dismissed the lawsuit due
Historical Background to lack of jurisdiction over Japan, stating that "[t]here is no
question that this court is not the appropriate forum in which
The comfort women system was the tragic legacy of the Rape plaintiffs may seek to reopen x x x discussions nearly half a
of Nanking. In December 1937, Japanese military forces century later x x x [E]ven if Japan did not enjoy sovereign
captured the city of Nanking in China and began a "barbaric immunity, plaintiffs' claims are non-justiciable and must be
campaign of terror" known as the Rape of Nanking, which dismissed."
included the rapes and murders of an estimated 20,000 to
80,000 Chinese women, including young girls, pregnant The District of Columbia Court of Appeals affirmed the lower
mothers, and elderly women.9 Document1zzF24331552898 court's dismissal of the case.26 On appeal, the US Supreme
Court granted the women’s petition for writ of certiorari,
In reaction to international outcry over the incident, the vacated the judgment of the District of Columbia Court of
Japanese government sought ways to end international Appeals, and remanded the case.27 On remand, the Court of
condemnation10 by establishing the "comfort women" Appeals affirmed its prior decision, noting that "much as we
system. Under this system, the military could simultaneously may feel for the plight of the appellants, the courts of the US
appease soldiers' sexual appetites and contain soldiers' simply are not authorized to hear their case."28 The women
activities within a regulated environment.11 Comfort stations again brought their case to the US Supreme Court which
would also prevent the spread of venereal disease among denied their petition for writ of certiorari on February 21, 2006.
soldiers and discourage soldiers from raping inhabitants of
occupied territories.12 Efforts at the United Nations

Daily life as a comfort woman was "unmitigated misery."13 In 1992, the Korean Council for the Women Drafted for Military
The military forced victims into barracks-style stations divided Sexual Slavery by Japan (KCWS), submitted a petition to the
into tiny cubicles where they were forced to live, sleep, and UN Human Rights Commission (UNHRC), asking for
have sex with as many 30 soldiers per day.14 The 30 minutes assistance in investigating crimes committed by Japan
allotted for sexual relations with each soldier were 30-minute against Korean women and seeking reparations for former
increments of unimaginable horror for the women.15 Disease comfort women.29 The UNHRC placed the issue on its
was rampant.16 Military doctors regularly examined the agenda and appointed Radhika Coomaraswamy as the
women, but these checks were carried out to prevent the issue's special investigator. In 1996, Coomaraswamy issued
spread of venereal diseases; little notice was taken of the a Report reaffirming Japan's responsibility in forcing Korean
frequent cigarette burns, bruises, bayonet stabs and even women to act as sex slaves for the imperial army, and made
broken bones inflicted on the women by soldiers. the following recommendations:
Document1zzF48331552898
A. At the national level
Fewer than 30% of the women survived the war.17 Their
agony continued in having to suffer with the residual physical, 137. The Government of Japan should:
psychological, and emotional scars from their former lives.
Some returned home and were ostracized by their families. (a) Acknowledge that the system of comfort stations set up by
Some committed suicide. Others, out of shame, never the Japanese Imperial Army during the Second World War
returned home.18 was a violation of its obligations under international law and
accept legal responsibility for that violation;
Efforts to Secure Reparation
(b) Pay compensation to individual victims of Japanese
The most prominent attempts to compel the Japanese military sexual slavery according to principles outlined by the
government to accept legal responsibility and pay Special Rapporteur of the Sub-Commission on Prevention of
compensatory damages for the comfort women system were Discrimination and Protection of Minorities on the right to
FINALS CONSTITUTIONAL LAW I ACJUCO NOV 11, 2017 45

restitution, compensation and rehabilitation for victims of The UN, since then, has not taken any official action directing
grave violations of human rights and fundamental freedoms. Japan to provide the reparations sought.
A special administrative tribunal for this purpose should be set
up with a limited time-frame since many of the victims are of a Women's International War Crimes
very advanced age;
Tribunal
(c) Make a full disclosure of documents and materials in its
possession with regard to comfort stations and other related The Women's International War Crimes Tribunal (WIWCT)
activities of the Japanese Imperial Army during the Second was a "people's tribunal" established by a number of Asian
World War; women and human rights organizations, supported by an
international coalition of non-governmental organizations.31
(d) Make a public apology in writing to individual women who First proposed in 1998, the WIWCT convened in Tokyo in
have come forward and can be substantiated as women 2000 in order to "adjudicate Japan's military sexual violence,
victims of Japanese military sexual slavery; in particular the enslavement of comfort women, to bring those
responsible for it to justice, and to end the ongoing cycle of
(e) Raise awareness of these issues by amending educational impunity for wartime sexual violence against women."
curricula to reflect historical realities;
After examining the evidence for more than a year, the
(f) Identify and punish, as far as possible, perpetrators "tribunal" issued its verdict on December 4, 2001, finding the
involved in the recruitment and institutionalization of comfort former Emperor Hirohito and the State of Japan guilty of
stations during the Second World War. crimes against humanity for the rape and sexual slavery of
women.32 It bears stressing, however, that although the
Gay J. McDougal, the Special Rapporteur for the UN Sub- tribunal included prosecutors, witnesses, and judges, its
Commission on Prevention of Discrimination and Protection of judgment was not legally binding since the tribunal itself was
Minorities, also presented a report to the Sub-Committee on organized by private citizens.
June 22, 1998 entitled Contemporary Forms of Slavery:
Systematic Rape, Sexual Slavery and Slavery-like Practices Action by Individual Governments
During Armed Conflict. The report included an appendix
entitled An Analysis of the Legal Liability of the Government On January 31, 2007, US Representative Michael Honda of
of Japan for 'Comfort Women Stations' established during the California, along with six co-sponsor representatives,
Second World War,30 which contained the following findings: introduced House Resolution 121 which called for Japanese
action in light of the ongoing struggle for closure by former
68. The present report concludes that the Japanese comfort women. The Resolution was formally passed on July
Government remains liable for grave violations of human 30, 2007,33 and made four distinct demands:
rights and humanitarian law, violations that amount in their
totality to crimes against humanity. The Japanese [I]t is the sense of the House of Representatives that the
Government’s arguments to the contrary, including arguments Government of Japan (1) should formally acknowledge,
that seek to attack the underlying humanitarian law prohibition apologize, and accept historical responsibility in a clear and
of enslavement and rape, remain as unpersuasive today as unequivocal manner for its Imperial Armed Forces' coercion
they were when they were first raised before the Nuremberg of young women into sexual slavery, known to the world as
war crimes tribunal more than 50 years ago. In addition, the "comfort women", during its colonial and wartime occupation
Japanese Government’s argument that Japan has already of Asia and the Pacific Islands from the 1930s through the
settled all claims from the Second World War through peace duration of World War II; (2) would help to resolve recurring
treaties and reparations agreements following the war questions about the sincerity and status of prior statements if
remains equally unpersuasive. This is due, in large part, to the the Prime Minister of Japan were to make such an apology as
failure until very recently of the Japanese Government to a public statement in his official capacity; (3) should clearly
admit the extent of the Japanese military’s direct involvement and publicly refute any claims that the sexual enslavement
in the establishment and maintenance of these rape centres. and trafficking of the "comfort women" for the Japanese
The Japanese Government’s silence on this point during the Imperial Army never occurred; and (4) should educate current
period in which peace and reparations agreements between and future generations about this horrible crime while
Japan and other Asian Governments were being negotiated following the recommendations of the international community
following the end of the war must, as a matter of law and with respect to the "comfort women."34
justice, preclude Japan from relying today on these peace
treaties to extinguish liability in these cases. In December 2007, the European Parliament, the governing
body of the European Union, drafted a resolution similar to
69. The failure to settle these claims more than half a century House Resolution 121.35 Entitled, "Justice for Comfort
after the cessation of hostilities is a testament to the degree to Women," the resolution demanded: (1) a formal
which the lives of women continue to be undervalued. Sadly, acknowledgment of responsibility by the Japanese
this failure to address crimes of a sexual nature committed on government; (2) a removal of the legal obstacles preventing
a massive scale during the Second World War has added to compensation; and (3) unabridged education of the past. The
the level of impunity with which similar crimes are committed resolution also stressed the urgency with which Japan should
today. The Government of Japan has taken some steps to act on these issues, stating: "the right of individuals to claim
apologize and atone for the rape and enslavement of over reparations against the government should be expressly
200,000 women and girls who were brutalized in "comfort recognized in national law, and cases for reparations for the
stations" during the Second World War. However, anything survivors of sexual slavery, as a crime under international law,
less than full and unqualified acceptance by the Government should be prioritized, taking into account the age of the
of Japan of legal liability and the consequences that flow from survivors."
such liability is wholly inadequate. It must now fall to the
Government of Japan to take the necessary final steps to The Canadian and Dutch parliaments have each followed suit
provide adequate redress. in drafting resolutions against Japan. Canada's resolution
demands the Japanese government to issue a formal apology,
to admit that its Imperial Military coerced or forced hundreds
FINALS CONSTITUTIONAL LAW I ACJUCO NOV 11, 2017 46

of thousands of women into sexual slavery, and to restore b) Prime Minister Tomiichi Murayama’s Statement in 1994
references in Japanese textbooks to its war crimes.36 The
Dutch parliament's resolution calls for the Japanese On the issue of wartime "comfort women", which seriously
government to uphold the 1993 declaration of remorse made stained the honor and dignity of many women, I would like to
by Chief Cabinet Secretary Yohei Kono. take this opportunity once again to express my profound and
sincere remorse and apologies"
The Foreign Affairs Committee of the United Kingdom’s
Parliament also produced a report in November, 2008 entitled, c) Letters from the Prime Minister of Japan to Individual
"Global Security: Japan and Korea" which concluded that Comfort Women
Japan should acknowledge the pain caused by the issue of
comfort women in order to ensure cooperation between Japan The issue of comfort women, with the involvement of the
and Korea. Japanese military authorities at that time, was a grave affront
to the honor and dignity of a large number of women.
Statements of Remorse made by representatives of the
Japanese government As Prime Minister of Japan, I thus extend anew my most
sincere apologies and remorse to all the women who endured
Various officials of the Government of Japan have issued the immeasurable and painful experiences and suffered incurable
following public statements concerning the comfort system: physical and psychological wounds as comfort women.

a) Statement by the Chief Cabinet Secretary Yohei Kono in I believe that our country, painfully aware of its moral
1993: responsibilities, with feelings of apology and remorse, should
face up squarely to its past history and accurately convey it to
The Government of Japan has been conducting a study on future generations.
the issue of wartime "comfort women" since December 1991.
I wish to announce the findings as a result of that study. d) The Diet (Japanese Parliament) passed resolutions in 1995
and 2005
As a result of the study which indicates that comfort stations
were operated in extensive areas for long periods, it is Solemnly reflecting upon the many instances of colonial rule
apparent that there existed a great number of comfort women. and acts of aggression that occurred in modern world history,
Comfort stations were operated in response to the request of and recognizing that Japan carried out such acts in the past
the military authorities of the day. The then Japanese military and inflicted suffering on the people of other countries,
was, directly or indirectly, involved in the establishment and especially in Asia, the Members of this House hereby express
management of the comfort stations and the transfer of deep remorse. (Resolution of the House of Representatives
comfort women. The recruitment of the comfort women was adopted on June 9, 1995)
conducted mainly by private recruiters who acted in response
to the request of the military. The Government study has e) Various Public Statements by Japanese Prime Minister
revealed that in many cases they were recruited against their Shinzo Abe
own will, through coaxing coercion, etc., and that, at times,
administrative/military personnel directly took part in the I have talked about this matter in the Diet sessions last year,
recruitments. They lived in misery at comfort stations under a and recently as well, and to the press. I have been consistent.
coercive atmosphere. I will stand by the Kono Statement. This is our consistent
position. Further, we have been apologizing sincerely to those
As to the origin of those comfort women who were transferred who suffered immeasurable pain and incurable psychological
to the war areas, excluding those from Japan, those from the wounds as comfort women. Former Prime Ministers, including
Korean Peninsula accounted for a large part. The Korean Prime Ministers Koizumi and Hashimoto, have issued letters
Peninsula was under Japanese rule in those days, and their to the comfort women. I would like to be clear that I carry the
recruitment, transfer, control, etc., were conducted generally same feeling. This has not changed even slightly. (Excerpt
against their will, through coaxing, coercion, etc. from Remarks by Prime Minister Abe at an Interview by NHK,
March 11, 2007).
Undeniably, this was an act, with the involvement of the
military authorities of the day that severely injured the honor I am apologizing here and now. I am apologizing as the Prime
and dignity of many women. The Government of Japan would Minister and it is as stated in the statement by the Chief
like to take this opportunity once again to extend its sincere Cabinet Secretary Kono. (Excerpt from Remarks by Prime
apologies and remorse to all those, irrespective of place of Minister Abe at the Budget Committee, the House of
origin, who suffered immeasurable pain and incurable Councilors, the Diet of Japan, March 26, 2007).
physical and psychological wounds as comfort women.
I am deeply sympathetic to the former comfort women who
It is incumbent upon us, the Government of Japan, to continue suffered hardships, and I have expressed my apologies for the
to consider seriously, while listening to the views of learned extremely agonizing circumstances into which they were
circles, how best we can express this sentiment. placed. (Excerpt from Telephone Conference by Prime
Minister Abe to President George W. Bush, April 3, 2007).
We shall face squarely the historical facts as described above
instead of evading them, and take them to heart as lessons of I have to express sympathy from the bottom of my heart to
history. We hereby reiterated our firm determination never to those people who were taken as wartime comfort women. As
repeat the same mistake by forever engraving such issues in a human being, I would like to express my sympathies, and
our memories through the study and teaching of history. also as prime minister of Japan I need to apologize to them.
My administration has been saying all along that we continue
As actions have been brought to court in Japan and interests to stand by the Kono Statement. We feel responsible for
have been shown in this issue outside Japan, the Government having forced these women to go through that hardship and
of Japan shall continue to pay full attention to this matter, pain as comfort women under the circumstances at the time.
including private researched related thereto. (Excerpt from an interview article "A Conversation with Shinzo
Abe" by the Washington Post, April 22, 2007).
FINALS CONSTITUTIONAL LAW I ACJUCO NOV 11, 2017 47

unusual need for unquestioning adherence to a political


x x x both personally and as Prime Minister of Japan, my heart decision already made; or the potentiality of embarrassment
goes out in sympathy to all those who suffered extreme from multifarious pronouncements by various departments on
hardships as comfort women; and I expressed my apologies question.
for the fact that they were forced to endure such extreme and
harsh conditions. Human rights are violated in many parts of In Tañada v. Cuenco,40 we held that political questions refer
the world during the 20th Century; therefore we must work to "to those questions which, under the Constitution, are to be
make the 21st Century a wonderful century in which no human decided by the people in their sovereign capacity, or in regard
rights are violated. And the Government of Japan and I wish to which full discretionary authority has been delegated to the
to make significant contributions to that end. (Excerpt from legislative or executive branch of the government. It is
Prime Minister Abe's remarks at the Joint Press Availability concerned with issues dependent upon the wisdom, not
after the summit meeting at Camp David between Prime legality of a particular measure."
Minister Abe and President Bush, April 27, 2007).
Certain types of cases often have been found to present
The Asian Women's Fund political questions.41 One such category involves questions
of foreign relations. It is well-established that "[t]he conduct of
Established by the Japanese government in 1995, the AWF the foreign relations of our government is committed by the
represented the government's concrete attempt to address its Constitution to the executive and legislative--'the political'--
moral responsibility by offering monetary compensation to departments of the government, and the propriety of what may
victims of the comfort women system.37 The purpose of the be done in the exercise of this political power is not subject to
AWF was to show atonement of the Japanese people through judicial inquiry or decision."42 The US Supreme Court has
expressions of apology and remorse to the former wartime further cautioned that decisions relating to foreign policy are
comfort women, to restore their honor, and to demonstrate delicate, complex, and involve large elements of prophecy.
Japan’s strong respect for women.38 They are and should be undertaken only by those directly
responsible to the people whose welfare they advance or
The AWF announced three programs for former comfort imperil. They are decisions of a kind for which the Judiciary
women who applied for assistance: (1) an atonement fund has neither aptitude, facilities nor responsibility.43
paying ¥2 million (approximately $20,000) to each woman; (2)
medical and welfare support programs, paying ¥2.5-3 million To be sure, not all cases implicating foreign relations present
($25,000-$30,000) for each woman; and (3) a letter of apology political questions, and courts certainly possess the authority
from the Japanese Prime Minister to each woman. Funding to construe or invalidate treaties and executive
for the program came from the Japanese government and agreements.44 However, the question whether the Philippine
private donations from the Japanese people. As of March government should espouse claims of its nationals against a
2006, the AWF provided ¥700 million (approximately $7 foreign government is a foreign relations matter, the authority
million) for these programs in South Korea, Taiwan, and the for which is demonstrably committed by our Constitution not
Philippines; ¥380 million (approximately $3.8 million) in to the courts but to the political branches. In this case, the
Indonesia; and ¥242 million (approximately $2.4 million) in the Executive Department has already decided that it is to the best
Netherlands. interest of the country to waive all claims of its nationals for
reparations against Japan in the Treaty of Peace of 1951. The
On January 15, 1997, the AWF and the Philippine government wisdom of such decision is not for the courts to question.
signed a Memorandum of Understanding for medical and Neither could petitioners herein assail the said determination
welfare support programs for former comfort women. Over the by the Executive Department via the instant petition for
next five years, these were implemented by the Department certiorari.
of Social Welfare and Development.
In the seminal case of US v. Curtiss-Wright Export Corp.,45
Our Ruling the US Supreme Court held that "[t]he President is the sole
organ of the nation in its external relations, and its sole
Stripped down to its essentials, the issue in this case is representative with foreign relations."
whether the Executive Department committed grave
abuse of discretion in not espousing petitioners’ claims It is quite apparent that if, in the maintenance of our
for official apology and other forms of reparations against international relations, embarrassment -- perhaps serious
Japan. embarrassment -- is to be avoided and success for our aims
achieved, congressional legislation which is to be made
The petition lacks merit. effective through negotiation and inquiry within the
international field must often accord to the President a degree
From a Domestic Law Perspective, the Executive of discretion and freedom from statutory restriction which
Department has the exclusive prerogative to determine would not be admissible where domestic affairs alone
whether to espouse petitioners’ claims against Japan. involved. Moreover, he, not Congress, has the better
opportunity of knowing the conditions which prevail in foreign
Baker v. Carr39 remains the starting point for analysis under countries, and especially is this true in time of war. He has his
the political question doctrine. There the US Supreme Court confidential sources of information. He has his agents in the
explained that: form of diplomatic, consular and other officials. x x x

x x x Prominent on the surface of any case held to involve a This ruling has been incorporated in our jurisprudence through
political question is found a textually demonstrable Bayan v. Executive Secretary46 and Pimentel v. Executive
constitutional commitment of the issue to a coordinate political Secretary;47 its overreaching principle was, perhaps, best
department or a lack of judicially discoverable and articulated in (now Chief) Justice Puno’s dissent in Secretary
manageable standards for resolving it, or the impossibility of of Justice v. Lantion:48
deciding without an initial policy determination of a kind clearly
for non-judicial discretion; or the impossibility of a court's x x x The conduct of foreign relations is full of complexities and
undertaking independent resolution without expressing lack of consequences, sometimes with life and death significance to
the respect due coordinate branches of government; or an the nation especially in times of war. It can only be entrusted
FINALS CONSTITUTIONAL LAW I ACJUCO NOV 11, 2017 48

to that department of government which can act on the basis United States v. Pink, 315 U.S. 203, 225, 62 S.Ct. 552, 563,
of the best available information and can decide with 86 L.Ed. 796 (1942). To resolve these difficulties, nations have
decisiveness. x x x It is also the President who possesses the often entered into agreements settling the claims of their
most comprehensive and the most confidential information respective nationals. As one treatise writer puts it,
about foreign countries for our diplomatic and consular international agreements settling claims by nationals of one
officials regularly brief him on meaningful events all over the state against the government of another "are established
world. He has also unlimited access to ultra-sensitive military international practice reflecting traditional international
intelligence data. In fine, the presidential role in foreign affairs theory." L. Henkin, Foreign Affairs and the Constitution 262
is dominant and the President is traditionally accorded a wider (1972). Consistent with that principle, the United States has
degree of discretion in the conduct of foreign affairs. The repeatedly exercised its sovereign authority to settle the
regularity, nay, validity of his actions are adjudged under less claims of its nationals against foreign countries. x x x Under
stringent standards, lest their judicial repudiation lead to such agreements, the President has agreed to renounce or
breach of an international obligation, rupture of state relations, extinguish claims of United States nationals against foreign
forfeiture of confidence, national embarrassment and a governments in return for lump-sum payments or the
plethora of other problems with equally undesirable establishment of arbitration procedures. To be sure, many of
consequences. these settlements were encouraged by the United States
claimants themselves, since a claimant's only hope of
The Executive Department has determined that taking up obtaining any payment at all might lie in having his
petitioners’ cause would be inimical to our country’s Government negotiate a diplomatic settlement on his behalf.
foreign policy interests, and could disrupt our relations But it is also undisputed that the "United States has
with Japan, thereby creating serious implications for sometimes disposed of the claims of its citizens without their
stability in this region. For us to overturn the Executive consent, or even without consultation with them, usually
Department’s determination would mean an assessment of without exclusive regard for their interests, as distinguished
the foreign policy judgments by a coordinate political branch from those of the nation as a whole." Henkin, supra, at 262-
to which authority to make that judgment has been 263. Accord, Restatement (Second) of Foreign Relations Law
constitutionally committed. of the United States § 213 (1965) (President "may waive or
settle a claim against a foreign state x x x [even] without the
In any event, it cannot reasonably be maintained that the consent of the [injured] national"). It is clear that the practice
Philippine government was without authority to negotiate the of settling claims continues today.
Treaty of Peace with Japan. And it is equally true that, since
time immemorial, when negotiating peace accords and Respondents explain that the Allied Powers concluded the
settling international claims: Peace Treaty with Japan not necessarily for the complete
atonement of the suffering caused by Japanese aggression
x x x [g]overnments have dealt with x x x private claims as during the war, not for the payment of adequate reparations,
their own, treating them as national assets, and as counters, but for security purposes. The treaty sought to prevent the
`chips', in international bargaining. Settlement agreements spread of communism in Japan, which occupied a strategic
have lumped, or linked, claims deriving from private debts with position in the Far East. Thus, the Peace Treaty compromised
others that were intergovernmental in origin, and concessions individual claims in the collective interest of the free world.
in regard to one category of claims might be set off against
concessions in the other, or against larger political This was also the finding in a similar case involving American
considerations unrelated to debts.49 victims of Japanese slave labor during the war.52 In a
consolidated case in the Northern District of California,53 the
Indeed, except as an agreement might otherwise provide, court dismissed the lawsuits filed, relying on the 1951 peace
international settlements generally wipe out the underlying treaty with Japan,54 because of the following policy
private claims, thereby terminating any recourse under considerations:
domestic law. In Ware v. Hylton,50 a case brought by a British
subject to recover a debt confiscated by the Commonwealth The official record of treaty negotiations establishes that a
of Virginia during the war, Justice Chase wrote: fundamental goal of the agreement was to settle the
reparations issue once and for all. As the statement of the
I apprehend that the treaty of peace abolishes the subject of chief United States negotiator, John Foster Dulles, makes
the war, and that after peace is concluded, neither the matter clear, it was well understood that leaving open the possibility
in dispute, nor the conduct of either party, during the war, can of future claims would be an unacceptable impediment to a
ever be revived, or brought into contest again. All violences, lasting peace:
injuries, or damages sustained by the government, or people
of either, during the war, are buried in oblivion; and all those Reparation is usually the most controversial aspect of
things are implied by the very treaty of peace; and therefore peacemaking. The present peace is no exception.
not necessary to be expressed. Hence it follows, that the
restitution of, or compensation for, British property On the one hand, there are claims both vast and just. Japan's
confiscated, or extinguished, during the war, by any of the aggression caused tremendous cost, losses and suffering.
United States, could only be provided for by the treaty of
peace; and if there had been no provision, respecting these On the other hand, to meet these claims, there stands a Japan
subjects, in the treaty, they could not be agitated after the presently reduced to four home islands which are unable to
treaty, by the British government, much less by her subjects produce the food its people need to live, or the raw materials
in courts of justice. (Emphasis supplied). they need to work. x x x

This practice of settling claims by means of a peace treaty is The policy of the United States that Japanese liability for
certainly nothing new. For instance, in Dames & Moore v. reparations should be sharply limited was informed by the
Regan,51 the US Supreme Court held: experience of six years of United States-led occupation of
Japan. During the occupation the Supreme Commander of the
Not infrequently in affairs between nations, outstanding claims Allied Powers (SCAP) for the region, General Douglas
by nationals of one country against the government of another MacArthur, confiscated Japanese assets in conjunction with
country are "sources of friction" between the two sovereigns. the task of managing the economic affairs of the vanquished
FINALS CONSTITUTIONAL LAW I ACJUCO NOV 11, 2017 49

nation and with a view to reparations payments. It soon fit, for it is its own right that the State is asserting. Should the
became clear that Japan's financial condition would render natural or legal person on whose behalf it is acting consider
any aggressive reparations plan an exercise in futility. that their rights are not adequately protected, they have no
Meanwhile, the importance of a stable, democratic Japan as remedy in international law. All they can do is resort to national
a bulwark to communism in the region increased. At the end law, if means are available, with a view to furthering their
of 1948, MacArthur expressed the view that "[t]he use of cause or obtaining redress. The municipal legislator may lay
reparations as a weapon to retard the reconstruction of a upon the State an obligation to protect its citizens abroad, and
viable economy in Japan should be combated with all possible may also confer upon the national a right to demand the
means" and "recommended that the reparations issue be performance of that obligation, and clothe the right with
settled finally and without delay." corresponding sanctions.1awwphi1 However, all these
questions remain within the province of municipal law and do
That this policy was embodied in the treaty is clear not only not affect the position internationally.58 (Emphasis supplied)
from the negotiations history but also from the Senate Foreign
Relations Committee report recommending approval of the The State, therefore, is the sole judge to decide whether its
treaty by the Senate. The committee noted, for example: protection will be granted, to what extent it is granted, and
when will it cease. It retains, in this respect, a discretionary
Obviously insistence upon the payment of reparations in any power the exercise of which may be determined by
proportion commensurate with the claims of the injured considerations of a political or other nature, unrelated to the
countries and their nationals would wreck Japan's economy, particular case.
dissipate any credit that it may possess at present, destroy the
initiative of its people, and create misery and chaos in which The International Law Commission’s (ILC’s) Draft Articles on
the seeds of discontent and communism would flourish. In Diplomatic Protection fully support this traditional view. They
short, [it] would be contrary to the basic purposes and policy (i) state that "the right of diplomatic protection belongs to or
of x x x the United States x x x. vests in the State,"59 (ii) affirm its discretionary nature by
clarifying that diplomatic protection is a "sovereign
We thus hold that, from a municipal law perspective, that prerogative" of the State;60 and (iii) stress that the state "has
certiorari will not lie. As a general principle – and particularly the right to exercise diplomatic protection on behalf of a
here, where such an extraordinary length of time has lapsed national. It is under no duty or obligation to do so."61
between the treaty’s conclusion and our consideration – the
Executive must be given ample discretion to assess the It has been argued, as petitioners argue now, that the State
foreign policy considerations of espousing a claim against has a duty to protect its nationals and act on his/her behalf
Japan, from the standpoint of both the interests of the when rights are injured.62 However, at present, there is no
petitioners and those of the Republic, and decide on that basis sufficient evidence to establish a general international
if apologies are sufficient, and whether further steps are obligation for States to exercise diplomatic protection of their
appropriate or necessary. own nationals abroad.63 Though, perhaps desirable, neither
state practice nor opinio juris has evolved in such a direction.
The Philippines is not under any international obligation If it is a duty internationally, it is only a moral and not a legal
to espouse petitioners’ claims. duty, and there is no means of enforcing its
fulfillment.641avvphi1
In the international sphere, traditionally, the only means
available for individuals to bring a claim within the international We fully agree that rape, sexual slavery, torture, and sexual
legal system has been when the individual is able to persuade violence are morally reprehensible as well as legally
a government to bring a claim on the individual’s behalf.55 prohibited under contemporary international law.65 However,
Even then, it is not the individual’s rights that are being petitioners take quite a theoretical leap in claiming that these
asserted, but rather, the state’s own rights. Nowhere is this proscriptions automatically imply that that the Philippines is
position more clearly reflected than in the dictum of the under a non-derogable obligation to prosecute international
Permanent Court of International Justice (PCIJ) in the 1924 crimes, particularly since petitioners do not demand the
Mavrommatis Palestine Concessions Case: imputation of individual criminal liability, but seek to recover
monetary reparations from the state of Japan. Absent the
By taking up the case of one of its subjects and by resorting consent of states, an applicable treaty regime, or a directive
to diplomatic action or international judicial proceedings on his by the Security Council, there is no non-derogable duty to
behalf, a State is in reality asserting its own right to ensure, in institute proceedings against Japan. Indeed, precisely
the person of its subjects, respect for the rules of international because of states’ reluctance to directly prosecute claims
law. The question, therefore, whether the present dispute against another state, recent developments support the
originates in an injury to a private interest, which in point of modern trend to empower individuals to directly participate in
fact is the case in many international disputes, is irrelevant suits against perpetrators of international crimes.66
from this standpoint. Once a State has taken up a case on Nonetheless, notwithstanding an array of General Assembly
behalf of one of its subjects before an international tribunal, in resolutions calling for the prosecution of crimes against
the eyes of the latter the State is sole claimant.56 humanity and the strong policy arguments warranting such a
rule, the practice of states does not yet support the present
Since the exercise of diplomatic protection is the right of the existence of an obligation to prosecute international crimes.67
State, reliance on the right is within the absolute discretion of Of course a customary duty of prosecution is ideal, but we
states, and the decision whether to exercise the discretion cannot find enough evidence to reasonably assert its
may invariably be influenced by political considerations other existence. To the extent that any state practice in this area is
than the legal merits of the particular claim.57 As clearly widespread, it is in the practice of granting amnesties,
stated by the ICJ in immunity, selective prosecution, or de facto impunity to those
who commit crimes against humanity."68
Barcelona Traction:
Even the invocation of jus cogens norms and erga omnes
The Court would here observe that, within the limits prescribed obligations will not alter this analysis. Even if we sidestep the
by international law, a State may exercise diplomatic question of whether jus cogens norms existed in 1951,
protection by whatever means and to whatever extent it thinks petitioners have not deigned to show that the crimes
FINALS CONSTITUTIONAL LAW I ACJUCO NOV 11, 2017 50

committed by the Japanese army violated jus cogens Of course, we greatly sympathize with the cause of
prohibitions at the time the Treaty of Peace was signed, or that petitioners, and we cannot begin to comprehend the
the duty to prosecute perpetrators of international crimes is an unimaginable horror they underwent at the hands of the
erga omnes obligation or has attained the status of jus Japanese soldiers. We are also deeply concerned that, in
cogens. apparent contravention of fundamental principles of law, the
petitioners appear to be without a remedy to challenge those
The term erga omnes (Latin: in relation to everyone) in that have offended them before appropriate fora. Needless to
international law has been used as a legal term describing say, our government should take the lead in protecting its
obligations owed by States towards the community of states citizens against violation of their fundamental human rights.
as a whole. The concept was recognized by the ICJ in Regrettably, it is not within our power to order the Executive
Barcelona Traction: Department to take up the petitioners’ cause. Ours is only the
power to urge and exhort the Executive Department to take
x x x an essential distinction should be drawn between the up petitioners’ cause.
obligations of a State towards the international community as
a whole, and those arising vis-à-vis another State in the field WHEREFORE, the Petition is hereby DISMISSED.
of diplomatic protection. By their very nature, the former are
the concern of all States. In view of the importance of the rights SO ORDERED.
involved, all States can be held to have a legal interest in their
protection; they are obligations erga
omnes.http://www.search.com/reference/Erga_omnes -
_note-0#_note-0

Such obligations derive, for example, in contemporary


international law, from the outlawing of acts of aggression, and
of genocide, as also from the principles and rules concerning
the basic rights of the human person, including protection from
slavery and racial discrimination. Some of the corresponding
rights of protection have entered into the body of general
international law … others are conferred by international
instruments of a universal or quasi-universal character.

The Latin phrase, ‘erga omnes,’ has since become one of the
rallying cries of those sharing a belief in the emergence of a
value-based international public order. However, as is so often
the case, the reality is neither so clear nor so bright. Whatever
the relevance of obligations erga omnes as a legal concept,
its full potential remains to be realized in practice.69

The term is closely connected with the international law


concept of jus cogens. In international law, the term "jus
cogens" (literally, "compelling law") refers to norms that
command peremptory authority, superseding conflicting
treaties and custom. Jus cogens norms are considered
peremptory in the sense that they are mandatory, do not admit
derogation, and can be modified only by general international
norms of equivalent authority.70

Early strains of the jus cogens doctrine have existed since the
1700s,71 but peremptory norms began to attract greater
scholarly attention with the publication of Alfred von
Verdross's influential 1937 article, Forbidden Treaties in
International Law.72 The recognition of jus cogens gained
even more force in the 1950s and 1960s with the ILC’s
preparation of the Vienna Convention on the Law of Treaties
(VCLT).73 Though there was a consensus that certain
international norms had attained the status of jus cogens,74
the ILC was unable to reach a consensus on the proper
criteria for identifying peremptory norms.

After an extended debate over these and other theories of jus


cogens, the ILC concluded ruefully in 1963 that "there is not
as yet any generally accepted criterion by which to identify a
general rule of international law as having the character of jus
cogens."75 In a commentary accompanying the draft
convention, the ILC indicated that "the prudent course seems
to be to x x x leave the full content of this rule to be worked
out in State practice and in the jurisprudence of international
tribunals."76 Thus, while the existence of jus cogens in
international law is undisputed, no consensus exists on its
substance,77 beyond a tiny core of principles and rules.78
FINALS CONSTITUTIONAL LAW I ACJUCO NOV 11, 2017 51

"G.R. No. 84642 -- (PEOPLE'S MOVEMENT FOR PRESS Delegates from other countries come and tell you in
FREEDOM, ET AL. V. HON. RAUL MANGLAPUS, ET AL.) confidence of their troubles at home and of their differences
with other countries and with other delegates; they tell you of
The petitioners, invoking the constitutional provisions on what they would do under certain circumstances and would
freedom of speech and the press (Constitution, Article III, not do under other circumstances.... If these reports ... should
section 7) seek to compel the respondents who are become public.... who would ever trust American Delegations
representatives of the President of the Philippine sin the on- in another conference? (United States Department of State,
going negotiations of the RP-US Military Base Agreement Press Releases, June 7, 1930, pp.
to: 282-284.)."

(1) Open to petitioners their negotiations/sessions with the "There is frequent criticism of the secrecy in which negotiation
U.S. counterparts on the RP-US Military Agreement: with foreign powers on nearly all subjects is concerned. This,
(2) Reveal and/or give petitioners access to the items which it is claimed, is incompatible with the substance of democracy.
they (respondents) have already agreed upon with their
American counterparts relative to the review of the RP-US As expressed by one writer 'It can be said that there is no more
Military Bases Agreement, and 92) reveal and/or make rigid system of silence anywhere in the world.' (E.J. Young,
accessible to petitioners the respective positions of Looking Behind the Censorship, J. B. Lippincott Co., 1938)
respondents and their US counterparts on items they have not President Wilson in starting his efforts for the conclusion of the
agreed upon, particularly the compensation package for the World War declared that we must have 'open covenants,
continued use by the U.S. of their military bases and facilities openly arrived at.' He quickly abandoned his thought.
in the Philippines.
"No one who has studied the question believes that such a
The petitioners argue that the closed-door negotiations method of publicity is possible. In the moment that
held by the two countries negotiating panels’ amount to negotiations are started, pressure groups attempt to 'muscle
the denial of the petitioners' right to press freedom as well in'. An ill-timed speech by one of the parties or a frank
as their right to information on matter of public concern. declaration of the concessions which are exacted or offered
They specifically mention the fact that their organization on both sides would quickly lead to widespread propaganda
should be given access to information on the negotiations so to block the negotiations. After a treaty has been drafted and
that they can pursue and protect, within the democratic its terms are fully published, there is ample opportunity for
framework, their legitimate and collective interests and discussion before it is approved." (The New American
aspirations thru peaceful and lawful means and can effectively Government and Its Works, James T. Young, 4thEdition, p.
and reasonably participate at this level of political decision as 194)
mandated by the Constitution.
The negotiation of treaties calls for a class of expertise,
Under the Constitution, the conduct of foreign relations experience, and sensitivity to national interest of an
of our Government especially the sensitive matter of extremely high order. It would be a sad day indeed if in
negotiating a treaty with a foreign government is lodged the negotiation leading to a treaty, the Philippine panel
with the political Departments of the Government. It has and the President would be hampered or embarrassed by
been ruled that the "propriety of what may be done in the criticisms or comments from persons with inadequate
exercise of this political power is not subject to judicial knowledge of the nuances of treaty negotiations or worse
inquiry or decision." (Otejen v. Central Leather Co., 246 US by publicity seekers
297, 306, 62, 2ed. 726, 732, 38 S Ct. 309, See also Baker v. or idle kibitzers.
Carr, 369 US 186, 7L ed 2nd 663, 82 S Ct. 691). The Court
finds no grave abuse of discretion amounting to lack or excess In United States v. Curtiss-Wright Export Corp. (299 U.S. 304,
of jurisdiction which would warrant the exercise of judicial 1936) and notwithstanding the power of the American Senate
power. to give "advice and consent" to the President in the making of
a treaty, it was emphasized that the President alone
As regards the President's power to conclude treaties, the negotiates. Not even the Senate or the House of
Constitution limits the role of the Supreme Court "to review, Representatives, unless they are asked, may intrude. The
revise, reverse, modify, or affirm on appeal or certiorari x x x pertinent portions of the decision read:
the constitutionality or validity of any treaty, international or
executive agreement, x x x (Article VIII, Section 5, number 2 xxx xxx xxx
paragraph (a), 1987 Constitution.
"x x x In this vast external realm, with its important,
Undoubtedly, this Court has no jurisdiction over a case complicated, delicate and manifold problems, the President
until the treaty or agreement has been concluded and its alone has the power to speak or listen as a representative of
terms questioned in an appropriate case. The Court the nation. He makes treaties with the advice and consent of
stresses that secrecy of negotiations with foreign countries is the Senate; but he alone negotiates. Into the field of
not violative of the constitutional provisions of freedom of negotiation the senate cannot intrude; and Congress itself is,
speech or of the press nor of the freedom of access to powerless to invade it. As Marshall said in his great argument
information -- of March 7, 1800, in the House of Representatives, 'The
President is the sole organ of the nation in its external
"The nature of diplomacy requires centralization of authority relations, and its sole representative with foreign
and expedition of decision which are inherent in executive nations.' Annals, 6th Cong., col. 613...
action. Another essential characteristic of diplomacy is its
confidential nature. Although much has been said about 'open' "It is important to bear in mind that we are here dealing not
and 'secret' diplomacy, with disparagement of the latter, alone with an authority vested in the President by an exertion
Secretaries of State Hughes and Stimson have clearly of legislative power, but with such an authority plus the very
analyzed and justified the practice. In the words of Mr. delicate, plenary and exclusive power of the President as the
Stimson: "A complicated negotiation... cannot be carried sole organ of the federal government in the field of
through without many, many private talks and discussions, international relations -- a power which does not require as a
man to man; many tentative suggestions and proposals. basis for its exercise an act of Congress, but which, of course,
FINALS CONSTITUTIONAL LAW I ACJUCO NOV 11, 2017 52

like every other governmental power, must be exercised in


subordination to the applicable provisions of the Constitution.
It is quite apparent that if, in the maintenance of our
international relations, embarrassment -- perhaps serious
embarrassment is to be avoided and success for our aims
achieved, congressional legislation which is to be made
effective through negotiation and inquiry within the
international field must often accord to the President a degree
of discretion and freedom admissible were domestic affairs
alone involved. Moreover, he, not Congress, has the better
opportunity of knowing the conditions which prevail in foreign
countries, and especially is this true in time of war. He has his
confidential sources of information. He has his agents in the
form of diplomatic, consular and other officials.

Secrecy in respect of information gathered by them may be


highly necessary, and the premature disclosure of it
productive of harmful results. Indeed, so clearly is this true that
the first President refused to accede to a request to lay before
the House of Representatives the instructions,
correspondence and documents relating to the negotiation of
the Jay Treaty- a refusal the wisdom of which was recognized
by the House itself and has never since been doubted. In his
reply to the request, President Washington said: " 'The nature
of foreign negotiations requires caution, and their success
must often depend on secrecy: and even when brought to a
conclusion a full disclosure of all the measures, demands, or
eventual concessions which may have been proposed or
contemplated would be extremely impolitic; fo this might have
a pernicious influence on future negotiations, or produce
immediate inconveniences, perhaps danger and mischief, in
relation to other powers. The necessity of such caution and
secrecy was one cogent reason for vesting the power of
making treaties in the President, with the advice and consent
ot the Senate, the principle on which that body was formed
confining it to a small number of members. To admit, then, a
right in the House of Representatives to demand and to have
as a matter of course all the papers respecting a negotiation
with a foreign power would be to establish a dangerous
precedent.' 1 Messages and Pares of the Presidents, p. 194."
(at pp. 297-298)

We have the same doctrine of separation of powers in the


Constitution and the same grant of authority over foreign
affairs to the President as in the American System. The same
reasoning applies to treaty negotiations by our Government.
WHEREFORE, the instant petition is DENIED for manifest
lack of merit."
FINALS CONSTITUTIONAL LAW I ACJUCO NOV 11, 2017 53

G.R. No. L-14279 October 31, 1961 Commissioner of Customs vs. Serree Investment Co., L-
12007 [May 16, 1960]; Commissioner of Customs vs. Serree
THE COMMISSIONER OF CUSTOMS and THE Investment Co., L-14274 [November 29, 1960]), for the
COLLECTOR OF CUSTOMS, petitioners, reason that the broad powers of the Central Bank, under
vs. its charter, to maintain our monetary stability and to
EASTERN SEA TRADING, respondent. preserve the international value of our currency, under
section 2 of Republic Act No. 265, in relation to section 14
Office of the Solicitor General for petitioners. of said Act — authorizing the bank to issue such rules
Valentin Gutierrez for respondent. and regulations as it may consider necessary for the
effective discharge of the responsibilities and the
CONCEPCION, J.: exercise of the powers assigned to the Monetary Board
and to the Central Bank — connote the authority to regulate
Petition for review of a judgment of the Court of Tax Appeals no-dollar imports, owing to the influence and effect that the
reversing a decision of the Commissioner of Customs. same may and do have upon the stability of our peso and its
international value.
Respondent Eastern Sea Trading was the consignee of
several shipments of onion and garlic which arrived at the Port The Court of Tax Appeals entertained doubts on the legality
of Manila from August 25 to September 7, 1954. Some of the executive agreement sought to be implemented by
shipments came from Japan and others from Hong Kong. In Executive Order No. 328, owing to the fact that our Senate
as much as none of the shipments had the certificate had not concurred in the making of said executive agreement.
required by Central Bank Circulars Nos. 44 and 45 for the The concurrence of said House of Congress is required by our
release thereof, the goods thus imported were seized and fundamental law in the making of "treaties" (Constitution of the
subjected to forfeiture proceedings for alleged violations Philippines, Article VII, Section 10[7]), which are, however,
of section 1363(f) of the Revised Administrative Code, in distinct and different from "executive agreements," which may
relation to the aforementioned circulars of the Central be validly entered into without such concurrence.
Bank. In due course, the Collector of Customs of Manila
rendered a decision on September 4, 1956, declaring said Treaties are formal documents which require ratification with
goods forfeited to the Government and — the goods having the approval of two thirds of the Senate. Executive
been, in the meantime, released to the consignees on surety agreements become binding through executive action
bonds, filed by the same, as principal, and the Alto Surety & without the need of a vote by the Senate or by Congress.
Insurance Co., Inc., as surety, in compliance with orders of the
Court of First Instance of Manila, in Civil Cases Nos. 23942 xxx xxx xxx
and 23852 thereof — directing that the amounts of said bonds
be paid, by said principal and surety, jointly and severally, to . . . the right of the Executive to enter into binding agreements
the Bureau of Customs, within thirty (30) days from notice. without the necessity of subsequent Congressional approval
has been confirmed by long usage. From the earliest days of
On appeal taken by the consignee, said decision was our history we have entered into executive agreements
affirmed by the Commissioner of Customs on December covering such subjects as commercial and consular relations,
27, 1956. Subsequently, the consignee sought a review of the most-favored-nation rights, patent rights, trademark and
decision of said two (2) officers by the Court of Tax Appeals, copyright protection, postal and navigation arrangements and
which reversed the decision of the Commissioner of Customs the settlement of claims. The validity of these has never been
and ordered that the aforementioned bonds be cancelled and seriously questioned by our courts.
withdrawn. Hence, the present petition of the Commissioner
of Customs for review of the decision of the Court of Tax xxx xxx xxx
Appeals.
Agreements with respect to the registration of trade-marks
The latter is based upon the following premises, namely: that have been concluded by the Executive with various countries
the Central Bank has no authority to regulate transactions under the Act of Congress of March 3, 1881 (21 Stat. 502).
not involving foreign exchange; that the shipments in Postal conventions regulating the reciprocal treatment of mail
question are in the nature of "no-dollar" imports; that, as matters, money orders, parcel post, etc., have been
such, the aforementioned shipments do not involve foreign concluded by the Postmaster General with various countries
exchange; that, insofar as a Central Bank license and a under authorization by Congress beginning with the Act of
certificate authorizing the importation or release of the goods February 20, 1792 (1 Stat. 232, 239). Ten executive
under consideration are required by Central Bank Circulars agreements were concluded by the President pursuant to
Nos. 44 and 45, the latter are null and void; and that the the McKinley Tariff Act of 1890 (26 Stat. 567, 612), and
seizure and forfeiture of the goods imported from Japan nine such agreements were entered into under the
cannot be justified under Executive Order No. 328,1 not only Dingley Tariff Act 1897 (30 Stat. 151, 203, 214). A very
because the same seeks to implement an executive much larger number of agreements, along the lines of the
agreement2 — extending the effectivity of our3 Trades and one with Rumania previously referred to, providing for
Financial Agreements4 with Japan — which (executive most-favored-nation treatment in customs and related
agreement), it believed, is of dubious validity, but, also, matters have been entered into since the passage of the
because there is no governmental agency authorized to issue Tariff Act of 1922, not by direction of the Act but in
the import license required by the aforementioned executive harmony with it.
order.
xxx xxx xxx
The authority of the Central Bank to regulate no-dollar imports
and the validity of the aforementioned Circulars Nos. 44, and International agreements involving political issues or changes
45 have already been passed upon and repeatedly upheld by of national policy and those involving international
this Court (Pascual vs. Commissioner of Customs, L-10979 arrangements of a permanent character usually take the form
[June 30, 1959]; Acting Commissioner of Customs vs. of treaties. But international agreements embodying
Leuterio, L-9142 [October 17, 1959] Commissioner of adjustments of detail carrying out well-established national
Customs vs. Pascual, L-9836 [November 18, 1959]; policies and traditions and those involving arrangements of a
FINALS CONSTITUTIONAL LAW I ACJUCO NOV 11, 2017 54

more or less temporary nature usually take the form of licenses was not vested exclusively upon the Import Control
executive agreements. Commission or Administration. Executive Order No. 328
provided for export or import licenses "from the Central
xxx xxx xxx Bank of the Philippines or the Import Control
Administration" or Commission. Indeed, the latter was
Furthermore, the United States Supreme Court has expressly created only to perform the task of implementing certain
recognized the validity and constitutionality of executive objectives of the Monetary Board and the Central Bank,
agreements entered into without Senate approval. (39 which otherwise had to be undertaken by these two (2)
Columbia Law Review, pp. 753-754) (See, also, U.S. vs. agencies. Upon the abolition of said Commission, the duty to
Curtis-Wright Export Corporation, 299 U.S. 304, 81 L. ed. 255; provide means and ways for the accomplishment of said
U.S. vs. Belmont, 301 U.S. 324, 81 L. ed. 1134; U.S. vs. Pink, objectives had merely to be discharged directly by the
315 U.S. 203, 86 L. ed. 796; Ozanic vs. U.S., 188 F. 2d. 288; Monetary Board and the Central Bank, even if the
Yale Law Journal, Vol. 15, pp. 1905-1906; California Law aforementioned Executive Order had been silent thereon.
Review, Vol. 25, pp. 670-675; Hyde on International Law
[Revised Edition], Vol. 2, pp. 1405, 1416-1418; Willoughby on WHEREFORE, the decision appealed from is hereby
the U.S. Constitutional Law, Vol. I [2d ed.], pp. 537-540; reversed and another one shall be entered affirming that of
Moore, International Law Digest, Vol. V, pp. 210-218; the Commissioner of Customs, with cost against respondent’s
Hackworth, International Law Digest, Vol. V, pp. 390-407). defendant-appellee, Eastern Sea Trading. It is so ordered.
(Emphasis supplied.)

In this connection, Francis B. Sayre, former U.S. High


Commissioner to the Philippines, said in his work on "The
Constitutionality of Trade Agreement Acts":

Agreements concluded by the President which fall short


of treaties are commonly referred to as executive
agreements and are no less common in our scheme of
government than are the more formal instruments —
treaties and conventions. They sometimes take the form of
exchanges of notes and at other times that of more formal
documents denominated "agreements" time or "protocols".

The point where ordinary correspondence between this and


other governments ends and agreements — whether
denominated executive agreements or exchanges of notes or
otherwise — begin, may sometimes be difficult of ready
ascertainment. It would be useless to undertake to discuss
here the large variety of executive agreements as such,
concluded from time to time. Hundreds of executive
agreements, other than those entered into under the trade-
agreements act, have been negotiated with foreign
governments. . . . It would seem to be sufficient, in order to
show that the trade agreements under the act of 1934 are not
anomalous in character, that they are not treaties, and that
they have abundant precedent in our history, to refer to certain
classes of agreements heretofore entered into by the
Executive without the approval of the Senate. They cover such
subjects as the inspection of vessels, navigation dues, income
tax on shipping profits, the admission of civil aircraft, customs
matters, and commercial relations generally, international
claims, postal matters, the registration of trademarks and
copyrights, etcetera. Some of them were concluded not by
specific congressional authorization but in conformity with
policies declared in acts of Congress with respect to the
general subject matter, such as tariff acts; while still others,
particularly those with respect of the settlement of claims
against foreign governments, were concluded independently
of any legislation." (39 Columbia Law Review, pp. 651, 755.)

The validity of the executive agreement in question is


thus patent. In fact, the so-called Parity Rights provided for
in the Ordinance Appended to our Constitution were, prior
thereto, the subject of an executive agreement, made without
the concurrence of two-thirds (2/3) of the Senate of the United
States.

Lastly, the lower court held that it would be unreasonable to


require from respondent-appellee an import license when the
Import Control Commission was no longer in existence and,
hence, there was, said court believed, no agency authorized
to issue the aforementioned license. This conclusion is
untenable, for the authority to issue the aforementioned
FINALS CONSTITUTIONAL LAW I ACJUCO NOV 11, 2017 55

G.R. No. L-23846 September 9, 1977 for deportation under the Immigration Law; that under section
37(3) of the law before an alien may be deported for having
GO TEK petitioner-appelle, been convicted and sentenced to imprisonment for a term of
vs. one year or more for a crime involving moral turpitude a
DEPORTATION BOARD, respondent-appellant. conviction is and that since Go Tek had not been convicted of
the offense punished in article 168, the deportation was
Teodoro C. Ronquillo for appelle. premature.

Solicitor-General Arturo A. Alafriz, Assistant Solicitor general The Board appealed to this Court on the ground that the
Felicisimo R. Rosete and Solicitor Octavio R. Ramirez for decision is contrary to law. The Solicitor General contends that
appellant. the trial court erred in assuming that the President may deport
undesirable aliens only to grounds enumerated by law; in
holding that mere possession of forged dollar checks is not a
AQUINO, J.: ground for deportation and that a criminal conviction is
necessary, and in not finding that the Board has jurisdiction
This is a deportation case. On March 3, 1964 the chief over Go Tek's case.
prosecutor of the Deportation Board filed a complaint against
Go Tek a Chinaman residing at Ilagan, Isabela and 1208-B, The parties stipulated that the Deportation Board is an of the
Misericordia Street, Sta. Cruz Manila. President of the Philippines charged with the investigation of
undesirable aliens and to report and recommend proper action
It was alleged in the complaint that in December, 1963 certain on the basis of its findings therein."
agents of the National Bureau of Investigation (NBI)
searched an office located at 1439 O'Donnel Street, Sta. The issue is whether the Deportation Board can entertain
Cruz, Manila believed to be the headquarters of a guerilla a deportation proceeding based on a ground which is not
unit of the "Emergency Intelligence Section, Army of the specified in section 37 of the Immigration Law and
United States" and that among those arrested thereat was although the aliens has not yet been convicted of the
Go Tek an alleged sector commander and intelligence offense imputed to him.
and record officer of that guerilla unit.
We hold that the Board has jurisdiction to investigate Go Tek
It was further alleged that fake dollar checks were found in for illegal possession of fake dollar checks (as well as his
Go Tek's possession and that, therefore, he had violated alleged "guerilla" activities) in spite of the fact that he has not
article 168 of the Revised Penal Code and rendered yet been convicted of illegal possession thereof under article
himself an undesirable alien. 168 of the Revised Penal Code and notwithstanding that act
is not the grounds for the deportation of undesirable aliens as
The prosecutor prayed that after trial the Board should enumerated in section 37 of the Immigration Law. The charge
recommend to the President of the Philippines the against Go Tek before- the Board was not premature.
immediate deportation of Go Tek as an undesirable alien,
"his presence in this country having been, and will always be The aforementioned obiter dictum the Qua Chee Gan case
and a menace to the peace , welfare, and security of the invoked by Go Tek and relied upon by the trial court, is not of
community". Case No. R1116). this case. In the Qua Chee Gan case the aliens were with
economic sabotage which is a ground for deportation under
Go Tek filed a motion to dismiss on the ground that the Republic Act No. 503.
complaint was premature bemuse them was a pending case
against him in the city fiscal's office of Manila for violation of The ratio decidendi of the Qua Chee Gan case is that the
article 168 (IS 64-7267). He contended that the board had no provision of Executive Order No. 398, of 1951, the Deportation
jurisdiction to try the case in view of the obiter dictum in Qua Board to issue a warrant of arrest upon the filing of formal
Chee Gan vs. Deportation Board, 118 Phil. 868, 875, that the charges against an alien, is "illegal" or unconstitutional
t may deport aliens only on the grounds specified in the law. because it is contrary to the provinsion in section 1(3), Article
III of the 1935 Constitution that warrants shall issue upon to
The Board, composed of Manuel A. Concordia, Arturo A. be de by the judge after e under oath the t and the witness he
Alafriz and Manuel V. Reyes, in its resolution of April 21, 1964 may produce. (Note that under section 3, Article IV of the 1973
denied Go Teks motion. The Board reasoned out that a Constitution cause may be determined "by the judge, or such
conviction is not a prerequisite before the State my exercise other responsible officer as may be authorized by law". See
its rights to deport an undesirable alien and that the Board is Santos vs. Commissioner of Immigration L-25694, November
only a fact finding body whose function is to make a report and 29, 1976, 74 SCRA 96, per Fernando, J.)
recommendation to the President in whom is lodged the
exclusive power to deport an alien or a deportation A thorough comprehension of the President's power to deport
proceeding. aliens may show the baselessness of the instant prohibition
action of Go Tek The President's power to deport aliens and
In view of the denial of his motion to quash, Go Tek on June the investigation of aliens subject to deportation are provided
10, 1964 filed in the Court of First Instance of Manila a for in the following provisions of the Revised Administrative
prohibition action against the Board. On July 8, 1964 the court Code:
issued a writ of preliminary injunction restraining the board
from hearing Go Tek's case. SEC. 69. Deportation of subject of foreign power. — A subject
of a foreign power residing in the Philippine Islands shall not
After hearing, the trial court (Judge Federico C. Alikpala be deported expelled, or excluded from said Islands or
presiding) in its decision of 31, 1964 granted the writ repatriated to his own country by the Governor-General
prohibition and the Board to desist from taking cognizance of except upon prior investigator, conducted by said Executive
the Go Tek. or his authorized agent, of the ground upon which such action
is contemplated. In such case the person concerned shall he
The court, citing the said obiter dictum in the Qua Chee Gan informed of the charge or charges against him and he shall be
case, held that mere ion of forged dollar checks is not a ground allowed not less than three days for the preparation of his
FINALS CONSTITUTIONAL LAW I ACJUCO NOV 11, 2017 56

defense. He shall also have the right to be heard by himself or evidence on which he acted. (Martin vs. Mott 12 Wheat., 19,
counsel, to produce witnesses in his own behalf, and to cross- 31, cited in In re McCulloch Dick, 38 Phil. 41, 62).
examine the opposing witnesses.
In the Dick case it was noted "that every alien forfeits his right
On the other hand, section 37 of the Immigration Law Provides to asylum in the country in which he resides, in the absence
that certain aliens may be arrested upon the warrant of the of treaty provisions to the contrary when his conduct or his
Commissioner of Immigration or of any other officer mode of life renders his prance there inimical to the public
designated by him for the purpose and deported upon the interests". "The reasons may be summed up and co in a single
Commissioner's warrant - "after a determination by the Board word: the public interest of the State." (38 Phil. 41, 47, 100).
of Commissioners of the existence of the ground for
deportation as charged against the alien." Thirteen classes of "It is fundamental that an executive order for deportation is not
aliens who may be deported by the Commissioner are dependent on a prior judicial conviction in a case" (Ang Bong
specified in section 37 (See Po Siok Pin vs. Vivo, L-24792, vs. Commissioner of Immigration, 100 Phil. 801, 803). Thus, it
February 14, 1975, 62 SCRA 363, 368). was held that the fact that an alien has been acquitted in a of
the charge does not prevent the deportation of such alien
So, under existing law; the deportation of an undesirable alien based on the same charge. Such acquittal does not constitute
may be effected (1) by order of the President, after due res judicata in the deportation proceedings. Conviction of a
investigation, pursuant to section 69 of the Revised crime is not n to warrant deportation. (3 C.J.S. 743, note 40,
Administrative Code and (2) by the Commissioner of citing Lewis vs. Frick, 233 U.S. 291, 58 L. Ed. 967 and U.S.
Immigration upon recommendation of the Board of ex.. Mastoras vs. McCandless 61 F. 2nd 366; Tama Miyake
Commissioners under section 37 of the immigration Law (Qua vs. U.S. 257 F. 732).
Chee Gan vs- Deportation Board, supra).
And in the Tan Tong case, supra, it was ruled that the
The State has the inherent power to deport undesirable aliens Deportation Board could take cognizance of the charge of
(Chuoco Tiaco vs. Forbes, 228 U.S. 549, 57 L. Ed. 960, 40 illegal importation against an alien as a ground for deportation,
Phil. 1122, 1125). That power may be exercise by the Chief even if he of the Deportation Board is merely
Executive "when he deems such action necessary for the recommendatory. The Chief Executive has to approve the
peace and domestic tranquility of the nation Justice Johnson's board's recommendatory Abuses or rents committed by the
is that there the Chief Executive rinds that there are aliens prosecutor or by the Board should first be brought to his
whose continued in the country is injurious to the public attention.
interest he may, even in the absence of express law,
deport them (Forbes vs. Chuoco Tiaco and Crossfield 16 WHEREFORE, the lower court's decision is reversed and set
Phil. 534, 568, 569; In re McCulloch Dick, 38 Phil. 41). aside. The writ of preliminary injunction is dissolved. The case
is to the Deportation Board for further proceedings. Costs
The right of a country to expel or deport aliens because against the petitioner-appellee.
their continued presence is rental to public welfare is
absolute and unqualified (Tiu Chun Hai and Go Tam vs. SO ORDERED.
Commissioner of Immigration and the Director of NBI, 104
Phil. 949, 956).

The Deportation Board is composed of the Undersecretary of


Justice as chairman, the solicitor General, and a
representative of the Secretary of National Defense
(Executive Order No. 455 dated June 25, 1951, 47 O.G. 28M).

SECTION 69 and Executive Order No. 398, the Deportation


Board, do not specify the grounds for deportation Paragraph
l(a) of Executive Order No. 398 merely provides that "the
Deportation Board, motu proprio or upon complaint of any
person is authorized to conduct investigations in the manner
prescribed in section 69 of the Revised Administrative Code
to determine whether a subject of a foreign power in the
Philippines is an undesirable alien or not, and thereafter to
recommend to the President of the Philippines the deportation
of such alien."

As observed by Justice Labrador, there is no legal nor


constitutes provision defining the power to deport aliens
because the intention of the law is to grant the Chief Executive
"full discretion to determine whether an alien's residence in the
country is so undesirable as to affect or injure the security
welfare or interest of the state. The adjudication of facts upon
which deportation is predicated also devolves on the Chief
Executive whose decision is final and executory." (Tan Tong
vs. Deportation Board 96 Phil. 934, 936; Tan Sin vs.
Deportation Board, 104 PhiL 868, 872).

It has been held that the Chief Executive is the sole and
exclusive judge of the existence of facts which warrant
the deportation of aliens as disclosed in an investigation
conducted in accordance with 69. No other tribunal is at
liberty to reexamine or to controvert the sufficiency of the
FINALS CONSTITUTIONAL LAW I ACJUCO NOV 11, 2017 57

G.R. No. 204605 Department of Foreign Affairs (DFA) that the Philippines
should accede to the Madrid Protocol.6
INTELLECTUAL PROPERTY ASSOCIATION OF THE
PHILIPPINES, Petitioner, After its own review, the DFA endorsed to the President the
vs. country's accession to the Madrid Protocol. Conformably with
HON. PAQUITO OCHOA, IN HIS CAPACITY AS its express authority under Section 9 of Executive Order No.
EXECUTIVE SECRETARY, HON. ALBERT DEL ROSARIO, 459 (Providing for the Guidelines in the Negotiation of
IN HIS CAPACITY AS SECRETARY OF THE International Agreements and its Ratification) dated
DEPARTMENT OF FOREIGN AFFAIRS, AND HON. November 25, 1997, the DFA determined that the Madrid
RICARDO BLANCAFLOR, IN HIS CAPACITY AS THE Protocol was an executive agreement.1âwphi1 The IPOPHL,
DIRECTOR GENERAL OF THE INTELLECTUAL the Department of Science and Technology, and the
PROPERTY OFFICE OF THE PHILIPPINES, Respondents. Department of Trade and Industry concurred in the
recommendation of the DFA.7
DECISION
On March 27, 2012, President Benigno C. Aquino III ratified
BERSAMIN, J.: the Madrid Protocol through an instrument of accession, The
instrument of accession was deposited with the Director
In this special civil action for certiorari and prohibition, the General of the World Intellectual Property Organization
Intellectual Property Association of the Philippines (IPAP) (WIPO) on April 25, 2012.8 The Madrid Protocol entered into
seeks to declare the accession of the Philippines to the force in the Philippines on July 25, 2012.9
Protocol Relating to the Madrid Agreement Concerning the
International Registration of Marks (Madrid Protocol) Petitioner IP AP, an association of more than 100 law firms
unconstitutional on the ground of the lack of concurrence by and individual practitioners in Intellectual Property Law whose
the Senate, and in the alternative, to declare the main objective is to promote and protect intellectual property
implementation thereof as unconstitutional because it conflicts rights in the Philippines through constant assistance and
with Republic Act No. 8293, otherwise known as the involvement in the legislation of intellectual property law,10
Intellectual Property Code of the Philippines (IP Code).1 has commenced this special civil action for certiorari and
prohibition11 to challenge the validity of the President's
We find and declare that the President's ratification is valid accession to the Madrid Protocol without the concurrence of
and constitutional because the Madrid Protocol, being an the Senate. Citing Pimentel, Jr. v. Office of the Executive
executive agreement as determined by the Department of Secretary, the IPAP has averred:
Foreign Affairs, does not require the concurrence of the
Senate. Nonetheless, while the President has the sole authority to
negotiate and enter into treaties, the Constitution provides a
Antecedents limitation to his power by requiring the concurrence of 2/3 of
all the members of the Senate for the validity of the treaty
The Madrid System for the International Registration of entered into by him. Section 21, Article VII of the 1987
Marks (Madrid System), which is the centralized system Constitution provides that "no treaty or international
providing a one-stop solution for registering and managing agreement shall be valid and effective unless concurred in by
marks worldwide, allows the trademark owner to file one at least two-thirds of all the Members of the Senate." The 1935
application in one language, and to pay one set of fees to and the 1973 Constitution also required the concurrence by
protect his mark in the territories of up to 97 member-states.2 the legislature to the treaties entered into by the executive.12
The Madrid System is governed by the Madrid Agreement,
concluded in 1891, and the Madrid Protocol, concluded in According to the IPAP, the Madrid Protocol is a treaty, not
1989.3 an executive agreement; hence, respondent DFA
Secretary Albert Del Rosario acted with grave abuse of
The Madrid Protocol, which was adopted in order to remove discretion in determining the Madrid Protocol as an
the challenges deterring some countries from acceding to the executive agreement.13
Madrid Agreement, has two objectives, namely: (1) to
facilitate securing protection for marks; and (2) to make The IPAP has argued that the implementation of the Madrid
the management of the registered marks easier in Protocol in the Philippines; specifically the processing of
different countries.4 foreign trademark applications, conflicts with the IP Code,14
whose Section 125 states:
In 2004; the Intellectual Property Office of the Philippines
(IPOPHL), the government agency mandated to administer Sec. 125. Representation; Address for Service. - If the
the intellectual property system of the country and to applicant is not domiciled or has no real and effective
implement the state policies on intellectual property; began commercial establishment in the Philippines; he shall
considering the country's accession to the Madrid Protocol. designate by a written document filed in the office, the name
However, based on its assessment in 2005, the IPOPHL and address of a Philippine resident who may be served
needed to first improve its own operations before making the notices or process in proceedings affecting the mark. Such
recommendation in favor of accession. The IPOPHL thus notices or services may be served upon the person so
implemented reforms to eliminate trademark backlogs and to designated by leaving a copy thereof at the address specified
reduce the turnaround time for the registration of marks.5 in the last designation filed. If the person so designated cannot
be found at the address given in the last designation, such
In the meanwhile, the IPOPHL mounted a campaign for notice or process may be served upon the Director. (Sec. 3;
information dissemination to raise awareness of the Madrid R.A. No. 166 a)
Protocol. It launched a series of consultations with
stakeholders and various business groups regarding the It has posited that Article 2 of the Madrid Protocol provides in
Philippines' accession to the Madrid Protocol. It ultimately contrast:
arrived at the conclusion that accession would benefit the
country and help raise the level of competitiveness for Filipino Article 2
brands. Hence, it recommended in September 2011 to the
FINALS CONSTITUTIONAL LAW I ACJUCO NOV 11, 2017 58

Securing Protection through International Registration (for example, in the event of a refusal of protection issued by
such an Office), who may act as a representative in such
(1) Where an application for the registration of a mark has cases and the method of appointment, are outside the scope
been filed with the Office of a Contracting Party, or where a of the Agreement, Protocol and Regulations and are governed
mark has been registered in the register of the Office of a by the law and practice of the Contracting Party concerned.
Contracting Party, the person in whose name that application
(hereinafter referred to as "the basic application;') or that which procedure is in conflict with that under Section 125 of
registration (hereinafter referred to as "the basic registration") the IP Code, and constitutes in effect an amendment of the
stands may, subject to the provisions of this Protocol secure local law by the Executive Department.16
protection for his mark in the territory of the Contracting
Parties, by obtaining the registration of that mark in the The IPAP has prayed that the implementation of the
register of the International Bureau of the World Intellectual Madrid Protocol in the Philippines be restrained in order
Property Organization (hereinafter referred to as "the to prevent future wrongs considering that the IP AP and
international registration," "the International Register," "the its constituency have a clear and unmistakable right not
International Bureau" and "the Organization'', respectively), to be deprived of the rights granted them by the IP Code
provided that, and existing local laws.17

(i) where the basic application has been filed with the Office of In its comment in behalf of the respondents, the Office of the
a Contracting State or where the basic registration has been Solicitor General (OSG) has stated that the IPAP does not
made by such an Office, the person in whose name that have the locus standi to challenge the accession to the Madrid
application or registration stands is a national of that Protocol; that the IPAP cannot invoke the Court's original
Contracting State, or is domiciled, or has a real and effective jurisdiction absent a showing of any grave abuse of discretion
industrial or commercial establishment, in the said Contracting on the part of the respondents; that the President's ratification
State, of the Madrid Protocol as an executive agreement is valid
because the Madrid Protocol is only procedural, does not
(ii) where the basic application has been filed with the Office create substantive rights, and does not require the
of a Contracting Organization or where the basic registration amendment of the IP Code; that the IPAP is not entitled to the
has been made by such an Office, the person in whose name restraining order or injunction because it suffers no damage
that application or registration stands is a national of a State from the ratification by the President, and there is also no
member of that Contracting Organization, or is domiciled, or urgency for such relief; and the IPAP has no clear
has a real and effective industrial or commercial unmistakable right to the relief sought.18
establishment, in the territory of the said Contracting
Organization. Issues

(2) The application for international registration (hereinafter The following issues are to be resolved, namely:
referred to as "the international application") shall be filed with
the International Bureau through the intermediary of the Office I. Whether or not the IP AP has locus standi to challenge the
with which the basic application was filed or by which the basic President's ratification of the Madrid Protocol;
registration was made (hereinafter referred to as "the Office of
origin"), as the case may be. II. Whether or not the President's ratification of the Madrid
Protocol is valid and constitutional; and
(3) Any reference in this Protocol to an "Office" or an "Office
of a Contracting Party" shall be construed as a reference to III. Whether or not the Madrid Protocol is in conflict with the IP
the office that is in charge, on behalf of a Contracting Party, of Code.
the registration of marks, and any reference in this Protocol to
"marks" shall be construed as a reference to trademarks and Ruling of the Court
service marks.
The petition for certiorari and prohibition is without merit.
(4) For the purposes of this Protocol, "territory of a Contracting
Party" means, where the Contracting Party is a State, the A.
territory of that State and, where the Contracting Party is an
intergovernmental organization, the territory in which the The issue of legal standing to sue, or locus standi
constituting treaty of that intergovernmental organization
applied. The IPAP argues in its reply19 that it has the locus standi to
file the present case by virtue of its being an association
The IPAP has insisted that Article 2 of the Madrid Protocol whose members stand to be injured as a result of the
means that foreign trademark applicants may file their enforcement of the Madrid Protocol in the Philippines; that the
applications through the International Bureau or the WIPO, injury pertains to the acceptance and approval of applications
and their applications will be automatically granted trademark submitted through the Madrid Protocol without local
protection without the need for designating their resident representation as required by Section 125 of the IP Code;20
agents in the country.15 and that such will diminish the rights granted by the IP Code
to Intellectual Property Law practitioners like the members of
Moreover, the IPAP has submitted that the procedure outlined the IPAP.21
in the Guide to the International Registration of Marks relating
to representation before the International Bureau is the The argument of the IPAP is untenable.
following, to wit:
Legal standing refers to "a right of appearance in a court of
Rule 3(1)(a) 09.02 References in the Regulations, justice on a given question."22 According to Agan, Jr. v.
Administrative Instructions or in this Guide to representation Philippine International Air Terminals Co., Inc.,23standing is
relate only to representation before the International Bureau. "a peculiar concept in constitutional law because in some
The questions of the need for a representative before the cases, suits are not brought by parties who have been
Office of origin or the Office of a designated Contracting Party personally injured by the operation of a law or any other
FINALS CONSTITUTIONAL LAW I ACJUCO NOV 11, 2017 59

government act but by concerned citizens, taxpayers or voters as are others, it is enough that he sufficiently demonstrates in
who actually sue in the public interest." his petition that he is entitled to protection or relief from the
Court in the vindication ofa public right.25
The Court has frequently felt the need to dwell on the issue of
standing in public or constitutional litigations to sift the worthy The injury that the IPAP will allegedly suffer from the
from the unworthy public law litigants seeking redress or relief. implementation of the Madrid Protocol is imaginary, incidental
The following elucidation in De Castro v. Judicial and Bar and speculative as opposed to a direct and material injury
Council24offers the general understanding of the context of required by the foregoing tenets on locus standi. Additionally,
legal standing, or locus standi for that purpose, viz. : as the OSG points out in the comment,26 the IPAP has
misinterpreted Section 125 of the IP Code on the issue of
In public or constitutional litigations, the Court is often representation. The provision only states that a foreign
burdened with the determination of the locus standi of the trademark applicant "shall designate by a written document
petitioners due to the ever-present need to regulate the filed in the office, the name and address of a Philippine
invocation of the intervention of the Court to correct any official resident who may be served notices or process in proceedings
action or policy in order to avoid obstructing the efficient affecting the mark;" it does not grant anyone in particular the
functioning of public officials and offices involved in public right to represent the foreign trademark applicant. Hence, the
service. It is required, therefore, that the petitioner must have IPAP cannot justly claim that it will suffer irreparable injury or
a personal stake in the outcome of the controversy, for, as diminution of rights granted to it by Section 125 of the IP Code
indicated in Agan, Jr. v. Philippine International Air Terminals from the implementation of the Madrid Protocol.
Co., Inc.:
Nonetheless, the IPAP also emphasizes that the paramount
The question on legal standing is whether such parties have public interest involved has transcendental importance
"'alleged such a personal stake in the outcome of the because its petition asserts that the Executive Department
controversy as to assure that concrete adverseness which has overstepped the bounds of its authority by thereby cutting
sharpens the presentation of issues upon which the court so into another branch's functions and responsibilities.27 The
largely depends for illumination of difficult constitutional assertion of the IPAP may be valid on this score. There is little
questions," Accordingly, it has been held that the interest of a question that the issues raised herein against the
person assailing the constitutionality of a statute must be implementation of the Madrid Protocol are of transcendental
direct and personal. He must be able to show, not only that importance. Accordingly, we recognize IPAP's locus standi to
the law or any government act is invalid, but also that he bring the present challenge. Indeed, the Court has adopted a
sustained or is in imminent danger of sustaining some direct liberal attitude towards locus standi whenever the issue
injury as a result of its enforcement, and not merely that he presented for consideration has transcendental significance to
suffers thereby in some indefinite way. It must appear that the the people, or whenever the issues raised are of paramount
person complaining has been or is about to be denied some importance to the public.28
right or privilege to which he is lawfully entitled or that he is
about to be subjected to some burdens or penalties by reason B.Accession to the Madrid Protocol was constitutional
of the statute or act complained of.
The IPAP submits that respondents Executive Secretary and
It is true that as early as in 1937, in People v. Vera, the Court DFA Secretary Del Rosario gravely abused their discretion in
adopted the direct injury test for determining whether a determining that there was no need for the Philippine Senate's
petitioner in a public action had locus standi. There, the Court concurrence with the Madrid Protocol; that the Madrid
held that the person who would assail the validity of a statute Protocol involves changes of national policy, and its
must have "a personal and substantial interest in the case being of a permanent character requires the Senate's
such that he has sustained, or will sustain direct injury as a concurrence,29 pursuant to Section 21, Article VII of the
result." Vera was followed in Custodio v. President of the Constitution, which states that "no treaty or international
Senate, Manila Race Horse Trainers' Association v. De la agreement shall be valid and effective unless concurred
Fuente, Anti-Chinese League of the Philippines v. Felix, and in by at least two-thirds of all the Members of the Senate."
Pascual v. Secretary of Public Works.
Before going further, we have to distinguish between treaties
Yet, the Court has also held that the requirement of locus and international agreements, which require the Senate's
standi, being a mere procedural technicality, can be waived concurrence, on one hand, and executive agreements, which
by the Court in the exercise of its discretion. For instance, in may be validly entered into without the Senate's concurrence.
1949, in Araneta v. Dinglasan, the Court liberalized the Executive Order No. 459, Series of 1997,30 notes the
approach when the cases had "transcendental importance." following definitions, to wit:
Some notable controversies whose petitioners did not pass
the direct injury test were allowed to be treated in the same Sec. 2. Definition of Terms.
way as in Araneta v. Dinglasan.
a. International agreement - shall refer to a contract or
In the 1975 decision in Aquino v. Commission on Elections, understanding, regardless of nomenclature, entered into
this Court decided to resolve the issues raised by the petition between the Philippines and another government in written
due to their "farreaching implications,'; even if the petitioner form and governed by international law, whether embodied in
had no personality to file the suit. The liberal approach of a single instrument or in two or more related instruments.
Aquino v. Commission on Elections has been adopted in
several notable cases, permitting ordinary citizens, legislators, b. Treaties - international agreements entered into by the
and civic organizations to bring their suits involving the Philippines which require legislative concurrence after
constitutionality or validity of laws, regulations, and rulings. executive ratification. This term may include compacts like
conventions, declarations, covenants and acts.
However, the assertion of a public right as a predicate for
challenging a supposedly illegal or unconstitutional executive c. Executive Agreements - similar to treaties except that they
or legislative action rests on the theory that the petitioner do not require legislative concurrence.
represents the public in general. Although such petitioner may
not be as adversely affected by the action complained against
FINALS CONSTITUTIONAL LAW I ACJUCO NOV 11, 2017 60

The Court has highlighted the difference between treaties and correspondence between this and other government’s ends
executive agreements in Commissioner of Customs v. and agreements - whether denominated executive
Eastern Sea Trading,31 thusly: agreements or exchanges of notes or otherwise - begin, may
sometimes be difficult of ready ascertainment. It would be
International agreements involving political issues or useless to undertake to discuss here the large variety of
changes of national policy and those involving executive agreements as such, concluded from time to time.
international arrangements of a permanent character Hundreds of executive agreements, other than those entered
usually take the form of treaties. But international into under the trade-agreements act, have been negotiated
agreements embodying adjustments of detail carrying with foreign governments. x x x It would seem to be sufficient,
out well-established national policies and traditions and in order to show that the trade agreements under the act of
those involving arrangements of a more or less 1934 are not anomalous in character, that they are not
temporary nature usually take the form of executive treaties, and that they have abundant precedent in our history,
agreements. to refer to certain classes of agreements heretofore entered
into by the Executive without the approval of the Senate. They
In the Philippines, the DFA, by virtue of Section 9, Executive cover such subjects as the inspection of vessels, navigation
Order No. 459,32 is initially given the power to determine dues, income tax on shipping profits, the admission of civil
whether an agreement is to be treated as a treaty or as an aircraft, customs matters, and commercial relations generally,
executive agreement. To determine the issue of whether DFA international claims, postal matters, the registration of
Secretary Del Rosario gravely abused his discretion in making trademarks and copyrights, etcetera. Some of them were
his determination relative to the Madrid Protocol, we review concluded not by specific congressional authorization but in
the jurisprudence on the nature of executive agreements, as conformity with policies declared in acts of Congress with
well as the subject matters to be covered by executive respect to the general subject matter, such as tariff acts; while
agreements. still others, particularly those with respect of the settlement of
claims against foreign governments, were concluded
The pronouncement in Commissioner of Customs v. Eastern independently of any legislation. (Emphasis ours)
Sea Trading33is instructive, to wit:
As the foregoing pronouncement indicates, the registration of
x x x The concurrence of said House of Congress is required trademarks and copyrights have been the subject of executive
by our fundamental law in the making of "treaties" agreements entered into without the concurrence of the
(Constitution of the Philippines; Article VII, Section 10[7]), Senate. Some executive agreements have been concluded in
which are, however, distinct and different from "executive conformity with the policies declared in the acts of Congress
agreements," which may be validly entered into without such with respect to the general subject matter.
concurrence.
It then becomes relevant to examine our state policy on
"Treaties are formal documents which require ratification with intellectual property in general, as reflected in Section 2 of our
the approval of two thirds of the Senate. Executive IP Code, to wit:
agreements become binding through executive action without
the need of a vote by the Senate or by Congress. Section 2. Declaration of State Policy. - The State
recognizes that an effective intellectual and industrial
xxxx property system is vital to the development of domestic
and creative activity, facilitates transfer of technology,
"x x x the right of the Executive to enter into binding attracts foreign investments, and ensures market access
agreements without the necessity of subsequent for our products. It shall protect and secure the exclusive
Congressional approval has been confirmed by long usage. rights of scientists, inventors, artists and other gifted
From the earliest days of our history we have entered into citizens to their intellectual property and creations,
executive agreements covering such subjects as commercial particularly when beneficial to the people, for such
and consular relations, most-favored-nation rights, patent periods as provided in this Act.
rights, trademark and copyright protection, postal and
navigation arrangements and the settlement of claims. The The use of intellectual property bears a social function.
validity of these has never been seriously questioned by our To this end, the State shall promote the diffusion of
courts. knowledge and information for the promotion of national
development and progress and the common good.
xxxx
It is also the policy of the State to streamline administrative
Agreements with respect to the registration of trademarks procedures of registering patents, trademarks and copyright,
have been concluded by the Executive with various countries to liberalize the registration on the transfer of technology; and
under the Act of Congress of March 3, 1881 (21 Stat. 502), x to enhance the enforcement of intellectual property rights in
xx the Philippines.

xxxx In view of the expression of state policy having been made by


the Congress itself, the IPAP is plainly mistaken in
In this connection, Francis B. Sayre, former U.S. High asserting that "there was no Congressional act that
Commissioner to the Philippines, said in his work on "The authorized the accession of the Philippines to the Madrid
Constitutionality of Trade Agreement Acts": Protocol."34

Agreements concluded by the President which fall short of Accordingly, DFA Secretary Del Rosario’s determination
treaties are commonly referred to as executive agreements and treatment of the Madrid Protocol as an executive
and are no less common in our scheme of government than agreement; being in apparent contemplation of the
are the more formal instruments - treaties and conventions. express state policies on intellectual property as well as
They sometimes take the form of exchanges of notes and at within his power under Executive Order No. 459, are
other times that or more formal documents denominated upheld. We observe at this point that there are no hard and
'agreements' or 'protocols'. The point where ordinary fast rules on the propriety of entering into a treaty or an
FINALS CONSTITUTIONAL LAW I ACJUCO NOV 11, 2017 61

executive agreement on a given subject as an instrument of


international relations. The primary consideration in the choice
of the form of agreement is the parties' intent and desire to
craft their international agreement in the form they so wish to
further their respective interests. The matter of form takes a
back seat when it comes to effectiveness and binding effect of
the enforcement of a treaty or an executive agreement;
inasmuch as all the parties; regardless of the form, become
obliged to comply conformably with the time-honored principle
of pacta sunt servanda.35The principle binds the parties to
perform in good faith their parts in the agreements.36

c. There is no conflict between the Madrid Protocol and


the IP Code.

The IPAP also rests its challenge on the supposed conflict


between the Madrid Protocol and the IP Code, contending that
the Madrid Protocol does away with the requirement of a
resident agent under Section 125 of the IP Code; and that the
Madrid Protocol is unconstitutional for being in conflict with the
local law, which it cannot modify.

The IPAP's contentions stand on a faulty premise. The


method of registration through the IPOPHL, as laid down
by the IP Code, is distinct and separate from the method
of registration through the WIPO, as set in the Madrid
Protocol. Comparing the two methods of registration
despite their being governed by two separate systems of
registration is thus misplaced.

In arguing that the Madrid Protocol conflicts with Section 125


of the IP Code, the IP AP highlights the importance of the
requirement for the designation of a resident agent. It
underscores that the requirement is intended to ensure that
non-resident entities seeking protection or privileges under
Philippine Intellectual Property Laws will be subjected to the
country's jurisdiction. It submits that without such resident
agent, there will be a need to resort to costly, time consuming
and cumbersome extraterritorial service of writs and
processes.37

The IPAP misapprehends the procedure for examination


under the Madrid Protocol, The difficulty, which the IPAP
illustrates, is minimal, if not altogether inexistent. The IPOPHL
actually requires the designation of the resident agent when it
refuses the registration of a mark. Local representation is
further required in the submission of the Declaration of Actual
Use, as well as in the submission of the license contract.38
The Madrid Protocol accords with the intent and spirit of the
IP Code, particularly on the subject of the registration of
trademarks. The Madrid Protocol does not amend or modify
the IP Code on the acquisition of trademark rights considering
that the applications under the Madrid Protocol are still
examined according to the relevant national law, In that
regard, the IPOPHL will only grant protection to a mark that
meets the local registration requirements.

WHEREFORE, this Court DISMISSES the petition for


certiorari and prohibition for lack of merit; and ORDERS the
petitioner to pay the costs of suit.

SO ORDERED.
FINALS CONSTITUTIONAL LAW I ACJUCO NOV 11, 2017 62

LEGISLATION upon the existence of which the operation of a specific law or


regulation is made to depend, shall be promulgated in
It must be noted also that much of the needed legislation of proclamations which shall have the force of an executive
the country today considered by Congress originates from the order.
executive branch. Each year after the President of the
Philippines outlines his legislative program in his State-of-the- Memorandum orders — Acts of the President on matters of
Nation Address, executive departments and agencies administrative detail, or of subordinate or temporary interest
transmit to the House and the Senate drafts of proposed which only concern a particular officer or government office
legislations to carry out the President’s program. shall be embodied in memorandum orders.

Article VII, Section 1, of the 1987 Constitution vests executive Memorandum circulars — Acts of the President on matters
power on the President of the Philippines. The President is the relating to internal administration, which the President desires
Head of State and Head of Government, and functions as the to bring to the attention of all or some of the departments,
commander-in-chief of the Armed Forces of the Philippines. agencies, bureaus, or offices of the government, for
As chief executive, the President exercises control over all the information or compliance, shall be embodied in
executive departments, bureaus, and offices. memorandum circulars.

THE PRESIDENT OF THE PHILIPPINES General or special orders — Acts and commands of the
The President of the Philippines is elected by direct vote by President in his capacity as commander-in-chief of the Armed
the people for a term of six years. He may only serve for one Forces of the Philippines shall be issued as general or special
term, and is ineligible for reelection. The term of the President orders.
of the Philippines starts at noon of the 30th day of June after
the election. It is important to note that during the term of President
Ferdinand E. Marcos, he used executive issuances known
QUALIFICATIONS as presidential decrees as a form of legislation. These
The qualifications for an individual aspiring to become the decrees have the full force and effect of laws because at the
President of the Philippines are outlined in Article VII, Section time the legislature did not exist and, when the 1973
2 of the 1987 Constitution. According to the constitution, an Constitution was put into full force and effect, it gave the power
individual may become President provided he meets the to the President to do as such. This continued until the first
following criteria: year of President Corazon C. Aquino’s term. However,
1. natural born Filipino; President Aquino opted to used executive orders instead of
2. a registered voter; presidential decrees. President Aquino’s executive orders,
3. must be able to read and write; however, still had the full force and effect of laws until the
4. 40 years of age at the day of the election; and ratification of the 1987 Constitution.
5. must have resided in the Philippines ten years before
the election is held. 3. POWER OVER ALIENS
The President of the Philippines has certain powers over non-
POWERS OF THE PRESIDENT Filipinos in the Philippines. The powers he may exercise over
Besides the constitution, the powers of the President of the foreigners in the country are as follows:
Philippines are specifically outlined in Executive Order No.
292, s. 1987, otherwise known as the Administrative Code of  The chief executive may have an alien in the
1987. The following powers are: Philippines deported from the country after due
process.
1. POWER OF CONTROL OVER THE  The President may change the status of a foreigner,
EXECUTIVE BRANCH as prescribed by law, from a non-immigrant status to
a permanent resident status without necessity of
The President of the Philippines has the mandate of control visa.
over all the executive departments, bureaus, and offices. This  The President may choose to overrule the Board of
includes restructuring, reconfiguring, and appointments of Commissioners of the Bureau of Immigration before
their respective officials. The Administrative Code also their decision becomes final and executory (after 30
provides for the President to be responsible for the days of the issuance of the decision). The Board of
abovementioned offices’ strict implementation of laws. Commissioners of the Bureau of Immigration has
jurisdiction over all deportation cases.
2. POWER ORDINANCE POWER  The president is also mandated by the Administrative
Code of 1987 to exercise powers as recognized by
the generally accepted principles of international law.
The President of the Philippines has the power to give
executive issuances, which are means to streamline the policy
and programs of an administration. There are six issuances 4. POWERS OF EMINENT DOMAIN,
that the President may issue. They are the following as
defined in the Administrative Code of 1987:
ESCHEAT, LAND RESERVATION AND
Executive orders — Acts of the President providing for rules RECOVERY OF ILL-GOTTEN WEALTH
of a general or permanent character in implementation or
execution of constitutional or statutory powers shall be The President of the Philippines has the authority to exercise
promulgated in executive orders. the power of eminent domain. The power of eminent domains
Administrative orders — Acts of the President which relate to means the state has the power to seize or authorize the
particular aspects of governmental operations in pursuance of seizure of private property for public use with just
his duties as the administrative head shall be promulgated in compensation. There are two constitutional provisions,
administrative orders. however, that limit the exercise of such power: Article III,
Proclamations — Acts of the President fixing a date or Section 9 (1) of the Constitution provides that no person shall
declaring a status or condition of public moment or interest, be deprived of his/her life, liberty, or property without due
FINALS CONSTITUTIONAL LAW I ACJUCO NOV 11, 2017 63

process of law. Furthermore, Article III, Section 9 (2), provides 7. VETO POWER
that private property shall not be taken for public use without
just compensation.
SECTION 26, ART VI - The President shall communicate his
Once the aforementioned conditions are met, the President veto of any bill to the House where it originated within thirty
may exercise the power of eminent domain which are as days after the date of receipt thereof; otherwise, it shall
follows: become a law as if he had signed it.

Power of eminent domain — The President shall determine (2) The President shall have the power to veto any particular
when it is necessary or advantageous to exercise the power item or items in an appropriation, revenue, or tariff bill, but the
of eminent domain in behalf of the national government, and veto shall not affect the item or items to which he does not
direct the solicitor general, whenever he deems the action object.
advisable, to institute expropriation proceedings in the proper
court. 8. EMERGENCY POWERS
Power to direct escheat or reversion proceedings — The SECTION 23 (2) In times of war or other national emergency,
President shall direct the solicitor general to institute escheat the Congress may, by law, authorize the President, for a
or reversion proceedings over all lands transferred or limited period and subject to such restrictions as it may
assigned to persons disqualified under the constitution to prescribe, to exercise powers necessary and proper to carry
acquire land. out a declared national policy. Unless sooner withdrawn by
resolution of the Congress, such powers shall cease upon the
Power to reserve lands of the public and private domain of the next adjournment thereof.
government —
(1) The president shall have the power to reserve for 9. OTHER POWERS
settlement or public use, and for specific public purposes, any
of the lands of the public domain, the use of which is not
otherwise directed by law. The reserved land shall thereafter Aside from the aforementioned powers of the President of the
remain subject to the specific public purpose indicated until Philippines, he can also exercise powers enumerated in the
otherwise provided by law or proclamation. constitution, and powers given to him by law.
(2) He shall also have the power to reserve from sale or other
disposition and for specific public uses or purposes, any land
belonging to the private domain of the government, or any of LINE OF SUCCESSION
the friar lands, the use of which is not otherwise directed by
law, and thereafter such land shall be used for the purposes The constitution provides for a line of succession in the event
specified by such proclamation until otherwise provided by that the elected President of the Philippines is not able to
law. discharge the duties of his office due to death, disability, or
Power over ill-gotten wealth — The President shall direct the resignation. The following is the line of succession:
solicitor general to institute proceedings to recover properties 1. Vice President — in cases of the death, disability, or
unlawfully acquired by public officials or employees, from resignation of the President
them or from their nominees or transferees. 2. Senate President — in cases of the death, disability,
Within the period fixed in, or any extension thereof authorized or resignation of the President and Vice President
by, the constitution, the President shall have the authority to 3. Speaker of the House of Representatives — in cases
recover ill-gotten properties amassed by the leaders and of the death, disability, or resignation of the
supporters of the previous regime, and protect the interest of President, Vice President, and Senate President
the people through orders of sequestration or freezing of Contrary to popular belief, the constitution doesn’t include the
assets or accounts. Chief Justice of the Supreme Court in the President’s line of
succession.

5. POWER OF APPOINTMENT The Congress of the Philippines is mandated enact a law


calling for a special election three days after the vacancy in
The President may appoint officials of the Philippine the Office of the President and Vice President. The special
government as provided by the constitution and laws of the election should occur 40 days after the enactment of the law,
Philippines. Some of these appointments, however, may need but not later than 60 days after the enactment of the law.
the approval of the Committee on Appointments (a committee
composed of members from the House of Representatives VICE PRESIDENT OF THE PHILIPPINES
and the Senate of the Philippines).
The Vice President of the Philippines is elected by direct vote
6. POWER OF GENERAL SUPERVISION by the people for a term of six years, and may run for
OVER LOCAL GOVERNMENTS reelection once. The term of the Vice President of the
Philippines starts at noon of the 30th day of June after
a regular election is held.
The President of the Philippines, as chief executive, has the
mandate to supervise local governments in the Philippines, QUALIFICATIONS
despite their autonomous status as provided by Republic Act
No. 7160 otherwise known as the Local Government Code of The qualifications for aspirants to the Office of the Vice
1991. President is outlined in Article VII, Section 3. According to the
constitution, the qualifications for the President is the same for
Traditionally, this is done by the Department of the Interior and the Vice President.
Local Government, headed by a cabinet secretary—an alter
ego of the President. DUTIES OF THE VICE PRESIDENT
FINALS CONSTITUTIONAL LAW I ACJUCO NOV 11, 2017 64

According to the constitution, the vice president may POWERS OF A CABINET SECRETARY
concurrently assume a cabinet position should the President As stated above, a cabinet secretary is the alter ego of the
of the Philippines offer the former one. The vice president will President in their respective departments. Thus, they posses
become a secretary concurrent to the position of vice the power to issue directives relative to their departments,
president. such as department orders. These orders only apply to offices
Aside from the cabinet post, the vice president is mandated to under a specific department under the cabinet secretary’s
assume the presidency in case of the death, disability, or jurisdiction. Cabinet secretaries also act as advisors to the
resignation of the incumbent President. President of the Philippines for their areas.

LINE OF SUCCESSION LOCAL GOVERNMENTS


Should there be a vacancy of the Office of the Vice President, The executive branch extends beyond the national
the President of the Philippines is required by the constitution government. According to Article X, Section 4 of the
to nominate a replacement with the concurrence of Committee constitution, the President of the Philippines is mandated to
on Appointments. supervise local governments all over the country. However,
because of Republic Act No. 7160, otherwise known as the
CABINET SECRETARIES Local Government Code of 1991, local governments enjoy
relative autonomy from the national government.
FUNCTIONS OF A CABINET SECRETARY Among the social services and facilities that local government
Cabinet secretaries act as the alter ego of the President should provide, as stipulated in Section 17 of the Local
executing, with his authority, the power of the Office of the Government Code, are the following:
President in their respective departments.  facilities and research services for agriculture and
The number of cabinet secretaries varies from time to time fishery activities, which include seedling nurseries,
depending on the need of an administration. According to the demonstration farms, and irrigation systems;
Administrative Code of 1987, the President of the Philippines  health services, which include access to primary
may create or dissolve any department as he sees fit. health care, maternal and child care, and medicines,
medical supplies and equipment;
APPOINTMENT OF CABINET SECRETARIES
 social welfare services, which include programs and
According to the Article 7, Section 16, the President may
projects for women, children, elderly, and persons
appoint anyone to executive departments with the consent of
with disabilities, as well as vagrants, beggars, street
the Commission on Appointments. Names of individuals
children, juvenile delinquents, and victims of drug
nominated to cabinet posts are submitted to the Commission
abuse;
on Appointments for their consideration.
An individual may not assume his post in a given department  information services, which include job placement
unless confirmed by the Commission on Appointments. information systems and a public library;
However, the constitution provides for individuals becoming  a solid waste disposal system or environmental
cabinet secretaries in an acting capacity before they are management system;
confirmed. According to Article VII, Section 16 of the  municipal/city/provincial buildings, cultural centers,
constitution, the president may appoint anyone to cabinet public parks, playgrounds, and sports facilities and
posts even if Congress is in recess. These appointments are equipment;
valid until the Commission on Appointments disapproves  infrastructure facilities such as roads, bridges, school
them, or at the end of the next session of Congress. buildings, health clinics, fish ports, water supply
Not all cabinet members, however, are subject to confirmation systems, seawalls, dikes, drainage and sewerage,
of the Commission on Appointments. According to the and traffic signals and road signs;
Commission of Appointments website, the following need  public markets, slaughterhouses, and other local
confirmation in order to assume their posts: enterprises;
1. Executive Secretary  public cemetery;
2. Secretary of Agrarian Reform
3. Secretary of Agriculture
 tourism facilities and other tourist attractions; and
4. Secretary of Budget and Management  sites for police and fire stations and substations and
5. Secretary of Education municipal jail.
6. Secretary of Energy Local government units also have the power to create its own
7. Secretary of Environment and Natural sources of revenue and to levy taxes, fees, and charges that
Resources shall accrue exclusively to them.
8. Secretary of Finance Each local government has its own chief executive. The
9. Secretary of Foreign Affairs following is the list of local chief executives:
10. Secretary of Health 1. barangay — punong barangay (barangay chairman)
11. Secretary of Justice 2. municipality — municipal mayor
12. Secretary of Labor and Employment 3. city — city mayor
13. Secretary of National Defense 4. province — provincial governor
14. Secretary of Public Works and Highways The local chief executives have the power to approve or veto
15. Secretary of Science and Technology local ordinances recommended by the local legislators.
16. Secretary of Social Welfare and
Development Punong barangay
17. Secretary of the Interior and Local The punong barangay, as the chief executive of the barangay
Government government, shall exercise and perform the following powers
18. Secretary of Trade and Industry and functions:
19. Secretary of Transportation and 1. enforce all laws and ordinances which are applicable
Communications within the barangay;
20. Secretary of Tourism 2. negotiate, enter into, and sign contracts for and in
21. Commission on Higher Education behalf of the barangay, upon authorization of the
22. Director General of the National Economic Sangguniang Barangay;
and Development Authority 3. maintain public order in the barangay;
FINALS CONSTITUTIONAL LAW I ACJUCO NOV 11, 2017 65

4. call and preside over the sessions of the 2. Enforce all laws and ordinances, and implement all
Sangguniang Barangay and the Barangay approved policies, programs, projects, services and
Assembly; activities of the municipality or city:
5. appoint or replace the barangay treasurer, the 1. issue executive orders as are necessary for
barangay secretary, and other appointive barangay the proper enforcement and execution of
officials; laws and ordinances;
6. organize and lead an emergency group for the 2. call conventions, seminars or meetings of
maintenance of peace and order or on occasions of any elective and appointive officials of the
emergency or calamity within the barangay; municipality or city;
7. prepare the annual executive and supplemental 3. formulate and implement the peace and
budgets of the barangay, in coordination with the order plan of the municipality or city; and
Barangay Development Council; 4. call upon the appropriate law enforcement
8. approve vouchers relating to the disbursement of agencies to suppress disorder, riot, lawless
barangay funds; violence, rebellion or sedition or to
9. enforce laws and regulations relating to pollution apprehend violators of the law.
control and protection of the environment; 3. Initiate and maximize the generation of resources
10. administer the operation of the Katarungang and revenues, to be used for the implementation of
Pambarangay; development plans, program objectives and
11. exercise general supervision over the activities of the priorities:
Sangguniang Kabataan; 1. prepare and submit to the Sangguniang
12. ensure the delivery of basic social services and Bayan or Panlungsod for approval the
access to facilities; annual executive and supplemental
13. conduct an annual palarong barangay which shall budgets of the municipality or city;
feature traditional sports and disciplines included in 2. ensure that all taxes and other revenues of
national and international games; and the municipality or city are collected;
14. promote the general welfare of the barangay. 3. issue, suspend or revoke licenses and
permits;
4. adopt measures to safeguard and conserve
land, mineral, marine, forest, and other
Municipal and city mayors resources of the municipality or city;
The municipal mayor and city mayor, as the chief executive of 5. provide efficient and effective property and
the municipal government and city government, respectively, supply management in the municipality or
shall exercise and perform the following powers and functions: city; and protect the funds, credits, rights
1. Exercise general supervision and control over all and other properties of the municipality or
programs, projects, services, and activities of the city; and
municipal or city government: 6. institute or cause to be instituted
1. determine the guidelines of municipal administrative or judicial proceedings for
policies and be responsible to the violation of ordinances in the collection of
Sangguniang Bayan or Panlungsod for the taxes, fees or charges, and for the recovery
program of government; of funds and property.
2. direct the formulation of the municipal or 4. Ensure the delivery of basic services and the
city development plan; provision of adequate facilities as provided for under
3. at the opening of the regular session of the Section 17 of the Local Government Code.
Sangguniang Bayan or Panlungsod,
present the program of government and Provincial governors
propose policies and projects for The provincial governor, as the chief executive of the
consideration; provincial government, shall exercise and perform the
4. initiate and propose legislative measures to following powers and duties:
the Sangguniang Bayan or Panlungsod; 1. Exercise general supervision and control over all
5. represent the municipality or city in all its programs, projects, services, and activities of the
business transactions and sign on its behalf provincial government:
all bonds, contracts, and obligations, upon 1. determine the guidelines of municipal
authorization by the Sangguniang Bayan; policies and be responsible to the
6. carry out emergency measures as may be Sangguniang Panlalawigan for the program
necessary during and in the aftermath of of government;
man-made and natural disasters; 2. direct the formulation of the provincial
7. examine the books, records and other development plan;
documents of all offices, officials, agents or 3. at the opening of the regular session of the
employees of the municipality or city; Sangguniang Panlalawigan, present the
8. visit component barangays of the program of government and propose
municipality or city at least once every six policies and projects for consideration;
months; 4. initiate and propose legislative measures to
9. solemnize marriages, any provision of law the Sangguniang Panlalawigan;
to the contrary notwithstanding; 5. represent the province in all its business
10. conduct a palarong bayan or panlungsod; transactions and sign on its behalf all
and bonds, contracts, and obligations, upon
11. submit to the provincial governor an annual authorization by the Sangguniang
report on the administration of the Panlalawigan;
municipality or city, and supplemental 6. carry out emergency measures as may be
reports when unexpected events and necessary during and in the aftermath of
situations such as calamities arise. man-made and natural disasters;
FINALS CONSTITUTIONAL LAW I ACJUCO NOV 11, 2017 66

7. examine the books, records and other


documents of all offices, officials, agents or
employees of the province;
8. furnish copies of executive orders issued by
him to the Office of the President within 72
hours after their issuance;
9. visit component cities and municipalities at
least once every six months;
10. represent the province in inter-provincial or
regional sports councils or committees, and
coordinate the efforts of component cities or
municipalities in the regional or national
palaro or sports development activities;
11. conduct an annual palarong panlalawigan;
and
12. submit to the Office of the President an
annual report on the administration and
development of the province, and
supplemental reports when unexpected
events and situations such as calamities
arise.
2. Enforce all laws and ordinances, and implement all
approved policies, programs, projects, services and
activities of the province:
1. issue executive orders as are necessary for
the proper enforcement and execution of
laws and ordinances;
2. call conventions, seminars or meetings of
any elective and appointive officials of the
province;
3. in coordination with the component cities
and municipalities, and the National Police
Commission, formulate and implement the
peace and order plan of the province; and
4. call upon the appropriate law enforcement
agencies to suppress disorder, riot, lawless
violence, rebellion or sedition or to
apprehend violators of the law.
3. Initiate and maximize the generation of resources
and revenues, to be used for the implementation of
development plans, program objectives and
priorities:
1. prepare and submit to the Sangguniang
Panlalawigan for approval the annual
executive and supplemental budgets of the
province;
2. ensure that all taxes and other revenues of
the province are collected;
3. issue, suspend or revoke licenses and
permits;
4. adopt measures to safeguard and conserve
land, mineral, marine, forest, and other
resources of the province;
5. provide efficient and effective property and
supply management in the province; and
protect the funds, credits, rights and other
properties of the province; and
6. institute or cause to be instituted
administrative or judicial proceedings for
violation of ordinances in the collection of
taxes, fees or charges, and for the recovery
of funds and property.
4. Ensure the delivery of basic services and the
provision of adequate facilities as provided for under
Section 17 of the Local Government Code.

TERM LIMITS
The offices of the abovementioned local chief executives are
limited to three consecutive three-year terms. Once they end
their third term, they may not run for reelection, but may run
again once they let one term pass.
FINALS CONSTITUTIONAL LAW I ACJUCO NOV 11, 2017 67

IMMUNITY FROM SUIT 1988. The motion for reconsideration was denied by the
Executive Secretary on May 16, 1988. With these
G.R. No. 82585 November 14, 1988 developments, petitioners' contention that they have been
denied the administrative remedies available under the law
MAXIMO V. SOLIVEN, ANTONIO V. ROCES, FREDERICK has lost factual support.
K. AGCAOLI, and GODOFREDO L. MANZANAS,
petitioners, It may also be added that with respect to petitioner Beltran,
vs. the allegation of denial of due process of law in the preliminary
THE HON. RAMON P. MAKASIAR, Presiding Judge of the investigation is negated by the fact that instead of submitting
Regional Trial Court of Manila, Branch 35, his counter- affidavits, he filed a "Motion to Declare
UNDERSECRETARY SILVESTRE BELLO III, of the Proceedings Closed," in effect waiving his right to refute the
Department of Justice, LUIS C. VICTOR, THE CITY complaint by filing counter-affidavits. Due process of law does
FISCAL OF MANILA and PRESIDENT CORAZON C. not require that the respondent in a criminal case actually file
AQUINO, respondents. his counter-affidavits before the preliminary investigation is
deemed completed. All that is required is that the respondent
G.R. No. 82827 November 14, 1988 be given the opportunity to submit counter-affidavits if he is so
minded.
LUIS D. BELTRAN, petitioner,
vs. The second issue, raised by petitioner Beltran, calls for an
THE HON. RAMON P. MAKASIAR, Presiding Judge of interpretation of the constitutional provision on the issuance of
Branch 35 of the Regional Trial Court, at Manila, THE HON. warrants of arrest. The pertinent provision reads:
LUIS VICTOR, CITY FISCAL OF MANILA, PEOPLE OF THE
PHILIPPINES, SUPERINTENDENT OF THE WESTERN Art. III, Sec. 2. The right of the people to be secure in their
POLICE DISTRICT, and THE MEMBERS OF THE PROCESS persons, houses, papers and effects against unreasonable
SERVING UNIT AT THE REGIONAL TRIAL COURT OF searches and seizures of whatever nature and for any
MANILA, respondents. purpose shall be inviolable, and no search warrant or warrant
of arrest shall issue except upon probable cause to be
G.R. No. 83979 November 14, 1988. determined personally by the judge after examination nder
oath or affirmation of the complainant and the witnesses he
LUIS D. BELTRAN, petitioner, may produce, and particularly describing the place to be
vs. searched and the persons or things to be seized.
EXECUTIVE SECRETARY CATALINO MACARAIG,
SECRETARY OF JUSTICE SEDFREY ORDOÑEZ, The addition of the word "personally" after the word
UNDERSECRETARY OF JUSTICE SILVESTRE BELLO III, "determined" and the deletion of the grant of authority by the
THE CITY FISCAL OF MANILA JESUS F. GUERRERO, and 1973 Constitution to issue warrants to "other responsible
JUDGE RAMON P. MAKASIAR, Presiding Judge of Branch officers as may be authorized by law," has apparently
35 of the Regional Trial Court, at Manila, respondents. convinced petitioner Beltran that the Constitution now requires
the judge to personally examine the complainant and his
Angara, Abello, Concepcion, Regala and Cruz for petitioners witnesses in his determination of probable cause for the
in G.R. No. 82585. issuance of warrants of arrest. This is not an accurate
interpretation.
Perfecto V. Fernandez, Jose P. Fernandez and Cristobal P.
Fernandez for petitioner in G.R. Nos. 82827 and 83979. What the Constitution underscores is the exclusive and
personal responsibility of the issuing judge to satisfy himself
RESOLUTION of the existence of probable cause. In satisfying himself of the
existence of probable cause for the issuance of a warrant of
PER CURIAM: arrest, the judge is not required to personally examine the
complainant and his witnesses. Following established
In these consolidated cases, three principal issues were doctrine and procedure, he shall: (1) personally evaluate the
raised: (1) whether or not petitioners were denied due report and the supporting documents submitted by the fiscal
process when informations for libel were filed against regarding the existence of probable cause and, on the basis
them although the finding of the existence of a prima facie thereof, issue a warrant of arrest; or (2) if on the basis thereof
case was still under review by the Secretary of Justice he finds no probable cause, he may disregard the fiscal's
and, subsequently, by the President; (2) whether or not report and require the submission of supporting affidavits of
the constitutional rights of Beltran were violated when witnesses to aid him in arriving at a conclusion as to the
respondent RTC judge issued a warrant for his arrest existence of probable cause.
without personally examining the complainant and the
witnesses, if any, to determine probable cause; and (3) Sound policy dictates this procedure, otherwise judges would
whether or not the President of the Philippines, under the be unduly laden with the preliminary examination and
Constitution, may initiate criminal proceedings against investigation of criminal complaints instead of concentrating
the petitioners through the filing of a complaint-affidavit. on hearing and deciding cases filed before their courts.

Subsequent events have rendered the first issue moot and On June 30, 1987, the Supreme Court unanimously adopted
academic. On March 30, 1988, the Secretary of Justice denied Circular No. 12, setting down guidelines for the issuance of
petitioners' motion for reconsideration and upheld the warrants of arrest. The procedure therein provided is
resolution of the Undersecretary of Justice sustaining the City reiterated and clarified in this resolution.
Fiscal's finding of a prima facie case against petitioners. A
second motion for reconsideration filed by petitioner Beltran It has not been shown that respondent judge has deviated
was denied by the Secretary of Justice on April 7, 1988. On from the prescribed procedure. Thus, with regard to the
appeal, the President, through the Executive Secretary, issuance of the warrants of arrest, a finding of grave abuse of
affirmed the resolution of the Secretary of Justice on May 2, discretion amounting to lack or excess of jurisdiction cannot
be sustained.
FINALS CONSTITUTIONAL LAW I ACJUCO NOV 11, 2017 68

this is the more important issue in these petitions and it should


Anent the third issue, petitioner Beltran argues that "the be resolved now rather that later.
reasons which necessitate presidential immunity from suit
impose a correlative disability to file suit." He contends that Consistent with our decision in Salonga v. Cruz Pano (134
if criminal proceedings ensue by virtue of the President's SCRA 438 [1985]), the Court should not hesitate to quash a
filing of her complaint-affidavit, she may subsequently criminal prosecution in the interest of more enlightened and
have to be a witness for the prosecution, bringing her substantial justice where it is not alone the criminal liability of
under the trial court's jurisdiction. This, continues Beltran, an accused in a seemingly minor libel case which is involved
would in an indirect way defeat her privilege of immunity from but broader considerations of governmental power versus a
suit, as by testifying on the witness stand, she would be preferred freedom.
exposing herself to possible contempt of court or perjury.
We have in these four petitions the unusual situation where
The rationale for the grant to the President of the privilege the highest official of the Republic and one who enjoys
of immunity from suit is to assure the exercise of unprecedented public support asks for the prosecution of a
Presidential duties and functions free from any hindrance newspaper columnist, the publisher and chairman of the
or distraction, considering that being the Chief Executive editorial board, the managing editor and the business
of the Government is a job that, aside from requiring all manager in a not too indubitable a case for alleged libel.
of the office holder's time, also demands undivided
attention. I am fully in accord with an all out prosecution if the effect will
be limited to punishing a newspaperman who, instead of
But this privilege of immunity from suit, pertains to the observing accuracy and fairness, engages in unwarranted
President by virtue of the office and may be invoked only personal attacks, irresponsible twisting of facts, of malicious
by the holder of the office; not by any other person in the distortions of half-truths which tend to cause dishonor,
President's behalf. Thus, an accused in a criminal case in discredit, or contempt of the complainant. However, this case
which the President is complainant cannot raise the is not a simple prosecution for libel. We have as complainant
presidential privilege as a defense to prevent the case a powerful and popular President who heads the investigation
from proceeding against such accused. and prosecution service and appoints members of appellate
courts but who feels so terribly maligned that she has taken
Moreover, there is nothing in our laws that would prevent the the unorthodox step of going to court inspite of the invocations
President from waiving the privilege. Thus, if so minded the of freedom of the press which would inevitably follow.
President may shed the protection afforded by the
privilege and submit to the court's jurisdiction. The I believe that this Court should have acted on this issue now
choice of whether to exercise the privilege or to waive it instead of leaving the matter to fiscals and defense lawyers to
is solely the President's prerogative. It is a decision that argue before a trial judge.
cannot be assumed and imposed by any other person.
There is always bound to be harassment inherent in any
As regards the contention of petitioner Beltran that he could criminal prosecution. Where the harassment goes beyond the
not be held liable for libel because of the privileged character usual difficulties encountered by any accused and results in
or the publication, the Court reiterates that it is not a trier of an unwillingness of media to freely criticize government or to
facts and that such a defense is best left to the trial court to question government handling of sensitive issues and public
appreciate after receiving the evidence of the parties. affairs, this Court and not a lower tribunal should draw the
demarcation line.
As to petitioner Beltran's claim that to allow the libel case to
proceed would produce a "chilling effect" on press freedom, As early as March 8, 1918, the decision in United States v.
the Court finds no basis at this stage to rule on the point. Bustos (37 Phil. 731) stated that "(c)omplete liberty to
comment on the conduct of public men is a scalpel in the case
The petitions fail to establish that public respondents, through of free speech. The sharp incision of its probe relieves the
their separate acts, gravely abused their discretion as to abscesses of officialdom. Men in public life may suffer under
amount to lack of jurisdiction. Hence, the writs of certiorari and a hostile and unjust accusation; the wound can be assuaged
prohibition prayed for cannot issue. with the balm of a clear conscience." The Court pointed out
that while defamation is not authorized, criticism is to be
WHEREFORE, finding no grave abuse of discretion expected and should be borne for the common good.
amounting to excess or lack of jurisdiction on the part of the
public respondents, the Court Resolved to DISMISS the In People v. Perfecto (43 Phil. 887 [1922]), the Court stated:
petitions in G. R. Nos. 82585, 82827 and 83979. The Order to
maintain the status quo contained in the Resolution of the xxx xxx xxx
Court en banc dated April 7, 1988 and reiterated in the
Resolution dated April 26, 1988 is LIFTED. ... No longer is there a Minister of the Crown own or a person
in authority of such exalted position that the citizen must speak
Fernan, C.J., Narvasa, Melencio-Herrera, Cruz, Paras, of him only with bated breath. "In the eye of our Constitution
Feliciano, Gancayco, Padilla, Bidin, Sarmiento, Cortes, Griño- and laws, every man is a sovereign, a ruler and a freeman,
Aquino Medialdea and Regalado, JJ., concur. and has equal rights with every other man." (at p. 900)

In fact, the Court observed that high official position, instead


Separate Opinions of affording immunity from slanderous and libelous charges,
GUTIERREZ, JR., J., concurring: would actually invite attacks by those who desire to create
sensation. It would seem that what would ordinarily be slander
I concur with the majority opinion insofar as it involves the if directed at the typical person should be examined from
three principal issues mentioned in its opening statement. various perspectives if directed at a high government official.
However, as to the more important issue on whether or not Again, the Supreme Court should draw this fine line instead of
the prosecution of the libel case would produce a "chilling leaving it to lower tribunals.
effect" on press freedom, I beg to reserve my vote. I believe
FINALS CONSTITUTIONAL LAW I ACJUCO NOV 11, 2017 69

This Court has stressed as authoritative doctrine in Elizalde v. ordinarily, libel is not protected by the free speech clause but
Gutierrez (76 SCRA 448 [1977]) that a prosecution for libel we have to understand that some provocative words, which if
lacks justification if the offending words find sanctuary within taken literally may appear to shame or disparage a public
the shelter of the free press guaranty. In other words, a figure, may really be intended to provoke debate on public
prosecution for libel should not be allowed to continue, where issues when uttered or written by a media personality. Will not
after discounting the possibility that the words may not be a criminal prosecution in the type of case now before us
really that libelous, there is likely to be a chilling effect, a dampen the vigor and limit the variety of public debate? There
patently inhibiting factor on the willingness of newspapermen, are many other questions arising from this unusual case which
especially editors and publishers to courageously perform have not been considered.
their critical role in society. If, instead of merely reading more
carefully what a columnist writes in his daily column, the I, of course, concur with the Court's opinion because it has
editors tell their people to lay off certain issues or certain decided to limit the issues to narrowly drawn ones. I see no
officials, the effect on a free press would be highly injurious. reason to disagree with the way the Court has resolved them.
The first issue on prematurity is moot. The second issue
Because many questions regarding press freedom are left discusses a procedure now embodied in the recently
unanswered by our resolution, I must call attention to our amended Rules of Court on how a Judge should proceed
decisions which caution that "no inroads on press freedom before he issues a warrant of arrest. Anent the third issue,
should be allowed in the guise of punitive action visited on considerations of public policy dictate that an incumbent
what otherwise should be characterized as libel." (Lopez v. President should not be sued. At the same time, the President
Court of Appeals, 34 SCRA 117 [1970]; See also the citations cannot stand by helplessly bereft of legal remedies if
in Elizalde v. Gutierrez, supra). somebody vilifies or maligns him or her.

The United States Supreme Court is even more emphatic, to The Court has decided to defer the "chilling effect" issue for a
wit: later day. To this, I take exception. I know that most of our
fiscals and judges are courageous individuals who would not
In deciding the question now, we are compelled by neither allow any considerations of possible consequences to their
precedent nor policy to give any more weight to the epithet careers to stand in the way of public duty. But why should we
"libel" than we have to other "mere labels" of state law. N. A. subject them to this problem? And why should we allow the
A. C. P. v. Button, 371 US 415, 429, 9L ed 2d 405, 415, 83 S possibility of the trial court treating and deciding the case as
Ct 328. Like insurrection, contempt, advocacy of unlawful one for ordinary libel without bothering to fully explore the
acts, breach of the peace, obscenity, solicitation of legal more important areas of concern, the extremely difficult issues
business, and the other various other formulae for the involving government power and freedom of expression.
repression of expression that have been challenged in this
Court, libel can claim no talismanic immunity from However, since we have decided to defer the "chilling effect"
constitutional limitations. It must be measured by standards issue for a later day, I limit myself to reiterating the dissenting
that satisfy the First Amendment. words of Mr. Justice Jackson in the American case of
Beaurnhais v. Illinois (343 U. S. 250) when he said:
xxx xxx xxx
If one can claim to announce the judgment of legal history on
Those who won our independence believed ... that public any subject, it is that criminal libel laws are consistent with the
discussion is a political duty; and that this should be a concept of ordered liberty only when applied with safeguards
fundamental principle of the American government. They evolved to prevent their invasion of freedom of expression.
recognized the risk to which all human institutions are subject.
But they knew that order cannot be secured merely through In the trial of the libel case against the petitioners, the
fear of punishment for its infraction; that it is hazardous to safeguards in the name of freedom of expression should be
discourage thought, hope and imagination; that fear breeds faithfully applied.
repression; that repression breeds hate; that hate menaces
stable government; that the path of safety lies in the Separate Opinions
opportunity to discuss freely supposed grievances and
proposed remedies; and that the fitting remedy for evil counsel GUTIERREZ, JR., J., concurring:
is good ones. Believing in the power of reason as applied
through public discussion, they eschewed silence coerced by I concur with the majority opinion insofar as it involves the
law—the argument of force in its worst form. ... three principal issues mentioned in its opening statement.
However, as to the more important issue on whether or not
Thus we consider this case against the background of a the prosecution of the libel case would produce a "chilling
profound national commitment to the principle that debate on effect" on press freedom, I beg to reserve my vote. I believe
public issues should be uninhibited, robust, and wide open, this is the more important issue in these petitions and it should
and that it may well include vehement, caustic, and sometimes be resolved now rather that later.
unpleasantly sharp attacks on government and public officials.
... (at pp. 700-701) Consistent with our decision in Salonga v. Cruz Pano (134
SCRA 438 [1985]), the Court should not hesitate to quash a
Shunting aside the individual liability of Mr. Luis Beltran, is criminal prosecution in the interest of more enlightened and
there a prima facie showing that Messrs. Maximo Soliven, substantial justice where it is not alone the criminal liability of
Antonio V. Roces, Frederick K. Agcaoili, and Godofredo L. an accused in a seemingly minor libel case which is involved
Manzanas knowingly participated in a wilful purveying of but broader considerations of governmental power versus a
falsehood? Considering the free speech aspects of these preferred freedom.
petitions, should not a differentiated approach to their
particular liabilities be taken instead of lumping up everybody We have in these four petitions the unusual situation where
with the offending columnist? I realize that the law includes the highest official of the Republic and one who enjoys
publishers and editors but perhaps the "chilling effect" issue unprecedented public support asks for the prosecution of a
applies with singular effectivity to publishers and editors vis-a- newspaper columnist, the publisher and chairman of the
vis newspaper columnists. There is no question that,
FINALS CONSTITUTIONAL LAW I ACJUCO NOV 11, 2017 70

editorial board, the managing editor and the business Because many questions regarding press freedom are left
manager in a not too indubitable a case for alleged libel. unanswered by our resolution, I must call attention to our
decisions which caution that "no inroads on press freedom
I am fully in accord with an all out prosecution if the effect will should be allowed in the guise of punitive action visited on
be limited to punishing a newspaperman who, instead of what otherwise should be characterized as libel." (Lopez v.
observing accuracy and fairness, engages in unwarranted Court of Appeals, 34 SCRA 117 [1970]; See also the citations
personal attacks, irresponsible twisting of facts, of malicious in Elizalde v. Gutierrez, supra).<äre||anº•1àw>
distortions of half-truths which tend to cause dishonor,
discredit, or contempt of the complainant. However, this case The United States Supreme Court is even more emphatic, to
is not a simple prosecution for libel. We have as complainant wit:
a powerful and popular President who heads the investigation
and prosecution service and appoints members of appellate In deciding the question now, we are compelled by neither
courts but who feels so terribly maligned that she has taken precedent nor policy to give any more weight to the epithet
the unorthodox step of going to court inspite of the invocations "libel" than we have to other "mere labels" of state law. N. A.
of freedom of the press which would inevitably follow. A. C. P. v. Button, 371 US 415, 429, 9L ed 2d 405, 415, 83 S
Ct 328. Like insurrection, contempt, advocacy of unlawful
I believe that this Court should have acted on this issue now acts, breach of the peace, obscenity, solicitation of legal
instead of leaving the matter to fiscals and defense lawyers to business, and the other various other formulae for the
argue before a trial judge. repression of expression that have been challenged in this
Court, libel can claim no talismanic immunity from
There is always bound to be harassment inherent in any constitutional limitations. It must be measured by standards
criminal prosecution. Where the harassment goes beyond the that satisfy the First Amendment.
usual difficulties encountered by any accused and results in
an unwillingness of media to freely criticize government or to xxx xxx xxx
question government handling of sensitive issues and public
affairs, this Court and not a lower tribunal should draw the Those who won our independence believed ... that public
demarcation line. discussion is a political duty; and that this should be a
fundamental principle of the American government. They
As early as March 8, 1918, the decision in United States v. recognized the risk to which all human institutions are subject.
Bustos (37 Phil. 731) stated that "(c)omplete liberty to But they knew that order cannot be secured merely through
comment on the conduct of public men is a scalpel in the case fear of punishment for its infraction; that it is hazardous to
of free speech. The sharp incision of its probe relieves the discourage thought, hope and imagination; that fear breeds
abscesses of officialdom. Men in public life may suffer under repression; that repression breeds hate; that hate menaces
a hostile and unjust accusation; the wound can be assuaged stable government; that the path of safety lies in the
with the balm of a clear conscience." The Court pointed out opportunity to discuss freely supposed grievances and
that while defamation is not authorized, criticism is to be proposed remedies; and that the fitting remedy for evil counsel
expected and should be borne for the common good. is good ones. Believing in the power of reason as applied
through public discussion, they eschewed silence coerced by
In People v. Perfecto (43 Phil. 887 [1922]), the Court stated: law—the argument of force in its worst form. ...

xxx xxx xxx Thus we consider this case against the background of a
profound national commitment to the principle that debate on
... No longer is there a Minister of the Crown own or a person public issues should be uninhibited, robust, and wide open,
in authority of such exalted position that the citizen must speak and that it may well include vehement, caustic, and sometimes
of him only with bated breath. "In the eye of our Constitution unpleasantly sharp attacks on government and public officials.
and laws, every man is a sovereign, a ruler and a freeman, ... (at pp. 700-701)
and has equal rights with every other man." (at p. 900)
Shunting aside the individual liability of Mr. Luis Beltran, is
In fact, the Court observed that high official position, instead there a prima facie showing that Messrs. Maximo Soliven,
of affording immunity from slanderous and libelous charges, Antonio V. Roces, Frederick K. Agcaoili, and Godofredo L.
would actually invite attacks by those who desire to create Manzanas knowingly participated in a wilful purveying of
sensation. It would seem that what would ordinarily be slander falsehood? Considering the free speech aspects of these
if directed at the typical person should be examined from petitions, should not a differentiated approach to their
various perspectives if directed at a high government official. particular liabilities be taken instead of lumping up everybody
Again, the Supreme Court should draw this fine line instead of with the offending columnist? I realize that the law includes
leaving it to lower tribunals. publishers and editors but perhaps the "chilling effect" issue
applies with singular effectivity to publishers and editors vis-a-
This Court has stressed as authoritative doctrine in Elizalde v. vis newspaper columnists. There is no question that,
Gutierrez (76 SCRA 448 [1977]) that a prosecution for libel ordinarily, libel is not protected by the free speech clause but
lacks justification if the offending words find sanctuary within we have to understand that some provocative words, which if
the shelter of the free press guaranty. In other words, a taken literally may appear to shame or disparage a public
prosecution for libel should not be allowed to continue, where figure, may really be intended to provoke debate on public
after discounting the possibility that the words may not be issues when uttered or written by a media personality. Will not
really that libelous, there is likely to be a chilling effect, a a criminal prosecution in the type of case now before us
patently inhibiting factor on the willingness of newspapermen, dampen the vigor and limit the variety of public debate? There
especially editors and publishers to courageously perform are many other questions arising from this unusual case which
their critical role in society. If, instead of merely reading more have not been considered.
carefully what a columnist writes in his daily column, the
editors tell their people to lay off certain issues or certain I, of course, concur with the Court's opinion because it has
officials, the effect on a free press would be highly injurious. decided to limit the issues to narrowly drawn ones. I see no
reason to disagree with the way the Court has resolved them.
The first issue on prematurity is moot. The second issue
FINALS CONSTITUTIONAL LAW I ACJUCO NOV 11, 2017 71

discusses a procedure now embodied in the recently


amended Rules of Court on how a Judge should proceed
before he issues a warrant of arrest. Anent the third issue,
considerations of public policy dictate that an incumbent
President should not be sued. At the same time, the President
cannot stand by helplessly bereft of legal remedies if
somebody vilifies or maligns him or her.

The Court has decided to defer the "chilling effect" issue for a
later day. To this, I take exception. I know that most of our
fiscals and judges are courageous individuals who would not
allow any considerations of possible consequences to their
careers to stand in the way of public duty. But why should we
subject them to this problem? And why should we allow the
possibility of the trial court treating and deciding the case as
one for ordinary libel without bothering to fully explore the
more important areas of concern, the extremely difficult issues
involving government power and freedom of expression.

However, since we have decided to defer the "chilling effect"


issue for a later day, I limit myself to reiterating the dissenting
words of Mr. Justice Jackson in the American case of
Beaurnhais v. Illinois (343 U. S. 250) when he said:

If one can claim to announce the judgment of legal history on


any subject, it is that criminal libel laws are consistent with the
concept of ordered liberty only when applied with safeguards
evolved to prevent their invasion of freedom of expression.

In the trial of the libel case against the petitioners, the


safeguards in the name of freedom of expression should be
faithfully applied.
FINALS CONSTITUTIONAL LAW I ACJUCO NOV 11, 2017 72

THE JUDICIAL DEPARTMENT Judicial Power and Jurisdiction


Under Article VIII, §1, the judicial power shall be vested in one
Supreme Court and in such lower courts as may be provided
JUDICIAL POWER- Judicial power rests with the Supreme by law. This power includes the duty to settle actual
Court and the lower courts, as established by law (Art. VIII, controversies involving rights that are legally demandable and
sec. 1 of the 1987 Constitution). Its duty is to settle actual enforceable and to determine if any branch or instrumentality
controversies involving rights which are legally demandable of government has acted with grave abuse of discretion
and enforceable (Art. VIII Sec. 1 (2). amounting to lack of excess of jurisdiction.
THE SUPREME COURT The Supreme Court has both original and appellate
jurisdiction. It exercises original jurisdiction (cases are directly
COMPOSITION filed with the SC in the first instance without passing through
The Supreme Court shall be composed of a Chief Justice and any of the lower courts) over cases affecting ambassadors,
fourteen Associate Justices. It may sit en banc or, in its other public ministers and consuls, and over petitions for
discretion, in divisions of three, five, or seven members. (Art. certiorari, prohibition, mandamus, quo warranto, and
VIII, §4) habeas corpus. (Art. VIII, §5(1)). It also has original
jurisdiction over writs of amparo, habeas data and the
APPOINTMENTS AND QUALIFICATIONS environmental writ of kalikasan. It exercises appellate
Its members shall be appointed by the President from a list of jurisdiction to review, revise, reverse, modify, or affirm final
at least three nominees prepared by the Judicial and Bar judgments, and orders of the lower courts in:
Council for every vacancy, without need of confirmation by the
Commission on Appointments. (Art.VIII, §9) (a) All cases in which the constitutionality or validity of any
treaty, international or executive agreement, law, presidential
Members of the Supreme Court are required to have proven decree, proclamation, order, instruction, ordinance, or
competence, integrity, probity and independence; they must regulation is in question.
be natural-born citizens of the Philippines, at least forty years
old, with at least fifteen years of experience as a judge of a (b) All cases involving the legality of any tax, impost,
lower court or law practice in the country. (Art. VIII, §7) assessment, or toll, or any penalty imposed in relation thereto.
Justices shall hold office during good behavior until they reach
the age of seventy years, or become incapacitated to (c) All cases in which the jurisdiction of any lower court is in
discharge the duties of office. (Art. VIII, §11) issue.
THE MEMBERS OF THE COURT (d) All criminal cases in which the penalty imposed is reclusion
The Chief Justice Hon. MARIA LOURDES P. A. SERENO perpetua or higher.
The Senior Associate Justice Hon. ANTONIO T. CARPIO
The Associate Justices Hon. PRESBITERO J. VELASCO, JR. (e) All cases in which only an error or question of law is
Hon. TERESITA J. LEONARDO-DE CASTRO involved. (Art. VIII, §5(1), (2))
Hon. ARTURO D. BRION
Hon. DIOSDADO M. PERALTA The Supreme Court has administrative supervision over all
Hon. LUCAS P. BERSAMIN courts and court personnel. (Article VIII, §6) It exercises this
Hon. MARIANO C. DEL CASTILLO power through the Office of the Court Administrator.
Hon. JOSE PORTUGAL PEREZ
Hon. JOSE CATRAL MENDOZA Rule-making Powers
Hon. BIENVENIDO L. REYES The Supreme Court has the exclusive power to promulgate
Hon. ESTELA M. PERLAS-BERNABE rules concerning the protection and enforcement of
Hon. MARVIC M. V. F. LEONEN constitutional rights, pleading, practice, and procedure in all
Hon. FRANCIS H. JARDELEZA courts, the admission to the practice of law, the integrated bar,
Hon. ALFREDO BENJAMIN S. CAGUIOA and legal assistance to the underprivileged. Any such rules
shall provide a simplified and inexpensive procedure for the
Administrative Officers speedy disposition of cases, shall be uniform for all courts of
Clerk of Court En Banc Atty. Felipa B. Anama the same grade, and shall not diminish, increase, or modify
Deputy Clerk of Court En Banc Atty. Anna-Li R. Papa-Gombio substantive rights. Rules of procedure of special courts and
First Division Clerk of Court Atty. Edgar O. Aricheta quasi-judicial bodies shall remain effective unless
Second Division Clerk of Court Atty. Ma. Lourdes C. Perfecto disapproved by the Supreme Court. (Art. VIII, §54(5))
Third Division Clerk of Court Atty. Wilfredo V. Lapitan

Office of Administrative Services


Atty. Eden T. Candelaria, Chief
Fiscal Management and Budget Office
Atty. Corazon G. Ferrer-Flores, Chief
Chief Attorney Atty. Maria Regina A. F. M. Ignacio, Acting
Chief
Office of the Reporter Atty. Edna B. Camba, Chief
Management Information Systems
Office
Atty. Carlos N. Garay, Acting Chief
Bar Confidant Atty. Ma. Cristina B. Layusa, Chief
Judicial Records Office Atty. Basilia T. Ringol, Chief
Library Services Mrs. Milagros S. Ong, Chief
Public Information Office Atty. Theodore O. Te, Chief
FINALS CONSTITUTIONAL LAW I ACJUCO NOV 11, 2017 73

NO NON-JUDCIAL WORK FOR JUDGES 1821 of the Civil Code. Citation of authority is hardly
necessary, except that it should be recalled that in the
Philippines, and in the United States for that matter, it has
G.R. No. L-37878 November 25, 1932 been held that a clause in a contract, providing that all matters
in dispute between the parties shall be referred to arbitrators
MANILA ELECTRIC COMPANY, petitioner, and to them alone, is contrary to public policy and cannot oust
vs. the courts of jurisdiction (Wahl and Wahl vs. Donaldson, Sims
PASAY TRANSPORTATION COMPANY, INC., ET AL., & Co. [1903], 2 Phil., 301; Puentebella vs. Negros Coal Co.
respondents. [1927], 50 Phil., 69; Vega vs. San Carlos Milling Co. [1924],
Ross, Lawrence & Selph for petitioner. 51 Phil., 908; District of Columbia vs. Bailey [1897], 171 U. S.,
Rivera & Francisco for respondent Pasay Transportation 161.)
Co.
P. A. Remigio for respondent E. B. Gutierrez. A. M. Zarate
for respondent Raymundo Transportation Co. We would not be understood as extending the principles
Vicente Ampil for respondent J. Ampil. governing arbitration and award too far. Unless the arbitration
agreement is such as absolutely to close the doors of the
courts against the parties, the courts should look with favor
upon such amicable arrangements. We can also perceive a
MALCOLM, J.: distinction between a private contract for submission to
arbitration and agreements to arbitrate falling within the terms
The preliminary and basic question presented by the petition of a statute enacted for such purpose and affecting others
of the Manila Electric Company, requesting the members of than the parties to a particular franchise. Here, however,
the Supreme Court, sitting as a board of arbitrators, to fix the whatever else may be said in extenuation, it remains true that
terms upon which certain transportation companies shall the decision of the board of arbitrators is made final, which if
be permitted to use the Pasig bridge of the Manila Electric literally enforced would leave a public utility, not a party to the
Company and the compensation to be paid to the Manila contract authorized by Act No. 1446, without recourse to the
Electric Company by such transportation companies, courts for a judicial determination of the question in dispute.
relates to the validity of section 11 of Act No. 1446 and to
the legal right of the members of the Supreme Court, Counsel for the petitioner rely principally on the case of
sitting as a board of arbitrators, to act on the petition. Act Tallassee Falls Mfg. Co. vs. Commissioner's Court [1908],
No. 1446 above referred to is entitled. "An Act granting a 158 Ala., 263. It was there held that an Act of a state
franchise to Charles M. Swift to construct, maintain, and legislature authorizing the commissioners' court of a certain
operate an electric railway, and to construct, maintain, and county to regulate and fix the rate of toll to be charged by the
operate an electric light, heat, and power system from a point owners of a bridge is not unconstitutional as delegating
in the City of Manila in an easterly direction to the town of legislative power to the courts. But that is not the question
Pasig, in the Province of Rizal." Section 11 of the Act provides: before us. Here the question is not one of whether or not there
"Whenever any franchise or right of way is granted to any has been a delegation of legislative authority to a court. More
other person or corporation, now or hereafter in existence, precisely, the issue concerns the legal right of the members of
over portions of the lines and tracks of the grantee herein, the the Supreme Court, sitting as a board of arbitrators the
terms on which said other person or corporation shall use decision of a majority of whom shall be final, to act in that
such right of way, and the compensation to be paid to the capacity.
grantee herein by such other person or corporation for said
use, shall be fixed by the members of the Supreme Court,
sitting as a board of arbitrators, the decision of a majority of We run counter to this dilemma. Either the members of the
whom shall be final." Supreme Court, sitting as a board of arbitrators, exercise
judicial functions, or the members of the Supreme Court,
sitting as board of arbitrators, exercise administrative or quasi
When the petition of the Manila Electric Company was filed in judicial functions. The first case would appear not to fall
this court, it was ordered that the petitioner be required to within the jurisdiction granted the Supreme Court. Even
serve copies on the Attorney-General and the transportation conceding that it does, it would presuppose the right to bring
companies affected by the petition. Thereafter, the Attorney- the matter in dispute before the courts, for any other
General disclaimed any interest in the proceedings, and construction would tend to oust the courts of jurisdiction and
opposition was entered to the petition by a number of public render the award a nullity. But if this be the proper
utility operators. On the submission of memoranda after an construction, we would then have the anomaly of a decision
oral hearing, the petition was made ready for resolution. by the members of the Supreme Court, sitting as a board of
arbitrators, taken therefrom to the courts and eventually
Examining the statutory provision which is here invoked, it is coming before the Supreme Court, where the Supreme Court
first noted that power is attempted to be granted to the would review the decision of its members acting as arbitrators.
members of the Supreme Court sitting as a board of
arbitrators and to the Supreme Court as an entity. It is next Or in the second case, if the functions performed by the
seen that the decision of a majority of the members of the members of the Supreme Court, sitting as a board of
Supreme Court is made final. And it is finally observed that the arbitrators, be considered as administrative or quasi judicial in
franchise granted the Manila Electric Company by the nature, that would result in the performance of duties which
Government of the Philippine Islands, although only a contract the members of the Supreme Court could not lawfully take it
between the parties to it, is now made to effect the rights of upon themselves to perform. The present petition also
persons not signatories to the covenant. furnishes an apt illustration of another anomaly, for we find the
Supreme Court as a court asked to determine if the members
The law calls for arbitration which represents a method of the of the court may be constituted a board of arbitrators, which is
parties' own choice. A submission to arbitration is a contract. not a court at all.lawphil.net
The parties to an arbitration agreement may not oust the
courts of jurisdiction of the matters submitted to arbitration. The Supreme Court of the Philippine Islands represents one
These are familiar rules which find support in articles 1820 and of the three divisions of power in our government. It is judicial
FINALS CONSTITUTIONAL LAW I ACJUCO NOV 11, 2017 74

power and judicial power only which is exercised by the


Supreme Court. Just as the Supreme Court, as the guardian
of constitutional rights, should not sanction usurpations by any
other department of the government, so should it as strictly
confine its own sphere of influence to the powers expressly or
by implication conferred on it by the Organic Act. The
Supreme Court and its members should not and cannot be
required to exercise any power or to perform any trust or to
assume any duty not pertaining to or connected with the
administering of judicial functions.

The Organic Act provides that the Supreme Court of the


Philippine Islands shall possess and exercise jurisdiction as
heretofore provided and such additional jurisdiction as shall
hereafter be prescribed by law (sec. 26). When the Organic
Act speaks of the exercise of "jurisdiction" by the Supreme
Court, it could not only mean the exercise of "jurisdiction" by
the Supreme Court acting as a court, and could hardly mean
the exercise of "jurisdiction" by the members of the Supreme
Court, sitting as a board of arbitrators. There is an important
distinction between the Supreme Court as an entity and the
members of the Supreme Court. A board of arbitrators is not
a "court" in any proper sense of the term, and possesses none
of the jurisdiction which the Organic Act contemplates shall be
exercised by the Supreme Court.lawph!l.net

In the last judicial paper from the pen of Chief Justice Taney,
it was said:

The power conferred on this court is exclusively judicial, and


it cannot be required or authorized to exercise any other. . . .
Its jurisdiction and powers and duties being defined in the
organic law of the government, and being all strictly judicial,
Congress cannot require or authorize the court to exercise any
other jurisdiction or power, or perform any other duty. . . . The
award of execution is a part, and an essential part of every
judgment passed by a court exercising judicial power. It is no
judgment, in the legal sense of the term, without it. Without
such an award the judgment would be inoperative and
nugatory, leaving the aggrieved party without a remedy. It
would be merely an opinion, which would remain a dead letter,
and without any operation upon the rights of the parties,
unless Congress should at some future time sanction it, and
pass a law authorizing the court to carry its opinion into effect.
Such is not the judicial power confided to this court, in the
exercise of its appellate jurisdiction; yet it is the whole power
that the court is allowed to exercise under this act of
Congress. . . . And while it executes firmly all the judicial
powers entrusted to it, the court will carefully abstain from
exercising any power that is not strictly judicial in its character,
and which is not clearly confided to it by the Constitution. . . .
(Gordon vs. United States [1864], 2 Wall., 561; 117 U. S., 697
Appendix.)

Confirming the decision to the basic question at issue, the


Supreme Court holds that section 11 of Act No. 1446
contravenes the maxims which guide the operation of a
democratic government constitutionally established, and that
it would be improper and illegal for the members of the
Supreme Court, sitting as a board of arbitrators, the decision
of a majority of whom shall be final, to act on the petition of
the Manila Electric Company. As a result, the members of the
Supreme Court decline to proceed further in the matter.
FINALS CONSTITUTIONAL LAW I ACJUCO NOV 11, 2017 75

A.M. No. 198-J May 31, 1971 6. That by his deliberate violation of his Oath of Office as a
PAZ M. GARCIA, complainant, District Judge of the Court of First Instance of Laguna and San
vs. Pablo, Branch VI he has manifested such moral bankruptcy
HON. CATALINO MACARAIG, JR., respondent. as to deny his fitness to perform or discharge official duties in
the administration of justice.
RESOLUTION
7. That on June 29, 1970, respondent Judge wrote to the
BARREDO, J.: Honorable Secretary of Justice informing him that he was
entering upon the performance of his duties, which letter of his
reads in full:
Administrative complaint filed by one Paz M. Garcia against
the Honorable Catalino Macaraig, Jr., formerly Judge of the
Court of First Instance of Laguna, Branch VI, now 'I have the honor to inform you that I am entering upon the
Undersecretary of Justice, in his former capacity as judge, for performance of the duties of the office of Judge of the Court
alleged "dishonesty, violation of his oath of office as judge ... of First Instance of Laguna and San Pablo City (Branch VI)
gross incompetence, violation of Republic Act 296 or the today, June 29, 1970.'
Judiciary Act of 1948, as amended, (particularly) Sections 5,
55 and 58 thereof, committed (allegedly) as follows: That such actuation of deliberately telling a deliberate
falsehood aggravates his moral bankruptcy incompatible to
2. That from July 1, 1970 up to February 28, 1971 inclusive, the requirements of the highest degree of honesty, integrity
as such incumbent Judge, respondent herein, has not and good moral character appertaining to holding the position
submitted his monthly reports containing the number of of Judge in the administration of justice.
cases filed, disposed of, decided and/or resolved, the
number of cases pending decisions for one month, two Upon being so required, in due time, respondent filed an
months to over three months, together with the title, answer alleging pertinently that:
number, number of hours of court session held a day,
etc., as evidenced by the certificate issued by Hon. Eulalio D. THE FACTS
Pichay, Judicial Superintendent, Dept. of Justice, copy of
which is hereto attached as Annex "A", Item No. 1, in violation
of Circular No. 10 of the Dept. of Justice dated February 6, Respondent took his oath as Judge of the Court of First
1952, copy of which is hereto attached as Annex "B"; Instance of Laguna and San Pablo City with station at
Calamba on June 29, 1970. The court, being one of the 112
newly created CFI branches, had to be organized from
3. That he has not submitted his certificate of service (New scratch. After consultations with the officials of the province of
Judicial Form No. 86, Revised 1966) from July to December, Laguna, the municipality of Calamba and the Department of
1970 and from January to February, 1971 inclusive as Justice, respondent decided to accept the offer of the
evidenced by the certificate issued by Judge Pichay, Judicial Calamba Municipal Government to supply the space for the
Superintendent, Dept. of Justice Annex "A", Item No. 2 courtroom and offices of the court; to utilize the financial
thereof; assistance promised by the Laguna provincial government for
the purchase of the necessary supplies and materials; and to
4. That as incumbent Judge of Branch VI, Court of First rely on the national government for the equipment needed by
Instance of Laguna and San Pablo and knowing fully well that the court (Under Section 190 of the Revised Administrative
he has never performed his official duties or discharged the Code, all these items must be furnished by the provincial
duties appertaining to his office, he has collected and was paid government. The provincial officials of Laguna, however,
his salaries from July to December, 1970 and from January to informed the respondent that the province was not in a
February 1971 as evidenced by the certificate issued by the position to do so).
cashier Mrs. Santos of the Department of Justice hereto
attached as Annex "C" and the certificate of Mr. Pichay Annex As to the space requirements of the court, the Municipal
"A", last paragraph thereof, aggravated by his repeated failure Mayor of Calamba assured the respondent that the court
to submit the certificate of service in flagrant violation of action could be accommodated in the west wing of the Calamba
5 of the Judiciary Act of 1948 as amended which provides as municipal building as soon as the office of the municipal
follows: treasurer and his personnel are transferred to another
location. When the projected transfer of the municipal
... District judges, judges of City Courts, and municipal Judges treasurer's office was about to be effected, the treasurer and
shall certify on their application for leave, and upon salary several municipal councilors objected. The municipal mayor
vouchers presented by them for payment, or upon the payrolls then requested the respondent to look over some of the office
upon which their salaries are paid, that all special spaces for rent in Calamba, with the commitment that the
proceedings, applications, petitions, motions, and all civil and municipal government will shoulder the payment of the
criminal cases which have been under submission for decision rentals. Respondent's first choice was the second floor of the
or determination for a period of ninety days or more have been Republic Bank branch in Calamba, but the negotiations failed
determined and decided on or before the date of making the when the owner of the building refused to reduce the rent to
certificate and ... no salary shall be paid without such P300 a month. The next suitable space selected by
certificate' (Emphasis supplied). respondent was the second floor of the Laguna Development
Bank. After a month's negotiations, the municipality finally
5. That his deliberate failure to submit the monthly reports signed a lease agreement with the owner on October 26,
from July to December, 1970 and from January, 1971 to 1970. Another month passed before the municipal
February, 1971 stating therein the number of hours of session government could release the amount necessary for the
that the Court holds daily, the accomplishments of the Court improvements to convert the space that was rented, which
constitutes a clear violation of Sections 55 and 58 of the was a big hall without partitions, into a courtroom and offices
Judiciary Act of 1948, as amended. for the personnel of the court and for the assistant provincial
fiscal. Thereafter, upon respondent's representations, the
FINALS CONSTITUTIONAL LAW I ACJUCO NOV 11, 2017 76

provincial government appropriated the amount of P5,000 for circular (please see enclosed certification of Judge Eulalio D.
the purchase of the supplies and materials needed by the Pichay, Judicial Superintendent, marked Annex 'C').
court. Early in December, 1970 respondent also placed his
order for the necessary equipment with the Property Officer of "Moreover, a reading of these sections and circular makes
the Department of Justice but, unfortunately, the appropriation evident the folly of requiring a judge who has not entered into
for the equipment of courts of first instance was released only the Performance of his judicial duties to comply with them.
on December 23, 1970 and the procurement of the equipment Taking Section 5, how could a judge who has not started to
chargeable against this allotment is still under way (please see discharge his judicial duties certify that 'all special
enclosed certification of the Financial Officer of the proceedings, applications, petitions, motions, and all civil and
Department of Justice marked Annex "A"). criminal cases, which have been under submission for
decision or determination for a period of ninety days or more
"When respondent realized that it would be sometime before have been determined and decided on or before the date of
he could actually preside over his court, he applied for an making the certificate.' And bow could such a judge hold court
extended leave (during the 16 years he had worked in the in his place of permanent station as required by Section 55;
Department of Justice, respondent had, due to pressure of observe the hours of daily sessions of the court as prescribed
duties, never gone on extended leave, resulting in his by Section 58; and render the reports required by Circular No.
forfeiting all the leave benefits he had earned beyond the 10 when his court is not yet in physical existence Clearly,
maximum ten months allowed by the law). The Secretary of therefore, Sections 5, 55 and 58 of the Judiciary Act and
Justice, however, prevailed upon respondent to forego his Circular No. 10 cannot apply to such a judge." .
leave and instead to assist him, without being extended a
formal detail, whenever respondent was not busy attending to In view of the nature of the allegations of complainant and
the needs of his court. respondent in their respective complaint and answer and
considering, in the light thereof, that the material facts are
"Charges Have No Basis -- . more or less undisputed, the Court feels that this case can be
disposed of without any further proceeding.
"Complainant has charged respondent with dishonesty,
violation of his oath of office, grave incompetence and After mature study and deliberation, the Court is convinced
violation of Sections 5, 55 and 58 of the Judiciary Act. that the complaint must be dismissed. To begin with, We
cannot discern any tinge of dishonesty in the actuations for
"It is respectfully submitted that -- . the respondent complained of. As We see it, the situation is
not exactly as complainant has attempted to portray it.
Complainant's theory is that respondent collected or received
"A. Respondent's inability to perform his judicial duties under salaries as judge when in fact he has never acted as such,
the circumstances mentioned above does not constitute since the date he took his oath up to the filing of the complaint.
incompetence. Respondent was like every lawyer who gets In the sense that respondent has not yet performed any
his first appointment to the bench, eager to assume his judicial judicial function, it may be admitted that respondent has not
duties and rid himself of the stigma of being 'a judge without a really performed the duties of judge. What is lost sight of,
sala', but forces and circumstances beyond his control however, is that after taking his oath and formally assuming
prevented him from discharging his judicial duties. this position as judge, respondent had a perfect right to earn
the salary of a judge even in the extreme supposition that he
"B. Respondent's collection of salaries as judge does not did not perform any judicial function for he could, while
constitute dishonesty because aside from the time, effort and preparing himself for his new job or for any good reason, take
money he spent in organizing the CFI at Calamba, he worked a leave, as in fact, he had planned to do, were it not for the
in the Department of Justice (please see enclosed certification request of the Secretary of Justice for him to forego the idea
of Undersecretary of Justice Guillermo S. Santos marked and, instead, help the Department in whatever way possible
Annex 'B'). Indeed, even if respondent did no more than exert which would not, it must be presumed, impair his position as
efforts to organize his court, he could, as other judges have a judge. This is more so, when, as in this case, the
done, have collected his salaries as judge without being guilty government officials or officers in duty bound to furnish him
of dishonesty. the necessary place and facilities for his court and the
performance of his functions have failed to provide him
"Incidentally, when respondent took his oath as CFI judge therewith without any fault on his part. That respondent took it
which position then carried a salary of P19,000 per annum, he upon himself to personally work for early action on the part of
automatically ceased to be Chief of the Technical Staff of the the corresponding officials in this direction and, in his spare
Department of Justice and Member of the Board of Pardons time, made himself available to the Department of Justice to
and Parole, positions from which he was receiving P16,200 assist the Secretary, what with his vast experience, having
and P8,000 per annum, respectively. Also, in anticipation of worked therein for sixteen years, is, far from being dishonesty,
the judicial duties which he was about to assume, respondent to his credit. In the circumstances, it was certainly not
took a leave of absence from his professorial lecturer's duties improper that he rendered some kind of service to the
in the U.P. College of Law where he was receiving government, since he was receiving salaries, while being
approximately P600 a month. unable to perform his regular duties as judge without any fault
on, his part. As to whether or not in doing so he, placed in
jeopardy the independence of the judiciary and failed to act
"C. Sections 5, 55 and 58 of the Judiciary Act and Circular No. according to the correct norm of conduct which a judge should
10 dated February 6, 1952 of the Department of Justice are observe vis-a-vis service to the other departments of the
not applicable to a Judge not actually discharging his judicial government will be discussed a non. At this juncture, the only
duties. point We settle is that complainant's theory of dishonesty
cannot hold water.
"The Department of Justice has never required judges who
have not actually started, to perform their judicial duties to Admittedly respondent has not prepared and submitted any of
comply with the abovementioned statutory-provisions and the reports of accomplishments and status of cases in his sala
FINALS CONSTITUTIONAL LAW I ACJUCO NOV 11, 2017 77

which are usually required of judges under existing laws as


well as the corresponding circulars of the Department of
Justice. The reason is simple. He has not yet started
performing any judicial functions. None of those laws and
circulars apply to him for all of them contemplate judges who
are actually holding trials and hearings and making decisions
and others. On the other hand, respondent Could not be
blamed for taking his oath as he did, for he had a valid
confirmed appointment in his favor. In other words, he simply
made himself available for the purpose for which he was
appointed. That he could not actually hold office in the court
to which he was appointed was not of his making. The other
officials in charge of providing him therewith seem to have
been caught unprepared and have not had enough time to
have it read. Conceivably, under the law, with the permission
of this Court, respondent could have been assigned to another
court pending all these preparations, but that is something
within the initiative control of the Secretary of Justice and nor
of the respondent.

Of course, none of these is to be taken as meaning that this


Court looks with favor at the practice of long standing to be
sure, of judges being detailed in the Department of Justice
to assist the Secretary even if it were only in connection
with his work of exercising administrative authority over
the courts. The line between what a judge may do and
what he may not do in collaborating or working with other
offices or officers under the other great departments of
the government must always be kept clear and jealously
observed, least the principle of separation of powers on
which our government rests by mandate of the people
thru the Constitution be gradually eroded by practices
purportedly motivated by good intentions in the interest
of the public service. The fundamental advantages and the
necessity of the independence of said three departments from
each other, limited only by the specific constitutional precepts
a check and balance between and among them, have long
been acknowledged as more paramount than the serving of
any temporary or passing governmental conveniences or
exigencies. It is thus of grave importance to the judiciary
under our present constitutional scheme of government
that no judge or even the lowest court in this Republic
should place himself in a position where his actuations
on matters submitted to him for action or resolution
would be subject to review and prior approval and, worst
still, reversal, before they can have legal effect, by any
authority other than the Court of Appeals or this Supreme
Court, as the case may be. Needless to say, this Court feels
very strongly that, it is best that this practice is discontinued.

WHEREFORE, the herein administrative complaint is hereby


dismissed. Let a copy of this resolution be furnished the
Secretary of Justice.

Concepcion, Reyes, J.B.L., Dizon, Makalintal, Zaldivar and


Villamor, JJ., concur.

Castro and Teekankee, JJ., took no part.

Fernando, J., concurs fully and in addition submits a brief


separate opinion. Makasiar, J., concurs with the opinion Mr.
Justice Fernando.
FINALS CONSTITUTIONAL LAW I ACJUCO NOV 11, 2017 78

SALARY authorized the continuation of the deduction of the withholding


tax from the salaries of the members of the Supreme Court,
as well as from the salaries of all other members of the
G.R. No. 78780 July 23, 1987 Judiciary. The Court hereby makes of record that it had then
DAVID G. NITAFAN, WENCESLAO M. POLO, and MAXIMO discarded the ruling in Perfecto vs. Meer and Endencia vs.
A. SAVELLANO, JR., petitioners, David, infra, that declared the salaries of members of the
vs. Judiciary exempt from payment of the income tax and
COMMISSIONER OF INTERNAL REVENUE and THE considered such payment as a diminution of their salaries
FINANCIAL OFFICER, SUPREME COURT OF THE during their continuance in office. The Court hereby reiterates
PHILIPPINES, respondents. that the salaries of Justices and Judges are properly subject
to a general income tax law applicable to all income earners
RESOLUTION and that the payment of such income tax by Justices and
Judges does not fall within the constitutional protection
MELENCIO-HERRERA, J.: against decrease of their salaries during their continuance in
office.

Petitioners, the duly appointed and qualified Judges presiding


over Branches 52, 19 and 53, respectively, of the Regional A comparison of the Constitutional provisions involved is
Trial Court, National Capital Judicial Region, all with stations called for. The 1935 Constitution provided:
in Manila, seek to prohibit and/or perpetually enjoin
respondents, the Commissioner of Internal Revenue and ... (The members of the Supreme Court and all judges of
the Financial Officer of the Supreme Court, from making inferior courts) shall receive such compensation as may be
any deduction of withholding taxes from their salaries. fixed by law, which shall not be diminished during their
continuance in office ... 1 (Emphasis supplied).
In a nutshell, they submit that "any tax withheld from their
emoluments or compensation as judicial officers constitutes a Under the 1973 Constitution, the same provision read:
decrease or diminution of their salaries, contrary to the
provision of Section 10, Article VIII of the 1987 Constitution The salary of the Chief Justice and of the Associate Justices
mandating that "(d)uring their continuance in office, their of the Supreme court, and of judges of inferior courts shall be
salary shall not be decreased," even as it is anathema to the fixed by law, which shall not be decreased during their
Ideal of an independent judiciary envisioned in and by said continuance in office. ... 2 (Emphasis ours).
Constitution."
And in respect of income tax exemption, another provision in
It may be pointed out that, early on, the Court had dealt with the same 1973 Constitution specifically stipulated:
the matter administratively in response to representations that
the Court direct its Finance Officer to discontinue the
withholding of taxes from salaries of members of the Bench. No salary or any form of emolument of any public officer
Thus, on June 4, 1987, the Court en banc had reaffirmed the or employee, including constitutional officers, shall be
Chief Justice's directive as follows: exempt from payment of income tax. 3

RE: Question of exemption from income taxation. — The The provision in the 1987 Constitution, which petitioners rely
Court REAFFIRMED the Chief Justice's previous and on, reads:
standing directive to the Fiscal Management and Budget
Office of this Court to continue with the deduction of the The salary of the Chief Justice and of the Associate Justices
withholding taxes from the salaries of the Justices of the of the Supreme Court, and of judges of lower courts shall be
Supreme Court as well as from the salaries of all other fixed by law. During their continuance in office, their salary
members of the judiciary. shall not be decreased. 4(Emphasis supplied).

That should have resolved the question. However, with the The 1987 Constitution does not contain a provision similar to
filing of this petition, the Court has deemed it best to settle the Section 6, Article XV of the 1973 Constitution, for which
legal issue raised through this judicial pronouncement. As will reason, petitioners claim that the intent of the framers is to
be shown hereinafter, the clear intent of the Constitutional revert to the original concept of "non-diminution "of salaries of
Commission was to delete the proposed express grant of judicial officers.
exemption from payment of income tax to members of the
Judiciary, so as to "give substance to equality among the three The deliberations of the 1986 Constitutional Commission
branches of Government" in the words of Commissioner relevant to Section 10, Article VIII, negate such contention.
Rigos. In the course of the deliberations, it was further
expressly made clear, specially with regard to Commissioner
Joaquin F. Bernas' accepted amendment to the amendment The draft proposal of Section 10, Article VIII, of the 1987
of Commissioner Rigos, that the salaries of members of the Constitution read:
Judiciary would be subject to the general income tax applied
to all taxpayers. Section 13. The salary of the Chief Justice and the Associate
Justices of the Supreme Court and of judges of the lower
This intent was somehow and inadvertently not clearly set courts shall be fixed by law. During their continuance in office,
forth in the final text of the Constitution as approved and their salary shall not be diminished nor subjected to income
ratified in February, 1987 (infra, pp. 7-8). Although the intent tax. Until the National Assembly shall provide otherwise, the
may have been obscured by the failure to include in the Chief Justice shall receive an annual salary of
General Provisions a proscription against exemption of any _____________ and each Associate Justice
public officer or employee, including constitutional officers, ______________ pesos. 5(Emphasis ours)
from payment of income tax, the Court since then has
FINALS CONSTITUTIONAL LAW I ACJUCO NOV 11, 2017 79

During the debates on the draft Article (Committee Report No. excludes them from income tax, but rather I would propose
18), two Commissioners presented their objections to the that the statement will read: "During their continuance in
provision on tax exemption, thus: office, their salary shall not be diminished BUT MAY BE
SUBJECT TO GENERAL INCOME TAX."IN support of this
MS. AQUINO. Finally, on the matter of exemption from tax of position, I would say that the argument seems to be that the
the salary of justices, does this not violate the principle of the justice and judges should not be subjected to income tax
uniformity of taxation and the principle of equal protection of because they already gave up the income from their practice.
the law? After all, tax is levied not on the salary but on the That is true also of Cabinet members and all other employees.
combined income, such that when the judge receives a salary And I know right now, for instance, there are many people who
and it is comingled with the other income, we tax the income, have accepted employment in the government involving a
not the salary. Why do we have to give special privileges to reduction of income and yet are still subject to income tax. So,
the salary of justices? they are not the only citizens whose income is reduced by
accepting service in government.
MR. CONCEPCION. It is the independence of the judiciary.
We prohibit the increase or decrease of their salary during Commissioner Rigos accepted the proposed amendment to
their term. This is an indirect way of decreasing their salary the amendment. Commissioner Rustico F. de los Reyes, Jr.
and affecting the independence of the judges. then moved for a suspension of the session. Upon
resumption, Commissioner Bernas announced:
MS. AQUINO. I appreciate that to be in the nature of a clause
to respect tenure, but the special privilege on taxation might, During the suspension, we came to an understanding with the
in effect, be a violation of the principle of uniformity in taxation original proponent, Commissioner Rigos, that his amendment
and the equal protection clause. 6 on page 6,. line 4 would read: "During their continuance in
office, their salary shall not be DECREASED."But this is on
the understanding that there will be a provision in the
xxx xxx xxx Constitution similar to Section 6 of Article XV, the General
Provisions of the 1973 Constitution, which says:
MR. OPLE. x x x
No salary or any form of emolument of any public officer
Of course, we share deeply the concern expressed by the or employee, including constitutional officers, shall be
sponsor, Commissioner Roberto Concepcion, for whom we exempt from payment of income tax.
have the highest respect, to surround the Supreme Court and
the judicial system as a whole with the whole armor of defense So, we put a period (.) after "DECREASED" on the
against the executive and legislative invasion of their understanding that the salary of justices is subject to tax.
independence. But in so doing, some of the citizens outside,
especially the humble government employees, might say that
in trying to erect a bastion of justice, we might end up with the When queried about the specific Article in the General
fortress of privileges, an island of extra territoriality under the Provisions on non-exemption from tax of salaries of public
Republic of the Philippines, because a good number of officers, Commissioner Bernas replied:
powers and rights accorded to the Judiciary here may not be
enjoyed in the remotest degree by other employees of the FR BERNAS. Yes, I do not know if such an article will be found
government. in the General Provisions. But at any rate, when we put a
period (.) after "DECREASED," it is on the understanding that
An example is the exception from income tax, which is a kind the doctrine in Perfecto vs. Meer and Dencia vs. David will not
of economic immunity, which is, of course, denied to the entire apply anymore.
executive department and the legislative. 7
The amendment to the original draft, as discussed and
And during the period of amendments on the draft Article, on understood, was finally approved without objection.
July 14, 1986, Commissioner Cirilo A. Rigos proposed that the
term "diminished" be changed to "decreased" and that the THE PRESIDING OFFICER (Mr. Bengzon). The
words "nor subjected to income tax" be deleted so as to "give understanding, therefore, is that there will be a provision under
substance to equality among the three branches in the the Article on General Provisions. Could Commissioner
government. Rosario Braid kindly take note that the salaries of officials of
the government including constitutional officers shall not be
Commissioner Florenz D. Regalado, on behalf of the exempt from income tax? The amendment proposed herein
Committee on the Judiciary, defended the original draft and and accepted by the Committee now reads as follows: "During
referred to the ruling of this Court in Perfecto vs. Meer 8 that their continuance in office, their salary shall not be
"the independence of the judges is of far greater importance DECREASED"; and the phrase "nor subjected to income tax"
than any revenue that could come from taxing their salaries." is deleted.9
Commissioner Rigos then moved that the matter be put to a
vote. Commissioner Joaquin G. Bernas stood up "in support The debates, interpellations and opinions expressed
of an amendment to the amendment with the request for a regarding the constitutional provision in question until it was
modification of the amendment," as follows: finally approved by the Commission disclosed that the true
intent of the framers of the 1987 Constitution, in adopting it,
FR. BERNAS. Yes. I am going to propose an amendment to was to make the salaries of members of the Judiciary taxable.
the amendment saying that it is not enough to drop the phrase The ascertainment of that intent is but in keeping with the
"shall not be subjected to income tax," because if that is all fundamental principle of constitutional construction that the
that the Gentleman will do, then he will just fall back on the intent of the framers of the organic law and of the people
decision in Perfecto vs. Meer and in Dencia vs. David [should adopting it should be given effect.10 The primary task in
be Endencia and Jugo vs. David, etc., 93 Phil. 696[ which constitutional construction is to ascertain and thereafter
assure the realization of the purpose of the framers and of the
FINALS CONSTITUTIONAL LAW I ACJUCO NOV 11, 2017 80

people in the adoption of the Constitution.11it may also be


safely assumed that the people in ratifying the Constitution
were guided mainly by the explanation offered by the
framers.12 1avvphi1

Besides, construing Section 10, Articles VIII, of the 1987


Constitution, which, for clarity, is again reproduced hereunder:

The salary of the Chief Justice and of the Associate Justices


of the Supreme Court, and of judges of lower courts shall be
fixed by law. During their continuance in office, their salary
shall not be decreased. (Emphasis supplied).

it is plain that the Constitution authorizes Congress to pass a


law fixing another rate of compensation of Justices and
Judges but such rate must be higher than that which they are
receiving at the time of enactment, or if lower, it would be
applicable only to those appointed after its approval. It would
be a strained construction to read into the provision an
exemption from taxation in the light of the discussion in the
Constitutional Commission.

With the foregoing interpretation, and as stated heretofore, the


ruling that "the imposition of income tax upon the salary of
judges is a dimunition thereof, and so violates the
Constitution" in Perfecto vs. Meer,13 as affirmed in Endencia
vs. David 14 must be declared discarded. The framers of the
fundamental law, as the alter ego of the people, have
expressed in clear and unmistakable terms the meaning and
import of Section 10, Article VIII, of the 1987 Constitution that
they have adopted Stated otherwise, we accord due respect
to the intent of the people, through the discussions and
deliberations of their representatives, in the spirit that all
citizens should bear their aliquot part of the cost of maintaining
the government and should share the burden of general
income taxation equitably.

WHEREFORE, the instant petition for Prohibition is


hereby dismissed.

Teehankee, C.J., Fernan, Narvasa, Gutierrez, Jr., Cruz,


Paras, Feliciano, Gancayco, Padilla, Bidin, Sarmiento and
Cortes, JJ., concur.

Yap, J., is on leave.


FINALS CONSTITUTIONAL LAW I ACJUCO NOV 11, 2017 81

A.M. No. 88-4-5433 April 15, 1988 removal from office, or any penalty service of which would
amount to removal from office.
IN RE FIRST INDORSEMET FROM HONORABLE RAUL M.
GONZALEZ DATED 16 MARCH 1988 REQUESTING The Court dealt with this matter in its Resolution of 17
HONORABLE JUSTICE MARCELO B. FERNAN TO February 1988 in Administrative Case No. 3135 in the
COMMENT ON AN ANONYMOUS LETTER-COMPLAINT. following terms:

RESOLUTION There is another reason why the complaining for disbarment


here must be dismissed. Members of the Supreme Court
PER CURIAM: must, under Article VIII (7) (1) of the Constitution, be
members of the Philippine Bar and may be removed from
office only by impeachment (Article XI [2], Constitution).
The Court CONSIDERED the 1st Indorsement dated 16 To grant a complaint for disbarment of a Member of the
March 1988 from Mr. Raul M. Gonzalez, Court during the Member's incumbency, would in effect
"Tanodbayan/Special; Prosecutor" forwarding to Mr. Justice be to circumbent and hence to run afoul of the
Marcelo B. Fernan a "letter-complaint, dated 14 December constitutional mandate theat Members of the Court may
1987 with enclosure of the Concerned Employees of the be removed from office only by impeachment for and
Supreme Court," together with a telegram of Miguel Cuenco, conviction of certain offenses listed in Article XI (2) of the
for "comment within ten (10) days from receipt hereof." Mr. Constitution. Precisely the same situation exists in respect of
Justice Fernan had brought this 1st Indorsement to the the Ombudsman and his deputies (Article XI [8] in relation to
attention of the Court en banc in view of the important Article XI [2], Id.), a majority of the members of the
implications of policy raised by said 1st Indorsement. Commission on Elections (Article IX [C] [1] [1] in relation to
Article XI [2], Id. and the members of the Commission on Audit
The mentioned 1st Indorsement has two (2) attachments. who are not certified public accountants (Article XI [D] [1][1],
First, an anonymous letter by "Concerned Employees of the Id.), all of whom are constitutionally required to be members
Supreme Court" addressed to Hon. Raul M. Gonzalez of the Philippine Bar. (Emphasis supplied)
referring to charges for disbarment brought by Mr. Miguel
Cuenco against Mr. Justice Marcelo B. Fernan and asking Mr. This is not the first time the Court has had occasion to rule on
Gonzalez "to do something about this." The second this matter. In Lecaroz v. Sandiganbayan, 1 the Court said:
attachment is a copy of a telegram from Mr. Miguel Cuenco
addressed to Hon. Raul M. Gonzalez, where Mr. Cuenco
refers to pleadings he apparently filed on 29 February 1988 The broad power of the New Constitution vests the
with the Supreme Court in Administrative Case No. 3135, respondent court with jurisdiction over "public officers and
which, in the opinion of Mr. Cuenco, made improper any employees, including those in government-owned or
"intervention" by Mr. Raul Gonzalez. Mr. Cuenco, controlled corporations." There are exceptions, however, like
nonetheless, encourages Mr. Gonzalez "to file responsive constitutional officers, particularly those declared to be
pleading Supreme Court en banc to comply with Petition removed by impeachment. Section 2, Article XIII of the 1973
Concerned Employees Supreme Court asking Tanodbayan's Constitution provides:
intervention.
Sec. 2 The President, the Members of the Supreme Court,
The Court DIRECTED the Clerk of Court to FURNISH Mr. and the Members of the Constitutional Commissions
Raul M Gonzales a copy of the per curiam Resolution, shall be removed from office on impeachment for, and
dated 17 February 1988 of the Court in Administrative Case conviction of, culpable violation of the Constitution,
No. 3135 entitled "Miguel Cuenco v. Honorable Marcelo B. treason, bribery, other high crimes, or graft and
Fernan" in which Resolution, the Court Resolved to dismiss corruption."
the charges made by complaint Cuenco against Mr.Justice
Fernan for utter lack of merit. In the same Resolution, the Thus, the above provision proscribes removal from office of
Court Resolved to require complainant Cuenco to show cause the aforementioned constitutional officers by any other
why he should not be administratively dealt with for making method; otherwise, to allow a public officer who may be
unfounded serious accusations against Mr. Justice Fernan. removed solely by impeachment to be charged criminally
Upon request of Mr. Cueco, the Court had granted him an while holding his office, would be violative of the clear
extension of up to 30 March 1988, Mr. Cuenco filed a pleading mandate of the fundamental law.
which appears to be an omnibus pleading relating to, inter alia,
Administrative Case No. 3135. Insofar as Administrative Case Chief Justice Enrique M. Fernando, in his authoritative
No. 3135 is concerned, the Court treated this pleading as a dissertation on the New Constitution, states that "judgement
Motion for Reconsideration. By a per curiam Resolution dated in cases of impeachment shall be limited to removal from
15 April 1988, the Court denied with finality Mr Cuenco's office and disqualification to hold any office of honor, trust, or
Motion for Reconsideration. profit under the Republic of the Philippines, but the party
convicted shall nevertheless be liable and subject to
It is important to underscore the rule of constitution law here prosecution trial, and punishment, in accordance with law. The
involved. This principle may be succinctly formulated in the above provision is a reproduction of what was found in the
following terms. A public officer who under the Constitution is 1935 Constitution. It is quite apparent from the explicit
required to be a Member of the Philippine Bar as a character of the above provision that the effect of
qualification for the office held by him and who may be impeachment is limited to the loss of position and
removed from office only by impeachment, cannot be charged disqualification to hold any office of honor, trust or profit under
with disbarment during the incumbency of such public officer. the Republic. It is equally manifest that the party this convicted
Further, such public officer, during his incumbency, cannot be may be proceeded against, tried and thereafter punished in
charged criminally before the Sandiganbayan or any other accordance with law. There can be no clearer expression of
court with any offence which carries with it the penalty of the constitutional intent as to the scope of the impeachment
process (The Constitution f the Philippines, pp. 465-466)." The
FINALS CONSTITUTIONAL LAW I ACJUCO NOV 11, 2017 82

clear implication is, the party convicted in the impeachment


proceeding shall nevertheless be liable and subject of
prosecution, trial and punishment according to law; and
that if the same does not result in a conviction and the
official is not thereby removed, the filing of a criminal
action "in accordance with law" may not prosper. 2

The provisions of the 1973 Constitution we referred to above


in Lecaroz v. Sandiganbayan are substantially reproduced in
Article XI of the 1987 Constitution:

Sec. 2 The President, the Vice-President, the Members of the


Supreme Court, the Members of the Constitutional
Commissions, and the Ombudsman may be removed from
office, on impeachment for, and conviction of, culpable
violation of the Constitution, treason, bribery, graft and
corruption, other high crimes, or betrayal of public trust. All
other public officers and employees may be removed from
office as provided by law, but not by impeachment.

Sec. 3 xxx xxx xxx

(7) Judgment in cases of impeachment shall not extend further


than removal from office and disqualification to hold any office
under the Republic of the Philippines, but the party convicted
shall nevertheless be liable and subject to prosecution, trial
and punishment according to law.

It is important to make clear that the Court is not here saying


that it Members or the other constitutional officers we referred
to above are entitled to immunity from liability for possibly
criminal acts or for alleged violation of the Canons of Judicial
Ethics or other supposed misbehavior. What the Court is
saying is that there is a fundamental procedural
requirements that must be observed before such liability
may be determined and enforced. A Member of the
Supreme Court must first be removed from office via the
constitutional route of impeachment under Sections 2
and 3 of Article XI of the 1987 Constitution. Should the
tenure of the Supreme Court Justice be thus terminated
by impeachment, he may then be held to answer either
criminally or administratively (by disbarment
proceedings) for any wrong or misbehavior that may be
proven against him in appropriate proceedings.

The above rule rests on the fundamental principles of judicial


independence and separation of powers. The rule is
important because judicial independence is important.
Without the protection of this rule, Members of the
Supreme Court would be brought against them by
unsuccessful litigants or their lawyers or by other parties
who, for any number of reasons might seek to affect the
exercise of judicial authority by the Court.

It follows from the foregoing that a fiscal or other prosecuting


officer should forthwith and motu proprio dismiss any charges
brought against a Member of this Court. The remedy of a
person with a legitimate grievance is to file impeachment
proceedings.

The Clerk of Court is hereby DIRECTED to serve a copy of


this Resolution upon Hon. Raul M. Gonzales and Mr Miguel
Cuenco.
FINALS CONSTITUTIONAL LAW I ACJUCO NOV 11, 2017 83

JURISDICTION constituted as the same was composed of all the Grade VI


teachers only, in violation of the Service Manual for Teachers
G.R. No. L-25024 March 30, 1970 of the Bureau of Public Schools which provides that the
committee to select the honor students should be composed
TEODORO C. SANTIAGO, JR. Minor, Represented by his of all teachers in Grades V and VI; that there are direct and
Mother, Mrs. Angelita C. Santiago, petitioner-appellant, circumstantial matters, which shall be proven during the trial,
vs. wherein respondents have exercised grave abuse of
MISS JUANITA BAUTISTA, ROSALINDA ALPAS, discretion and irregularities, such as the changing of the final
REBECCA MATUGAS, MILKITA INAMAC, ROMEO ratings on the grading sheets of Socorro Medina and Patricia
AGUSTIN, AIDA CAMINO, LUNA SARMAGO, AURORA Liñgat from 80% to 85%, and some teachers giving petitioner
LORENA, SOLEDAD FRANCISCO and MR. FLOR a starting grade of 75% in Grade VI, which proves that there
MARCELO, respondents-appellees. has already an intention to pull him to a much lower rank at
the end of the school year; that several district examinations
Teodoro M. Santiago for petitioner-appellant. outside of teachers' daily units and other than periodical tests
Ramon C. Carag for respondent-apellees. were given, ratings in which were heavily considered in the
determination of periodical ratings, whereas according to the
Academic Supervisor and Acting Division Superintendent of
BARREDO, J.: schools of the place such district examinations were not
advisable; that there was a unanimous agreement and
Appeal from the order of the Court of First Instance of understanding among the respondent teachers to insult and
Cotabato dismissing, on a motion to dismiss, its Civil Case No. prejudice the second and third honors by rating Socorro
2012 — for certiorari, injunction and damages — on the Medina with a perfect score, which is very unnatural; that the
ground that the complaint therein states no cause of action, words "first place" in petitioner's certificate in Grade I was
and from the subsequent order of the court a quo denying the erased and replaced with the words "second place", which is
motion for the reconsideration of the said order of dismissal. an instance of the unjust and discriminating abuses committed
by the respondent teachers in the disputed selection of honor
The record shows that at the time Civil Case No. 2012 was pupils they made; that petitioner personally appealed the
commenced in the court below, appellant Teodoro matter to the School Principal, to the District Supervisor, and
Santiago, Jr. was a pupil in Grade Six at the public school to the Academic Supervisor, but said officials "passed the
named Sero Elementary School in Cotabato City. As the buck to each other" to delay his grievances, and as to appeal
school year 1964-1965 was then about to end, the to higher authorities will be too late, there is no other speedy
"Committee on the Rating of Students for Honor" was and adequate remedy under the circumstances; and, that
constituted by the teachers concerned at said school for the petitioner and his parents suffered mental and moral damages
purpose of selecting the "honor students" of its graduating in the amount of P10,000.00. They prayed the court, among
class. With the school Principal, Mrs. Aurora Lorena, as others, to set aside the final list of honor students in Grade VI
chairman, and Juanita Bautista, Rosalinda Alpas, Rebecca of the Sero Elementary School for that school year 1964-1965,
Matugas, Milkita Inamac, Romeo Agustin, Aida Camino and and, during the pendency of the suit, to enjoin the respondent
Luna Sarmago, as members, the above-named committee teachers from officially and formally publishing and
deliberated and finally adjudged Socorro Medina, Patricia proclaiming the said honor pupils in Grade VI in the graduation
Liñgat and Teodoro C. Santiago, Jr. as first, second and third exercises the school was scheduled to hold on the 21st of May
honors, respectively. The school's graduation exercises were of that year 1965. The injunction prayed for was denied by the
thereafter set for May 21, 1965; but three days before that lower court in its order of May 20, 1965, the said court
date, the "third placer" Teodoro Santiago, Jr., reasoning out that the graduation exercises were then already
represented by his mother, and with his father as counsel, set on the following day, May 21, 1965, and the restraining of
sought the invalidation of the "ranking of honor students" the same would be shocking to the school authorities, parents,
thus made, by instituting the above-mentioned civil case and the community who had eagerly looked forward to the
in the Court of First Instance of Cotabato, against the coming of that yearly happy event. As scheduled, the
above-named committee members along with the District graduation exercises of the Sero Elementary School for the
Supervisor and the Academic Supervisor of the place. school year 1964-1965 was held on May 21, with the same
protested list of honor students.

The corresponding complaint filed alleged, inter alia: that


plaintiff-petitioner Teodoro C. Santiago, Jr. is a sixth grader at Having been required by the above-mentioned order to
the Sero Elementary School in Cotabato City scheduled to be answer the petition within ten (10) days, respondents moved
graduated on May 21st, 1965 with the honor rank of third for the dismissal of the case instead. Under date of May 24,
place, which is disputed; that the teachers of the school had 1965, they filed a motion to dismiss, on the grounds (1) that
been made respondents as they compose the "Committee on the action for certiorari was improper, and (2) that even
the Rating of Student for Honor", whose grave abuse of official assuming the propriety of the action, the question brought
discretion is the subject of suit, while the other defendants before the court had already become academic. This was
were included as Principal, District Supervisor and Academic opposed by petitioner.
Supervisor of the school; that Teodoro Santiago, Jr. had been
a consistent honor pupil from Grade I to Grade V of the Sero In an order dated June 4, 1965, the motion to dismiss of
Elementary School, while Patricia Liñgat (second placer in the respondents was granted, the court reasoning thus:
disputed ranking in Grade VI) had never been a close rival of
petitioner before, except in Grade V wherein she ranked third; The respondents now move to dismiss the petition for
that Santiago, Jr. had been prejudiced, while his closest rival being improper and for being academic. In order to resolve
had been so much benefited, by the circumstance that the the motion to dismiss, the Court has carefully examined the
latter, Socorro Medina, was coached and tutored during the petition to determine the sufficiency of the alleged cause of
summer vacation of 1964 by Mrs. Alpas who became the action constituting the special civil action of certiorari.
teacher of both pupils in English in Grade VI, resulting in the
far lead Medina obtained over the other pupil; that the
committee referred to in this case had been illegally
FINALS CONSTITUTIONAL LAW I ACJUCO NOV 11, 2017 84

The pertinent portions of the petition alleging 'grave abuse of aggrieved thereby may file a verified petition in the proper
discretion' are found in paragraphs 3, 4, 5, 6, 7, 8, 9 and 10. court alleging the facts with certainty and praying that
These allegations may be substantially summarized as judgment be rendered annulling or modifying the proceedings,
follows: Paragraph 3 alleges that since grades one to six, the as the law requires, of such tribunal, board or officer.'
students closely contending for class honors were Socorro
Medina, Teodoro Santiago, Jr., Dolores Dalican and Patricia 'The petition shall be accompanied by a certified true copy of
Liñgat. the judgment or order subject thereof, together with copies of
all pleadings and documents relevant and pertinent thereto.'
Socorro Medina obtained first honor thrice (grades I, V and
VI); once second honor (grade IV), and twice third place It is striking, indeed, that this petition has not been
(grades II and III). accompanied by a certified true copy of the judgment or order
complained of, together with all pleadings and documents
Teodoro Santiago, Jr. obtained first place once (grade IV); which are relevant thereto, as required by the second,
four times second place (grades I, II, III, and V) and once third paragraph of the aforequoted rule. This violation renders the
place (grade VI). petition extremely indefinite and uncertain. There is no written
formal judgment or order of respondents that is submitted for
Dolores Dalican obtained twice first place (grades II, III); once revision or correction of this Court. This violation is fatal to the
third place (grade I). petition.

Patricia Liñgat once third place (grade V); and once second ADMINISTRATIVE REMEDIES NEGLECTED
place (grade VI).
All that the petition alleges is that the petitioner personally
That as now ranked in the graduation Liñgat is given second appealed to the school authorities who only 'passed the buck
place while Teodoro Santiago, Jr., is given the third place only. to each other.' This allegation does not show that petitioner
This is the ranking now disputed by petitioner, Teodoro formally availed of and exhausted the administrative remedies
Santiago, Jr. of the Department of Education. The petition implies that this
is the first formal complaint of petitioner against his teachers.
The administrative agencies of the Department of Education
Paragraph 4 alleges that Socorro Medina was tutored in the could have investigated the grievances of the petitioner with
summer of 1964 by Mrs. Rosalinda Alpas who became her dispatch and give effective remedies, but petitioner
English teacher in the sixth grade; that as such, Mrs. Alpas negligently abandoned them. Petitioner cannot now claim that
unjustly favored Socorro against her rivals. he lacked any plain, speedy and adequate remedy.

Paragraph 5 alleges that the teachers who composed the NO GRAVE ABUSE OF DISCRETION
committee on honor students are all grade six teachers while
the Service Manual For Teachers provides that the committee
shall be composed of the teachers from the fifth and sixth Allegations relating to the alleged 'grave abuse of discretion'
grades. on the part of teachers refer to errors, mistakes, or
irregularities rather than to real grave abuse of discretion that
would amount to lack of jurisdiction. Mere commission of
Paragraph 6 alleges that there are direct and circumstantial errors in the exercise of jurisdiction may not be corrected by
evidence showing the change of ratings of Socorro Medina means of certiorari.
and Patricia Liñgat from 80% to 85% and the intention to junk
petitioner to a lower rank.
In view of the foregoing, the Court is of the opinion, and so
holds, that the petition states no cause of action and
Paragraph 7 alleges that the giving of district examinations should be, as it is hereby dismissed.
upon which ratings were partly based were not advisable.
Upon receipt of a copy of the above-quoted order, the
Paragraph 8 alleges that the teachers rated Socorro Medina petitioner moved for the reconsideration thereof, but the same
a perfect pupil which is unnatural. proved to be futile, hence, this appeal.

Paragraph 9 alleges that on the first grade certificate of the Appellant here assails the holding of the lower court that his
petitioner the word "First Place" was erased and changed to petition states no cause of action on the grounds — discussed
"Second Place". by the court a quo in the appealed order above-quoted — (1)
that the petition does not comply with the second paragraph
Paragraph 10 alleges that petitioner personally appealed to of Sec. 1 of Rule 65 because it has not been accompanied by
the school authorities but they only 'passed the buck to each a certified true copy of the judgment or order subject thereof,
other.' together with copies of all pleadings and documents relevant
and pertinent thereto; (2) that administrative remedies were
SECOND PARAGRAPH VIOLATED not first exhausted; and (3) that there was no grave abuse of
discretion on the part of the teachers who constituted the
committee referred to. On the other hand, appellees maintain
Rule 65, Section 1 of the Rules of Court provides: that the court below did not err in dismissing the case on said
grounds. Further, they argue in favor of the questioned order
'Section 1. Petition for certiorari. — When any tribunal, board, of dismissal upon the additional ground that the "committee
or officer exercising judicial functions, has acted without or in on the ratings of students for honor" whose actions are here
excess of its or his jurisdiction, or with grave abuse of condemned by appellant is not the "tribunal, board or officer
discretion and there is no appeal, nor any plain, speedy, and exercising judicial functions" against which an action for
adequate remedy in the ordinary course of law, a person certiorari may lie under Section 1 of Rule 65.
FINALS CONSTITUTIONAL LAW I ACJUCO NOV 11, 2017 85

The last point raised by appellees deserves first consideration, some matter incidental thereto, and of which they have
for if really the said committee of teachers does not fall within jurisdiction; the power of a court to decide and pronounce a
the category of the tribunal, board, or officer exercising judicial judgment; the power which adjudicates upon and protects the
functions contemplated by Rule 65, further discussion of the rights and interests of individual citizens, and to that end
issues raised by appellant may no longer be necessary. To construes and applies the law. "Judicial power" implies the
resolve this problem the following tests may be employed: construction of laws and the adjudication of legal rights. It
includes the power to hear and determine but not everyone
In this jurisdiction certiorari is a special civil action instituted who may hear and determine has judicial power. The term
against 'any tribunal, board, or officer exercising judicial "judicial power" does not necessarily include the power to hear
functions.' (Section 1, Rule 67.) A judicial function is an act and determine a matter that is not in the nature of a suit or
performed by virtue of judicial powers; the exercise of a action between the parties.' (34 C.J. 1183-1184.) .
judicial function is the doing of something in the nature of the
action of the court (34 C.J. 1182). In order that a special civil (3) the tribunal, board or officer must pertain to that branch of
action of certiorari may be invoked in this jurisdiction the the sovereign power which belongs to the judiciary, or at least,
following circumstances must exist: (1) that there must be a which does not belong to the legislative or executive
specific controversy involving rights of persons or property department.
and said controversy is brought before a tribunal, board or
officer for hearing and determination of their respective rights ... the distinction between legislative or ministerial functions
and obligations. and judicial functions is difficult to point out. What is a judicial
function does not depend solely upon the mental operation by
'Judicial action is an adjudication upon the rights of parties which it is performed or the importance of the act. In solving
who in general appear or are brought before the tribunal by this question, due regard must be had to the organic law of
notice or process, and upon whose claims some decision or the state and the division of power of government. In the
judgment is rendered. It implies impartiality, discharge of executive and legislative duties, the exercise of
disinterestedness, a weighing of adverse claims, and is discretion and judgment of the highest order is necessary, and
inconsistent with discretion on the one hand — for the tribunal matters of the greatest weight and importance are dealt with.
must decide according to law and the rights of the parties — It is not enough to make a function judicial that it requires
or with dictation on the other; for in the first instance it must discretion, deliberation, thought, and judgment. It must be the
exercise its own judgment under the law, and not act under a exercise of discretion and judgment within that subdivision of
mandate from another power. ... The character of its action in the sovereign power which belongs to the judiciary, or, at
a given case must decide whether that action is judicial, least, which does not belong to the legislative or executive
ministerial, or legislative, or whether it be simply that of a department. If the matter, in respect to which it is exercised,
public agent of the country or State, as in its varied belongs to either of the two last-named departments of
jurisdictions it may by turns be each.' (In Re Saline County government, it is not judicial. As to what is judicial and what is
Subscription, 100 Am. Dec. 337, 338, cited in Southeastern not seems to be better indicated by the nature of a thing, than
Greyhound Lines v. Georgia Public Service Commission, 181 its definition.' (Whealing & Elm Grove Railroad Co. Appt. v.
S. E. 836-837.) Town of Triadelphia, et al., 4 L.R.A. (N. S.) pp. 321, 328-329.)
[Emphasis supplied]1
'It may be said generally that the exercise of judicial function
is to determine what the law is, and what the legal rights of 'WHAT ARE JUDICIAL OR QUASI JUDICIAL ACTS. It is
parties are, with respect to a matter in controversy; and difficult, if not impossible, precisely to define what are judicial
whenever an officer is clothed with that authority, and or quasi judicial acts, and there is considerable conflict in the
undertakes to determine those questions, he acts judicially.' decisions in regard thereto, in connection with the law as to
(State ex rel. Board of Commissioners of St. Louis County, et the right to the writ of certiorari. It is clear, however, that it is
al. v. Dunn, 90 N. W. 772-773.) the nature of the act to be performed, rather than of the office,
board, or body which performs it, that determines whether or
(2) the tribunal, board or officer before whom the controversy not it is the discharge of a judicial or quasi-judicial function. It
is brought must have the power and authority to pronounce is not essential that the proceedings should be strictly and
judgment and render a decision on the controversy construing technically judicial, in the sense in which that word is used
and applying the laws to that end. when applied to the courts of justice, but it is sufficient if they
are quasi judicial. It is enough if the officers act judicially in
making their decision, whatever may be their public character.
'The phrase "judicial power" is not capable of a precise ...' "In State ex rel. Board of Commrs. vs. Dunn (86 Minn. 301,
definition which would be applicable to all cases. The term has 304), the following statements were made:
been variously defined as the authority to determine the rights
of persons or property by arbitrating between adversaries in
specific controversies at the instance of a party thereto; the 'The precise line of demarkation between what are judicial and
authority exercised by that department of government which what are administrative or ministerial functions is often difficult
is charged with the declaration of what the law is and its to determine. The exercise of judicial functions may involve
construction so far as it is written law; the authority or power the performance of legislative or administrative duties, and the
vested in the judges or in the courts; the authority vested in performance of administrative or ministerial duties, may, in a
some court, officer, or persons to hear and determine when measure, involve the exercise of judicial functions. It may be
the rights of persons or property or the propriety of doing an said generally that the exercise of judicial functions is to
act is the subject matter of adjudication; the power belonging determine what the law is, and what the legal rights of parties
to or emanating from a judge as such; the power conferred are, with respect to a matter in controversy; and whenever an
upon a public officer, involving the exercise of judgment and officer is clothed with that authority, and undertakes to
discretion in the determination of questions of right in specific determine those questions, he acts judicially.'2
cases affecting the interest of persons or property, as
distinguished from ministerial power or authority to carry out It is evident, upon the foregoing authorities, that the so called
the mandates of judicial power or the law; the power exercised committee on the rating of students for honor whose actions
by courts in hearing and determining cases before them, or are questioned in this case exercised neither judicial nor quasi
FINALS CONSTITUTIONAL LAW I ACJUCO NOV 11, 2017 86

judicial functions in the performance of its assigned task. From legal principle not in harmony with the generally accepted
the above-quoted portions of the decision cited, it will be views thereon. (See C.J.S. Vol. 1, p. 1012.)
gleaned that before tribunal board, or officer may exercise
judicial or quasi judicial acts, it is necessary that there be a We observe that in assuming jurisdiction over the matter, the
law that give rise to some specific rights of persons or property respondent judge reasoned out that where there is a wrong
under which adverse claims to such rights are made, and the there is a remedy and that courts of first instance are courts of
controversy ensuing therefrom is brought, in turn, before the general jurisdiction.
tribunal, board or officer clothed with power and authority to
determine what that law is and thereupon adjudicate the
respective rights of the contending parties. As pointed out by The flaw in his reasoning lies in the assumption that Imperial
appellees,3 however, there is nothing on record about any rule suffered some wrong at the hands of the board of judges. If at
of law that provides that when teachers sit down to assess the all, there was error on the part of one judge, at most. Error and
individual merits of their pupils for purposes of rating them for wrong do not mean the same thing. 'Wrong' as used in the
honors, such function involves the determination of what the aforesaid principle is the deprivation or violation of a right. As
law is and that they are therefore automatically vested with stated before, a contestant has no right to the prize unless and
judicial or quasi judicial functions. Worse still, this Court has until he or she is declared winner by the board of referees or
not even been appraised by appellant of the pertinent judges.
provisions of the Service Manual of Teachers for Public
Schools appellees allegedly violated in the composition of the Granting that Imperial suffered some loss or injury, yet in law
committee they constituted thereunder, and, in the there are instances of 'damnum absque injuria'. This is one of
performance of that committee's duties. them. If fraud or malice had been proven, it would be a
different proposition. But then her action should be directed
At any rate, the situation brought before Us in this case, the against the individual judge or judges who fraudulently or
seemingly one of first impression, is not without substantial maliciously injured her. Not against the other judges.
parallel. In the case of Felipe vs. Leuterio, etc., et al.,4 the
issue presented for determination was whether or not the But even were We to assume for the moment, as the court
courts have the authority to reverse the award of the board of below apparently did, that judicial intervention might be sought
judges of an oratorical contest, and this Court declared that in cases of this nature, still, We are inclined to sustain the
the judiciary has no power to reverse the award of the board order of dismissal appealed from for failure on the part of
of judges of that contest and, for that matter, it would not appellant to comply with the requirements of Section 1 of Rule
interfere in literary contests, beauty contests and similar 65. To be sure, the lower court's holding that appellant's failure
competitions. It was reasoned out thus: to accompany his petition with a copy of the judgment or order
subject thereof together with copies of all pleadings and
For more than thirty years oratorical tilts have been held documents relevant and pertinent thereto "is fatal to his
periodically by schools and colleges in this islands. Inter- cause" is supported not only by the provision of that Rule but
collegiate oratorical competitions are of more recent origin. by precedents as well. In the case of Alajar, et al. vs. Court of
Members of this court have taken part in them either as Industrial Relations,5where it was claimed by therein
contestants in their school days (In the College of Law, U.P. petitioners that the respondent court had acted with grave
annual oratorical contest, first prize was awarded to Justice abuse of discretion in estimating certain rice harvests involved
Montemayor in 1914 and to Justice Labrador in 1916), or as in the case in terms of cavans instead of cans, allegedly in
members of the board of judges afterwards. They know some complete disregard of the decision of the Court of First
few verdicts did not reflect the audience's preference and that Instance of Batangas in Expropriation Proceedings No. 84
errors have sometimes been ascribed to the award of the and of this Court in G.R. No.
judges. Yet no party ever presumed to invoke judicial
intervention; for it is unwritten law in such contests that the L-6191,6 and in ordering thereafter the division of the said rice
board's decision is final and unappealable. harvests on the ratio of 70-30 in favor of the tenants, this Court
denied the petition for certiorari on the ground, among others,
Like the ancient tournaments of the Sword, these tournaments of failure on the part of said petitioners to attach to their
of the Word apply the highest tenets of sportsmanship: finality petition copies of the decisions allegedly violated. Speaking
of referee's verdict. No alibis, no murmurs of protest. The thru Mr. Justice J.B.L. Reyes then, this Court held:
participants are supposed to join the competition to contribute
to its success by striving their utmost: the prizes are The petition is patently without merit. In the first place, it is not
secondary. even sufficient in form and substance to justify the issuance of
the writ of certiorari prayed for. It charges that the Court of
No rights to the prizes may be asserted by the contestants, Industrial Relations abused its discretion in disregarding the
because theirs was merely the privilege to compete for the decision of the Court of First Instance of Batangas in
prize, and that privilege did not ripen into a demandable right Expropriation Proceedings No. 84 and of this Court in G.R.
unless and until they were proclaimed winners of the No. L-6191; yet it does not attach to the petition the decisions
competition by the appointed arbiters or referees or judges. allegedly violated by the Court below and point out which
particular portion or portions thereof have been disregarded
by the respondent Court.
Incidentally, these school activities have been imported from
the United States. We found in American jurisprudence no
litigation questioning the determination of the board of judges. The same principle was applied in the more recent case of
NAWASA vs. Municipality of Libmanan, et al.,7 wherein this
Court dismissed (by Resolution) the petition for certiorari and
Now, the fact that a particular action has had no precedent mandamus filed by the National Waterworks and Sewerage
during a long period affords some reason for doubting the Authority against the Court of First Instance of Camarines Sur,
existence of the right sought to be enforced, especially where and the municipality of Libmanan. In the following language,
occasion for its assertion must have often arisen; and courts this Court emphasized the importance of complying with the
are cautious before allowing it, being loath to establish a new said requirement of Rule 65:
FINALS CONSTITUTIONAL LAW I ACJUCO NOV 11, 2017 87

While paragraph 3 of the petition speaks of the complaint filed


by the respondent municipality with the respondent court for
recovery of property with damages (Civil Case No. L-161) no
copy thereof is attached to the petition.

Similarly, paragraph 4 of the petition mentions the decision


rendered by the respondent court on December 10, 1965, but
no copy thereof is attached to the petition.

Again, paragraph 5 of the petition speaks of the order of


default entered by the respondent court and of the motion for
reconsideration filed by petitioner in the case above-
mentioned, but no copy of the order of default is attached to
its petition.

Bearing in mind that the petition under consideration was filed


for the purpose of enjoining the respondent court from
executing the decision rendered in Civil Case No. L-161, the
importance of the missing pleadings is obvious.

Moreover, the petition is also for the purpose of securing an


order commanding the respondent court to approve either the
original or the amended record on appeal filed petition, but no
copy of either is attached to its petition.

In view of the foregoing, the petition under consideration is


dismissed.

It might be true, as pointed out by appellant, that he received


a copy of the programme of the graduation exercises held by
the Sero Elementary School in the morning of the very day of
that graduation exercises, implying that he could not have
attached then a copy thereof (to show the decision of the
committee of teachers in the ranking of students complained
of) to his petition. The stubborn fact remains, however, that
appellant had known of such decision of the said committee
of teachers much earlier, as shown by the circumstance that
according to him, even before the filing of his petition with the
lower court on the 19th of May, 1965, he had personally
appealed the said committee's decision with various higher
authorities of the above-named school, who merely passed
the buck to each other. Moreover, appellant mentions in his
petition various other documents or papers — as the Service
Manual for Teachers allegedly violated by appellees in the
constitution of their committee; altered grading sheets; and
erasures in his Grade I certificate — which appellant never
bothered to attach to his petition. There could be no doubt
then that he miserably failed to comply with the requirement
of Rule 65 above-mentioned. With this conclusion, it is no
longer necessary to pass upon the other two errors assigned
by appellant.

FOR THE FOREGOING CONSIDERATIONS, the judgment


appealed from is affirmed, with costs against appellant.

Concepcion, C.J., Reyes, J.B.L., Dizon, Makalintal, Zaldivar,


Castro, Fernando, Teehankee and Villamor, JJ., concur.
FINALS CONSTITUTIONAL LAW I ACJUCO NOV 11, 2017 88

G.R. No. L-4606 May 30, 1952 were added, and the contestant receiving the lowest number
got first prize, the next second prize, etc.
RAMON B. FELIPE, SR., as Chairman, Board of Judges,
petitioner, 7. The sums for the first four winners were: Nosce 10; Imperial
10; Benevides 17, General 17, the Board of judges having
vs. voted as follows:

HON. JOSE N. LEUTERIO, Judge, Court of First Instance Judge Nosce Imperial Buenavides General
of Camarines Sur, EMMA IMPERIAL, represented by her
guardian-ad-litem JUSTO V. IMPERIAL, and SOUTHERN Felipe Sr. ......... 3 1 2 4
LUZON COLLEGE, respondents.
Obias .............. 1 2 4 3
Ramon Felipe, Jr., and L. B. Karingal for petitioner.
Rodriguez .......... 1 4 5 3
Ezequiel S. Grageda and Victoriano Yamson for respondents
Judge Leuterio and Emma Imperial. Prado .............. 3 2 1 3

Padilla and San Juan for respondent Southern Luzon College. Moll ............... 2 1 5 4

BENGZON, J.: 10 10 17 17

Statement of the case. The issue in the litigation is whether 8. It appearing that Nestor Nosce and Emma Imperial had tied
the courts have the authority to reverse the award of the board for the first place, the Chairman, apparently with the consent
of judges of an oratorical competition. of the board, broke the tie awarding first honors to Nosce and
second honors to Imperial.
In an oratorical contest held in Naga, Camarines Sur, first
honor was given by the board of five judges to Nestor Nosce, 9. For the convenience of the judges the typewritten forms
and second honor to Emma Imperial. Six days later, Emma contained blank spaces in which, after the names of the rival
asked the court of the first instance of that province to orators and their respective orations, the judge could not jot
reversed that award, alleging that one of the judges had fallen down the grades he thought the contestants deserved
to error in grading her performance. After a hearing, and over according to "Originality", "Timeliness", "English", "Stage
the objection of the other four judges of the contest, the court Personality", "Pronunciation and Enunciation" and "Voice".
declared Emma Imperial winner of the first place. Hence this From such data he made up his vote.
special civil action challenging the court's power to modify the
board's verdict.
10. It was discovered later that the form filed by Delfin
Rodriguez, one of the Judges, gave Imperial and General the
The facts. There is no dispute about the facts: following ratings under the above headings; Imperial 19-15-
15-18-14-14 Total 94-Place 4th General 19-15-15 or 14-19-
1. On March 12, 1950 a benefit inter-collegiate oratorical 14-14 Total 95-Place 3rd.
contest was held in Naga City. The contestants were eight,
among them Nestor Nosce, Emma Imperial, and Luis General, 11. Imperial asserts that her total should be 95 instead of 94
Jr. and therefore should rank 3rd place in Rodriguez' vote. And if
she got 3 from Rodriguez, her total vote should have been 9
2. There were five judges of the competition, the petitioner instead of ten, with the result that she copped first place in the
Ramon B. Felipe, Sr. being the Chairman. speaking joust.

3. After the orators had delivered their respective pieces, and 12. Rodriguez testified that he made a mistake in adding up
after the judges had expressed their votes, the Chairman Imperial's ratings; that she should have been given a total of
publicly announced their decision awarding first price to 95, or placed No. 3, the same as General; that he was not
Nestor Nosce, second price to Emma Imperial, third price to disposed to break the tie between her and General and
Menandro Benavides and fourth place to Luis General, Jr. insisted that he wanted to give rank 3 to Imperial and rank 3
also to General.
4. Four days afterwards, Emma Imperial addressed a letter to
the Board of Judges protesting the verdict, and alleging that Discussion. Although it would seem anomalous for one judge
one of the Judges had committed a mathematical mistake, to give the same rank to two contestants, we will concede for
resulting in her second place only, instead of the first, which the moment that Delfin Rodriguez could have given 3 to
she therefore claimed. Imperial to General.

5. Upon refusal of the Board to amend their award, she filed a However if deductions are to be made from his recorded vote
complaint in the court of first instance. (Exhibit 3) one may infer that after the contest and before
submitting his vote he decided to give General an edge over
6. At the contest the five judges were each furnished a blank Imperial. How? Under the caption "English" General was
form wherein he give the participants grades according to his given by himself at first "14", later increased to "15". Evidently
estimate of their abilities, giving number 1 to the best, number because after he had added the ratings of Imperial and
2 to the second best etc., down to number 8. Then the grades (erroneously) reached the sum of 94, he added the ratings of
General (which were the same as Imperial with 14 under
FINALS CONSTITUTIONAL LAW I ACJUCO NOV 11, 2017 89

"English") and (mistakenly) reached 94 also. So what did he The flaw in his reasoning lies in the assumption that Imperial
also? He raised the 14 to 15 and thus gave general 95 to place suffered some wrong at the hands of the board of judges. If at
him over Imperial's 94. (Mistakingly again, because with 15 all, there was error on the part of one judge, at most. Error and
General got 96 instead of 95). wrong do not mean the same thing. "Wrong" as used in the
aforesaid legal principle is the deprivation or violation of a
But to us the important thing is Rodriguez' vote during and right. As stated before, a contestant has no right to the prize
immediately after the affair. His vote in Exhibit 3 definitely unless and until he or she is declared winner by the board of
gave General place No. 3 and Imperial place No. 4. His referees or judges.
calculations recorded on Exhibit 3 were not material. In fact
the Chairman did not bother to fill out the blank spaces in his Granting that Imperial suffered some loss or injury, yet in
own form, and merely set down his conclusions giving one to law there are instances of "damnum absque injuria". This
Imperial, 2 to Benavides etc. without specifying the ratings for is one of them. If fraud or malice had been proven, it
"Voice", "English", "Stage Personality" etc. In other words would be a different proposition. But then her action
what counted was the vote. should be directed against the individual judge or judges
who fraudulently or maliciously injured her. Not against
Probably for the above reasons the board refused to "correct" the other judges.
the alleged error.
By the way what is here in stated must not be understood as
The situation then is this: Days after a contest has been applying to those activities which the government has chosen
conducted and the winners announced, one of the judges to regulate with the creation of the Games and Amusements
confesses he made a mistake, that the ratings he gave the Board in Executive Order No. 392, Series 1950.
second place winner should have been such as would entitle
her to first place. The other judges refuse to alter their verdict. Judgment. In view of all the foregoing, we are of the opinion
May the matter be brought to the court to obtain a new and so declare, that the judiciary has no power to reverse
award, reversing the decision of the board of judges? the award of the board of judges of an oratorical contest.
For that matter it would not interfere in literary contests, beauty
For more than thirty years oratorical tilts have been held contests and similar competitions.
periodically by schools and colleges in these islands. Inter-
collegiate oratorical competitions are of more recent origin. Wherefore the order in controversy is hereby set aside. No
Members of this court have taken part in them either as costs.
contestants in their school days1, or as members of the board
of judges afterwards. They know some (few) verdicts did not Paras, C.J., Pablo, Tuason, Montemayor, Bautista Angelo
reflect the audience's preference and that errors have and Ladrador, JJ., concur.
sometimes been ascribed to the award of the judges. Yet no
party ever presumed to invoke judicial intervention; for it is
unwritten law in such contests that the board's decision is final Feria, J., concurs in the result.
and unappealable.

Like the ancient tournaments of the Sword, these tournaments


of the Word apply the highest tenets of sportmanship: finally
of the referee's verdict. No alibis, no murmurs of protest. The
participants are supposed to join the competition to contribute
to its success by striving their utmost: the prizes are
secondary.

No rights to the prizes may be asserted by the contestants,


because their's was merely the privilege to compete for the
prize, and that privilege did not ripen into a demandable right
unless and until they were proclaimed winners of the
competition by the appointed arbiters or referees or judges.

Incidentally, these school activities have been imported from


the United States. We found in American jurisprudence no
litigation questioning the determination of the board of judges.

Now, the fact that a particular action has had no precedent


during a long period affords some reason for doubting the
existence of the right sought to be enforced, especially where
occasion for its assertion must have often arisen; and courts
are cautious before allowing it, being loath to establish a new
legal principle not in harmony with the generally accepted
views thereon. (See C.J.S. Vol. 1, p. 1012).

We observe that in assuming jurisdiction over the matter, the


respondent judge reasoned out that where there is a wrong
there is a remedy and that courts of first instance are courts of
general jurisdiction.
FINALS CONSTITUTIONAL LAW I ACJUCO NOV 11, 2017 90

REQUIREMENTS AS TO DECISIONS before the Regional Director, the transfer was motivated by
the fact:
G.R. No. L-52364 March 25, 1983
xxx xxx xxx
RICARDO VALLADOLID, petitioner,
That as such switchboard operator numerous telephone
conversations and communications relating to business and
vs. confidential matters were intercepted and relayed to
Tropicana Apartment-Hotel, a competitor;
HON. AMADO G. INCIONG, Deputy Minister of Labor, and
COPACABANA APARTMENT-HOTEL, respondents. That to confirm suspicion on Ricardo Valladolid as the person
responsible for said interception and relay, Mrs. Lourdes T.
G.R. No.L-53349 March 25, 1983 Yu, President of JRM & Co., Inc. sent him on an errand to
Manila Hotel to bring flowers on the occasion of Wedding
J.R.M. & CO., INC. as owner and operator of Copacabana Anniversary of Mr. & Mrs. Yu Hong Ty. Matters which Mrs.
Apartment-Hotel petitioners, Lourdes Yu told him in confidence and admonitions not to tell
anyone, reached Tropicana people;

vs.
xxx xxx xxx 1

HON. AMADO G. INCIONG, as Deputy Minister of


Labor,HON. FRANCISCO L. ESTRELLA, as Regional The affidavit further disclosed:
Director of the National Capital Region, Ministry of Labor, nd
RICARDO VALLADOLID, respondents. xxx xxx xxx

Daniel Co for petitioner Ricardo Valadolid. That while serving in his capacity as clerk/collector, copies of
Accounts Receivables, reach Tropicana Management
The Solicitor General for respondents. although said copies were not referred to them;

Vicente V. Ocampo & Antonio V. de Ocampo for J.R.M. & Co., That conferred (sic) on numerous confidential matters taken
Inc. in the office of Copacabana Apartment-Hotel reached
Tropicana Apartment-Hotel;

MELENCIO-HERRERA, J.:
That to finally and fully confirmed suspicions that Ricardo
Valladolid was the person responsible for the aforementioned
The Order dated December 26, 1979 of the Deputy Minister disclosures, a plan for the entrapment was conceived by the
of Labor affirming the Order of May 2, 1979 for management of Copacabana Apartment- Hotel;
reinstatement without backwages issued by Regional
Director Francisco L. Estrella in Case No. R4-STF-2-1316-
79 entitled, "Ricardo C. Valladolid, Jr. vs. Copacabana That on November 9, 1979, pursuance of said plan, a cash
Apartment-Hotel," is being assailed by the parties in these voucher for P500,000.00 supposedly in payment for
petitions. representation expenses to myself with the corresponding
check were prepared and issued respectively by Juan V.
Bermudo, Apartment-Hotel Manager, who thereafter called
J. R. M. & Co., Inc. (hereinafter referred to as JRM), as Ricardo Valladolid and asked the latter to bring the said cash
petitioner in G.R. No. 53349, is also the respondent in G.R. voucher and check to my room which he did; few minutes later
No. 52364 named therein as Copacabana Apartment-Hotel. I came down to the office and asked Mr. Ricardo Valladolid to
JRM originally owned and operated not only Copacabana but prepare the corresponding deposit slip to Pacific Banking
also Tropicana Apartment-Hotel. The principal stockholders of Corporation for said check;
JRM were the brothers Joseph, Manuel, Vicente and Roman,
all surnamed Yu. Upon the death of Joseph on October 12,
1975, although both Copacabana and Tropicana continued That thereafter, the aforementioned cash voucher,
technically as owned by JRM, the controlling (70%) interest in corresponding check and deposit slip were kept in the hotel
Copacabana was lodged in the surviving heirs of Joseph, with vault with no other person other than myself, Juan Bermudo
brothers Manuel and Roman having a 15% interest each. JRM and Ricardo Valladolid having any knowledge of preparation
was placed under the management of the heirs of Joseph. The and existence thereof;
brothers Manuel, Roman and Vicente were allowed 100%
equity interest in Tropicana, which was operated separately That unknown to Ricardo Villadolid, the aforementioned
from JRM. Eventually, Tropicana and Copacabana became check, cash voucher and deposit slip were cancelled;
competing businesses.
That on December 4, 1978, Mr. Manuel Yu Chua, came to
Ricardo Valladolid, petitioner in G.R. No. 52364 and Copacabana Apartment-Hotel as minority stockholder of the
respondent in G.R. No. 53349, after the death of Joseph, was latter, vehemently demanding for an accounting of
employed by JRM in 1977 as a telephone switchboard Copacabana books;
operator. He was subsequently transferred to the position of
clerk-collector by Mrs. Lourdes T. Yu, President of JRM. That he strongly charged that information reached him that I
received a disbursement of P500,000.00 from Copacabana
According to the affidavit of Daniel T. Yu, Executive Vice- Apartment-Hotel as representation expenses in my capacity
President, attached to the position paper submitted by JRM as Executive Vice-President thereof;
FINALS CONSTITUTIONAL LAW I ACJUCO NOV 11, 2017 91

That at this juncture, I brought out the cancelled cash voucher, clearance and Valladolid's complaint for Illegal Dismissal were
check and deposit slip with mouth agape Manuel Yu Chua, consolidated and docketed as R4-STF-2-1316-79. The parties
could do nothing else but admit that in fact, his informer within submitted their respective position papers and documentary
Copacabana Apartment-Hotel was no other than Mr. Ricardo evidence. On May 2, 1979, the Regional Director issued the
C. Villadolid; following challenged Order:

That I then informed Manuel Yu Chua, that under the WHEREFORE, premises considered, the application for
circumstances, I could no longer repose any trust whatsoever clearance with preventive suspension is hereby denied.
on Ricardo Valladolid and requested him to take the latter to Respondent is hereby ordered to reinstate complainant to his
Tropicana Apartment-Hotel and just swap him with someone former position without backwages and without loss of
else; Mr. Manuel Yu Chua directed me to tell Valladolid to see seniority rights. Let the time this case was pending be
him; considered as complainant's suspension for his absences.

That after few days, Ricardo Valladolid came back and told The claim for vacation sick leave pay is dismissed for failure
me that Manuel Yu Chua has no place for him at Tropicana to substantiate the same.
Apartment-Hotel; in this conversation, Ricardo Valladolid
apologized for having betrayed the trust that we had reposed Valladolid appealed the foregoing order to the Minister of
on him, especially after Mrs. Lourdes T. Yu had told him to Labor seeking modification of the same, praying for the award
stay impartial; that he then having done this for Manuel Yu of backwages from the time he was illegally dismissed on
Chua, the latter could not even accept him in Tropicana February 16, 1979 to the date of his actual reinstatement. JRM
Apartment-Hotel; also appealed the said Order.

xxx xxx xxx 2 On December 26, 1979, the Deputy Minister of Labor, in a
succinct Order, dismissed both appeals after finding "no
The entrapment scheme was corroborated by the affidavits of sufficient justification or valid reason to alter, modify, much
Sofia Mo. Gianan, External Auditor of J.R.M. & Co., Inc., and less reverse the Order appealed from."
Juan V. Bermudo, Copacabana Apartment-Hotel Manager,
which affidavits formed part of JRM's position paper filed On January 21, 1980, Valladolid filed a Petition for certiorari
before the agency below. 3 The cancelled Cash Voucher, the with this Court, docketed as G.R. No. 52364, praying for a
uncashed check, and the unused deposit slip, all in the modification of the Order of December 26, 1979 of the Deputy
respective amounts of P500,000.00 were also attached to the Minister of Labor so as to grant him backwages. This Court
same position paper as Exhibits "4", "5" and "6". resolved. on February 4, 1980, to give due course to the
petition, and required the parties to submit simultaneous
On December 29, 1978, or after the entrapment scheme had memoranda.
been effected, Valladolid filed a written request for a five (5)
day vacation leave starting December 30, 1978 with the On March 12, 1980, JRM also filed a petition for certiorari with
Manager of Copacabana, stating therein that he would report this Court assailing that same Order. This Court gave due
for work on January 5, 1979. 4 He did not report for work on course to the petition and consolidated the same with G.R.
January 5 but sent a telegram from Bicol on January 8, 1979 No. 52364. Thereafter, the parties filed their respective
requesting for 15 days sick leave as he was confined for flu at memoranda.
the Dr. Estrellado Clinic. 5 On January 23, 1979, Valladolid's
wife allegedly called up JRM informing the company through
its accountant, Eddie Escueta, that her husband was still sick The non-award of backwages is the only issue being raised
and requested for 30 days sick leave, which was allegedly by Valladolid claiming that the Orders in question are contrary
granted. This was denied by JRM. to law and evidence, and were issued arbitrarily and
capriciously with grave abuse of discretion, amounting to
excess or lack of jurisdiction.
Valladolid reported for work on February 16, 1979. The
Executive Vice- President, Mr. Daniel Yu, allegedly refused to
admit him and instead asked him to resign. JRM maintains JRM, on the other hand, assails the said Orders on the
that Valladolid left the office that same day and never following grounds:
returned, because he was reprimanded for his unauthorized
absences. I

On February 22, 1979, Valladolid filed a Complaint for Illegal That respondent Deputy Minister of Labor committed grave
Dismissal with vacation and sick leave pay. 6 abuse of discretion when in his questioned order in effect
sustained the finding of respondent Regional Director that
On February 24, 1979, JRM sent a letter to Valladolid signed there is no evidence to support the dismissal of private
by Daniel T. Yu, advising him of his preventive suspension respondent.
effective February 26, 1979 preparatory to the termination of
his services 10 days from receipt of a copy of the application II
for clearance to dismiss him. The grounds given were: (1)
Willful Breach of Trust for having divulged, in various That respondent Deputy Minister Amado Inciong and
instances, confidential business matters to competitors of the Regional Director Francisco Estrella committed grave abuse
company; and (2) Gross Neglect of Duty for having been of discretion when they arbitrarily failed to consider in their
absent without leave or notice for more than 25 days, to the respective orders under review, established jurisprudence.
detriment of the company. 7

III
On February 28, 1979, JRM filed said application for
clearance with the Ministry of Labor. 8 The application for
FINALS CONSTITUTIONAL LAW I ACJUCO NOV 11, 2017 92

That respondent Regional Director committed grave abuse of facts relative to the business of Copacabana, which, if
discretion when he held that preventive suspension is divulged to Tropicana would be to the former's prejudice.
equivalent to dismissal.
Moreover, we find basis for the finding of the Regional Director
IV that Valladolid was terminated without prior clearance. J.R.M.
sent a memorandum to Valladolid on February 24, 1979
That the order of respondent Hon. Amado Inciong was a advising him of his preventive suspension effective February
capricious and whimsical exercise of judgment when it failed 26, 1979 pending approval of the application for clearance to
to state the facts and conclusion of law upon which it is based. dismiss him. The clearance application was filed on February
28, 1979. However, even prior to that date, or on February 22,
1979, Valladolid had already filed a complaint for Illegal
V Dismissal. This shows that Valladolid was indeed refused
admittance on February 16, 1979 when he reported back to
That respondent Regional Director Francisco Estrella acted in work, so that he was practically dismissed before he was
excess of his jurisdiction when, without any statutory authority formally notified of his suspension leading to his dismissal, in
or transcending beyond his jurisdiction, he absolutely violation of the requirement of Section 3, Rule XIV, Book V,
disregarded procedural requirement in the hearing of the Rules & Regulation Implementing the Labor Code. 12 And as
present controversy, thus depriving petitioner of its right to due provided in Section 2 of the same Rule, any dismissal without
process. prior clearance shall be "conclusively presumed to be
termination of employment without a just cause."
Valladolid, in his affidavit dated March 29, 1979, denied
having committed any breach of trust. 9 In corroboration, he JRM cannot claim that it was deprived of due process
presented the affidavits of Mr. Manuel Yu dated March 20, considering that applications for clearance have to be
1979 and March 29, 1979, wherein the latter stated that summarily investigated and a decision required to be rendered
Valladolid was "one of Copacabana's most hard-working and within ten (10) days from the filing of the opposition 13 As this
efficient employees;" that Valladolid's work is "mere routinary Court had occasion to hold there is no violation of due process
collection and clerical in nature which do not involve trust (or) where the Regional Director merely required the submission
confidential business or trade secrets which he may 'divulge' of position papers and resolved the case summarily thereafter.
to other companies." 10 14

On this issue, the Regional Director ruled that "there is no Nor is the questioned Order of the Deputy Minister of Labor
evidence on record that Valladolid furnished copies of violative of Section 9, Article X of the Constitution, which
receivables or divulged confidential business matters to Mr. requires a statement of the facts and the conclusions of law
Manuel Yu and the 'Tropicana People' including the upon which it is based. That prescription applies to decisions
P500,000.00 'entrapment scheme.'" of Courts of record. The Ministry of Labor is an administrative
body with quasi-judicial functions. Section 5, Rule XIII, Book
That finding is not supported by the records. The affidavits V, Ibid, states that proceedings in the NLRC shall be non-
attached to petitioner's position paper adequately show that litigious and summary in nature without regard to legal
JRM did not act on mere suspicion but on the contrary, acted technicalities obtaining in courts of law. As the Deputy Minister
prudently when it first transferred Valladolid from switchboard was in full accord with the findings of fact and the conclusions
operator where he could eavesdrop on telephone of law drawn from those facts by the Regional Director, there
conversations, to a less crucial position of clerk-collector. But was no necessity of discussing anew the issues raised
even in the latter capacity, JRM's fears were confirmed as therein.
shown by the entrapment scheme. Manuel Yu's certification
as to Valladolid's trustworthiness cannot be given much JRM admits that Valladolid requested for leave for 5 days from
weight not only because it was disproved by the entrapment December 30, 1978, and thereafter for 15 days, but denies
contrived but more so because even Manuel Yu himself that he notified the company of his absences subsequent to
refused to employ him at Tropicana when Daniel Yu had this. The Regional Director ruled that the absences of
suggested that Tropicana absorb Valladolid because JRM Valladolid were unauthorized but did not amount to gross
had lost confidence in the latter. And although Manuel Yu, who neglect of duty or abandonment of work which requires
owns 15% of the equity holding of Copacabana, and being a deliberate refusal to resume employment or a clear showing
member of the Board of Directors of JRM had a right to know in terms of specific circumstances that the worker does not
the business standing of said establishment, there is basis to intend to report for work. We agree. But as Valladolid had
believe that he would not have been able to pinpoint the been AWOL, no error was committed by respondent Regional
particular "disbursement" of P500,000.00, if the same had not Director in ordering his reinstatement without backwages. 16
been leaked out to him.
WHEREFORE, both Petitions for certiorari are hereby denied.
Loss of confidence is a valid ground for dismissing an No costs.
employee. Proof beyond reasonable doubt of the employee's
misconduct is not required, it being sufficient that there is SO ORDERED.
some basis for the same or that the employer has reasonable
ground to believe that the employee is responsible for the
misconduct and his participation therein renders him unworthy Plana, Vasquez, Relova and Gutierrez, Jr., JJ., concur.
of the trust and confidence demanded of his position. 11
However, as this was Valladolid's first offense, as found by the Teehankee, J., took no part.
Regional Director, dismissal from the service is too harsh a
punishment, considering that he had not been previously
admonished, warned or suspended for any misdemeanor.
Besides as clerk-collector, he need not be given access to
FINALS CONSTITUTIONAL LAW I ACJUCO NOV 11, 2017 93

G.R. No. 78648 January 24, 1989 On 5 December 1984, petitioner and his wife, on the one
hand, and on the other, Mayor Dans in his capacity both as
RAFAEL N. NUNAL, petitioner, Municipal Mayor and as Presiding Officer of the Sangguniang
Bayan of Isabela, Basilan, the Municipal Treasurer and the
Provincial Fiscal (p. 4, Reply To Comment of COA), entered
vs. into a Compromise Agreement stipulating, among others, that:

COMMISSION ON AUDIT AND MUNICIPALITY OF l. The respondents shall pay petitioner Rafael Nunal all back
ISABELA, BASILAN, respondents. salaries and other emoluments due him by reason of his
employment as Municipal Administrator of Isabela, Basilan,
Romulo D. Plagata for petitioner. covering the period from January 1, 1980 to August 15, 1984,
together with accumulated vacation/sick leaves, mid-year and
RESOLUTION Christmas bonuses in 1982 and 1983, and separation pay
under the Local Government Code, which are reflected in the
computation hereto attached and made an integral part
MELENCIO-HERRERA, J.: hereof... (p. 13, Rollo)

For resolution is petitioner's Motion for Reconsideration of the Under the same Compromise Agreement, petitioner was also
Minute Resolution of this Court of 11 May 1988 dismissing the considered as "retired" upon receipt of the monetary
Petition for certiorari "for failure of the petitioner to sufficiently considerations mentioned therein.
show that the public respondent had committed grave abuse
of discretion in holding, among others, that the compromise
agreement of the parties is not enforceable against the On 12 December 1984, the Court approved the Compromise
Municipality of Isabela, the latter not having been impleaded Agreement.
as an indispensable party in the case.
On 1 April 1985, petitioner collected his retirement benefits
In the present Motion, petitioner contends: although, concededly, no provision for the same had been
included in the Compromise Agreement (Petition, p. 6; Rollo,
P. 9).
1. The decision does not clearly and distinctly express the
facts and the law on which it is based;
On 17 September 1985, petitioner filed his claim for
separation pay in the amount of P54,092.50 to which he is
2. The Municipality of Isabela, Basilan, is bound by the allegedly entitled due to the abolition of the position of
compromise agreement; and Municipal Administrator, which separation pay is provided for
by the Local Government Code (B.P. 337, Section 76).
3. Public respondent "Commission on Audit (COA, for short)
gravely abused its discretion in denying the lawful claim for On 6 January 1986 the Municipal treasurer forwarded
separation pay by your petitioner." (Motion for petitioner's claim to the Provincial Auditor of Basilan. On 11
Reconsideration, p. 1; Rollo, p. 67) January, 1986, in a First Indorsement, the Provincial Auditor
opined that the claim was legal and proper but payment
The facts disclose that on 24 February 1986 petitioner was thereof was made subject to availability of funds and the ruling
appointed as Municipal Administrator of Isabela, Basilan. On of the Regional Office of the Commission on Audit, Region IX,
1 February 1980 he was administratively charged and Zamboanga City.
dismissed from the service for dishonesty, misconduct and for
lack of confidence. On appeal, the Merit Systems Board On 12 February 1986, in a 2nd Indorsement, the Regional
exonerated petitioner and reinstated him to his position as Director of the Commission on Audit, Region IX, Zamboanga
Municipal Administrator on 8 May 1980. City, reversed the Provincial Auditor of Basilan and denied
petitioner's claim for separation pay. Petitioner's Motion for
On 29 January 1981 petitioner was again dismissed for lack Reconsideration was forwarded to the Commission on Audit
of confidence by then Municipal Mayor Alvin Dans under (COA), Central Office, Quezon City.
Administrative Order No. 54, Series of 1981. Upon denial of
his Motion for Reconsideration, petitioner filed Case No. 43, a On 13 October 1986 the COA Central Office, in its Decision
suit for mandamus and Damages with Preliminary Injunction No. 388, not only denied petitioner's claim for separation pay
against the Municipal Mayor, the Municipal Treasurer, and the but also disallowed the other payments made to petitioner. It
Sangguniang Bayan of Isabela, Basilan, before the then Court held:.
of First Instance in Basilan Province, Branch 1, praying for
reinstatement "with full backwages and other rights inherent
in the position." He also filed Case No. 45 with the same Court Premises considered, and it appearing that Mr. Nunal has
seeking that he and his wife be paid their back salaries from 1 been paid back salaries and other emoluments in the total
February 1980 to 31 May 1980 pursuant to the Decision of the amount of P90,362.96 pursuant to the Compromise
Merit Systems Board on 16 February 1981. Agreement, supra, this Commission hereby directs that any
and all payments made to Mr. Nunal corresponding to the
period when he was no longer in the government service
On 20 February 1984, during the pendency of the said case, should be disallowed in audit without prejudice to his right of
the Sangguniang Bayan of Isabela, Basilan, abolished the recourse against the officials personally liable for his unlawful
subject position in its Resolution No. 902, Series of 1984, and dismissal. (pp. 15-16, Rollo)
Ordinance No. 336, pursuant to the provisions of the Local
Government Code.
Thus, this recourse by petitioner alleging grave abuse of
discretion by COA, which Petition we had previously
FINALS CONSTITUTIONAL LAW I ACJUCO NOV 11, 2017 94

dismissed in our Resolution of 11 May 1988 as heretofore To grant double gratuity is unwarranted (See Cajiuat, et al. vs.
adverted to. Mathay, Sr., G.R. No. L-39743, 124 SCRA 710, September
24, 1983).
It appearing, however, that the Compromise agreement was
duly signed by Mayor Alvin Dans as Mayor and as Presiding It may be that the matter of separation pay was included in the
Officer of the Sangguniang Bayan, by the Municipal Compromise Agreement. Nonetheless, it could not be granted
Treasurer, and by the Provincial Fiscal as their lawyer (Motion outright but still had to be claimed and passed in audit, and
for Reconsideration, p. 3); that the case was one for has been aptly denied by COA. And although petitioner did file
reinstatement and backwages; and following the ruling of this suit against the Municipality for reinstatement, it does not
Court in Gementiza vs. Court of Appeals (G.R. Nos. L-41717- follow that he was not effectively dismissed such that he could
33, 113 SCRA 477, April 12, 1982), the Municipality of Isabela still be considered an incumbent whose position had been
should be deemed as impleaded in this case, it being apparent abolished. A dismissed employee can be considered as not
that the officials concerned had been sued in their official having left his office only upon reinstatement and should be
capacity. given a comparable position and compensation at the time of
reinstatement (Cristobal vs. Melchor, No. L-43203, 101 SCRA
It should be noted that before the Court below, respondents 857, December 29, 1980).
sued petitioner Mayor alone. However, respondents, too,
prayed for a Writ of mandamus to compel petitioner Mayor to Finally, a word on petitioner's contention that the Resolution
reinstate them with back salaries and damages. of this Court under date of 11 May 1988 is not in accordance
Respondents, therefore, actually intended to sue petitioner in with Section 14, Article VIII of the 1987 Constitution, which
his official capacity. Failure to implead the Municipality and provides:
other municipal authorities should not deter this Court, in the
interests of justice and equity, from including them herein as Sec. 14. No decision shall be rendered by any Court without
respondents. (at p. 488) expressing therein clearly and distinctly the facts and the law
on which it is based.
The Compromise Agreement, therefore, must be held binding
on the Municipality of Isabela, which was not, in any way, No petition for review or motion for reconsideration of a
deprived of its day in Court (Gabutas vs. Castellanes, L- decision of the Court shall be refused due course or denied
17323, 14 SCRA 376, June 23, 1965). Thus, the payments to without stating the legal basis therefor.
petitioner of the sums of P68,389.25 as back salaries,
P21,387.71 as total accumulated vacation/sick leaves,
P772.75 as Christmas bonus, and the back salaries of Mrs. In the first place, our "Resolution" of 11 May 1988 was not a
Nanie B. Nunal in the sum of P3,096.00, have to be upheld. It "Decision" within the meaning of the Constitutional
likewise appears that retirement benefits bad also been requirement. This mandate is applicable only in cases
collected by petitioner on 1 April 1985. "submitted for decision," i.e., given due course and after the
filing of Briefs or Memoranda and/or other pleadings, as the
case may be. It is not applicable to an Order or Resolution
In respect, however, of the separation pay claimed by refusing due course to a Petition for Certiorari. In the second
petitioner, we uphold the ruling of the COA reading in part: place, the assailed Resolution does state the legal basis for
the dismissal of the Petition and thus complies with the
Anent the second issue, this Commission believes and so Constitutional provision. (Tayamura, et al., vs. IAC, et al., G.R.
holds that the instant claim for separation pay in addition to No. 76355, May 21, 1987 [en banc]; see also Que vs. People,
the retirement benefits earlier received by claimant is bereft of G.R. Nos. L-75217-18, 154 SCRA 160, September 21, 1987).
any legal basis. Culled from the records is the fact that Mr.
Nunal was dismissed from the service on January 29, 1981 It may be added that the Writ of certiorari dealt with in Rule 65
and has not been reinstated to the service until his position of of the Rules of Court is a prerogative Writ, never demandable
Municipal Administrator of Isabela was abolished. In other as a matter of right, "never issued except in the exercise of
words, he was no longer in, or had already been separated judicial discretion." (Bouvier's Law Dictionary, 3d Rev. [8th
from, the service when the said position was abolished. ed.]; Francisco, The Revised Rules, 1972 ed., Vol. IV- B, pp.
Evidently then, his separation from the service was not 4546, citing 14 C.J.S., 121-122).
attributable to the abolition of the position but was due to his
dismissal and, therefore, Section 76 of Batas Pambansa Blg.
337 which provides — ACCORDINGLY, the Resolution of this Court of 11 May 1988
is hereby PARTIALLY RECONSIDERED in that the
disallowance by respondent Commission on Audit of the
'Section 76.-Abolition of Position. When the position of an amounts ordered paid by the Court of First Instance of
official or employee under the civil service is abolished by law Basilan, Branch 1, in its Decision dated 12 December 1984, is
or ordinance, the official or employee so affected shall be hereby SET ASIDE, but its disallowance of petitioner's claim
reinstated in another vacant position without diminution of for separation pay of P54,092.50, is hereby SUSTAINED. No
salary. Should such position not be available, the official or costs.
employee affected shall be granted a separation pay
equivalent to one month salary for every year of service over
and above the monetary privileges granted to officials and SO ORDERED.
employees under existing law.' cannot be validly invoked as
legal basis for the claim for separation pay. Moreover, the fact Paras, Padilla, Sarmiento and Regalado, JJ., concur.
remains that as earlier seen Mr. Nunal has already been paid
his retirement benefits under the existing retirement law. His
entitlement, therefore, to separation pay under Batas
Pambansa Blg. 337 is offensive to the general policy of the
government prohibiting payment of double retirement benefits
to an employee. (p. 4, COA Decision No. 388; p. 15, Rollo)
FINALS CONSTITUTIONAL LAW I ACJUCO NOV 11, 2017 95

[G.R. Nos. 110817-22. June 13, 1997] 31160, the additional cases were raffled to Branch 97 of the
same court. These cases were eventually consolidated and
THE PEOPLE OF THE PHILIPPINES, plaintiff-appellee, assigned to Branch 88.
vs. MARCELINO A. BUGARIN, accused-appellant.
The informations in the six cases alleged as follows:
DECISION
Crim. Case No. 92-31157
MENDOZA, J.:
That on or about the month of June 1990 in Quezon City,
This is an appeal from the decision,[1] dated February 11, Philippines, the said accused by means of force and
1993, which the Regional Trial Court, Branch 97 of Quezon intimidation, did then and there, wilfully and feloniously have
City rendered in Criminal Cases Nos. Q-92-28785 to 86 and carnal knowledge of the undersigned MARY JANE BUGARIN
Q-92-31157 to 31160, finding accused-appellant Marcelino y ASUNCION, a minor, 15 years of age, without her consent
Bugarin guilty of four counts of consummated rape and one and against her will, to the damage and prejudice of the latter.
count of attempted rape and sentencing him as follows:
The crime was attended by the aggravating circumstance of
WHEREFORE, this Court finds the accused GUILTY beyond relationship.
reasonable doubt as charged of multiple (3 Counts) rape and
one count of attempted rape, and in accordance with Article Crim. Case No. 92-31158
335 of the Revised Penal Code sentences him to prison terms
as follows:
That on or about the month of November, 1989 in Quezon
City, Philippines, the said accused, by means of force and
1) For each of the four counts of the above rape, reclusion intimidation, did then and there, wilfully and feloniously have
perpetua. carnal knowledge with the undersigned MARY JANE
BUGARIN y ASUNCION without her consent and against her
2) For the attempted rape, two (2) years and four (4) months will, to the damage and prejudice of the latter.
in the minimum penalty to four (4) years in the maximum
period and to indemnify the private complainant in the amount The crime was attended by the aggravating circumstance of
of P50,000.00 as moral damages and exemplary damages of relationship.
P50,000.00 to deter sexual crimes of the sort committed by
accused.
Crim. Case No. 92-31159
SO ORDERED.
That on or about the 14th day of March, 1991 in Quezon City,
Philippines, the said accused, by means of force and
The complainant, Maryjane Bugarin, is the daughter of intimidation, did then and there, wilfully and feloniously have
accused-appellant. On February 22, 1992, accompanied by carnal knowledge of the undersigned MARYJANE BUGARIN
her mother, Regina Bugarin, and her maternal aunt, Nena y ASUNCION, a minor, 15 years of age, without her consent
Padecio, she complained to the Central Police District and against her will, to the damage and prejudice of the latter.
Command that she had been repeatedly raped by accused-
appellant. In her sworn statement she related how, on nine
different occasions between November 1989 and January 17, The crime was attended by the aggravating circumstance of
1992, her father entered the common sleeping area of their relationship.
house in Payatas, Quezon City and, after holding her knees
and spreading her legs, succeeded in inserting his penis into Crim. Case No. 92-31160
her vagina and kissed her breasts. She claimed that, on
January 17, 1992, her father molested her by kissing her That on or about the month of May 1990 in Quezon City,
vagina and that only by repeatedly kicking him did he desist Philippines, the said accused by means of force and
from molesting her any further. intimidation, did then and there wilfully and feloniously have
carnal knowledge of the undersigned MARYJANE BUGARIN
Complainant was examined on the same date by Emmanuel y ASUNCION, a minor, 15 years of age, without her consent
I. Aranas, PNP Medico-Legal Officer, who found that she was and against her will, to the damage and prejudice of the latter.
in non-virgin state physically.[2] On February 25, 1992, she
returned to the police station to file formal charges against her The crime was attended by the aggravating circumstance of
father. The case was referred to the Office of the Quezon City relationship.
Prosecutor which found probable cause and accordingly filed
charges for consummated rape and attempted rape by means
of force and intimidation committed on December 23, 1991 Crim. Case No. 92-28785
and January 17, 1992 against accused-appellant Marcelino
Bugarin. No bail was recommended considering that the That on or about the 17th day of January, 1992, in Quezon
evidence of guilt of the respondent is strong. The cases were City, Metro Manila, Philippines, and within the jurisdiction of
docketed as Criminal Cases Nos. Q-92-28785 and Q-92- this Honorable Court, the above-named accused, with lewd
28786 and raffled to Branch 88 of the Quezon City Regional designs and by means of force and intimidation, did then and
Trial Court. there wilfully, unlawfully and feloniously commence the
commission of the crime of Rape directly by overt acts, by then
On May 7, 1992, four more charges for rape by means of force and there kissing the nipples and the vagina of the
and intimidation committed on November 1989, May 1990, undersigned MARYJANE BUGARIN Y ASUNCION, a minor,
June 1990, and March 14, 1991 were filed against accused- and about to lay on top of her, all against her will, however,
appellant. Docketed as Criminal Cases Nos. Q-92-31157 to the said accused did not perform all the acts of execution
FINALS CONSTITUTIONAL LAW I ACJUCO NOV 11, 2017 96

which would have produced the crime of Rape by reason of Sec. 2. Form and contents of judgment. - The judgment must
some causes other than his own spontaneous desistance, be written in the official language, personally and directly
that is, undersigned complainant push him away, to the prepared by the judge and signed by him and shall contain
damage and prejudice of the undersigned in such amount as clearly and distinctly a statement of the facts proved or
may be awarded to her under the provisions of the New Civil admitted by the accused and the law upon which the judgment
Code. is based.

Crim. Case No. 92-28786 If it is of conviction, the judgment shall state (a) the legal
qualification of the offense constituted by the acts committed
That on or about the 23rd day of December, 1991, in Quezon by the accused, and the aggravating or mitigating
City, Metro Manila, Philippines, and within the jurisdiction of circumstances attending the commission thereof, if there are
this Honorable Court, the above-named accused, with lewd any; (b) the participation of the accused in the commission of
designs and by means of force and intimidation, did then and the offense, whether as principal, accomplice, or accessory
there wilfully, unlawfully and feloniously have sexual after the fact; (c) the penalty imposed upon the accused; and
intercourse with the undersigned MARYJANE BUGARIN Y (d) the civil liability or damages caused by the wrongful act to
ASUNCION, a minor, without her consent and against her will, be recovered from the accused by the offended party, if there
to her damage and prejudice in such amount as may be is any, unless the enforcement of the civil liability by a
awarded to her under the provisions of the New Civil Code. separate action has been reserved or waived.

Upon arraignment, accused-appellant pleaded not guilty in In case of acquittal, unless there is a clear showing that the
each case, after which trial ensued. Under questioning by the act from which the civil liability might arise did not exist, the
prosecutor, Maryjane Bugarin narrated how her father judgment shall make a finding on the civil liability of the
sexually assaulted her in their familys common sleeping area accused in favor of the offended party.
while no one was at home and threatened her if she told
anyone about what happened. The decision of the trial court falls short of this requirement in
at least three respects. First, it does not contain an evaluation
Accused-appellant denied the charges against him. He of the evidence of the parties and a discussion of the legal
claimed to be God fearing and morally upright and that his questions involved. It does not explain why the trial court
wife, Regina Bugarin, must have induced their daughter to file considered the complainants testimony credible despite the
the complaints against him because his wife blamed him for fact that, as accused-appellant points out, complainant could
financially neglecting their family since 1989. not remember the time of the day when she was allegedly
raped. It does not explain why accused-appellants licking of
complainants genital constituted attempted rape and not
In rebuttal, the prosecution presented Regina Bugarin who another crime.Second, the complainant testified that she had
testified that a good mother would not expose her child to been raped five times, to wit, in November 1989, on December
humiliation just to get back at her husband. She further 24, 1989, in June 1990, on March 14, 1991, and on December
claimed that her daughter, who had been raised properly and 23, 1991, and that once, on January 17, 1992, she was
taught to be honest, could not have fabricated the charges molested by her father who licked her private part, for which
against the accused-appellant. reason six informations were filed against him, but the
decision found the accused-appellant guilty of only four counts
In a two-page decision, promulgated on February 11, 1993, of rape (which the trial court erroneously said three counts)
the trial court, after giving a summary of the testimonies of the and one count of attempted rape, without explaining whether
complainant and accused-appellant, laconically ruled: accused-appellant was being acquitted of one charge of rape.
Third, the decision is so carelessly prepared that it finds the
The issue is simple. Is the private complainant credible in her accused-appellant guilty of three counts of consummated
story of how she was raped? The answer of this Court is an rape but sentences him to suffer the penalty of reclusion
undoubtful and a definite yes. perpetua for each of the four counts of . . . rape.

Accused-appellant questions the trial courts decision on the Maryjane claimed she had been raped on December 24,
ground that: (1) the testimony of Maryjane Bugarin is not 1989, but the information in Criminal Case No. Q-92-31160 is
credible; (2) the elements of force and intimidation had not for rape allegedly committed in May 1990. It must be for this
been proved; and (3) the decision of the trial court does not reason that the trial court convicted accused-appellant of only
state the facts and law upon which it was based. four counts of rape, instead of five. But the trial court should
have explained so, if this was really the reason, and expressly
acquitted the accused-appellant of the charge under this
On the other hand, the Solicitor General, representing the information.
prosecution, contends that complainant, who was only 15
years old when she reported the crime, was not likely to
concoct charges against her father and that the moral The requirement that the decisions of courts must be in writing
ascendancy of the father over her took the place of force and and that they must set forth clearly and distinctly the facts and
intimidation in rape. the law on which they are based serves many functions. It is
intended, among other things, to inform the parties of the
reason or reasons for the decision so that if any of them
We take up first accused-appellants charge that the decision appeals, he can point out to the appellate court the findings of
of the trial court does not state the grounds therefor. Indeed, facts or the rulings on points of law with which he disagrees.
the Constitution provides in part in Art. VIII, 14 that No More than that, the requirement is an assurance to the parties
decision shall be rendered by any court without expressing that, in reaching judgment, the judge did so through the
therein clearly and distinctly the facts and the law on which it processes of legal reasoning. It is, thus, a safeguard against
is based. This requirement is reiterated and implemented by the impetuosity of the judge, preventing him from deciding by
the 1985 Rules of Criminal Procedure which provides in Rule ipse dixit. Vouchsafed neither the sword nor the purse by the
120, 2: Constitution but nonetheless vested with the sovereign
FINALS CONSTITUTIONAL LAW I ACJUCO NOV 11, 2017 97

prerogative of passing judgment on the life, liberty or property Q- When you say inside me, what do you mean?
of his fellowmen, the judge must ultimately depend on the
power of reason for sustained public confidence in the A- His sex organ entered my what, I am fertile.
justness of his decision. The decision of the trial court in this
case disrespects the judicial function.
Q- What do you mean by what?
We would normally remand this case to the trial court for
compliance with the constitutional requirement for decisions. A- My sex organ, sir.
But this case has been pending for sometime and further
delay can be avoided if the Court simply reviews the whole Q- What did you do when your father tried to put his sex organ
evidence. After all, the records of the trial court contain the to your sex organ?
transcript of stenographic notes, the complainants sworn
statement dated February 22, 1992, the resolution of the A- I was trying to push his body away from me, and I said to
prosecutor, and the statement of the arresting officer, on the him, father, I dont like it, ayoko po.
basis of which the Court may properly decide the case.[3] For
this reason the Court has decided to review this case despite
the failure of the trial court to make detailed findings of facts Q- Despite your pleas, what happened next?
and a statement of the reasons underlying its decision.
A- He still continued what he is doing, and when I fainted, he
Now it is settled that when the complainant in a rape case, suddenly moved back.
more so if she is a minor,[4] testifies that she has been raped,
she says, in effect, all that is necessary to prove the Q- After he moved back, what transpired afterwards?
commission of the crime.[5] Care must be taken, however,
that her testimony is credible for a conviction to be justified
A- He went out and I was left crying.
based on her testimony alone.[6] In this case, Maryjane
Bugarin testified on November 25, 1992[7] as follows:
Q- After this first incident, were there any other similar incident
which happened, if any?
Q- On November 1989, was your father residing with you or
was he living with you?
A- There are, sir, but I cant recall when.
A- Yes, sir.
Q- How many times more or less?
Q- Now, on November, 1989 do you remember any unusual
incident that happened, if any? A- Around four (4) times.

A- Yes, sir, when he entered the room. Q- After November 1989?

Q- When you say he, are you referring to Marcelino Bugarin? A- Including November 1989.

A- Yes, sir. Q- How about on December 24, 1989, do you remember


where you were?
Q- If Marcelino Bugarin is present today, would you be able to
identify him? A- Yes, sir. I was in our house.

A- Yes, sir. (witness is pointing to a man wearing a green t- Q- Do you remember any unusual or extraordinary incident
shirt who answers by the name Marcelino Bugarin when that happened on December 24, 1989?
asked by the Court).
A- Yes, sir. It is the same thing that he did to me.
Q- On November 1989, you were mentioning that he, referring
to the accused, entered your room, what happened? Q- Please explain what the same thing he do to you?

A- (witness crying) When he entered the room, he embraced A- He embraced and kissed me on my cheek, my neck and
me and touched the different parts of my body and he also my breast.
informed me that when I grow up, I would not become
innocent.
Q- What happened next?

Q- What do you mean by those words that he told you that


A- He was spreading my legs.
when you grow up you would not be innocent?

Q- Then, after spreading your legs, what did the accused do?
A- So that when I grow up I will know what he will be doing.

A- He let his sex organ touched my sex organ.


Q- What did he do?

Q- After that, what happened next?


A- He touched my knees and spread them out and then
holding my breast and he put his sex organ inside me.
A- I cried again.
FINALS CONSTITUTIONAL LAW I ACJUCO NOV 11, 2017 98

Q- When you cried again, what did your father do, if any? Q- Would you kindly tell what happened in your house on this
day?
A- His face became scaring.
A- He licked my sex organ.
Q- Now, after December 24, 1989 incident, do you remember
where you were sometime in June 1990? Q- After that, what did he do next?

A- I was in our house. A- He was threatening me.

Q- Specifically, in June of 1990, do you remember any Q- What did you do when he threatened you?
unusual incident that happened?
A- I was so afraid. (witness is crying)
A- The same thing happened, over and over.
Q- What did the accused to after threatening you?
Q- What do you mean by the same thing happened, over and
over? A- He was doing nothing. He was just walking beside me.

A- He would embraced me and then kissed me and touched Q- What happened after you saw him walking just beside you
my breast and kissed my nipples. on that date?

Q- And, besides in embracing, kissing and kissing your A- None, sir. I was just crying.
nipples, what else did your father do on June 1990?
The accused-appellant claims that Maryjanes testimony
A- He kissed also my sex organ. contains inconsistencies which indicate that the charges
against him were fabricated. He points to the failure of
Q- Beside kissing your sex organ, what else did he do, if any? complainant on cross-examination to state in some instances
the exact date and time she was allegedly raped, and to the
A- He placed inside my sex organ his sex organ. fact that it took complainant two years before reporting the
incidents and that the prosecution did not present the medico-
legal officer who examined the complainant. Accused-
Q- In March 14, 1991, where were you? appellant also claims that no evidence was adduced to prove
that the rape was committed by force and intimidation.
A- I was also in our house.
The failure of the complainant to state in some cases the exact
Q- Do you remember what happened, if any, On March 14, date and time of the commission of rape is a minor matter and
1991? can be expected when a witness is recounting the details of a
humiliating experience which are painful and difficult to recall
A- Thats it again, I was in the room and again he embraced in open court and in the presence of other people.[8] Indeed,
me made me lie down then kiss my sex organ and then, he this Court has ruled that complainants failure to recall some
placed again his sex organ inside my sex organ. details of the crime, instead of suggesting prevarication,
precisely indicates spontaneity and is to be expected from a
witness who is of tender age and unaccustomed to court
Q- How about December 23, 1991, do you remember where proceedings.
you were?
Besides, the date of the commission of the rape is not an
A- I was also in the house. essential element of the crime.[9] The precise time of the
crime has no substantial bearing on its commission,[10]
Q- What happened? especially since in this case the date and time of the
commission of the crime is not material to the accused-
appellants defense. Indeed, accused-appellants contention is
A- I was lying down and he lied there beside me and told me
only that he could not have raped his daughter in the common
to accede to his desire.
bedroom at nighttime because the place where they sleep is
shut off from the rest of their house by a curtain.
Q- What do you mean to accede to his desire?
Suffice it to state that lust is no respecter of time and place.[11]
A- He wanted to use me again. Our cases record instances of rape committed inside family
dwellings when other occupants are asleep.[12] In the case at
Q- Then, he actually used you? bar, Maryjane testified that the accused-appellant was able to
rape her by sending out her siblings to play with their
neighbors children, and while her mother was at work from 3
A- Yes, sir.
p.m. to 11 p.m. Complainant explained her apparent inability
to recall the exact dates of the assaults upon her, thus:
Q- Now, on January 17, 1992, do you remember where you
were?
Q- Madam witness, you mentioned that you were raped
sometime November 1989, June 1990, December 24, 1989,
A- I was also in our house. March 14, 1991, December 23, 1991, how come that you
knew very well the date as December 24, March 14,
FINALS CONSTITUTIONAL LAW I ACJUCO NOV 11, 2017 99

December 23; or rather, how come that your complaint is only to live with him. This fact makes it unlikely that she would use
sometime in the early part of 1991? her daughter to destroy her husband more than ten years
later. A mother would not expose her child to public trial, if the
A- I remember because that was closed to the birthday of my charges she makes are not true.[21]
brother.
We find no evidence, however, to find accused-appellant
Q- How about March 14, how come you knew very well that guilty of the charge in Criminal Case No. Q-92-31160 for
you were molested by your father? alleged rape committed in May 1990. There is no evidence to
prove that accused-appellant raped complainant on that date.
Her testimony is to the effect that she was raped on another
A- Because at that time, our class will almost end and we were date, December 24, 1989. But accused-appellant cannot be
given clearances. convicted for this as no complaint was formally filed regarding
it. Accused-appellant must accordingly be acquitted of the
Q- What day is your last school day? charge in Criminal Case No. Q-92-31160.

A- I cannot remember, sir. Nor do we think that accused-appellant is guilty of attempted


rape committed on January 17, 1992 as the trial court held.
Q- Is it usual that you knew very well March 14, and you do Maryjane testified:
not know very well your last day of your school day?
Q- Now, on January 17, 1992, do you remember where you
A- Because March 14 is our clearance. were?

Neither does the delay in making a criminal accusation impair A- I was also in our house.
the credibility of a witness if such delay is satisfactorily
explained.[13] In People v. Coloma,[14] where the Q- Would you kindly tell what happened in your house on this
complainant was also only 13 years old when first molested day?
by her father, the Court adverted to the fathers moral and
physical control over the young complainant in explaining the A- He licked my sex organ.
delay of eight years before the complaint against her father
was made. In this case, Maryjane must have been
overwhelmed by fear and confusion, and shocked that her Q- After that, what did he do next?
own father had defiled her. After all, she had been very close
to him. She also testified that she was afraid to tell her mother A- He was threatening me.
because the latter might be angered, so that she finally
confided to her aunt. Indeed, a survey conducted by the Q- What did you do when he threatened you?
University of the Philippines Center for Womens Studies
showed that victims of rape committed by their fathers took
much longer in reporting the incidents to the authorities than A- I was so afraid. (witness is crying)
did other victims. Many factors account for this difference: the
fact that the father lives with the victim and constantly exerts Q- What did the accused to after threatening you?
moral authority over her, the threat he might make against her,
the victims fear of her mother and other relatives. A- He was doing nothing. He was just walking beside me.

Nor is it entirely true that no evidence of force and intimidation Q- What happened after you saw him walking just beside you
had been adduced during the trial. Maryjane testified that she on that date?
tried to resist her fathers advances but, on several occasions,
she was overpowered by him. She was embraced and thus
prevented from escaping.[15] At other times she was A- None, sir. I was just crying.
intimidated by menacing looks cast on her[16] and by threats
of harm.[17] Indeed, even if there was no violence or force The intent to commit rape is not apparent from the act
employed against her, the moral influence of accused- described. It cannot be inferred from this act (licking
appellant over the complainant sufficed to make the crime complainants genital) alone that his intention was to have
rape.[18] sexual intercourse with her because it has not been shown
that he had at least placed himself on top of the
Nor is a medical examination an indispensable element in complainant.[22] The act imputed to him cannot be considered
prosecutions for rape.[19] That the prosecution did not present a preparatory act to sexual intercourse.[23] Accused-
the medico-legal officer is, therefore, not an obstacle to a appellant is instead guilty of acts of lasciviousness. It can at
finding of guilt in this case. least be inferred from his act of kissing the genital of the
complainant that he was moved by lewd designs.[24]
We think the evidence in this case proves beyond all
reasonable doubt that Maryjane had been raped on four Although relationship, as an aggravating circumstance, is
occasions by accused-appellant: November 1989, June 1990, alleged only in Criminal Cases Nos. Q-92-31157 to 31160, this
March 14, 1991, and December 23, 1991. Complainant has circumstance was nonetheless proved during the trial
no motive to incriminate her father. To the contrary, she inCriminal Case No. Q-92-28785 and, therefore, should also
testified that she was close to him. The absence of a motive be appreciated in that case to justify the imposition of the
lends greater credence to her testimony.[20] Neither does her penalty in its maximum period.
mother have any reason to falsely accuse Marcelino Bugarin.
Regina Bugarin suspected her husband of having an affair WHEREFORE, the decision dated February 11, 1993 of the
with her sister in 1980 and confronted him, but she continued Regional Trial Court of Quezon City is SET ASIDE and
FINALS CONSTITUTIONAL LAW I ACJUCO NOV 11, 2017 100

another one is RENDERED finding accused-appellant


Marcelino Bugarin GUILTY of four counts of consummated
rape in Criminal Cases Nos. Q-92-28786, Q-92-31157, Q-92-
31158, and Q-92-31159 and SENTENCED to reclusion
perpetua and ORDERED to INDEMNIFY the complainant
Maryjane Bugarin in the amount of P30,000.00 in damages for
each count of rape committed; and of acts of lasciviousness
in Criminal Case No. Q-92-28785, for which he is
SENTENCED to suffer imprisonment from 6 months of arresto
mayor, as minimum, to six 6 years of prision correccional, as
maximum.

In Criminal Case No. Q-92-31160, accused-appellant is


hereby ACQUITTED.

SO ORDERED.

Regalado, (Chairman), Romero, Puno, and Torres, Jr., JJ.,


concur.
FINALS CONSTITUTIONAL LAW I ACJUCO NOV 11, 2017 101

G.R. No. 104874 December 14, 1993 reclusion temporal, as maximum and to pay the offended party
Remedios de Leon in the amount of P100,000.00 value of the
DANILO HERNANDEZ, petitioner, jewelries embezzled and to pay the costs; Crim. case No. 29-
87, for Viol. of BP 22, he is hereby sentenced to eight (8)
months of prision correccional, and to pay the costs (Rollo, pp.
vs. 49-50).

THE COURT OF APPEALS AND THE PEOPLE OF THE It appears that sometime in August 1986, petitioner was
PHILIPPINES, respondents. introduced to Remedios de Leon by his aunt, as one engaged
in the business of buying and selling jewelry (TSN, March 17,
Marcelo Y. Hernandez for petitioner. 1987, pp. 18, 20).

The Solicitor General for People of the Philippines. In their first transaction, petitioner paid in cash the several
pieces of jewelry which he bought from de Leon. In their
QUIASON, J.: subsequent dealings, petitioner either paid in cash or by way
of postdated checks (TSN, March 17, 1987, pp. 22-23). On
one occasion, petitioner issued post-dated checks with the
This is a petition for review on certiorari under Rule 45 of the aggregate amount of P275,000.00. These checks bounced.
Revised Rules of Court seeking to set aside the Decision of However, upon notice of dishonor by the drawee banks
the Court of Appeals in concerned, petitioner paid de Leon cash in exchange for the
dishonored checks. Some checks were likewise exchanged
CA-G.R. CR No. 05877, the dispositive portion of which reads with cash even prior to their due date (TSN, March 17, 1987,
as follows: pp. 33, 37).

WHEREFORE, FINDING the decision appealed from to be in Several days before October 20, 1986, petitioner told de Leon
accordance with law and evidence, the same is hereby that he was interested in buying some more pieces of jewelry
AFFIRMED except as to Criminal Case No. 21-87 where, for (TSN, March 17, 1987, p. 60). On that date, at around 10:00
reasons above discussed, the accused-appellant is A.M., petitioner, together with his common-law wife,
ACQUITTED (Rollo, p. 33). Rosemarie Rodriguez, and two other companions, went to the
house of de Leon in Cavite City (TSN, March 17, 1987, p. 70).
Petitioner selected a pair of 2-carat diamond earrings worth
CA-G.R. CR No. 05877 was an appeal by petitioner from the
P150,000.00 for which he issued BPI Check No. 798246
decision of the Regional Trial Court, Branch 17, Cavite City in
payable to "cash" in the said amount and post-dated it to
Criminal Cases Nos. 21-87 to 29-87, the dispositive portion of
October 26, 1986. The amount of the check was filled in by
which reads as follows:
Rosemarie Rodriguez and petitioner affixed his signature as
drawer (TSN, March 17, 1987, p. 69).
WHEREFORE, in view of the foregoing, the Court finds Danilo
Hernandez guilty beyond reasonable doubt in the following
Petitioner and Rodriguez returned to de Leon's house at about
cases: Crim. Case No. 21-87, for Estafa and he is hereby
7:00 P.M. and bought one choker with 20 diamond stones and
ordered sentenced to an indeterminate prison term from
one bracelet with 16 diamond stones, for which he issued BPI
Twelve (12) years and one (1) day of reclusion temporal, as
Check No. 798247 payable to "cash" in the amount of
minimum to Twenty (20) years of reclusion temporal, as
P250,000.00 and postdated it to October 27, 1986. Petitioner
maximum and to pay the offended party Remedios de Leon in
specifically instructed de Leon to give him one week to confer
the amount of P150,000.00 corresponding to the value of
with his buyer before de Leon negotiates the check. The check
jewelries embezzled and unreturned and to pay the costs;
was also prepared by Rodriguez and signed by petitioner
Crim. Case No. 22-87, for Estafa, he is hereby ordered
(TSN, March 17, 1987, pp. 73-79).
sentenced for an indeterminate prison term of Twelve (12)
years and one (1) day of reclusion temporal, as minimum to
Twenty (20) years of reclusion temporal. as maximum and to In the evening of October 22, 1986, petitioner again went to
pay the offended party Remedios de Leon in the amount of the house of de Leon and bought one heart-shaped diamond
P100,000.00 for the value of the jewelries embezzled and to set. In payment thereof, he issued BPI Check No. 798248
pay the costs; Crim. Case No. 23-87, Violation of BP 22, he is payable to "cash" in the amount of P280,000.00 and post-
hereby sentenced to eight (8) months of prision correccional dated it to November 9, 1986 (TSN, March 17, 1987, pp. 84-
and to pay the costs; Crim. Case No. 25-87, for Estafa, he is 89).
hereby sentenced to an indeterminate prison term of from
Twelve (12) years and one (1) day of reclusion temporal, as On October 23, 1986, petitioner made a long-distance call to
minimum to Twenty (20) years of reclusion temporal, as de Leon and inquired whether she still had jewelry for sale.
maximum and to pay the offended party Remedios de Leon in When she said that she did, petitioner made an appointment
the amount of P250,000.00 for the value of the jewelries with her. Petitioner arrived at de Leon's house at around 2:30
embezzled and to pay the costs; Crim. Case No. 26-87, for P.M. The two went to a restaurant, where petitioner selected
Viol. of BP 22, he is hereby sentenced to eight (8) months of a set of earrings and a ring, each piece with a 1-carat diamond
prision correccional and to pay the costs; Crim. Case No. 27- (TSN, March 31, 1987, pp. 15-16). In payment for the set,
87, for Estafa, he is hereby sentenced to an indeterminate petitioner issued to de Leon BPI Check No. 798250 payable
prison term of from Twelve (12) years and one (1) day of to "cash" in the amount of P100,000.00 and dated that same
reclusion temporal, as minimum to twenty (20) years of day (October 23, 1992). Petitioner told de Leon that the check
reclusion temporal, as maximum and to pay the offended party was funded and that she could even withdraw the amount on
Remedios de Leon in the amount of P280,000.00 for the value that day. De Leon did not encash the check for the bank was
of jewelries embezzled and to pay the costs; Crim. Case No. closed (TSN, March 31, 1987, pp. 16, 19).
28-87, for Estafa, he is hereby sentenced to an indeterminate
prison term of from twelve (12) years and one (1) day of
reclusion temporal, as minimum to twenty (20) years of
FINALS CONSTITUTIONAL LAW I ACJUCO NOV 11, 2017 102

In the evening of October 24, 1986, petitioner with Rodriguez INCULPATORY EVIDENCE AND PRESENCE OF
went to de Leon's house and bought a 5-carat diamond piece. NUMEROUS PROOFS NEGATING THE SOUNDNESS OF
In payment thereof, petitioner indorsed to de Leon ASB Check SUCH VERDICT.
No. 245964 in the amount of P150,000.00 post-dated to
November 10, 1986 and issued by one Enrique Araneta (TSN, VIII
March 31, 1987, pp. 21-26).
IN PRONOUNCING THE ACCUSED, THROUGH A SINGLE
BPI Checks Nos. 798246, 798247 and 798250 were drawn JUDGMENT, GUILTY OF ALL THE NINE DISTINCT
against insufficient funds, while BPI Check no. 798248 and OFFENSES COVERED BY THE NINE SEPARATE
ASB Check No. 245964 were drawn against a closed account. INFORMATIONS SUBJECT MATTER OF CRIMINAL CASES
NOS. 21-87 TO 29-87, INCLUSIVE (Rollo, p. 56).
Petitioner was charged in nine informations with estafa and
violation of B.P. Blg. 22. Obviously, the Court of Appeals did not deem it necessary to
make a separate findings of fact for said assigned errors,
At his arraignment, petitioner pleaded not guilty to the charges because they were just the necessary consequences of the
(Rollo, pp. 16-17). previous, assigned errors.

After a joint trial, petitioner was convicted of the nine charges Petitioner next questions: (i) as violative of the constitutional
in a joint decision. mandate that decisions shall contain the facts and the law on
which they are based (1987 Constitution, Art. VIII, sec. 14,
On appeal to the Court of Appeals, the conviction of petitioner par. 1), the decision of the Court of Appeals which merely
was affirmed as to Criminal Cases nos. 22-87, 23-87, 24-87, adopted the statement of facts of the Solicitor general in the
25-87, 26-87, 27-87, 28-87 and 29-87, and reversed as to appellee's brief, and (ii) as violative of the constitutional
Criminal Case No. 21-87. mandate requiring that any denial of a motion for
reconsideration must state the legal basis thereof (1987
Constitution, Art. VIII, Sec. 14, par. 2), the denial of his motion
In this petition, petitioner claims that his conviction of nine of reconsideration on the basis of a comparison of said motion
distinct offenses subject of nine separate informations in a with the "comment thereon" (Rollo, pp. 10-12).
single judgment is reversible error, This contention is
untenable.
In its decision the Court of Appeals merely stated: "The facts
of the case as summarized in the Appellee's Brief are as
The case of United States v. Tanjuatco, 1 Phil. 116, relied follows: " and the quoted in full the statement of facts of the
upon by petitioner, is distinguishable from the instant case. In Solicitor General (Rollo, p. 20). According to petitioner, the
Tanjuatco, we held that the trial court improperly rendered a Court of Appeals did not make its own "independent judicial
single judgment for two offenses in one of the two criminal opinion" by such act of adopting the statement of facts made
cases, in the absence of a consolidation of the two cases. In by the advance party (Rollo, p. 11).
the case at bench, the trial court rendered a judgment for each
of the nine separate informations, albeit in the same decision.
What the Court of Appeals, in effect, said was that it found the
facts as presented by the Solicitor General as supported by
We further stated in Tanjuatco that the trial court violated "an the evidence. The constitutional mandate only requires that
essential right of the accused, inasmuch as he is entitled, the decision should state the facts on which it is based. There
although accused of two offenses, to a trial in each of the two is no proscription made in the briefs or memoranda of the
cases upon the proofs adduced in each individual case, and parties, instead of rewriting the same in its own words.
upon the allegations set forth in each information. It is not
permissible to take into account or consider in one case the
facts proved in the other, and vice versa" (at pp. 117-118). Precisely briefs or memoranda are required in order to aid the
courts in the writing of decisions.
The trial of the nine criminal cases was conducted jointly
without any objection from petitioner. Even had he signified We note that aside from adopting the statement of facts of the
his opposition to the joint trail, such opposition would have Solicitor General, the Court of Appeals also made findings of
been unavailing. A consolidation of trails, at the court's facts in the course of its discussion of the assignment of
discretion, is allowed in "charges for offenses founded on the errors.
same facts, or forming part of a series of offenses of similar
character" (1985 Rules on Criminal procedure, Rule 119, Sec. As to the denial of the motion for reconsideration, the Court of
14). Appeals stated in its Resolution dated March 30, 1992:

Petitioner also complains that while he assigned eight errors, Acting on the motion for reconsideration filed by the accused-
the Court of Appeals did not make a complete findings of fact appellant of the decision dated December 13, 1991 and the
as to the last two assigned errors (Rollo, pp. 11-12). comment thereon of the Solicitor General, the Court finds no
cogent reason that could justify a modification or reversal of
In the last two assigned errors, petitioner claims that the trial the decision sought to be reconsidered.
court erred:
Accordingly, the instant motion for reconsideration is hereby
VII DENIED for lack of merit (Rollo, p. 35).

IN FINDING THE ACCUSED GUILTY BEYOND The denial, therefore, was based on the ground that the Court
REASONABLE DOUBT IN CRIMINAL CASES NOS. 21-87 of Appeals did not find any "cogent reason that could justify a
TO 29-87 NOTWITHSTANDING ABSENCE OF
FINALS CONSTITUTIONAL LAW I ACJUCO NOV 11, 2017 103

modification or reversal of the decision sought to be The trial court based the civil indemnity on the actual price of
reconsidered." the jewelry as agreed upon by petitioner and de Leon at the
time of the transactions and this is reflected by the face value
Petitioner claims that the decision of the trial court is not of the checks.
supported by the evidence, which is contrary to the findings of
the Court of Appeals that said decision is "in accordance with Petitioner further contends that the trial court erred in
law and the evidence" (Rollo, p. 12). He points out that the admitting the checks in evidence without the prosecution first
appellate court should not have believed the trial court's proving that his signature on the checks were authentic (Rollo,
conclusion that "the sole testimony of the offended party pp. 13-14). If petitioner claims that his signatures on the
would have sufficed to sustain her assertions (Rollo, p. 47). checks were forged, the burden is on him to prove such fact.
He claims that self-serving declarations of a party favorable to He who alleges must prove his allegations.
himself are not admissible and that none of the alleged
witnesses to the transactions were presented. In the trial court, petitioner presented a certification issued by
the San Juan Police Station to the effect that he reported as
The common objection known as "self-serving" is not correct lost several blank checks, to wit: BPI Checks Nos. 798246,
because almost all testimonies are self-serving. The proper 798247, 798248, 798249, and 798250, and AB Check No.
basis for objection is "hearsay" (Wenke, Making and Meeting 245964. We agree with the trial court when it gave little weight
Objections, 69). to the certification. Like the trial court, we wonder why
petitioner never filed a criminal case against de Leon, if said
Petitioner fails to take into account the distinction between checks were really stolen.
self-serving statements and testimonies made in court. Self-
serving statements are those made by a party out of court Lastly, petitioner contends that because "the amounts
advocating his own interest; they do not include a party's covered by the checks were deposited by the offended party
testimony as a witness in court (National Development Co. v. in her savings account with the Prudential Bank, it becomes
Workmen's Compensation Commission, 19 SCRA 861 the liability of the bank by its acceptance to pay for the
[1967]). amounts of the checks" (Rollo, pp. 14-15).

Self-serving statements are inadmissible because the The case of Banco de Oro v. Equitable Banking Corporation,
adverse party is not given the opportunity for cross- 157 SCRA 188 (1988) cited by petitioner as authority, dealt
examination, and their admission would encourage fabrication with the negligence of a collecting bank which facilitated the
of testimony. This cannot be said of a party's testimony in payment by the drawee bank of the value of a check with a
court made under oath, with full opportunity on the part of the forged endorsement and signature of the payee. No such
opposing party for cross-examination. issue is involved in the case at bench.

It is not true that none of the alleged witnesses to the WHEREFORE, the petition is DENIED and the decision of the
transactions was presented in court (Rollo, p. 13). Yolanda Court of Appeals is AFFIRMED. Costs de oficio.
Dela Rosa, an eye-witness to some of the transactions,
testified for the prosecution. Assuming that Dela Rosa was not SO ORDERED.
presented as a witness, the testimony of de Leon sufficed to
sustain the conviction of petitioner. The conviction of an
accused may be on the basis of the testimony of a single Cruz, Davide, Jr. and Bellosillo, JJ., concur.
witness (People v. Rumeral, 200 SCRA 194 [1991]). In
determining the value and credibility of evidence, witnesses
are to be weighed, not counted (People v. Villalobos, 209
SCRA 304 [1992] ).

Petitioner also contends that while he was condemned by the


trial court to pay civil indemnity, no evidence was actually
presented in court to prove the existence, ownership and
worth of the pieces of jewelry other than the checks (Rollo, p.
14).

The existence of the jewelry was established by de Leon


herself, who testified that petitioner even selected the pieces
of jewelry before buying them.

As to the ownership of the jewelry, we held in People v. Dy,


109 SCRA 400 [1981] that:

Ownership is not a necessary element of the crime of estafa,


. . . In estafa, the person prejudiced or then immediate victim
of the fraud need not be the owner of the goods. This, Article
315 of the Revised Penal Code provides that "Any person who
shall defraud another (it does say 'owner') by any means
mentioned herein below shall be punished . . . ." All that is
necessary is that the loss should have fallen on someone
other than the perpetrators of the crime. . . ." (at p. 408).
FINALS CONSTITUTIONAL LAW I ACJUCO NOV 11, 2017 104

[G.R. No. 132428. October 24, 2000] logo. Mr. de la Cruz added that only Hankuk Stars of Korea
manufactured GE starters and if these were imported by
GEORGE YAO, petitioner, PEMCO, they would cost P7.00 each locally. As TCCs starters
cost P1.60 each, the witnesses agreed that the glaring
differences in the packaging, design and costs indisputably
vs. HON. COURT OF APPEALS, and THE PEOPLE OF proved that TCCs GE starters were counterfeit.
THE PHILIPPINES, respondents.
The defense presented YAO as its lone witness. YAO
DECISION admitted that as general manager, he has overall supervision
of the daily operation of the company. As such, he has the
DAVIDE, JR., C.J.: final word on the particular brands of products that TCC would
purchase and in turn sold. He also admitted that TCC is not
In this petition for review on certiorari, George Yao (hereafter an accredited distributor of GE starters. However, he
YAO) assails the 25 April 1995 Resolution of the Court of disclaimed liability for the crime charged since (1) he had no
Appeals in CA-G.R. No. 16893 which dismissed his appeal knowledge or information that the GE starters supplied to TCC
and ordered the remand of the records of the case to the were fake; (2) he had not attended any seminar that helped
Metropolitan Trial Court, Branch 52, Caloocan* City (hereafter him determine which TCC products were counterfeit; (3) he
MeTC) for execution. YAO was convicted by said MeTC for had no participation in the manufacture, branding, stenciling
unfair competition. of the GE names or logo in the starters; (4) TCCs suppliers of
the starters delivered the same already branded and boxed;
and (5) he only discussed with the suppliers matters regarding
YAOs legal dilemma commenced in June 1990 when the pricing and peak-volume items.
Philippine Electrical Manufacturing Company (hereafter
PEMCO) noticed the proliferation locally of General Electric
(GE) lamp starters. As the only local subsidiary of GE-USA, In its 13-page 20 October 1993 decision,[2] the MeTC
PEMCO knew that it was a highly unlikely market situation acquitted Roxas but convicted YAO. In acquitting Roxas, the
considering that no GE starter was locally manufactured or trial court declared that the prosecution failed to prove that he
imported since 1983.PEMCO commissioned Gardsmarks, was still one of the Board of Directors at the time the goods
Inc. to conduct a market survey. Gardsmarks, Inc., thru its were seized. It anchored its conviction of YAO on the
trademark specialist, Martin Remandaman, discovered that following: (1) YAOs admission that he knew that the starters
thirty (30) commercial establishments sold GE starters. All were not part of GEs line products when he applied with
these establishments pointed to Tradeway Commercial PEMCO for TCCs accreditation as distributor; (2) the
Corporation (hereafter TCC) as their source. Remandaman prosecutions evidence (Exhibit G-7), a delivery receipt dated
was able to purchase from TCC fifty (50) pieces of fluorescent 25 May 1989 issued by Country Supplier Center, on which a
lamp starters with the GE logo and design. Assessing that TCC personnel noted that the 2000 starters delivered were
these products were counterfeit, PEMCO applied for the GE starters despite the statement therein that they were China
issuance of a search warrant. This was issued by the MeTC, starters; this fact gave rise to a presumption that the TCC
Branch 49, Caloocan City. Eight boxes, each containing personnel knew of the anomaly and that YAO as general
15,630 starters, were thereafter seized from the TCC manager and overall supervisor knew and perpetrated the
warehouse in Caloocan City. deception of the public; (3) the fact that no genuine GE starter
could be sold from 1986 whether locally manufactured or
imported or at the very least in such large commercial quantity
Indicted before the MeTC, Branch 52, Caloocan City for unfair as those seized from TCC; and (4) presence of the elements
competition under Article 189 of the Revised Penal Code were of unfair competition.
YAO, who was TCCs President and General Manager, and
Alfredo Roxas, a member of TCCs Board of Directors. The
indictment[1] charged YAO and Roxas of having mutually and The dispositive portion of the decision reads as follows:
in conspiracy sold fluorescent lamp starters which have the
General Electric (GE) logo, design and containers, making For the failure of the prosecution to prove the guilt of the
them appear as genuine GE fluorescent lamp starters; and accused, Alfredo Roxas, of Unfair Competition under Article
inducing the public to believe them as such, when they were 189 (1) of the Revised Penal Code ... i.e., to prove that he was
in fact counterfeit. The case was docketed as Criminal Case Chairman of the Board of the Tradeway Commercial
No. C-155713. Corporation on October 10, 1990, as well as to have him
identified in open court during the trial, he is acquitted of the
Both accused pleaded not guilty. At the trial, the prosecution same.
presented evidence tending to establish the foregoing
narration of facts. Further, the State presented witnesses Atty. But because the prosecution proved the guilt of the other
Hofilena of the Castillo Laman Tan and Pantaleon Law Offices accused, George Yao, beyond reasonable doubt as principal
who underwent a familiarization seminar from PEMCO in 1990 under the said Article 189 (1) for Unfair Competition, he is
on how to distinguish a genuine GE starter from a counterfeit, convicted of the same. In the absence of any aggravating or
and Allan de la Cruz, PEMCOs marketing manager. Both mitigating circumstances alleged/proven, and considering the
described a genuine GE starter as having a stenciled silk- provisions of the Indeterminate Sentence Law, he is
screen printing which includes the GE logo... back to back sentenced to a minimum of four (4) months and twenty-one
around the starter, a drumlike glowbulb and a (21) days of arresto mayor to a maximum of one (1) year and
condenser/capacitor shaped like an M&M candy with the five (5) months of prision correccional.
numbers .006." They then compared and examined random
samples of the seized starters with the genuine GE products. This case was prosecuted by the law offices of Castillo Laman
They concluded that the seized starters did not possess the Tan and Pantaleon for ... PEMCO ... Considering that no
full design complement of a GE original. They also observed document was submitted by the private complainant to show
that some of the seized starters did not have capacitors or if how the claim ofP300,000 for consequential damages was
they possessed capacitors, these were not shaped like M&M. reached and/or computed, the court is not in a position to
Still others merely had sticker jackets with prints of the GE make a pronouncement on the whole amount. However, the
FINALS CONSTITUTIONAL LAW I ACJUCO NOV 11, 2017 105

offender, George Yao, is directed to pay PEMCO the amount From the Manifestation, filed on March 24, 1995, by City
of P20,000 by way of consequential damages under Article Prosecutor Gabriel N. dela Cruz, Kalookan City, it would
2202 of the New Civil Code, and to pay the law offices of appear that:
Castillo, Laman Tan and Pantaleon the amount of another
P20,000.00 as PEMCOs attorneys fees under Article 2208 xxx
(11) of the same.
2. George Yao received a copy of the RTCs decision on
This decision should have been promulgated in open court on August 16, 1994, and filed a motion for reconsideration on
July 28, 1993 but the promulgation was reset for August 31, August 30, 1994. On October 3, 1994, George Yao received
1993 in view of the absence of parties; it was again re-set for a copy of the RTCs order, dated September 28, 1994, denying
today. his motion for reconsideration.

Promulgated this 20th day of October, 1993 in Kalookan City, 3. On October 4, 1994, George Yao filed a notice of appeal by
Philippines.[3] registered mail.

YAO filed a motion for reconsideration, which the MeTC We will assume from the said Manifestation that the decision
denied in its order[4] of 7 March 1994. of the RTC and the order denying YAOs motion for
reconsideration were sent to and received by YAOs counsel.
YAO appealed to the Regional Trial Court of Caloocan City
(RTC). The appeal was docketed as Criminal Case No. C- Proceeding from said assumption, Yao had fifteen (15) days
47255(94) and was assigned to Branch 121 of the court. from August 16, 1994 to elevate his case to this Court. On
August 30, 1994, or fourteen (14) days thereafter, Yao filed a
On 24 May 1994, Presiding Judge Adoracion G. Angeles of motion for reconsideration. When he received the Order
Branch 121 issued an order[5] directing the parties to file their denying his aforesaid motion on October 3, 1994, he had one
respective memoranda. more day left to elevate his case to this Court by the proper
mode of appeal, which is by petition for review. Yao, however,
On 4 July 1994 YAO filed his Appeal Memorandum.[6] on October 4, 1994, filed a notice of appeal by registered mail
informing the RTC that he is appealing his conviction to the
Court of Appeals. By then, the fifteen (15) day period had
Without waiting for the Memorandum on Appeal of the already elapsed.
prosecution, which was filed only on 20 August 1994,[7] Judge
Adoracion Angeles rendered on 27 July 1994 a one-page
Decision[8] which affirmed in toto the MeTC decision. In so That notwithstanding, the Branch Clerk of Court, RTC, Branch
doing, she merely quoted the dispositive portion of the MeTC 121, transmitted to this Court the entire records of the case,
and stated that [a]after going over the evidence on record, the thru a transmittal letter, dated October 13, 1994, and received
Court finds no cogent reason to disturb the findings of the by the Criminal Section of this Court on October 28, 1994.
Metropolitan Trial Court. YAOs counsel, on February 20, 1995, filed with this Court, a
motion for extension of period to file brief for accused-
appellant which was granted in Our resolution mentioned in
YAO filed a motion for reconsideration[9] and assailed the the opening paragraph of this resolution.
decision as violative of Section 2, Rule 20 of the Rules of
Court.[10] In its order[11] of 28 September 1994, the RTC
denied the motion for reconsideration as devoid of merit and Petitions for review shall be filed within the period to appeal.
reiterated that the findings of the trial court are entitled to great This period has already elapsed even when Yao filed a notice
weight on appeal and should not be disturbed on appeal of appeal by registered mail, with the RTC of Kalookan City.
unless for strong and cogent reasons. Worse, the notice of appeal is procedurally infirm.

On 4 October 1994, YAO appealed to the Court of Appeals by YAO filed an Urgent Motion to Set Aside Entry of Judgment
filing a notice of appeal.[12] contending that the 25 April 1995 resolution did not specifically
dismiss the appeal, for which reason, there was no
judgmenton which an entry of judgment could be issued. He
The appealed case was docketed as CA-G.R. CR No. 16893. also argued that the attendant procedural infirmities in the
In its Resolution[13] of 28 February 1995, the Court of Appeals appeal, if any, were cured with the issuance of the 28
granted YAO an extension of twenty (20) days from 10 February 1995 resolution granting him twenty (20) days from
February or until 12 March 1995 within which to file the 10 February 1995 or until 12 March 1995 within which to file
Appellants Brief. However, on 25 April 1995 the Court of an appellants brief and in compliance thereto, consequently
Appeals promulgated a Resolution[14] declaring that [t]he filed his appellants brief on 2 March 1995.[15]
decision rendered on July 27, 1994 by the Regional Trial
Court, Branch 121, has long become final and executory and
ordering the records of the case remanded to said court for In its Resolution[16] of 26 January 1998, the Court of Appeals
the proper execution of judgment. The pertinent portion of the denied the Urgent Motion to Set Aside the Entry of Judgment
Resolution reads: for lack of merit. It considered the 25 April 1995 resolution as
having in effect dismissed the appeal, [hence] the Entry of
Judgment issued on May 26, 1995... was proper.
In Our resolution, dated February 28, 1995, accused-
appellant was granted an extension of twenty (20) days from
February 10, 1995, or until March 12, 1995 within which to file In this petition for review on certiorari, YAO reiterates the
appellants brief. arguments he raised in his Urgent Motion to Set Aside the
Entry of Judgment of the Court of Appeals, thus: (1) that the
entry of judgment was improvidently issued in the absence of
To date, no appellants brief has been filed. a final resolution specifically dismissing the appeal; (2) the
procedural infirmity in the appeal, if any, has been cured; and
FINALS CONSTITUTIONAL LAW I ACJUCO NOV 11, 2017 106

(3) the Court of Appeals committed grave abuse of discretion of the RTCs order denying the motion for reconsideration on
amounting to lack of jurisdiction in denying him (YAO) due 3 October 1994. He had therefore, only one day left, 4 October
process of law. 1994 as the last day, within which to file with the Court of
Appeals a petition for review.[18]However, on said date, YAO
In support of his first argument, YAO cites Section 1, Rule 11 filed a notice of appeal. He palpably availed of the wrong
of the Revised Internal Rules of the Court of Appeals, thus: mode of appeal. And since he never instituted the correct one,
he lost it.
SEC. 1. Entry of Judgment. -- Unless a motion for
reconsideration is filed or an appeal is taken to the Supreme The right to appeal is not a constitutional, natural or inherent
Court, judgments and final resolutions of the Court of Appeals right. It is a statutory privilege of statutory origin and, therefore,
shall be entered upon the expiration of fifteen (15) days after available only if granted or provided by statute.[19] Since the
notice to parties. right to appeal is not a natural right nor a part of due process,
it may be exercised only in the manner and in accordance with
the provisions of law.[20] Corollarily, its requirements must be
YAO claims that the 25 April 1995 resolution of the Court of strictly complied with.
Appeals was not a judgment on his appeal nor was it a final
resolution contemplated in the Internal Rules since it did not
specifically dismiss his appeal. A fortiori, the entry of judgment That an appeal must be perfected in the manner and within
was improvidently issued for lack of legal basis. the period fixed by law is not only mandatory but
jurisdictional.[21] Non-compliance with such legal
requirements is fatal,[22]for it renders the decision sought to
YAO also repeats his argument that any procedural infirmity be appealed final and executory,[23] with the end result that
in the appeal was cured when the RTC gave due course to no court can exercise appellate jurisdiction to review the
the appeal, elevated the records to the Court of Appeals which decision.[24]
in turn issued on 13 December 1994 a notice to file his
Appellants Brief and granted him until 12 March 1995 within
which to file the appellants brief. In the light of these procedural precepts, YAOs petition
appears to be patently without merit and does not deserve a
second look. Hence, the reasons he enumerated to persuade
Finally, YAO asserts that he was denied due process this Court to grant his petition and reinstate his appeal are
considering that (1) none of the elements of unfair competition obviously frivolous if not downright trivial. They need not even
are present in this case; (2) he filed his appeal to the Court of be discussed here.
Appeals within the reglementary period; and (3)
notwithstanding his filing of a notice of appeal (instead of a
petition for review), it was a mere procedural lapse, a In the normal and natural course of events, we should dismiss
technicality which should not bar the determination of the case the petition outright, if not for an important detail which augurs
based on intrinsic merits. YAO then invokes the plethora of well for YAO and would grant him a reprieve in his legal battle.
jurisprudence wherein the Supreme Court in the exercise of The decision of the RTC affirming the conviction of YAO
equity jurisdiction decided to disregard technicalities; decided palpably transgressed Section 14, Article VIII of the
[the case] on merits and not on technicalities; found manifest Constitution, which states:
in the petition strong considerations of substantial justice
necessitating the relaxing of the stringent application of Sec. 14. No decision shall be rendered by any court without
technical rules, or heeded petitioners cry for justice because expressing therein clearly and distinctly the facts and the law
the basic merits of the case warrant so, as where the petition on which it is based.
embodies justifying circumstances; discerned not to sacrifice
justice to technicality; discovered that the application of res xxx
judicata and estoppel by judgment amount to a denial of
justice and or a bar to a vindication of a legitimate
grievance.[17] Let us quote in full the RTC judgment:

In its Comment, the Office of the Solicitor General prays that This is an appeal from the decision of the Metropolitan Trial
the petition should be dismissed for lack of merit. It maintains Court, Branch 52, Kalookan City, in Crim. Case No. C-155713,
that although the 25 April 1995 resolution did not specifically the dispositive portion of which reads as follows:
state that the appeal was being dismissed, the intent and
import are clear and unequivocable. It asserts that the appeal xxx
was obviously dismissed because the RTC decision has long
become final and executory. YAO failed to challenge the RTC But because the prosecution proved the guilt of the other
decision, within the reglementary period, by filing a petition for accused, George Yao; beyond reasonable doubt as principal
review of the same with the Court of appeals pursuant to under the said Article 189 (1) for Unfair Competition, he is
Section 1 of Rule 42 of the Rules of Court. Instead, he filed an convicted of the same. In the absence of any aggravating or
ordinary appeal by way of a notice of appeal. Hence, the mitigating circumstances alleged/proven, and considering the
period to file the correct procedural remedy had lapsed. provisions of the Indeterminate Sentence Law, he is
sentenced to a minimum of four (4) months and twenty-one
There is no dispute that YAO availed of the wrong procedural (21) days of arresto mayor to a maximum of one (1) year and
remedy in assailing the RTC decision. It is clear from the five (5) months of prision correccional.
records that YAO received a copy of the adverse RTC
judgment on 16 August 1994. He has fifteen (15) days or until xxx
31 August 1994 within which to file either a motion for
reconsideration or a petition for review with the Court of
Appeals. Fourteen (14) days thereafter or on 30 August 1994, After going over the evidence on record, the Court finds no
YAO opted to file a motion for reconsideration the pendency cogent reason to disturb the findings of the Metropolitan Trial
of which tolled the running of the period. He received a copy Court.
FINALS CONSTITUTIONAL LAW I ACJUCO NOV 11, 2017 107

WHEREFORE, this Court affirms in toto the decision of the only, such as ordinary collection cases, where the appeal is
Metropolitan Trial Court dated October 20, 1993. obviously groundless and deserves no more than the time
needed to dismiss it.
SO ORDERED.
xxx
That is all there is to it.
Henceforth, all memorandum decisions shall comply with the
We have sustained decisions of lower courts as having requirements herein set forth both as to the form prescribed
substantially or sufficiently complied with the constitutional and the occasions when they may be rendered. Any deviation
injunction notwithstanding the laconic and terse manner in will summon the strict enforcement of Article VIII, Section 14
which they were written and even if there (was left) much to of the Constitution and strike down the flawed judgment as a
be desired in terms of (their) clarity, coherence and lawless disobedience.
comprehensibility provided that they eventually set out the
facts and the law on which they were based,[25] as when they Tested against these standards, we find that the RTC decision
stated the legal qualifications of the offense constituted by the at bar miserably failed to meet them and, therefore, fell short
facts proved, the modifying circumstances, the participation of of the constitutional injunction. The RTC decision is brief
the accused, the penalty imposed and the civil liability;[26] or indeed, but it is starkly hallow, otiosely written, vacuous in its
discussed the facts comprising the elements of the offense content and trite in its form. It achieved nothing and attempted
that was charged in the information, and accordingly rendered at nothing, not even at a simple summation of facts which
a verdict and imposed the corresponding penalty;[27] or could easily be done. Its inadequacy speaks for itself.
quoted the facts narrated in the prosecutions memorandum
but made their own findings and assessment of evidence, We cannot even consider or affirm said RTC decision as a
before finally agreeing with the prosecutions evaluation of the memorandum decision because it failed to comply with the
case.[28] measures of validity laid down in Francisco v. Permskul. It
merely affirmed in toto the MeTC decision without saying
We have also sanctioned the use of memorandum more. A decision or resolution, especially one resolving an
decisions,[29] a specie of succinctly written decisions by appeal, should directly meet the issues for resolution;
appellate courts in accordance with the provisions of Section otherwise, the appeal would be pointless.[33]
40, B.P. Blg. 129[30] on the grounds of expediency,
practicality, convenience and docket status of our courts. We We therefore reiterate our admonition in Nicos Industrial
have also declared that memorandum decisions comply with Corporation v. Court of Appeals,[34] in that while we conceded
the constitutional mandate.[31] that brevity in the writing of decisions is an admirable trait, it
should not and cannot be substituted for substance; and again
In Francisco v. Permskul,[32] however, we laid down the in Francisco v. Permskul,[35] where we cautioned that
conditions for the of validity of memorandum decisions, thus: expediency alone, no matter how compelling, cannot excuse
non-compliance with the constitutional requirements.
The memorandum decision, to be valid, cannot incorporate
the findings of fact and the conclusions of law of the lower This is not to discourage the lower courts to write abbreviated
court only by remote reference, which is to say that the and concise decisions, but never at the expense of scholarly
challenged decision is not easily and immediately available to analysis, and more significantly, of justice and fair play, lest
the person reading the memorandum decision. For the the fears expressed by Justice Feria as the ponente in
incorporation by reference to be allowed, it must provide for Romero v. Court of Appeals[36] come true, i.e., if an appellate
direct access to the facts and the law being adopted, which court failed to provide the appeal the attention it rightfully
must be contained in a statement attached to the said deserved, said court deprived the appellant of due process
decision. In other words, the memorandum decision since he was not accorded a fair opportunity to be heard by a
authorized under Section 40 of B.P. Blg. 129 should actually fair and responsible magistrate. This situation becomes more
embody the findings of fact and conclusions of law of the lower ominous in criminal cases, as in this case, where not only
court in an annex attached to and made an indispensable part property rights are at stake but also the liberty if not the life of
of the decision. a human being.

It is expected that this requirement will allay the suspicion that Faithful adherence to the requirements of Section 14, Article
no study was made of the decision of the lower court and that VIII of the Constitution is indisputably a paramount component
its decision was merely affirmed without a proper examination of due process and fair play.[37] It is likewise demanded by
of the facts and the law on which it is based. The proximity at the due process clause of the Constitution.[38] The parties to
least of the annexed statement should suggest that such an a litigation should be informed of how it was decided, with an
examination has been undertaken. It is, of course, also explanation of the factual and legal reasons that led to the
understood that the decision being adopted should, to begin conclusions of the court. The court cannot simply say that
with, comply with Article VIII, Section 14 as no amount of judgment is rendered in favor of X and against Y and just leave
incorporation or adoption will rectify its violation. it at that without any justification whatsoever for its action. The
losing party is entitled to know why he lost, so he may appeal
The Court finds necessary to emphasize that the to the higher court, if permitted, should he believe that the
memorandum decision should be sparingly used lest it decision should be reversed. A decision that does not clearly
become an addictive excuse for judicial sloth. It is an and distinctly state the facts and the law on which it is based
additional condition for the validity that this kind of decision leaves the parties in the dark as to how it was reached and is
may be resorted to only in cases where the facts are in the precisely prejudicial to the losing party, who is unable to
main accepted by both parties and easily determinable by the pinpoint the possible errors of the court for review by a higher
judge and there are no doctrinal complications involved that tribunal.[39] More than that, the requirement is an assurance
will require an extended discussion of the laws involved. The to the parties that, in reaching judgment, the judge did so
memorandum decision may be employed in simple litigations through the processes of legal reasoning. It is, thus, a
FINALS CONSTITUTIONAL LAW I ACJUCO NOV 11, 2017 108

safeguard against the impetuosity of the judge, preventing him ASIDE and the 27 July 1994 decision of the Regional Trial
from deciding ipse dixit. Vouchsafed neither the sword nor the Court, Branch 121 of Kalookan City rendered in its appellate
purse by the Constitution but nonetheless vested with the jurisdiction is NULLIFIED. The records are hereby remanded
sovereign prerogative of passing judgment on the life, liberty to said Regional Trial Court for further proceedings and for the
or property of his fellowmen, the judge must ultimately depend rendition of judgment in accordance with the mandate of
on the power of reason for sustained public confidence in the Section 14, Article VIII of the Constitution.
justness of his decision.[40]
No costs.
Thus the Court has struck down as void, decisions of lower
courts and even of the Court of Appeals whose careless SO ORDERED.
disregard of the constitutional behest exposed their
sometimes cavalier attitude not only to their magisterial
responsibilities but likewise to their avowed fealty to the Puno, Pardo, and Ynares-Santiago, JJ., concur.
Constitution.
Kapunan, J., on leave.
Thus, we nullified or deemed to have failed to comply with
Section 14, Article VIII of the Constitution, a decision,
resolution or order which: contained no analysis of the
evidence of the parties nor reference to any legal basis in
reaching its conclusions; contained nothing more than a
summary of the testimonies of the witnesses of both
parties;[41] convicted the accused of libel but failed to cite any
legal authority or principle to support conclusions that the
letter in question was libelous;[42] consisted merely of one (1)
paragraph with mostly sweeping generalizations and failed to
support its conclusion of parricide;[43] consisted of five (5)
pages, three (3) pages of which were quotations from the
labor arbiters decision including the dispositive portion and
barely a page (two [2] short paragraphs of two [2] sentences
each) of its own discussion or reasonings[44]; was merely
based on the findings of another court sans transcript of
stenographic notes;[45] or failed to explain the factual and
legal bases for the award of moral damages.[46]

In the same vein do we strike down as a nullity the RTC


decision in question.

In sum, we agree with YAO that he was denied due process


but not on the grounds he ardently invoked but on the reasons
already extensively discussed above. While he indeed
resorted to the wrong mode of appeal and his right to appeal
is statutory, it is still an essential part of the judicial system that
courts should proceed with caution so as not to deprive a party
of the prerogative, but instead afford every party-litigant the
amplest opportunity for the proper and just disposition of his
cause, freed from the constraints of technicalities.[47]

In the interest of substantial justice, procedural rules of the


most mandatory character in terms of compliance, may be
relaxed.[48] In other words, if strict adherence to the letter of
the law would result in absurdity and manifest injustice[49] or
where the merit of a partys cause is apparent and outweighs
consideration of non-compliance with certain formal
requirements,[50]procedural rules should definitely be
liberally construed. A party-litigant is to be given the fullest
opportunity to establish the merits of his complaint or defense
rather than for him to lose life, liberty, honor or property on
mere technicalities.[51] We therefore withhold legal
approbation on the RTC decision at bar for its palpable failure
to comply with the constitutional and legal mandates thereby
denying YAO of his day in court. We also remind all
magistrates to heed the demand of Section 14, Article VIII of
the Constitution. It is their solemn and paramount duty to
uphold the Constitution and the principles enshrined therein,
lest they be lost in the nitty-gritty of their everyday judicial
work.

WHEREFORE, in view of all the foregoing, the petition in this


case is GRANTED. The questioned 25 April 1995 resolution
of the Court of Appeals in CA-G.R. No. 16893 is hereby SET
FINALS CONSTITUTIONAL LAW I ACJUCO NOV 11, 2017 109

[A.M. No. RTJ-96-1338. September 5, 1997] Court for re-raffle. The case was eventually assigned to Judge
Manuel F. Dumatol of Branch 113 of the Pasay City RTC.
ENGINEER FERNANDO S. DIZON, complainant, vs.
JUDGE LILIA C. LOPEZ, Regional Trial Court, Branch 109, Judge Lopez claims that on April 22, 1993, when the judgment
Pasay City, respondent. was promulgated with the reading of the dispositive portion,
her decision was already prepared, although to prevent
DECISION leakage in the process of preparing it, she withheld its
dispositive portion until the day of its promulgation.
Respondent judge states that after the dispositive portion had
MENDOZA, J.: been read to complainant, respondent gave it to Ma. Cleotilde
Paulo (Social Worker II, presently OIC of Branch 109) for
This is a complaint charging Judge Lilia C. Lopez of the typing and incorporation into the text of the decision. The court
Regional Trial Court, Branch 109, Pasay City, with violation of found complainant guilty beyond reasonable doubt of
the Constitution, serious misconduct, inefficiency, and falsification of private document under Art. 172, par. 2 of the
falsification in connection with her decision in Criminal Case Revised Penal Code. Respondent states that the delay in
No. 91-0716 entitled People of the Philippines v. Engineer furnishing complainant with a copy of the decision was
Fernando S. Dizon. unintentional.

It appears that on April 22, 1993, judgment was rendered, Respondent judge referred to difficulties she had in preparing
convicting complainant of falsification of private document. her decision and to a series of personal problems which
The promulgation of the judgment consisted of reading the contributed to this delay in the release of her decision, to wit:
dispositive portion of the decision sentencing him to she has only two (2) stenographers to attend to daily trials in
imprisonment, without serving a copy of the decision on him. her court, making it necessary for her to make use of the
The accused and his counsel were told to return in a few days Social Worker assigned to her to type her decisions. During
for their copy of the decision, but although petitioner and his the period January to December 1993 she had to dispose of
father by turns went to the court to obtain a copy of the 285 cases, apart from the fact that there was an unusually big
decision they were not able to do so. To protect his right, number of criminal, civil, and land registration cases as well
complainant filed a partial motion for reconsideration on May as special proceedings filed in her court which required the
5, 1993, expressly reserving his right to submit a more holding of hearings in the mornings and in the afternoons.
elaborate one upon receipt of the decision. The hearing of the During the same period, she went through some personal
motion for reconsideration was scheduled on May 12, 1993, tragedies. She lost her niece, Gloria Lopez Roque, whom she
but the case was not called as complainants counsel was told had raised from childhood, due to a hospital accident. This
that the decision had not yet been finished. On November 29, was followed by the death on March 1, 1992 of her mother,
1994, complainant filed an Omnibus Motion to Annul Margarita Lopez, who had been under respondents care for
Promulgation of Sentence and to Dismiss the case. On the past eight years after suffering a stroke. On September
December 16, 1994, the date set for hearing the motion, 17, 1993, respondents father died of diabetes, renal failure,
complainant was served a copy of the decision, dated April pneumonia, and cardiac arrest.Respondent was the one who
22, 1993, the dispositive portion of which states: single-handedly brought them in and out of the hospital
because all her able-bodied relatives are abroad. Respondent
In view of all the foregoing, the Court finds the accused herself was found to be suffering from diabetes and
Fernando Dizon guilty beyond reasonable doubt of the crime hypertension, necessitating her treatment and leave of
of Falsification of Private Document as defined and penalized absence from September 27, 1994 to December 12, 1994, in
under Art. 172, par. 2 in relation to Art. 171 par. 2 and 4 thereof addition to her other leaves of absence. Aside from these,
and hereby sentences him to imprisonment of Two (2) Years, respondents family suffered financial reverses because of
Four (4) Months and One (1) Day to Six (6) Years and a fine estafa committed against them.
of P5,000.00.
On February 19, 1996, Deputy Court Administrator Bernardo
Complainant alleges that the failure of respondent judge to P. Abesamis submitted a memorandum, finding the charge of
furnish him a copy of the decision until almost one year and violation of the Constitution to be without merit. He called
eight months after the promulgation of its dispositive portion attention to the written decision of respondent judge, which,
on April 22, 1993 constitutes a violation of Art. VIII, 14 of the albeit delivered to complainant late, nonetheless states the
Constitution which prohibits courts from rendering decisions facts and law on which it is based. He likewise finds the charge
without expressing therein clearly and distinctly the facts and of serious misconduct and falsification to be without basis in
law on which they are based and 15 of the same Art. VIII, view of the absence of malice. However, he finds the charge
which provides that in all cases lower courts must render their of inefficiency to be well founded on the basis of respondents
decisions within three months from the date of their failure to furnish complainant or his counsel a copy of the
submission. He alleges further that he was denied the right to decision within a reasonable time after its promulgation.
a speedy trial in violation of Art. III, 14(2) of the Constitution Hence, the Deputy Court Administrator believes that Judge
and that Judge Lopez falsified her decision by antedating it Lopez should be given admonition for her negligence, but
and including therein, as additional penalty, a fine of recommends that the other charges against her for violation
P5,000.00. of the Constitution, serious misconduct, and falsification be
dismissed for lack of merit.

On December 26, 1994, complainant filed another motion for


reconsideration after receiving a copy of the full decision of The Court finds that respondent violated Art. VIII, 15(1) of the
the court. On January 3, 1995, he moved to disqualify Constitution which provides:
respondent from hearing the motions for reconsideration
which he had filed. Respondent judge responded by All cases or matters filed after the effectivity of this
voluntarily inhibiting herself from further consideration of the Constitution must be decided or resolved within twenty-four
case and ordered it forwarded to the Office of the Clerk of months from date of submission for the Supreme Court, and,
unless reduced by the Supreme Court, twelve months for all
FINALS CONSTITUTIONAL LAW I ACJUCO NOV 11, 2017 110

lower collegiate courts and three months for all other lower Abay, Sr. v. Garcia.[2] Respondent only succeeds in showing
courts. that the judgment promulgated on April 22, 1993 was a sin
perjuicio judgment which was incomplete and needed a
Although respondent judge promulgated her decision within statement of the facts and law upon which the judgment was
three months of the submission of the case for decision, the based. As early as 1923, this Court already expressed its
fact is that only the dispositive portion was read at such disapproval of the practice of rendering sin perjuicio
promulgation. She claims that on April 22, 1993 the text of her judgments, what with all the uncertainties entailed because of
decision, containing her findings and discussion of the implied reservation that it is subject to modification when
complainants liability, had already been prepared although it the decision is finally rendered.[3] This Court has expressed
had to be put in final form by incorporating the dispositive approval of the practice of some judges of withholding the
portion. However, the fact is that it took a year and eight dispositive portion from their opinions until the very last
months more before this was done and a copy of the complete moment of promulgation of their judgment in order to prevent
decision furnished the complainant on December 16, 1994. leakage,[4] but that refers to the preparation of their decision,
Rule 120 of the Rules on Criminal Procedure provides: not its promulgation. What must be promulgated must be the
complete decision. There would be no more reason to keep
the dispositive portion a secret at the stage of promulgation of
1. Judgment defined. - The term judgment as used in this Rule judgment.
means the adjudication by the court that the accused is guilty
or is not guilty of the offense charged, and the imposition of
the proper penalty and civil liability provided for by law on the However, the Court finds the other charges against
accused. respondent to be without merit.

2. Form and contents of judgment. - The judgment must be First, the claim that complainant was deprived of his right to a
written in the official language, personally and directly speedy trial by reason of respondents failure to furnish him
prepared by the judge and signed by him and shall contain with a copy of the decision until after one year and eight
clearly and distinctly a statement of the facts proved or months is without basis. It appears that despite the destruction
admitted by the accused and the law upon which the judgment of records by fire in the Pasay City Hall on January 18, 1992
is based. the parties were required to submit simultaneously their
memoranda on August 18, 1992. The delay, if any, was not
such vexatious, capricious, and oppressive delay[5] as to
6. Promulgation of judgment. - The judgment is promulgated justify finding a denial of the right to a speedy trial. The fact is
by reading the same in the presence of the accused and any that the reading of the sentence on April 22, 1993, albeit not
judge of the court in which it was rendered. However, if the in compliance with the requirement for promulgation of
conviction is for a light offense, the judgment may be judgments, nonetheless put an end to trial.
pronounced in the presence of his counsel or representative.
When the judge is absent or outside of the province or city,
the judgment may be promulgated by the clerk of court. Second, the delay in furnishing complainant a copy of the
complete decision did not prejudice his right to appeal or file a
motion for reconsideration. It is true that an accused must be
It is clear that merely reading the dispositive portion of the given a copy of the decision in order to apprise him of the basis
decision to the accused is not sufficient. It is the judgment that of such decision so that he can intelligently prepare his appeal
must be read to him, stating the facts and the law on which or motion for reconsideration. However, in accordance with
such judgment is based. Since this was done only on the ruling in Director of Lands v. Sanz,[6] complainants period
December 16, 1994 when a copy of the complete decision to appeal or file a motion for reconsideration did not begin to
was served on complainant, it is obvious that the respondent run until after he actually received a copy of the judgment on
failed to render her decision within three months as required December 16, 1994. He therefore suffered no prejudice. If at
by Art. VIII, 15 of the Constitution. all, complainant suffered from the anxiety to refute a
conviction which he could not do for lack of a statement of the
If indeed all that had to be done after the dispositive portion basis of the conviction.
had been read in open court on April 22, 1993 was to
incorporate it in the text of the decision allegedly then already Nonetheless, certain factors mitigate respondent judges
prepared, it is difficult to see why it took respondent judge one culpability. Except for this incident, respondents record of
year and eight more months before she was able to do so. public service as legal officer and agent of the National Bureau
Respondent claims that she was prevented from putting out of Investigation, as State Prosecutor, and later Senior State
her decision by a series of personal and other problems which Prosecutor, of the Department of Justice for 17 years and as
leads the Court to believe that when she promulgated her Regional Trial Judge for more than 13 years now is unmarred
sentence she had not finished the preparation of the entire by malfeasance, misfeasance or wrongdoing. This is the first
decision. At all events, she could have applied for extension time she is required to answer an administrative complaint
of time to decide the case and put off the promulgation of against her. Her failure to decide the case of complainant was
judgment until she had finished it. brought about by factors not within her control, to wit, lack of
stenographers and unusually big number of cases; and her
What respondent did in this case was to render what is known personal loss as a result of the death of her niece and both
as a sin perjuicio judgment, which is a judgment without a her parents, financial reverses of the family, and poor health
statement of the facts in support of its conclusion to be later as a result of diabetes and hypertension.
supplemented by the final judgment.[1] That is why, in answer
to complainants charge that the dispositive portion of the In Mangulabnan v. Tecson,[7] a joint decision in two criminal
judgment read to him did not impose a fine, respondent cases was rendered by respondent judge on February 24,
contends that the addition of the fine of P5,000.00 was within 1978, six months and eight days from submission of the case,
her power to do even if no such fine had been included in the and a copy was delivered to complainant on September 28,
oral sentence given on April 22, 1993. As respondent judge 1979, over 19 months after rendition of the decision. Two
states, because the decision was not complete it could be complaints were filed for violation of the constitutional
modified and cites in support of her contention the case of provision requiring submitted cases to be decided by lower
FINALS CONSTITUTIONAL LAW I ACJUCO NOV 11, 2017 111

courts within three months and for violation of complainants On September 13, 1985, the High Court of Malaya
right to a speedy trial. Respondent judge blamed the delay in (Commercial Division) rendered judgment in favor of the
deciding the cases on the fact that his clerks had misfiled the petitioner and against the private respondent which is also
records. As to the delay in furnishing complainant with a copy designated therein as the "2nd Defendant. "
of the decision, the judge attributed this to the mistake of his
clerk who did not think complainant was entitled to receive the The judgment reads in full:
same. The judge was reprimanded. The reason for the delay
in that case was even less excusable than the difficulties
experienced by respondent Judge Lopez, i.e., deaths in SUIT NO. C638 of 1983
respondents family, her own poor state of health, financial
reverses suffered by her family, and the volume of work done Between
within the period in question, which somewhat mitigate her
liability. The Court believes that a similar penalty would be Asiavest Merchant Bankers (M) Berhad Plaintiffs
appropriate.

And
In view of the foregoing, respondent is hereby
REPRIMANDED with WARNING that repetition of the same
acts complained of will be dealt with more severely. 1. Asiavest -CDCP Sdn. Bhd. Defendant

SO ORDERED. 2. Construction & Development Corporation of the Philippines

Romero, Puno, and Torres, Jr. JJ., concur.


JUDGMENT
Regalado, J., (Chairman), on leave.
The 2nd Defendant having entered appearance herein and
the Court having under Order 14, rule 3 ordered that judgment
as hereinafter provided be entered for the Plaintiffs against the
2nd Defendant.
G.R. No. 110263 July 20, 2001
IT IS THIS DAY ADJUDGED that the 2nd defendant do pay
ASIAVEST MERCHANT BANKERS (M) BERHAD, petitioner, the Plaintiffs the sum of $5, 108,290.23 (Ringgit Five million
one hundred and eight thousand two hundred and ninety and
vs. Sen twenty-three) together with interest at the rate of 12% per
annum on
COURT OF APPEALS and PHILIPPINE NATIONAL
CONSTRUCTION CORPORATION, respondents. (i) the sum of $2,586,866.91 from the 2nd day of March 1983
to the date of payment; and
DELEON, JR., J.:
(ii) the sum of $2,521,423.32 from the 11th day of March 1983
to the date of payment; and $350.00 (Ringgit Three Hundred
Before us is a petition for review on certiorari of the Decision1 and Fifty) costs.
of the Court of Appeals dated May 19,1993 in CA-G.R. CY No.
35871 affirming the Decision2 dated October 14,1991 of the
Regional Trial Court of Pasig, Metro Manila, Branch 168 in Dated the 13th day of September, 1985.
Civil Case No. 56368 which dismissed the complaint of
petitioner Asiavest Merchant Bankers (M) Berhad for the Senior Assistant Registrar, High Court, Kuala Lumpur
enforcement of the money of the judgment of the High Court
of Malaysia in Kuala Lumpur against private respondent This Judgment is filed by Messrs. Skrine & Co., 3rd Floor,
Philippine National Construction Corporation.1âwphi1.nêt Straits Trading Building, No.4, Leboh Pasar, Besar, Kuala
Lumpur, Solicitors for the Plaintiffs abovenamed.
The petitioner Asiavest Merchant Bankers (M) Berhad is a (VP/Ong/81194.7/83)4
corporation organized under the laws of Malaysia while private
respondent Philippine National Construction Corporation is a On the same day, September 13, 1985, the High Court of
corporation duly incorporated and existing under Philippine Malaya issued an Order directing the private respondent (also
laws. designated therein as the "2nd Defendant") to pay petitioner
interest on the sums covered by the said Judgment, thus:
It appears that sometime in 1983, petitioner initiated a suit for
collection against private respondent, then known as
Construction and Development Corporation of the Philippines,
before the High Court of Malaya in Kuala Lumpur entitled
"Asiavest Merchant Bankers (M) Berhad v. Asiavest CDCP SUIT NO. C638 of 1983
Sdn. Bhd. and Construction and Development Corporation of
the Philippines."3 Between

Petitioner sought to recover the indemnity of the performance Asiavest Merchant Bankers (M) Berhad Plaintiffs
bond it had put up in favor of private respondent to guarantee
the completion of the Felda Project and the nonpayment of the
And
loan it extended to Asiavest-CDCP Sdn. Bhd. for the
completion of Paloh Hanai and Kuantan By Pass; Project.
FINALS CONSTITUTIONAL LAW I ACJUCO NOV 11, 2017 112

1. Asiavest -CDCP Sdn. Bhd. Defendants On May 22, 1989, private respondent filed its Answer with
Compulsory Counter claim's10 and therein raised the grounds
2. Construction & Development Corporation of the Philippines it brought up in its motion to dismiss. In its Reply filed11 on
June 8, 1989, the petitioner contended that the High Court of
Malaya acquired jurisdiction over the Person of private
respondent by its voluntary submission the court's jurisdiction
BEFORE THE SENIOR ASSISTANT REGISTRAR through its appointed counsel, Mr. Khay Chay Tee.
Furthermore, private respondent's counsel waived any and all
CIK SUSILA S. PARAM THIS 13th DAY OF SEPTEMBER objections to the High Court's jurisdiction in a pleading filed
1985 IN CHAMBERS before the court.

ORDER In due time, the trial court rendered its Decision dated October
14, 1991 dismissing petitioner's complaint. Petitioner
Upon the application of Asiavest Merchant Bankers (M) interposed an appeal with the Court of Appeals, but the
Berhad, the Plaintiffs in this action AND UPON READINGthe appellate court dismissed the same and affirmed the decision
Summons in Chambers dated the 16th day of August, 1984 of the trial court in a Decision dated May 19, 1993.
and the Affidavit of Lee Foong Mee affirmed on the 14th day
of August 1984 both filed herein AND UPON HEARING Mr. T. Hence, the instant Petition which is anchored on two (2)
Thomas of Counsel for the Plaintiffs and Mr. Khaw Chay Tee assigned errors,12 to wit:
of Counsel for the 2nd Defendant abovenamed on the 26th
day of December 1984 IT WAS ORDERED that the Plaintiffs I
be at liberty to sign final judgment against the 2nd Defendant
for the sum of $5,108,290.23 AND IT WAS ORDERED that
the 2nd Defendant do pay the Plaintiffs the costs of suit at THE COURT OF APPEALS ERRED IN HOLDING THAT THE
$350.00 AND IT WAS FURTHER ORDERED that the plaintiffs MALAYSIAN COURT DID NOT ACQUIRE PERSONAL
be at liberty to apply for payment of interest AND upon the JURISDICTION OVER PNCC, NOTWITHSTANDING THAT
application of the Plaintiffs for payment of interest coming on (a) THE FOREIGN COURT HAD SERVED SUMMONS ON
for hearing on the 1st day of August in the presence of Mr. PNCC AT ITS MALAYSlA OFFICE, AND (b) PNCC ITSELF
Palpanaban Devarajoo of Counsel for the Plaintiffs and Mr. APPEARED BY COUNSEL IN THE CASE BEFORE THAT
Khaw Chay Tee of Counsel for the 2nd Defendant above- COURT.
named AND UPON HEARING Counsel as aforesaid BY
CONSENT IT WAS ORDERED that the 2nd Defendant do pay II
the Plaintiffs interest at a rate to be assessed AND the same
coming on for assessment this day in the presence of Mr. THE COURT OF APPEALS ERRED IN DENYING
Palpanaban Devarajoo of Counsel for the Plaintiffs and Mr. RECOGNITION AND ENFORCEMENT TO (SIC) THE
Khaw Chay Tee of Counsel for the 2nd Defendant AND UPON MALAYSIAN COURT JUDGMENT.
HEARING Counsel as aforesaid BY CONSENT IT IS
ORDERED that the 2nd Defendant do pay the Plaintiffs
interest at the rate of 12% per annum on: Generally, in the absence of a special compact, no sovereign
is bound to give effect within its dominion to a judgment
rendered by a tribunal of another country;13 however, the
(i) the sum of $2,586,866.91 from the 2nd day of March 1983 rules of comity, utility and convenience of nations have
to the date of payment; and established a usage among civilized states by which final
judgments of foreign courts of competent jurisdiction are
(ii) the sum Of $2,521,423.32 from the 11th day of March 1983 reciprocally respected and rendered efficacious under certain
to the date of Payment. conditions that may vary in different countries.14

Dated the 13th day of September,1985. In this jurisdiction, a valid judgment rendered by a foreign
tribunal may be recognized insofar as the immediate parties
Senior Assistant Registrar, High Court, Kuala Lumpur.5 and the underlying cause of action are concerned so long as
it is convincingly shown that there has been an opportunity for
a full and fair hearing before a court of competent jurisdiction;
Following unsuccessful attempts6 to secure payment from that the trial upon regular proceedings has been conducted,
private respondent under the judgment, petitioner initiated on following due citation or voluntary appearance of the
September 5, 1988 the complaint before Regional Trial Court defendant and under a system of jurisprudence likely to
of Pasig, Metro Manila, to enforce the judgment of the High secure an impartial administration of justice; and that there is
Court of Malaya.7 nothing to indicate either a prejudice in court and in the system
of laws under which it is sitting or fraud in procuring the
Private respondent sought the dismissal of the case via a judgment.15
Motion to Dismiss filed on October 5, 1988, contending that
the alleged judgment of the High Court of Malaya should be A foreign judgment is presumed to be valid and binding in the
denied recognition or enforcement since on in face, it is tainted country from which it comes, until a contrary showing, on the
with want of jurisdiction, want of notice to private respondent, basis of a presumption of regularity of proceedings and the
collusion and/or fraud, and there is a clear mistake of law or giving of due notice in the foreign forum Under Section
fact.8 Dismissal was, however, denied by the trial court 50(b),16 Rule 39 of the Revised Rules of Court, which was the
considering that the grounds relied upon are not the proper governing law at the time the instant case was decided by the
grounds in a motion to dismiss under Rule 16 of the Revised trial court and respondent appellate court, a judgment, against
Rules of Court. 9 a person, of a tribunal of a foreign country having jurisdiction
to pronounce the same is presumptive evidence of a right as
between the parties and their successors in interest by a
subsequent title. The judgment may, however, be assailed by
FINALS CONSTITUTIONAL LAW I ACJUCO NOV 11, 2017 113

evidence of want of jurisdiction, want of notice to the party, (g) Statement of Claim, Writ of Summons and Affidavit of
collusion, fraud, or clear mistake of law or fact. In addition, Service of such writ in petitioner's suit against private
under Section 3(n), Rule 131 of the Revised Rules of Court, a respondent before the Malaysian High Court;30
court, whether in the Philippines or elsewhere, enjoys the
presumption that it was acting in the lawful exercise of its (h) Memorandum of Conditional Appearance dated March 28,
jurisdiction. Hence, once the authenticity of the foreign 1983 filed by counsel for private respondent with the
judgment is proved, the party attacking a foreign judgment, is Malaysian High Court;31
tasked with the burden of overcoming its presumptive validity.
(i) Summons in Chambers and Affidavit of Khaw Chay Tee,
In the instant case, petitioner sufficiently established the cotmsel for private respondent, submitted during the
existence of the money judgment of the High Court of Malaya proceedings before the Malaysian High Court;32
by the evidence it offered. Vinayak Prabhakar Pradhan,
presented as petitioner's sole witness, testified to the effect
that he is in active practice of the law profession in (j) Record of the Court's Proceedings in Civil Case No. C638
Malaysia;17 that he was connected with Skrine and Company of 1983.33
as Legal Assistant up to 1981;18 that private respondent, then
known as Construction and Development Corporation of the (k) Petitioner 's verified Application for Summary Judgment
Philippines, was sued by his client, Asiavest Merchant dated August 14, 1984;34 and
Bankers (M) Berhad, in Kuala Lumpur;19that the writ of
summons were served on March 17, 1983 at the registered (l) Letter dated November 6, 1985 from petitioner's Malaysian
office of private respondent and on March 21, 1983 on Cora Counsel to private respondent's counsel in Malaysia.35
S. Deala, a financial planning officer of private respondent for
Southeast Asia operations;20that upon the filing of the case,
Messrs. Allen and Gledhill, Advocates and Solicitors, with Having thus proven, through the foregoing evidence, the
address at 24th Floor, UMBC Building, Jalan Sulaiman, Kuala existence and authenticity of the foreign judgment, said
Lumpur, entered their conditional appearance for private foreign judgment enjoys presumptive validity and the burden
respondent questioning the regularity of the service of the writ then fell upon the party who disputes its validity, herein private
of summons but subsequently withdrew the same when it respondent, to prove otherwise.
realized that the writ was properly served;21 that because
private respondent failed to file a statement of defense within Private respondent failed to sufficiently discharge the burden
two (2) weeks, petitioner filed an application for summary that fell upon it - to prove by clear and convincing evidence
judgment and submitted affidavits and documentary evidence the grounds which it relied upon to prevent enforcement of the
in support of its claim;22 that the matter was then heard before Malaysian High Court judgment, namely, (a) that jurisdiction
the High Court of Kuala Lumpur in a series of dates where was not acquired by the Malaysian Court over the person of
private respondent was represented by counsel; 23 and that private respondent due to alleged improper service of
the end result of all these proceedings is the judgment sought summons upon private respondent and the alleged lack of
to be enforced. authority of its counsel to appear and represent private
respondent in the suit; (b) the foreign judgment is allegedly
In addition to the said testimonial evidence, petitioner offered tainted by evident collusion, fraud and clear mistake of fact or
the following documentary evidence: law; and (c) not only were the requisites for enforcement or
recognition allegedly not complied with but also that the
Malaysian judgment is allegedly contrary to the Constitutional
(a) A certified and authenticated copy of the Judgment prescription that the "every decision must state the facts and
promulgated by the Malaysian High Court dated September law on which it is based."36
13, 1985 directing private respondent to pay petitioner the
sum of $5,108,290.23 Malaysian Ringgit plus interests from
March 1983 until fully paid;24 Private respondent relied solely on the testimony of its two (2)
witnesses, namely, Mr. Alfredo. Calupitan, an accountant of
private respondent, and Virginia Abelardo, Executive
(b) A certified and authenticated copy of the Order dated Secretary and a member of the staff of the Corporate
September 13,1985 issued by the Malaysian High Court in Secretariat Section of the Corporate Legal Division, of private
Civil Suit No. C638 of 1983;25 respondent, both of whom failed to shed light and amplify its
defense or claim for non-enforcement of the foreign judgment
(c) Computation of principal and interest due as of January 31, against it.
1990 on the amount adjudged payable to petitioner by private
respondent;26 Mr. Calupitan's testimony centered on the following: that from
January to December 1982 he was assigned in Malaysia as
(d) Letter and Statement of Account of petitioner's counsel in Project Comptroller of the Pahang Project Package A and B
Malaysia indicating the costs for prosecuting and for road construction under the joint venture of private
implementing the Malaysian High Court's Judgment;27 respondent and Asiavest Holdings;37 that under the joint
venture, Asiavest Holdings would handle the financial aspect
(e) Letters between petitioner's Malaysian counsel, Skrine and of the project, which is fifty-one percent (51 %) while private
Co., and its local counsel, Sycip Salazar Law Offices, relative respondent would handle the technical aspect of the project,
to institution of the action in the Philippines;28 or forty-nine percent (49%);38 and, that Cora Deala was not
authorized to receive summons for and in behalf of the private
respondent.39 Ms. Abelardo's testimony, on the other hand,
(f) Billing Memorandum of Sycip Salazar Law Offices dated focused on the following: that there was no board resolution
January 2, 1990 showing attorney's fees paid by and due from authorizing Allen and Gledhill to admit all the claims of
petitioner; 29 petitioner in the suit brought before the High Court of
Malaya,40 though on cross-examination she admitted that
Allen and Gledhill were the retained lawyers of private
respondent in Malaysia. 41
FINALS CONSTITUTIONAL LAW I ACJUCO NOV 11, 2017 114

The foregoing reasons or grounds relied upon by private was also sought against Asiavest-CDCP Sdn. Bhd. but the
respondent in preventing enforcement and recognition of the same was found insolvent.49 Furthermore, even when the
Malaysian judgment primarily refer to matters of remedy and foreign judgment is based on the drafts prepared by counsel
procedure taken by the Malaysian High Court relative to the for the successful party, such is not per se indicative of
suit for collection initiated by petitioner. Needless to stress, the collusion or fraud. Fraud to hinder the enforcement within the
recognition to be accorded a foreign judgment is not jurisdiction of a foreign judgment must be extrinsic, i.e., fraud
necessarily affected by the fact that the procedure in the based on facts not controverted or resolved in the case where
courts of the country in which such judgment was rendered judgment is rendered,50 or that which would go to the
differs from that of the courts of the country in which the jurisdiction of the court or would deprive the party against
judgment is relied on.42 Ultimately, matters of remedy and whom judgment is rendered a chance to defend the action to
procedure such as those relating to the service of summons which he has a meritorious defense.51 Intrinsic fraud is one
or court process upon the defendant, the authority of counsel which goes to the very existence of the cause of action is
to appear and represent a defendant and the formal deemed already adjudged, and it, therefore, cannot militate
requirements in a decision are governed by the lex fori or the against the recognition or enforcement of the foreign
internal law of the forum,43 i.e., the law of Malaysia in this judgment.52 Evidence is wanting on the alleged extrinsic
case. fraud. Hence, such unsubstantiated allegation cannot give
rise to liability therein.
In this case, it is the procedural law of Malaysia where the
judgment was rendered that determines the validity of the Lastly, there is no merit to the argument that the foreign
service of court process on private respondent as well as other judgment is not enforceable in view of the absence of any
matters raised by it. As to what the Malaysian procedural law statement of facts and law upon which the award in favor of
is, remains a question of fact, not of law. It may not be taken the petitioner was based. As aforestated, the lex fori or the
judicial notice of and must be pleaded and proved like any internal law of the forum governs matters of remedy and
other fact. Sections 24 and 25 of Rule 132 of the Revised procedure.53 Considering that under the procedural rules of
Rules of Court provide that it may be evidenced by an official the High Court of Malaya, a valid judgment may be rendered
publication or by a duly attested or authenticated copy thereof. even without stating in the judgment every fact and law upon
It was then incumbent upon private respondent to present which the judgment is based, then the same must be accorded
evidence as to what that Malaysian procedural law is and to respect and the courts in the jurisdiction cannot invalidate the
show that under it, the assailed service of summons upon a judgment of the foreign court simply because our rules provide
financial officer of a corporation, as alleged by it, is invalid. It otherwise.
did not. Accordingly, the presumption of validity and regularity
of service of summons and the decision thereafter rendered All in all, private respondent had the ultimate duty to
by the High Court of Malaya must stand.44 demonstrate the alleged invalidity of such foreign judgment,
being the party challenging the judgment rendered by the High
On the matter of alleged lack of authority of the law firm of Court of Malaya. But instead of doing so, private respondent
Allen and Gledhill to represent private respondent, not only did merely argued, to which the trial court agreed, that the burden
the private respondent's witnesses admit that the said law firm lay upon petitioner to prove the validity of the money
of Allen and Gledhill were its counsels in its transactions in judgment. Such is clearly erroneous and would render
Malaysia,45 but of greater significance is the fact that meaningless the presumption of validity accorded a foreign
petitioner offered in evidence relevant Malaysian judgment were the party seeking to enforce it be required to
jurisprudence46 to the effect that (a) it is not necessary under first establish its validity.54
Malaysian law for counsel appearing before the Malaysian
High Court to submit a special power of attorney authorizing WHEREFORE, the instant petition is GRANTED. The
him to represent a client before said court, (b) that counsel Decision of the Court of Appeals dated May 19,1993 in CA-
appearing before the Malaysian High Court has full authority G.R CY No. 35871 sustaining the Decision dated October 14,
to compromise the suit, and (c) that counsel appearing before 1991 in Civil Case No. 56368 of the Regional Trial Court of
the Malaysian High Court need not comply with certain pre- Pasig, Branch 168 denying the enforcement of the Judgment
requisites as required under Philippine law to appear and dated September 13, 1985 of the High Court of Malaya in
compromise judgments on behalf of their clients before said Kuala Lumpur is REVERSED and SET ASIDE, and another in
court.47 its stead is hereby rendered ORDERINGprivate respondent
Philippine National Construction Corporation to pay petitioner
Furthermore, there is no basis for or truth to the appellate Asiavest Merchant Bankers (M) Berhad the amounts
court's conclusion that the conditional appearance of private adjudged in the said foreign Judgment, subject of the said
respondent's counsel who was allegedly not authorized to case.
appear and represent, cannot be considered as voluntary
submission to the jurisdiction of the High Court of Malaya, Costs against the private respondent.
inasmuch as said conditional appearance was not premised
on the alleged lack of authority of said counsel but the
conditional appearance was entered to question the regularity SO ORDERED.
of the service of the writ of summons. Such conditional
appearance was in fact subsequently withdrawn when
counsel realized that the writ was properly served.48

On the ground that collusion, fraud and, clear mistake of fact


and law tainted the judgment of the High Court of Malaya, no
clear evidence of the same was adduced or shown. The facts
which the trial court found "intriguing" amounted to mere
conjectures and specious observations. The trial court's
finding on the absence of judgment against Asiavest-CDCP
Sdn. Bhd. is contradicted by evidence on record that recovery
FINALS CONSTITUTIONAL LAW I ACJUCO NOV 11, 2017 115

[A. M. No. 00-8-05-SC. November 28, 2001] NOW, THEREFORE, in view of the foregoing, the Board of
Governors of the Integrated Bar of the Philippines hereby
RE: PROBLEM OF DELAYS IN CASES BEFORE THE resolves as follows:
SANDIGANBAYAN.
1. To recommend to the Supreme Court that Supreme Court
RESOLUTION Administrative Circular No. 10-94 be made applicable to the
Sandiganbayan in regard cases over which the
Sandiganbayan has original jurisdiction; and
PARDO, J.:
2. To recommend to the Supreme Court an inquiry into the
The Case causes of delay in the resolution of incidents and motions and
in the decision of cases before the Sandiganbayan for the
Submitted to the Court for consideration is a resolution of the purpose of enacting measures intended at avoiding such
Board of Governors, Integrated Bar of the Philippines delays.
(hereafter, the IBP) recommending an inquiry into the causes
of delays in the resolution of incidents and motions and in the Done in Los Baos, Laguna, this 29th day of July, 2000.
decision of cases pending before the Sandiganbayan.
On August 8, 2000, the Court required Sandiganbayan
The Antecedents Presiding Justice Francis E. Garchitorena to comment on the
letter of the IBP and to submit a list of all Sandiganbayan
On July 31, 2000, the IBP, through its National President, cases pending decision, or with motion for reconsideration
Arthur D. Lim, transmitted to the Court a Resolution[1] pending resolution, indicating the dates they were deemed
addressing the problem of delays in cases pending before the submitted for decision or resolution.[4]
Sandiganbayan (hereafter, the Resolution).[2] We quote the
Resolution in full:[3] On September 27, 2000, complying with the order, Presiding
Justice Francis E. Garchitorena submitted a report[5]
WHEREAS, Section 16, Article III of the Constitution (hereafter, the compliance) admitting a number of cases
guarantees that, [a]ll persons shall have the right to a speedy submitted for decision and motion for reconsideration pending
disposition of their cases before all judicial, quasi-judicial, or resolution before its divisions. We quote:
administrative bodies,
Cases Submitted W/ Motions For
WHEREAS, Canon 12 of the Code of Professional
Responsibility for Lawyers mandates that [a] lawyer shall exert For Decision Reconsideration
every effort and consider it his duty to assist in the speedy and
efficient administration of justice;
1st Division 341 None
WHEREAS, it is the duty of the Integrated Bar of the
Philippines to undertake measures to assist in the speedy 2nd Division 5 None
disposition of cases pending before the various courts and
tribunals; 3rd Division 12 None

WHEREAS, the Integrated Bar of the Philippines has received 4th Division 5 None
numerous complaints from its members about serious delays
in the decision of cases and in the resolution of motions and 5th Division 52 1
other pending incidents before the different divisions of the
Sandiganbayan;
Total 415[6]
WHEREAS, Supreme Court Administrative Circular No. 10-94
requires all Regional Trial Courts, Metropolitan Trial Courts, Thus, the Sandiganbayan has a total of four hundred fifteen
Municipal Trial Courts and Municipal Circuit Trial Courts to (415) cases for decision remaining undecided long beyond the
submit to the Supreme Court a bi-annual report indicating the reglementary period to decide, with one case submitted as
title of the case, its date of filing, the date of pre-trial in civil early as May 24, 1990,[7] and motion for reconsideration
cases and arraignment in criminal cases, the date of initial which has remained unresolved over thirty days from
trial, the date of last hearing and the date that the case is submission.[8]
submitted for decision, and to post, in a conspicuous place
within its premises, a monthly list of cases submitted for On October 20, 2000, Sandiganbayan Presiding Justice
decision; Francis E. Garchitorena submitted a schedule of cases
submitted for decision, the schedule indicating the number of
WHEREAS, Supreme Court Administrative Circular No. 10-94 detained prisoners, of which there are (were) none.[9]
has not been made applicable to the Sandiganbayan;
On October 26, 2000, the IBP submitted its reply to the
WHEREAS, considering that the Sandiganbayan is also a trial compliance stating: First, that it was not in a position to
court, the requirements imposed upon trial courts by Supreme comment on the accuracy of the compliance; nonetheless, it
Court Administrative Circular No. 10-94 should also be showed that there was much to be desired with regard to the
imposed upon the Sandiganbayan; expeditious disposition of cases, particularly in the
Sandiganbayans First Division, where cases submitted for
decision since 1990 remained unresolved. Second, the
compliance did not include pending motions, and it is a fact
FINALS CONSTITUTIONAL LAW I ACJUCO NOV 11, 2017 116

that motions not resolved over a long period of time would The Issues
suspend and delay the disposition of a case. Third, since the
Sandiganbayan is a trial court, it is required to submit the The issues presented are the following: (1) What is the
same reports required of Regional Trial Courts. Fourth, the reglementary period within which the Sandiganbayan must
Constitution[10]states that, all lower collegiate courts must decide/resolve cases falling within its jurisdiction? (2) Are
decide or resolve cases or matters before it within twelve (12) there cases submitted for decision remaining undecided by
months from date of submission; however, the the Sandiganbayan or any of its divisions beyond the afore-
Sandiganbayan, as a trial court, is required to resolve and stated reglementary period? (3) Is Supreme Court
decide cases within a reduced period of three (3) months like Administrative Circular No. 1094 applicable to the
regional trial courts, or at the most, six (6) months from date Sandiganbayan?[19]
of submission.[11]
The Courts Ruling
On November 21, 2000, the Court resolved to direct then
Court Administrator Alfredo L. Benipayo (hereafter, the OCA)
to conduct a judicial audit of the Sandiganbayan, especially We resolve the issues presented in seriatim.
on the cases subject of this administrative matter, and to
submit a report thereon not later than 31 December 2000.[12] 1. Period To Decide/Resolve Cases.-- There are two views.
The first view is that from the time a case is submitted for
On December 4, 2000, in a letter addressed to the Chief decision or resolution, the Sandiganbayan has twelve (12)
Justice, Presiding Justice Francis E. Garchitorena admitted months to decide or resolve it.[20] The second view is that as
that the First Division of the Sandiganbayan[13] has a backlog a court with trial function, the Sandiganbayan has three (3)
of cases; that one case[14] alone made the backlog of the months to decide the case from the date of submission for
First Division so large, involving 156 cases but the same has decision.[21]
been set for promulgation of decision on December 8, 2000,
which would reduce the backlog by at least fifty percent Article VIII, Section 15 (1) and (2), of the 1987 Constitution
(50%).[15] provides:

On January 26, 2001, the Court Administrator submitted a "Sec. 15. (1) All cases or matters filed after the effectivity of
memorandum to the Court[16] stating that the causes of delay this Constitution must be decided or resolved within twenty-
in the disposition of cases before the Sandiganbayan are:[17] four months from date of submission to the Supreme Court,
and, unless reduced by the Supreme Court, twelve months for
(1) Failure of the Office of the Special Prosecutor to submit all lower collegiate courts, and three months for all other lower
reinvestigation report despite the lapse of several years; courts.

(2) Filing of numerous incidents such as Motion to Dismiss, (2) A case or matter shall be deemed submitted for decision
Motion to Quash, Demurrer to Evidence, etc. that remain or resolution upon the filing of the last pleading, brief or
unresolved for years; memorandum required by the Rules of Court or by the court
itself.[22]
(3) Suspension of proceedings because of a pending petition
for certiorari and prohibition with the Supreme Court; The above provision does not apply to the Sandiganbayan.
The provision refers to regular courts of lower collegiate level
that in the present hierarchy applies only to the Court of
(4) Cases remain unacted upon or have no further settings Appeals.[23]
despite the lapse of considerable length of time; and
The Sandiganbayan is a special court of the same level as the
(5) Unloading of cases already submitted for decision even if Court of Appeals and possessing all the inherent powers of a
the ponente is still in service. court of justice,[24] with functions of a trial court.[25]

We consider ex mero motu the Resolution of the Integrated Thus, the Sandiganbayan is not a regular court but a special
Bar of the Philippines (IBP) as an administrative complaint one.[26] The Sandiganbayan was originally empowered to
against Presiding Justice Francis E. Garchitorena for serious promulgate its own rules of procedure.[27] However, on March
delays in the decision of cases and in the resolution of motions 30, 1995, Congress repealed the Sandiganbayans power to
and other pending incidents before the different divisions of promulgate its own rules of procedure[28] and instead
the Sandiganbayan, amounting to incompetence, inefficiency, prescribed that the Rules of Court promulgated by the
gross neglect of duty and misconduct in office. Supreme Court shall apply to all cases and proceedings filed
with the Sandiganbayan.[29]
We find no need to conduct a formal investigation of the
charges in view of the admission of Justice Francis E. Special courts are judicial tribunals exercising limited
Garchitorena in his compliance of October 20, 2000, that there jurisdiction over particular or specialized categories of actions.
are indeed hundreds of cases pending decision beyond the They are the Court of Tax Appeals, the Sandiganbayan, and
reglementary period of ninety (90) days from their submission. the Sharia Courts.[30]
In one case, he not only admitted the delay in deciding the
case but took sole responsibility for such inaction for more
than ten (10) years that constrained this Court to grant Under Article VIII, Section 5 (5) of the Constitution Rules of
mandamus to dismiss the case against an accused to give procedure of special courts and quasi-judicial bodies shall
substance and meaning to his constitutional right to speedy remain effective unless disapproved by the Supreme Court.
trial.[18]
In his report, the Court Administrator would distinguish
between cases which the Sandiganbayan has cognizance of
FINALS CONSTITUTIONAL LAW I ACJUCO NOV 11, 2017 117

in its original jurisdiction,[31] and cases which fall within the According to the compliance submitted by the
appellate jurisdiction of the Sandiganbayan.[32] The Court Sandiganbayan, three hundred and forty one (341) cases
Administrator posits that since in the first class of cases, the were submitted for decision but were undecided as of
Sandiganbayan acts more as a trial court, then for that September 15, 2000. A number of the cases were submitted
classification of cases, the three (3) month reglementary for decision as far back as more than ten (10) years ago. As
period applies. For the second class of cases, the of September 15, 2000, the following cases[43] had not been
Sandiganbayan has the twelve-month reglementary period for decided:[44]
collegiate courts.[33] We do not agree.
First Division
The law creating the Sandiganbayan, P.D. No. 1606[34] is
clear on this issue.[35] It provides: Case Title Case No. Date Submitted for
Decision
Sec. 6. Maximum period for termination of cases As far as
practicable, the trial of cases before the Sandiganbayan once (1) People v. Paares 12127 May 24, 1990
commenced shall be continuous until terminated and the
judgment shall be rendered within three (3) months from the
date the case was submitted for decision. (2) People v. Gabriel Duero 11999 December 11,
1990
On September 18, 1984, the Sandiganbayan promulgated its
own rules,[36] thus:[37] (3) People v. RhizaMonterozo 133533 December 14,
1990
Sec. 3 Maximum Period to Decide Cases The judgment or
final order of a division of the Sandiganbayan shall be (4) People v. Zenon R. Perez 13353 January 7, 1991
rendered within three (3) months from the date the case was
submitted for decision(underscoring ours). (5) People v. Bernardo B. Dayao, Jr. 12305-12306
February 7, 1991
Given the clarity of the rule that does not distinguish, we hold
that the three (3) month period, not the twelve (12) month (6) People v. Melquiades Ribo 13521 May 7, 1991
period, to decide cases applies to the Sandiganbayan.
Furthermore, the Sandiganbayan presently sitting in five (5) (7) People v. Carlos Benitez 12102 June 19, 1991
divisions,[38] functions as a trial court. The term trial is used
in its broad sense, meaning, it allows introduction of evidence
by the parties in the cases before it.[39]The Sandiganbayan, (8) People v. Salvador P. Nopre, et. al. 11156-11160
in original cases within its jurisdiction, conducts trials, has the August 9, 1991
discretion to weigh the evidence of the parties, admit the
evidence it regards as credible and reject that which they (9) People v. Delfina A. Letegio 12289 August 28,
consider perjurious or fabricated.[40] 1991

Compliance with its Own Rules (10) People v. Rodolfo A. Lasquite 13618 August 28,
1991
In Department of Agrarian Reform Adjudication Board
(DARAB) v. Court of Appeals,[41] the Court faulted the (11) People v. Potenciana Evangelista 13679-13680
DARAB for violating its own rules of procedure. We reasoned September 3, 1991
that the DARAB does not have unfettered discretion to
suspend its own rules. We stated that the DARAB should have
(12) People v. Ramon N. Guico, Jr. et. al 16516
set the example of observance of orderly procedure.
December 2, 1991
Otherwise, it would render its own Revised Rules of
Procedure uncertain and whose permanence would be
dependent upon the instability of its own whims and caprices. (13) People v. Ruperto N. Solares 16239 January 10,
1992
Similarly, in Cabagnot v. Comelec,[42] this Court held that the
Commission on Elections ought to be the first one to observe (14) People v. Socorro Alto 13708 March 9, 1992
its own Rules. Its departure from its own rules constitutes
arrogance of power tantamount to abuse. Such inconsistency (15) People v. TomasBaguio 130151 March 11, 1992
denigrates public trust in its objectivity and dependability. The
Court reminded the Comelec to be more judicious in its
(16) People v. Felipa D. de Veyra 13672 April 13, 1992
actions and decisions and avoid imprudent volte-face moves
that undermine the public's faith and confidence in it.
(17) People v. Felicidad Tabang 12139 July 23, 1992
The ratio decidendi in the afore-cited cases applies mutatis
mutandis to the Sandiganbayan. The Sandiganbayan ought to (18) People v. Jose S.Buguia 14227 September 9,
be the first to observe its own rules. It cannot suspend its rules, 1992
or except a case from its operation.
(19) People v. Eleno T. Regidor, et al. 13689-13695
2. Undecided Cases Beyond the Reglementary Period.-- We January 6, 1993
find that the Sandiganbayan has several cases undecided
beyond the reglementary period set by the statutes and its (20) People v. Serafin Unilongo 14411 February 2,
own rules, some as long as more than ten (10) years ago. 1993
FINALS CONSTITUTIONAL LAW I ACJUCO NOV 11, 2017 118

(21) People v. Manuel Parale, et al. 15168 June 21, 1993 (45) People v. Engr.Antonio B. Laguador 14195 March
31, 1997
(22) People v. Robert P. Wa-acon 14375 June 21, 1993
(46) People v. Paterno C. Belcia, Jr. 16583-16585 March
(23) People v. Linda J. Necessito 13668 July 13, 1993 31, 1997

(24) People v. Simon Flores 16946 August 4, 1993 (47) People v. SPO3Serafin V. Reyes 21608 March
31, 1997
(25) People v. Alejandro F. Buccat 14986 August 31,
1993 (48) People v. MayorSamuel F. Bueser, et al.
22195-22196 March 31, 1997
(26) People v. IrmaCollera Monge 15301 March 9, 1994
(49) People v. Romeo C. Monteclaro 14223 May 6,
1997
(27) People v. Melencio F. Ilajas 9977 May 10, 1994
(50) People v. Rodolfo E.Aguinaldo 20948-20949
(28) People v.Buenaventura Q.Sindac, et al. 13747-13748 October 17, 1997
August 19, 1994
(51) People v. Aniceto M. Sobrepea 23324 October 27,
(29) People v. Jesus A. Bravo 17514 August 24, 1997
1994
(52) People v. Marietta T. Caugma, et al. 17001
(30) People v. Raul S. Tello 15006 November 15, November 26, 1997
1994
(53) People v. MayorMeliton Geronimo, et al. 19708
(31) People v. Celso N. Jacinto 14975 January 10, February 23, 1998
1995
(54) People v. Fernando Miguel, et al. 17600 April 7,
(32) People v. MayorAntonio Abad Santos, et al. 17670 1998
January 24, 1995
(55) People v. Rogelio A. Aniversario 17601 April 7,
(33) People v. Lamberto R. Te 20588 February 14, 1998
1995
(56) People v. Corazon Gammad Leao 9812-9967
(34) People v. AleFrancisco 21020 July 18, 1995 May 8, 1998

(35) People v. Dir. Felix R. Gonzales, et al. 13563 July (57) People v. Teresita S. Lazaro 17901 June 8, 1998
25, 1995
(58) People v. Brig. Gen. Raymundo Jarque, et al. 20688
(36) People v. MayorAdelina Gabatan, et al. 14324 October 19, 1998
January 3, 1996
(59) People v. Pros. Filotea Estorninos 23509
(37) People v. Victoria Posadas-Adona 17202 October 19, 1998
January 4, 1996
(60) People v. Orlando Mina 19534-19545
(38) People v. Roberto Estanislao Chang, et al. 16854 October 20, 1998
January 22, 1996
(61) People v. Vice Gov. Milagros A. Balgos 23042
(39) People v. Godofredo Yambao, et al. 16927-16928 October 20, 1998
March 13, 1996
(62) People v. Ceferino Paredes, Jr., et al. 18857
(40) People v. Honesto G. Encina 13171 April 26, 1996 November 17, 1998

(41) People v. Pablito Rodriguez 13971 May 10, 1996 (63) People v. Brig. Gen. Rayundo Jarque, et al. 18696
January 15, 1999
(42) People v. Leandro A. Suller 17759 June 28, 1996
(64) People v. MayorAgustin R. Escao, Jr. 23336
(43) People v. Trinidad M. Valdez 16695 August 26, January 15, 1999
1996
(65) People v. MayorEdgar V. Teves, et al. 23374
(44) People v. Vivencio B. Patagoc 19651 January 27, January 15, 1999
1997
(66) People v. C/Supt. Alfonso T. Clemente, et al. 22832
January 29, 1999
FINALS CONSTITUTIONAL LAW I ACJUCO NOV 11, 2017 119

(67) People v. Dominica Santos 19059-19063 (4)People v. Alfredo Sarmiento, et al. 24407-24408
February 18, 1999 August 11, 2000

(68) People v. Edith G. Tico 23273 April 20, 1999 Third Division**

(69) People v. Sec.Hilarion J. Ramiro, et al. 23511 Case Title Case No. Date Submitted for
August 6, 1999 Decision

(70) People v. Timoteo A. Garcia, et al. 24042-24098 (1) People v. Sergia Zoleta A/R # 016 November 16,
August 6, 1999 1999

(71) People v. Mayor Jeceju L. Manaay 24402 (2) People v. Manuel Solon Y Tenchaves A/R # 029
August 6, 1999 December 9, 1999

(72) People v. Dir.Rosalinda Majarais, et al. 24355 (3) People v. Eliseo L. Ruiz 13861-13863 April 6,
August 18, 1999 2000

(73) People v. Victor S. Limlingan 24281 August 13, (4) People v. Manuel R. Galvez, et al. 13889
1999 September 30, 1999

(74) People v. Nestor S. Castillo, et al. 24631 (5) People v. Tolentino Mendoza, et al. 16756
August 31, 1999 August 28, 1999

(75) People v. Apolinar Candelaria 22145 September 6, (6) People v. Rodrigo Villas 19563 April 6, 2000
1999
(7) People v. Ernesto Vargas 19574 April 6, 2000
(76) People v. Bernardo Billote Resoso 19773-19779
October 11, 1999 (8) People v. Ernesto, Vargas, et al. 20053 April 6, 2000

(77) People v. Atty.Alfredo Fordan Rellora, et al. (9) People v. Marcelo T.Abrenica, et al. 23522 July 6,
24433-24434 October 11, 1999 2000

(78) People v. Faustino Balacuit 98 December 22, (10) People v. Florencio Garay, et al. 25657 May 5,
1999 2000

(79) People v. MayorBernardino Alcaria, Jr., et al. Fourth Division***


23418-23423 January 6, 2000
Case Title Case No. Date Submitted for
(80) People v. Joel R.Lachica, et al. 24319-24329 Decision
January 6, 2000
(1) People v. Jaime Alos, et Al. 17664 August 31,
(81) People v. JoseMicabalo, et al. 24531-24534 April 1999
27, 2000
(2) People v. Antonio R. De Vera 23366 November 26,
(82) People v. MayorEduardo Alarilla 23069 May 1999
29, 2000
(3) People v. Aurora Mantele 24841-42 May 9,
(83) People v. Pros. Nilo M. Sarsaba, et al. 23323 May 2000
29, 2000
(4) People v. Olegario Clarin, Jr., et al. 25198 July
(84) People v. Philip G. Zamora 24150 May 29, 2000 12, 2000

Second Division* Fifth Division****

Case Title Case No. Date Submitted for Case Title Case No. Date Submitted for
Decision Decision

(1) People v. Marcelino Cordova, et al. 18435 (1) People v. Nestor A. Pablo 13344 January 16,
August 11, 2000 1998

(2) People v. Benjamin T.Damian 22858 August 11, (2) People v. Hernand D.Dabalus, et al. 14397
2000 January 13, 1999

(3)People v. Lino L. Labis, et al. 22398 July 18, 2000 (3) People v. Eduardo Pilapil 16672 March 23, 2000
FINALS CONSTITUTIONAL LAW I ACJUCO NOV 11, 2017 120

(4) People v. P/Sgt. Nazario Marifosque 17030 April (26) People v. Mayor Walfrido A. Siasico 23427
16, 1998 January 16, 1998

(5) People v. Ignacio B. Bueno 17055 September 12, The Sandiganbayan is a special court created in an effort
1995 to maintain honesty and efficiency in the bureaucracy,
weed out misfits and undesirables in the government and
(6) People v. Corazon G. Garlit 17072 March 31, 1997 eventually stamp out graft and corruption.45 We have held
consistently that a delay of three (3) years in deciding a single
case is inexcusably long.46 We can not accept the excuses of
(7) People v. Mayor RufoPabelonia, et al. 17538 Presiding Justice Sandiganbayan Francis E. Garchitorena
November 14, 1995 that the court was reorganized in 1997; that the new justices
had to undergo an orientation and that the Sandiganbayan
(8) People v. Enrique B. Lenon, et al. 17617 March relocated to its present premises which required the packing
13, 1996 and crating of records; and that some boxes were still
unopened.47
(9) People v. Constancio Bonite, et al. 17618-17619
May 1, 1995 We likewise find unacceptable Presiding Justice
Garchitorenas excuse that one case alone48 comprises more
(10) People v. Jesus Villanueva 17884 January 9, 1996 that fifty percent (50%) of the First Divisions backlog and that
the same has been set for promulgation on December 8,
2000.49 As we said, a delay in a single case cannot be
(11) People v. Ricardo T.Liwanag, et al. 18008 March tolerated, para muestra, basta un boton. (for an example, one
9, 1998 button suffices). It is admitted that there are several other
cases submitted for decision as far back as ten (10) years ago
(12) People v. Ma. Lourdes L. Falcon 18036 that have remained undecided by the First Division, of which
January 18, 1995 Justice Garchitorena is presiding justice and chairman.
Indeed, there is even one case, which is a simple motion to
withdraw the information filed by the prosecutor. This has
(13) People v. Luis D. Montero, et al. 18684 July
remained unresolved for more than seven (7) years (since
24, 1998
1994).50 The compliance submitted by the Sandiganbayan
presiding justice incriminates him. The memorandum
(14) People v. Roel D. Morales 18699 December 22, submitted by the Court Administrator likewise testifies to the
1995 unacceptable situation in the Sandiganbayan. Indeed, there is
a disparity in the reports submitted by the Sandiganbayan
(15) People v. Diosdado T. Gulle 18759 October 18, presiding justice and the OCA. According to the Court
1995 Administrator, the cases submitted for decision that were still
pending promulgation51 before the five divisions of the
Sandiganbayan are:52
(16) People v. Benjamin Sapitula, et al. 18785
August 31, 1995
First Division
(17) People v. Danilo R. Santos, et al. 18932
November 4, 1997 Case Number Date Submitted

(18) People v. Pat. DaniloMaraon 19039 May 24, 1995 Criminal Cases

(19) People v. Romeo Cabando, et al. 19378-19379 1. 11156 8/9/91


May 27, 1996
2. 11157 8/9/91
(20) People v. SPO2 Rodolfo Burbos 19593 July 6,
1998 3. 11158 8/9/91

(21) People v. Guillermo M. Viray, et al. 19614 4. 11159 8/9/91


August 31, 1998
5. 11160 8/9/91
(22) People v. Mayor Bonifacio Balahay 20427
November 5, 1999
6. 11999 12/10/90

(23) People v. Enrique Sy, et al. 20487 December 17,


7. 12102 7/1/91
1998

8. 12127 2/12/90
(24) People v. PO2 Manuel L. Bien 20648-20649 March
31, 1998
9. 12139 6/10/92
(25) People v. Felipe L. Laodenio 23066 September 28,
1999 10. 12289 8/28/91

11. 12305 2/7/91


FINALS CONSTITUTIONAL LAW I ACJUCO NOV 11, 2017 121

12. 12306 2/7/91 43. 15006 11/19/94

13. 13015 3/2/92 44. 15168 3/25/93

14. 13171 11/16/95 45. 15301 3/16/94

15. 13353 10/6/90 46. 16239 12/26/91

16. 13521 12/12/99 47. 16516 11/19/91

17. 13563 7/4/95 48. 16583 8/13/96

18. 13618 7/14/91 49. 16584 8/13/96

19. 13668 6/13/93 50. 16585 8/13/96

20. 13672 3/5/92 51. 16695 8/15/96

21. 13679 8/6/91 52. 16854 1/15/96

22. 13680 8/6/91 53. 16927 12/17/95

23. 13689 11/14/92 54. 16928 12/17/95

24. 13690 11/14/92 55. 16946 8/4/93

25. 13691 11/14/92 56. 17001 9/4/97

26. 13692 11/14/92 57. 17278 5/2/94

27. 13693 11/14/92 58. 17447 9/6/94

28. 13694 11/14/92 59. 17448 9/6/94

29. 13695 11/14/92 60. 17514 8/19/94

30. 13708 3/9/92 61. 17600 8/30/97

31. 13747 8/19/94 62. 17601 8/30/97

32. 13748 8/19/94 63. 17670 11/25/94

33. 13971 3/12/95 64. 17759 6/25/96

34. 14223 3/7/97 65. 17901 5/28/98

35. 14227 9/5/92 66. 18283 2/21/95

36. 14230 11/30/90 67. 18696 8/9/98

37. 14287 7/3/94 68. 18857 10/21/98

38. 14324 11/5/95 69. 19059 2/11/99

39. 14375 5/22/95 70. 19060 2/11/99

40. 14411 1/24/93 71. 19061 2/11/99

41. 14975 9/29/94 72. 19062 2/11/99

42. 14986 12/11/92 73. 19063 2/11/99


FINALS CONSTITUTIONAL LAW I ACJUCO NOV 11, 2017 122

74. 19534 9/2/98 105. 23422 10/15/99

75. 19535 9/2/98 106. 23423 10/15/99

76. 19651 11/15/96 107. 23509 9/5/98

77. 19708 8/25/98 108. 23511 4/23/99

78. 19773 5/21/99 109. 23540 10/15/99

79. 19774 5/21/99 110. 24042 4/28/99

80. 19775 5/21/99 111. 24043 4/28/99

81. 19976 5/21/99 112. 24044 4/28/99

82. 19977 5/21/99 113. 24045 4/28/99

83. 19978 5/21/99 114. 24046 4/28/99

84. 19979 5/21/99 115. 24047 4/28/99

85. 20588 2/14/95 116. 24048 4/28/99

86. 20688 7/9/98 117. 24049 4/28/99

87. 20948 10/9/97 118. 24050 4/28/99

88. 20949 10/9/97 119. 24051 4/28/99

89. 21020 7/4/95 120. 24052 4/28/99

90. 22145 7/7/99 121. 24053 4/28/99

91. 22195 6/14/96 122. 24054 4/28/99

92. 22196 6/14/96 123. 24055 4/28/99

93. 22832 10/21/98 124. 24056 4/28/99

94. 23042 8/27/98 125. 24057 4/28/99

95. 23146 11/13/00 126. 24058 4/28/99

96. 23273 4/19/99 127. 24059 4/28/99

97. 23323 3/23/00 128. 24060 4/28/99

98. 23324 8/3/97 129. 24061 4/28/99

99. 23336 9/4/97 130. 24062 4/28/99

100. 23374 12/17/98 131. 24063 4/28/99

101. 23418 10/15/99 132. 24064 4/28/99

102. 23419 10/15/99 133. 24065 4/28/99

103. 23420 10/15/99 134. 24066 4/28/99

104. 23421 10/15/99 135. 24067 4/28/99


FINALS CONSTITUTIONAL LAW I ACJUCO NOV 11, 2017 123

136. 24068 4/28/99 167. 24150 1/31/00

137. 24069 4/28/99 168. 24236 2/14/00

138. 24070 4/28/99 169. 24237 2/14/00

139. 24071 4/28/99 170. 24281 5/9/99

140. 24072 4/28/99 171. 24319 11/4/99

141. 24073 4/28/99 172. 24320 11/4/99

142. 24074 4/28/99 173. 24321 11/4/99

143. 24075 4/28/99 174. 24322 11/4/99

144. 24076 4/28/99 175. 24323 11/4/99

145. 24077 4/28/99 176. 24324 11/4/99

146. 24078 4/28/99 177. 24325 11/4/99

147. 24079 4/28/99 178. 24326 11/4/99

148. 24080 4/28/99 179. 24327 11/4/99

149. 24081 4/28/99 180. 24328 11/4/99

150. 24082 4/28/99 181. 24329 11/4/99

151. 24083 4/28/99 182. 24339 10/20/00

152. 24084 4/28/99 183. 24355 2/18/99

153. 24085 4/28/99 184. 24395 7/13/99

154. 24086 4/28/99 185. 24402 6/17/99

155. 24087 4/28/99 186. 24433 9/6/99

156. 24088 4/28/99 187. 24434 9/6/99

157. 24089 4/28/99 188. 24531 12/16/99

158. 24090 4/28/99 189. 24532 12/16/99

159. 24091 4/28/99 190. 24533 12/16/99

160. 24092 4/28/99 191. 24534 12/16/99

161. 24093 4/28/99 192. 24631 8/9/99

162. 24094 4/28/99 193. 24768 7/8/00

163. 24095 4/28/99 194. 6672 7/11/90

164. 24096 4/28/99 195. 9977 5/10/94

165. 24097 4/28/99 Civil Case

166. 24098 4/28/99 1. 0112 1/11/92


FINALS CONSTITUTIONAL LAW I ACJUCO NOV 11, 2017 124

2. 0116 10/16/91 27. 13828 8/30/00

3. 0156 3/14/97 28. 13829 8/30/00

Second Division 29. 13830 8/30/00

Case No. Date Submitted 30. 13831 8/30/00

Criminal Case 31. 13832 8/30/00

1. 19542 4/16/99 32. 18965 11/30/00

2. 19004 9/10/96 33. 19848 3/28/96

3. 22934 10/14/00 34. 20765 8/30/96

4. 20483 8/28/96 35. 20816 3/11/98

5. 20484 8/28/96 36. 19692 8/27/00

6. 23529 10/23/00 37. 19693 8/27/00

7. 23530 10/23/00 38. 19694 8/27/00

8. 23338 12/2/99 39. 19695 8/27/00

9. 18786 11/28/00 40. 19696 8/27/00

10. 19686 07/2/97 41. 19697 8/27/00

11. 184403 12/4/98 42. 19698 8/27/00

12. 184404 12/4/98 43. 19699 8/27/00

13. 184405 12/4/98 44. 19700 8/27/00

14. 184406 12/4/98 45. 19701 8/27/00

15. 184407 12/4/98 46. 19702 8/27/00

16. 184408 12/4/98 47. 19703 8/27/00

17. 184409 12/4/98 48. 19704 8/27/00

18. 184410 12/4/98 49. 19705 8/27/00

19. 184411 12/4/98 50. 19706 8/27/00

20. 184412 12/4/98 51. 19707 8/27/00

21. 184413 12/4/98 52. 23262 10/11/00

22. 184414 12/4/98 53. AR#035 12/9/00

23. 184415 12/4/98 54. 24994 8/17/00

24. 184416 12/4/98 55. 21097 12/13/00

25. 184417 12/4/98 56. 20660 12/20/00

26. 13827 8/30/00 57. 23111 11/27/00


FINALS CONSTITUTIONAL LAW I ACJUCO NOV 11, 2017 125

58. 24407 7/27/00 23. 19563 4/6/00

59. 24408 7/27/00 24. 19574 4/6/00

60. 18435 3/21/00 25. 19622 4/6/00

61. 22858 8/4/00 26. 19623 4/6/00

62. 22976 5/4/99 27. 19624 4/6/00

Civil Case 28. 20053 4/6/00

1. 0171 7/10/00 29. 20054 4/6/00

Third Division 30. 20271 12/18/00

Case Number Date Submitted 31. 22143 12/18/00

1.SCA/005 12/18/00 32. 23014 9/23/00

2.A/R 016 8/5/99 33. 23522 7/6/00

3.A/R 029 10/2/00 34. 23699 3/22/00

4.487 4/8/98 35. 23700 3/22/00

5.488 4/8/98 36. 23701 3/22/00

6.489 4/8/98 37. 23802 9/10/00

7.490 4/8/98 38. 23803 9/10/00

8.491 4/8/98 39. 24153 12/18/00

9.11794 6/10/00 40. 24697 9/10/00

10.13861 4/6/00 41. 24698 9/10/00

11. 13862 4/6/00 42. 24741 12/7/00

12. 13863 4/6/00 43. 24779 10/28/00

13. 13889 3/25/99 44. 24780 10/28/00

14. 16756 8/25/99 45. 24781 10/28/00

15. 17532 12/11/00 46. 25657 5/5/00

16. 18867 10/5/00 Fourth Division

17. 18868 10/5/00 Case No. Date Submitted

18. 18869 10/5/00 1. 11960 09/21/98

19. 18870 10/5/00 2.17664 01/29/98

20. 18871 10/5/00 3.13036 02/22/99

21. 18872 10/5/00 4.13037 02/22/99

22. 19182 4/6/00 5.13593 05/21/96


FINALS CONSTITUTIONAL LAW I ACJUCO NOV 11, 2017 126

6.13594 05/21/96 37. 20475 07/07/00

7.13757 03/21/97 38. 20476 07/07/00

8.14380 02/14/95 39. 20664 06/29/96

9.16809 03/26/00 40. 20685 02/18/00

10.17015 06/06/94 41. 20828 09/13/00

11.17016 06/06/94 42. 21093 08/07/99

12.17140 06/13/96 43. 21131 08/04/96

13.17141 06/13/96 44. 21778 09/29/97

14.17209 12/27/96 45. 21779 09/29/97

15.17805 02/15/00 46. 21780 09/29/97

16.17806 02/15/00 47. 22891 03/02/00

17.17809 02/15/00 48. 22892 03/02/00

18. 17856 04/02/00 49. 23007 05/24/99

19. 18005 05/07/96 50. 23058 04/27/00

20. 18006 05/07/96 51. 23059 04/27/00

21. 18257 09/22/97 52. 23060 04/27/00

22. 18894 11/17/00 53. 23061 04/27/00

23. 18895 11/17/00 54. 23062 04/27/00

24. 18896 11/17/00 55. 23366 03/28/99

25. 18900 10/28/00 56. 23415 05/25/00

26. 18935 06/16/00 57. 23534 12/15/00

27. 18936 06/16/00 58. 23708 09/27/00

28. 18937 06/16/00 59. 24447 09/18/00

29. 19567 05/21/96 60. 24448 09/18/00

30. 20338 05/19/97 61. 24464 07/26/00

31. 20469 07/07/00 62. 24465 07/26/00

32. 20470 07/07/00 63. 24742 10/10/00

33. 20471 07/07/00 64. 24841 03/22/00

34. 20472 07/07/00 65. 24842 03/22/00

35. 20473 07/07/00 66. 24851 10/29/00

36. 20474 07/07/00 67. 25198 05/31/00


FINALS CONSTITUTIONAL LAW I ACJUCO NOV 11, 2017 127

68. 25389 09/26/00 26. 23450 9/16/00

69. 25543 12/27/00 27. 23515 1/29/00

70. 25658 07/28/00 28. 24155 11/30/00

Fifth Division 29. 24379 8/27/00

Case Number Date Submitted 30. 24759 5/5/00

Criminal Cases 31. 24858 12/28/00

1. 14397 1/4/99 We find that Presiding Justice Francis E. Garchitorena


failed to devise an efficient recording and filing system to
2. 16672 2/13/00 enable him to monitor the flow of cases and to manage
their speedy and timely disposition. This is his duty on
which he failed.53
3. 17030 2/19/98
Memorandum of the Court Administrator
4. 17826 12/9/00
On November 14, 2001, the Court required the Office of the
5. 17827 12/9/00 Court Administrator54 to update its report.55

6. 18478 8/21/00 On November 16, 2001, OCA Consultant Pedro A. Ramirez


(Justice, Court of Appeals, Retired) submitted a compliance
7. 18684 5/29/98 report with the Courts order. The compliance report shows
that to this day, several cases that were reported pending by
8. 18880 12/6/00 the Sandiganbayan on September 26, 2000, and likewise
reported undecided by the OCA on January 26, 2001, have
not been decided/resolved. We quote the compliance
9. 19510 12/4/00 report:56

10. 19511 12/4/00 First Division

11. 19512 12/4/00 Case Number Date Submitted Ponente


Assigned Reason for Not Deciding Case
12. 19593 6/5/98
194. 11999 12/10/90 Garchitorena Under
13. 19614 7/31/98 study, submitted before the reorganization

14. 19668 7/26/98 195. 12102 7/1/91 Garchitorena Under


study, submitted before the reorganization
15. 20194 1/8/01
196. 12127 2/12/90 Not reported;
unaccounted for by Sandiganbayan report
16. 20427 11/3/99

197. 12139 6/10/92 Castaneda*


17. 20648 1/4/98

Under studysubmitted before the reorganization


18. 20649 1/4/98

198. 12289 8/28/91 Castaneda Under


19. 20694 3/11/98
studysubmitted before the reorganization

20. 21882 8/12/00


199. 12305-06 2/7/91 Castaneda Under
studysubmitted before the reorganization
21. 22184 12/16/00
200. 13015 3/2/92 Garchitorena Under
22. 22873 12/4/99 studysubmitted before the reorganization

23. 22926 11/13/00 201. 13171 11/16/95 Castaneda Under


studysubmitted before the reorganization
24. 23066 8/16/99
202. 13353 10/6/90 Garchitorena Under
25. 23319 9/30/00 studysubmitted before the reorganization
FINALS CONSTITUTIONAL LAW I ACJUCO NOV 11, 2017 128

203. 13521 12/12/99 Garchitorena Under 224. 15301 3/16/94 Castaneda Under study
studysubmitted before the reorganization submitted before the reorganization

204. 13563 7/4/95 Garchitorena Under 225. 16239 12/26/91 Castaneda Under study
studysubmitted before the reorganization submitted before the reorganization

205. 13618 7/14/91 Castaneda Under 226. 16516 11/19/91 Castaneda Under study
studysubmitted before the reorganization submitted before the reorganization

206. 13668 6/13/93 Castaneda Under 227. 16583-85 8/13/96 Castaneda Under study
studysubmitted before the reorganization submitted before the reorganization

207. 13672 3/5/92 Castaneda Under 228. 16695 8/15/96 Castaneda Under study
studysubmitted before the reorganization submitted before the reorganization

208. 13679-80 8/6/91 Castaneda Under 229. 16854 1/15/96 Castaneda Under study
studysubmitted before the reorganization submitted before the reorganization

209. 13689-95 11/14/92 Castaneda Under 230. 16927-28 12/17/95 Castaneda Under study
studysubmitted before the reorganization submitted before the reorganization

210. 13708 3/9/92 Castaneda Under 231. 16946 8/4/93 Castaneda Under study
studysubmitted before the reorganization submitted before the reorganization

211. 13747-48 8/19/94 Castaneda Under 232. 17001 9/4/97 Not yet assigned
studysubmitted before the reorganization
233. 17278 5/2/94 Death of accused is
212. 13971 3/12/95 Castaneda Under unconfirmed and dismissal of the case was held in abeyance.
studysubmitted before the reorganization (Ong, J.)

213. 14223 3/7/97 Death of accused is 234. 17600 8/30/97 Not yet assigned
unconfirmed and dismissal of the case was held in abeyance.
(Ong, J.)* 235. 17601 8/30/97 Not yet assigned

236. 17759 6/25/96 Ong Decided and set for


214. 14227 9/5/92 Castaneda Under study promulgation
submitted before the reorganization
237. 17901 5/28/98 Not yet assigned
215. 14230 11/30/90 Castaneda Under study
submitted before the reorganization
238. 18696 8/9/98 Not yet assigned
216. 14287 7/3/94 Castaneda Under study
submitted before the reorganization 239. 18857 10/21/98 Not yet assigned

217. 14324 11/5/95 Castaneda Under study 240. 19059-63 2/11/99 Not yet assigned
submitted before the reorganization
241. 19534-35 9/2/98 Not yet assigned
218. 14375 5/22/95 Castaneda Under study
submitted before the reorganization 242. 19708 8/25/98 Not yet assigned

219. 14411 1/24/93 Garchitorena Under study 243. 19773-79 5/21/99 Not yet assigned
submitted before the reorganization
244. 20688 7/9/98 Not yet assigned
220. 14975 9/29/94 Castaneda Under study
submitted before the reorganization 245. 20948 10/9/97 Not reported;
unaccounted for by Sandiganbayan report
221. 14986 12/11/92 Castaneda Under study
submitted before the reorganization 246. 20949 10/9/97 Not reported;
unaccounted for by Sandiganbayan report
222. 15006 11/19/94 Castaneda Under study
submitted before the reorganization 247. 21020 7/4/95 Ong Set for Promulgation on
November 27, 2001
223. 15168 3/25/93 Castaneda Under study
submitted before the reorganization 248. 22145 7/7/99 Not yet assigned
FINALS CONSTITUTIONAL LAW I ACJUCO NOV 11, 2017 129

249. 22195-96 6/14/96 Castaneda Under study, 277. 0112 1/11/92 Not reported;
submitted before the reorganization unaccounted for by Sandiganbayan report

250. 22832 10/21/98 Not yet assigned 278. 0116 10/16/91 Not reported;
unaccounted for by Sandiganbayan report
251. 23042 8/27/98 Not yet assigned
279. 0156 3/14/97 Not reported;
252. 23146 11/13/00 Not yet assigned unaccounted for by Sandiganbayan report

253. 23273 4/19/99 Not yet assigned Summary/Tally

254. 23323 3/23/00 Not yet assigned Cases Assigned to Garchitorena, PJ. 9

255. 23324 8/3/97 Not yet assigned Cases Assigned to Castaneda, J. 42

256. 23336 9/4/97 Not yet assigned Cases Assigned to Ong, J. 5

257. 23374 12/17/98 Not yet assigned Cases not yet assigned 73

258. 23418-23 10/15/99 Not yet assigned Cases not accounted for or reported 9

259. 23509 9/5/98 Not yet assigned __________

260. 23511 4/23/99 Not yet assigned Total 138

261. 23540 10/15/99 Not yet assigned Second Division

262. 24042-98 4/28/99 Ong Set for Promulgation on Case Number Date Submitted Ponente Assigned
November 27, 2001 Reason for Not Deciding Case

263. 24150 1/31/00 Not yet assigned 63. 19542 4/16/99 For retaking of testimony due to
incomplete TSN
264. 24236-37 2/14/00 Not yet assigned
64. 13827-32 8/30/00 Victorino For promulgation
265. 24281 5/9/99 Not yet assigned
65. 18965 11/30/00 For retaking of testimony due to
incomplete TSN
266. 24319-29 11/4/99 Not yet assigned
Third Division
267.24319-29 11/4/99 Not reported;
unaccounted for by Sandiganbayan report
Case Number Date Submitted Ponente Assigned
Reason for Not Deciding Case
268. 24355 2/18/99 Not yet assigned
47. SCA/005 12/18/00 Ilarde --
269.24395 7/13/99 Not reported;
unaccounted for by Sandiganbayan report
48. A/R 029 10/2/00 Illarde
270. 24402 6/17/99 Not yet assigned
49. 487-491 4/8/98 With pending demurrer to
evidence, submitted, 01/26/01 re Submitted, 03/20/01
271. 24433-34 9/6/99 Not yet assigned
50. 11794 6/10/00 De Castro --
272. 24531-34 12/16/99 Not yet assigned
51. 17532 12/11/00 Ilarde --
273. 24631 8/9/99 Not yet assigned
52. 18867-72 10/5/00 Pending trial per order dated
274. 24768 7/8/00 Not yet assigned 08/17/00

275. 6672 7/11/90 Garchitorena Under Study, 53. 19182 4/6/00 Unloaded to the 5th Division,
before the reorganization 10/13/97

276. 9977 5/10/94 Garchitorena Under Study, 54. 19563 4/6/00 No Assignment --
before the reorganization
55. 19574 4/6/00 No Assignment --
FINALS CONSTITUTIONAL LAW I ACJUCO NOV 11, 2017 130

56. 19622-24 4/6/00 Unloaded to the 5th Division, 32. 14397 1/4/99 Badoy, Jr. Inherited
10/13/97 case/lack of personnel

57. 20053-54 4/6/00 Not with the 3rd Division 33. 16672 2/13/00 Badoy, Jr. Inherited
case/lack of personnel
58. 20271 12/18/00 Illarde --
34. 17030 2/19/98 Badoy, Jr. Inherited
59. 22143 12/18/00 De Castro -- case/lack of personnel

60. 23014 9/23/00 De Castro -- 35. 18478 8/21/00 Estrada Inherited case/lack of
personnel
61. 23699-701 3/22/00 Ilarde --
36. 18684 5/29/98 Badoy, Jr. Inherited
case/lack of personnel
62. 23802-03 9/10/00 No Assignment --
37. 18880 12/6/00 Badoy, Jr. Inherited
63. 24153 12/18/00 No Assignment -- case/lack of personnel

64. 24697-98 9/10/00 Ilarde -- 38. 19510-12 12/4/00 Estrada Inherited case/lack of
personnel
65. 24741 12/7/00 De Castro --
39. 19593 6/5/98 Badoy, Jr. Inherited
66. 24779-81 10/28/00 No Assignment -- case/lack of personnel

67. 25657 5/5/00 With Defense pending motion for 40. 19614 7/31/98 Badoy, Jr. Inherited
the re-examination of the Information and the parties case/lack of personnel
affidavits, etc. Order dated 08/31/01
41. 20194 1/8/01 Chico-Nazario Complicated
Summary/Tally Issues

Cases Assigned to Illarde, J. 9 42. 20427 11/3/99 Badoy, Jr. Inherited


case/lack of personnel
Cases Assigned to De Castro, J. 4
43. 20648-49 1/4/98 Badoy, Jr. Inherited
Cases not yet assigned 8 case/lack of personnel

Others 18 44. 20694 3/11/98 Estrada Inherited case/lack of


personnel

____________
45. 22926 11/13/00 No report, Unaccounted for by the
Sandiganbayan report
Total 39
46. 23066 8/16/99 Badoy, Jr. Inherited
Fourth Division** case/lack of personnel

Case Number Date Submitted Ponente Assigned 47. 24155 11/30/00 Estrada Not yet due
Reason for Not Deciding Case
48. 24379 8/27/00 Estrada Draft decision released
71. 11960 09/21/98 Draft of decision penned by J. 7/31/01
Nario in view of the dissenting opinion of one Justice was
referred to a Division of five (5) composed of Nario, Palattao,
Ferrer, Badoy, Jr. and De Castro, JJ.

72. 16809 03/26/00 Palattao -- Summary/Tally

73. 23058-62 04/27/00 Nario -- Cases Assigned to Badoy, J. *** 11

74. 25389 09/26/00 Nario -- Cases Assigned to Estrada, J. 7

Fifth Division Cases Assigned to Chico-Nazario, J. 1

Case Number Date Submitted Ponente Assigned No report/Unaccounted For 1


Reason for Not Deciding Case
_________
FINALS CONSTITUTIONAL LAW I ACJUCO NOV 11, 2017 131

Total 20 case and initialled the last page thereof, and I certify that the
results of the inventory are correctly reflected in the above
3. Applicability of SC Adm. Circular No. 10-94.-- Supreme tabulation.
Court Circular No. 10-94 applies to the Sandiganbayan.
_________.
Administrative Circular 10-9457 directs all trial judges to make
a physical inventory of the cases in their dockets. The docket _____________________ Presiding Judge
inventory procedure is as follows:58
Given the rationale behind the Administrative Circular, we hold
a. Every trial judge shall submit not later than the last week of that it is applicable to the Sandiganbayan with respect to
February and the last week of August of each year a tabulation cases within its original and appellate jurisdiction.
of all pending cases which shall indicate on a horizontal
column the following data: Mora Decidendi

1. Title of the case We reiterate the admonition we issued in our resolution of


October 10, 2000:59
2. Date of Filing
This Court has consistently impressed upon judges
3. Date arraignment in criminal cases of Pre-trial in civil cases (which includes justices) to decide cases promptly and
and expeditiously on the principle that justice delayed is
justice denied. Decision making is the primordial and
4. Date of initial trial most important duty of the member of the bench.60
Hence, judges are enjoined to decide cases with dispatch.
Their failure to do so constitutes gross inefficiency61 that
5. Date of last hearing warrants disciplinary sanction, including fine,62 suspension63
and even dismissal.64 The rule particularly applies to justices
6. Date submitted for Decision of the Sandiganbayan. Delays in the disposition of cases
erode the faith and confidence of our people in the judiciary,
b. The tabulation shall end with a certification by the trial judge lower its standards, and bring it into disrepute.65 Delays
that he/she has personally undertaken an inventory of the cannot be sanctioned or tolerated especially in the anti-graft
pending cases in his/her court; that he/she has examined court, the showcase of the nations determination to succeed
each case record and initialled the last page thereof. The in its war against graft (underscoring ours).
judge shall indicate in his/her certification the date when
inventory was conducted. In Yuchengco v. Republic,66 we urged the Sandiganbayan to
promptly administer justice. We stated that the
c. The Tabulation and Certification shall be in the following Sandiganbayan has the inherent power to amend and control
form. its processes and orders to make them conformable to law
and justice. The Sandiganbayan as the nations anti-graft court
must be the first to avert opportunities for graft, uphold the
Docket Inventory for the Period right of all persons to a speedy disposition of their cases and
avert the precipitate loss of their rights.
January __ to June ___, ___/July
Practice of Unloading Cases
To December ___, ___
According to the memorandum submitted by the OCA, there
(Indicate Period) is a practice in the first and third divisions of the
Sandiganbayan of unloading cases to other divisions despite
the fact that these cases have been submitted for decision
Court and Station ________
before them. We cite relevant portions of the memorandum:67

Presiding Judge ________


Cases Submitted for Decision When Unloaded to the Fourth
Division
Title of Case Date Filed Pretrial/
Case No. Title of the Case Division where case
Arraignment Initial Hearing Date of Last Hearing originated Date Submitted for
Date submitted for Decision
Decision

1) 17015 PP vs. Raul Zapatos 3rd 06/06/94

2) 17016 PP vs. Raul Zapatos 3rd 06/06/94

CERTIFICATION:
3) 14380 PP vs. Francisco Ramoran 3rd 02/14/95

I hereby certify that on (Date/Dates___), I personally


4) 18005 PP vs. Panfilo Bongcac 3rd 05/07/96
conducted a physical inventory of pending cases in the docket
of this court, that I personally examined the records of each
FINALS CONSTITUTIONAL LAW I ACJUCO NOV 11, 2017 132

5) 18006 PP vs. Panfilo Bongcac 3rd 05/07/96 15. 17567 2/24/93

6) 13593 PP vs. Dominador Meninguito 3rd 16. 17598 8/3/94


05/30/96
17. 17617 3/28/96
7) 13594 PP vs. Dominador Meninguito 3rd
05/30/96 18. 17618 4/6/95

8) 19567 PP vs. Dominador Meninguito 3rd 19. 17619 4/6/95


05/30/96
20. 17640 6/12/95
9) 17140 PP vs. Jose Caf, et. al. 3rd 06/13/96
21. 17661 12/15/94
10) 17141 PP vs. Jose Caf, et. al. 3rd
06/13/96
22. 17666 8/25/97
11) 20064 PP vs. Ben dela Pena 3rd
07/01/96 23. 17884 11/12/95

12) 21131 PP vs. Rufino Mamanguin 3rd 24. 17902 4/16/95


08/05/96
25. 18008 9/15/97
13) 17209 PP vs. Isidro Catapang 3rd
12/27/96 26. 18423 1/15/96

14) 13757 PP vs. Catalino Daganzo 3rd 27. 18687 9/30/94


03/21/97
28. 18759 10/12/95
15) 18257 PP vs. Zenaida Sazon 1st
09/22/97 29. 18785 7/13/95

Cases Submitted for Decision When Unloaded to the Fifth 30. 18932 4/20/97
Division

31. 18988 10/25/95


Case Number Date Submitted

32. 18999 12/21/95


1. 10264 12/22/90

33. 19039 5/6/95


2. 13344 5/14/97

34. 19378 4/17/96


3. 16223 4/25/94

35. 19379 4/17/96


4. 16574 5/30/95

36. 19679 10/5/95


5. 16760 5/25/95

37. 19712 2/18/95


6. 16810 1/23/96

38. 19907 6/22/95


7. 17018 7/20/94

39. 20487 12/14/96


8. 17055 7/5/95

40. 20624 7/15/95


9. 17139 4/24/94

41. 23427 7/25/97


10. 17162 2/23/95

We suggest a review of the practice of unloading cases


11. 17193 3/8/94 that greatly contributes to the backlog of undecided
cases. When a case has been heard and tried before a
12. 17426 2/12/94 division of the Sandiganbayan, it is ideal that the same division
and no other must decide it as far as practicable.
13. 17480 3/22/94
We further note that several cases which were earlier reported
14. 17538 11/20/95 as undecided by the Sandiganbayan and the OCA have been
FINALS CONSTITUTIONAL LAW I ACJUCO NOV 11, 2017 133

decided since the reports of September 26, 2000 and January 20483-20484 July 26, 1995 April 6, 2001
26, 2001.Nonetheless, the delay in deciding these cases is Victorino
patent and merits reprobation. According to the compliance
report submitted by the OCA on November 16, 2001, there are 20660 December 20, 2000 August 2, 2001
several cases decided way beyond the reglementary period Legaspi
prescribed by law, even assuming without granting, a
reglementary period of twelve months from the time a case is
submitted for decision.68 20765 August 30, 1996 February 23, 2001
Victorino
In a case brought before this Court, Presiding Justice
Garchitorena admitted fault and that the fault is exclusively his 20816 March 11, 1998 January 25, 2001 Victorino
own, in failing to decide the case, though submitted for
decision as early as June 20, 1990.69 This case was not even 21097 December 13, 2000 June 15, 2001
included among pending cases in the Sandiganbayan report Victorino
of September 26, 2000.
22858 August 11, 2000 January 31, 2001 Victorino
The following cases were decided, though beyond the
prescribed period: 22934 October 14, 2000 February 15, 2001
Sandoval
First Division
22976 May 4, 1999 March 1, 2001 Sandoval
Case Number Submitted for Decision Date of
Promulgation Ponente 23111 November 27, 2000 March 14, 2001
Sandoval
14195 March 31, 1997 November 10, 2000 Ong
23262 October 11, 2000 May 16, 2001 Victorino
21608 March 31, 1997 November 15, 2000 Ong
23338 December 2, 1999 December 14, 2000
20588 February 14, 1998 January 12, 2001 Ong Sandoval

19651 November 15, 1996 January 26, 2001 Ong 23529-23530 October 23, 2000 March 28, 2001
Victorino
17670 November 25, 1994 January 26, 2001 Ong
24407-24408 August 11, 2000 January 24, 2001
17447-48 September 6, 1994 February 22, Legaspi
2001 Ong
24994 August 17, 2000 May 30, 2001 Sandoval
18283 February 21, 1995 February 23, 2001
Ong AR#035 December 9, 2000 August 28, 2001
Legaspi
17514 August 19, 1994 April 24, 2001 Ong
Third Division
Second Division
Case Number Submitted for Decision Date of
Case Number Submitted for Decision Date of Promulgation Ponente
Promulgation Ponente
A/R 016 November 16, 1999 January 26, 2001 Ilarde
18403-18417 December 4, 1998 February 2,
2001 Victorino 13861-13863 April 6, 2000 December 22, 2000
Del Rosario
18435 August 11, 2000 March 26, 2001 Victorino
13889 September 30, 1999 May 10, 2001 Ilarde
18786 November 28, 2000 March 28, 2001
Legaspi 16756 August 28, 1999 December 11, 2000 Del
Rosario
19004 September 10, 1996 March 16, 2001
Victorino 23522 July 6, 2000 January 12, 2001 Del Rosario

19692-19707 August 27, 2000 February 26, 2001 Fourth Division


Sandoval
Case Number Submitted for Decision Date of
19848 March 28, 1996 January 29, 2001 Victorino Promulgation Ponente

17664 August 31, 1999 June 1, 2000 Pallatao


FINALS CONSTITUTIONAL LAW I ACJUCO NOV 11, 2017 134

17016 June 6, 1994 March 27, 2001 Ferrer 23366 November 26, 1999 October 29, 2001 Ferrer

17140-41 June 13, 1996 February 6, 2001 Nario 23415 May 25, 2000 May 28, 2001 Palattao

17209 December 27, 1996 April 30, 2001 Ferrer 23534 December 15, 2000 February 28, 2001
Palattao
17805-09; 17814 February 15, 2000 October 10,
2001 Palattao 23708 September 27, 2000 September 10, 2001
Nario
17856 April 2, 2000 June 25, 2001 Palattao
24464-65 July 26, 2000 June 26, 2001 Nario
18005-06 May 7, 1996 May 18, 2001 Ferrer
24742 October 10, 2000 March 22, 2001 Ferrer
18257 September 22, 1997 July 26, 2001 Ferrer
24841-42 May 9, 2000 March 7, 2001 Ferrer
18894-96 November 17, 2000 March 20, 2001
Palattao 25198 July 12, 2000 February 6, 2001 Nario

18900 October 28, 2000 March 23, 2001 Ferrer 25543 December 27, 2000 February 26, 2001
Palattao
18935-37 June 16, 2000 January 18, 2001
Palattao 25658 July 28, 2000 July 20, 2001 Palattao

19567 May 21, 1996 January 15, 2001 Ferrer 24447-48 September 18, 2000 December 7,
2001 Palattao
20338 May 19, 1997 February 9, 2001 Ferrer
Fifth Division
20469 July 7, 2000 June 25, 2001 Palattao
Case Number Submitted for Decision Date of
13036-37 February 22, 1999 February 28, Promulgation Ponente
2001 Ferrer
17826-17827 December 9, 2000 March 28, 2001
13593-94 May 21, 1996 January 15, 2001 Ferrer Chico-Nazario

20470-76 July 7, 2000 June 25, 2001 19668 July 26, 1998 February 9, 2001 Badoy, Jr.
Palattao
21882 August 12, 2000 July 25, 2001 Chico- Nazario
20664 June 29, 1996 February 20, 2001 Ferrer
22184 December 16, 2000 May 21, 2001 Chico-
20685 February 18, 2000 March 2, 2001 Nazario
Palattao
22873 December 4, 1999 May 31, 2001 Chico-
20828 September 13, 2000 October 8, 2001 Nazario
Palattao
23319 September 30, 2000 April 23, 2001 Chico-
21093 August 7, 1999 January 15, 2001 Palattao Nazario

21131 August 4, 1996 February 13, 2001 Ferrer 23450 September 16, 2000 March 16, 2001 Chico-
Nazario
21778-80 September 29, 1997 June 21, 2001
Ferrer 23515 January 29, 2000 May 28, 2001 Cortez-Estrada

22891-92 March 2, 2000 December 13, 2000 24759 May 5, 2000 July 10, 2001 Cortez-Estrada
Ferrer
24858 December 28, 2000 May 31, 2001 Chico-
23007 May 24, 1999 March 14, 2000 Ferrer Nazario

13757 March 21, 1997 July 2, 2001 Ferrer Relief of Presiding Justice

14380 February 14, 1995 April 23, 2001 Ferrer At this juncture, the Court cites the case of Canson v.
Garchitorena.70 In that case, we admonished respondent
Presiding Justice Francis E. Garchitorena. General Jewel F.
17015 June 6, 1994 March 27, 2001 Ferrer Canson, Police Chief Superintendent, National Capital Region
FINALS CONSTITUTIONAL LAW I ACJUCO NOV 11, 2017 135

Command Director, complained of deliberate delayed action hundred ninety eight (198) cases reported submitted for
of the Presiding Justice on the transfer of Criminal Cases Nos. decision before the First Division.80 Even in the updated
23047-23057 to the Regional Trial Court of Quezon City, report, there are one hundred thirty eight (138) cases still
depriving complainant of his right to a just and speedy trial. undecided in the First Division.
Due to a finding of lack of bad faith on the part of respondent
justice, we issued only a warning. However, the dispositive In fact, Presiding Justice Francis E. Garchitorena admitted
portion of the decision cautioned respondent justice that a that he has a backlog.81 He claimed that one (1) case alone
repetition of the same or similar act in the future shall be dealt comprises fifty percent (50%) of the backlog. We find this
with more severely.71 claim exaggerated. We cannot accept that a backlog of three
hundred forty one (341) cases in the First Division could be
Presiding Justice Francis E. Garchitorena sits as the eliminated by the resolution of a single consolidated case of
Chairman, First Division, with a backlog of cases pending one hundred fifty six (156) counts. A consolidated case is
decision. At least seventy-three cases have been unassigned considered only as one case. The cases referred to were
for the writing of the extended opinion, though submitted for consolidated as Criminal Case Nos. 9812-9967, People v.
decision. It may be the thinking of the Presiding Justice, Corazon Gammad-Leao, decided on December 8, 2000.
Sandiganbayan that an unassigned case is not counted in its What about the one hundred eighty five (185) cases that
backlog of undecided cases. This is not correct. It is the duty unfortunately remained undecided to this date? Worse, the
of the Presiding Justice and the Chairmen of divisions to motion for reconsideration of the decision in said cases,
assign the ponente as soon as the case is declared submitted submitted as of January 11, 2001, has not been resolved to
for decision, if not earlier. If he fails to make the assignment, this date.82 The First Division has only thirty (30) days from
he shall be deemed to be the ponente. submission to resolve the same. It is now ten (10) months from
submission. The expediente and the motion were transmitted
The Constitution provides that a case shall be deemed to the ponente, Presiding Justice Francis E. Garchitorena, on
submitted for decision or resolution upon the filing of the last that date, but to this day the case remains unresolved.83
pleading, brief, or memorandum required by the Rules of Unfortunately, even other divisions of the Sandiganbayan may
Court or by the court itself.72 In Administrative Circular No. be following his example.84
28, dated July 3, 1989, the Supreme Court provided that A
case is considered submitted for decision upon the admission In the first report of the Court Administrator, he indicated a
of the evidence of the parties at the termination of the trial. total of one hundred ninety five (195) criminal cases and three
The ninety (90) days period for deciding the case shall (3) civil cases, or a total of one hundred ninety eight (198)
commence to run from submission of the case for decision cases submitted for decision as of December 21, 2000.85
without memoranda; in case the court requires or allows its Almost a year later, as of November 16, 2001, there are still
filing, the case shall be considered submitted for decision one hundred thirty eight (138) cases undecided submitted
upon the filing of the last memorandum or the expiration of the long ago. For almost one year, not one case was
period to do so, whichever is earlier. Lack of transcript of decided/resolved by the Presiding Justice himself.86
stenographic notes shall not be a valid reason to interrupt or
suspend the period for deciding the case unless the case was Directive
previously heard by another judge not the deciding judge in
which case the latter shall have the full period of ninety (90)
days from the completion of the transcripts within which to WHEREFORE, in view of all the foregoing, the Court resolves:
decide the same.73 The designation of a ponente to a case is
not a difficult administrative task. (1) To IMPOSE on Presiding Justice Francis E. Garchitorena
a fine of twenty thousand pesos (P20,000.00), for inefficiency
Administrative sanctions must be imposed. Mora reprobatur in and gross neglect of duty.
lege.74 Again, we reiterate the principle that decision-making
is the most important of all judicial functions and (2) Effective December 1, 2001, to RELIEVE Presiding Justice
responsibilities.75 In this area, Presiding Justice Francis E. Francis E. Garchitorena of his powers, functions and duties as
Garchitorena, as the ponente assigned to the cases submitted the Presiding Justice, Sandiganbayan, and from presiding
for decision/resolution long ago, some as far back as more over the trial of cases as a justice and Chairman, First
than ten (10) years ago, has been remiss constituting gross Division, so that he may DEVOTE himself exclusively to
neglect of duty and inefficiency.76 As we said in Canson,77 DECISION WRITING, until the backlog of cases assigned to
unreasonable delay of a judge in resolving a case amounts to him as well as cases not assigned to any ponente, of which
a denial of justice, bringing the Sandiganbayan into disrepute, he shall be deemed the ponente in the First Division, are
eroding the public faith and confidence in the judiciary.78 finally decided. There shall be no unloading of cases to other
divisions, or to the First Division inter se.
Consequently, Presiding Justice Francis E. Garchitorena
should be relieved of all trial and administrative work as In the interim, Associate Justice Minita V. Chico-Nazario, as
Presiding Justice and as Chairman, First Division so that he the most senior associate justice, shall TAKE OVER and
can devote himself full timeto decision-making until his exercise the powers, functions, and duties of the office of the
backlog is cleared. He shall finish this assignment not later Presiding Justice, Sandiganbayan, until further orders from
than six (6) months from the promulgation of this resolution. this Court.

We have, in cases where trial court judges failed to decide (3) To DIRECT Presiding Justice Francis E. Garchitorena and
even a single case within the ninety (90) day period, imposed the associate justices of the Sandiganbayan to decide/resolve
a fine ranging from five thousand pesos (P5,000.00) to the the undecided cases submitted for decision as of this date,
equivalent of their one months salary.79 According to the within three (3) months from their submission, and to resolve
report of the Sandiganbayan, as of September 26, 2000, there motions for new trial or reconsiderations and petitions for
were three hundred forty one (341) cases submitted for review within thirty (30) days from their submission. With
decision before its first division headed by the Presiding respect to the backlog of cases, as hereinabove enumerated,
Justice. In the memorandum of the OCA, there were one the Sandiganbayan shall decide/resolve all pending cases
FINALS CONSTITUTIONAL LAW I ACJUCO NOV 11, 2017 136

including incidents therein within six (6) months from notice of


this resolution.

(4) To ORDER the Sandiganbayan to comply with Supreme


Court Administrative Circular 10-94, effective immediately.

(5) To DIRECT the Sandiganbayan en banc to adopt not later


than December 31, 2001 internal rules to govern the allotment
of cases among the divisions, the rotation of justices among
them and other matters leading to the internal operation of the
court, and thereafter to submit the said internal rules to the
Supreme Court for its approval.87

This directive is immediately executory.

SO ORDERED.

Davide, Jr., C.J., Bellosillo, Melo, Puno, Vitug, Kapunan,


Mendoza, Panganiban, Quisumbing, Ynares-Santiago,
Sandoval-Gutierrez, and Carpio, JJ., concur.

De Leon, Jr., J., see dissenting and concurring opinion.

Buena, J., on official leave.


FINALS CONSTITUTIONAL LAW I ACJUCO NOV 11, 2017 137

[A.M. No. 00-7-09-CA. March 27, 2001] Daosos, Go Teng Kok, Yu Yuk Lai, MTC Judge Orlando
Siapno, Peter Young, Atty. Reinerio Paas, lawyer of Go Teng
In Re: Derogatory news Items Charging Court of Appeals Kok, Danilo J. Mijares, bodyguard of Go Teng Kok, and Luisito
Artiaga, official of the Philippine Amateur Track and Field
Association (PATAFA).
Associate Justice Demetrio Demetria with Interference on
Behalf of a Suspected Drug Queen:
The facts as borne out by the evidence presented by the
prosecution are quite clear. In an Information dated 9
Court of Appeals Associate Justice Demetrio G. Demetria, December 1998, SP Formaran III charged Yu Yuk Lai,
respondent. together with her supposed nephew, a certain Kenneth
Monceda y Sy alias William Sy, before the RTC of Manila, Br.
DECISION 18,[5] with violation of Sec. 15, Art. III, RA 6425, as
amended, for "conspiring, confederating and mutually
PER CURIAM: helping one another, with deliberate intent and without
authority of law x x x (to) willfully, unlawfully and
feloniously sell and deliver to a poseur-buyer three (3)
Men and Women of the courts must conduct themselves with kilograms, more or less, of methylamphetamine
honor, probity, fairness, prudence and discretion. Magistrates hydrochloride (shabu), which is a regulated drug."[6]
of justice must always be fair and impartial. They should avoid Accused of non-bailable offense, both Yu Yuk Lai and
not only acts of impropriety, but all appearances of Kenneth Monceda were held at the detention cell of the PNP
impropriety. Their influence in society must be consciously Narcotics Group in Camp Crame, Quezon City. On 25 June
and conscientiously exercised with utmost prudence and 1999, accused Yu Yuk Lai filed a Petition for Bail on the
discretion. For, theirs is the assigned role of preserving the ground that the evidence of her guilt was not strong.
independence, impartiality and integrity of the Judiciary.
On 10 November 1999, upon receiving information that the
The Code of Judicial Conduct mandates a judge to accused, especially Yu Yuk Lai, had been seen regularly
"refrain from influencing in any manner the outcome of playing in the casinos of Heritage Hotel and the Holiday Inn
litigation or dispute pending before another court or Pavilion, SP Formaran III filed an Urgent Ex-Parte Motion to
administrative agency."[1] The slightest form of interference Transfer the Detention of the Accused to the City Jail.[7] On
cannot be countenanced. Once a judge uses his influence to the same day, Judge Perfecto A. S. Laguio, Jr., granted the
derail or interfere in the regular course of a legal or judicial motion and ordered the immediate transfer of the two (2)
proceeding for the benefit of one or any of the parties therein, accused to the Manila City Jail.[8]
public confidence in the judicial system is diminished, if not
totally eroded.
On 18 January 2000, Judge Laguio, Jr., concluded that
"the evidence standing alone and unrebutted, is strong
Such is this administrative charge triggered by newspaper and sufficient to warrant conviction of the two accused
accounts which appeared on the 21 July 2000 issues of The for the crime charged" and denied the petition for bail of
Manila Standard, The Manila Times, Malaya, The Philippine accused Yu Yuk Lai for lack of merit.[9] Consequently, both
Daily Inquirer and Today. The national dailies collectively accused filed a Joint Motion for Inhibition arguing that the trial
reported that Court of Appeals Associate Justice Demetrio G. court's actuations "do not inspire the belief that its decision
Demetria tried to intercede on behalf of suspected Chinese would be just and impartial."[10] On 28 January 2000, Judge
drug queen Yu Yuk Lai, alias Sze Yuk Lai, who went in and Laguio, Jr., believing that the joint motion was utterly without
out of prison to play in a Manila casino.[2] merit but considering the gravity of the offense and for the
peace of mind of the accused, inhibited himself.[11]
That same day, 21 July 2000, Chief Justice Hilario G. Davide,
Jr., issued a Memorandum to Justice Demetria directing him The case was re-raffled to Branch 53, presided by Judge
to comment on the derogatory allegations in the news Angel V. Colet. Accused Yu Yuk Lai then filed a Motion to
items.[3] On 24 July 2000, Justice Demetria submitted his Order the Confinement of the Accused in a Hospital. Before
Compliance. Subsequently, Chief State Prosecutor (CSP) Judge Colet could resolve the motion, the case was handled
Jovencito R. Zuo, who disclosed to the media the name by the Branch's Pairing Judge Manuel T. Muro.
of Justice Demetria, and State Prosecutor (SP) Pablo C.
Formaran III, a member of the Task Force on Anti-
Narcotics Cases of the Department of Justice (DOJ) On 15 May 2000 Judge Muro granted accused Yu Yuk Lai's
prosecuting the case of the suspected Chinese drug motion and allowed her to be confined at the Manila Doctors
queen, filed their respective Comments on the Hospital for a period not exceeding seven (7) days,[12]
Compliance of Justice Demetria.[4] contrary to the recommendation of Dr. Jose Estrada Rosal,
Chief of the Health Services of the Manila City Jail, that Yu
Yuk Lai be confined at the Philippine General Hospital.[13]
On 8 August 2000, the Court En Banc ordered an investigation
and designated Mme. Justice Carolina C. Grio-Aquino as
Investigator and Court Administrator Alfredo L. Benipayo as On 5 June 2000 Judge Muro granted Yu Yuk Lai's Urgent
Prosecutor. An investigation then commenced on 22 August Motion for Extension of Medical Confinement "for a period of
2000 and continued until 16 November 2000. one (1) month, or until such time that she is fit to be discharged
from the said hospital."[14] On 7 July 2000 Judge Muro also
granted Yu Yuk Lai's Motion for Leave of Court to File
The Prosecution presented four (4) witnesses, namely, CSP Demurrer to Evidence with Motion to Admit Demurrer to
Zuo, SP Formaran III, Agnes P. Tuason, secretary of SP Evidence.[15] Soon, rumors circulated in the Manila City Hall
Formaran, III, and Jose H. Afalla, an employee from the Office that Judge Muro was partial towards accused Yu Yuk Lai.
of Asst. CSP (ACSP) Leonardo Guiyab, Jr. The defense on
the other hand presented ten (10) witnesses: respondent
Justice Demetria, Asst. Chief State Prosecutor (ACSP) The rumors did not end there, On 6 July 2000 unidentified
Severino Gana, Jr., Senior State Prosecutor (SSP) Romeo employees of the RTC Manila calling themselves
FINALS CONSTITUTIONAL LAW I ACJUCO NOV 11, 2017 138

"CONCERNED COURT EMPLOYEES" wrote the Secretary Thereafter, SP Formaran III went to see CSP Zuo and
of Justice, copy furnished the Chief State Prosecutor, the informed the latter of what had transpired. CSP Zuo replied,
Ombudsman, and Judge Muro. The letter alleged that Judge "No way!" SP Formaran III also told ACSP Guiyab, Jr., who
Muro ordered the hospitalization of Yu Yuk Lai "even if she gave the same reply.[22]
(was) not sick and there (was) already a rumor circulating
around the City Hall, that the notorious Judge had given the At around 3:00 o'clock that same afternoon, CSP Zuo
go signal to the counsel of the accused to file the Motion to received a call from Justice Demetria who requested him to
Quash, which (would) be granted for a consideration of instruct SP Formaran III to withdraw the motion for inhibition
millions of pesos and the contact person (was) allegedly the of Judge Muro so that the Judge could already issue an order.
daughter of the Judge, who is an employee in the said "Pakisabi mo nga kay State Prosecutor Formaran na
branch."[16] iwithdraw na iyong kanyang Motion to Inhibit para naman
makagawa na ng Order si Judge Muro," Justice Demetria was
Accordingly on 14 July 2000, SP Formaran III filed a Motion quoted as saying.[23] Politely, CSP Zuo said that he would
for Inhibition praying that Judge Muro inhibit himself see what he could do. Tingnan ko po kung ano ang
"from further handling this case and/or from resolving the magagawa ko."[24]
demurrer to evidence filed by the accused Yu Yuk Lai as
well as any other pending incidents therein."[17] On 20 July 2000, The Philippine Daily Inquirer reported that a
"Supreme Court Justice x x x and an outspoken sports person
On 16 July 2000, at around 7:30 o'clock in the morning, while and leader"[25] had been exerting "undue pressure" on the
she was supposed to be confined at the Manila Doctors DOJ to go slow in prosecuting re-arrested drug queen Yu Yuk
Hospital, accused Yu Yuk Lai was arrested inside the VIP Lai. That same afternoon, the names of Justice Demetria and
room of the Casino Filipino at the Holiday Inn Pavilion, Manila, Mr. Go Teng Kok were disclosed to the media to clear the
while playing baccarat, She was unescorted at the time of her name of the Supreme Court justices who might have been
arrest. affected by the erroneous news report. The following day, 21
July 2000, several newspapers named Justice Demetria and
On 18 July 2000, at 9:00 o'clock in the morning, the Motion for Go Teng Kok as "drug lawyers."
Inhibition of Judge Muro was heard and submitted for
resolution. Later, at around 11:30 o'clock, when SP Formaran Also on 20 July 2000 the DOJ received a Copy of an Order
III arrived in his office from the hearing, he was informed by dated 19 July 200 of Judge Muro inhibiting himself from further
his secretary, Agnes Tuason, that the staff of Court of Appeals hearing the case of Yu Yuk Lai and Kenneth Monceda.[26]
Justice Demetrio Demetria had called earlier and said that the
Justice wanted to speak with him. The caller requested for a Respondent Justice Demetria, for his part, vehemently denied
return call. As requested, SP Formaran III immediately having interceded for Yu Yuk Lai. While he admitted that he
returned the call of Justice Demetria but the Justice had indeed visited the DOJ on 18 July 2000, he went there to "visit
already gone out for lunch. old friends" and his meeting Go Teng Kok whom he did not
know until that time was purely accidental. Expectedly, Atty.
Later in the afternoon, between 1:30 and 2:00 o'clock, Justice Paas and Go Teng Kok corroborated the claim of respondent
Demetria, PATAFA President Go Teng Kok and Atty. Reinerio Justice.
Paas, lawyer of Go Teng Kok and a close friend of Justice
Demetria, went to the office of SP Formaran III in the DOJ Justice Demetria explained that he merely requested SP
which SP Formaran III shares with SP Albert Fonacier. Formaran III "to do something to help Go Teng Kok about the
Apparently, Justice Demetria was not familiar with SP case" without ever specifying the kind of "help" that he
Formaran III as he greeted SP Fonacier "Kamusta ka, requested. He averred that it was purely on the basis of
Prosecutor Formaran?"[18] erroneous impression and conjecture on the part of SP
Formaran III that he impliedly asked him to withdraw the
Soon the visitors were seated. Go Teng Kok immediately motion "because that is what Mr. Go Teng Kok was appealing
pleaded with SP Formaran III to withdraw his motion to inhibit and requesting."[27] Respondent claimed that the "help" he
Judge Muro as this would purportedly delay the resolution of was requesting could well be "within legal bounds or line of
the case. Go Teng Kok also expressed his apprehension that duty."
if Judge Muro would inhibit, a new judge might convict his
friend, accused Yu Yuk Lai, who was then already receiving Justice Demetria claimed that if ever he said anything else
bad publicity. during the discussion between Go Teng Kok and SP
Formaran III, such was not a form of intervention. He only
Justice Demetria then asked about the status of the case. SP admonished Go Teng Kok "to cool it" when the discussion
Formaran III informed the Justice that a motion for inhibition between the prosecutor and Go Teng Kok became heated.
has been submitted for resolution, one basis of which was the While he asked about the status of the case this, he said,
unsigned letter of the concerned court employees. Justice demonstrated his lack of knowledge about the case and
Demetria opined that it was a bit dangerous to anchor the bolstered his claim that he could not have possibly interceded
inhibition of a judge on an unsigned, anonymous letter. The for Yu Yuk Lai.
Justice then advised Go Teng Kok who was becoming
persistent to "keep his cool" and asked SP Formaran III if he Respondent Justice likewise argued that the bases of his
could do something to help Go Teng Kok. Apparently, prior to identification by CSP Zuo as the Justice exerting undue
18 July 2000, Go Teng Kok had already been asking SP pressure on the DOJ were all hearsay. Respondent submited
Formaran III to go slow in prosecuting accused Yu Yuk that CSP Zuo based his identification from a newspaper
Lai.[19] SP Formaran III at first politely declined the request. account, from the statement of his secretary that it was he
But later, "just to put an end to (the) conversation,"[20] he told (Justice Demetria) who was on the other end of the telephone
them that he would bring the matter to CSP Zuo. "Iyon pala," and from SP Formaran III when the latter consulted the Chief
Justice Demetria replied. The Justice then stood up, bade State Prosecutor about the visit of the Justice and Go Teng
good bye and left. Atty. Paas and Go Teng Kok followed Kok impliedly asking him to withdraw the motion.
closely behind.[21]
FINALS CONSTITUTIONAL LAW I ACJUCO NOV 11, 2017 139

In defense of respondent Justice, Atty. Paas stated that it was Appeals and a former Undersecretary and at one time Acting
actually he, not Justice Demetria, who later called up CSP Zuo Secretary of the DOJ.
to inquire about the latter's decision regarding the withdrawal
of the motion to inhibit since SP Formaran III had earlier told Even the requested "help" for Go Teng Kok, whom respondent
Go Teng Kok that the matter would be taken up with his Justice claims he did not know and met only that time, could
superiors. not have meant any other assistance but the withdrawal of the
motion to inhibit Judge Muro. True, Justice Demetria never
In fine, respondent Justice Demetria maintains that it is categorically asked SP Formaran III to withdraw his Motion.
inconceivable for him to ask SP Formaran III whom he just But when respondent Justice Demetria asked the state
met for the first time to do something for Go Teng Kok whom prosecutor at that particular time "to do something x x x to help
he claims he just likewise met for the first time. Neither did he Mr. Go Teng Kok," the latter was pleading for the withdrawal
know Yu Yuk Lai, a claim Yu Yuk Lai herself corroborated. It of the motion, and nothing else. That was the only form of
would be unthinkable for him to intercede in behalf of "help" that Go Teng Kok wanted. The subtle pressure exerted
someone he did not know. Indeed respondent Justice simply pointed to one particular act. Thus, subsequently
asserted that his meeting Go Teng Kok on 18 July 2000 at the respondent Justice called CSP Zuo to ask for just that - the
DOJ was purely coincidence, if not accidental. withdrawal of the motion to inhibit Judge Muro.

So, did respondent Justice Demetria really intercede in behalf Justice Demetria also claimed that he, together with Atty.
of suspected drug queen Yu Yuk Lai? Paas, went to the DOJ, first, to see Secretary Artemio Tuquero
and seek assistance in the appointment of Atty. Paas to the
Investigating Justice Carolina C. Grio-Aquino believes so. In Court of Appeals, and second, to "visit old friends,"[32] and
her Report dated 5 January 2001, she found respondent that the meeting with Go Teng Kok was purely accidental. But
Justice Demetria "guilty of violating Rule 2.04, Canon 2, Code respondent Justice never mentioned in his earlier Compliance
of Judicial Conduct" and recommended that "appropriate to the Memorandum of the Chief Justice that his primary
disciplinary action be taken against him by this Honorable purpose in going to the DOJ was to see Sec. Tuquero, and
Court."[28] since Sec. Tuquero was not in, he instead decided to see
some officials/prosecutors whom he had not visited for a long
time.
Only rightly so. The evidence is clear, if not overwhelming, and
damning. Thus, even the Senate Committee on Justice and
Human Rights, after a hearing, found that "there was a We find this assertion difficult to accept. For, even his very
conspiracy to commit the following offenses on the part of CA own witnesses belied his alibi. ACSP Gaa, Jr. testified and
Associate Justice Demetrio Demetria and PATAFA President confirmed that Justice Demetria only said "hi."[33] SSP
Go Teng Kok and Miss Yu Yuk Lai: obstruction of justice Daosos, denied seeing him and claimed that it was only Atty.
punishable under PD No.1829 and Article 3(a) of RA 3019, or Paas who peeped into his room.[34] Suspiciously, it was really
the Anti-Graft and Corrupt Practices Act."[29] in the office of SP Formaran III, whom respondent Justice
Demetria did not know, where Justice Demetria, Atty. Paas
and Go Teng Kok decided to "stay a while."[35]
While Justice Demetria vehemently denied interfering with the
criminal case, his denial cannot stand against the positive
assertions of CSP Zuo and SP Formaran III,[30] which are Thus, as found by Mme. Justice Carolina C. Grio-Aquino, the
consistent with natural human experience. To accept the Investigating Justice, Justice Demetria and company could
testimony of the defense witnesses that it was Atty. Paas who not have been there to exchange pleasantries with SPs
telephoned CSP Zuo, and not Justice Demetria, and that the Formaran III and Fonacier since they were not acquainted with
"help" the respondent Justice was requesting SP Formaran III each other. Prior to this incident, Justice Demetria did not
was something "within legal bounds or line of duty" other than personally know either SP Formaran III or SP Fonacier, a fact
the withdrawal of the motion is to strain too far one's corroborated by respondent himself.[36]
imagination.
All of these contradict and belie respondent Justice Demetria's
The testimony of CSP Zuo is plainly unambiguous and earlier Compliance to the Memorandum of the Chief Justice
indubitably consistent with the other facts and circumstances that "[b]ecause Prosecutor Formaran is also a friend, we
surrounding the case - decided to drop by his office x x x (and) I stayed a while."[37]

CSP Zuo: As far as I could recall Justice Demetria said, As pointed out by the Investigating Justice, respondent Justice
"Pakisabi mo nga kay State Prosecutor Formaran na was there "to join forces with Go Teng Kok in arguing for the
iwithdraw na iyong kanyang Motion to Inhibit para naman withdrawal of Formaran's Motion for Inhibition of Judge Muro,
makagawa ng Order si Judge Muro."[31] which was the real purpose of their visit to SP Formaran and
to the DOJ. The uncanny coincidence in the timing of Justice
Demetria's visit to SP Formaran's office, and that of Go Teng
In his discussion with Go Teng Kok and Justice Demetria, SP Kok, could not have been 'accidental' but pre-arranged."[38]
Formaran III said that he would consult his superiors regarding And, "visiting old friends" only came as an afterthought. The
the proposal to withdraw the motion. The timely telephone call circumstances simply show that Justice Demetria and Atty.
to CSP Zuo was thus a logical follow-up. And no one could Paas, together with Go Teng Kok, did not go to the DOJ to
have made the call except respondent Justice since it is not see Sec. Tuquero, but to visit, if not "pressure," CSP Zuo and
uncommon for anyone to believe that CSP Zuo would SP Formaran III.
recognize the voice of respondent Justice who was CSP Zuo's
former superior in the DOJ. Thus, the confident utterance
"[p]akisabi mo nga kay State Prosecutor Formaran na Justice Demetria also claimed that it is inconceivable for him
iwithdraw na iyong kanyang Motion to Inhibit para naman to help Yu Yuk Lai and Go Teng Kok, both of whom he did not
makagawa ng Order si Judge Muro" could not have come from personally know, and more unthinkable that he would be
anyone else but from respondent Justice who had moral asking help from SP Formaran III whom he had just met for
ascendancy over CSP Zuo, he being a Justice of the Court of the first time.
FINALS CONSTITUTIONAL LAW I ACJUCO NOV 11, 2017 140

The argument cannot be sustained. It is admitted that


respondent is a very close friend of Atty. Paas, lawyer of Go
Teng Kok. And, it is not necessary that respondent Justice
Demetria be acquainted with Go Teng Kok, Yu Yuk Lai or SP
Formaran III for him to intercede in behalf of the accused. It is
enough that he is a close friend of the lawyer of Go Teng Kok,
who has been helping the accused, and that he wields
influence as a former DOJ Undersecretary and later, Acting
Secretary, and now, a Justice of the Court of Appeals.

In sum, we find the testimonies of the prosecution witnesses


convincing and trustworthy, as compared to those of the
defense which do not only defy natural human experience but
are also riddled with major inconsistencies which create well-
founded and overriding doubts.

The conduct and behavior of everyone connected with an


office charged with the dispensation of justice is circumscribed
with the heavy burden of responsibility. His at all times must
be characterized with propriety and must be above
suspicion.[39] His must be free of even a whiff of impropriety,
not only with respect to the performance of his judicial duties,
but also his behavior outside the courtroom and as a private
individual.

Unfortunately, respondent Justice Demetrio Demetria failed to


live up to this expectation. Through his indiscretions, Justice
Demetria did not only make a mockery of his high office, but
also caused incalculable damage to the entire Judiciary. The
mere mention of his name in the national newspapers,
allegedly lawyering for a suspected drug queen and interfering
with her prosecution, seriously undermined the integrity of the
entire Judiciary.

Although every office in the government service is a public


trust, no position exacts a greater demand on moral
righteousness and uprightness than a seat in the
Judiciary.[40] High ethical principles and a sense of propriety
should be maintained, without which the faith of the people in
the Judiciary so indispensable in an orderly society cannot be
preserved.[41] There is simply no place in the Judiciary for
those who cannot meet the exacting standards of judicial
conduct and integrity.[42]

WHEREFORE, we sustain the findings of the Investigating


Justice and hold Justice Demetrio G. Demetria GUILTY of
violating Rule 2.04 of the Code of Judicial Conduct. He is
ordered DISMISSED from the service with forfeiture of all
benefits and with prejudice to his appointment or
reappointment to any government office, agency or
instrumentality, including any government owned or controlled
corporation or institution.

SO ORDERED.

Davide, Jr., C.J., Bellosillo, Melo, Vitug, Kapunan, Mendoza,


Panganiban, Quisumbing, Pardo, Buena, Gonzaga-Reyes,
Ynares-Santiago, De Leon, Jr., and Sandoval-Gutierrez, JJ.,
concur.

Puno, J., abroad on official business.


FINALS CONSTITUTIONAL LAW I ACJUCO NOV 11, 2017 141

RE: LETTER OF PRESIDING JUSTICE CONRADO M. On May 29, 2008, Antonio V. Rosete, Manuel M. Lopez,
VASQUEZ, JR. ON CA-G.R. SP NO. 103692 [Antonio Felipe B. Alfonso, Jesus P. Francisco, Christian S. Monsod,
Rosete, et al. v. Securities and Exchange Commission, et al.] Elpidio L. Ibaez, and Francis Giles B. Puno, as officers,
A.M. No. 08-8-11-CA directors and/or representatives of the Manila Electric
Company (hereinafter to be collectively referred to as
PER CURIAM: Meralco), filed with the Court of Appeals a petition for certiorari
and prohibition with prayer for the issuance of a writ of
preliminary injunction and temporary restraining order (TRO)
The Judiciary, which is acclaimed as the firmest pillar of our against the Securities and Exchange Commission (SEC),
democratic institutions, is vested by the Constitution with the Commissioner Jesus Enrique G. Martinez, Commissioner
power to settle disputes between parties and to determine Hubert B. Guevarra, and the Government Service Insurance
their rights and obligations under the law. For judicial System (GSIS). [3] Aside from the application for immediate
decisions, which form part of the law of the land, to be credible issuance of a TRO, petitioners prayed for the issuance of a
instruments in the peaceful and democratic resolution of preliminary injunction that should thereafter be declared
conflicts, our courts must be perceived to be and, in fact be, permanent, as well as a declaration of nullity of the cease and
impartial, independent, competent and just. To accomplish desist and show cause orders issued by the SEC through
this end, it is imperative that members of the Judiciary from its Commissioner Martinez. The petition was received by the CA
highest magistrates to its humblest employees adhere to the at 10:49 a.m. on May 29, 2008 and docketed as CA-G.R. SP
strictest code of ethics and the highest standards of propriety No. 103692.
and decorum. Indeed, it is unfortunate that one of the countrys
second highest courts, the Court of Appeals, should be
presently embroiled in scandal and controversy. It is this On the same day, petitioners simultaneously filed at 10:48
Courts bounden duty to determine the culpability or innocence a.m. an urgent motion for a special raffle. Presiding Justice
of the members of the Judiciary involved in the said Vasquez granted the motion in a handwritten note on the face
controversy and to discipline any one whose conduct has of the urgent motion,[4] and CA-G.R. No. 103692 was raffled
failed to conform to the canons of judicial ethics, which uphold to Justice Vicente Q. Roxas (Justice Roxas).[5] At 3:10 p.m.,
integrity, independence, impartiality, competence and the Office of Presiding Justice Vasquez received a letter from
propriety in the performance of official functions. Atty. Estrella C. Elamparo (Atty. Elamparo), Chief Legal
Counsel of the GSIS, requesting the re-raffling of the case in
the presence of the parties in the interest of transparency and
The present administrative matter arose from the Letter dated fairness.[6] At 4:10 p.m. on that day, the GSIS filed an ex-
August 1, 2008 of Court of Appeals Presiding Justice Conrado parte motion to defer action on any incident in the petition
M. Vasquez, Jr. (Presiding Justice Vasquez), referring to this pending the resolution of their motion for the re-raffle of the
Court for appropriate action the much publicized dispute and case.[7]
charges of impropriety among the justices of the Court of
Appeals (CA) involved in CA-G.R. SP No. 103692 entitled
Antonio Rosete, et al. v. Securities and Exchange Atty. Elamparo, accompanied by Atty. Orlando P. Polinar, also
Commission, et al. of the GSIS Law Office, personally filed the urgent motion to
defer action on the petition pending the resolution of their
motion to re-raffle the case. Since the receiving clerk of the
To assist in its investigation of this sensitive matter, the Court Court of Appeals could not assure them that the motion would
in its Resolution dated August 4, 2008 constituted a three- be transmitted to the Court of Appeals Division, Attys.
person panel (the Panel of Investigators) composed of retired Elamparo and Polinar allegedly went to the office of Justice
Justices of the Court; namely, Mme. Justice Carolina Grio- Roxas for the sole purpose of personally furnishing him a copy
Aquino as Chairperson, Mme. Justice Flerida Ruth P. Romero of the motion.[8] They initially talked to a male clerk who
and Mr. Justice Romeo J. Callejo, Sr. as Members. The Panel referred them to one of the lawyers, who, however, told them
of Investigators was tasked to investigate the (a) alleged that it was not possible for them to personally hand a copy of
improprieties of the actions of the Justices of the Court of the motion to Justice Roxas. Thus, Attys. Elamparo and
Appeals in CA-G.R. SP No. 103692 (Antonio V. Rosete, et al. Polinar left a copy of the motion to the staff but no one wanted
v. SEC, et al.); and (b) alleged rejected offer or solicitation of to sign and acknowledge receipt of the copy.[9]
bribe disclosed respectively by Mr. Justice Jose Sabio and Mr.
Francis de Borja.
On May 30, 2008, Justice Reyes filed an application for the
extension of his leave until June 6, 2008.[10] In the meantime,
A narration of relevant events and facts, as found by the Justice Mendoza, who had been designated to replace Justice
Investigating Panel, follows: Reyes during the latters absence, informed Justice Roxas
through a letter that he (Justice Mendoza) was inhibiting from
On April 15, 2008, Justice Bienvenido L. Reyes (Justice the case on the ground that he used to be a lawyer of the
Reyes), then Chairperson of the Ninth Division of the CA, filed Meralco.[11] Hence, in an Emergency Request for Raffle,
an application for leave from May 15, 2008 to June 5, 2008.[1] Justice Roxas informed the Raffle Committee about the
inhibition.[12]
In Office Order No. 149-08-CMV dated May 14, 2008 issued
by Presiding Justice Vasquez, Justice Jose C. Mendoza Justice Jose L. Sabio, Jr. (Justice Sabio) was assigned as
(Justice Mendoza) was designated by the Raffle Committee Acting Chairman of the Ninth Division by raffle, in lieu of
as Acting Chairman of the Ninth Division during the absence Justice Mendoza.[13] At 11:30 a.m., the office of Justice
of Justice Reyes. Apart from his duties as regular senior Myrna Dimaranan-Vidal (Justice Dimaranan-Vidal) received a
member of the Fifth Division, Justice Mendoza was authorized notice of emergency deliberation with the new Acting
to act on all cases submitted to the Ninth Division for final Chairman of the Special Ninth Division, apparently sent by
resolution and/or appropriate action, except ponencia, from Justice Roxas, stating that her presence and that of Justice
May 15, 2008 to June 5, 2008 or until Justice Reyes reports Sabio, Jr. were indispensable on account of the national
back for duty. The said office order likewise applied to the interest involved in CA-G.R. SP No. 103692.[14]
other Division(s) where Justice Reyes had participated or took
part as regular member or in an acting capacity.[2]
FINALS CONSTITUTIONAL LAW I ACJUCO NOV 11, 2017 142

Meanwhile, Atty. Elamparo received a telephone call from had taken up the cudgels for the long suffering public, who are
somebody who did not identify herself but (who) said that she at the mercy of MERALCO.
had important information regarding the Meralco case. The
unidentified caller told Atty. Elamparo that a TRO was already x x x x x x x x x.
being prepared and that certain Meralco lawyers had in fact
been talking to Justice Roxas. The caller warned Atty.
Elamparo against Justice Roxas who had administrative
cases and was very notorious, but when prodded, the caller
would not disclose more details.[15] Immediately, I tried to contact Justice Sabio. But due to the
noise I could not hear him. So I waited until we would arrive in
Manila.

At about 1:30 p.m. also on May 30, 2008, Justice Sabio As we were leaving the Airport, I again got in touch with
received a telephone call in his chambers from his older Justice Sabio. After, he confirmed that he was in fact in the
brother, Chairman Camilo Sabio (Chairman Sabio) of the Division to which the petition of MERALCO had been raffled.
Presidential Commission on Good Government (PCGG).[16] I impressed upon him the character and essence of the
Chairman Sabio informed his brother that he (Justice Sabio) controversy. I asked him to help GSIS if the legal situation
had been named the third member of the division to which the permitted. He said he would decide according to his
MERALCO-GSIS case had been raffled. Justice Sabio was conscience. I said: of course.
surprised as he had not yet been officially informed about the
matter. Chairman Sabio likewise informed him that a TRO had x x x x x x x x x.
been prepared. Chairman Sabio then tried to convince Justice
Sabio of the rightness of the stand of the GSIS and the SEC, On the same day, May 30, 2008, GSIS filed an urgent ex-parte
and asked his brother to help the GSIS, which represents the motion to inhibit Justice Roxas from CA-G.R. No. SP
interest of the poor people. Justice Sabio told his brother that 103692.[18] The Special Cases Section of the Court of
he would vote according to [his] conscience and that the most Appeals received a copy of the motion at 11:58 a.m.[19]
that he could do was to have the issuance of the TRO and the
injunctive relief scheduled for oral arguments, at which the
respondents must be able to convince him that the TRO Claiming that the TRO was issued to pre-empt the hearing
indeed had no legal basis. scheduled in the afternoon of that day before the SEC, the
GSIS Law Office, through Atty. Marcial C. Pimentel, Jr., set
forth its reason for the motion for inhibition as follows:

3. Unfortunately, reports have reached respondent GSIS that


In his signed testimony,[17] which he read before the Panel of the Honorable ponente has been in contact with certain
Investigators, Chairman Sabio narrated the circumstances of lawyers of MERALCO and has in fact already prepared a draft
this call to his brother on May 30, 2008. It appears to have resolution granting the TRO without affording respondents
been prompted by a call from a member of the Board of even a summary hearing. The records of this case was (sic),
Trustees of GSIS. To quote from Chairman Sabios testimony: per information, immediately transmitted to the Honorable
ponente upon his instructions. The worries of the respondent
Last May 30, 2008 I was in Davao City Airport with my wife, were exacerbated when it learned that there are supposedly
Marlene, waiting for our 1:25 P.M. PAL flight to Manila. xxx two administrative cases pending against the Honorable
xxx xxx. ponente, both of which involve allegations of bias and
prejudice.
As we were boarding, I received a call from Atty. Jesus I.
Santos, a Member of the Board of Trustees of GSIS. We had It turned out, however, that at that time, Justice Roxas had not
known each other and had become friends since before yet been officially notified by the Raffle Committee that the
Martial Law because as Chief Counsel of the Federation of case was raffled to him.[20] Moreover, contrary to the
Free Farmers (FFF) we were opposing counsel in various allegation of Atty. Elamparo that the raffle was rigged, Justice
cases in Bulacan. Roxas had no hand in the raffle proceeding, which was
handled by the Division chaired by Justice Mariano del Castillo
Attorney Santos informed me that the dispute between the with the use of a fool-proof Las Vegas tambiolo, like the lotto
GSIS and MERALCO was now in the Court of Appeals; and, machine.[21]
that as a matter of fact, my brother, Justice Sabio, was chair
of the Division to which the case had been assigned. Being a Justice Roxas brought to the office of Justice Sabio, for the
Trustee, Attorney Santos requested me to help. I readily latters signature, the TRO which he had prepared, already
welcomed the request for help and thanked him. There was signed by himself and Justice Dimaranan-Vidal. Convinced of
no mystery about his having known of the results of the raffle the urgency of the TRO, Justice Sabio signed it on condition
because the lawyers are notified thereof and are present that the case will be set for oral arguments.
thereat. As a Trustee, Attorney Santos should be concerned
and involved. As such it is his duty to seek assistance for the Thus, at 2:08 p.m. on May 30, 2008,[22] the Special Ninth
GSIS where he could legitimately find it. He was right in Division composed of Justices Sabio, Roxas, and Dimaranan-
seeking my assistance Vidal, issued the Resolution granting the TRO prayed for by
the petitioners and directing the respondents to file their
I was aware of the controversy between the GSIS and respective comments (not a motion to dismiss) to the petition
MERALCO. In essence this was in fact a controversy between within ten days from notice, with the petitioners given five days
the long suffering public and the mighty financially and from receipt of that comment within which to file their reply.
politically controlling owners of MERALCO. MERALCO is not The Special Ninth Division also set the hearing on the
only a public utility but also a monopoly. Fortunately, GSIS application for the issuance of a writ of preliminary injunction
for 10:00 a.m. on June 23 and 24, 2008. In the same
FINALS CONSTITUTIONAL LAW I ACJUCO NOV 11, 2017 143

Resolution, parties were directed to file their respective On either June 17 or 18, 2008, Justice Sabio requested the
memorandum of authorities in connection with the application rollo of CA-G.R. SP No. 103692 from Justice Roxas so that
for a writ of preliminary injunction together with their he could study the case before the hearing.[34]Justice Roxas
comments/reply. After the parties had filed their memorandum asked him whether Justice Reyes would preside over the
of authorities relative to the application for a writ of preliminary hearing. Justice Sabio explained the reason why he, not
injunction, the prayer for the said writ would be considered Justice Reyes, should preside. Justice Roxas promised to
submitted for resolution forty five (45) days from promulgation instruct the Division Clerk of Court to send the rollo over to
of this Resolution. The SEC received a copy of the Resolution Justice Sabio. The next day, the Division Clerk of Court told
at 4:03 p.m. on that day.[23] Justice Sabio that the rollo was with Justice Reyes. When the
rollo was eventually transmitted to Justice Sabio, the Division
For Justice Roxas, the issuance of the TRO was an implied Clerk of Court asked him whether the rollo should be with
denial of the motion for inhibition filed against him. There was Justice Reyes. Justice Sabio explained why the rollo should
no need to put in writing the action on the motion for be with him.
inhibition.[24]
On June 18, 2008, petitioners filed a motion for an extension
At 3:00 p.m., the Special Cases Section of the Court of of five days or until June 23, 2008 within which to file their
Appeals received the Urgent Motion to Lift Temporary consolidated memoranda of authorities and reply to the
Restraining Order and To Hold Its Enforcement in Abeyance comment of the SEC.[35]
filed by the GSIS.[25] Justice Roxas did not act on the Urgent
Motion because he did not consider it meritorious.[26] On June 19, 2008, MERALCO filed an ex-parte manifestation
together with their reply to the comment of the GSIS.[36]
On May 31, 2008, Justice Sabio received a cellular phone call Meanwhile, Justice B. L. Reyes asked Atty. Custodio to report
from Mr. Francis De Borja (Mr. De Borja), a person he had lost on what transpired between her and Justice Sabio when she
contact with for almost a year already.[27] Mr. De Borja returned the cartilla. Teary-eyed, Atty. Custodio begged off
greeted him with: Mabuhay ka, Justice. When Justice Sabio, from making a report.[37]
Jr. asked Mr. De Borja why he said that, Mr. De Borja told him
that the Makati Business Club was happy with his having Justice Reyes decided to consult the Presiding Justice to
signed the TRO, to which Justice Sabio retorted, I voted avoid an ugly confrontation with the Justices on the highly
according to my conscience. politicized case involving giants of the Philippine society. He
explained to the Presiding Justice his understanding of the
On June 5, 2008, the GSIS Law Office received a letter dated relevant IRCA rules and the actual practice in similar
June 2, 2008 of Presiding Justice Vasquez, Jr. informing GSIS situations in the past. The Presiding Justice promised to talk
Chief Legal Counsel, Atty. Elamparo, that the Court of with Justice Sabio and, for the sake of transparency and future
Appeals could not grant her request for the re-raffling of CA- reference, Justice Reyes requested permission to write an
G.R. SP No. 103692 in the presence of the parties in the inquiry on the matter.[38]
interest of transparency and fairness, as the case had been
raffled in accordance with the procedure under the IRCA.[28] On the same day, Justice Reyes wrote Presiding Justice
Vasquez a letter[39] calling the attention of Justice Edgardo
On June 10, 2008, Justice B. L. Reyes reported back to P. Cruz (Justice Cruz), Chairperson of the Committee on
work.[29] Rules, to the dilemma as to who between him and Justice
Sabio should receive CA-G.R. SP No. 103692. Justice Reyes
posed these questions before the Presiding Justice:
On June 11, 2008, at 3:50 p.m.,[30] the Office of the Solicitor
General (OSG), appearing for the SEC, filed a manifestation
and motion praying for the admission of the comment (to the Will the case remain with Justice Jose Sabio, Jr. as Acting
petition) attached thereto, as well as the advance and Chairman of the Special 9th Division and who participated in
additional copies of the memorandum of authorities. the initial Resolution of the case?

On June 12, 2008, at 4:53 p.m., the GSIS filed its Will the case revert to the regular 9th Division with the
comment/opposition to the petition in CA-G.R. SP No. undersigned as Chairman?
103692,[31] as well as its memorandum of authorities.
For Justice Reyes, the dilemma was engendered by this
On June 16, 2008, the Division Clerk of Court, Atty. Teresita provision of Section 2 of Rule VI of the IRCA:
Custodio (Atty. Custodio), delivered to Justice Reyes the
cartilla of the Meralco case, and informed him that a hearing (2) When, in an original action or petition for review, any of
on the prayer for the issuance of a preliminary injunction had these actions or proceedings, namely: (1) giving due course;
been scheduled at 10:00 a.m. on June 23 and 24, 2008.[32] (2) granting writ of preliminary injunction; (3) granting new trial;
However, on the same day, the Division Clerk of Court came and (4) granting execution pending appeal have been taken,
back to retrieve the cartilla upon instructions of Justice Sabio. the case shall remain with the Justice to whom the case is
Justice Reyes instructed his staff to return the cartilla and assigned for study and report and the Justices who
when he asked the Division Clerk of Court why she was participated therein, regardless of their transfer to other
retrieving it, she said that Justice Sabio demanded that it be Divisions in the same station.
returned back to him. Personally affronted by the domineering
and superior stance of Justice Sabio, Justice Reyes read and The hearing on the application for preliminary injunction
re-read Secs. 1, 2(d) & 5, Rule VI (Process of Adjudication) having been scheduled for June 23 and 24, 2008, Justice
until he was satisfied that he should sit as Division Chairman Reyes considered it necessary that the issues be resolved
in the Meralco case.[33] before that date. Moreover, the referral of the controversy to
the Presiding Justice would give him sufficient time to
seriously study the case before the hearing.[40]
FINALS CONSTITUTIONAL LAW I ACJUCO NOV 11, 2017 144

not have been leaked by Justice Bato but by a member of the


Division.[48]
On June 20, 2008, Presiding Justice Vasquez referred the
letter of Justice Reyes to Justice Cruz, Chairperson of the The Presiding Justice did not do anything anymore to prevent
Committee on Rules, noting some urgency involved as the an unpalatable situation at the scheduled June 23, 2008
hearing of the case is on Monday, June 23, 2008.[41] hearing, notwithstanding the conflicting opinions of Justices
Reyes and Sabio. The personal view of the Presiding Justice
On that same day, Justice Cruz wrote Justice Reyes a was at the time with Justice Cruz but Justice Sabio had a
letter[42] quoting Section 2 (d), Rule VI of the IRCA and different interpretation. Neither did the Presiding Justice
stating that the [i]ssuance of a TRO is not among the suggest that the Rules Committee be convened because the
instances where the Justices who participated in the case Committee then had only two members. He felt that it would
shall remain therein. Hence, Justice Cruz opined that be better if Justices Reyes and Sabio could settle it between
[n]otwithstanding the issuance of the TRO (not writ of themselves. The Presiding Justice was seeing the Justices
preliminary injunction), the case reverted to the regular practically everyday because he did not want these things to
Chairman (Justice Reyes) of the Ninth Division upon his blow up. However, neither did it enter the mind of the
return. Justice Reyes received a copy of the letter of Justice Presiding Justice that the hearing on June 23 could be reset.
Cruz in the afternoon of that day.[43] Had he known that there was a motion to inhibit Justice
Roxas, he would have changed his position that it should be
the Sabio group.[49]
During the hearings of this case, Justice Cruz explained his
opinion before the Panel. He opined that the motion to lift the
TRO is not a motion for reconsideration because Rule 52 of Also on June 20, 2008, the GSIS requested permission to
the Rules of Court states that a motion for reconsideration conduct a power-point presentation during the hearing.[50]
may be filed with respect to a decision or a final resolution. A Likewise the SEC, through the OSG prayed that it be allowed
TRO is not a final resolution but an interlocutory order. the use of Microsoft Powerpoint Application at the June 23 and
Moreover, since the subject of the hearing on June 23, 2008 24, 2008 hearings.[51] Justice Roxas did not act on the
was on the application for preliminary injunction, Justice Sabio motions.
had no right to participate in the hearing because as an Acting
Chairman, his authority was only to act on the motion to lift the On June 21, 2008, Justice Sabio came to know that it was the
TRO. Under the IRCA, the position of Justice Sabio invoked Division chaired by Justice Reyes that would handle the case
the exception to the general rule in the IRCA. However, the on account of the opinion of Justice Cruz.[52]
settled principle is to construe a rule strictly against the
exception. The participation of Justice Sabio in the hearing on In the morning of June 23, 2008, Justice Sabio consulted with
June 23, 2008 was a passport to participation in the decision- Justice Martin Villarama, Jr. (Justice Villarama) who advised
making process, in violation of the IRCA.[44] him, in no uncertain terms, that his stand was correct and that
he should remain in the case.[53] Justice Villarama said that
Justice Reyes having consulted with him, the Presiding the case should remain with the Special Ninth Division
Justice referred the matter to Justice Sabio who in turn, opined regardless of the transfer of the ponente to the Eighth Division
that a temporary restraining order is part of the injunctive relief because of the pending motion to lift TRO, which the Special
or at least its initial action such that he should be the one to Ninth Division should resolve following the general rule that
chair the Division. [45] In his office after that consultation with when a decision or resolution is rendered by a division, a
the Presiding Justice, Justice Reyes found that the Division motion for reconsideration thereof should be acted upon by all
Clerk of Court had given him a copy of the cartilla just in case the Members of that division, whether regular or special, which
he would preside over the hearing. In the evening, the participated in the rendition of the decision or resolution,
Presiding Justice called up Justice Reyes to inform him that except in case of death, retirement or resignation of such
Justice Sabio insisted that he would preside over the hearing Member.[54]
of the case, and that the opinion of Justice Cruz, who was
junior to Justice Sabio was no better than his own opinion.[46] That morning, Justice Roxas also consulted Justice Villarama.
The latter told the former that since there was a motion to lift
It turned out that, upon receipt of a copy of the letter of Justice the TRO, Justice Roxas should first rule on the motion. He
Cruz, Justice Sabio told the Presiding Justice by telephone also advised Justice Roxas to inhibit himself from the case, as
that he disagreed with the opinion of Justice Cruz because he there might be a problem (mag-inhibit ka baka magka-
did not sign in an official capacity as Chairman of the Rules problema). Justice Roxas told Justice Villarama that he would
Committee, but in his personal capacity and hence, the follow his suggestion.[55]
opinion of Justice Sabio was as good as his, as in fact I
(Justice Sabio, Jr.) am even more senior than he.[47] Justice Justice Reyes also went to the office of Justice Villarama to
Sabio told the Presiding Justice that he smelled something tell him of his strong conviction that the issuance of a TRO is
fishy about the move to transfer the case to the Ninth Division not among the instances provided in Sec. 2 (d), Rule VI when
especially because Justice Reyes did not inform him about it the case shall remain with those Justices who participated in
despite the fact that they were seated together on three the case regardless of their transfer to other division(s).
occasions. Justice Villarama told Justice Reyes that per his
understanding and interpretation of said provision, x x x the
Justice Sabio smelled something fishy because a couple or so case should remain with the Special Ninth Division.[56]
weeks ago, he attended a Chairpersons meeting regarding
the leakage of the ponencia of Justice Bato, with Justice At 9:50 a.m., the Office of the Division Clerk of Court called
Reyes as Chairperson and Justice Jose Mendoza as senior Justice Reyes to inform him that the parties and their counsels
member. The meeting was called because prior to the were already in the hearing room. Justice Reyes informed the
promulgation of the decision of Justice Bato, the losing party caller that he could not preside as Justice Sabio had
already filed a motion for the inhibition of the ponente. apparently hardened his position and he wanted to avoid an
According to Justice Sabio information on the decision could ugly spectacle. His name plate was displayed in the hearing
FINALS CONSTITUTIONAL LAW I ACJUCO NOV 11, 2017 145

room but Justice Sabio moved to another hearing room.[57] That same afternoon, Mr. De Borja again called up Justice
Allegedly, the removal of the nameplate of Justice Reyes was Sabio, seeking to meet with him for an important matter.
the talk of the Court of Appeals for weeks.[58] Because Justice Sabio had 6-8 p.m. classes at the Ateneo
Law School, they agreed to meet after his classes but not for
Villaraza Cruz Marcelo and Angangco entered its appearance long because his wife and his daughter, Atty. Silvia Jo Sabio
as counsel for Meralco.[59] At the hearing, Justice Sabio who is an Attorney VI in the Office of the Chief Justice,[70]
presided with Justices Roxas and Dimaranan-Vidal in would be waiting for him.[71] According to Justice Sabio, the
attendance. Justice Roxas, the ponente, did not ask a single conversation at that meeting with Francis de Borja went as
question.[60] Not one of the Justices in attendance brought up follows:
the motion for inhibition filed by the GSIS against Justice
Roxas.[61] In open court, the parties in CA-G.R. SP No. 17. By the time my class was finished at 8 pm, Mr. De Borja
103692 agreed to submit, within 15 days, simultaneous was already waiting for me at the Lobby Lounge of the 3rd
memoranda on the injunctive relief prayed for by the Floor of the Ateneo Law School. His first words to me were:
petitioners, after which the application for preliminary Alam mo Justice kung sino ang kasama ko sa kotse? Si
injunction would be deemed submitted for resolution.[62] Manolo Lopez. Then he said: Noong tinatawagan kita at sinabi
kong Mabuhay ka Justice, si Manolo Lopez ang katabi ko
On June 25, 2008, or about two days after the separate noon. Nasa Amerika siya, kaya ako na lang ang pumunta dito
conversations of Justice Villaram with Justices Sabio and para makiusap sa yo. Alam mo, itong kaso na ito is a matter
Reyes, the Presiding Justice also consulted Justice Villarama of life and death for the Lopezes. And alam mo naman what
about the letter-queries of Justices Roxas and Reyes on which the Marcoses did to them, which is being done now by the
Division should resolve the matter of injunctive relief or issue Arroyos.
the decision in CA-G.R. SP No. 103692.[63]
At that point he mentioned the impasse between Justice
The Presiding Justice issued Office Order No. 196-08-CMV Bienvenido Reyes and myself. He said: Alam naming may
reconstituting the Committee on Rules and designating problema kayo ni Justice Reyes tungkol sa chairmanship.
Justice Cruz as the Chairperson, with Justices Rebecca De
Guia-Salvador, Reyes, Hakim Abdulwahid, and Noel G. Tijam, I was surprised how he came to know about it, as this
as members.[64] The Committee on Rules was tasked to was an internal matter of the Court of Appeals which only
propose amendments to the IRCA on or before August 15, happened fairly recently and many associate justices of the
2008 for submission and adoption of the Court en banc. (The CA were not even aware of this. Just the same, I explained
office order was later amended by Office Order No. 196-08- my stand and why I could not relinquish the chairmanship to
CMV on August 4, 2008 to include as members Justices Mario Justice Reyes.
L. Guaria III, Lucas P. Bersamin, and Teresita Dy-Liacco
Flores.[65]) The Rules Committee used to be composed of He then replied: Alam mo, Justice ang opinion dito ni
only three members, namely: Justices Cruz, Abdulwahid, and Nonong Cruz ay i-challenge ang stand mo. Kaya lang,
Roberto Barrios, now deceased, as members, with Justice mayroon namang nagsabi na it might become messy.
Cruz as chairperson.[66]
Then he bragged to me: Ako din ang responsible sa pag-
It was also on June 25, 2008 that Presiding Justice Vasquez recommend at pag-hire ng Villaraza Law Firm.
issued Office Order No. 200-08-CMV stating that, in view of
the retirement of Justices Enrique Lanzanas, Lucenito N.
Tagle, Agustin S. Dizon, and Rodrigo Cosico, and the Then he explained that he was there to offer me a win-
appointments of Justices Ruben C. Ayson and Edgardo L. win situation.
delos Santos, the Divisions would have a new composition
effective July 4, 2008.[67] Under that office order, Justice He said: Justice, mayroon kaming P10 million. Ready.
Sabio became the Chairman of the Sixth Division, with Justice Just give way to Justice Reyes.
Dimaranan-Vidal as a member. Justice Reyes became the
Chairman of the Eighth Division, with Justices Roxas and Then I said: Bakit ganun. Nakasisiguro sila sa kanya, sa
Apolinario D. Bruselas, Jr. (Justice Bruselas) as members. akin hindi?

He said: Mas komportable lang sila sa kanya.

On June 29, 2008, Justice Reyes went on official leave of At that point, I was shocked that he had a very low regard
absence to use a business class airplane ticket to Sydney, for me. He was treating me like there was a price on my
Australia that he had won in an APT Golf Tournament in person. I could not describe my feelings. I was stunned. But
January 2008. He was still on official leave when the at the same time, hindi ko rin magawang bastusin siya
reorganization of the Court of Appeals took place on July 4, because I had known him since 1993 and this was the first
2008.[68] time that he had ever treated me like this, or shown that he
believed I could be bought.

So I just told him: Francis, I cannot in conscience agree


On July 1, 2008, Justice Roxas told Justice Sabio that he did to that.
not attend the Access to Courts (sic) summit on June 30 and
July 1, 2008 at the Court of Appeals Auditorium because he His answer was: Sabi ko nga sa kanila, mahirap ka
was busy with the Meralco case. Justice Sabio was taken talaga papayag. Kasi may anak iyang Opus Dei. Numerary
aback because at that time the parties had not yet submitted pa.
their memoranda.[69]
FINALS CONSTITUTIONAL LAW I ACJUCO NOV 11, 2017 146

At this point, I just wanted to leave, so I told him I could acquaintance of Mr. De Borja at the Ateneo grade school. Mr.
not stay long. I told him my wife and lawyer daughter were Lopez did not ask him (Mr. De Borja) to contact Justice Sabio.
waiting. At a party where Mr. De Borja met Mr. Lopez, Mr. De Borja
informed him that he knew Justice Sabio but Mr. Lopez did not
Even then, he was already insistent. His parting words say anything.
before I left were: Just think about it, Justice.[72]

Mr. De Borja denied having offered P10 million to Justice


At that time, Mr. De Borja was carrying a sealed brown paper Sabio. Instead, he claimed that Justice Sabio informed him
bag, which he was handling as if something important was that the government has offered him (Justice Sabio) money
inside. However, Justice Sabio did not know if the bag and a promotion to the Supreme Court to favor GSIS. When
contained P10 million.[73] Mr. De Borja asked what would it take for Justice Sabio to
resist the governments offer, Justice Sabio allegedly replied:
Fifty Million.[76] He alleged that it was Justice Sabio who
In his car, Justice Sabio told his wife and his daughter, Silvia called up after that July 1, 2008 meeting to feel his reaction to
Jo, about the offer of Mr. De Borja for Meralco.[74] the P50 million solicitation. Justice Sabio asked him: O, ano,
kumusta, ano ang nangyayari.

In his affidavit submitted to the Panel of Investigators, Mr. De


Borja describes himself as a businessman, a deal maker, and Mr. De Borja admitted having given P300,000 to Justice
project packager. On July 1, 2008, he invited Justice Sabio for Sabio, some 15 years ago, as a balato because he came to
dinner to touch base and for chismis about the MERALCO- value the friendship of Justice Sabio that developed while the
GSIS case. As the latter would have evening classes at the latter was helping the Roa family in a business transaction.
Ateneo Law School, and his wife and daughter would be Mr. De Borja earned more than P25 million although he
waiting in their car after his classes, they just agreed to meet received only P3 million as down payment out of the sale of
at the lobby-lounge of the School. What Mr. De Borja knew 100 hectares of the Roa property. He gave the balato of 10%
about the MERALCO case allegedly came from news reports of the P3 million to Justice Sabio in cash at the Roa-owned
but he was interested in the news because he is a confirmed bank in Cagayan de Oro. Since the Roas had a lot of legal
free-enterpriser. Moreover, De Borja thought that there was problems, Justice Sabio rendered advice and consultation at
[n]othing like hearing things directly from the horses the time that he was an RTC judge in Cagayan de Oro. After
mouth.[75] the promotion of Justice Sabio to the Court of Appeals, Mr. De
Borja invited him for dinner. They would see each other at get-
togethers of the Roas with whom Mr. De Borja is related, even
at a gathering in the house of Mr. De Borjas mother.[77]
When Mr. De Borja and Justice Sabio met, Mr. De Borja
averred he was indeed carrying a bag, not an expensive
looking luggage. After parking his car at the Rockwell
basement, he took the escalator, intending to walk out of the On July 2, 2008, Justice Sabio that informed Presiding Justice
mall. On his way, he passed by the Kenneth Cole shop and, Vasquez that he (Justice Sabio) was offered a bribe (which he
since it was still early, he looked in and saw a T-shirt he liked. rejected) to have him ousted from the Meralco case. The news
He bought the T-shirt, which he brought before the Panel of allegedly shocked the Presiding Justice. Justice Sabio also
Investigators in the grey Kenneth Cole Reaction bag. The went to Justice Villarama who was both shocked and amused.
photographs of the bag and the T-shirt costing P1,650.00 are Justice Sabio. did not tell them who the offeror was. However,
marked Exhibits A-De Borja and A-1-De Borja and attached to a day or two later, Justice Sabio found out that Mr. De Borja
the rollo of A.M. No. 08-8-11-CA, while the photograph of the had called their mutual friend, Mrs. Evelyn Clavano, who was
receipt issued by the Kenneth Cole Boutique, marked as also shocked that Mr. De Borja had the gall to ask her to
Exhibit A-2-De Borja, shows that the purchase was made on convince Justice Sabio to accept the bribe.[78]
July 1, 2008 at 19:47. He stressed the bag did not contain P10
million.

Although Justice Sabio told the Presiding Justice that the offer
of P10 million to a Justice was, in the words of Justice Sabio,
Before the Panel, Justice Sabio claimed that the bag Mr. De bastusan na ito, and he knew that bribing a Justice is a
Borja brought during the hearing was not the bag that Mr. De criminal act, the Presiding Justice did nothing because he
Borja was carrying when Justice Sabio saw him on July 1, could not advise a fellow Justice on what to do the Justice
2008. What Mr. De Borja allegedly brought with him to the would know what he should do. Neither did he think of
lobby-lounge of the Ateneo Law School was a brown bag with consulting Justices Roxas and Dimaranan-Vidal on the
paper handle about 2/3 (of the Kenneth Cole bag) in size. chairmanship impasse.[79]
Justice Sabio was told by the Panel that it could be the subject
of rebuttal evidence but he did not present such evidence.

On July 3, 2008, to stop Mr. De Borja from pestering him with


phone calls and text messages, Justice Sabio called up Mr.
According to Mr. De Borja, Manolo Lopez (Mr. Lopez), the De Borja who told him: Mabuti naman Justice tumawag ka,
owner of MERALCO whose wife was a member of Marthas kasi malapit na ang deadline ng submission ng memorandum.
Vineyard just like Mr. De Borjas wife, was also an Pinag-isipan mo bang mabuti ang offer namin? Kasi sayang
FINALS CONSTITUTIONAL LAW I ACJUCO NOV 11, 2017 147

din kung di mo tatanggapin, Kasi kahit aabot itong kaso sa


Supreme Court, matatalo ka din. Sayang lang yung P10
million. Baka sisihin ka pa ng mga anak mo. Shocked by what Since July 4, 2008, Justice Bruselas alleged that he acted on
he heard, Justice Sabio said No. Since Mr. De Borja did not all the ponencias of Justices Reyes and Roxas, just as they
seem to understand why he kept saying No, Justice Sabio had acted on his ponencias.[86]
explained to him: If I accept that, my conscience will bother
me forever. How can I face my wife and two daughters? One
a lawyer and the other a Numerary member of Opus Dei? And
besides, how can I reconcile my being a member of PHILJAs
Ethics and Judicial Conduct Department; being a lecturer of On July 7, 2008, the GSIS filed its memorandum.
the MCLE; and being a pre-bar reviewer of the Ateneo Law
School on Legal and Judicial Ethics? Mr. De Borja retorted:
Wala naman kaming pinapagawa sa iyo na illegal, eh. Then
he added: You know Justice, after two or three weeks,
makakalimutan na ito ng mga tao. Meron naman diyang mga On or about July 8, 2008, Atty. Silvia Sabio, to help her father,
Atenista na tumatanggap. Justice Sabio said: I dont know sought the advice of Atty. Jose Midas Marquez (Atty.
about them, but I am different. Mr. De Borja then said: Well, if Marquez) regarding the bribery attempt. Atty. Marquez
you will not accept, we will be forced to look for other ways. advised that Justice Sabio should write the Chief Justice about
To this, Justice Sabio said: But they will have to contend with the incident, detailing not only the bribery attempt but all that
me. In parting, Mr. De Borja said: Justice, no matter what, has transpired relative to the chairmanship issue. Atty. Silvia
saludo talaga ako sa iyo. Sabio immediately called her father and relayed Atty.
Marquezs advice. Later that date, Justice Sabio handed his
daughter, Silvia, a handwritten letter for her to deliver to the
Chief Justice.[87] The handwritten letter, in essence,
requested permission for Justice Sabio to unburden himself
Mr. De Borja admitted that Justice Sabio called him up, but before the Chief Justice on the Meralco case.[88]
denied the above conversation with Justice Sabio.

At around 2:30 p.m., Justice Reyes went to see Justice Sabio.


On July 4, 2008, the reorganization of the Court of Appeals The conversation between them, as recalled by Justice Sabio,
became effective and brought Justices Reyes, Roxas and was as follows:
Bruselas to the Eighth Division. Justice Reyes went to see the
Presiding Justice about the urgent motion for him to assume As soon as he came in, I said: Why did you stab me
the chairmanship of the Division, which shows on its face that behind my back? He said, Why, what did I do? I asked him
the Urgent Motion dated July 10, 2008 was received by the Why is it that you have to resort to that strategy of seeking the
Court of Appeals at 2:08 p.m. on July 10, 2008 and by Atty. opinion of Ed Cruz, in his personal capacity, when we could
Teresita C. Custodio on July 9, 2008. Justice Reyes have discussed the matter with the PJ?
expressed to the Presiding Justice his apprehension that
should he fail to assume the chairmanship, he would face
administrative liability for nonfeasance or dereliction of duty. I reminded him that we were seated three times near
The Presiding Justice suggested that the respondents in the each other on different occasions only recently and he never
case be required to comment on the Urgent Motion in a mentioned to me about the plan to oust me.
resolution to be issued by the former 9th Division of Justice
J.L. Sabio, Jr. since to allow the new Division of Justice B.L. He said: Perhaps that was my fault. I should have talked
Reyes to issue the resolution x x x would render moot and to you.
academic the same motion. Justice Reyes agreed and told the
Presiding Justice that he would be sending over the records
I told him, that all the while I thought we were friends.
to him so that the Presiding Justice could place a note thereon
Why did you have to do these things behind my back and not
as to what had been agreed upon. However, the records of
discuss the matter with me face to face?
the case did not reach the Presiding Justice.[80]

Then he said it just came about due to the urgent motion;


that he was afraid Meralco would take action against him for
nonfeasance for not doing his job.
For Justice Roxas, the July 4, 2008 reorganization was
mandatory and the Meralco case followed him as its ponente
It was then that I said: Are you aware that I was offered
to the Eighth Division. By the reorganization, Justice Sabio
10M for me to give way to you?
was moved from the disbanded Special Ninth Division to the
Sixth Division, as the reorganization did not spare any
Justice.[81] Moreover, the IRCA does not require that the I further asked him the following: In the first place, how
Justices that issued a TRO be the same Justices that will was the Meralco emissary able to know that there was an
render the decision.[82] This is because the TRO does not impasse between you and me when that was supposed to be
appear in Section 2 (d), Rule VII of the IRCA. Accordingly, only an internal matter?
the issuance of a preliminary injunction could be an exception
to the July 4, 2008 reorganization of the CA.[83] He believes → If you will now insist on assuming the chairmanship after I
the IRCA does not require that the Justices who heard the told you of the 10Million offer, what will I think of you?
case should also decide it because the CA is a court of record
and Justices may rely on the transcript of stenographic
→ Are you a Trojan horse? Can you blame me if I think you
notes.[84] And so, once the three Justices have signed the
are part of this whole scheme or shenanigan?
decision, the ponente has the pressing duty to promulgate the
decision.[85]
FINALS CONSTITUTIONAL LAW I ACJUCO NOV 11, 2017 148

→ Does not the timing alone stink of corruption? After they TRO.[90] She also concurred with the portion of the decision
failed to convince me of their offer, now they will use you to recommending administrative sanctions against the GSIS
oust me? Is it because they are certain of your loyalty and they lawyers because she believed the OSG or the OGCC should
are uncertain with mine? have appeared for the GSIS.[91]

→ And why did they file this stupid urgent motion to assume?
In my nine years in this court, I have never seen such an
animal as this. This is a cowardly act, and whoever advised Also late that day, Justice Villarama told Justice Sabio that he
this stupid motion is also stupid. Why do you have to dignify had advised Justice Reyes to lay off the case and allow
such a foolish motion? They should file a motion for me to Justice Sabio to continue and to resolve the urgent motion for
inhibit or recuse myself. Justice Reyes to assume the chairmanship. Justice Villarama
recalled that Justice Reyes repeatedly said: Wala talaga ako
→ Why is it that Meralco actively participated in the hearing dito Jun, Wala akong personal interest dito.
on the 23rd and never raised any question on the alleged
irregularity of my having presided over the hearing?

→ Why do you insist on assuming the case? Are you not After a careful and judicious study of the more than 56-page
aware that several days after the issuance of the TRO, decision of Justice Roxas, Justice Dimaranan-Vidal signed it.
respondents filed a motion for inhibition of Justice Vicente True to his word, Justice Roxas personally picked up the
Roxas and a motion to lift the TRO. Who then had the right to decision that day purportedly for the action of the Acting
resolve such motion? Chairman, Justice Sabio, who was then on leave of absence
until July 11, 200.[92] Notwithstanding the fact that the parties
→ Under the circumstances, anong iisipin ko sa yo? Ano ang had not submitted their respective memoranda, Justice
tingin ko ngayon sa iyo? Dimaranan-Vidal signed the convincing ponencia, including
three copies of the signature page, because Justice Roxas
His feeble answer was: you. He then said he did not was insistent of the urgency of the signing of the decision due
know of those pending motions. (Incidentally, these motions to the impending lapse of the TRO on July 29, 2008.[93]
were never resolved.) He also said, wala talaga akong interest Justice Sabio thought otherwise.[94]
dito kundi ayaw ko lang ma charge ng non-feasance for failing
to do my duty.

I answered him: Malayo yung non-feasance. Hindi ito However, Justice Roxas denied that the decision he gave to
nonfeasance. I taught the subject for many years and this is Justice Dimaranan-Vidal was the final decision. He denied
not one of them. that he gave it to her for her signature. He said it was only for
her to read because she asked to read it. He said it was a
So I told him, I have made my decision on the matter. mere draft as everything was unofficial there was no rollo or
Bahala ka na. Then I stood up to show him to the door. He logbook with it, it was not placed in an envelope, and it did not
was silent after that and before he left, he put his arm around have the special seal of Justice Roxas. It allegedly was thrown
me. in the garbage can.

For his part, Justice Reyes kept on repeating: Wala talaga ako On July 9, 2008, the OSG filed the memorandum for the SEC.
dito, wala akong interest kung di yun lang hindi ako ma non-
feasance. Justice Sabio thought otherwise.

On July 10, 2008, Meralco filed an urgent motion praying that


Justice Reyes assume the chairmanship of the Division,[95]
Meanwhile, Justice Roxas brought to the office of Justice alleging the reasons for the urgent motion as follows:
Dimaranan-Vidal the final decision on the MERALCO case
bearing his signature, which he gave to Justice Dimaranan- 5. At the scheduled oral arguments on 23 June 2008 in the
Vidal for concurrence/dissent. According to Justice instant case, the parties were first directed to one of the
Dimaranan-Vidal, Justice Roxas explained to her the rationale Hearing Rooms of the Court of Appeals. At the said room, the
for his conclusion. Justice Roxas went out for a while and name plate of Justice Reyes was already placed on the table
returned with an expensive looking travelling bag from where for the justices. Thus, petitioners were of the impression that
he pulled out the purported final decision. Before the close of the leave of absence of Justice Reyes was over and that he
office hours, Justice Roxas returned to the chambers of would be presiding over the oral arguments as Chairman of
Justice Dimaranan-Vidal to check if he (Justice Roxas) had the Ninth Division of the Honorable Court.
signed his decision. When she replied that yes, he had signed
it, Justice Roxas said he would pick it up the next day.[89]

6. However, when the parties were directed to transfer to


another Room of the Court of Appeals for the oral arguments
Justice Dimaranan-Vidal signed the decision notwithstanding in the instant case, petitioners saw that the name plates on
that on July 8, 2008 the Court of Appeals had been the table for the justices included that of Justice Sabio, Jr.,
reorganized because she believed that the Special Ninth together with that (sic) of Justices Roxas and Dimaranan-
Division was still existing on account of its having issued the Vidal. Thereafter, Justice Sabio presided over the oral
FINALS CONSTITUTIONAL LAW I ACJUCO NOV 11, 2017 149

arguments as Chairman of the Special Ninth Division of the Sabio told him that he needed ample time to read the
Honorable Court. Petitioners were, thus, of the impression that memoranda of the parties. Justice Roxas promised to send to
the regular Chairman of the Ninth Division, Justice Reyes, was Justice Sabio the memoranda immediately.[97]
still on temporary leave of absence.

At 4:00 p.m., Justice Reyes received from the Eighth Division


7. Subsequently, it has come to the attention of the petitioners Clerk of Court a copy of Meralcos Urgent Motion for him to
that Justice Reyes has already returned from his temporary assume the chairmanship of the Ninth Division.
leave of absence and has resumed his duties as Chairman of
the Ninth Division of the Honorable Court.

On Monday, July 14, 2008 at the flag ceremony, Justice Sabio


requested Justice Roxas to meet with him as he had by then
8. Under the Internal Rules of the Court of Appeals, Justice read the memoranda of the parties. Justice Roxas initially
Sabio, Jr. should now refrain from acting as the chairman of agreed to the meeting but he later informed Justice Sabio that
the Division hearing the instant case as he is already he had another matter to attend to; neither was he available
disqualified from acting as such upon the return of Justice in the afternoon. Justice Roxas had become scarce. Justice
Reyes. Sabio learned that Justice Dimaranan-Vidal was also looking
for Justice Roxas.[98]

8.1. With due respect, Justice Reyes cannot shirk from his
bounden judicial responsibility of performing his duties and Justice Sabio prepared a resolution on the motion for the
functions as Chairman of the Ninth Division of the Honorable reconsideration of the TRO and informed Justices Roxas and
Court. Dimaranan-Vidal that he wanted to discuss it with them. The
resolution he prepared never saw light.[99]
8.2. Specifically, under Section 3 (d), Rule IV of the 2002
Internal Rules of the Court of Appeals, a case can remain with
the justices who participated therein only when any of the
following actions have been taken: (a) giving due course; (b) At 10 a.m., Justice Roxas, with his messenger, brought the
granting of a writ of preliminary injunction; (c) granting of a rollo of CA G.R. SP No. 103692 to Justice Reyes, and told the
new trial; or (d) granting of execution pending appeal: latter that he and Justice Bruselas would be coming over to
deliberate on the case. Ten minutes later, the Eighth Division
x x x x x x x x x. deliberated on the case.[100] After a cursory examination of
the rollo, Justice Reyes found that the decision had been
signed by Justices Roxas and Bruselas but Justice Reyes
asked for more time to study the case.[101]
9. None of the foregoing instances apply with respect to
Justice Sabio, Jr.s continuing hold on the case. Although
Justice Sabio, Jr. was one of the Justices who issued the
temporary restraining order in favour of the petitioners in the A transcript of the Final Deliberation on July 14, 2008 is
instant case, this circumstance is not among the grounds as attached to page 1926 of Volume III of the rollo of CA-G.R. SP
above-quoted, when a justice of the Court of Appeals may No. 103692 and marked as Exh. 2- Roxas on page 279 of the
remain in the Division. rollo of A.M. No. 08-8-11-CA. According to Justice Roxas, it
was he who prepared the transcript from memory to lend
credence to the certification of Justice Reyes at the end of the
decision pursuant to Article VIII, Section 13 of the
Constitution.[102] Justice Reyes denied having seen it or
10. As above-quoted, the rule is categorical that it is not the having authorized its transcription. Justice Bruselas did not
grant of a temporary restraining order but rather the grant of a sign any transcript of the deliberation as he was not aware that
writ of preliminary injunction that sanctions a justices a transcript was being taken. There was no stenographer
remaining with the Division. Thus, the continued participation present, as only the three of them, Justices Reyes, Roxas,
of Justice Sabio, Jr., in the instant case, considering the clear and Bruselas were present at the deliberation. Neither was
Rules of the Honorable Court, is not only irregular but may there a recording machine. Justice Roxas admittedly prepared
lead one to conclude that he is exhibiting undue interest in the the transcript from memory.[103]
instant case.

The statement attributed to Justice Reyes in the transcript that


On this day, Justice Reyes reported back to work after his trip there were previous deliberations were really meetings, which
to Australia.[96] they had twice, in the office of Justice Reyes, according to
Justice Roxas.[104]

On July 11, 2008, Justice Sabio was on leave when Justice


Roxas called him up for a meeting to discuss the case. Justice
FINALS CONSTITUTIONAL LAW I ACJUCO NOV 11, 2017 150

On July 15, 2008, when she felt that the timing was right, Atty. Presiding Justice Vasquez allegedly told Justice Roxas that
Silvia Sabio testified that she handed her fathers letter to the as Presiding Justice, he had no authority to rule on the
Chief Justice through his private secretary, Ms. Jasmin Interpleader Petition, which is not an administrative concern
Mateo.[105] A few days later, however, Presiding Justice over which the Presiding Justice must intervene.
Vasquez told Justice Sabio that the Chief Justice would no Nevertheless, to avoid further discussion, the Presiding
longer meet with him, as the Presiding Justice had apprised Justice told Justice Roxas that he would study the matter.[115]
the Chief Justice about the matter.[106]

On July 22, 2008, Justice Reyes wrote the Presiding Justice


According to Justice Reyes, at 2:00 p.m. that day, the Office a letter on what was discussed between us last 17 July 2008
of the Presiding Justice informed him that Justice Sabio was at around 3:30 p.m.[116] Apparently the Presiding Justice had
waiting for him in his office. As soon as Justice Reyes was suggested to endorse the case and have the Special Ninth
seated, Justice Sabio berated him and accused him of Division direct the respondents to file their simultaneous
orchestrating matters. Justice Sabio told him that an emissary comments on the petitioners Urgent Motion (For Honorable
of MERALCO had offered him P10 million to drop off the case, BIENVENIDO L. REYES to Assume Chairmanship of the
hence, he asked that if he was offered that much, how much Division in the Instant Case) dated 10 July 2008.
could have been offered to the principals?[107]

Justice Reyes expressed doubts that the suggestion was most


On July 17, 2008, Justice Reyes went back to the office of the prudent, as the dispute revolves around the correct
Presiding Justice and informed him of the episode in the office interpretation of the IRCA. He believed that since the question
of Justice Sabio. He also went to ask Justice Villarama for his was purely internal, the CA should not seek enlightenment
opinion as to who was the rightful claimant to the from the litigants for it would only be construed against its
chairmanship of the Division that should decide the Meralco competence. He shared Justice Cruzs and Roxas
case. Justice Villarama allegedly replied that they were both interpretation of the IRCA. Hence, he urged the Presiding
correct. Justice to decide the matter; otherwise, he would interpret the
rules according to his best lights and act accordingly.

On July 18, 2008, at the pre-launching meeting for the CA-


CMIS, Justice Villarama had a brief chat with Justice Bruselas. On July 23, 2008, Presiding Justice Vasquez asked for the
The former told the latter that both Justices Sabio and Reyes rollo of CA G. R. No. SP No. 103692 so he could properly
are correct in the sense that one (1) [of] them can properly submit the requested opinion. It was then that he came across
assume chairmanship either under the exception provided in the unresolved motion praying for the inhibition of Justice
Sec. 2 (d), Rule VI of the 2002 IRCA depending on the final Roxas and the pending urgent motion to lift the TRO or to hold
disposition of the prayer for injunctive relief, or pursuant to the its enforcement in abeyance. The Presiding Justice
general rule enshrined in Sec. 7 (b), Rule VI.[108] considered the latter as a motion for reconsideration of the
Resolution issuing the TRO.[117]

On July 21, 2008, Justice Roxas personally filed with the


Presiding Justice[109] an Interpleader Petition[110] praying Meanwhile, at noon of that day, as Justice Reyes had not yet
that Presiding Justice Vasquez decide which division received any reaction from the Presiding Justice, he signed
Chairman (Justice Sabios Former Special 9th Division or the decision as well as the Certification. It was promulgated
Justice B. L. Reyes 8th Division) should sign the Preliminary on the same day.
Injunction or Decision.[111] Justice Roxas averred that [t]he
impasse between two Chairmen from two Divisions has to be
resolved much earlier than July 30, 2008 because July 30,
2008 is the expiration date of the TRO issued by the Special
9th Division (signed by Justice Jose L. Sabio, Jr., Justice The decision was promulgated without waiting for the
Vicente Q. Roxas [ponente] and Justice Myrna Dimaranan- Presiding Justices opinion on whether it was the Eighth or
Vidal). He opined that the two Chairpersons differed in the Special Ninth Division that should decide the case. Justice
interpretation of Sections 1 and 2 (d) in relation to Section 5 of Roxas alleged that he did not expect the Presiding Justice to
Rule VI on Process of Adjudication of the Internal Rules of the answer or resolve the matter anyway.
Court of Appeals (IRCA).[112] His stand was that the IRCA
should be strictly applied because [w]hen the provisions are
clear, there is no room for interpretation.
On July 24, 2008, Presiding Justice Vasquez issued his reply
to Justice Reyes letter and Justice Roxas Interpleader-
Petition. The Presiding Justice claimed having doubts on
Justice Roxas endorsed his Interpleader Petition to Justice whether he possessed the authority to decide the subject
Reyes for his signature or dissent to the finalized MERALCO conflict simply because under the IRCA, the Presiding Justice
Decision, which had been in Justice Reyes possession since has control and supervision only over administrative affairs of
July 14, 2008.[113] He also gave the rollo of the case to the Court. The controversy was certainly not an administrative
Justice Reyes.[114] matter but Section 11 of Rule VIII of the IRCA provides that
the Presiding Justice has the authority to act on any matter
FINALS CONSTITUTIONAL LAW I ACJUCO NOV 11, 2017 151

not covered by the Rules although such action should be My deepest regret is that the undersigned who already signed
reported to the Court en banc. the supposed final draft of the Decision in the instant case
which bears the signature of the ponente, was not even
informed by the latter as a judicial courtesy at least, of the
hurried easing out of the undersigned from the case. This
inevitably posed even to an unprejudiced mind the following
The Presiding Justice expressed in his letter the view that the questions: under what basis was the case suddenly
(Special Ninth) Division that issued the temporary restraining transferred to the 8th Division and why is it that neither the
order should continue resolving the injunctive prayer in the undersigned nor the Acting Chairman Justice SABIO, of the
petition because it was the Division that issued the Resolution Special 9th Division not consulted thereof? and, foremost,
granting the TRO and setting the hearing on the application what happened to the Decision which the undersigned signed
for the issuance of a writ of preliminary injunction, aside from after devoting her precious time and effort in carefully and
the fact that the parties did not contest the authority of Justice laboriously examining the voluminous records and rollo of the
Sabio as Division Chairman at the time, although Justice case?
Reyes had reported back to work. Moreover, the motion for
inhibition and the urgent motion to lift the TRO have a bearing
on the application of Section 2 of Rule VI of the IRCA,
especially because Section 7 (b) of Rule VI[118] points to the
retention of the case by the Special Ninth Division. Sad to say the circumstance obtaining herein constitute a
Furthermore, the new Division headed by Justice Reyes may flagrant violation of the provision of Canon 5 particularly
not be allowed to resolve the pending incidents because two Sections 2 and 3 thereof of the New Code of Judicial Conduct
of its members, Justices Reyes and Bruselas did not for the Philippine Judiciary (A.M. No. 03-05-01-SC).
participate in the hearing on June 23, 2008. He did not believe
that Justice Reyes would be charged with dereliction of duty
should he not assume the chairmanship. The Presiding
Justice ended his letter with the hope that the matter would be
laid to rest and that whoever would be dissatisfied with its On July 25, 2008, Justice Bruselas wrote the Presiding Justice
outcome may elevate the matter to the Supreme Court. a letter,[122] which was prompted by a disturbing telephone
call he received from Justice Sabio in the morning of July 24,
2008. Justice Sabio informed Justice Bruselas that, after the
injunction hearing on June 23, 2008, Meralco offered him P10
Million to either favor them or yield the chair to Justice Reyes.
At 2:00 p.m. that day, Justice Sabio informed the Presiding Justice Sabio told Justice Bruselas that he had informed the
Justice that a decision had been promulgated in the Meralco Presiding Justice of the bribery incident and that he was
case the previous day. The Presiding Justice was surprised disgusted over the turn of events because he should have
because Justices Roxas and Reyes had asked him to resolve remained chair of the Special 9th Division that issued the TRO
the impasse on the Division chairmanship. Upon inquiry, the on the case. Justice Bruselas informed Justice Sabio that it
Presiding Justice found that the decision had indeed been was the first time that he heard of the matter and that he had
promulgated at 4:10 p.m. on July 23, 2008.[119] participated in the deliberation on the case and concurred with
the ponencia of Justice Roxas without such information ever
being taken up. Justice Sabio told Justice Bruselas that he
would not leave the matter as it is because he would bring it
up in the open, to media, etc. Justice Sabio asked Justice
It was also on July 24, 2008 that Justice Dimaranan-Vidal Bruselas that if P10M was offered to him, how much would
received a call from Justice Sabio, informing her that Meralco have been offered to the others.
had offered him a bribe of P10 million in exchange for his
voluntary stepping out from the Meralco case in order to give
way to Justice B. L. Reyes, and that the decision in the
Meralco case had been promulgated by the Eighth
Division.[120] Shocked that Justice Roxas did not inform her Troubled by the information, Justice Bruselas went to the
as a matter of judicial courtesy of the scrapping of the decision Presiding Justice where Justice Dimaranan-Vidal, who had
which she signed on July 8, 2008, Justice Dimaranan-Vidal received the same call from Justice Sabio, joined them. After
wrote a letter to the Presiding Justice dated July 24, that meeting with the Presiding Justice, Justice Bruselas
2008,[121] bringing to his attention the apparent and obvious called up Justice Reyes who confirmed that he had heard
irregularities in the handing of CA-G.R. SP No. 103692, and about the bribe offer but that he did not reveal the same to
complaining about Justice Roxas lack of judicial courtesy in Justice Bruselas as it escaped his mind. The effort of Justice
discarding for reasons she would not know, his purported final Bruselas to get in touch with Justice Roxas proved futile.
Decision that he had asked her to sign and which she signed
after a judicious study of the records and rollo thereof. Justice
Roxas gave the lame excuse that he had to incorporate
therein some ten pages which he forgot to include in his
Decision. Allegedly prompted by the manner by which the decision x x x
was arrived at, and how the decision was promulgated, and
that unless an immediate and thorough investigation thereon
be undertaken by the Court of Appeals, both the individual and
institutional integrity of the justices and of the Court of Appeals
Justice Dimaranan-Vidal expressed surprise and would undoubtedly be tarnished, Justice Sabio wrote on July
consternation when she learned on even date that a Decision 26, 2008 a letter[123] to the Presiding Justice, which
in the case had been promulgated on July 23, 2008 by the precipitated the present investigation.
Eighth Division chaired by Justice Reyes, with Justices Roxas
and Bruselas as members. She said:
FINALS CONSTITUTIONAL LAW I ACJUCO NOV 11, 2017 152

On July 28, 2008, the Philippine Daily Inquirer carried an examined the ponencia with more than 50 pages, after
account of the letter of Justice Dimaranan-Vidal to the devoting her precious time to such study, and affixing her
Presiding Justice, without her knowing how her confidential concurrence thereto. Justice Dimaranan-Vidal reiterated her
letter to the Presiding Justice leaked out.[124] prayer for an investigation of the matter.

Before Justice Bruselas delivered his letter to the Presiding Meanwhile, on that day, Mr. De Borja, executed an affidavit
Justice, he received a copy of the letter of Justice Sabio and, admitting that he was the businessman referred to by Justice
through a telephone call, reiterated his full agreement with his Sabio, Jr. in his letter to Presiding Justice Vasquez. Mr. De
desired investigation. Borja publicly claimed having learned from the news that
Justice Sabio was one of the justices in the case arising from
the order of the SEC to nullify the proxies issued in favor of
the MERALCO management. He also alleged that Justice
Sabio told him about the blandishments coming from the
The Presiding Justice called the Court of Appeals to an government side, that he was being offered a promotion to the
emergency en banc session at 10:00 a.m. on July 31, 2008 at Supreme Court and money to favor the GSIS position. Mr. De
the Session Hall to elicit the reaction of the Court and on the Borja asked Justice Sabio, Jr., What would it take for you to
possible effect on the decision rendered. The session was resist the governments offer? and that the response of Justice
also called in order that the predicament experienced in CA- Sabio, Jr. was Fifty Million.
G.R. SP No. 103692 could be deliberated upon by the
Committee on Rules with a view to amending the IRCA on the
reorganization of the Court of Appeals. The Executive Justices
of Cebu and Cagayan de Oro, Justices Antonio L. Villamor
and Romulo V. Borja, respectively, were instructed to attend Justice Sabio asked permission from the Presiding Justice to
the en banc session to report to the other Justices in their hold a press conference the next day on account of the
stations what transpired at the session, and to collect the publicized affidavit of Mr. De Borja. The Presiding Justice told
personal reaction, comment or view of the Justices on the Justice Sabio that this is a matter of self-defense on his part,
matter.[125] hence, the Presiding Justice cannot stop him from doing so.

In its closed door en banc session on July 31, 2008, after a Justice Sabio issued a signed statement as an initial response
torrid discussion of all the issues, the Court of Appeals to the affidavit of Mr. De Borja, vehemently denying that Mr.
decided, as follows: De Borja asked him what it would take for him to inhibit from
the case, and that he never asked for money from him.[128]
(1) Refer the propriety of the actions of the Justices concerned
to the Supreme Court, through the Office of the Court
Administrator;
On August 1, 2008, Justice Sabio called the press conference
to read a signed statement entitled My Reaction to Mr. Francis
De Borjas Affidavit dated July 31, 2008 on the Meralco-SEC
(2) Leave the matter regarding the validity of the decision Case.
rendered in the above-entitled case to the parties for them to
take whatever legal steps they may deem appropriate in the
usual course of procedure; and
Expressing anger at the filthy lie of Mr. De Borja, Justice Sabio
decided to narrate almost word for word his conversations with
Mr. De Borja.
(3) Refer the conflict in the interpretation of our Internal Rules
to the Committee on Rules of the Court of Appeals in order to
prevent the recurrence of a similar situation.[126]
In an affidavit dated August 1, 2008, which Evelyn
Clavano[129] executed in Davao City, she stated that -

After the en banc session, Justice Dimaranan-Vidal


expressed in a letter for the Presiding Justice[127] her strong
reaction to the paper of Justice Roxas falsely imputing to her Francis de Borja requested me if I have the cell phone number
grandstanding before the media or resorting to media- of Justice Jose L. Sabio Jr. He related that because he was
recourse instead of just filing an administrative complaint very close to the Lopezes of Meralco, he wanted to call him
before the Supreme Court, and taking exception to the equally regarding his possible inhibition in a certain Meralco case,
outrageous, revolting and baseless accusation that she is wherein he was designated as a substitute member of the
allegedly clinging to the case. She asserted that she never division vice a justice who was temporarily on leave by reason
leaked a copy of her letter to the Philippine Daily Inquirer, as of sickness. He further said that the Lopezes desire that the
her letter was only intended to bring to the attention of the same Justice, with whom the Lopezes are more comfortable,
Presiding Justice the impropriety done by Justice Roxas in the to sit in the division.
MERALCO case that resulted in her having been eased out of
the case notwithstanding that she carefully and judiciously
FINALS CONSTITUTIONAL LAW I ACJUCO NOV 11, 2017 153

So, I gave Francis de Borja the cell phone number of Justice b) GSIS Urgent Ex-Parte Motion to Inhibit Justice Roxas,
Jose. L. Sabio, Jr. through business card. which was filed on May 30, 2008. As the motion raised a
prejudicial question, Justice Roxas should have resolved it
before issuing the TRO sought by Meralco, but he never did
(Rollo, pp. 220-223).
x x x x x x x x x.

(c) GSIS Motion to Lift TRO which was filed on May 30, 2008
(Rollo, pp. 187-210)
On August 4, 2008, the Supreme Court constituted the Panel
of Investigators to investigate (1) alleged improprieties of the
actions of the Justices of the Court of Appeals in CA-G.R. SP
No. 103692 (Antonio V. Rosete, et al. v. SEC, et al.) and (2)
the alleged rejected offer or solicitation of bribe disclosed (d) GSIS Motion filed on June 18, 2008, praying that it be
respectively by Mr. Justice Jose Sabio and Mr. Francis de allowed to use Power point at the hearing on June 23, 2008 .
Borja. On June 20, 2008, the SEC filed a similar motion. Both
motions were not acted upon by Justice Roxas (Rollo, pp.
593-621,)

The Panel of Investigators held hearings from August 8 to 23,


2008. Affidavits were submitted to the Panel to serve as the
parties direct testimonies upon which they were cross- (e) Meralcos Motion for Extension of Time to file their
examined by the Panel and the other parties. Consolidated Memorandum of Authorities and Reply to
Repondent SECs Comment filed on June 25, 2008 (Rollo, pp.
981- 987).

On September 4, 2008, the Panel of Investigators submitted (f) Meralcos Urgent Motion for Honorable Justice Bienvenido
its Report of even date to the Court en banc. L. Reyes to Assume Chairmanship of the Division in the
Instant Case, which was filed on July 10, 2008 (Rollo, pp.
1262-1274).[131] (emphasis supplied)

According to the Report, the investigation has revealed


irregularities and improprieties committed by the Court of
Appeals Justices in connection with the MERALCO case, CA- We agree with the Panel of Investigators that by ignoring or
G.R. SP No. 103692, which are detrimental to the proper refusing to act on the motion for his inhibition, Justice Roxas
administration of justice and damaging to the institutional violated Rule V, Section 3, third paragraph of the IRCA, which
integrity, independence and public respect for the provides that he should resolve such motion in writing with
Judiciary.[130] copies furnished the other members of the Division, the
Presiding Justice, the Raffle Committee, and the Division
Clerk of Court. The pertinent portion of the said provision
states:

Findings regarding the conduct of Associate Justice Vicente


Q. Roxas
Sec. 3. Motion to Inhibit a Division or a Justice. x x x

xxx
Justice Roxas inexcusably failed to act on a number of
motions of the parties prior to the promulgation of the
Decision. A motion for voluntary inhibition of a Justice shall be acted
upon by him alone in writing, copy furnished the other
members of the Division, the Presiding Justice, the Raffle
Committee and the Division Clerk of Court.

As found by the Panel of Investigators, several motions were


not resolved or acted upon by Justice Roxas. These were
enumerated in the Report as follows:
This Court cannot agree with Justice Roxas proposition that
the issuance of the TRO constitutes an implied denial of the
motion to inhibit since under IRCA the obligation of the Justice
to act on such a motion is mandatory.
(a) The Urgent Ex-Parte Motion to Defer Action on
any Incident of the Petition Pending Resolution of Re-Raffle
filed by GSIS on May 29, 2008 soon after this case was filed
on that date (Rollo, pp. 185-186).
FINALS CONSTITUTIONAL LAW I ACJUCO NOV 11, 2017 154

Furthermore, the Court finds well-taken the Panels finding that deliberation, shall be kept. The so-called transcript is a
Justice Roxas failure to act on the other motions of the parties fabrication designed to deceive that there had been
violated Canon 3, Rule 3.05 of the 1989 Code of Judicial compliance when actually there was none -- with the
Conduct (which applies in a suppletory manner to the New prerequisite of the IRCA that consultation and/or deliberation
Code of Judicial Conduct for the Philippine Judiciary) among the members of the Division must precede the drafting
providing that: of a decision.

Rule 3.05. A judge shall dispose of the courts business (b) The statement in the transcript that it was a recap from our
promptly and decide cases within the required periods. previous deliberations was another falsehood because there
had been no previous deliberations.

(c) The reference in the transcript to a Final Report of Justice


Even Section 5, Canon 6 of the New Code of Judicial Conduct Roxas was also false for Justice Roxas admittedly did not
mandates that [j]udges shall perform all judicial duties, submit a report as ponente, as required by Sec. 9, Rule VI of
including the delivery of reserved decisions, efficiently, fairly the IRCA, for deliberation by the Eighth Division on July 14,
and with reasonable promptness. Thus, it has become well- 2008. The Final Report which he submitted was admittedly the
settled in jurisprudence that even just undue delay in the decision itself which he and Justice Bruselas, Jr. had already
resolving pending motions or incidents within the signed. The Final Report was merely the title of the page that
reglamentary period fixed by law is not excusable and served as the cover of the decision. Hence, Justice B.L. Reyes
constitutes gross inefficiency.[132] With more reason, this supposed closing statement in the transcript that -- We have
Court finds suspicious and reprehensible the failure of Justice covered every angle of the Final Report of Justice Roxas
Roxas to act at all on pending motions and incidents in CA- extensively is also false. Justice B.L. Reyes testified at the
G.R. SP No. 103692. investigation that he had not seen the transcript until the copy
in the rollo was shown to him by Justice Callejo, Sr. during his
cross-examination of Justice B. L. Reyes on August 26, 2008.

This is in fact not the first time that Justice Roxas has been
cited administratively for failure to resolve pending incidents
in cases assigned to him. In Orocio v. Roxas, A.M. Nos. 07- xxx xxx xxx
115-CA-J and CA-08-46-J, this Court imposed a P15,000 fine
on Justice Roxas for unwarranted delay in resolving two
motions for reconsideration in another case and sternly
warned him that future commission any act of impropriety will (e) Justice Roxas testimony that when he brought the Meralco
be dealt with more severely. decision to Justice Dimaranan-Vidal on July 8, 2008, it was
only a draft for her to read, because she asked if she may read
it, not for her to sign it, is completely false. This testimony was
labelled by Justice Dimaranan-Vidal as a lie, and she called
Justice Roxas is guilty of gross dishonesty. Justice Roxas a liar, because she did not ask to borrow the
decision for her reading pleasure, but Justice Roxas
personally brought it to her office for her to sign as a member
of the Special Ninth Division. After poring over it the whole
night, she signed it, as well as three (3) additional signature
pages which were to be attached to three (3) other copies of
the decision.[133]
Apart from Justice Roxas inexcusable inaction on pending
incidents in the Meralco case, the Panel of Investigators found
that he had been dishonest and untruthful in relation to the
said case. The Court adopts the following findings of the xxx xxx xxx
Panel:

Indeed, the fabrications and falsehoods that Justice Roxas


2. Justice Roxas was dishonest and untruthful. blithely proferred to the Panel in explanation/justification of his
questioned handling of the Meralco case demonstrated that
he lacks the qualification of integrity and honesty expected of
a magistrate and a member of the appellate court.
(a) Justice Roxas admitted that the Transcript of Final
Decision, which is supposed to be a transcript of the
deliberation on July 14, 2008 of the Eighth Division on the final
decision in the Meralco case was not a true transcript of the Under Rule 140 of the Rules of Court, dishonesty is
minutes of the meeting, but purely a transcript from memory considered a serious offense that may warrant the penalty of
because no notes were taken, no stenographer was present, dismissal from the service. Under the Rule IV, Section 52 of
and no tape recorder was used. It was in fact a drama which the Uniform Rules on Administrative Cases in the Civil
he composed from my recollection to comply with Sec. 9, Rule Service, dishonesty is likewise considered a grave offense
VI of the IRCA which requires that minutes of the meeting, i.e.,
FINALS CONSTITUTIONAL LAW I ACJUCO NOV 11, 2017 155

and warrants the penalty of dismissal even for the first offense. (g) J. Roxas was disrespectful to Presiding Justice Vasquez,
In the past, the Court has had the occasion to rule that: Jr. whose ruling on his Interpleader Petition he sought on July
21, 2008, but he promulgated the Meralco decision two (2)
dishonesty and falsification are considered grave offenses days later, on July 23, 2008, without waiting for Presiding
warranting the penalty of dismissal from service upon the Justice Vasquez, Jr.s ruling which came out on July 24, 2008,
commission of the first offense. On numerous occasions, the only three (3) days after the Interpleader Petition was filed by
Court did not hesitate to impose such extreme punishment on him, and two (2) days after Justice B.L. Reyes also reiterated
employees found guilty of these offenses. in writing his request for Presiding Justice Vasquez, Jr. to
resolve the same chairmanship issue raised in the
Interpleader. Presiding Justice Vasquez, Jr. was embarrassed
Dishonesty, being in the nature of a grave offense, carries the and humiliated by Justices B.L. Reyes and Roxas lack of
extreme penalty of dismissal from the service with forfeiture of courtesy and respect for his position as head of the Court.
retirement benefits except accrued leave credits, and
perpetual disqualification for re-employment in the
government service. Dishonesty has no place in the
judiciary.[134]
xxx xxx xxx

Justice Roxas showed a lack of courtesy and respect for his


colleagues in the Court of Appeals. There is an old adage which says to gain respect one must
learn to give it. If judges and justices are expected to treat
litigants, counsels and subordinates with respect and fairness,
with more reason, that judges and justices should give their
fellow magistrates the courtesy and professional regard due
to them as their colleagues in the Judiciary. Thus, in Canon 5,
Section 3 of the New Code of Judicial Conduct, judges are
The Panel of Investigators reported on this matter in this wise: expected to carry out judicial duties with appropriate
consideration for all persons, such as the parties, witnesses,
xxx xxx xxx lawyers, court staff and judicial colleagues, without
differentiation on any irrelevant ground, immaterial to the
proper performance of such duties.
(f) Justice Roxas was thoughtlessly disrespectful to a
colleague and a lady at that, when he unceremoniously
discarded, shredded, and burned the decision that Justice
Dimaranan-Vidal had signed, because he allegedly forgot that
Justice Dimaranan-Vidal and Justice Sabio, Jr. had already
been reorganized out of the Special Ninth Division as of July
4, 2008, hence, out of the Meralco case. Out of courtesy, he This Court cannot view lightly the discourteous manner that
should have explained to Justice Dimaranan-Vidal the reason Justice Roxas, in his apparent haste to promulgate his
why he was not promulgating the decision which she had decision in the Meralco case, treated his colleagues in the
signed. Court of Appeals. It behooves the Court to remind all
magistrates that their high office demands compliance with the
most exacting standards of propriety and decorum.

The truth, it seems, is that Justice Roxas, who had consulted


Justice Villarama, Jr. on which Division should decide the
Meralco case, may have been convinced that it should be the Justice Roxas questionable handling of the Meralco case
Special Ninth Division. That is why he brought his decision to demonstrates his undue interest therein.
Justice Dimaranan-Vidal for her signature. However,
somehow, somewhere, during the night, while Justice
Dimaranan-Vidal was patiently poring over his decision,
Justice Roxas was persuaded to bring his decision to the
Eighth Division (to which he and Justice B.L. Reyes belong In the Report, the Panel of Investigators observed that Justice
after the July 4, 2008 reorganization of the Court), it may have Roxas in fact began drafting his decision even prior to the
dawned on him that if the case remained in the Special Ninth submission of the parties memoranda. As discussed in the
Division, Justice Sabio, Jr. might dissent, requiring the Report:
Presiding Justice to constitute a special division of five. If he
(Justice Roxas) should fail to obtain a majority of the Division
on his side, he would lose his ponencia; someone else would
become the ponente (perhaps Justice Sabio, Jr.). That may
be the reason why he junked Justices Sabio, Jr. and xxx xxx xxx
Dimaranan-Vidal (even if the latter concurred with his
decision) because he was unsure of Justice Sabio, Jr. He
chose to cast his lot with his companions in the Eighth Division
-- Justices B. L. Reyes and Bruselas, Jr. -- with whom he and (d) Although the parties were given 15 days after the hearing
Meralco were comfortable. on June 23, 2008, or up to July 8, 2008, to simultaneously
submit their memoranda and memoranda of authorities, and
actually submitted:
FINALS CONSTITUTIONAL LAW I ACJUCO NOV 11, 2017 156

On July 7, 2008 GSISs 39 page- memorandum In sum, this Court finds that Justice Roxas multiple violations
of the canons of the Code of Judicial Conduct constitute grave
On July 9, 2008 SECs 62 page-memorandum misconduct, compounded by dishonesty, undue interest and
conduct prejudicial to the best interest of the service, which
warrant his DISMISSAL from the service.
On July 10, 2008 MERALCOs 555 page- memorandum (by
messenger) with memorandum of authorities

Findings regarding the conduct of Associate Justice Jose L.


Sabio, Jr.
Justice Roxas prepared the decision before the parties had
filed their memoranda in the case and submitted it to Justice
Dimaranan-Vidal for her signature on July 8, 2008. His rush to
judgment was indicative of undue interest and unseemly
haste, according to J.Romero. In the Report, the Panel found that Justice Sabio likewise
committed improprieties in relation to the Meralco case.

He cheated the parties counsel of the time, effort, and energy


that they invested in the preparation of their ponderous The circumstances of the telephone call of Chairman Sabio to
memoranda which, as it turned out, neither he nor the other his brother Justice Sabio showed that Justice Sabio failed to
members of the Eighth Division bothered to read before uphold the standard of independence and propriety expected
signing his decision. He made a mockery of his own order for of him as a magistrate of the appellate court.
the parties to submit memoranda, and rendered their
compliance a futile exercise.

xxx xxx xxx In his testimony before the Panel, Chairman Sabio admits that
he called up Justice Sabio on May 30, 2008 from Davao City,
(underscoring supplied) in response to a resquest for help from a member of the Board
of Trustees of Meralco. Notwithstanding the fact that
Chairman Sabio called to relay to Justice Sabio the rightness
of the GSIS cause and asked him to help GSIS and that
Justice Sabio allegedly told his brother that he would act in
We agree with Mme. Justice Romeros observation that the accordance with his conscience, the same still constituted a
rush to judgment (even before the filing of the parties violation of Canon 13 of the Code of Professional
memoranda) was indicative of Justice Roxas undue interest Responsibility for lawyers, which provides that:
and unseemly haste, especially when taken together with
other circumstances. This inexplicable haste in resolving the
case on the merits is likewise apparent in Justice Roxas failure A lawyer shall x x x refrain from any impropriety which tends
to resolve the several pending incidents and instead jumping to influence, or gives the appearance of influencing the Court.
ahead to deciding the case on the merits; his rushing of
Justice Dimaranan-Vidal into signing his draft Decision on July
8, 2008 when the parties memoranda have not yet all been
filed with the CA; his precipitate transfer of the case to the As they were both members of the Bar, it is incomprehensible
Eighth Division for promulgation of decision, without notice to to this Court how the brothers can justify their improper
Justice Dimaranan-Vidal of the Special Ninth Division who had conversation regarding the Meralco case. As the Panel
already signed his draft Decision and despite the unresolved observed in its Report:
Chairmanship dispute between Justice Reyes and Justice
Sabio which he (Justice Roxas) even submitted to the
Presiding Justice for appropriate action, just a few days before
the promulgation.
Ironically, both of them found nothing wrong with brother
Camilos effort to influence his younger brothers action in the
Meralco case, because both believe that our Filipino culture
allows brother-to-brother conversation, even if the purpose of
We reiterate here that as the visible representation of the law one is to influence the other, provided the latter does not agree
and justice, judges are expected to conduct themselves in a to do something illegal.[137]
manner that would enhance respect and confidence of the
people in the judicial system. The New Code of Judicial
Conduct for the Philippine Judiciary mandates that judges
must not only maintain their independence, integrity and
impartiality; but they must also avoid any appearance of For the Panel, Justice Sabio violated Sections 1, 4, and 5,
impropriety or partiality, which may erode the peoples faith in Canon 1 of the New Code of Judicial Conduct for the
the judiciary. This standard applies not only to the decision Philippine Judiciary, which provide that
itself, but also to the process by which the decision is
made.[135] This Court will not hesitate to sanction with the Sec. 1. Judges shall exercise the judicial function
highest penalty magistrates who exhibit manifest undue independently x x x free from extraneous influence,
interest in their assigned cases.[136] inducement, pressure, threat or interference, direct or indirect,
from any quarter or for any reason.
FINALS CONSTITUTIONAL LAW I ACJUCO NOV 11, 2017 157

On this matter, the Court accepts the following findings in the


Report:
xxx xxx xxx

Knowing the nature of De Borjas profession, Justice Sabio, Jr.


Sec. 4. Judges shall not allow family, social, or other should have been wary of the former. He should have
relationships to influence judicial conduct or judgment. The foreseen that De Borja had the Meralco case on his mind
prestige of judicial office shall not be used or lent to advance when he called Justice Sabio, Jr. True enough, De Borja
the private interests of others, nor convey or permit others to mentioned the Meralco case and congratulated Justice Sabio,
convey the impression that they are in a special position to Jr. for having signed the TRO in favour of Meralco.
influence the judge.

But that was not the last time Justice Sabio, Jr. would hear
Sec. 5. Judges shall not only be free from inappropriate from De Borja. A month later, after Justice Sabio, Jr. had
connections with, and influence by, the executive and presided at the hearing of Meralcos prayer for preliminary
legislative branches of government, but must also appear to injunction on June 23, 2008, and the case was ripening for
be free therefrom to a reasonable observer. decision or resolution, De Borja again called up Justice Sabio,
Jr. and asked to meet him over dinner to chit chat about the
Meralco case.

Instead of telling off De Borja that he could not, and would not,
In the Investigators mind, although Justice Sabio signed the talk about the Meralco case, Justice Sabio, Jr. agreed to meet
TRO in favour of Meralco contrary to his brothers advice, De Borja in the lobby-lounge of the Ateneo Law School after
Justice Sabios unusual interest in holding on to the Meralco his evening class in Legal Ethics in said school.
case, seemed to indicate that he may have been actually
influenced by his brother to help GSIS. In arriving at this
conclusion, the Panel noted the following circumstances: (1)
Justice Sabio adamantly refused to yield the chairmanship of
the Special Ninth Division although the regular chairman, Justice Sabio Jr.s action of discussing the Meralco case with
Justice Reyes had returned to duty on June 10, 2008; and, (2) De Borja was highly inappropriate and indiscreet. First, in talks
Justice Sabio officiously prepared and signed a resolution (a with his brother; the second time in conversation with De
chore for the ponente Justice V. Roxas to perform), requiring Borja, Justice Sabio, Jr. broke the shield of confidentiality that
the GSIS and the SEC to comment on Meralcos Motion for covers the disposition of cases in the Court in order to
Justice B. Reyes to Assume the Chairmanship of the 9th preserve and protect the integrity and independence of the
Division, which he probably intended to delay the decision on Court itself. He ignored the injunction in Canon 1, Section 8 of
the preliminary injunction beyond the life of the TRO to the the New Code of Judicial Conduct for the Philippine Judiciary
prejudice of Meralco and the advantage of the GSIS. that: Judges shall exhibit and promote high standards of
judicial conduct (and discretion) in order to reinforce public
confidence in the judiciary which is fundamental to the
maintenance of judicial independence.
Based on the facts on record, the Court is wary of declaring
that Justice Sabio had been influenced by his brother by
speculating that he would have favored GSIS had he been a
part of the division which rendered the decision in the Meralco It was during that meeting with De Borja in the lobby-lounge
case. However, we do find that it was improper for Justice of the Ateneo Law School, that De Borja allegedly offered him
Sabio to hold on to the chairmanship of the Ninth Division the P10 million, in behalf of Meralco, to step out of the case and
despite the return of Justice Reyes, when Justice Sabios allow Justice Bienvenido Reyes to assume the chairmanship
designation as acting chairman was clearly only for the of the Special Ninth Division because Meralco was not
duration of Justice Reyes leave of absence. We likewise note comfortable with him (Justice Sabio, Jr.). He rejected the bribe
with disfavor his stubborn insistence on his own interpretation offer because he could not in conscience accept it.
of the IRCA and hostile, dismissive attitude towards equally
well-reasoned positions of his colleagues on the proper
interpretation of their rules. Such conduct on the part of
Justice Sabio did nothing to aid in the swift and amicable
resolution of his dispute with Justice Reyes but rather fanned Justice Sabio, Jr. was allegedly shocked and insulted that De
the flames of resentment between them. We deem this sort of Borja would think that he (Justice Sabio, Jr.) could be bribed
behavior unbecoming for a magistrate of his stature. or bought. The Panel is, however, honestly perplexed why in
spite of his outraged respectability, Justice Sabio, Jr. called up
De Borja two (2) days later (on July 3, 2008), to tell De Borja
to stop pestering him with his calls. The Panel is nonplussed
because, normally, a person who has been insulted would
Justice Sabios conversations with Mr. De Borja were improper never want to see, much less speak again, to the person who
and indiscreet. had disrespected him. He could have just shut off his cell
phone to De Borjas calls. De Borja denied that he reiterated
his offer of P10 million to Justice Sabio, Jr. He denied saying
FINALS CONSTITUTIONAL LAW I ACJUCO NOV 11, 2017 158

that even if the case should go up to the Supreme Court, GSIS As previously discussed, Justice Reyes appealed to Presiding
would still lose, hence, saying lang yung P10 million; baka Justice Vazquez in a letter dated July 22, 2008, reiterating his
sisihin ka pa ng mga anak mo. He testified that his reply to (Justice Reyes) request that the Presiding Justice render an
Justice Sabio, Jr.s call was deadma or indifference. Justice opinion which Division of the Court of Appeals the Eighth
Sabio, Jr. blamed that call of his to a lapse in judgment on his Division with him as chairman, or the Special Ninth Division
part. chaired by Justice Sabio should resolve the Meralco case.
This was in conjunction with an Interpleader filed by Justice
Roxas on the same issue with the Presiding Justice. Yet,
despite the fact that the Presiding Justice informed Justices
Reyes and Roxas that he would study the matter, Justices
Be that as it may, the Investigating Panel finds more credible Reyes and Justice Roxas, together with Justice Bruselas,
Justice Sabio, Jr.s story about De Borjas P10 million-bribe- promulgated the decision in the Meralco case on July 23,
offer on behalf of Meralco, than De Borjas denial that he made 2008. Justice Reyes and Justice Roxas did not withdraw their
such an offer. Why does the Panel believe him, and not De request for a ruling nor did either of them advise the Presiding
Borja? Justice beforehand of their intention to proceed with the
resolution of the Meralco case. Thus, when the Presiding
Justice issued his ruling on the chairmanship dispute on July
24, 2008, he was unaware of the promulgation of the Meralco
First, because Justice Sabio, Jr. verbally reported the rejected decision on July 23, 2008, under the aegis of Justice Reyes
bribe offer to CA Presiding Justice Conrado M. Vasquez, Jr. Eighth Division. As found by the Panel, Presiding Justice
the next day a fact admitted by Presiding Justice Vasquez, Jr. Vasquez, Jr. was completely taken aback when he learned
about it on July 24, 2008, the same day that he issued his
opinion on the chairmanship issue which by then had become
functus oficio. He felt belittled and humiliated by the
discourtesy of the two justices to him.
Second, even though Justice Sabio, Jr. did not mention the
bribe-offerors name in both his verbal and written reports to
Presiding Justice Vasquez, Jr., De Borja identified himself to
the media as the person alluded to.
It bears repeating here that under Canon 5, Section 3 of the
New Code of Judicial Conduct, judges are mandated to show
the appropriate consideration and respect for their colleagues
in the Judiciary.
Third, De Borjas allegation, that Justice Sabio, Jr. wanted P50
million, not P10 million, is not believable, for, if Justice Sabio,
Jr. quoted P50 million as his price, he would not have reported
the P10 million bribe offer to Presiding Justice Vasquez, Jr.
He would have waited for Meralcos reply to his counter- Thus, we adopt the finding of the Panel on this point and find
offer.[138] Justice Reyes guilty of simple misconduct, which is mitigated
by the fact that he repeatedly asked Presiding Justice
Vasquez to act on his request to rule on the conflicting
xxx xxx xxx interpretation of the IRCA. However, Justice Reyes should be
reprimanded for taking part in the decision of the subject case
without awaiting the ruling of the Presiding Justice.

Indeed, the Court agrees with the Panel that the allegation of
solicitation on the part of Justice Sabio is not credible.
Nevertheless, the continued communications between Justice Findings regarding the conduct of Justice Myrna Dimaranan-
Sabio and Mr. De Borja even after the latters rejected bribery Vidal
attempt is highly inappropriate and shows poor judgment on
the part of Justice Sabio who should have acted in
preservation of the dignity of his judicial office and the The Court finds well-taken and adopts the findings of the
institution to which he belongs. Panel of Investigators, to wit:

Premises considered, this Court is of the view that Justice Justice Dimaranan-Vidal deviated from the IRCA when she
Sabios indiscreet and imprudent conversations regarding the allowed herself to be rushed by Justice Roxas to sign the
Meralco case with his brother and Mr. De Borja and his Meralco decision on July 8, 2008, without reading the parties
actuations in the chairmanship dispute with Justice Reyes memoranda and without the deliberation among members of
constitute simple misconduct and conduct unbecoming of a the Division required by the IRCA. She knew that the TRO
justice of the Court of Appeals which warrant the penalty of would not expire until July 30, 2008 some three (3) weeks
two (2) months suspension without pay. away from July 8, 2008 yet she allowed herself to believe
Justice Roxas misrepresentation that signing the decision was
urgent. Her compliance with certain dissembling practices of
other justices of the Court, in violation of the IRCA, showed
weakness and lack of independence on her part.[139]
Findings regarding the conduct of Associate Justice
Bienvenido L. Reyes.
FINALS CONSTITUTIONAL LAW I ACJUCO NOV 11, 2017 159

The following sections of Canon 1 of the Code of Judicial He should have convened the Court en banc as soon as the
Conduct are instructive in this regard: alleged bribery attempt on Justice Sabio, Jr. was reported to
him, for it was an attempt to corrupt a member of the Court,
calling for the protection and preservation of the integrity of
the judicial processes of the Court, hence, an administrative
matter cognizable by the Court en banc. Section 5 (c), Rule I
SEC. 1. Judges shall exercise the judicial function of the IRCA, provides:
independently on the basis of their assessment of the facts
and in accordance with a conscientious understanding of the
law, free of any extraneous influence, inducement, pressure,
threat or interference, direct or indirect, from any quarter or for
any reason. Sec. 5. Matters cognizable by the Court en banc.- The Court
en banc

shall, inter alia:


SEC. 2. In performing judicial duties, judges shall be
independent from judicial colleagues in respect of decisions (a) xxx
which the judge is obliged to make independently.
(b) Adopt uniform administrative measures,
procedures, and policies for the protection and preservation of
the integrity of the judicial processes, x x x.
Allowing a fellow justice to induce her to deviate from
established procedure constitutes conduct unbecoming a
justice for which Justice Dimaranan-Vidal should be
ADMONISHED to be more circumspect in the performance of Presiding Justice Vasquez admitted his lapses in
her judicial duties. judgment.[140]

Findings regarding the conduct of Presiding Justice Conrado In the light of the foregoing observations of the Panel, this
M. Vasquez Court is of the view that much of the trouble now being faced
by the Court of Appeals could have been averted by timely,
judicious and decisive action on the part of the Presiding
Justice. Certainly, this unpleasant and trying episode in failure
It is the view of the Panel of Investigators that Presiding to act in the early part of his tenure as Presiding Justice has
Justice Vasquez failed to provide the leadership expected of indelibly impressed upon him what is required of him as leader
him as head of the Court of Appeals. The following quote from of the second highest court in the land. Nevertheless,
the Report summarizes the perceived lapses on the part of the Presiding Justice Vasquez is hereby severely reprimanded for
Presiding Justice: his failure to act promptly and decisively on the controversy as
required of him by the IRCA.

Clearly, Presiding Justice Vasquez, Jr. had been indecisive in


dealing with the turmoil arising from the Meralco case. He Findings regarding other personalities involved in the Meralco
vacillated and temporized on resolving the impasse between case
Justice Sabio, Jr. and Justice B. L. Reyes over the
chairmanship of the Division that should hear and decide the
Meralco case. He failed to take action on the reported bribe-
offer by Meralco to J. Sabio, Jr. He hesitated to assert his
leadership of the Court even when the parties repeatedly
urged him to lay down the rule for them to follow. Was he
hampered by the fact that he has relatives two daughters Although the Presiding Justice in his letter dated August 1,
employed in the GSIS, and a sister who is a consultant 2008 only referred to this Court the propriety of the actions of
thereof? He pleaded lack of authority. Was he not aware then, the Justices concerned in the Meralco case, we cannot simply
or did he discover too late, that under Section 11, Rule VIII of turn a blind eye to the facts brought to light during the
the IRCA, he is in fact authorized to act on any matter involving investigation that relate to potential liabilities of other
the Court and its members? That Rule provides: personalities in the Meralco case.

Sec. 11. x xx the Presiding Justice or any one acting in his With respect to Chairman Sabio, this Court has the power to
place is authorized to act on any matter not covered by these discipline members of the Bar and his attempt to influence a
Rules. Such action shall, however, be reported to the Court member of the Judiciary, his brother at that, should be referred
en banc. to the Bar Confidant for appropriate action.
FINALS CONSTITUTIONAL LAW I ACJUCO NOV 11, 2017 160

With respect to Mr. De Borja, the present investigation has


given this Court reason to believe that Mr. De Borja may be
criminally liable for his attempt to bribe a magistrate of the (5) Associate Justice Myrna Dimaranan-Vidal is found guilty
Court of Appeals. This matter should be referred to the of conduct unbecoming a Justice of the Court of Appeals and
Department of Justice for appropriate action. is ADMONISHED to be more circumspect in the discharge of
her judicial duties.

Pursuant to Section 13, Article VIII of the Constitution, this per


curiam decision was reached after deliberation of the Court en (6) PCGG Chairman Camilo L. Sabios act to influence the
banc. At the outset, the offer of three (3) members of the Court judgment of a member of the Judiciary in a pending case is
to recuse themselves was denied by the Court. Except for two hereby referred to the Bar Confidant for appropriate action;
members of the Court who were allowed to inhibit themselves
from the case, the Justices voted as follows: Twelve Justices
voted for the dismissal from service of Associate Justice
Vicente Q. Roxas and one (1) voted for his suspension from
the service for six (6) months. Ten (10) Justices voted for two (7) Justice Jose L. Sabio, Jr.s charge against Mr. Francis R.
(2) month suspension from service without pay of Associate De Borja for attempted bribery of a member of the Judiciary is
Justice Jose L. Sabio, one (1) voted for six-month suspension, hereby referred to the Department of Justice for appropriate
one (1) for reprimand only as he should be credited for being action.
a whistle blower and one (1) for his dismissal from the service.
Eight (8) Justices voted to reprimand Associate Justice
Bienvenido L. Reyes and five (5) for his suspension from the
service for one (1) month. As to the rest, the voting was
unanimous. This Decision shall take effect immediately.

WHEREFORE, the Court RESOLVES as follows: SO ORDERED.

(1) Associate Justice Vicente Q. Roxas is found guilty of


multiple violations of the canons of the Code of Judicial
Conduct, grave misconduct, dishonesty, undue interest and
conduct prejudicial to the best interest of the service, and is
DISMISSED from the service, with FORFEITURE of all
benefits, except accrued leave credits if any, with prejudice to
his re-employment in any branch or service of the government
including government-owned and controlled corporations;

(2) Associate Justice Jose L. Sabio, Jr. is found guilty of


simple misconduct and conduct unbecoming of a justice of the
Court of Appeals and is SUSPENDED for two (2) months
without pay, with a stern warning that a repetition of the same
or similar acts will warrant a more severe penalty;

(3) Presiding Justice Conrado M. Vasquez, Jr. is SEVERELY


REPRIMANDED for his failure to act promptly and decisively
in order to avert the incidents that damaged the image of the
Court of Appeals, with a stern warning that a repetition of the
same or similar acts will warrant a more severe penalty;

(4) Associate Justice Bienvenido L. Reyes is found guilty of


simple misconduct with mitigating circumstance and is
REPRIMANDED, with a stern warning that a repetition of the
same or similar acts will warrant a more severe penalty;
FINALS CONSTITUTIONAL LAW I ACJUCO NOV 11, 2017 161

G.R. No. L-35377-78 July 31, 1975 SO ORDERED.

THE PEOPLE OF THE PHILIPPINES, plaintiff-appellee, G.R. No. L-41313 November 6, 1975

vs. ALIPIO MONDIGUING and ANDRES DUNUAN, petitioners,

CAMILO PILOTIN, VINCENT CRISOLOGO, ISIDRO PUGAL vs.


and ERNING ABANO, defendants-appellants.
HON. FRANCISCO MEN ABAD, as Judge of the Court of
RESOLUTION First Instance of Ifugao; PEOPLE OF THE PHILIPPINES;
MARIANO PACTIW, alias Bugbug; DULMOG ABLUYEN
AQUINO, J.: and ANGELINA ABLUYEN, respondents..

Vincent Crisologo through counsel filed a verified motion R E S O L U T I O N.


praying for the transfer to the New Bilibid Prisons or,
alternatively, to Camps Crame, Aguinaldo or Olivas, of the AQUINO, J.:
place of trial of Criminal Case No. 3949 of the municipal
court of Vigan, Ilocos Sur, wherein he, as sole defendant, Alipio Mondiguing and Andres Dunuan are two of the ten
is charged with illegal possession of firearms and defendants accused of double murder, frustrated murder and
ammunitions. attempted murder in Criminal Case No. 140 of the Court of
First Instance of Ifugao Province (People vs. George Bayucca
As justificatory ground, he alleged that his life would be in et al.). That case was filed in connection with an ambuscade
jeopardy if he were to be confined in the Vigan municipal which was perpetuated on July 23, 1970 at Baag, Banaue,
jail during the trial because there are many political Ifugao. As a result of that incident, Governor Gualberto
enemies of the Crisologo family in that vicinity; some of Lumauig of Ifugao was wounded and his executive assistant
the adherents of the Crisologos had in fact been and his driver were killed. Up to this time the accused in that
murdered in Ilocos Sur, and his father, Congressman case have not been arraigned. .
Floro Crisologo, was shot to death while hearing mass at
the Vigan cathedral. On September 4, 1975 Mondiguing and Dunuan filed in this
Court a petition to transfer the venue of the case to Baguio
Bluntly, he affirmed that inside that jail he would be a sitting City or Quezon City. They claimed that they could not expect
duck for a gunwielder or grenade-thrower who wants to a fair and impartial trial in Lagawe, Ifugao because Judge
assassinate him. He could even be lynched or shot to death Francisco Men Abad of the Court of First Instance of that
on the specious pretext that he was trying to escape. province is a protege' of Governor Lumauig and his brother,
former Congressman Romulo Lumauig, and because their
Asked to comment on the motion, the Provincial Fiscal of witnesses would be afraid to testify for fear of harassment and
Ilocos Sur signified his conformity to the transfer of the venue reprisals. The petitioners further claimed that, as may be
of the trial to the New Bilibid Prisons. inferred from previous incidents recounted in the petition, their
lives and the lives of their witnesses and lawyers would be in
grave danger in Ifugao because of the tensions and
Section 5(4), Article X of the Constitution expressly empowers antagonisms spawned by the case and the political rivalry
this Court to "order a change of venue or place of trial to avoid between the Lumauig and Mondiguing factions. (The
a miscarriage of justice". Here, what is involved is not merely accused, George Bayucca was killed on October 28, 1970 and
a miscarriage of justice but the personal safety of movant Alipio Mondiguing resigned as mayor of Banaue and took
Crisologo, the accused. It would be absurd to compel him to refuge in Baguio City). .
undergo trial in a place where his life would be imperilled.
The Acting Solicitor General interposed no objection to the
Present hostile sentiment against the accused at the place of change of venue but he invited the Court's attention to the
trial is a justification for transfer of venue (See State vs. Siers, suggestion of Governor Lumauig that the case may be
136 S. E. 503, 103, W. Va. 30; 22 C.J.S. 310).1äwphï1.ñët transferred to the proper court in Isabela in view of its
proximity to Ifugao. .
We find Crisologo's motion to be meritorious. The change of
venue involves not merely the change of the place of hearing Respondent Judge Francisco Men Abad in his comment
but also the transfer of the expediente of Criminal Case No. disputed the correctness or truth of the grounds relied upon
3949 to another court. According to Crisologo's motion, the for the change of venue and prayed that the petition be
alleged evidence against him is in the custody of the dismissed. He said that, if there would be bias on his part, he
authorities at Camp Crame, Quezon City. The transfer of would be biased in favor of the People of the Philippines. He
Criminal Case No. 3949 to the City Court of Quezon City and said that the crime charged was not "committed personally
the holding of the trial at Camp Crame appear to be the most against" Governor Lumauig. That statement is not correct
convenient arrangement. since the governor is one of the victims mentioned in the
information. .
WHEREFORE, the municipal court of Vigan is directed to
transfer the record of Criminal Case No. 3949 to the City Court Judge Abad revealed that petitioner Dunuan sent to the court
of Quezon City where it should be re-docketed and raffled to a letter dated August 30, 1975 wherein he declined the
any Judge thereof. The case may be tried at Camp Crame. services of Atty. Jose W. Diokno (who filed the instant petition
The usual precautions and security measures should be for transfer of venue). In view of that disclosure, the petition
adopted in bringing defendant Crisologo to Camp Crame on herein should be regarded as having been filed only by Alipio
the occasion of the hearing. Mondiguing. .
FINALS CONSTITUTIONAL LAW I ACJUCO NOV 11, 2017 162

The fact is that this Court in Paredes vs. Abad, L-36927-28, G.R. No. L-56158-64 March 17, 1981
April 15, 1974, 56 SCRA 522, 534, disqualified Judge Abad
from trying the electoral protests filed by Crescencio Paredes PEOPLE OF THE PHILIPPINES, petitioner,
and Venancio Uyan against Gualberto Lumauig and John
Langbayan. In that case it was alleged that Judge Abad was
a political leader of Governor Lumauig and was recommended vs.
to his present position by the Lumauig brothers. .
MAYOR PABLO SOLA, SANGGUNIANG BAYAN MEMBER
The issue is whether Mondiguing's plea for a change of venue FRANCISCO (ECOT) GARCIA, RICARDO (CADOY)
is justified. A change of the place of trial in criminal cases GARCIA, JOSE BETHOVEN (ATSONG) CABRAL,
should not be granted for whimsical or flimsy reasons. "The CAPTAIN FLORENDO BALISCAO, JOHN, PETER,
interests of the public require that, to secure the best results OSCAR, OMAR, JACK, RICHARD, JAMES, DONALD,
and effects in the punishment of crime, it is necessary to WILLIAM, ROBERT, HOMER, JESSIE, ANDY, PAUL, all
prosecute and punish the criminal in the very place, as near surnamed DOES respondents.
as may be, where he committed his crime" (Manila Railroad
Co. vs. Attorney General, 20 Phil. 523, 562). . FERNANDO, C.J.:

This Court is invested with the prerogative of ordering "a The power of this Tribunal, constitutionally mandated, 1 to
change of venue or place of trial to avoid a miscarriage of order a change of venue to avoid any miscarriage of justice as
justice" (Sec. 5[4], Art. X of the Constitution). It "possesses well as the procedure ordained in the implementation of the
inherent power and jurisdiction to decree that the trial and right to bail 2 are involved in this petition which, even if not so
disposition of a case pending in a Court of First Instance be denominated, partakes of the nature of a certiorari. It must
transferred to another Court of First Instance within the same have been the zeal of private prosecutors Francisco Cruz and
district whenever the interest of justice and truth so demand, Renecio Espiritu, 3 no doubt under the conviction that there
and there are serious and weighty reasons to believe that a was no time to lose, that must have led them to devote less
trial by the court that originally had jurisdiction over the case than that full measure of attention to certain fundamentals.
would not result in a fair and impartial trial and lead to a They ignored the principle that the responsibility for the
miscarriage of justice" (People vs. Gutierrez, L-32282-83, conduct of the prosecution is with the public officials
November 26, 1970, 36 SCRA 172, 185). . concerned. Nonetheless, the importance of the questions
raised, the need for a change of venue and the cancellation of
A change of venue was ordered by this Court in a case where the bail bonds, necessitated that further action be taken.
it was shown that the accused might be liquidated by his Accordingly, in a resolution dated February 12, 1981, one day
enemies in the place where the trial was originally scheduled after the filing of the petition, the Court required the comment
to be held (People vs. Pilotin Vincent Crisologo, movant, L- of the Solicitor General as well as of the private respondents,
3537778, July 31, 1975). 4the accused in six pending criminal cases before the Court
of First Instance of Negros Occidental.
After a careful consideration of the circumstances recited in
Mondiguing's petition to support his request for a change of On March 4, 1981, the Comment was submitted by Solicitor
the place of trial, we have reached the conclusion that his General Estelito P. Mendoza. 5 It opened with this preliminary
petition is meritorious. . statement: "The present petition was filed by the private
prosecutors in Criminal Cases Nos. 1700-1706, People v.
Pablo Sola, et al., pending trial before the Court of First
In the interest of a fair and impartial trial and to avoid a Instance of Negros Occidental. Rightly, any petition before this
miscarriage of justice and considering that his life would be in Honorable Court on behalf of the People of the Philippines
danger if he were to be tried in Lagawe, Ifugao, he should be can, under the law, be instituted only by the Solicitor General.
tried by the Circuit Criminal Court in the City of Baguio. . The assertion of the petitioner private prosecutors that they
are instituting the action 'subject to the control and supervision
The other relief sought by Mondiguing, which is that he be of the Fiscal' will not, therefore, improve their legal standing."
transferred from the Philippine Constabulary headquarters at 6 Nonetheless, it did not press the legal point but instead
Lagawe, Ifugao to Camp Crame should be submitted for the adopted "the two-pronged trusts of the petition: 1. the setting
consideration of the Circuit Criminal Court. . aside, by certiorari, of the order of the Municipal Court of
Kabankalan, presided over by Judge Rafael Gasataya,
WHEREFORE, the petition of Alipio Mondiguing for the granting bail to the accused in the criminal cases mentioned
transfer of the venue of Criminal Case No. 140 of the Court of above, and 2. the petition for a change of venue or place of
First Instance of Ifugao is granted. The said case should be trial of the same criminal cases to avoid a miscarriage of
transferred to the Circuit Criminal Court of the Second Judicial justice. 7
District so that it may be heard in Baguio City. .
The facts were therein narrated thus: "On September 15,
SO ORDERED. . 1980, acting on the evidence presented by the Philippine
Constabulary commander at Hinigaran, Negros Occidental,
the Court of First Instance of that province issued a search
Barredo (Actg. Chairman), Antonio, Concepcion, Jr. and warrant for the search and seizure of tile deceased bodies of
Martin, JJ., concur. seven persons believed in the possession of the accused
Pablo Sola in his hacienda at Sta. Isabel, Kabankalan, Negros
Fernando J., is on leave. Occidental. * * * On September 16, 1980 armed with the above
warrant, elements of the of the 332nd PC/INP Company
proceeded to the place of Sola. Diggings made in a canefield
yielded two common graves containing the bodies of
Fernando Fernandez, Mateo Olimpos, Alfredo Perez,
Custodio Juanica, Arsolo Juanica, Rollie Callet and
FINALS CONSTITUTIONAL LAW I ACJUCO NOV 11, 2017 163

Bienvenido Emperado. On September 23 and October 1, Bacolod City. The Constitution is quite explicit. The Supreme
1980, the PC provincial commander of Negros Occidental filed Court could order "a change of venue or place of trial to avoid
seven (7) separate complaints for murder against the accused a miscarriage of justice." 10 The Constitutional Convention of
Pablo Sola, Francisco Garcia, Ricardo Garcia, Jose Bethoven 1971 wisely incorporated the ruling in the landmark decision
Cabral, Florendo Baliscao and fourteen (14) other persons of of People v. Gutierrez, 11 where Justice J. B. L. Reyes as
unknown names. The cases were docketed as Criminal Cases ponente vigorously and categorically affirmed: "In the
No. 4129, 4130, 4131, 4137, 4138, 4139 and 4140 of the particular case before Us, to compel the prosecution to
Municipal Court of Kabankalan. After due preliminary proceed to trial in a locality where its witnesses will not be at
examination of the complainant's witnesses and his other liberty to reveal what they know is to make a mockery of the
evidence, the municipal court found probable cause against judicial process, and to betray the very purpose for which
the accused. It thus issued an order for their a. rest. However, courts have been established." 12 Why a change of venue is
without giving the prosecution the opportunity to prove that the imperative was made clear in the Comment of the Solicitor
evidence of guilt of the accused is strong, the court granted General. Thus: "The exercise by this Honorable Court of its
them the right to post bail for their temporary release. The above constitutional power in this case will be appropriate.
accused Pablo Sola, Francisco Garcia, and Jose Bethoven The witnesses in the case are fearful for their lives. They are
Cabral availed themselves of this right and have since been afraid they would be killed on their way to or from Himamaylan
released from detention. In a parallel development. the during any of the days of trial. Because of qqqts fear, they may
witnesses in the murder cases informed the prosecution of either refuse to testify or testimony falsely to save their lives.
their fears that if the trial is held at the Court of First Instance 13 Respondent Florendo Baliscao was not averse to such
branch in Himamaylan which is but 10 kilometers from transfer, but his preference is for a court anywhere in Metro
Kabankalan, their safety could be jeopardized. At least two of Manila. 14 Respondent Francisco Garcia confined his
the accused are officials with power and influence in comment to the question of the cancellation of the bail bonds.
Kabankalan and they have been released on bail. In addition, Respondent Pablo Sola made clear that he had "no objection
most of the accused remained at large. Indeed, there have to the transfer. 15 It may be added that there may be cases
been reports made to police authorities of threats made on the where the fear, objectively viewed, may, to some individuals,
families of the witnesses." 8 The facts alleged argue strongly be less than terrifying, but the question must always be the
for the remedies sought, namely a change of venue and the effect it has on the witnesses who will testify. The primordial
cancellation of the bail bonds. aim and intent of the Constitution must ever be kept in mind.
In case of doubt, it should be resolved in favor of a change of
On the very next day, March 15, 1981, this Court issued the venue. As a matter of fact, there need not be a petition of this
following resolution: "The Court Resolved to: (a) [Note] the character filed before this Court. Such a plea could have been
comment of the Solicitor General on the urgent petition for done administratively. In this particular case, however, there
change of venue and cancellation of bail bonds, adopting the is justification for the procedure followed in view of the fact that
plea of the petition, namely, (1) the setting aside, by certiorari, along with the change of venue, the cancellation of the bail
of the order of the Municipal Court of Kabankalan, presided bonds was also sought.
over by Judge Rafael Gasataya, granting bail to the accused
in Criminal Cases Nos. 4129, 4130, 4131, 4137, 4138, 4139 2. Equally so the cancellation of the bail bonds is more than
and 4140, all entitled "People of the Philippines v. Mayor justified. Bail was granted to the accused in the Order of the
Pablo Sola. et al."; (2) the petition for a change of venue or Municipal Court without hearing the prosecution That is to
place of trial of the same criminal cases to avoid a miscarriage disregard the authoritative doctrine enunciated in People v.
of Justice; (b) [Transfer] the venue of the aforesaid criminal San Diego. 16 As pointed out by Justice Capistrano, speaking
cases to Branch V of the Court of First Instance of Negros for the Court: "The question presented before us is, whether
Occidental at Bacolod City, presided by Executive Judge the prosecution was deprived of procedural due process. The
Alfonso Baguio, considering that District Judge Ostervaldo answer is in the affirmative. We are of the considered opinion
Emilia of the Court of First Instance, Negros Occidental, that whether the motion for bail of a defendant who is in
Branch VI at Himamaylan has an approved leave of absence custody for a capital offense be resolved in a summary
covering the period from January 12 to March 12, 1981 due to proceeding or in the course of a regular trial, the prosecution
a mild attack of cerebral thrombosis and that the said Branch must be given an opportunity to present, within a reasonable
V is the nearest court station to Himamaylan: and (c) [Await] time, all the evidence that it may desire to introduce before the
the comment of respondents on the petition to cancel bail, court should resolve the motion for bail. If, as in the criminal
without prejudice to the public officials concerned taking the case involved in the instant special civil action, the prosecution
necessary measures to assure the safety of the witnesses of should be denied such an opportunity, there would be a
the prosecution." 9 Thus, the issue of a change of venue has violation of procedural due process, and the order of the court
become moot and academic. The comments respectively granting bail should be considered void on that ground." 17
submitted by respondent Florendo Baliscao on March 5, 1981, These words of Justice Cardozo come to mind: "The law, as
respondent Francisco Garcia on March 11, 1981 and we have seen, is sedulous in maintaining for a defendant
respondent Pablo Sola on March 16, 1981, dealt solely with charged with crime whatever forms of procedure are of the
the question of the cancellation of the bail bonds. Such essence of an opportunity to defend. Privileges so
comments were considered as answers, with the case fundamental as to be inherent in every concept of a fair trial
thereafter deemed submitted for decision. that could be acceptable to the thought of reasonable men will
be kept inviolate and inviolable, however crushing may be the
The sole remaining issue of the cancellation of the bail bonds pressure of incriminating proof. But justice, though due to the
of respondents, there being a failure to abide by the basic accused, is due to the accuser also. The concept of fairness
requirement that the prosecution be heard in a case where the must not be strained till it is narrowed to a filament. We are to
accused is charged with a capital offense, prior to bail being keep the balance true." 18 This norm which is of the very
granted, must be decided in favor of petitioner. The bail bonds essence of due process as the embodiment of justice requires
must be cancelled and the case remanded to the sala of that the prosecution be given the opportunity to prove that
Executive Judge Alfonso Baguio for such hearing. So we rule. there is strong evidence of guilt. It does not suffice, as
asserted herein, that the questions asked by the municipal
judge before bail was granted could be characterized as
1. It may not be amiss to say a few words on the question of
transferring the place of trial, in this case, from Himamaylan to
FINALS CONSTITUTIONAL LAW I ACJUCO NOV 11, 2017 164

searching. That fact did not cure an infirmity of a jurisdictional


character. 19

WHEREFORE, the assailed order of Judge Rafael Gasataya


granting bail to private respondents is nullified, set aside, and
declared to be without force and effect. Executive Judge
Alfonso Baguio of the Court of First Instance of Negros
Occidental, to whose sala the cases had been transferred by
virtue of the resolution of this Court of March 5, 1981, is
directed forthwith to hear the petitions for bail of private
respondents, with the prosecution being duly heard on the
question of whether or not the evidence of guilt against the
respondents is strong. This decision is immediately executory.
No costs.

Teehankee, Makasiar, Aquino, Concepcion, Jr., Fernandez,


Guerrero, De Castro and Melencio-Herrera JJ., concur.

Barredo and Abad Santos, JJ., are on leave.


FINALS CONSTITUTIONAL LAW I ACJUCO NOV 11, 2017 165

G.R. No. 79690-707 February 1, 1989 opportunity to present all defenses, arguments and evidence
that he wanted to present for the consideration of this Court.
ENRIQUE A. ZALDIVAR, petitioner, The Court did not summarily impose punishment upon the
respondent which it could have done under Section 1 of Rule
71 of the Revised Rules of Court had it chosen to consider
vs. respondent's acts as constituting "direct contempt."

THE HONORABLE SANDIGANBAYAN and HONORABLE 2. In his point C, respondent's counsel argues that it was "error
RAUL M. GONZALEZ, claiming to be and acting as for this Court to charge respondent under Rule 139 (b) and
Tanodbayan-Ombudsman under the 1987 Constitution, not 139 of the Revised Rules of Court."
respondents.
In its per curiam Resolution, the Court referred to Rule 139 (b)
G.R. No. 80578 February 1, 1989 of the Revised Rules of Court pointing out that:

ENRIQUE A. ZALDIVAR, petitioner, [R]eference of complaints against attorneys either to the


Integrated Bar of the Philippines or to the Solicitor General is
vs. not mandatory upon the Supreme Court such reference to the
Integrated Bar of the Philippines or to the Solicitor General is
HON. RAUL M. GONZALES, claiming to be and acting as certainly not an exclusive procedure under the terms of Rule
Tanodbayan-Ombudsman under the 1987 Constitution, 139 (b) of the Revised Rules of Court, especially where the
respondent. charge consists of acts done before the Supreme Court.

RESOLUTION The above statement was made by the Court in response to


respondent's motion for referral of this case either to the
Solicitor General or to the Integrated Bar of the Philippines
PER CURIAM: under Rule 139 (b). Otherwise, there would have been no
need to refer to Rule 139 (b). It is thus only necessary to point
We have examined carefully the lengthy and vigorously out that under the old rule, Rule 139, referral to the Solicitor
written Motion for Reconsideration dated October 18, 1988 General was similarly not an exclusive procedure and was not
filed by counsel for respondent Raul M. Gonzalez, relating to the only course of action open to the Supreme Court. It is well
the per curiam Resolution of the Court dated October 7, 1988. to recall that under Section 1 (entitled "Motion or complaint")
We have reviewed once more the Court's extended per of Rule 139, "Proceedings for the removal or suspension of
curiam Resolution, in the light of the argument adduced in the attorneys may be taken by the Supreme Court, (1) on its own
Motion for Reconsideration, but must conclude that we find no motion, or (2) upon the complaint under oath of another in
sufficient basis for modifying the conclusions and rulings writing" (Parentheses supplied). The procedure described in
embodied in that Resolution. The Motion for Reconsideration Sections 2 et seq. of Rule 139 is the procedure provided for
sets forth copious quotations and references to foreign texts suspension or disbarment proceedings initiated upon sworn
which, however, whatever else they may depict, do not reflect complaint of another person, rather than a procedure required
the law in this jurisdiction. for proceedings initiated by the Supreme Court on its own
motion. It is inconceivable that the Supreme Court would
initiate motu proprio proceedings for which it did not find
Nonetheless, it might be useful to develop further, in some
probable cause to proceed against an attorney. Thus, there is
measure, some of the conclusions reached in the per curiam
no need to refer a case to the Solicitor General, which referral
Resolution, addressing in the process some of the "Ten (10)
is made "for investigation to determine if there is sufficient
Legal Points for Reconsideration," made in the Motion for
ground to proceed with the prosecution of the respondent"
Reconsideration.
(Section 3, Rule 139), where the Court itself has initiated
against the respondent. The Court may, of course, refer a
1. In respondent's point A, it is claimed that it was error for this case to the Solicitor General if it feels that, in a particular case,
Court "to charge respondent [with] indirect contempt and further factual investigation is needed. In the present case, as
convict him of direct contempt." pointed out in the per curiam Resolution of the Court (page
18), there was "no need for further investigation of facts in the
In the per curiam Resolution (page 50), the Court concluded present case for it [was] not substantially disputed by
that "respondent Gonzalez is guilty both of contempt of court respondent Gonzalez that he uttered or wrote certain
in facie curiae and of gross misconduct as an officer of the statements attributed to him" and that "in any case,
court and member of the bar." The Court did not use the respondent has had the amplest opportunity to present his
phrase "in facie curiae" as a technical equivalent of "direct defense: his defense is not that he did not make the
contempt," though we are aware that courts in the United statements ascribed to him but that those statements give rise
States have sometimes used that phrase in speaking of "direct to no liability on his part, having been made in the exercise of
contempts' as "contempts in the face of the courts." Rather, his freedom of speech. The issues which thus need to be
the court sought to convey that it regarded the contumacious resolved here are issues of law and of basic policy and the
acts or statements (which were made both in a pleading filed Court, not any other agency, is compelled to resolve such
before the Court and in statements given to the media) and issues."
the misconduct of respondent Gonzalez as serious acts
flaunted in the face of the Court and constituting a frontal In this connection, we note that the quotation in page 7 of the
assault upon the integrity of the Court and, through the Court, Motion for Reconsideration is from a dissenting opinion of Mr.
the entire judicial system. What the Court would stress is that Justice Black in Green v. United State. 1 It may be pointed out
it required respondent, in its Resolution dated 2 May 1988, to that the majority in Green v. United States, through Mr. Justice
explain "why he should not be punished for contempt of court Harlan, held, among other things, that: Federal courts do not
and/or subjected to administrative sanctions" and in respect lack power to impose sentences in excess of one year for
of which, respondent was heard and given the most ample criminal contempt; that criminal contempts are not subject to
FINALS CONSTITUTIONAL LAW I ACJUCO NOV 11, 2017 166

jury trial as a matter of constitutional right; nor does the (US) applied by courts. In Logunzad v. Vda. de Gonzales, 3 this
Constitution require that contempt subject to prison terms of Court, speaking through Mme. Justice Melencio-Herrera said:
more than one year be based on grand jury indictments.
...The right of freedom of expression indeed, occupies a
In his concurring opinion in the same case, Mr. Justice preferred position in the "hierarchy of civil liberties" (Philippine
Frankfurter said: Blooming Mills Employees Organization v. Philippine
Blooming Mills Co., Inc., 51 SCRA 191 [1963]. It is not,
Whatever the conflicting views of scholars in construing more however, without limitations. As held in Gonzales v.
or less dubious manuscripts of the Fourteenth Century, what Commission on Elections, 27 SCRA 835, 858 [1960]:
is indisputable is that from the foundation of the United States
the constitutionality of the power to punish for contempt "From the language of the specific constitutional provision, it
without the intervention of a jury has not been doubted. The would appear that the right is not susceptible of any limitation.
First Judiciary Act conferred such a power on the federal No law may be passed abridging the freedom of speech and
courts in the very act of their establishment, 1 State 73, 83, of the press. The realities of life in a complex society preclude
and of the Judiciary Committee of eight that reported the bill however, a literal interpretation. Freedom of expression is not
to the Senate, five member including the chairman, Senator, an absolute. It would be too much to insist that all times and
later to be Chief Justice, Ellsworth, had been delegates to the under all circumstances it should remain unfettered and
Constitutional Convention (Oliver Ellsworth, Chairman, unrestrained. There are other societal values that press for
William Paterson, Caleb Strong, Ricard Basett, William Few. recognition."
1 Annals of Cong 17). In the First Congress itself no less than
nineteen member including Madison who contemporaneously The prevailing doctrine is that the clear and present danger
introduced the Bill of Rights, had been delegates to the rule is such a limitation. Another criterion for permissible
Convention. And when an abuse under this power manifested limitation on freedom of speech and of the press, which
itself, and led Congress to define more explicitly the summary includes such vehicles of the mass media as radio, television
power vested in the courts, it did not remotely deny the and the movies, is the "balancing-of-interests test" (Chief
existence of the power but merely defined the conditions for Justice Enrique M. Fernando on the Bill of Rights, 1970 ed.,
its exercise more clearly, in an Act "declaratory of the law p. 79). The principle "requires a court to take conscious and
concerning contempts of court." Act of Mar. 2, 1831, 4 Stat detailed consideration of the interplay of interests observable
487. in a given situation or type of situation (Separate Opinion of
the late Chief Justice Castro in Gonzales v. Commission on
xxxxxxxxx Elections, supra, p. 899). (Emphasis Supplied) 4

Nor has the constitutionality of the power been doubted by this Under either the "clear and present danger" test or the
Court throughout its existence . In at least two score cases in "balancing-of-interest test," we believe that the statements
this Court, not to mention the vast mass of decisions in the here made by respondent Gonzalez are of such a nature and
lower federal courts, the power to punish summarily has been were made in such a manner and under such circumstances,
accepted without question. ... 2 as to transcend the permissible limits of free speech. This
conclusion was implicit in the per curiam Resolution of
To say that a judge who punishes a contemnor judges his own October 7, 1988. It is important to point out that the
cause, is simplistic at best. The judge who finds himself "substantive evil" which the Supreme Court has a right and a
compelled to exercise the power to punish for contempt does duty to prevent does not, in the instant case, relate to threats
so not really to avenge a wrong inflicted upon his own person; of physical disorder or overt violence or similar disruptions of
rather he upholds and vindicates the authority, dignity and public order. 5 What is here at stake is the authority of the
integrity of the judicial institution and its claim to respectful Supreme Court to confront and prevent a "substantive evil"
behaviour on the part of all persons who appears before it, consisting not only of the obstruction of a free and fair hearing
and most especially from those who are officers of the court. of a particular case but also the avoidance of the broader evil
of the degradation of the judicial system of a country and the
destruction of the standards of professional conduct required
3. In his point D, respondent counsel urges that it is error "for from members of the bar and officers of the courts. The
this Court to apply the "visible tendency" rule rather than the "substantive evil" here involved, in other words, is not as
"clear and present danger" rule in disciplinary and contempt palpable as a threat of public disorder or rioting but is certainly
charges." no less deleterious and more far reaching in its implications
for society.
The Court did not purport to announce a new doctrine of
"visible tendency," it was, more modestly, simply paraphrasing 4. In his point H, respondent's counsel argues that it is error
Section 3 (d) of Rule 71 of the Revised Rules of Court which "for this Court to hold that intent is irrelevant in charges of
penalizes a variety of contumacious conduct including: "any misconduct." What the Court actually said on this point was:
improper conduct tending, directly or indirectly, to impede,
obstruct or degrade the administration of justice."
Respondent Gonzalez disclaims an intent to attack and
denigrate the Court. The subjectivities of the respondent are
The "clear and present danger" doctrine invoked by irrelevant so far as characterization of his conduct or
respondent's counsel is not a magic incantation which misconduct is concerned. He will not, however, be allowed to
dissolves all problems and dispenses with analysis and disclaim the natural and plain import of his words and acts. It
judgment in the testing of the legitimacy of claims to free is, upon the other hand, not irrelevant to point out that the
speech, and which compels a court to exonerate a defendant respondent offered no apology in his two (2) explanations and
the moment the doctrine is invoked, absent proof of impending exhibited no repentance (Resolution, p. 7; footnotes omitted).
apocalypse. The clear and present danger" doctrine has been
an accepted method for marking out the appropriate limits of
freedom of speech and of assembly in certain contexts. It is The actual subjectivities of the respondent are irrelevant
not, however, the only test which has been recognized and because such subjectivities (understood as pyschological
FINALS CONSTITUTIONAL LAW I ACJUCO NOV 11, 2017 167

phenomena) cannot be ascertained and reached by the


processes of this Court. Human intent can only be shown
derivatively and implied from an examination of acts and
statements. Thus, what the Court was saying was that
respondent's disclaimer of an intent to attack and denigrate
the Court, cannot prevail over the plain import of what he did
say and do. Respondent cannot negate the clear import of his
acts and statements by simply pleading a secret intent or state
of mind incompatible with those acts or statements. It is
scarcely open to dispute that, e.g., one accused of homicide
cannot successfully deny his criminal intent by simply
asserting that while he may have inserted a knife between the
victim's ribs, he actually acted from high motives and kind
feelings for the latter.

5 In his point 1, respondent's counsel argues that it is error


"for this Court to punish respondent for contempt of court for
out of court publications."

Respondent's counsel asks this Court to follow what he


presents as alleged modern trends in the United Kingdom and
in the United States concerning the law of contempt. We are,
however, unable to regard the texts that he cites as binding or
persuasive in our jurisdiction. The Court went to some length
to document the state of our case law on this matter in its per
curiam Resolution. There is nothing in the circumstances of
this case that would suggest to this Court that that case law,
which has been followed for at least half a century or so, ought
to be reversed.

6. In his point J, respondent's counsel pleads that the


imposition of indefinite suspension from the practice of law
constitutes "cruel, degrading or inhuman punishment". The
Court finds it difficult to consider this a substantial
constitutional argument. The indefiniteness of the
respondent's suspension, far from being "cruel" or "degrading"
or "inhuman," has the effect of placing, as it were, the key to
the restoration of his rights and privileges as a lawyer in his
own hands. That sanction has the effect of giving respondent
the chance to purge himself in his own good time of his
contempt and misconduct by acknowledging such
misconduct, exhibiting appropriate repentance and
demonstrating his willingness and capacity to live up to the
exacting standards of conduct rightly demanded from every
member of the bar and officer of the courts.

ACCORDINGLY, the Court Resolved to DENY the Motion for


Reconsideration for lack of merit. The denial is FINAL.

The Court also NOTED the Ex-Parte Manifestation and


Motion, dated October 25, 1988 and the Supplemental
Manifestation, dated October 27, 1988, filed by respondent

Fernan, C.J., Narvasa, Melencio-Herrera, Gutierrez, Jr., Cruz,


Paras, Feliciano, Gancayco, Padilla, Bidin, Sarmiento, Cortes,
Griño-Aquino, Medialdea and Regalado, JJ., concur.
FINALS CONSTITUTIONAL LAW I ACJUCO NOV 11, 2017 168

Resolution March 18, 1954 per cent in the nineteen hundred and fifty-two bar
examinations; seventy-two per cent in the in the nineteen
In the Matter of the Petitions for Admission to the Bar of hundred and fifty-three bar examinations; seventy-three per
Unsuccessful Candidates of 1946 to 1953; cent in the nineteen hundred and fifty-four bar examinations;
seventy-four per cent in the nineteen hundred and fifty-five bar
examinations without a candidate obtaining a grade below fifty
ALBINO CUNANAN, ET AL., petitioners. per cent in any subject, shall be allowed to take and subscribe
the corresponding oath of office as member of the Philippine
Jose M. Aruego, M.H. de Joya, Miguel R. Cornejo, and Bar: Provided, however, That for the purpose of this Act, any
Antonio Enrile Inton for petitioners. exact one-half or more of a fraction, shall be considered as
one and included as part of the next whole number.
Office of the Solicitor General Juan R. Liwag for respondent.
SEC. 2. Any bar candidate who obtained a grade of seventy-
DIOKNO, J.: five per cent in any subject in any bar examination after July
fourth, nineteen hundred and forty-six shall be deemed to
have passed in such subject or subjects and such grade or
In recent years few controversial issues have aroused so grades shall be included in computing the passing general
much public interest and concern as Republic Act No. average that said candidate may obtain in any subsequent
972, popularly known as the "Bar Flunkers' Act of 1953." examinations that he may take.
Under the Rules of Court governing admission to the bar, "in
order that a candidate (for admission to the Bar) may be
deemed to have passed his examinations successfully, he SEC. 3. This Act shall take effect upon its approval.
must have obtained a general average of 75 per cent in all
subjects, without falling below 50 per cent in any subject." Enacted on June 21, 1953, without the Executive approval.
(Rule 127, sec. 14, Rules of Court). Nevertheless, considering
the varying difficulties of the different bar examinations held After its approval, many of the unsuccessful postwar
since 1946 and the varying degree of strictness with which the candidates filed petitions for admission to the bar invoking its
examination papers were graded, this court passed and provisions, while others whose motions for the revision of their
admitted to the bar those candidates who had obtained an examination papers were still pending also invoked the
average of only 72 per cent in 1946, 69 per cent in 1947, 70 aforesaid law as an additional ground for admission. There are
per cent in 1948, and 74 per cent in 1949. In 1950 to 1953, also others who have sought simply the reconsideration of
the 74 per cent was raised to 75 per cent. their grades without, however, invoking the law in question. To
avoid injustice to individual petitioners, the court first reviewed
Believing themselves as fully qualified to practice law as those the motions for reconsideration, irrespective of whether or not
reconsidered and passed by this court, and feeling conscious they had invoked Republic Act No. 972. Unfortunately, the
of having been discriminated against (See Explanatory Note court has found no reason to revise their grades. If they are to
to R.A. No. 972), unsuccessful candidates who obtained be admitted to the bar, it must be pursuant to Republic Act No.
averages of a few percentage lower than those admitted to 972 which, if declared valid, should be applied equally to all
the Bar agitated in Congress for, and secured in 1951 the concerned whether they have filed petitions or not. A complete
passage of Senate Bill No. 12 which, among others, reduced list of the petitioners, properly classified, affected by this
the passing general average in bar examinations to 70 per decision, as well as a more detailed account of the history of
cent effective since 1946. The President requested the views Republic Act No. 972, are appended to this decision as
of this court on the bill. Complying with that request, seven Annexes I and II. And to realize more readily the effects of the
members of the court subscribed to and submitted written law, the following statistical data are set forth:
comments adverse thereto, and shortly thereafter the
President vetoed it. Congress did not override the veto. (1) The unsuccessful bar candidates who are to be benefited
Instead, it approved Senate Bill No. 371, embodying by section 1 of Republic Act No. 972 total 1,168, classified as
substantially the provisions of the vetoed bill. Although the follows:
members of this court reiterated their unfavorable views on
the matter, the President allowed the bill to become a law on
June 21, 1953 without his signature. The law, which 1946 (August) 206 121 18
incidentally was enacted in an election year, reads in full as
follows: 1946 (November) 477 228 43

REPUBLIC ACT NO. 972 1947 749 340 0

AN ACT TO FIX THE PASSING MARKS FOR BAR 1948 899 409 11
EXAMINATIONS FROM NINETEEN HUNDRED AND
FORTY-SIX UP TO AND INCLUDING NINETEEN 1949 1,218 532 164
HUNDRED AND FIFTY-FIVE.
1950 1,316 893 26
Be it enacted by the Senate and House of Representatives of
the Philippines in Congress assembled:
1951 2,068 879 196
SECTION 1. Notwithstanding the provisions of section
fourteen, Rule numbered one hundred twenty-seven of the 1952 2,738 1,033 426
Rules of Court, any bar candidate who obtained a general
average of seventy per cent in any bar examinations after July 1953 2,555 968 284
fourth, nineteen hundred and forty-six up to the August
nineteen hundred and fifty-one bar examinations; seventy-one
FINALS CONSTITUTIONAL LAW I ACJUCO NOV 11, 2017 169

TOTAL 12,230 5,421 1,168 the inadequacy of the preparation of students who took up law
soon after the liberation.
Of the total 1,168 candidates, 92 have passed in subsequent
examination, and only 586 have filed either motions for Of the 9,675 candidates who took the examinations from 1946
admission to the bar pursuant to said Republic Act, or mere to 1952, 5,236 passed. And now it is claimed that in addition
motions for reconsideration. 604 candidates be admitted (which in reality total 1,094),
because they suffered from "insufficiency of reading
(2) In addition, some other 10 unsuccessful candidates are to materials" and of "inadequacy of preparation."
be benefited by section 2 of said Republic Act. These
candidates had each taken from two to five different By its declared objective, the law is contrary to public interest
examinations, but failed to obtain a passing average in any of because it qualifies 1,094 law graduates who confessedly had
them. Consolidating, however, their highest grades in different inadequate preparation for the practice of the profession, as
subjects in previous examinations, with their latest marks, they was exactly found by this Tribunal in the aforesaid
would be sufficient to reach the passing average as provided examinations. The public interest demands of legal profession
for by Republic Act No. 972. adequate preparation and efficiency, precisely more so as
legal problem evolved by the times become more difficult. An
(3) The total number of candidates to be benefited by this adequate legal preparation is one of the vital requisites for the
Republic Acts is therefore 1,094, of which only 604 have filed practice of law that should be developed constantly and
petitions. Of these 604 petitioners, 33 who failed in 1946 to maintained firmly. To the legal profession is entrusted the
1951 had individually presented motions for reconsideration protection of property, life, honor and civil liberties. To approve
which were denied, while 125 unsuccessful candidates of officially of those inadequately prepared individuals to
1952, and 56 of 1953, had presented similar motions, which dedicate themselves to such a delicate mission is to create a
are still pending because they could be favorably affected by serious social danger. Moreover, the statement that there was
Republic Act No. 972, — although as has been already stated, an insufficiency of legal reading materials is grossly
this tribunal finds no sufficient reasons to reconsider their exaggerated. There were abundant materials. Decisions of
grades this court alone in mimeographed copies were made available
to the public during those years and private enterprises had
also published them in monthly magazines and annual
UNCONSTITUTIONALITY OF REPUBLIC ACT NO. 972 digests. The Official Gazette had been published
continuously. Books and magazines published abroad have
Having been called upon to enforce a law of far-reaching entered without restriction since 1945. Many law books, some
effects on the practice of the legal profession and the even with revised and enlarged editions have been printed
administration of justice, and because some doubts have locally during those periods. A new set of Philippine Reports
been expressed as to its validity, the court set the hearing of began to be published since 1946, which continued to be
the afore-mentioned petitions for admission on the sole supplemented by the addition of new volumes. Those are
question of whether or not Republic Act No. 972 is facts of public knowledge.
constitutional.
Notwithstanding all these, if the law in question is valid, it has
We have been enlightened in the study of this question by the to be enforced.
brilliant assistance of the members of the bar who have amply
argued, orally an in writing, on the various aspects in which The question is not new in its fundamental aspect or from the
the question may be gleaned. The valuable studies of Messrs. point of view of applicable principles, but the resolution of the
E. Voltaire Garcia, Vicente J. Francisco, Vicente Pelaez and question would have been easier had an identical case of
Buenaventura Evangelista, in favor of the validity of the law, similar background been picked out from the jurisprudence we
and of the U.P. Women's Lawyers' Circle, the Solicitor daily consult. Is there any precedent in the long Anglo-Saxon
General, Messrs. Arturo A. Alafriz, Enrique M. Fernando, legal history, from which has been directly derived the judicial
Vicente Abad Santos, Carlos A. Barrios, Vicente del Rosario, system established here with its lofty ideals by the Congress
Juan de Blancaflor, Mamerto V. Gonzales, and Roman of the United States, and which we have preserved and
Ozaeta against it, aside from the memoranda of counsel for attempted to improve, or in our contemporaneous judicial
petitioners, Messrs. Jose M. Aruego, M.H. de Joya, Miguel R. history of more than half a century? From the citations of those
Cornejo and Antonio Enrile Inton, and of petitioners Cabrera, defending the law, we can not find a case in which the validity
Macasaet and Galema themselves, has greatly helped us in of a similar law had been sustained, while those against its
this task. The legal researchers of the court have exhausted validity cite, among others, the cases of Day (In re Day, 54 NE
almost all Philippine and American jurisprudence on the 646), of Cannon (State vs. Cannon, 240 NW, 441), the opinion
matter. The question has been the object of intense of the Supreme Court of Massachusetts in 1932 (81 ALR
deliberation for a long time by the Tribunal, and finally, after 1061), of Guariña (24 Phil., 37), aside from the opinion of the
the voting, the preparation of the majority opinion was President which is expressed in his vote of the original bill and
assigned to a new member in order to place it as humanly as which the postponement of the contested law respects.
possible above all suspicion of prejudice or partiality.
This law has no precedent in its favor. When similar laws in
Republic Act No. 972 has for its object, according to its author, other countries had been promulgated, the judiciary
to admit to the Bar, those candidates who suffered from immediately declared them without force or effect. It is not
insufficiency of reading materials and inadequate preparation. within our power to offer a precedent to uphold the disputed
Quoting a portion of the Explanatory Note of the proposed bill, law.
its author Honorable Senator Pablo Angeles David stated:
To be exact, we ought to state here that we have examined
The reason for relaxing the standard 75 per cent passing carefully the case that has been cited to us as a favorable
grade is the tremendous handicap which students during the precedent of the law — that of Cooper (22 NY, 81), where the
years immediately after the Japanese occupation has to Court of Appeals of New York revoked the decision of the
overcome such as the insufficiency of reading materials and Supreme court of that State, denying the petition of Cooper to
FINALS CONSTITUTIONAL LAW I ACJUCO NOV 11, 2017 170

be admitted to the practice of law under the provisions of a examination by the court; and as rendering the latter
statute concerning the school of law of Columbia College examination, to which no definite period of preliminary study
promulgated on April 7, 1860, which was declared by the was essential, unnecessary and burdensome.
Court of Appeals to be consistent with the Constitution of the
state of New York. The act was obviously passed with reference to the learning
and ability of the applicant, and for the mere purpose of
It appears that the Constitution of New York at that time substituting the examination by the law committee of the
provided: college for that of the court. It could have had no other object,
and hence no greater scope should be given to its provisions.
They (i.e., the judges) shall not hold any other office of public We cannot suppose that the Legislature designed entirely to
trust. All votes for either of them for any elective office except dispense with the plain and explicit requirements of the
that of the Court of Appeals, given by the Legislature or the Constitution; and the act contains nothing whatever to indicate
people, shall be void. They shall not exercise any power of an intention that the authorities of the college should inquire
appointment to public office. Any male citizen of the age of as to the age, citizenship, etc., of the students before granting
twenty-one years, of good moral character, and who a diploma. The only rational interpretation of which the act
possesses the requisite qualifications of learning and ability, admits is, that it was intended to make the college diploma
shall be entitled to admission to practice in all the courts of this competent evidence as to the legal attainments of the
State. (p. 93). applicant, and nothing else. To this extent alone it operates as
a modification of pre-existing statutes, and it is to be read in
connection with these statutes and with the Constitution itself
According to the Court of Appeals, the object of the in order to determine the present condition of the law on the
constitutional precept is as follows: subject. (p.89)

Attorneys, solicitors, etc., were public officers; the power of xxx xxx xxx
appointing them had previously rested with the judges, and
this was the principal appointing power which they possessed.
The convention was evidently dissatisfied with the manner in The Legislature has not taken from the court its jurisdiction
which this power had been exercised, and with the restrictions over the question of admission, that has simply prescribed
which the judges had imposed upon admission to practice what shall be competent evidence in certain cases upon that
before them. The prohibitory clause in the section quoted was question. (p.93)
aimed directly at this power, and the insertion of the provision"
expecting the admission of attorneys, in this particular section From the foregoing, the complete inapplicability of the case of
of the Constitution, evidently arose from its connection with Cooper with that at bar may be clearly seen. Please note only
the object of this prohibitory clause. There is nothing indicative the following distinctions:
of confidence in the courts or of a disposition to preserve any
portion of their power over this subject, unless the Supreme (1) The law of New York does not require that any candidate
Court is right in the inference it draws from the use of the word of Columbia College who failed in the bar examinations be
`admission' in the action referred to. It is urged that the admitted to the practice of law.
admission spoken of must be by the court; that to admit means
to grant leave, and that the power of granting necessarily
implies the power of refusing, and of course the right of (2) The law of New York according to the very decision of
determining whether the applicant possesses the requisite Cooper, has not taken from the court its jurisdiction over the
qualifications to entitle him to admission. question of admission of attorney at law; in effect, it does not
decree the admission of any lawyer.
These positions may all be conceded, without affecting the
validity of the act. (p. 93.) (3) The Constitution of New York at that time and that of the
Philippines are entirely different on the matter of admission of
the practice of law.
Now, with respect to the law of April 7, 1860, the decision
seems to indicate that it provided that the possession of a
diploma of the school of law of Columbia College conferring In the judicial system from which ours has been evolved, the
the degree of Bachelor of Laws was evidence of the legal admission, suspension, disbarment and reinstatement of
qualifications that the constitution required of applicants for attorneys at law in the practice of the profession and their
admission to the Bar. The decision does not however quote supervision have been disputably a judicial function and
the text of the law, which we cannot find in any public or responsibility. Because of this attribute, its continuous and
accessible private library in the country. zealous possession and exercise by the judicial power have
been demonstrated during more than six centuries, which
certainly "constitutes the most solid of titles." Even considering
In the case of Cooper, supra, to make the law consistent with the power granted to Congress by our Constitution to repeal,
the Constitution of New York, the Court of Appeals said of the alter supplement the rules promulgated by this Court
object of the law: regarding the admission to the practice of law, to our judgment
and proposition that the admission, suspension, disbarment
The motive for passing the act in question is apparent. and reinstatement of the attorneys at law is a legislative
Columbia College being an institution of established function, properly belonging to Congress, is unacceptable.
reputation, and having a law department under the charge of The function requires (1) previously established rules and
able professors, the students in which department were not principles, (2) concrete facts, whether past or present,
only subjected to a formal examination by the law committee affecting determinate individuals. and (3) decision as to
of the institution, but to a certain definite period of study before whether these facts are governed by the rules and principles;
being entitled to a diploma of being graduates, the Legislature in effect, a judicial function of the highest degree. And it
evidently, and no doubt justly, considered this examination, becomes more undisputably judicial, and not legislative, if
together with the preliminary study required by the act, as fully previous judicial resolutions on the petitions of these same
equivalent as a test of legal requirements, to the ordinary individuals are attempted to be revoked or modified.
FINALS CONSTITUTIONAL LAW I ACJUCO NOV 11, 2017 171

We have said that in the judicial system from which ours has Our conclusion may be epitomized as follows: For more than
been derived, the act of admitting, suspending, disbarring and six centuries prior to the adoption of our Constitution, the
reinstating attorneys at law in the practice of the profession is courts of England, concededly subordinate to Parliament
concededly judicial. A comprehensive and conscientious since the Revolution of 1688, had exercise the right of
study of this matter had been undertaken in the case of State determining who should be admitted to the practice of law,
vs. Cannon (1932) 240 NW 441, in which the validity of a which, as was said in Matter of the Sergeant's at Law, 6
legislative enactment providing that Cannon be permitted to Bingham's New Cases 235, "constitutes the most solid of all
practice before the courts was discussed. From the text of this titles." If the courts and judicial power be regarded as an entity,
decision we quote the following paragraphs: the power to determine who should be admitted to practice
law is a constituent element of that entity. It may be difficult to
This statute presents an assertion of legislative power without isolate that element and say with assurance that it is either a
parallel in the history of the English speaking people so far as part of the inherent power of the court, or an essential element
we have been able to ascertain. There has been much of the judicial power exercised by the court, but that it is a
uncertainty as to the extent of the power of the Legislature to power belonging to the judicial entity and made of not only a
prescribe the ultimate qualifications of attorney at law has sovereign institution, but made of it a separate independent,
been expressly committed to the courts, and the act of and coordinate branch of the government. They took this
admission has always been regarded as a judicial function. institution along with the power traditionally exercise to
This act purports to constitute Mr. Cannon an attorney at law, determine who should constitute its attorney at law. There is
and in this respect it stands alone as an assertion of legislative no express provision in the Constitution which indicates an
power. (p. 444) intent that this traditional power of the judicial department
should in any manner be subject to legislative control.
Perhaps the dominant thought of the framers of our
Under the Constitution all legislative power is vested in a constitution was to make the three great departments of
Senate and Assembly. (Section 1, art. 4.) In so far as the government separate and independent of one another. The
prescribing of qualifications for admission to the bar are idea that the Legislature might embarrass the judicial
legislative in character, the Legislature is acting within its department by prescribing inadequate qualifications for
constitutional authority when it sets up and prescribes such attorneys at law is inconsistent with the dominant purpose of
qualifications. (p. 444) making the judicial independent of the legislative department,
and such a purpose should not be inferred in the absence of
But when the Legislature has prescribed those qualifications express constitutional provisions. While the legislature may
which in its judgment will serve the purpose of legitimate legislate with respect to the qualifications of attorneys, but is
legislative solicitude, is the power of the court to impose other incidental merely to its general and unquestioned power to
and further exactions and qualifications foreclosed or protect the public interest. When it does legislate a fixing a
exhausted? (p. 444) standard of qualifications required of attorneys at law in order
that public interests may be protected, such qualifications do
Under our Constitution the judicial and legislative departments not constitute only a minimum standard and limit the class
are distinct, independent, and coordinate branches of the from which the court must make its selection. Such legislative
government. Neither branch enjoys all the powers of qualifications do not constitute the ultimate qualifications
sovereignty which properly belongs to its department. Neither beyond which the court cannot go in fixing additional
department should so act as to embarrass the other in the qualifications deemed necessary by the course of the proper
discharge of its respective functions. That was the scheme administration of judicial functions. There is no legislative
and thought of the people setting upon the form of government power to compel courts to admit to their bars persons deemed
under which we exist. State vs. Hastings, 10 Wis., 525; by them unfit to exercise the prerogatives of an attorney at
Attorney General ex rel. Bashford vs. Barstow, 4 Wis., 567. (p. law. (p. 450)
445)
Furthermore, it is an unlawful attempt to exercise the power of
The judicial department of government is responsible for the appointment. It is quite likely true that the legislature may
plane upon which the administration of justice is maintained. exercise the power of appointment when it is in pursuance of
Its responsibility in this respect is exclusive. By committing a a legislative functions. However, the authorities are well-nigh
portion of the powers of sovereignty to the judicial department unanimous that the power to admit attorneys to the practice of
of our state government, under 42a scheme which it was law is a judicial function. In all of the states, except New Jersey
supposed rendered it immune from embarrassment or (In re Reisch, 83 N.J. Eq. 82, 90 A. 12), so far as our
interference by any other department of government, the investigation reveals, attorneys receive their formal license to
courts cannot escape responsibility fir the manner in which the practice law by their admission as members of the bar of the
powers of sovereignty thus committed to the judicial court so admitting. Cor. Jur. 572; Ex parte Secombre, 19 How.
department are exercised. (p. 445) 9,15 L. Ed. 565; Ex parteGarland, 4 Wall. 333, 18 L. Ed. 366;
Randall vs. Brigham, 7 Wall. 53, 19 L. Ed. 285; Hanson vs.
Grattan, 48 Kan, 843, 115 P. 646, 34 L.R.A. 519; Danforth vs.
The relation at the bar to the courts is a peculiar and intimate Egan, 23 S. D. 43, 119 N.W. 1021, 130 Am. St. Rep. 1030, 20
relationship. The bar is an attache of the courts. The quality of Ann. Cas. 413.
justice dispense by the courts depends in no small degree
upon the integrity of its bar. An unfaithful bar may easily bring
scandal and reproach to the administration of justice and bring The power of admitting an attorney to practice having been
the courts themselves into disrepute. (p.445) perpetually exercised by the courts, it having been so
generally held that the act of the court in admitting an attorney
to practice is the judgment of the court, and an attempt as this
Through all time courts have exercised a direct and severe on the part of the Legislature to confer such right upon any
supervision over their bars, at least in the English speaking one being most exceedingly uncommon, it seems clear that
countries. (p. 445) the licensing of an attorney is and always has been a purely
judicial function, no matter where the power to determine the
After explaining the history of the case, the Court ends thus: qualifications may reside. (p. 451)
FINALS CONSTITUTIONAL LAW I ACJUCO NOV 11, 2017 172

In that same year of 1932, the Supreme Court of mere ministerial power. It is the exercise of judicial power, and
Massachusetts, in answering a consultation of the Senate of has been so held in numerous cases. It was so held by the
that State, 180 NE 725, said: court of appeals of New York in the matter of the application
of Cooper for admission. Re Cooper 22 N. Y. 81. "Attorneys
It is indispensible to the administration of justice and to and Counselors", said that court, "are not only officers of the
interpretation of the laws that there be members of the bar of court, but officers whose duties relate almost exclusively to
sufficient ability, adequate learning and sound moral proceedings of a judicial nature; and hence their appointment
character. This arises from the need of enlightened assistance may, with propriety, be entrusted to the court, and the latter,
to the honest, and restraining authority over the knavish, in performing his duty, may very justly considered as engaged
litigant. It is highly important, also that the public be protected in the exercise of their appropriate judicial functions." (pp. 650-
from incompetent and vicious practitioners, whose opportunity 651).
for doing mischief is wide. It was said by Cardoz, C.L., in
People ex rel. Karlin vs. Culkin, 242 N.Y. 456, 470, 471, 162 We quote from other cases, the following pertinent portions:
N.E. 487, 489, 60 A.L.R. 851: "Membership in the bar is a
privilege burden with conditions." One is admitted to the bar Admission to practice of law is almost without exception
"for something more than private gain." He becomes an conceded everywhere to be the exercise of a judicial function,
"officer of the court", and ,like the court itself, an instrument or and this opinion need not be burdened with citations in this
agency to advance the end of justice. His cooperation with the point. Admission to practice have also been held to be the
court is due "whenever justice would be imperiled if exercise of one of the inherent powers of the court. — Re
cooperation was withheld." Without such attorneys at law the Bruen, 102 Wash. 472, 172 Pac. 906.
judicial department of government would be hampered in the
performance of its duties. That has been the history of
attorneys under the common law, both in this country and Admission to the practice of law is the exercise of a judicial
England. Admission to practice as an attorney at law is almost function, and is an inherent power of the court. — A.C.
without exception conceded to be a judicial function. Petition Brydonjack, vs. State Bar of California, 281 Pac. 1018; See
to that end is filed in courts, as are other proceedings invoking Annotation on Power of Legislature respecting admission to
judicial action. Admission to the bar is accomplish and made bar, 65, A.L. R. 1512.
open and notorious by a decision of the court entered upon its
records. The establishment by the Constitution of the judicial On this matter there is certainly a clear distinction between the
department conferred authority necessary to the exercise of functions of the judicial and legislative departments of the
its powers as a coordinate department of government. It is an government.
inherent power of such a department of government ultimately
to determine the qualifications of those to be admitted to The distinction between the functions of the legislative and the
practice in its courts, for assisting in its work, and to protect judicial departments is that it is the province of the legislature
itself in this respect from the unfit, those lacking in sufficient to establish rules that shall regulate and govern in matters of
learning, and those not possessing good moral character. transactions occurring subsequent to the legislative action,
Chief Justice Taney stated succinctly and with finality in Ex while the judiciary determines rights and obligations with
parte Secombe, 19 How. 9, 13, 15 L. Ed. 565, "It has been reference to transactions that are past or conditions that exist
well settled, by the rules and practice of common-law courts, at the time of the exercise of judicial power, and the distinction
that it rests exclusively with the court to determine who is is a vital one and not subject to alteration or change either by
qualified to become one of its officers, as an attorney and legislative action or by judicial decree.
counselor, and for what cause he ought to be removed."
(p.727)
The judiciary cannot consent that its province shall be invaded
by either of the other departments of the government. — 16
In the case of Day and others who collectively filed a petition C.J.S., Constitutional Law, p. 229.
to secure license to practice the legal profession by virtue of a
law of state (In re Day, 54 NE 646), the court said in part:
If the legislature cannot thus indirectly control the action of the
courts by requiring of them construction of the law according
In the case of Ex parte Garland, 4 Wall, 333, 18 L. Ed. 366, to its own views, it is very plain it cannot do so directly, by
the court, holding the test oath for attorneys to be settling aside their judgments, compelling them to grant new
unconstitutional, explained the nature of the attorney's office trials, ordering the discharge of offenders, or directing what
as follows: "They are officers of the court, admitted as such by particular steps shall be taken in the progress of a judicial
its order, upon evidence of their possessing sufficient legal inquiry. — Cooley's Constitutional Limitations, 192.
learning and fair private character. It has always been the
general practice in this country to obtain this evidence by an
examination of the parties. In this court the fact of the In decreeing the bar candidates who obtained in the bar
admission of such officers in the highest court of the states to examinations of 1946 to 1952, a general average of 70 per
which they, respectively, belong for, three years preceding cent without falling below 50 per cent in any subject, be
their application, is regarded as sufficient evidence of the admitted in mass to the practice of law, the disputed law is not
possession of the requisite legal learning, and the statement a legislation; it is a judgment — a judgment revoking those
of counsel moving their admission sufficient evidence that promulgated by this Court during the aforecited year affecting
their private and professional character is fair. The order of the bar candidates concerned; and although this Court
admission is the judgment of the court that the parties possess certainly can revoke these judgments even now, for justifiable
the requisite qualifications as attorneys and counselors, and reasons, it is no less certain that only this Court, and not the
are entitled to appear as such and conduct causes therein. legislative nor executive department, that may be so. Any
From its entry the parties become officers of the court, and are attempt on the part of any of these departments would be a
responsible to it for professional misconduct. They hold their clear usurpation of its functions, as is the case with the law in
office during good behavior, and can only be deprived of it for question.
misconduct ascertained and declared by the judgment of the
court after opportunity to be heard has been afforded. Ex parte That the Constitution has conferred on Congress the power to
Hoyfron, admission or their exclusion is not the exercise of a repeal, alter or supplement the rule promulgated by this
FINALS CONSTITUTIONAL LAW I ACJUCO NOV 11, 2017 173

Tribunal, concerning the admission to the practice of law, is The case of Guariña (1913) 24 Phil., 37, illustrates our
no valid argument. Section 13, article VIII of the Constitution criterion. Guariña took examination and failed by a few points
provides: to obtain the general average. A recently enacted law
provided that one who had been appointed to the position of
Section 13. The Supreme Court shall have the power to Fiscal may be admitted to the practice of law without a
promulgate rules concerning pleading, practice, and previous examination. The Government appointed Guariña
procedure in all courts, and the admission to the practice of and he discharged the duties of Fiscal in a remote province.
law. Said rules shall be uniform for all courts of the same grade This tribunal refused to give his license without previous
and shall not diminish, increase or modify substantive rights. examinations. The court said:
The existing laws on pleading, practice and procedure are
hereby repealed as statutes, and are declared Rules of Court, Relying upon the provisions of section 2 of Act No. 1597, the
subject to the power of the Supreme Court to alter and modify applicant in this case seeks admission to the bar, without
the same. The Congress shall have the power to repeal, alter, taking the prescribed examination, on the ground that he holds
or supplement the rules concerning pleading, practice, and the office of provincial fiscal for the Province of Batanes.
procedure, and the admission to the practice of law in the
Philippines. — Constitution of the Philippines, Art. VIII, sec. Section 2 of Act No. 1597, enacted February 28, 1907, is as
13. follows:

It will be noted that the Constitution has not conferred on Sec. 2. Paragraph one of section thirteen of Act Numbered
Congress and this Tribunal equal responsibilities concerning One Hundred and ninety, entitled "An Act providing a Code of
the admission to the practice of law. the primary power and Procedure in Civil Actions and Special Proceedings in the
responsibility which the Constitution recognizes continue to Philippine Islands," is hereby amended to read as follows:
reside in this Court. Had Congress found that this Court has
not promulgated any rule on the matter, it would have nothing
over which to exercise the power granted to it. Congress may 1. Those who have been duly licensed under the laws and
repeal, alter and supplement the rules promulgated by this orders of the Islands under the sovereignty of Spain or of the
Court, but the authority and responsibility over the admission, United States and are in good and regular standing as
suspension, disbarment and reinstatement of attorneys at law members of the bar of the Philippine Islands at the time of the
and their supervision remain vested in the Supreme Court. adoption of this code; Provided, That any person who, prior to
The power to repeal, alter and supplement the rules does not the passage of this act, or at any time thereafter, shall have
signify nor permit that Congress substitute or take the place of held, under the authority of the United States, the position of
this Tribunal in the exercise of its primary power on the matter. justice of the Supreme Court, judge of the Court of First
The Constitution does not say nor mean that Congress may Instance, or judge or associate judge of the Court of Land
admit, suspend, disbar or reinstate directly attorneys at law, Registration, of the Philippine Islands, or the position of
or a determinate group of individuals to the practice of law. Its Attorney General, Solicitor General, Assistant Attorney
power is limited to repeal, modify or supplement the existing General, assistant attorney in the office of the Attorney
rules on the matter, if according to its judgment the need for a General, prosecuting attorney for the City of Manila, city
better service of the legal profession requires it. But this power attorney of Manila, assistant city attorney of Manila, provincial
does not relieve this Court of its responsibility to admit, fiscal, attorney for the Moro Province, or assistant attorney for
suspend, disbar and reinstate attorneys at law and supervise the Moro Province, may be licensed to practice law in the
the practice of the legal profession. courts of the Philippine Islands without an examination, upon
motion before the Supreme Court and establishing such fact
to the satisfaction of said court.
Being coordinate and independent branches, the power to
promulgate and enforce rules for the admission to the practice
of law and the concurrent power to repeal, alter and The records of this court disclose that on a former occasion
supplement them may and should be exercised with the this appellant took, and failed to pass the prescribed
respect that each owes to the other, giving careful examination. The report of the examining board, dated March
consideration to the responsibility which the nature of each 23, 1907, shows that he received an average of only 71 per
department requires. These powers have existed together for cent in the various branches of legal learning upon which he
centuries without diminution on each part; the harmonious was examined, thus falling four points short of the required
delimitation being found in that the legislature may and should percentage of 75. We would be delinquent in the performance
examine if the existing rules on the admission to the Bar of our duty to the public and to the bar, if, in the face of this
respond to the demands which public interest requires of a affirmative indication of the deficiency of the applicant in the
Bar endowed with high virtues, culture, training and required qualifications of learning in the law at the time when
responsibility. The legislature may, by means of appeal, he presented his former application for admission to the bar,
amendment or supplemental rules, fill up any deficiency that it we should grant him license to practice law in the courts of
may find, and the judicial power, which has the inherent these Islands, without first satisfying ourselves that despite his
responsibility for a good and efficient administration of justice failure to pass the examination on that occasion, he now
and the supervision of the practice of the legal profession, "possesses the necessary qualifications of learning and
should consider these reforms as the minimum standards for ability."
the elevation of the profession, and see to it that with these
reforms the lofty objective that is desired in the exercise of its But it is contented that under the provisions of the above-cited
traditional duty of admitting, suspending, disbarring and statute the applicant is entitled as of right to be admitted to the
reinstating attorneys at law is realized. They are powers bar without taking the prescribed examination "upon motion
which, exercise within their proper constitutional limits, are not before the Supreme Court" accompanied by satisfactory proof
repugnant, but rather complementary to each other in that he has held and now holds the office of provincial fiscal of
attaining the establishment of a Bar that would respond to the the Province of Batanes. It is urged that having in mind the
increasing and exacting necessities of the administration of object which the legislator apparently sought to attain in
justice. enacting the above-cited amendment to the earlier statute,
and in view of the context generally and especially of the fact
that the amendment was inserted as a proviso in that section
FINALS CONSTITUTIONAL LAW I ACJUCO NOV 11, 2017 174

of the original Act which specifically provides for the admission character, or as other authorities say, merely to fix the
of certain candidates without examination. It is contented that minimum conditions for the license.
this mandatory construction is imperatively required in order
to give effect to the apparent intention of the legislator, and to The law in question, like those in the case of Day and Cannon,
the candidate's claim de jure to have the power exercised. has been found also to suffer from the fatal defect of being a
class legislation, and that if it has intended to make a
And after copying article 9 of Act of July 1, 1902 of the classification, it is arbitrary and unreasonable.
Congress of the United States, articles 2, 16 and 17 of Act No.
136, and articles 13 to 16 of Act 190, the Court continued: In the case of Day, a law enacted on February 21, 1899
required of the Supreme Court, until December 31 of that year,
Manifestly, the jurisdiction thus conferred upon this court by to grant license for the practice of law to those students who
the commission and confirmed to it by the Act of Congress began studying before November 4, 1897, and had studied for
would be limited and restricted, and in a case such as that two years and presented a diploma issued by a school of law,
under consideration wholly destroyed, by giving the word or to those who had studied in a law office and would pass an
"may," as used in the above citation from Act of Congress of examination, or to those who had studied for three years if
July 1, 1902, or of any Act of Congress prescribing, defining they commenced their studies after the aforementioned date.
or limiting the power conferred upon the commission is to that The Supreme Court declared that this law was
extent invalid and void, as transcending its rightful limits and unconstitutional being, among others, a class legislation. The
authority. Court said:

Speaking on the application of the law to those who were This is an application to this court for admission to the bar of
appointed to the positions enumerated, and with particular this state by virtue of diplomas from law schools issued to the
emphasis in the case of Guariña, the Court held: applicants. The act of the general assembly passed in 1899,
under which the application is made, is entitled "An act to
In the various cases wherein applications for the admission to amend section 1 of an act entitled "An act to revise the law in
the bar under the provisions of this statute have been relation to attorneys and counselors," approved March 28,
considered heretofore, we have accepted the fact that such 1884, in force July 1, 1874." The amendment, so far as it
appointments had been made as satisfactory evidence of the appears in the enacting clause, consists in the addition to the
qualifications of the applicant. But in all of those cases we had section of the following: "And every application for a license
reason to believe that the applicants had been practicing who shall comply with the rules of the supreme court in regard
attorneys prior to the date of their appointment. to admission to the bar in force at the time such applicant
commend the study of law, either in a law or office or a law
school or college, shall be granted a license under this act
In the case under consideration, however, it affirmatively notwithstanding any subsequent changes in said rules". — In
appears that the applicant was not and never had been re Day et al, 54 N.Y., p. 646.
practicing attorney in this or any other jurisdiction prior to the
date of his appointment as provincial fiscal, and it further
affirmatively appears that he was deficient in the required . . . After said provision there is a double proviso, one branch
qualifications at the time when he last applied for admission to of which is that up to December 31, 1899, this court shall grant
the bar. a license of admittance to the bar to the holder of every
diploma regularly issued by any law school regularly
organized under the laws of this state, whose regular course
In the light of this affirmative proof of his defieciency on that of law studies is two years, and requiring an attendance by the
occasion, we do not think that his appointment to the office of student of at least 36 weeks in each of such years, and
provincial fiscal is in itself satisfactory proof if his possession showing that the student began the study of law prior to
of the necessary qualifications of learning and ability. We November 4, 1897, and accompanied with the usual proofs of
conclude therefore that this application for license to practice good moral character. The other branch of the proviso is that
in the courts of the Philippines, should be denied. any student who has studied law for two years in a law office,
or part of such time in a law office, "and part in the aforesaid
In view, however, of the fact that when he took the law school," and whose course of study began prior to
examination he fell only four points short of the necessary November 4, 1897, shall be admitted upon a satisfactory
grade to entitle him to a license to practice; and in view also examination by the examining board in the branches now
of the fact that since that time he has held the responsible required by the rules of this court. If the right to admission
office of the governor of the Province of Sorsogon and exists at all, it is by virtue of the proviso, which, it is claimed,
presumably gave evidence of such marked ability in the confers substantial rights and privileges upon the persons
performance of the duties of that office that the Chief named therein, and establishes rules of legislative creation for
Executive, with the consent and approval of the Philippine their admission to the bar. (p. 647.)
Commission, sought to retain him in the Government service
by appointing him to the office of provincial fiscal, we think we Considering the proviso, however, as an enactment, it is
would be justified under the above-cited provisions of Act No. clearly a special legislation, prohibited by the constitution, and
1597 in waiving in his case the ordinary examination invalid as such. If the legislature had any right to admit
prescribed by general rule, provided he offers satisfactory attorneys to practice in the courts and take part in the
evidence of his proficiency in a special examination which will administration of justice, and could prescribe the character of
be given him by a committee of the court upon his application evidence which should be received by the court as conclusive
therefor, without prejudice to his right, if he desires so to do, of the requisite learning and ability of persons to practice law,
to present himself at any of the ordinary examinations it could only be done by a general law, persons or classes of
prescribed by general rule. — (In re Guariña, pp. 48-49.) persons. Const. art 4, section 2. The right to practice law is a
privilege, and a license for that purpose makes the holder an
It is obvious, therefore, that the ultimate power to grant license officer of the court, and confers upon him the right to appear
for the practice of law belongs exclusively to this Court, and for litigants, to argue causes, and to collect fees therefor, and
the law passed by Congress on the matter is of permissive creates certain exemptions, such as from jury services and
FINALS CONSTITUTIONAL LAW I ACJUCO NOV 11, 2017 175

arrest on civil process while attending court. The law United States to follow any lawful calling, business or
conferring such privileges must be general in its operation. No profession he may choose, subject only to such restrictions as
doubt the legislature, in framing an enactment for that are imposed upon all persons of like age, sex, and condition."
purpose, may classify persons so long as the law establishing This right may in many respects be considered as a
classes in general, and has some reasonable relation to the distinguishing feature of our republican institutions. Here all
end sought. There must be some difference which furnishes a vocations are all open to every one on like conditions. All may
reasonable basis for different one, having no just relation to be pursued as sources of livelihood, some requiring years of
the subject of the legislation. Braceville Coal Co. vs. People, study and great learning for their successful prosecution. The
147 Ill. 66, 35 N.E. 62; Ritchie vs. People, 155 Ill. 98, 40 N.E. interest, or, as it is sometimes termed, the "estate" acquired in
454; Railroad Co. vs. Ellis, 165 U.S. 150, 17 Sup. Ct. 255. them — that is, the right to continue their prosecution — is
often of great value to the possessors and cannot be arbitrarily
The length of time a physician has practiced, and the skill taken from them, any more than their real or personal property
acquired by experience, may furnish a basis for classification can be thus taken. It is fundamental under our system of
(Williams vs. People 121 Ill. 48, II N.E. 881); but the place government that all similarly situated and possessing equal
where such physician has resided and practiced his qualifications shall enjoy equal opportunities. Even statutes
profession cannot furnish such basis, and is an arbitrary regulating the practice of medicine, requiring medications to
discrimination, making an enactment based upon it void (State establish the possession on the part of the application of his
vs. Pennyeor, 65 N.E. 113, 18 Atl. 878). Here the legislature proper qualifications before he may be licensed to practice,
undertakes to say what shall serve as a test of fitness for the have been challenged, and courts have seriously considered
profession of the law, and plainly, any classification must have whether the exemption from such examinations of those
some reference to learning, character, or ability to engage in practicing in the state at the time of the enactment of the law
such practice. The proviso is limited, first, to a class of persons rendered such law unconstitutional because of infringement
who began the study of law prior to November 4, 1897. This upon this general principle. State vs. Thomas Call, 121 N.C.
class is subdivided into two classes — First, those presenting 643, 28 S.E. 517; see, also, The State ex rel. Winkler vs.
diplomas issued by any law school of this state before Rosenberg, 101 Wis. 172, 76 N.W. 345; State vs. Whitcom,
December 31, 1899; and, second, those who studied law for 122 Wis. 110, 99 N.W. 468.
the period of two years in a law office, or part of the time in a
law school and part in a law office, who are to be admitted This law singles out Mr. Cannon and assumes to confer upon
upon examination in the subjects specified in the present rules him the right to practice law and to constitute him an officer of
of this court, and as to this latter subdivision there seems to this Court as a mere matter of legislative grace or favor. It is
be no limit of time for making application for admission. As to not material that he had once established his right to practice
both classes, the conditions of the rules are dispensed with, law and that one time he possessed the requisite learning and
and as between the two different conditions and limits of time other qualifications to entitle him to that right. That fact in no
are fixed. No course of study is prescribed for the law school, matter affect the power of the Legislature to select from the
but a diploma granted upon the completion of any sort of great body of the public an individual upon whom it would
course its managers may prescribe is made all-sufficient. Can confer its favors.
there be anything with relation to the qualifications or fitness
of persons to practice law resting upon the mere date of A statute of the state of Minnesota (Laws 1929, c. 424)
November 4, 1897, which will furnish a basis of classification. commanded the Supreme Court to admit to the practice of law
Plainly not. Those who began the study of law November 4th without examination, all who had served in the military or naval
could qualify themselves to practice in two years as well as forces of the United States during the World War and received
those who began on the 3rd. The classes named in the a honorable discharge therefrom and who (were disabled
proviso need spend only two years in study, while those who therein or thereby within the purview of the Act of Congress
commenced the next day must spend three years, although approved June 7th, 1924, known as "World War Veteran's Act,
they would complete two years before the time limit. The one 1924 and whose disability is rated at least ten per cent
who commenced on the 3rd. If possessed of a diploma, is to thereunder at the time of the passage of this Act." This Act
be admitted without examination before December 31, 1899, was held |unconstitutional on the ground that it clearly violated
and without any prescribed course of study, while as to the the quality clauses of the constitution of that state. In re
other the prescribed course must be pursued, and the diploma Application of George W. Humphrey, 178 Minn. 331, 227 N.W.
is utterly useless. Such classification cannot rest upon any 179.
natural reason, or bear any just relation to the subject sought,
and none is suggested. The proviso is for the sole purpose of
bestowing privileges upon certain defined persons. (pp. 647- A good summary of a classification constitutionally acceptable
648.) is explained in 12 Am. Jur. 151-153 as follows:

In the case of Cannon above cited, State vs. Cannon, 240 The general rule is well settled by unanimity of the authorities
N.W. 441, where the legislature attempted by law to reinstate that a classification to be valid must rest upon material
Cannon to the practice of law, the court also held with regards differences between the person included in it and those
to its aspect of being a class legislation: excluded and, furthermore, must be based upon substantial
distinctions. As the rule has sometimes avoided the
constitutional prohibition, must be founded upon pertinent and
But the statute is invalid for another reason. If it be granted real differences, as distinguished from irrelevant and artificial
that the legislature has power to prescribe ultimately and ones. Therefore, any law that is made applicable to one class
definitely the qualifications upon which courts must admit and of citizens only must be based on some substantial difference
license those applying as attorneys at law, that power can not between the situation of that class and other individuals to
be exercised in the manner here attempted. That power must which it does not apply and must rest on some reason on
be exercised through general laws which will apply to all alike which it can be defended. In other words, there must be such
and accord equal opportunity to all. Speaking of the right of a difference between the situation and circumstances of all the
the Legislature to exact qualifications of those desiring to members of the class and the situation and circumstances of
pursue chosen callings, Mr. Justice Field in the case of Dent. all other members of the state in relation to the subjects of the
vs. West Virginia, 129 U.S. 114, 121, 9 S. Ct. 232, 233, 32 L. discriminatory legislation as presents a just and natural cause
Ed. 626, said: "It is undoubtedly the right of every citizen of the
FINALS CONSTITUTIONAL LAW I ACJUCO NOV 11, 2017 176

for the difference made in their liabilities and burdens and in Article 2 of the law in question permits partial passing of
their rights and privileges. A law is not general because it examinations, at indefinite intervals. The grave defect of this
operates on all within a clause unless there is a substantial system is that it does not take into account that the laws and
reason why it is made to operate on that class only, and not jurisprudence are not stationary, and when a candidate finally
generally on all. (12 Am. Jur. pp. 151-153.) receives his certificate, it may happen that the existing laws
and jurisprudence are already different, seriously affecting in
Pursuant to the law in question, those who, without a grade this manner his usefulness. The system that the said law
below 50 per cent in any subject, have obtained a general prescribes was used in the first bar examinations of this
average of 69.5 per cent in the bar examinations in 1946 to country, but was abandoned for this and other disadvantages.
1951, 70.5 per cent in 1952, 71.5 per cent in 1953, and those In this case, however, the fatal defect is that the article is not
will obtain 72.5 per cent in 1954, and 73.5 per cent in 1955, expressed in the title will have temporary effect only from 1946
will be permitted to take and subscribe the corresponding oath to 1955, the text of article 2 establishes a permanent system
of office as members of the Bar, notwithstanding that the rules for an indefinite time. This is contrary to Section 21 (1), article
require a minimum general average of 75 per cent, which has VI of the Constitution, which vitiates and annuls article 2
been invariably followed since 1950. Is there any motive of the completely; and because it is inseparable from article 1, it is
nature indicated by the abovementioned authorities, for this obvious that its nullity affect the entire law.
classification ? If there is none, and none has been given, then
the classification is fatally defective. Laws are unconstitutional on the following grounds: first,
because they are not within the legislative powers of Congress
It was indicated that those who failed in 1944, 1941 or the to enact, or Congress has exceeded its powers; second,
years before, with the general average indicated, were not because they create or establish arbitrary methods or forms
included because the Tribunal has no record of the that infringe constitutional principles; and third, because their
unsuccessful candidates of those years. This fact does not purposes or effects violate the Constitution or its basic
justify the unexplained classification of unsuccessful principles. As has already been seen, the contested law
candidates by years, from 1946-1951, 1952, 1953, 1954, suffers from these fatal defects.
1955. Neither is the exclusion of those who failed before said
years under the same conditions justified. The fact that this Summarizing, we are of the opinion and hereby declare that
Court has no record of examinations prior to 1946 does not Republic Act No. 972 is unconstitutional and therefore, void,
signify that no one concerned may prove by some other and without any force nor effect for the following reasons, to
means his right to an equal consideration. wit:

To defend the disputed law from being declared 1. Because its declared purpose is to admit 810 candidates
unconstitutional on account of its retroactivity, it is argued that who failed in the bar examinations of 1946-1952, and who, it
it is curative, and that in such form it is constitutional. What admits, are certainly inadequately prepared to practice law, as
does Rep. Act 972 intend to cure ? Only from 1946 to 1949 was exactly found by this Court in the aforesaid years. It
were there cases in which the Tribunal permitted admission to decrees the admission to the Bar of these candidates,
the bar of candidates who did not obtain the general average depriving this Tribunal of the opportunity to determine if they
of 75 per cent: in 1946 those who obtained only 72 per cent; are at present already prepared to become members of the
in the 1947 and those who had 69 per cent or more; in 1948, Bar. It obliges the Tribunal to perform something contrary to
70 per cent and in 1949, 74 per cent; and in 1950 to 1953, reason and in an arbitrary manner. This is a manifest
those who obtained 74 per cent, which was considered by the encroachment on the constitutional responsibility of the
Court as equivalent to 75 per cent as prescribed by the Rules, Supreme Court.
by reason of circumstances deemed to be sufficiently
justifiable. These changes in the passing averages during 2. Because it is, in effect, a judgment revoking the resolution
those years were all that could be objected to or criticized. of this Court on the petitions of these 810 candidates, without
Now, it is desired to undo what had been done — cancel the having examined their respective examination papers, and
license that was issued to those who did not obtain the although it is admitted that this Tribunal may reconsider said
prescribed 75 per cent ? Certainly not. The disputed law resolution at any time for justifiable reasons, only this Court
clearly does not propose to do so. Concededly, it approves and no other may revise and alter them. In attempting to do it
what has been done by this Tribunal. What Congress directly Republic Act No. 972 violated the Constitution.
lamented is that the Court did not consider 69.5 per cent
obtained by those candidates who failed in 1946 to 1952 as
sufficient to qualify them to practice law. Hence, it is the lack 3. By the disputed law, Congress has exceeded its legislative
of will or defect of judgment of the Court that is being cured, power to repeal, alter and supplement the rules on admission
and to complete the cure of this infirmity, the effectivity of the to the Bar. Such additional or amendatory rules are, as they
disputed law is being extended up to the years 1953, 1954 ought to be, intended to regulate acts subsequent to its
and 1955, increasing each year the general average by one promulgation and should tend to improve and elevate the
per cent, with the order that said candidates be admitted to practice of law, and this Tribunal shall consider these rules as
the Bar. This purpose, manifest in the said law, is the best minimum norms towards that end in the admission,
proof that what the law attempts to amend and correct are not suspension, disbarment and reinstatement of lawyers to the
the rules promulgated, but the will or judgment of the Court, Bar, inasmuch as a good bar assists immensely in the daily
by means of simply taking its place. This is doing directly what performance of judicial functions and is essential to a worthy
the Tribunal should have done during those years according administration of justice. It is therefore the primary and
to the judgment of Congress. In other words, the power inherent prerogative of the Supreme Court to render the
exercised was not to repeal, alter or supplement the rules, ultimate decision on who may be admitted and may continue
which continue in force. What was done was to stop or in the practice of law according to existing rules.
suspend them. And this power is not included in what the
Constitution has granted to Congress, because it falls within 4. The reason advanced for the pretended classification of
the power to apply the rules. This power corresponds to the candidates, which the law makes, is contrary to facts which
judiciary, to which such duty been confided. are of general knowledge and does not justify the admission
to the Bar of law students inadequately prepared. The
FINALS CONSTITUTIONAL LAW I ACJUCO NOV 11, 2017 177

pretended classification is arbitrary. It is undoubtedly a class Number of candidates whose grades were raised
legislation. 12

5. Article 2 of Republic Act No. 972 is not embraced in the title 73'S 6
of the law, contrary to what the Constitution enjoins, and being
inseparable from the provisions of article 1, the entire law is 72'S 6
void.
Number of candidates who passed 85
6. Lacking in eight votes to declare the nullity of that part of
article 1 referring to the examinations of 1953 to 1955, said
part of article 1, insofar as it concerns the examinations in Number of candidates who failed 121
those years, shall continue in force.
Number of those affected by Republic Act No. 972
RESOLUTION 18

Upon mature deliberation by this Court, after hearing and Percentage of success (per cent) 41.62
availing of the magnificent and impassioned discussion of the
contested law by our Chief Justice at the opening and close of Percentage of failure (per cent) 58.74
the debate among the members of the Court, and after
hearing the judicious observations of two of our beloved Passing grade (per cent) 72
colleagues who since the beginning have announced their
decision not to take part in voting, we, the eight members of
the Court who subscribed to this decision have voted and November, 1946
resolved, and have decided for the Court, and under the
authority of the same: Board of Examiners: The same as that of August, 1946,
except Hon. Jose Teodoro who was substituted by Atty.
1. That (a) the portion of article 1 of Republic Act No. 972 Honesto K. Bausan.
referring to the examinations of 1946 to 1952, and (b) all of
article 2 of said law are unconstitutional and, therefore, void Number of candidates 481
and without force and effect.
Number of candidates whose grades were raised
2. That, for lack of unanimity in the eight Justices, that part of 19
article 1 which refers to the examinations subsequent to the
approval of the law, that is from 1953 to 1955 inclusive, is valid
(72 per cent and above 73 per cent ---
and shall continue to be in force, in conformity with section 10,
article VII of the Constitution.
Minutes of March 31, 1947)
Consequently, (1) all the above-mentioned petitions of the
candidates who failed in the examinations of 1946 to 1952 Number of candidates who passed 249
inclusive are denied, and (2) all candidates who in the
examinations of 1953 obtained a general average of 71.5 per Number of candidates who failed 228
cent or more, without having a grade below 50 per cent in any
subject, are considered as having passed, whether they have
Number of those affected by Republic Act No. 972
filed petitions for admission or not. After this decision has
43
become final, they shall be permitted to take and subscribe
the corresponding oath of office as members of the Bar on the
date or dates that the chief Justice may set. So ordered. Percentage of success (per cent) 52.20

Bengzon, Montemayor, Jugo, Labrador, Pablo, Padilla, and Percentage of failure (per cent) 47.80
Reyes, JJ., concur.
Passing grade
ANNEX I
(By resolution of the Court). (per cent) 72
PETITIONERS UNDER REPUBLIC ACT NO. 972
October, 1947
A resume‚ of pertinent facts concerning the bar examinations
of 1946 to 1953 inclusive follows: Board of Examiners: Hon. Cesar Bengzon, Chairman, Hon.
Guillermo B. Guevara, Atty. Antonio Araneta, Atty. Simon
August, 19461 Cruz, Hon. Sixto de la Costa, Atty. Celso B. Jamora, Hon.
Emilio Peña, Atty. Federico Agrava, Atty. Carlos B. Hilado,
Members.
Board of Examiners: Hon. Pedro Tuason, Chairman, Prof.
Gerardo Florendo, Atty. Bernardino Guerrero, Atty. Joaquin
Ramirez, Atty. Crispin Oben, Hon. Jose Teodoro, Atty. Number of candidates 749
Federico Agrava, Atty. Jose Perez Cardenas, and Hon.
Bienvenido A. Tan, members. Number of candidates whose grades were raised
43
Number of candidates 206
FINALS CONSTITUTIONAL LAW I ACJUCO NOV 11, 2017 178

70.55 per cent with 2 subject below 50 per cent 1 Araneta, Hon. Pastor M. Endencia, Atty. Federico Agrava,
Hon. Mariano H. de Joya, Hon. Felipe Natividad, Atty.
Emeterio Barcelon, Members.
69 per cent 40
Number of candidates 1,218
68 per cent 2
Number of candidates whose grades were raised (74's)
Number of candidates who passed 409 55

Number of candidates who failed 340 Number of candidates who passed 686

Number of those affected by Republic Act No. 972 Number of candidates who failed 532
972
Number of those affected by Republic Act No. 972
Percentage of success (per cent) 54.59 164

Percentage of failure (per cent) 45.41 Percentage of success (per cent) 56.28

Passing grade (per cent) 69 Percentage of failure (per cent) 43.72

(by resolution of the Court). Passing grade (per cent) 74

Note.--In passing the 2 whose grades were 68.95 per cent and (by resolution of the Court).
68.1 per cent respectively, the Court found out that they were
not benefited at all by the bonus of 12 points given by the August, 1950
Examiner in Civil Law.
Board of Examiners: Hon. Fernando Jugo,2 Chairman, Hon.
August, 1948 Guillermo B. Guevara, Atty. Enrique Altavas, Atty. Marcial P.
Lichauco, Atty. Carlos B. Hilado, Atty. J. Antonio Araneta,
Board of Examiners: Hon. Marceliano R. Montemayor, Hon. Enrique V. Filamor, Hon. Francisco A. Delgado, Hon.
Chairman Hon. Luis P. Torres, Hon. Felipe Natividad, Hon. Antonio Horrilleno, Members.
Jose Teodoro, Sr., Atty. Federico Agrava, Atty. Macario
Peralta, Sr., Hon. Jesus G. Barrera, Hon. Rafael Amparo, Atty. Number of candidates 1,316
Alfonso Ponce Enrile, Members.
Number of candidates whose grades were raised
Number of candidates 899 38

Number of candidates whose grades were raised (The grade of 74 was raised to 75 per cent by
64 recommendation and authority

71's 29 of the examiner in Remedial Law, Atty. Francisco Delgado).

70's 35 Number of candidates who passed 432

Number of candidates who passed 490 Number of candidates who failed 894

Number of candidates who failed 409 Number of those affected by Republic Act No. 972
26
Number of those affected by Republic Act No. 972
11 Percentage of success (per cent) 32.14

Percentage of success (per cent) 62.40 Percentage of failure (per cent) 67.86

Percentage of failure (per cent) 37.60 Passing grade (per cent) 75

Passing grade (per cent) 70 August, 1951

(by resolution of the Court). Board of Examiners: Hon. Guillermo F. Pablo, Chairman, Hon.
Pastor M. Endencia, Atty. Enrique Altavas, Hon. Manuel Lim,
August, 1949 Hon. Felipe Natividad, Hon. Vicente Albert, Atty. Arturo Alafriz,
Hon. Enrique V. Filamor, Hon. Alfonso Felix, Members.
Board of Examiners: Hon. Sabino Padilla, Chairman, Hon.
Fernando Jugo, Hon. Enrique Filamor, Atty. Salvador Number of candidates 2,068
FINALS CONSTITUTIONAL LAW I ACJUCO NOV 11, 2017 179

Number of candidates whose grades were raised (74's) Percentage of failure (per cent) 38.96
112
Passing grade (per cent) 75
Number of candidates who passed 1,189
A list of petitioners for admission to the Bar under Republic
Number of candidates who failed 879 Act No. 972, grouped by the years in which they took the bar
examinations, with annotations as to who had presented
Number of those affected by Republic Act No. 972 motions for reconsideration which were denied (MRD), and
196 who filed mere motions for reconsideration without invoking
said law, which are still pending, follows:
Percentage of success (per cent) 57.49
PETITIONER UNDER THE BAR FLUNKERS' LAW
Percentage of failure (per cent) 42.51
Civ. Land Merc. Int. Pol.
Crim. Rem. Leg. Gen. Av.
Passing grade (per cent) 75
MRD- 1. Agunod, Filemon L. 66 71 61
August, 1952 76 80 83 73 75 71.4

Board of Examiners: Hon. Sabino Padilla, Chairman, Hon. MRD- 2. Cunanan, Albino 76 72 74 75
Pastor M. Endencia, Hon. Enrique V. Filamor, Atty. Francisco 70 70 65 72 71.45
Ortigas, Hon. Emilio Peña, Atty. Emilio P. Virata, Hon. Alfonso
Felix, Hon. Felipe Natividad, Atty. Macario Peralta, Sr.,
Members. MRD- 3. Mejia, Flaviano V. 64 64 65 68
83 74 68 80 69.85
Number of candidates 2,738
1948
Number of candidates whose grades were raised (74's)
163 MRD- 4. Orlina, Soledad R. 71 68 66
75 63 75 70 88 69.9
Number of candidates who passed 1,705
MRD- 5. Vivero, Antonio Lu. 75 73 73
65 63 66 65 80 69.95
Number of candidates who failed 1,033
MRD- 6. Gatchalian, Salud 72 66 71 75
Number of those affected by Republic Act No. 972 78 68 65 50 69.65
426
1949
Percentage of success (per cent) 62.27
7. Abaya, Jesus A. 69 79 75 75
Percentage of failure (per cent) 37.73 71 89 55 75 70.8

Passing grade (per cent) 75 MRD- 8. Advincula, David D. 76 80 62


86 81 72 60 65 70.5
August, 1953
9. Agraviador, Alfredo L. 63 85 70
Board of Examiners: Hon. Fernando Jugo, Chairman, Hon. 77 80 81 65 80 71.8
Pastor M. Endencia, Atty. Enrique Altavas, Atty. Francisco
Ortigas, Jr., Hon. Emilio Peña, Atty. Jose S. de la Cruz, Hon. 10. Alacar, Pascual C. 61 63 83
Alfonso Felix, Hon. Felipe Natividad, Hon. Mariano L. de la 79 71 85 65 80 72.05
Rosa, Members.
11. Amog, Pedro M. 75 66 76 78
Number of candidates 2,555 81 74 55 85 72.2

Number of candidates whose grades were raised (74's) 12. Apolinario, Miguel S. 75 84 78
100 78 70 70 60 75 71.95

Number of candidates who passed 1,570 13. Aquino, Maximo G. 82 77 71


77 76 77 60 75 73.15
Number of candidates who failed 986
14. Asinas, Candido D. 75 83 69
Number of those affected by Republic Act No. 972 80 81 83 55 85 72.65
284
15. Baldivino, Jose B. 75 65 72 82
Percentage of success (per cent) 61.04 82 69 60 80 71.95
FINALS CONSTITUTIONAL LAW I ACJUCO NOV 11, 2017 180

16. Balintona, Bernardo 75 80 64 37. Garcia, Freidrich M. 76 80 66


78 74 67 65 70 70 75 72 70 60 75 69.7

17. Banawa, Angel L. 78 70 70 75 38. Garcia, Julian L. 64 77 68 82


81 83 60 60 72.3 89 77 65 75 72.15

18. Bandala, Anacleto A. 66 80 66 39. Garcia, Leon Mo. 77 86 71 80


71 93 72 55 70 69.6 60 82 65 75 71.85

19. Bandon, Alawadin L. 74 79 69 40. Garcia, Pedro V. 76 82 73 81


77 91 73 60 80 73.35 74 83 60 85 73.6

20. Baquero, Benjamin 76 79 64 41. Garcia, Santiago C. 62 91 79


77 85 72 65 75 72.5 75 72 75 65 80 71.8

21. Blanco, Jose 75 75 70 75 42. Genoves, Pedro 75 83 70 78


77 76 60 90 72.5 87 76 55 80 72.7

22. Buenaluz, Victoriano T. 75 71 72 43. Gonzales, Amado P. 75 71 71


78 67 82 60 75 70.85 75 86 75 60 75 72.65

23. Canda, Benjamin S. 75 72 75 44. Guia, Odon R. de 77 76 66 81


82 76 77 65 75 73.55 74 76 60 75 70.9

24. Canon, Guillermo 77 86 67 88 45. Fernandez, Simeon 62 68 71


75 69 70 85 73.9 80 74 90 65 75 70.85

25. Carlos, Estela S. 75 81 81 79 46. Jakosalem, Filoteo 82 83 73


72 73 65 70 73.8 82 61 87 65 70 73.6

26. Cerezo, Gregorio O. 69 76 76 47. Jesus, Felipe D. de 75 83 67


79 71 80 55 80 70.4 79 78 85 60 75 72.45

27. Clarin, Manuel L. 75 82 76 81 48. Jocom, Jacobo M. 77 77 74


73 69 70 75 73.95 77 74 64 55 85 70.65

28. Claudo, Conrado O. 76 62 78 49. Juares, Nicolas 77 84 56 76


77 73 72 60 70 71.4 73 82 60 85 70

29. Condevillamar, Antonio V. 68 65 74 50. Kalalang, Remigio 65 75 74


80 85 75 60 75 71.65 80 70 70 65 85 70.3

MRD- 30. Cornejo, Crisanto R. 72 75 51. Layumas, Vicente L. 67 84 65


69 82 83 79 65 80 75 89 66 60 80 70.3
73.4
52. Leyson, Amancio F. 69 83 75
31. Corona, Olvido D. 68 76 73 81 76 81 75 65 75 73.15
81 72 60 75 71.15
53. Libanan, Marcelino 71 83 61
32. Dizon, Marcial C. 76 86 69 83 77 80 81 65 85 71.75
75 74 65 80 73.1
54. Lim, Jose E. 77 77 72 76
33. Enriquez, Agustin P. 75 77 70 72 64 65 70 71.15
81 81 77 65 80 73.75
55. Lim, Jose F. 70 75 62 83
34. Espiritu, Irineo E. 80 88 69 75 80 71 65 80 70.4
76 77 65 75 73.8
56. Linao, Mariano M. 66 84 76
35. Fernandez, Macario J. 63 82 76 78 80 75 60 75 71.75
75 81 84 65 75 72.95
57. Lopez, Angelo P. 67 81 75 72
36. Gallardo, Amando C. 78 79 67 79 81 55 80 71
77 76 75 60 65 70.95
FINALS CONSTITUTIONAL LAW I ACJUCO NOV 11, 2017 181

58. Lopez, Eliezar M. 77 75 60 75 79. Rodil, Francisco C. 68 69 70


77 85 60 75 70.7 81 76 75 65 75 70.75

59. Lopez, Nicanor S. 72 71 70 78 80. Rodriguez, Mariano I. 80 75 69


77 84 60 75 71.55 80 72 80 65 80 73.35

60. Manoleto, Proceso D. 72 70 65 81. Romero, Crispulo P. 78 75 66


78 81 90 60 80 71.95 77 76 83 65 75 72.85

61. Mancao, Alfredo P. 67 64 71 82. Saez, Porfirio D. 75 75 72 81


83 76 76 65 80 70.95 69 77 60 75 71

62. Manera, Mariano A. 75 78 75 83. Saliguma, Crisogono D. 79 79 74


75 68 79 60 65 71 78 69 65 65 70 71.8

63. Mercado, Arsenio N. 67 64 71 84. Samano, Fortunato A. 75 84 72


83 76 76 65 80 70.95 77 70 82 60 75 71.9

64. Miranda, Benjamin G. 76 81 67 85. Santos, Faustina C. 71 68 68


82 74 77 65 80 72.55 76 75 85 55 75 69.5

65. Manad, Andres B. 77 75 68 86. Santos, Josefina R. 68 69 76


82 69 72 65 75 71.15 71 77 82 65 75 72.3

1948 87. Seludo, Ananias G. 75 80 69


79 77 82 65 75 73.25
66. Orosco, Casimiro P. 72 84 69
81 70 82 65 75 71.9 88. Semilia, Rafael I. 68 85 55 83
89 79 65 80 71.25
67. Padua, Manuel C. 76 76 68
80 79 79 50 75 70.1 89. Telan, Gaudencio 77 79 70 75
70 75 60 75 70.85
68. Palang, Basilio S. 71 75 82 71
55 87 55 75 69.6 90. Tesorero, Leocadio T. 75 71 63
75 82 62 65 63 69.65
69. Palma, Cuadrato 62 75 69 93
80 79 55 80 69.5 91. Torre, Valentin S. de la 85 81 71
76 69 65 55 70 70.4
70. Pañganiban, Jose V. 67 83 61
81 91 74 60 75 70.6 92. Torres, Ariston L. 78 71 72 81
61 84 55 85 70.4
71. Pareja, Felipe 66 71 75 81
67 74 60 70 68.75 93. Veyra, Zosimo C. de 70 75 71
79 65 80 65 80 70.65
72. Patalinjug, Eriberto 73 77 78
73 78 71 55 75 71.25 94. Viado, Jose 67 70 74 75
75 90 55 80 70.7
73. Paulin, Jose C. 66 69 71 77
83 82 65 75 72.1 95. Villacarlos, Delfin A. 73 87 71
82 69 70 75 85 73.85
74. Pido, Serafin C. 72 78 63 80
71 85 70 80 72.05 96. Villamil, Leonor S. 73 81 76
86 86 73 55 85 73.6
75. Pimentel, Luis P. 77 75 76 81
76 68 55 80 71.6 97. Zabala, Amando A. 76 70 67
75 76 76 60 75 70.6
76. Plantilla, Rodrigo C. 72 78 68
89 79 81 65 85 73.55 1950

77. Regalario, Benito B. 72 80 64 MRD-98. Cruz, Filomeno de la 70 71


80 75 81 55 80 69.55 78 81 76 72 64 96
73.4
78. Robis, Casto P. 62 77 74 73
68 80 70 80 70.9
FINALS CONSTITUTIONAL LAW I ACJUCO NOV 11, 2017 182

99. Española, Pablo S. 71 78 55 119. Biason, Sixto F. 73 82 67 65


76 85 69 65 93 70.2 66 72 77 68 71.25

100. Foronda, Clarencio J. 60 78 68 MRD-120. Briñas, Isagani A. 71 69 74


79 84 88 62 93 71.9 70 76 52 79 72 71.95

101. Hechanova, Vicente 59 76 75 121. Buela, Arcadio P. 72 77 61 70


75 69 68 75 96 71.3 71 58 79 71 69.75

MRD-102. Peñalosa, Osias R. 80 78 122. Cabilao, Leonardo S. 73 50 75


61 76 61 77 66 85 75 75 60 71 79 71.25
70.2
123. Cabrera, Ireneo M. 75 66 70
103. Sarmiento, Floro A. 65 86 63 65 72 81 70 79 72.4
82 89 72 60 72 70.15
124. Cacacho, Emilio V.
MRD-104. Torre, Catalino P. 75 85 68
78 69 67 65 69 70.25
125. Calilung, Soledad C. 64 73 73
105. Ungson, Fernando S. 61 87 75 80 73 57 75 59 69.65
70 57 85 83 82 72.8
MRD-126. Calimlim, Jose B. 64 73 73
1951 80 73 57 75 59 69.65

106. Abasolo, Romulo 77 70 64 65 127. Calimlim, Pedro B. 66 82 69


76 70 76 64 71.7 60 69 52 83 75 70

107. Adeva, Daniel G. 75 59 74 65 128. Camello, Sotero H. 70 77 63


69 51 78 67 70.4 65 75 66 84 64 71.55

108. Aguilar, Vicente Z. 73 63 68 129. Campos, Juan A. 71 88 70 75


75 70 69 75 75 71.25 64 69 71 62 70.15

109. Amodia, Juan T. 75 76 66 75 130. Castillo, Antonio del 78 78 70


76 60 77 76 72.35 60 79 67 69 76 72.65

MRD-110. Añosa, Pablo S. 76 78 63 MRD-131. Castillo, Dominador Ad. 75 61


75 74 61 75 79 71.6 72 75 74 71 67 66
71.1
111. Antiola, Anastacio R. 68 76 75
70 71 70 81 66 73.05 MRD-132. Castro, Jesus B. 72 86 72
75 65 75 76 71 72.85
112. Aquino, S. Rey A. 70 71 71 60
74 62 76 77 71.1 133. Casuga, Bienvenido B. 75 72 72
70 69 61 75 60 70.95
113. Atienza, Manuel G. 71 78 68
80 86 51 82 75 73.85 134. Cabangbang, Santiago B. 77 67 61
80 73 59 83 76 72.2
114. Avanceña, Alfonso 71 71 65
75 70 72 78 80 71.8 135. Cruz, Federico S. 69 74 75 75
68 65 76 70 71.65
MRD-115. Balacuit, Camilo N. 75 73
75 70 72 65 75 76 136. Dacanay, Eufemio P. 70 73 62
73.25 75 72 69 85 71 72.05

116. Barinaga, Jeremias L. 68 69 73 137. Deysolong, Felisberto 66 62 72


70 74 50 80 79 71.2 75 70 62 83 62 70.85

MRD-117. Barrientos, Ambrosio D. 76 60 MRD-138. Dimaano, Jr., Jose N. 78 79


67 55 74 63 77 62 63 75 73 75 81 59
70.25 73.5

MRD-118. Benitez, Tomas P. 67 75 139. Espinosa, Domingo L. 78 63 58


75 60 73 72 75 78 70 70 67 87 63 71.6
72.2
FINALS CONSTITUTIONAL LAW I ACJUCO NOV 11, 2017 183

MRD-140. Farol, Evencia C. 80 78 66 MRD-161. Magsino, Encarnacion 77 66


75 81 72 62 73 72.25 70 70 76 71 75 61
72.75
141. Felix, Conrado S. 71 71 75 65
70 58 75 69 70.75 MRD-162. Maligaya, Demetrio M. 70 61
75 65 75 50 91 51
142. Fernan, Pablo L. 67 88 66 85 72.3
73 68 78 75 72.35
163. Manio, Gregorio 67 67 69 80
143. Gandioco, Salvador G. 64 58 66 71 67 75 75 70.65
65 76 70 89 75 72.1
164. Puzon, Eduardo S. 72 82 60
144. Gastardo, Crispin B. 70 69 68 60 69 70 68 72 62.05
75 78 66 86 72 73.9
MRD-165. Marcial, Meynardo R. 66 75
145. Genson, Angelo B. 75 57 73 74 70 75 67 81 75
65 67 54 78 56 69.55 73.15

146. Guiani, Guinald M. 68 60 75 166. Martin, Benjamin S. 68 72 63


65 74 67 75 77 71.5 75 69 63 84 62 70.1

147. Guina, Graciano P. 66 69 67 MRD-167. Monterroyo, Catalina S. 70 80


60 78 52 83 61 69.6 75 80 76 66 82 51
73.95
MRD-148. Homeres, Praxedes P. 74 74
75 75 71 69 75 71 MRD-168. Montero, Leodegario C. 73 67
73.35 66 80 81 65 81 75
73.75
149. Ibarra, Venancio M. 60 75 74
70 74 70 80 75 71.9 169. Monzon, Candido T. 70 72 74
75 67 70 77 69 72.05
150. Imperial, Monico L. 72 78 75
75 72 56 82 77 73.7 170. Natividad, Alberto M. 73 79 68
65 73 69 75 79 72.2
MRD-151. Ibasco, Jr., Emiliano M. 71 70
63 85 71 60 85 53 MRD-171. Navallo, Capistrano C. 70 72
70.85 68 85 81 66 71 74
72.1
152. Inandan, Fortunato C. 77 77 67
53 73 75 79 57 72.5 172. Nisce, Camilo Z. 66 66 75 65
79 68 85 62 73.5
153. Jimenez, Florencio C. 75 70 70
75 72 61 75 78 72.05 MRD-173. Ocampo, Antonio F. de 75 81
76 65 74 67 75 69
73.75
154. Kintanar, Woodrow M. 70 83 72
65 76 73 75 69 72.95
174. Olaviar, Jose O. 72 70 69 55
66 70 77 75 70.5
155. Languido, Cesar V. 63 71 63
85 70 61 85 79 70.55
MRD-175. Perez, Cesario Z. 75 76 66
80 72 63 82 69 72.95
156. Lavilles, Cesar L. 61 89 75 55
73 63 75 78 70.55
176. Pogado, Causin O. 70 66 65
70 75 64 75 70 69.95
157. Llenos, Francisco U. 64 70 65
60 72 65 92 75 71.75
177. Ramos-Balmori, Manuela 75 73 62
65 78 59 75 66 70.2
158. Leon, Marcelo D. de 63 73 60
85 75 75 90 70 72.75
178. Recinto, Ireneo I. 73 76 68 75
74 68 80 53 72.3
159. Llanto, Priscilla 72 68 60 65
76 67 84 68 71.35
MRD-179. Redor, Francisco K. 62 77
73 75 69 64 76 69
160. Machachor, Oscar 68 59 78 70
70 67 57 75 75 70.15
FINALS CONSTITUTIONAL LAW I ACJUCO NOV 11, 2017 184

MRD-180. Regis, Deogracias A. 76 74 201. Villagonzalo, Job R. 78 67 74


68 65 65 65 88 75 65 72 51 69 71 70.25
73.35
202. Villarama, Jr., Pedro 75 74 75
181. Rigor, Estelita C. 67 78 61 80 55 75 66 67 75 71.45
71 77 79 65 70.9
1952
MRD-182. Rimorin-Gordo, Estela 70 72
62 60 88 66 67 79 203. Abacon, Pablo 75 72 78 81
70.15 78 72 64 55 72.7

183. Rosario, Prisco del 70 64 70 MRP-204. Abad, Agapito 73 76 73


70 72 73 85 57 72.65 85 75 63 62 75 70.95

184. Rosario, Vicente D. del 75 91 65 MRP-205. Abella, Ludovico B. 70 81


75 68 68 79 62 72.2 76 81 70 66 77 58
72.7
185. Saavedra, Felipe 73 80 63 75
76 73 68 62 70.35 MRP-206. Abellera, Geronimo F. 75 79
79 87 76 51 63 70
186. Salazar, Alfredo N. 66 72 73 71.7
75 67 68 77 69 70.85
MRP-207. Abenojar, Agapito N. 71 72
187. Salem, Romulo R. 77 81 72 78 84 70 75 69 70
65 73 60 76 75 73 72.9

188. Foz, Julita A. 75 72 75 75 208. Alandy, Doroteo R. 64 83 93


65 70 76 64 72.5 91 68 59 60 60 71.2

189. Santa Ana, Candido T. 77 69 65 209. Alano, Fabian T. 70 83 61 83


75 81 75 70 75 73 72 87 72 70 71.9

190. Santos, Aquilino 72 66 69 65 MRP-210. Alcantara, Pablo V. 71 79


68 70 81 71 71.7 80 81 73 70 72 62
73.65
191. Santos, Valeriano V. 76 72 75
75 68 62 76 79 73.1 211. Arcangel, Agustin Ag. 75 85 71
73 76 65 68 65 71.85
192. Suico, Samuel 73 79 72 75
71 59 84 65 73.3 212. Acosta, Dionisio N. 75 81 78
87 56 65 77 70 72.8
193. Suson, Teodorico 74 68 66 80
66 59 79 67 70.35 MRP-213. Abinguna, Agapito C. 66 85
80 84 75 58 76 75
194. Tado, Florentino P. 64 76 67 73.65
65 76 72 76 53 69.7
214. Adove, Nehemias C. 76 86 78
195. Tapayan, Domingo A. 69 72 69 77 66 78 69 62 73.55
70 76 73 82 79 73.75
215. Adrias, Inocencio C. 75 83 61
MRD-196. Tiausas, Miguel V. 67 60 88 76 67 79 75 73.4
71 75 79 67 84 60
72.7 216. Aglugub, Andres R. 75 83 73
88 72 62 72 62 72.65
197. Torres, Carlos P. 68 71 71 70
70 63 82 71 71.6 217. Andrada, Mariano L. 76 85 66
87 63 77 75 77 73.
198. Tria, Hipolito 69 72 75 60
69 54 78 66 70.05 MRP-218. Almeda, Serafin V. 72 72
75 81 61 67 73 65
199. Velasco, Avelino A. 65 72 75 70.75
75 71 67 78 76 72.1
219. Almonte-Peralta, Felicidad 73 71 72
200. Villa, Francisco C. 65 80 73 91 75 67 65 53 70.7
75 68 79 65 75 70.2
FINALS CONSTITUTIONAL LAW I ACJUCO NOV 11, 2017 185

MRP-220. Amodia, Juan T. 75 79 68 MRP-240. Bejec, Conceso D. 79 80


85 62 64 75 78 71.4 73 82 63 77 75 50
73.15
MRP-221. Antonio, Felino A. 71 76 81
83 79 52 72 70 73.3 MRP-241. Beltran, Gervasio M. 72 75
81 73 75 57 75 80
MRP-222. Antonio, Jose S. 75 92 90 73.95
68 65 64 68 60 73.75
MRP-242. Benaojan, Robustiano O. 74 84
223. Añonuevo, Ramos B. 71 87 78 77 84 75 63 68 62
81 64 63 74 76 72.7 72.85

224. Aquino, S. Rey A. 67 77 57 78 MRP-243. Beriña, Roger C. 70 80 79


69 70 69 80 67.7 79 68 72 64 78 71.85

225. Arteche, Filomeno D. 78 83 50 MRP-244. Bihis, Marcelo M. 75 86 65


89 76 77 70 70 70.8 92 64 64 84 75 73.45

MRP-226. Arribas, Isaac M. 75 78 70 MRP-245. Binaoro, Vicente M. 73 69


81 73 70 67 78 72.2 78 83 73 59 70 82
72.75
MRP-227. Azucena, Ceferino D. 72 67
78 89 72 67 77 65 MRP-246. Bobila, Rosalio B. 76 86 76
73.95 83 68 59 71 78 73.05

228. Atienza, Ricardo 72 87 70 79 247. Buenafe, Avelina R. 78 80 75


66 55 75 75 70.85 75 70 55 72 80 72.75

229. Balacuit, Camilo N. 75 78 89 248. Bueno, Anastacio F. 73 78 71


75 70 54 66 75 73.3 78 71 67 71 60 71.15

MRP-230. Baclig, Cayetano S. 77 84 249. Borres, Maximino L. 67 85 62


83 80 69 70 61 65 91 72 63 76 80 70.9
73
MRP-250. Cabegin, Cesar V. 72 71
231. Balcita, Oscar C. 75 77 79 90 76 75 74 70 71 60
64 60 67 50 70.65 72.2

232. Barilea, Dominador Z. 71 67 82 MRP-251. Cabello, Melecio F. 72 78


77 64 61 65 80 70.5 78 89 58 70 67 71
70.5
MRP-233. Banta, Jose Y. 75 80 77
81 75 63 71 75 73.95 MRP-252. Cabrera, Irineo M. 79 88
53 91 71 85 75 76
73.3
MRP-234. Barrientos, Ambrosio D. 76 70
67 80 67 65 70 81
70.7 253. Cabreros, Paulino N. 71 79 83
84 60 62 71 50 70.85
235. Batucan, Jose M. 66 76 78 88
62 76 67 78 71.2 254. Calayag, Florentino R. 69 79 66
88 69 75 68 76 70.6
236. Bautista, Atilano C. 70 82 84
85 58 61 71 62 71.25 MRP-255. Calzada, Cesar de la 76 72
80 67 62 71 66 62
70.85
237. Bautista, Celso J. 71 68 63 87
80 67 80 70 72.75
256. Canabal, Isabel 70 82 81 77
78 51 75 75 73.7
238. Belderon, Jose 76 81 76 92
70 66 67 62 72.65
MRP-257. Cabugao, Pablo N. 76 87
69 80 58 64 78 75
MRP-239. Belo, Victor B. 76 77 64 71.8
73 75 71 76 76 72.85
258. Calañgi, Mateo C. 73 93 71
87 70 66 69 62 71.8
FINALS CONSTITUTIONAL LAW I ACJUCO NOV 11, 2017 186

259. Canda, Benjamin S. 72 71 77 MRP-279. Cornejo, Crisanto R. 68 87


90 62 75 66 82 71.95 78 86 79 50 80 60
73.7
260. Cantoria, Eulogio 71 80 71 89
70 55 72 75 71 MRP-280. Cruz, Raymundo 75 81 79
85 72 57 68 75 72.95
261. Capacio, Jr., Conrado 67 78 71
90 65 75 72 60 70.65 MRP-281. Cunanan, Jose C. 78 92
63 83 76 72 68 65
262. Capitulo, Alejandro P. 75 70 53 72.4
87 78 63 76 91 71.2
282. Cunanan, Salvador F. 70 82 64
MRP-263. Calupitan, Jr., Alfredo 75 93 92 67 75 73 76 71.45
81 76 64 75 68 56
73.15 283. Cimafranca, Agustin B. 71 76 76
80 70 71 75 71 73.35
MRP-264. Caluya, Arsenio V. 75 86
70 87 77 52 77 82 284. Crisol, Getulio R. 70 91 78 85
73.9 68 55 71 50 70.8

MRP-265. Campanilla, Mariano B. 80 75 MRP-285. Dusi, Felicisimo R. 76 82


78 77 73 71 63 76 69 82 66 62 80 71
73.65 72.85

MRP-266. Campos, Juan A. 66 85 83 MRP-286. Datu, Alfredo J. 70 75 72


84 67 61 80 57 73.25 86 80 55 68 79 71.5

267. Cardoso, Angelita G. 78 71 73 287. Dacuma, Luis B. 71 67 87 83


76 79 56 69 60 71.8 71 50 65 70 71.25

268. Cartagena, Herminio R. 71 72 65 MRP-288. Degamo, Pedro R. 73 80


89 64 73 80 70 71.65 82 74 80 67 67 57
73.65
MRP-269. Castro, Daniel T. 65 75 77
76 85 60 75 69 73.15 289. Delgado, Vicente N. 70 84 82
84 77 52 73 50 72.65
270. Cauntay, Gaudencio V. 70 78 72
73 77 69 64 80 71.2 MRP-290. Diolazo, Ernesto A. 75 83
86 73 54 54 75 75
271. Castro, Pedro L. de 70 68 69 72.25
87 76 75 72 70 73.35
291. Dionisio, Jr., Guillermo 73 84 64
272. Cerio, Juan A. 75 82 75 86 89 71 78 75 66 72.8
60 54 76 75 71.75
MRP-292. Dichoso, Alberto M. 71 77
273. Colorado, Alfonso R. 68 75 80 71 81 69 75 80 70
74 77 66 67 80 72.6 73.65

274. Chavez, Doroteo M. 73 65 79 MRP-293. Dipasupil, Claudio R. 70 76


84 73 69 66 84 73.1 82 73 79 70 72 56
73.9
275. Chavez, Honorato A. 77 76 79
86 74 53 71 75 73.65 MRP-294. Delgado, Abner 75 84 63
67 64 60 70 72 68.35
MRP-276. Cobangbang, Orlando B. 69 81
74 82 76 61 78 80 MRP-295. Domingo, Dominador T. 70 69
73.85 81 82 68 63 71 75
72.2
277. Cortez, Armando R. 78 60 88
86 60 66 69 64 73.1 296. Ducusin, Agapito B. 70 78 53
88 75 77 62 76 68.05
278. Crisostomo, Jesus L. 76 87 74
76 62 55 76 66 71.45 MRP-297. Duque, Antonio S. 75 77
78 86 76 72 64 75
73.9
FINALS CONSTITUTIONAL LAW I ACJUCO NOV 11, 2017 187

298. Duque, Castulo 75 80 73 83 MRP-317. Fuente, Jose S. de la 76 88


66 67 65 66 70.65 72 74 60 71 79 79
73.55
299. Ebbah, Percival B. 70 80 85
76 66 63 76 75 73.95 318. Fohmantes, Nazario S. 72 79 71
77 68 61 76 60 70.9
300. Edisa, Sulpicio 65 77 75 89
75 62 75 65 72 MRP-319. Fuggan, Lorenzo B. 76 81
74 69 71 71 73 60
301. Edradan, Rosa C. 70 75 84 84 72.85
71 59 69 86 73.4
320. Gabuya, Jesus S. 70 83 82 83
MRP-302. Enage, Jacinto N. 66 70 88 70 63 75 65 73.75
93 72 67 65 75 73.2
321. Galang, Victor N. 69 83 84 76
MRP-303. Encarnacion, Alfonso B. 75 86 70 57 71 60 71.95
73 81 63 77 69 75
72.65 322. Gaerlan, Manuel L. 73 87 77
90 67 61 72 75 73.15
304. Encarnacion, Cesar 65 78 58
68 66 64 75 78 67.1 323. Galem, Nestor R. 72 79 86 78
60 61 75 70 73.05
305. Estoista, Agustin A. 78 76 74
86 58 67 70 76 71.7 324. Gallardo, Jose Pe B. 75 88 75
75 63 70 70 65 71.85
MRP-306. Fabros, Jose B. 66 75 80
82 80 71 67 70 73.05 MRP-325. Gallos, Cirilo B. 70 78 84
91 80 51 65 70 72.85
MRP-307. Fajardo, Balbino P. 77 69
82 83 65 60 75 75 326. Galindo, Eulalio D. 70 89 87
73.9 65 78 71 62 62 73.4

308. Fajardo, Genaro P. 70 79 77 327. Galman, Patrocinio G. 72 72 80


79 79 50 73 75 72.5 85 71 56 70 53 71.15

309. Evangelista, Felicidad P. 75 75 72 328. Gamalinda, Carlos S. 76 79 81


87 63 63 77 70 72.15 86 67 63 69 55 72.55

310. Familara, Raymundo Z. 68 75 87 329. Gamboa, Antonio G. 71 67 70


83 64 65 68 65 71.85 72 76 60 75 68 70.95

311. Fariñas, Dionisio 70 78 89 66 330. Gannod, Jose A. 69 80 75 81


65 75 70 50 72.75 68 62 73 68 71.25

312. Favila, Hilario B. 71 84 74 70 MRP-331. Garcia, Matias N. 67 78 74


75 67 73 59 72.2 90 79 59 76 65 72.8

MRP-313. Feliciano, Alberto I. 71 69 MRP-332. Ganete, Carmelo 75 87 77


70 85 69 81 72 70 82 74 57 68 81 73.3
72.25
333. Gilbang, Gaudioso R. 75 67 80
MRP-314. Fernando, Lope F. 73 77 82 67 57 64 70 70.5
86 79 70 76 64 50
73 334. Gofredo, Claro C. 68 78 72 86
78 52 70 76 70.9
MRP-315. Flores, Dionisio S. 78 72
77 83 67 60 68 73 335. Gomez, Jose S. 71 76 71 81
72.05 76 63 69 62 70.85

MRP-316. Fortich, Benjamin B. 70 82 MRP-336. Gosiaoco, Lorenzo V. 68 93


70 70 78 65 64 75 85 78 64 69 70 54
70.35 72.35
FINALS CONSTITUTIONAL LAW I ACJUCO NOV 11, 2017 188

MRP-337. Gonzales, Rafael C. 77 75 MRP-356. Jose, Nestor L. 78 61 64


71 89 55 70 70 60 73 68 76 64 80 69.7
70.05
357. La Q, Jose M. 75 71 75 72
MRP-338. Gracia, Eulalia L. de 66 68 70 67 81 59 73.5
90 84 77 59 69 65
73.3 358. Leon, Brigido C. de 67 75 78
91 78 51 72 80 72.55
339. Grageda, Jose M. A. 70 85 72
67 70 60 73 73 70.75 359. Leones, Constante B. 68 81 79
84 73 60 77 60 73
340. Guzman, Juan de 75 86 69 84
64 79 75 76 73.6 360. Liboro, Horacio T. 72 69 80 87
73 62 70 61 72.4
MRP-341. Guzman, Mateo de 76 79
79 73 72 69 68 80 361. Llanera, Cesar L. 77 81 80 78
73.9 64 59 75 63 73

342. Guzman, Salvador B. 71 61 74 362. Lomontod, Jose P. 75 76 69


72 61 66 78 75 70.75 70 73 76 74 75 73.2

343. Guzman, Salvador T. de 75 84 64 363. Luna, Lucito 70 75 69 83


81 74 61 78 58 71.75 59 53 74 75 68.4

344. Habelito, Geronimo E. 71 76 71 MRP-364. Luz, Lauro L. 76 90 78


87 73 60 67 55 69.65 88 64 58 75 77 73.95

345. Hedriana, Naterno G. 75 68 84 MRP-365. Macasaet, Tomas S. 73 81


76 66 58 76 60 72.9 72 83 66 75 72 70
72.5
346. Hernandez, Quintin B. 67 75 72
81 72 72 66 76 70.6 366. Magbiray, Godofredo V. 80 67 84
76 70 62 65 68 73.05
1952
367. Majarais, Rodolfo P. 70 62 64
347. Homeres, Agustin R. 73 84 65 82 88 75 71 79 72.85
86 70 77 63 76 70.7
MRP-368. Makabenta, Eduardo 75 90
348. Ines, Leonilo F. 65 88 71 88 77 83 59 71 72 78
77 73 61 70 70.55 73.3

349. Jamer, Alipio S. 68 75 83 89 MRP-369. Malapit, Justiniano S. 74 83


80 61 65 50 72 74 89 58 60 72 76
71.1
MRP-350. Ibasco, Jr., Emiliano M. 75 65
68 85 76 70 83 54 370. Maloles, Iluminado M. 70 87 73
73.8 76 77 50 76 76 72.3

MRP-351. Jardinico, Jr., Emilio 73 86 371. Maniquis, Daniel R. 75 80 73


72 78 82 67 67 64 91 69 71 65 70 72.1
72.8
372. Maraña, Arsenio 65 79 60 72
MRP-352. Jaen, Justiniano F. 76 75 73 51 75 86 67.9
78 84 71 66 70 77
73.85 373. Marasigan, Napoleon 75 71 83
75 69 62 69 70 72.75
353. Jaring, Antonio S. 72 77 79 70
72 57 71 50 70.75 MRP-374. Marco, Jaime P. 75 67 74
76 64 75 75 57 71.9
MRP-354. Javier, Aquilino M. 75 84
79 78 77 61 66 66 MRP-375. Martir, Osmundo P. 70 86
73.05 76 78 72 71 75 53
72.95
355. Jomuad, Francisco 75 75 72
88 78 58 76 43 72.4
FINALS CONSTITUTIONAL LAW I ACJUCO NOV 11, 2017 189

MRP-376. Masancay, Amando E. 73 87 394. Nono, Pacifico G. 67 77 78 67


75 77 72 50 78 80 75 59 71 76 71.35
73.2
MRP-395. Nuval, Manuel R. 78 72 67
MRP-377. Mati-ong, Ignacio T. 62 87 90 72 68 78 67 73.65
72 79 73 76 69 77
71.3 396. Ocampo, Augusto 75 90 77
72 69 55 65 67 60.7
378. Mara, Guillermo L. 70 78 78
89 75 67 66 65 72.35 397. Oliveros, Amado A. 72 75 68
72 84 50 75 79 71.9
MRP-379. Mercado, Felipe A. 73 77
82 82 78 52 69 85 398. Opiña, Jr., Pedro 76 77 74 67
73.9 73 66 68 70 71.85

MRP-380. Miculob, Eugenio P. 70 82 MRP-399. Olaviar, Jose O. 70 62 85


73 86 77 52 79 65 81 74 50 68 79 71.8
72.8
MRP-400. Olandesca, Per O. 70 91
381. Mison, Rafael M. Jr., 79 78 73 76 87 72 66 70 79
75 71 68 69 53 71.95 73.45

MRP-382. Monponbanua, Antonio D. 79 79 401. Orden, Apolonio J. 72 65 84


68 88 64 78 69 83 86 66 50 72 68 71.45
73.1
402. Ortiz, Melencio T. 71 75 78 81
MRP-383. Montero, Leodegario C. 72 89 66 67 70 78 72.1
69 89 70 68 70 75
72.15
MRP-403. Pablo, Fedelino S. 72 64
76 86 72 61 76 75
384. Morada, Servillano S. 75 76 67 72.95
71 65 66 75 76 70.9
404. Pacifico, Vicente V. 76 79 69
385. Mocorro, Generoso 78 84 78 80 76 52 72 80 71.95
84 60 73 68 70 73
MRP-405. Paderna, Perfecto D. 75 69
MRP-386. Mosquera, Estanislao L. 75 78 72 75 78 58 75 70
75 85 72 55 77 66 72.6
73.15
406. Padlan, Crispin M. 71 66 76
387. Motus, Rodentor P. 80 78 70 79 68 67 74 66 71.65
94 72 75 70 57 73.75
407. Padilla, Jose C. 70 65 67 82
388. Macario, Pedro R. 70 67 74 78 75 78 75 73.3
86 78 63 72 66 72.15
408. Padilla, Jr., Estanislao E. 71 88 78
MRP-389. Nadela, Geredion T. 72 64 86 59 75 78 50 72.95
64 81 73 50 75 75
69.15
MRP-409. Palma, Bartolome 67 81 80
82 71 75 69 75 73.25
MRP-390. Nazareno, Romeo P. 67 70
71 76 76 79 75 57
72.05 MRP-410. Papa, Angel A. 75 72 85
85 77 59 63 71 73.45
391. Nieto, Benedicto S. 69 79 77
77 72 62 76 76 72.9 MRP-411. Parayno, Mario V. 71 88
74 89 69 66 76 73
73.65
MRP-392. Noguera, Raymundo 71 86
81 80 73 56 72 70
73.15 412. Pariña, Santos L. 70 87 85 77
64 67 63 76 71.85
MRP-393. Nodado, Domiciano R. 70 70
69 73 57 37 64 72 MRP-413. Pasion, Anastacio 63 80
63.6 68 81 82 79 76 58
72.55
FINALS CONSTITUTIONAL LAW I ACJUCO NOV 11, 2017 190

414. Pastrana, Rizal R. 69 76 71 MRP-434. Rayos, Victor S. 75 86 79


76 68 63 77 83 71.65 91 71 67 67 70 73.9

MRP-415. Paulin, Jose O. 70 66 80 435. Revilla, Mariano S. 75 78 81


87 75 50 65 80 70.9 90 70 54 69 81 73.35

MRP-416. Pelaez, Jr., Vicente C. 79 87 436. Reyes, Abdon L. 72 64 81 78


73 83 69 71 68 65 76 73 69 53 72.85
73.2
437. Reyes, Domingo B. 72 87 78
417. Peña, Jesus 75 75 75 62 83 72 75 62 70 72.7
75 70 60 66 70.4
438. Reyes, Francisco M. 75 85 84
418. Perez, Toribio R. 71 64 81 92 68 75 71 68 50 73.9
69 58 67 70 71.25
439. Reyes, Lozano M. 80 57 78
419. Pestaño, Melquiades 77 81 74 79 78 65 64 79 73.35
87 59 68 76 75 73.2
MRP-440. Reyes, Oscar R. 75 75 82
MRP-420. Pido, Serafin C. 77 81 72 82 76 64 68 60 73.65
82 69 71 60 75 71.15
441. Rigonan, Cesar V. 71 85 65
421. Pinlac, Filemon 67 76 74 86 86 75 70 76 70 72.7
65 79 65 72 70.55
442. Rivera, Honorio 71 56 70 90
422. Poblete, Celso B. 72 79 82 76 71 65 75 71 71.2
66 64 74 50 72.15
MRP-443. Rivero, Buenaventura A. 72 88
MRP-423. Piza, Luz 68 70 75 72 94 68 73 66 80
87 74 67 64 75 70.8 72.6

424. Puzon, Eduardo S. 72 80 81 MRP-444. Robles, Enrique 75 77 75


69 72 53 67 70 71.05 77 82 64 69 70 73.7

425. Quetulio, Josefina D. 75 90 60 445. Rodriguez, Orestes Arellano 76 75


93 64 78 76 83 72.9 76 63 69 77 65 78
72.25
MRP-426. Quipanes, Melchor V. 69 88
79 82 65 62 71 66 446. Roldan, Jose V. 67 80 79 83
71.55 73 71 75 70 73.9

MRP-427. Quietson, Bayani R. 73 75 447. Rosario, Adelaida R. del 80 75 65


76 77 70 81 71 53 70 68 72 80 70 73.15
72.85
448. Rosario, Restituto F. del 75 75 79
428. Racho, Macario D. 68 75 81 90 68 65 66 63 72.1
82 78 53 66 54 70.55
MRP-449. Sabelino, Conrado S. 71 81
429. Ramirez, Sabas P. 71 80 73 69 75 77 71 75 70
87 62 62 75 80 71.65 72.95

MRP-430. Raffiñan, Jose A. 80 83 79 450. San Juan, Damaso 77 86 72


79 62 72 68 65 73.25 89 59 76 65 72 71.6

MRP-431. Ramos, Patricio S. 75 87 451. Sañiel, Felix L. 72 93 76 80


76 75 72 72 61 75 67 75 66 62 72.1
72.25
452. Samaniego, Jesus B. 75 80 76
MRP-432. Ramos-Balmori, Manuela 78 84 72 60 67 68 70 70.6
76 90 48 75 80 65
73.45 MRP-453. Sandoval, Emmanuel M. 75 83
70 83 77 67 77 60
MRP-433. Raro, Celso 75 81 76 73.95
67 75 77 55 77 71.4
FINALS CONSTITUTIONAL LAW I ACJUCO NOV 11, 2017 191

MRP-454. Sanidad, Emmanuel Q. 71 75 MRP-473. Tolentino, Jesus C. 75 89


81 90 62 64 76 68 63 84 85 73 73 50
72.95 73.4

455. Santiago, Jr., Cristobal 75 76 84 474. Torrijas, Alfredo A. 77 66 67


93 63 65 59 70 71.8 83 68 75 71 63 71.3

456. Santillan, Juanito Ll. 76 89 83 MRP-475. Tobias, Artemio M. 69 58


83 63 58 65 52 71.25 74 81 71 55 65 57
67.55
MRP-457. Santos, Rodolfo C. 75 75
78 82 73 76 66 70 MRP-476. Trillana, Jr., Apolonio 76 86
73.7 76 86 70 68 75 50
73.8
MRP-458. Santos, Ruperto M. 67 54
69 76 63 64 71 60 MRP-477. Trinidad, Manuel O. 66 91
66.75 83 75 63 66 67 65
70.8
MRP-459. Santos, Aquilino C. 72 71
73 79 73 79 71 85 478. Trinidad, Pedro O. 66 78 78
73.8 85 78 51 64 75 70.8

MRP-460. Santos, Rufino A. 75 81 79 MRP-479. Udarbe, Flavio J. 80 82 77


85 74 72 66 54 73.3 82 67 56 68 75 72.6

461. Suanding, Bantas 75 67 67 92 480. Umali, Osmundo C. 68 75 81


79 59 76 76 73.1 80 71 69 68 60 71.7

MRP-462. Sulit, Feliz M. 76 79 76 481. Umayam, Juanito C. 77 75 87


78 72 75 68 67 73.5 85 56 56 66 60 71

463. Songco, Felicisimo G. 70 68 82 MRP-482. Usita, Gelacio U. 75 72 75


84 60 69 76 65 73.35 74 73 76 71 70 73.55

464. Soriano, Aniceto S. 64 79 77 483. Valino, Francisco M. 72 81 80


80 80 53 70 65 70.7 84 62 78 71 75 73.7

465. Suarez, Pablo D. 73 85 70 87 484. Varela, Dominador M. 67 75 81


76 70 64 70 71.9 86 72 57 81 70 73.85

MRP-466. Sybico, Jesus L. 79 70 70 485. Vega, Macairog L. de 78 62 79


72 75 75 72 60 73.05 87 70 70 71 65 73.8

467. Tabaque, Benjamin R. 69 68 77 MRP-486. Velasco, Emmanuel D. 71 80


79 74 68 72 60 71.85 74 85 60 66 76 76
71.85
MRP-468. Tan Kiang, Clarita 81 79
72 80 62 75 73 80 487. Velez, Maria E. 73 70 89 80
73.95 56 50 72 67 71.05

MRP-469. Tando, Amado T. 71 82 78 MRP-488. Venal, Artemio V. 78 91 58


83 71 61 71 60 72 67 76 55 75 73 73.65

470. Tasico, Severo E. 71 69 75 89 489. Venus, Conrado B. 69 81 74


70 75 67 63 71.65 85 62 66 72 77 77.05

471. Tiburcio, Ismael P. 73 82 72 MRP-490. Verzosa, Federico B. 75 79


93 76 57 68 54 71.15 72 88 76 68 74 59
73.7
MRP-472. Tiongson, Federico T. 70 70
76 84 77 75 75 50 MRP-491. Villafuerte, Eduardo V. 75 83
73.45 70 76 64 64 75 65
71.2
FINALS CONSTITUTIONAL LAW I ACJUCO NOV 11, 2017 192

MRP-492. Villanueva, Cecilio C. 75 85 1953 57 74 68 68 76 52


79 88 66 77 67 70 71 76 66.7
73.95
3. Blanco, Jose B.
493. Villar, Custodio R. 73 69 70
88 76 66 69 50 70.75
MRD-1949 75 75 70 75 77
MRP-494. Villaseñor, Leonidas F. 80 85 76 60 90 72.15
67 77 62 75 76 73
73.15 1951 64 71 58 65 68 70
75 71 66.95
495. Viterbo, Jose H. 80 77 65 93
70 65 65 65 70.65 4. Condeno, Mateo

496. Yaranon, Pedro 70 77 76 85


72 50 75 75 71.85 1950 71 80 62 75 75 81
55 92 69.3
MRP-497. Yasay, Mariano R. 75 75
72 76 63 77 70 60 1951 70 60 61 65 77 64
71.1 67 81 67.85

MRP-498. Ygay, Venancio M. 73 80 5. Ducusin, Agapito B.


83 84 62 59 72 77
72.65
MRD-1949 69 70 76 73 76
499. Yulo, Jr., Teodoro 73 82 78 75 71 55 60 68.65
60 81 75 75 73.95
1950 60 71 55 67 67 75
500. Zamora, Alberto 70 65 76 79 56 89 68.1
62 77 69 82 71.3
6. Garcia, Manuel N.
501. Rigonan, Felipe C. 70 79 69
89 76 62 71 64 71.2
MRD-1949 60 70 82 79 70
A list of those who petitioned for the consolidation of their 69 60 80 69.25
grades in subjects passed in previous examinations, showing
the years in which they took the examinations together with
their grades and averages, and those who had filed motions 1950 57 65 51 69 54 85
for reconsideration which were denied, indicated by the initials 56 84 60.3
MRD, follows:
7. Luna, Lucito A.
PETITIONERS UNDER REPUBLIC ACT NO. 72

Civ. Land Merc. Int. Pol. 1946 63 53 69 76 75 76


Crim. Rem. Leg. Gen. Av. 57 69 66.55

1. Amao, Sulpicio M. 1952 70 75 69 83 59 53


74 75 68.4

1946 68 67 76 76 73 73 8. Maraña, Arsenio s.


49 50 66.5

1950 59 80 67 77 62 80 1949 72 68 68 75 75 72
71 57 67.4 60 75 69.35

2. Baldo, Olegario Ga. 1952 65 79 60 72 73 51


75 86 67.9

1951 65 76 58 55 59 63 9. Montano, Manuel M.


75 72 64.9

1952 65 68 75 84 72 59 1951 61 60 58 60 70 63
73 57 69.75 75 64 64.8
FINALS CONSTITUTIONAL LAW I ACJUCO NOV 11, 2017 193

1952 70 77 65 79 66 52 MRD-1949 67 56 69 75 72
70 50 66.4 77 60 75 68

1953 78 64 66 68 81 50 1951 70 59 55 60 68 57
71 78 70.65 78 67 65.8

10. Peña, Jesus S. 16. Santos, Constantino

1950 25 75 45 75 45 52 1952 62 76 54 82 72 77
46 71 46.2 66 65 66.65

1951 70 77 65 79 66 52 1953 73 71 70 65 78 64
70 50 66.4 65 78 70.4

1952 75 75 75 62 75 70 17. Santos, Salvador H.


60 66 70.4

11. Placido, Sr., Isidro 1951 60 64 55 70 68 52


70 75 62.85

1950 68 78 70 75 69 70 1952 75 64 70 81 76 55
58 69 67.75 61 75 69.1

1951 65 62 75 60 73 57 1953 70 71 79 65 72 54
75 71 66.8 66 80 70

12. Rementizo, Filemon S. 18. Sevilla, Macario C.

1949 65 75 72 75 60 75 MRD-1948 50 64 76 66 66
55 85 66.65 69 60 52 63.1

1951 68 57 48 60 91 66 MRD-1949 47 66 78 64 71
55 75 64.05 86 65 85 68

1952 68 53 68 67 58 56 1950 35 65 40 75 63 57
75 64 65.7 27 49 45

13. Amao, Sulpicio M. MRD-1951 68 59 72 55 69


65 75 75 69.3

1952 67 80 51 69 69 77 1953 70 73 74 70 81 56
73 53 66.35 69 71 71.05

1953 65 67 78 74 75 62 Finally, with regards to the examinations of 1953, while some


69 80 70.9 candidates--85 in all--presented motions for reconsideration
of their grades, others invoked the provisions of Republic Act
14. Rodulfa, Juan T. No. 972. A list of those candidates separating those who filed
mere motions for reconsideration (56) from those who invoked
the aforesaid Republic act, is as follows:
1951 67 60 70 65 68 56
75 66 67.75 1953 PETITIONERS FOR RECONSIDERATION

1952 70 71 67 78 67 75 Civ. Land Merc. Int. Pol.


71 70 70.1 Crim. Rem. Leg. Gen. Av.

15. Sanchez, Juan J. 1. Acenas, Calixto R. 73 70 68


62 82 51 67 77 73.45

1948 39 69 82 75 76 72 2. Alcantara, Pedro N. 67 70 75


55 50 63.5 85 87 54 71 80 72.8

3. Alejandro, Exequiel 67 72 71
75 80 76 75 77 73.4
FINALS CONSTITUTIONAL LAW I ACJUCO NOV 11, 2017 194

4. Andres, Gregorio M. 70 73 86 25. Ilejay, Abraham I. 77 70 76 77


58 79 50 71 78 72.7 81 62 70 68 73.7

5. Arnaiz, Antonio E. 66 80 76 26. Leon, Benjamin La. De 66 66 75


58 79 68 77 81 73.4 70 77 55 71 82 70.35

6. Asis, Floriano U. de 66 78 75 27. Lugtu, Felipe L. 62 70 78 65


81 77 55 73 69 71.25 78 56 69 81 69.9

7. Bacaiso, Celestino M. 71 65 76 28. Lukman, Abdul-Hamid 76 64 67


68 76 50 75 70 70.95 69 73 59 73 75 70.45

8. Bala, Florencio F. 64 82 47 70 29. Maloles, Jr., Benjamin G. 77 76 68


82 58 75 82 67 68 71 51 75 78 70.85

9. Baldo, Olegario A. 57 74 68 30. Maloles, Julius G. 77 71 60 71


68 76 52 71 76 66.7 79 62 68 72 69.75

10. Barrios, Benjamin O. 65 71 76 31. Mandi, Santiago P. 65 76 70


75 80 62 83 73 73.95 61 79 68 75 72 71.1

11. Buhay, Eduardo L. 73 76 71 32. Margete, Rufino C. 70 76 66


91 76 61 74 78 73.35 75 85 73 71 75 72.75

12. Burgos, Dominador C. 72 80 89 33. Melocoton, Nestorio B. 70 81 73


61 66 37 69 68 70.05 78 83 52 72 75 72.35

13. Cariño, Eldo J. 79 81 60 75 34. Molina, Manuel C. 75 78 70


74 74 76 74 73 61 75 63 66 85 70.95

14. Casar, Dimapuro 67 73 84 79 35. Muñoz, Mariano A. 75 80 86


77 61 71 74 73.35 67 74 57 68 76 73.75

15. Castañeda, Gregorio 70 73 80 36. Navarro, Buenaventura M. 80 75 65


71 75 70 73 78 73.95 75 83 55 73 79 73

16. Estrellado, Benjamin R. 67 79 64 37. Nodado, Domiciano R. 60 67 67


73 82 62 71 74 70.2 50 70 50 56 75 61.7

17. Fabunan, Edilberto C. 70 72 68 38. Papas, Sisenando B. 65 62 71


69 77 60 76 74 71.1 61 70 56 66 67 66

18. Feril, Domingo B. 75 71 84 65 39. Pagulayan-Sy, Fernando 63 75 71


70 60 65 70 71.6 62 83 67 70 72 70.4

19. Fernandez, Alejandro G. 65 75 87 40. Padula, Benjamin C. 70 77 54


80 81 63 61 80 72.8 62 74 78 75 68 69.05

20. Gapus, Rosita S. (Miss) 76 80 86 41. Pasno, Enrique M. 78 72 66


77 64 74 66 69 73.9 54 71 58 72 78 69.85

21. Garcia, Rafael B. 70 86 70 75 42. Peña, Jr., Narciso 70 95 81 78


73 63 73 75 71.65 67 66 67 73 72.55

22. Gracia, Miguel L. de 73 68 75 43. Peralta, Rodolfo P. 70 70 52


59 80 51 72 71 71 81 68 63 59 69 63.7

23. Gungon, Armando G. 68 76 76 44. Pigar, Leopoldo R. 76 75 78


84 77 57 77 83 73.6 61 72 72 71 79 73.75

24. Gutierrez, Antonio S. 68 77 66 45. Publico, Paciano L. 68 69 76


70 72 59 71 74 69.1 76 70 59 74 67 70.6
FINALS CONSTITUTIONAL LAW I ACJUCO NOV 11, 2017 195

46. Radaza, Leovigildo 75 78 76 9. Fernandez, Alejandro Q. 65 75 87


61 77 50 71 86 72.2 80 81 63 61 80 72.8

47. Ramos, Bernardo M. 64 62 75 10. Fernandez, Luis N. 70 75 77


93 81 52 66 80 70.1 75 78 67 72 73 73.35

48. Rabaino, Andres D. 68 72 75 11. Figueroa, Alfredo A. 70 75 87


73 78 55 69 76 70.65 78 75 50 68 68 72.3

49. Ravanera, Oscar N. 70 77 80 12. Formilleza, Pedro 65 75 89 68


71 82 62 69 78 73.6 83 51 70 75 73.25

50. Renovilla, Jose M. 65 75 80 13. Garcia, Manuel M. 69 68 83


68 79 52 62 78 69.5 83 73 62 62 70 71

51. Sabaot, Solomon B. 69 73 80 14. Grospe, Vicente E. 68 75 78


69 82 69 69 79 73.85 66 79 61 69 82 71.6

52. Sumaway, Ricardo S. 66 76 69 15. Galema, Nestor R. (1952) 72 79 86


76 74 56 72 68 69.1 78 60 61 75 70 73.05

53. Torrefiel, Sofronio O. 70 77 74 16. Jacobo, Rafael F. 76 76 75 74


75 73 50 68 72 69.55 76 50 72 76 72.3

54. Vera, Federico V. de 60 61 47 17. Macalindong, Reinerio L. 67 77 79


77 69 50 67 77 60.9 79 74 72 68 77 72.75

55. Viray, Venancio Bustos 65 67 67 18. Mangubat, Antonio M. 70 70 78


52 73 64 71 65 67.15 61 80 74 62 70 71.45

56. Ylaya, Angela P. (Miss) 63 70 56 19. Montano, Manuel M. 78 64 66


75 68 54 70 77 64.5 68 81 50 71 78 70.65

PETITIONERS UNDER REPUBLIC ACT NO. 972 20. Plomantes, Marcos 73 67 74


58 68 70 76 71 71.6
Civ. Land Merc. Int. Pol.
Crim. Rem. Leg. Gen. Av. 21. Ramos, Eugenio R. 70 80 76
67 72 69 72 79 72.6
1. Ala, Narciso 70 71 73 59
73 74 81 77 73.5 22. Reyes, Juan R. 71 73 77 76
81 59 72 74 73.2
2. Alcantara, Pedro N. 67 70 75
85 87 54 71 80 72.8 23. Reyes, Santiago R. 65 78 83
60 76 75 70 70 72.9
3. Arellano, Antonio L. 74 66 73
60 78 63 78 72 72.9 24. Rivera, Eulogio J. 65 67 78 74
75 62 69 80 70.9
4. Buhay, Eduardo L. 73 76 71
91 76 61 74 78 73.35 25. Santos, Constantino P. 73 71 70
65 78 64 65 78 70.4
5. Calautit, Celestino R. 71 78 84
75 75 61 68 72 73.2 26. Santos, Salvador H. 70 71 79
65 72 54 66 80 70
6. Casuncad, Sulvio P. 61 73 82
69 81 68 71 84 73.05 27. Sevilla, Macario C. 70 73 74
70 81 56 69 71 71.05
7. Enriquez, Pelagio y Concepcion 84 69
76 75 82 50 58 79 28. Villavicencio, Jose A. 78 75 70
72.05 67 69 77 64 77 73.2

8. Estonina, Severino 80 74 64 29. Viray, Ruperto G. 76 73 76 73


89 81 56 68 82 72.4 80 58 68 83 73.25
FINALS CONSTITUTIONAL LAW I ACJUCO NOV 11, 2017 196

There are the unsuccessful candidates totaling 604 The next amendment is of section 14 of Rule 127. One part of
directly affected by this resolution. Adding 490 this amendment provides that if a bar candidate obtains 70 per
candidates who have not presented any petition, they cent or higher in any subject, although failing to pass the
reach a total of 1,094. examination, he need not be examined in said subject in his
next examination. This is a sort of passing the Bar
The Enactment of Republic Act No. 972 Examination on the installment plan, one or two or three
subjects at a time. The trouble with this proposed system is
that although it makes it easier and more convenient for the
As will be observed from Annex I, this Court reduced to 72 per candidate because he may in an examination prepare himself
cent the passing general average in the bar examination of on only one or two subjects so as to insure passing them, by
august and November of 1946; 69 per cent in 1947; 70 per the time that he has passed the last required subjects, which
cent in 1948; 74 per cent in 1949; maintaining the prescribed may be several years away from the time that he reviewed
75 per cent since 1950, but raising to 75 per cent those who and passed the firs subjects, he shall have forgotten the
obtained 74 per cent since 1950. This caused the introduction principles and theories contained in those subjects and
in 1951, in the Senate of the Philippines of Bill No. 12 which remembers only those of the one or two subjects that he had
was intended to amend Sections 5, 9, 12, 14 and 16 of Rule last reviewed and passed. This is highly possible because
127 of the Rules of Court, concerning the admission of there is nothing in the law which requires a candidate to
attorneys-at-law to the practice of the profession. The continue taking the Bar examinations every year in
amendments embrace many interesting matters, but those succession. The only condition imposed is that a candidate,
referring to sections 14 and 16 immediately concern us. The on this plan, must pass the examination in no more that three
proposed amendment is as follows: installments; but there is no limitation as to the time or number
of years intervening between each examination taken. This
SEC. 14. Passing average. — In order that a candidate would defeat the object and the requirements of the law and
may be deemed to have passed the examinations the Court in admitting persons to the practice of law. When a
successfully, he must have obtained a general average of person is so admitted, it is to be presumed and presupposed
70 per cent without falling below 50 per cent in any that he possesses the knowledge and proficiency in the law
subject. In determining the average, the foregoing subjects and the knowledge of all law subjects required in bar
shall be given the following relative weights: Civil Law, 20 per examinations, so as presently to be able to practice the legal
cent; Land Registration and Mortgages, 5 per cent; profession and adequately render the legal service required
Mercantile Law, 15 per cent; Criminal Law, 10 per cent; by prospective clients. But this would not hold true of the
Political Law, 10 per cent; International Law, 5 per cent; candidates who may have obtained a passing grade on any
Remedial Law, 20 per cent; Legal Ethics and Practical five subjects eight years ago, another three subjects one year
Exercises, 5 per cent; Social Legislation, 5 per cent; later, and the last two subjects the present year. We believe
Taxation, 5 per cent. Unsuccessful candidates shall not that the present system of requiring a candidate to obtain a
be required to take another examination in any subject in passing general average with no grade in any subject below
which they have obtained a rating of 70 per cent or higher 50 per cent is more desirable and satisfactory. It requires one
and such rating shall be taken into account in to be all around, and prepared in all required legal subjects at
determining their general average in any subsequent the time of admission to the practice of law.
examinations: Provided, however, That if the candidate
fails to get a general average of 70 per cent in his third xxx xxx xxx
examination, he shall lose the benefit of having already
passed some subjects and shall be required to the
examination in all the subjects. We now come to the last amendment, that of section 16 of
Rule 127. This amendment provides that any application who
has obtained a general average of 70 per cent in all subjects
SEC. 16. Admission and oath of successful applicants. — without failing below 50 per cent in any subject in any
Any applicant who has obtained a general average of 70 examination held after the 4th day of July, 1946, shall be
per cent in all subjects without falling below 50 per cent allowed to take and subscribe the corresponding oath of
in any examination held after the 4th day of July, 1946, or office. In other words, Bar candidates who obtained not less
who has been otherwise found to be entitled to admission than 70 per cent in any examination since the year 1946
to the bar, shall be allowed to take and subscribe before without failing below 50 per cent in any subject, despite their
the Supreme Court the corresponding oath of office. non-admission to the Bar by the Supreme Court because they
(Arts. 4 and 5, 8, No. 12). failed to obtain a passing general average in any of those
years, will be admitted to the Bar. This provision is not only
With the bill was an Explanatory Note, the portion pertinent to prospective but retroactive in its effects.
the matter before us being:
We have already stated in our comment on the next preceding
It seems to be unfair that unsuccessful candidates at bar amendment that we are not exactly in favor of reducing the
examinations should be compelled to repeat even those passing general average from 75 per cent to 70 per cent to
subjects which they have previously passed. This is not govern even in the future. As to the validity of making such
the case in any other government examination. The Rules of reduction retroactive, we have serious legal doubts. We
Court have therefore been amended in this measure to give a should not lose sight of the fact that after every bar
candidate due credit for any subject which he has previously examinations, the Supreme Court passes the corresponding
passed with a rating of 75 per cent or higher." resolution not only admitting to the Bar those who have
obtained a passing general average grade, but also rejecting
Senate Bill No. 12 having been approved by Congress on May and denying the petitions for reconsideration of those who
3, 1951, the President requested the comments of this have failed. The present amendment would have the effect of
Tribunal before acting on the same. The comment was signed repudiating, reversing and revoking the Supreme Court's
by seven Justices while three chose to refrain from making resolution denying and rejecting the petitions of those who
any and one took no part. With regards to the matter that may have obtained an average of 70 per cent or more but less
interests us, the Court said: than the general passing average fixed for that year. It is clear
that this question involves legal implications, and this phase
FINALS CONSTITUTIONAL LAW I ACJUCO NOV 11, 2017 197

of the amendment if finally enacted into law might have to go Instead Bill No. 371 was presented in the Senate. It reads as
thru a legal test. As one member of the Court remarked during follows:
the discussion, when a court renders a decision or promulgate
a resolution or order on the basis of and in accordance with a AN ACT TO FIX THE PASSING MARKS FOR BAR
certain law or rule then in force, the subsequent amendment EXAMINATIONS FROM 1946 UP TO AND INCLUDING 1953
or even repeal of said law or rule may not affect the final
decision, order, or resolution already promulgated, in the
sense of revoking or rendering it void and of no effect. Be it enacted by the Senate and House of Representatives of
the Philippines in Congress assembled:
Another aspect of this question to be considered is the fact
that members of the bar are officers of the courts, including SECTION 1. Notwithstanding the provisions of section 14,
the Supreme Court. When a Bar candidate is admitted to the Rule 127 of the Rules of Court, any bar candidate who
Bar, the Supreme Court impliedly regards him as a person fit, obtained a general average of 70 per cent in any bar
competent and qualified to be its officer. Conversely, when it examinations after July 4, 1946 up to the August 1951 Bar
refused and denied admission to the Bar to a candidate who examinations; 71 per cent in the 1952 bar examinations; 72
in any year since 1946 may have obtained a general average per cent in the 1953 bar examinations; 73 per cent in the 1954
of 70 per cent but less than that required for that year in order bar examinations; 74 per cent in 1955 bar examinations
to pass, the Supreme Court equally and impliedly considered without a candidate obtaining a grade below 50 per cent in any
and declared that he was not prepared, ready, competent and subject, shall be allowed to take and subscribe the
qualified to be its officer. The present amendment giving corresponding oath of office as member of the Philippine Bar;
retroactivity to the reduction of the passing general average Provided, however, That 75 per cent passing general average
runs counter to all these acts and resolutions of the Supreme shall be restored in all succeeding examinations; and
Court and practically and in effect says that a candidate not Provided, finally, That for the purpose of this Act, any exact
accepted, and even rejected by the Court to be its officer one-half or more of a fraction, shall be considered as one and
because he was unprepared, undeserving and unqualified, included as part of the next whole number.
nevertheless and in spite of all, must be admitted and allowed
by this Court to serve as its officer. We repeat, that this is SEC. 2. Any bar candidate who obtained a grade of 75 per
another important aspect of the question to be carefully and cent in any subject in any bar examination after July 4, 1945
seriously considered. shall be deemed to have passed in such subject or subjects
and such grade or grades shall be included in computing the
The President vetoed the bill on June 16, 1951, stating the passing general average that said candidate may obtain in
following: any subsequent examinations that he may take.

I am fully in accord with the avowed objection of the bill, SEC. 3. This bill shall take effect upon its approval.
namely, to elevate the standard of the legal profession and
maintain it on a high level. This is not achieved, however, by With the following explanatory note:
admitting to practice precisely a special class who have failed
in the bar examination, Moreover, the bill contains provisions This is a revised Bar bill to meet the objections of the President
to which I find serious fundamental objections. and to afford another opportunity to those who feel themselves
discriminated by the Supreme Court from 1946 to 1951 when
Section 5 provides that any applicant who has obtained a those who would otherwise have passed the bar examination
general average of 70 per cent in all subjects without failing but were arbitrarily not so considered by altering its previous
below 50 per cent in any subject in any examination held after decisions of the passing mark. The Supreme Court has been
the 4th day of July, 1946, shall be allowed to take and altering the passing mark from 69 in 1947 to 74 in 1951. In
subscribed the corresponding oath of office. This provision order to cure the apparent arbitrary fixing of passing grades
constitutes class legislation, benefiting as it does specifically and to give satisfaction to all parties concerned, it is proposed
one group of persons, namely, the unsuccessful candidates in in this bill a gradual increase in the general averages for
the 1946, 1947, 1948, 1949 and 1950 bar examinations. passing the bar examinations as follows; For 1946 to 1951 bar
examinations, 70 per cent; for 1952 bar examination, 71 per
The same provision undertakes to revoke or set aside final cent; for 1953 bar examination, 72 per cent; for 1954 bar
resolutions of the Supreme Court made in accordance with examination, 73 percent; and for 1955 bar examination, 74 per
the law then in force. It should be noted that after every bar cent. Thus in 1956 the passing mark will be restored with the
examination the Supreme Court passes the corresponding condition that the candidate shall not obtain in any subject a
resolution not only admitting to the Bar those who have grade of below 50 per cent. The reason for relaxing the
obtained a passing general average but also rejecting and standard 75 per cent passing grade, is the tremendous
denying the petitions for reconsideration of those who have handicap which students during the years immediately after
failed. The provision under consideration would have the the Japanese occupation has to overcome such as the
effect of revoking the Supreme Court's resolution denying and insufficiency of reading materials and the inadequacy of the
rejecting the petitions of those who may have failed to obtain preparation of students who took up law soon after the
the passing average fixed for that year. Said provision also liberation. It is believed that by 1956 the preparation of our
sets a bad precedent in that the Government would be morally students as well as the available reading materials will be
obliged to grant a similar privilege to those who have failed in under normal conditions, if not improved from those years
the examinations for admission to other professions such as preceding the last world war.
medicine, engineering, architecture and certified public
accountancy. In this will we eliminated altogether the idea of having our
Supreme Court assumed the supervision as well as the
Consequently, the bill was returned to the Congress of the administration of the study of law which was objected to by the
Philippines, but it was not repassed by 2/3 vote of each House President in the Bar Bill of 1951.
as prescribed by section 20, article VI of the Constitution.
FINALS CONSTITUTIONAL LAW I ACJUCO NOV 11, 2017 198

The President in vetoing the Bar Bill last year stated among Respectfully returned to the Honorable, the Acting Executive
his objections that the bill would admit to the practice of law "a Secretary, Manila, with the information that, with respect to
special class who failed in the bar examination". He Senate Bill No. 371, the members of the Court are taking the
considered the bill a class legislation. This contention, same views they expressed on Senate Bill No. 12 passed by
however, is not, in good conscience, correct because Congress in May, 1951, contained in the first indorsement of
Congress is merely supplementing what the Supreme Court the undersigned dated June 5, 1951, to the Assistant
have already established as precedent by making as low as Executive Secretary.
69 per cent the passing mark of those who took the Bar
examination in 1947. These bar candidates for who this bill (Sgd.) RICARDO PARAS
should be enacted, considered themselves as having passed
the bar examination on the strength of the established
precedent of our Supreme Court and were fully aware of the The President allowed the period within which the bill should
insurmountable difficulties and handicaps which they were be signed to pass without vetoing it, by virtue of which it
unavoidably placed. We believe that such precedent cannot became a law on June 21, 1953 (Sec. 20, Art. VI, Constitution)
or could not have been altered, constitutionally, by the numbered 972 (many times erroneously cited as No. 974).
Supreme Court, without giving due consideration to the rights
already accrued or vested in the bar candidates who took the It may be mentioned in passing that 1953 was an election
examination when the precedent was not yet altered, or in year, and that both the President and the author of the Bill
effect, was still enforced and without being inconsistent with were candidates for re-election, together, however, they lost
the principles of their previous resolutions. in the polls.

If this bill would be enacted, it shall be considered as a simple Separate Opinions


curative act or corrective statute which Congress has the
power to enact. The requirement of a "valid classification" as LABRADOR, J., concurring and dissenting:
against class legislation, is very expressed in the following
American Jurisprudence:
The right to admit members to the Bar is, and has always
been, the exclusive privilege of this Court, because lawyers
A valid classification must include all who naturally belong to are members of the Court and only this Court should be
the class, all who possess a common disability, attribute, or allowed to determine admission thereto in the interest of the
classification, and there must be a "natural" and substantial principle of the separation of powers. The power to admit is
differentiation between those included in the class and those judicial in the sense that discretion is used in is exercise. This
it leaves untouched. When a class is accepted by the Court power should be distinguished from the power to promulgate
as "natural" it cannot be again split and then have the rules which regulate admission. It is only this power (to
dissevered factions of the original unit designated with promulgate amendments to the rules) that is given in the
different rules established for each. (Fountain Park Co. vs. Constitution to the Congress, not the exercise of the discretion
Rensier, 199 Ind. 95, N. E. 465 (1926). to admit or not to admit. Thus the rules on the holding of
examination, the qualifications of applicants, the passing
Another case penned by Justice Cardozo: "Time with its tides grades, etc. are within the scope of the legislative power. But
brings new conditions which must be cared for by new laws. the power to determine when a candidate has made or has
Sometimes the new conditions affect the members of a class. not made the required grade is judicial, and lies completely
If so, the correcting statute must apply to all alike. Sometimes with this Court.
the condition affect only a few. If so, the correcting statute may
be as narrow as the mischief. The constitution does not I hold that the act under consideration is an exercise of the
prohibit special laws inflexibly and always. It permits them judicial function, and lies beyond the scope of the
when there are special evils with which the general laws are congressional prerogative of amending the rules. To say that
incompetent to cope. The special public purpose will sustain candidates who obtain a general average of 72 per cent in
the special form. . . . The problem in the last analysis is one of 1953, 73 per cent in 1954, and 74 per cent in 1955 should be
legislative policy, with a wide margin of discretion conceded to considered as having passed the examination, is to mean
the lawmakers. Only in the case of plain abuse will there be exercise of the privilege and discretion judged in this Court. It
revision by the court. (In Williams vs. Mayor and City Council is a mandate to the tribunal to pass candidates for different
of Baltimore, 286 U. S. 36, 77 L. Ed. 1015, 53 Sup. Ct. 431). years with grades lower than the passing mark. No reasoning
(1932) is necessary to show that it is an arrogation of the Court's
judicial authority and discretion. It is furthermore objectionable
This bill has all the earmarks of a corrective statute which as discriminatory. Why should those taking the examinations
always retroacts to the extent of the care of correction only as in 1953, 1954 and 1955 be allowed to have the privilege of a
in this case from 1946 when the Supreme Court first deviated lower passing grade, while those taking earlier or later are
from the rule of 75 per cent in the Rules of Court. not?

For the foregoing purposes the approval of this bill is earnestly I vote that the act in toto be declared unconstitutional, because
recommended. it is not embraced within the rule-making power of Congress,
because it is an undue interference with the power of this
(Sgd.) PABLO ANGELES DAVID Court to admit members thereof, and because it is
discriminatory.
Senator
PARAS, C.J., dissenting:
Without much debate, the revised bill was passed by
Congress as above transcribed. The President again asked Under section 145 of Rule of Court No. 127, in order that a bar
the comments of this Court, which endorsed the following: candidate "may be deemed to have passed his examinations
successfully, he must have obtained a general average of 75
FINALS CONSTITUTIONAL LAW I ACJUCO NOV 11, 2017 199

per cent in all subjects, without falling below 50 per cent in any that any bar candidate who obtained a grade of 75 per cent in
subject.' This passing mark has always been adhered to, with any subject in any examination after July 4, 1946, shall be
certain exception presently to be specified. deemed to have passed in such subject or subjects and such
grade or grades shall be included in computing the passing in
With reference to the bar examinations given in August, 1946, any subsequent examinations.
the original list of successful candidates included only those
who obtained a general average of 75 per cent or more. Upon Numerous candidates who had taken the bar examinations
motion for reconsideration, however, 12 candidates with previous to the approval of Republic Act No. 972 and failed to
general averages ranging from 72 to 73 per cent were raised obtain the necessary passing average, filed with this Court
to 75 per cent by resolution of December 18, 1946. In the mass or separate petitions, praying that they be admitted to
examinations of November, 1946 the list first released the practice of law under and by virtue of said Act, upon the
containing the names of successful candidates covered only allegation that they have obtained the general averages
those who obtained a general average of 75 per cent or more; prescribed therein. In virtue of the resolution of July 6, 1953,
but, upon motion for reconsideration, 19 candidates with a this Court held on July 11, 1953 a hearing on said petitions,
general average of 72 per cent were raised to 75 per cent by and members of the bar, especially authorized
resolution of March 31, 1947. This would indicate that in the representatives of bar associations, were invited to argue or
original list of successful candidates those having a general submit memoranda as amici curiae, the reason alleged for
average of 73 per cent or more but below 75 per cent were said hearing being that some doubt had "been expressed on
included. After the original list of 1947 successful bar the constitutionality of Republic Act No. 972 in so far as it
candidates had been released, and on motion for affects past bar examinations and the matter" involved "a new
reconsideration, all candidates with a general average of 69 question of public interest."
per cent were allowed to pass by resolution of July 15, 1948.
With respect to the bar examinations held in August, 1948, in All discussions in support of the proposition that the power to
addition to the original list of successful bar candidates, all regulate the admission to the practice of law is inherently
those who obtained a general average of 70 per cent or more, judicial, are immaterial, because the subject is now governed
irrespective of the grades in any one subject and irrespective by the Constitution which in Article VII, section 13, provides as
of whether they filed petitions for reconsideration, were follows:
allowed to pass by resolution of April 28, 1949. Thus, for the
year 1947 the Court in effect made 69 per cent as the passing
average, and for the year 1948, 70 per cent; and this The Supreme Court shall have the power to promulgate rules
amounted, without being noticed perhaps, to an amendment concerning pleading, practice, and procedure in all courts, and
of section 14 of Rule 127. the admission to the practice of law. Said rules shall be
uniform for all courts of the same grade and shall not diminish,
increase or modify substantive right. The existing laws on
Numerous flunkers in the bar examinations held subsequent pleading, practice, and procedure are hereby repealed as
to 1948, whose general averages mostly ranged from 69 to 73 statutes and are declared Rules of Court, subject to the power
per cent, filed motions for reconsideration invoking the of the Supreme Court to alter and modify the same. The
precedents set by this Court in 1947 and 1948, but said Congress shall have the power to repeal, alter, or supplement
motions were uniformly denied. the rules concerning pleading, practice, and procedure, and
the admission to the practice of law in the Philippines.
In the year 1951, the Congress, after public hearings where
law deans and professors, practising attorneys, presidents of Under this constitutional provision, while the Supreme Court
bar associations, and law graduates appeared and argued has the power to promulgate rules concerning the admission
lengthily pro or con, approved a bill providing, among others, to the practice of law, the Congress has the power to repeal,
for the reduction of the passing general average from 75 per alter or supplement said rules. Little intelligence is necessary
cent to 70 per cent, retroactive to any bar examination held to see that the power of the Supreme Court and the Congress
after July 4, 1946. This bill was vetoed by the President mainly to regulate the admission to the practice of law is concurrent.
in view of an unfavorable comment of Justices Padilla,
Tuason, Montemayor, Reyes, Bautista and Jugo. In 1953, the
Congress passed another bill similar to the previous bill The opponents of Republic Act No. 972 argue that this Act, in
vetoed by the President, with the important difference that in so far as it covers bar examinations held prior to its approval,
the later bill the provisions in the first bill regarding (1) the is unconstitutional, because it sets aside the final resolutions
supervision and regulation by the Supreme Court of the study of the Supreme Court refusing to admit to the practice of law
of law, (2) the inclusion of Social Legislation and Taxation as the various petitioners, thereby resulting in a legislative
new bar subjects, (3) the publication of the bar examiners encroachment upon the judicial power. In my opinion this view
before the holding of the examination, and (4) the equal is erroneous. In the first place, resolutions on the rejection of
division among the examiners of all the admission fees paid bar candidates do not have the finality of decisions in
by bar applicants, were eliminated. This second bill was justiciable cases where the Rules of Court expressly fix certain
allowed to become a law, Republic Act No. 972, by the periods after which they become executory and unalterable.
President by merely not signing it within the required period; Resolutions on bar matters, specially on motions for
and in doing so the President gave due respect to the will of reconsiderations filed by flunkers in any give year, are subject
the Congress which, speaking for the people, chose to repass to revision by this Court at any time, regardless of the period
the bill first vetoed by him. within which the motion were filed, and this has been the
practice heretofore. The obvious reason is that bar
examinations and admission to the practice of law may be
Under Republic Act No. 972, any bar candidates who obtained deemed as a judicial function only because said matters
a general average of 70 per cent in any examinations after happen to be entrusted, under the Constitution and our Rules
July 4, 1946 up to August 1951; 71 per cent in the 1952 bar of Court, to the Supreme Court. There is no judicial function
examinations; 72 per cent in 1953 bar examinations; 73 per involved, in the subject and constitutional sense of the word,
cent in the 1954 bar examinations; and 74 per cent in the 1955 because bar examinations and the admission to the practice
bar examinations, without obtaining a grade below 50 per cent of law, unlike justiciable cases, do not affect opposing litigants.
in any subject, shall be allowed to pass. Said Act also provides It is no more than the function of other examining boards. In
FINALS CONSTITUTIONAL LAW I ACJUCO NOV 11, 2017 200

the second place, retroactive laws are not prohibited by the bar examinations and 70 per cent for the 1948 examinations.
Constitution, except only when they would be ex post facto, Anyway, we should not inquire into the wisdom of the law,
would impair obligations and contracts or vested rights or since this is a matter that is addressed to the judgment of the
would deny due process and equal protection of the law. legislators. This Court in many instances had doubted the
Republic Act No. 972 certainly is not an ex post facto propriety of legislative enactments, and yet it has consistently
enactment, does not impair any obligation and contract or refrained from nullifying them solely on that ground.
vested rights, and denies to no one the right to due process
and equal protection of the law. On the other hand, it is a mere To say that the admission of the bar candidates benefited
curative statute intended to correct certain obvious under Republic Act 972 is against public interest, is to assume
inequalities arising from the adoption by this Court of different that the matter of whether said Act is beneficial or harmful to
passing general averages in certain years. the general public was not considered by the Congress. As
already stated, the Congress held public hearings, and we are
Neither can it be said that bar candidates prior to July 4, 1946, bound to assume that the legislators, loyal, as do the
are being discriminated against, because we no longer have members of this Court, to their oath of office, had taken all the
any record of those who might have failed before the war, circumstances into account before passing the Act. On the
apart from the circumstance that 75 per cent had always been question of public interest I may observe that the Congress,
the passing mark during said period. It may also be that there representing the people who elected them, should be more
are no pre-war bar candidates similarly situated as those qualified to make an appraisal. I am inclined to accept
benefited by Republic Act No. 972. At any rate, in the matter Republic Act No. 972 as an expression of the will of the people
of classification, the reasonableness must be determined by through their duly elected representatives.
the legislative body. It is proper to recall that the Congress
held public hearings, and we can fairly suppose that the I would, however, not go to the extent of admitting that the
classification adopted in the Act reflects good legislative Congress, in the exercise of its concurrent power to repeal,
judgment derived from the facts and circumstances then alter, or supplement the Rules of Court regarding the
brought out. admission to the practice of law, may act in an arbitrary or
capricious manner, in the same way that this Court may not
As regards the alleged interference in or encroachment upon do so. We are thus left in the situation, incidental to a
the judgment of this Court by the Legislative Department, it is democracy, where we can and should only hope that the right
sufficient to state that, if there is any interference at all, it is men are put in the right places in our Government.
one expressly sanctioned by the Constitution. Besides,
interference in judicial adjudication prohibited by the Wherefore, I hold that Republic Act No. 972 is
Constitution is essentially aimed at protecting rights of litigants constitutional and should therefore be given effect in its
that have already been vested or acquired in virtue of entirety.
decisions of courts, not merely for the empty purpose of
creating appearances of separation and equality among the
three branches of the Government. Republic Act No. 972 has
not produced a case involving two parties and decided by the
Court in favor of one and against the other. Needless to say,
the statute will not affect the previous resolutions passing bar
candidates who had obtained the general average prescribed
by section 14 of Rule 127. A law would be objectionable and
unconstitutional if, for instance, it would provide that those
who have been admitted to the bar after July 4, 1946, whose
general average is below 80 per cent, will not be allowed to
practice law, because said statute would then destroy a right
already acquired under previous resolutions of this Court,
namely, the bar admission of those whose general averages
were from 75 to 79 per cent.

Without fear of contradiction, I think the Supreme Court, in the


exercise of its rule-making power conferred by the
Constitution, may pass a resolution amending section 14 of
Rule 127 by reducing the passing average to 70 per cent,
effective several years before the date of the resolution.
Indeed, when this Court on July 15, 1948 allowed to pass all
candidates who obtained a general average of 69 per cent or
more and on April 28, 1949 those who obtained a general
average of 70 per cent or more, irrespective of whether they
filed petitions for reconsideration, it in effect amended section
14 of Rule 127 retroactively, because during the examinations
held in August 1947 and August 1948, said section (fixing the
general average at 75 per cent) was supposed to be in force.
In stands to reason, if we are to admit that the Supreme Court
and the Congress have concurrent power to regulate the
admission to the practice of law, that the latter may validly
pass a retroactive rule fixing the passing general average.

Republic Act No. 972 cannot be assailed on the ground that it


is unreasonable, arbitrary or capricious, since this Court had
already adopted as passing averages 69 per cent for the 1947
FINALS CONSTITUTIONAL LAW I ACJUCO NOV 11, 2017 201

[B. M. No. 1036. June 10, 2003] On 22 May 2001, the Court issued a resolution allowing
respondent to take the lawyers oath but disallowed him from
DONNA MARIE S. AGUIRRE, complainant, vs. EDWIN L. signing the Roll of Attorneys until he is cleared of the charges
RANA, respondent. against him. In the same resolution, the Court required
respondent to comment on the complaint against him.
DECISION
In his Comment, respondent admits that Bunan sought his
specific assistance to represent him before the MBEC.
CARPIO, J.: Respondent claims that he decided to assist and advice
Bunan, not as a lawyer but as a person who knows the law.
The Case Respondent admits signing the 19 May 2001 pleading that
objected to the inclusion of certain votes in the canvassing. He
Before one is admitted to the Philippine Bar, he must possess explains, however, that he did not sign the pleading as a
the requisite moral integrity for membership in the legal lawyer or represented himself as an attorney in the pleading.
profession. Possession of moral integrity is of greater
importance than possession of legal learning. The practice of On his employment as secretary of the Sangguniang Bayan,
law is a privilege bestowed only on the morally fit. A bar respondent claims that he submitted his resignation on 11
candidate who is morally unfit cannot practice law even if he May 2001 which was allegedly accepted on the same date.
passes the bar examinations. He submitted a copy of the Certification of Receipt of
Revocable Resignation dated 28 May 2001 signed by Vice-
The Facts Mayor Napoleon Relox. Respondent further claims that the
complaint is politically motivated considering that complainant
is the daughter of Silvestre Aguirre, the losing candidate for
Respondent Edwin L. Rana (respondent) was among those mayor of Mandaon, Masbate. Respondent prays that the
who passed the 2000 Bar Examinations. complaint be dismissed for lack of merit and that he be
allowed to sign the Roll of Attorneys.
On 21 May 2001, one day before the scheduled mass oath-
taking of successful bar examinees as members of the On 22 June 2001, complainant filed her Reply to respondents
Philippine Bar, complainant Donna Marie Aguirre Comment and refuted the claim of respondent that his
(complainant) filed against respondent a Petition for Denial of appearance before the MBEC was only to extend specific
Admission to the Bar. Complainant charged respondent with assistance to Bunan. Complainant alleges that on 19 May
unauthorized practice of law, grave misconduct, violation of 2001 Emily Estipona-Hao (Estipona-Hao) filed a petition for
law, and grave misrepresentation. proclamation as the winning candidate for mayor. Respondent
signed as counsel for Estipona-Hao in this petition. When
The Court allowed respondent to take his oath as a member respondent appeared as counsel before the MBEC,
of the Bar during the scheduled oath-taking on 22 May 2001 complainant questioned his appearance on two grounds: (1)
at the Philippine International Convention Center. However, respondent had not taken his oath as a lawyer; and (2) he was
the Court ruled that respondent could not sign the Roll of an employee of the government.
Attorneys pending the resolution of the charge against him.
Thus, respondent took the lawyers oath on the scheduled date Respondent filed a Reply (Re: Reply to Respondents
but has not signed the Roll of Attorneys up to now. Comment) reiterating his claim that the instant administrative
case is motivated mainly by political vendetta.
Complainant charges respondent for unauthorized practice of
law and grave misconduct. Complainant alleges that On 17 July 2001, the Court referred the case to the Office of
respondent, while not yet a lawyer, appeared as counsel for a the Bar Confidant (OBC) for evaluation, report and
candidate in the May 2001 elections before the Municipal recommendation.
Board of Election Canvassers (MBEC) of Mandaon, Masbate.
Complainant further alleges that respondent filed with the
MBEC a pleading dated 19 May 2001 entitled Formal OBCs Report and Recommendation
Objection to the Inclusion in the Canvassing of Votes in Some
Precincts for the Office of Vice-Mayor. In this pleading, The OBC found that respondent indeed appeared before the
respondent represented himself as counsel for and in behalf MBEC as counsel for Bunan in the May 2001 elections. The
of Vice Mayoralty Candidate, George Bunan, and signed the minutes of the MBEC proceedings show that respondent
pleading as counsel for George Bunan (Bunan). actively participated in the proceedings. The OBC likewise
found that respondent appeared in the MBEC proceedings
On the charge of violation of law, complainant claims that even before he took the lawyers oath on 22 May 2001. The
respondent is a municipal government employee, being a OBC believes that respondents misconduct casts a serious
secretary of the Sangguniang Bayan of Mandaon, Masbate. doubt on his moral fitness to be a member of the Bar. The
As such, respondent is not allowed by law to act as counsel OBC also believes that respondents unauthorized practice of
for a client in any court or administrative body. law is a ground to deny his admission to the practice of law.
The OBC therefore recommends that respondent be denied
admission to the Philippine Bar.
On the charge of grave misconduct and misrepresentation,
complainant accuses respondent of acting as counsel for vice
mayoralty candidate George Bunan (Bunan) without the latter On the other charges, OBC stated that complainant failed to
engaging respondents services. Complainant claims that cite a law which respondent allegedly violated when he
respondent filed the pleading as a ploy to prevent the appeared as counsel for Bunan while he was a government
proclamation of the winning vice mayoralty candidate. employee. Respondent resigned as secretary and his
resignation was accepted. Likewise, respondent was
authorized by Bunan to represent him before the MBEC.
FINALS CONSTITUTIONAL LAW I ACJUCO NOV 11, 2017 202

The Courts Ruling various pleadings, without license to do so. Evidence clearly
supports the charge of unauthorized practice of law.
We agree with the findings and conclusions of the OBC Respondent called himself counsel knowing fully well that he
that respondent engaged in the unauthorized practice of was not a member of the Bar. Having held himself out as
law and thus does not deserve admission to the counsel knowing that he had no authority to practice law,
Philippine Bar. respondent has shown moral unfitness to be a member of the
Philippine Bar.[3]
Respondent took his oath as lawyer on 22 May 2001.
However, the records show that respondent appeared as The right to practice law is not a natural or constitutional right
counsel for Bunan prior to 22 May 2001, before but is a privilege. It is limited to persons of good moral
respondent took the lawyers oath. In the pleading entitled character with special qualifications duly ascertained and
Formal Objection to the Inclusion in the Canvassing of Votes certified.The exercise of this privilege presupposes
in Some Precincts for the Office of Vice-Mayor dated 19 May possession of integrity, legal knowledge, educational
2001, respondent signed as counsel for George Bunan. In the attainment, and even public trust[4] since a lawyer is an officer
first paragraph of the same pleading respondent stated that of the court. A bar candidate does not acquire the right to
he was the undersigned Counsel for, and in behalf of Vice practice law simply by passing the bar examinations. The
Mayoralty Candidate, GEORGE T. BUNAN. Bunan himself practice of law is a privilege that can be withheld even from
wrote the MBEC on 14 May 2001 that he had authorized Atty. one who has passed the bar examinations, if the person
Edwin L. Rana as his counsel to represent him before the seeking admission had practiced law without a license.[5]
MBEC and similar bodies.
The regulation of the practice of law is unquestionably strict.
On 14 May 2001, mayoralty candidate Emily Estipona-Hao In Beltran, Jr. v. Abad,[6] a candidate passed the bar
also retained respondent as her counsel. On the same date, examinations but had not taken his oath and signed the Roll
14 May 2001, Erly D. Hao informed the MBEC that Atty. Edwin of Attorneys. He was held in contempt of court for practicing
L. Rana has been authorized by REFORMA LM-PPC as the law even before his admission to the Bar. Under Section 3 (e)
legal counsel of the party and the candidate of the said party. of Rule 71 of the Rules of Court, a person who engages in the
Respondent himself wrote the MBEC on 14 May 2001 that he unauthorized practice of law is liable for indirect contempt of
was entering his appearance as counsel for Mayoralty court.[7]
Candidate Emily Estipona-Hao and for the REFORMA LM-
PPC. On 19 May 2001, respondent signed as counsel for True, respondent here passed the 2000 Bar Examinations
Estipona-Hao in the petition filed before the MBEC praying for and took the lawyers oath. However, it is the signing in the
the proclamation of Estipona-Hao as the winning candidate for Roll of Attorneys that finally makes one a full-fledged lawyer.
mayor of Mandaon, Masbate. The fact that respondent passed the bar examinations is
immaterial. Passing the bar is not the only qualification to
All these happened even before respondent took the lawyers become an attorney-at-law.[8] Respondent should know that
oath. Clearly, respondent engaged in the practice of law two essential requisites for becoming a lawyer still had to be
without being a member of the Philippine Bar. performed, namely: his lawyers oath to be administered by
this Court and his signature in the Roll of Attorneys.[9]
In Philippine Lawyers Association v. Agrava,[1] the Court
elucidated that: On the charge of violation of law, complainant contends that
the law does not allow respondent to act as counsel for a
private client in any court or administrative body since
The practice of law is not limited to the conduct of cases or respondent is the secretary of the Sangguniang Bayan.
litigation in court; it embraces the preparation of pleadings and
other papers incident to actions and special proceedings, the
management of such actions and proceedings on behalf of Respondent tendered his resignation as secretary of the
clients before judges and courts, and in addition, Sangguniang Bayan prior to the acts complained of as
conveyancing. In general, all advice to clients, and all action constituting unauthorized practice of law. In his letter dated 11
taken for them in matters connected with the law,incorporation May 2001 addressed to Napoleon Relox, vice- mayor and
services, assessment and condemnation services presiding officer of the Sangguniang Bayan, respondent
contemplating an appearance before a judicial body, the stated that he was resigning effective upon your
foreclosure of a mortgage, enforcement of a creditor's claim in acceptance.[10] Vice-Mayor Relox accepted respondents
bankruptcy and insolvency proceedings, and conducting resignation effective 11 May 2001.[11] Thus, the evidence
proceedings in attachment, and in matters of estate and does not support the charge that respondent acted as counsel
guardianship have been held to constitute law practice, as do for a client while serving as secretary of the Sangguniang
the preparation and drafting of legal instruments, where the Bayan.
work done involves the determination by the trained legal mind
of the legal effect of facts and conditions. (5 Am. Jur. p. 262, On the charge of grave misconduct and misrepresentation,
263). (Italics supplied) x x x evidence shows that Bunan indeed authorized respondent to
represent him as his counsel before the MBEC and similar
In Cayetano v. Monsod,[2] the Court held that practice of law bodies. While there was no misrepresentation, respondent
means any activity, in or out of court, which requires the nonetheless had no authority to practice law.
application of law, legal procedure, knowledge, training and
experience. To engage in the practice of law is to perform acts WHEREFORE, respondent Edwin L. Rana is DENIED
which are usually performed by members of the legal admission to the Philippine Bar.
profession. Generally, to practice law is to render any kind of
service which requires the use of legal knowledge or skill. SO ORDERED.

Verily, respondent was engaged in the practice of law when


he appeared in the proceedings before the MBEC and filed
FINALS CONSTITUTIONAL LAW I ACJUCO NOV 11, 2017 203

A.M. No. 1928 August 3, 1978 SEC. 10. Effect of non-payment of dues. — Subject to the
provisions of Section 12 of this Rule, default in the payment of
In the Matter of the IBP Membership Dues Delinquency of annual dues for six months shall warrant suspension of
Atty. MARCIAL A. EDILION (IBP Administrative Case No. membership in the Integrated Bar, and default in such
MDD-1) payment for one year shall be a ground for the removal of the
name of the delinquent member from the Roll of Attorneys.
RESOLUTION
The all-encompassing, all-inclusive scope of membership in
the IBP is stated in these words of the Court Rule:
CASTRO, C.J.:
SECTION 1. Organization. — There is hereby organized an
The respondent Marcial A. Edillon is a duly licensed practicing official national body to be known as the 'Integrated Bar of the
attorney in the Philippines. Philippines,' composed of all persons whose names now
appear or may hereafter be included in the Roll of Attorneys
On November 29, 1975, the Integrated Bar of the Philippines of the Supreme Court.
(IBP for short) Board of Governors unanimously adopted
Resolution No. 75-65 in Administrative Case No. MDD-1 The obligation to pay membership dues is couched in the
(In the Matter of the Membership Dues Delinquency of following words of the Court Rule:
Atty. Marcial A. Edillon) recommending to the Court the
removal of the name of the respondent from its Roll of
Attorneys for "stubborn refusal to pay his membership SEC. 9. Membership dues. Every member of the Integrated
dues" to the IBP since the latter's constitution Bar shall pay such annual dues as the Board of Governors
notwithstanding due notice. shall determine with the approval of the Supreme Court. ...

On January 21, 1976, the IBP, through its then President The core of the respondent's arguments is that the above
Liliano B. Neri, submitted the said resolution to the Court for provisions constitute an invasion of his constitutional rights in
consideration and approval, pursuant to paragraph 2, Section the sense that he is being compelled, as a pre-condition to
24, Article III of the By-Laws of the IBP, which reads: maintaining his status as a lawyer in good standing, to be a
member of the IBP and to pay the corresponding dues, and
that as a consequence of this compelled financial support of
.... Should the delinquency further continue until the following the said organization to which he is admittedly personally
June 29, the Board shall promptly inquire into the cause or antagonistic, he is being deprived of the rights to liberty and
causes of the continued delinquency and take whatever action property guaranteed to him by the Constitution. Hence, the
it shall deem appropriate, including a recommendation to the respondent concludes, the above provisions of the Court Rule
Supreme Court for the removal of the delinquent member's and of the IBP By-Laws are void and of no legal force and
name from the Roll of Attorneys. Notice of the action taken effect.
shall be sent by registered mail to the member and to the
Secretary of the Chapter concerned.
The respondent similarly questions the jurisdiction of the
Court to strike his name from the Roll of Attorneys,
On January 27, 1976, the Court required the respondent to contending that the said matter is not among the
comment on the resolution and letter adverted to above; he justiciable cases triable by the Court but is rather of an
submitted his comment on February 23, 1976, reiterating his "administrative nature pertaining to an administrative
refusal to pay the membership fees due from him. body."

On March 2, 1976, the Court required the IBP President and The case at bar is not the first one that has reached the Court
the IBP Board of Governors to reply to Edillon's comment: on relating to constitutional issues that inevitably and inextricably
March 24, 1976, they submitted a joint reply. come up to the surface whenever attempts are made to
regulate the practice of law, define the conditions of such
Thereafter, the case was set for hearing on June 3, 1976. After practice, or revoke the license granted for the exercise of the
the hearing, the parties were required to submit memoranda legal profession.
in amplification of their oral arguments. The matter was
thenceforth submitted for resolution. The matters here complained of are the very same issues
raised in a previous case before the Court, entitled
At the threshold, a painstaking scrutiny of the respondent's "Administrative Case No. 526, In the Matter of the Petition for
pleadings would show that the propriety and necessity of the the Integration of the Bar of the Philippines, Roman Ozaeta,
integration of the Bar of the Philippines are in essence et al., Petitioners." The Court exhaustively considered all
conceded. The respondent, however, objects to particular these matters in that case in its Resolution ordaining the
features of Rule of Court 139-A (hereinafter referred to as the integration of the Bar of the Philippines, promulgated on
Court Rule) 1 — in accordance with which the Bar of the January 9, 1973. The Court there made the unanimous
Philippines was integrated — and to the provisions of par. 2, pronouncement that it was
Section 24, Article III, of the IBP By-Laws (hereinabove cited).
... fully convinced, after a thoroughgoing conscientious study
The authority of the IBP Board of Governors to recommend to of all the arguments adduced in Adm. Case No. 526 and the
the Supreme Court the removal of a delinquent member's authoritative materials and the mass of factual data contained
name from the Roll of Attorneys is found in par. 2 Section 24, in the exhaustive Report of the Commission on Bar
Article Ill of the IBP By-Laws (supra), whereas the authority of Integration, that the integration of the Philippine Bar is
the Court to issue the order applied for is found in Section 10 'perfectly constitutional and legally unobjectionable'. ...
of the Court Rule, which reads:
FINALS CONSTITUTIONAL LAW I ACJUCO NOV 11, 2017 204

Be that as it may, we now restate briefly the posture of the Phil 218), for, as the Latin maxim goes, "Salus populi est
Court. supreme lex." The public welfare is the supreme law. To this
fundamental principle of government the rights of individuals
An "Integrated Bar" is a State-organized Bar, to which are subordinated. Liberty is a blessing without which life is a
every lawyer must belong, as distinguished from bar misery, but liberty should not be made to prevail over authority
associations organized by individual lawyers themselves, because then society win fall into anarchy (Calalang vs.
membership in which is voluntary. Integration of the Bar Williams, 70 Phil. 726). It is an undoubted power of the State
is essentially a process by which every member of the Bar to restrain some individuals from all freedom, and all
is afforded an opportunity to do his share in carrying out individuals from some freedom.
the objectives of the Bar as well as obliged to bear his
portion of its responsibilities. Organized by or under the But the most compelling argument sustaining the
direction of the State, an integrated Bar is an official constitutionality and validity of Bar integration in the
national body of which all lawyers are required to be Philippines is the explicit unequivocal grant of precise power
members. They are, therefore, subject to all the rules to the Supreme Court by Section 5 (5) of Article X of the 1973
prescribed for the governance of the Bar, including the Constitution of the Philippines, which reads:
requirement of payment of a reasonable annual fee for the
effective discharge of the purposes of the Bar, and Sec. 5. The Supreme Court shall have the following powers:
adherence to a code of professional ethics or
professional responsibility breach of which constitutes
sufficient reason for investigation by the Bar and, upon xxx xxx xxx
proper cause appearing, a recommendation for discipline
or disbarment of the offending member. 2 (5) Promulgate rules concerning pleading, practice, and pro.
procedure in all courts, and the admission to the practice of
The integration of the Philippine Bar was obviously law and the integration of the Bar ...,
dictated by overriding considerations of public interest
and public welfare to such an extent as more than and Section 1 of Republic Act No. 6397, which reads:
constitutionally and legally justifies the restrictions that
integration imposes upon the personal interests and SECTION 1. Within two years from the approval of this Act,
personal convenience of individual lawyers. 3
the Supreme Court may adopt rules of Court to effect the
integration of the Philippine Bar under such conditions as it
Apropos to the above, it must be stressed that all legislation shall see fit in order to raise the standards of the legal
directing the integration of the Bar have been uniformly and profession, improve the administration of justice, and enable
universally sustained as a valid exercise of the police power the Bar to discharge its public responsibility more effectively.
over an important profession. The practice of law is not a
vested right but a privilege, a privilege moreover clothed with Quite apart from the above, let it be stated that even without
public interest because a lawyer owes substantial duties not the enabling Act (Republic Act No. 6397), and looking solely
only to his client, but also to his brethren in the profession, to to the language of the provision of the Constitution granting
the courts, and to the nation, and takes part in one of the most the Supreme Court the power "to promulgate rules concerning
important functions of the State — the administration of justice pleading, practice and procedure in all courts, and the
— as an officer of the court. 4 The practice of law being admission to the practice of law," it at once becomes
clothed with public interest, the holder of this privilege must indubitable that this constitutional declaration vests the
submit to a degree of control for the common good, to the Supreme Court with plenary power in all cases regarding the
extent of the interest he has created. As the U. S. Supreme admission to and supervision of the practice of law.
Court through Mr. Justice Roberts explained, the expression
"affected with a public interest" is the equivalent of "subject to
the exercise of the police power" (Nebbia vs. New York, 291 Thus, when the respondent Edillon entered upon the legal
U.S. 502). profession, his practice of law and his exercise of the said
profession, which affect the society at large, were (and are)
subject to the power of the body politic to require him to
When, therefore, Congress enacted Republic Act No. 6397 conform to such regulations as might be established by the
5 authorizing the Supreme Court to "adopt rules of court
proper authorities for the common good, even to the extent of
to effect the integration of the Philippine Bar under such
interfering with some of his liberties. If he did not wish to
conditions as it shall see fit," it did so in the exercise of submit himself to such reasonable interference and
the paramount police power of the State. The Act's avowal regulation, he should not have clothed the public with an
is to "raise the standards of the legal profession, improve the interest in his concerns.
administration of justice, and enable the Bar to discharge its
public responsibility more effectively." Hence, the Congress in
enacting such Act, the Court in ordaining the integration of the On this score alone, the case for the respondent must already
Bar through its Resolution promulgated on January 9, 1973, fall.
and the President of the Philippines in decreeing the
constitution of the IBP into a body corporate through The issues being of constitutional dimension, however,
Presidential Decree No. 181 dated May 4, 1973, were we now concisely deal with them seriatim.
prompted by fundamental considerations of public welfare and
motivated by a desire to meet the demands of pressing public
1. The first objection posed by the respondent is that the Court
necessity.
is without power to compel him to become a member of the
Integrated Bar of the Philippines, hence, Section 1 of the Court
The State, in order to promote the general welfare, may Rule is unconstitutional for it impinges on his constitutional
interfere with and regulate personal liberty, property and right of freedom to associate (and not to associate). Our
occupations. Persons and property may be subjected to answer is: To compel a lawyer to be a member of the
restraints and burdens in order to secure the general Integrated Bar is not violative of his constitutional freedom to
prosperity and welfare of the State (U.S. vs. Gomez Jesus, 31 associate. 6
FINALS CONSTITUTIONAL LAW I ACJUCO NOV 11, 2017 205

Integration does not make a lawyer a member of any group of proceeding was confirmed and disbarment ordered, the court,
which he is not already a member. He became a member of sustaining the Bar Integration Act of Kentucky, said: "The
the Bar when he passed the Bar examinations. 7 All that power to regulate the conduct and qualifications of its officers
integration actually does is to provide an official national does not depend upon constitutional or statutory grounds. It is
organization for the well-defined but unorganized and a power which is inherent in this court as a court —
incohesive group of which every lawyer is a ready a member. appropriate, indeed necessary, to the proper administration of
8 justice ... the argument that this is an arbitrary power which
the court is arrogating to itself or accepting from the legislative
Bar integration does not compel the lawyer to associate with likewise misconceives the nature of the duty. It has limitations
anyone. He is free to attend or not attend the meetings of his no less real because they are inherent. It is an unpleasant task
Integrated Bar Chapter or vote or refuse to vote in its elections to sit in judgment upon a brother member of the Bar,
as he chooses. The only compulsion to which he is subjected particularly where, as here, the facts are disputed. It is a grave
is the payment of annual dues. The Supreme Court, in order responsibility, to be assumed only with a determination to
to further the State's legitimate interest in elevating the quality uphold the Ideals and traditions of an honorable profession
of professional legal services, may require that the cost of and to protect the public from overreaching and fraud. The
improving the profession in this fashion be shared by the very burden of the duty is itself a guaranty that the power will
subjects and beneficiaries of the regulatory program — the not be misused or prostituted. ..."
lawyers.9
The Court's jurisdiction was greatly reinforced by our 1973
Assuming that the questioned provision does in a sense Constitution when it explicitly granted to the Court the power
compel a lawyer to be a member of the Integrated Bar, such to "Promulgate rules concerning pleading, practice ... and the
compulsion is justified as an exercise of the police power of admission to the practice of law and the integration of the Bar
the State. 10 ... (Article X, Sec. 5(5) the power to pass upon the fitness of
the respondent to remain a member of the legal profession is
indeed undoubtedly vested in the Court.
2. The second issue posed by the respondent is that the
provision of the Court Rule requiring payment of a
membership fee is void. We see nothing in the Constitution We thus reach the conclusion that the provisions of Rule of
that prohibits the Court, under its constitutional power and Court 139-A and of the By-Laws of the Integrated Bar of the
duty to promulgate rules concerning the admission to the Philippines complained of are neither unconstitutional nor
practice of law and the integration of the Philippine Bar (Article illegal.
X, Section 5 of the 1973 Constitution) — which power the
respondent acknowledges — from requiring members of a WHEREFORE, premises considered, it is the unanimous
privileged class, such as lawyers are, to pay a reasonable fee sense of the Court that the respondent Marcial A. Edillon
toward defraying the expenses of regulation of the profession should be as he is hereby disbarred, and his name is hereby
to which they belong. It is quite apparent that the fee is indeed ordered stricken from the Roll of Attorneys of the Court.
imposed as a regulatory measure, designed to raise funds for
carrying out the objectives and purposes of integration. 11 Fernando, Teehankee, Barredo, Makasiar, Antonio, Muñoz
Palma, Aquino, Concepcion, Jr., Santos, Fernandez and
3. The respondent further argues that the enforcement of the Guerrero, JJ., concur.
penalty provisions would amount to a deprivation of property
without due process and hence infringes on one of his
constitutional rights. Whether the practice of law is a property
right, in the sense of its being one that entitles the holder of a
license to practice a profession, we do not here pause to
consider at length, as it clear that under the police power of
the State, and under the necessary powers granted to the
Court to perpetuate its existence, the respondent's right to
practise law before the courts of this country should be and is
a matter subject to regulation and inquiry. And, if the power to
impose the fee as a regulatory measure is recognize, then a
penalty designed to enforce its payment, which penalty may
be avoided altogether by payment, is not void as
unreasonable or arbitrary. 12

But we must here emphasize that the practice of law is not a


property right but a mere privilege, 13 and as such must bow
to the inherent regulatory power of the Court to exact
compliance with the lawyer's public responsibilities.

4. Relative to the issue of the power and/or jurisdiction of the


Supreme Court to strike the name of a lawyer from its Roll of
Attorneys, it is sufficient to state that the matters of admission,
suspension, disbarment and reinstatement of lawyers and
their regulation and supervision have been and are
indisputably recognized as inherent judicial functions and
responsibilities, and the authorities holding such are legion. 14

In In Re Sparks (267 Ky. 93, 101 S.W. (2d) 194), in which the
report of the Board of Bar Commissioners in a disbarment
FINALS CONSTITUTIONAL LAW I ACJUCO NOV 11, 2017 206

A.M. No. 491 October 6, 1989 association, namely, Attorneys Nereo Paculdo, Ramon Nisce,
and Violeta C. Drilon, the alleged use of government planes,
IN THE MATTER OF THE INQUIRY INTO THE 1989 and the officious intervention of certain public officials to
ELECTIONS OF THE INTEGRATED BAR OF THE influence the voting, all of which were done in violation of the
PHILIPPINES. IBP By-Laws which prohibit such activities. The Supreme
Court en banc, exercising its power of supervision over the
Integrated Bar, resolved to suspend the oath-taking of the IBP
PER CURIAM: officers-elect and to inquire into the veracity of the reports.

In the election of the national officers of the Integrated Bar of It should be stated at the outset that the election process itself
the Philippines (hereafter "IBP") held on June 3, 1989 at the (i.e. the voting and the canvassing of votes on June 3, 1989)
Philippine International Convention Center (or PICC), the which was conducted by the "IBP Comelec," headed by
following were elected by the House of Delegates (composed Justice Reynato Puno of the Court of Appeals, was
of 120 chapter presidents or their alternates) and proclaimed unanimously adjudged by the participants and observers to be
as officers: above board. For Justice Puno took it upon himself to device
safeguards to prevent tampering with, and marking of, the
NAME POSITION ballots.

Atty. Violeta Drilon President What the Court viewed with considerable concern was the
reported electioneering and extravagance that characterized
Atty. Bella Tiro Executive Vice-President the campaign conducted by the three candidates for president
of the IBP.

Atty. Salvador Lao Chairman, House of Delegates


I. MEDIA ACCOUNT OF THE ELECTION CAMPAIGN.

Atty. Renato F. Ronquillo Secretary, House of Delegates


Emil Jurado, in his column "IBP Group Questions Drilon
Election" (Manila Standard, Sunday, June 17, 1989), Luis
Atty. Teodoro Quicoy Treasurer, House of Delegates Mauricio, in two successive columns: "The Invertebrated Bar"
(Malaya, June 10, 1989) and "The Disintegrating Bar"
Atty. Oscar Badelles Sergeant at Arms, House of (Malaya, June 20, 1989), and Teodoro Locsin Jr. in an article,
Delegates entitled "Pam-Pam" (The Philippines Free Press, July 8,1989),
and the editorial, entitled 'Wrong Forum" of the Daily Globe
(June 8, 1989), were unanimously critical of the "vote-buying
Atty. Justiniano Cortes Governor & Vice-President for
and pressure tactics" allegedly employed in the campaign by
Northern Luzon
the three principal candidates: Attys. Violeta C. Drilon, Nereo
Paculdo and Ramon Nisce who reportedly "poured heart,
Atty. Ciriaco Atienza Governor & Vice-President for soul, money and influence to win over the 120 IBP delegates."
Central Luzon
Mr. Jurado mentioned the resentment of Atty. Drilon's rivals
Atty. Mario Jalandoni Governor & Vice-President for who felt at a disadvantage because Atty. Drilon allegedly used
Metro Manila PNB helicopters to visit far-flung IBP chapters on the pretext
of distributing Bigay Puso donations, and she had the added
Atty. Jose Aguilar Grapilon Governor & Vice-President for advantage of having regional directors and labor arbiters of
Southern Luzon the Department of Labor and Employment (who had been
granted leaves of absence by her husband, the Labor
Secretary) campaigning for her. Jurado's informants alleged
Atty. Teodoro Almine Governor & Vice-President for
that there was rampant vote-buying by some members of the
Bicolandia
U.P. Sigma Rho Fraternity (Secretary Drilon's fraternity), as
well as by some lawyers of ACCRA (Angara, Concepcion,
Atty. Porfirio Siyangco Governor & Vice-President for Cruz, Regala and Abello Law Office) where Mrs. Drilon is
Eastern Visayas employed, and that government positions were promised to
others by the office of the Labor Secretary.
Atty. Ricardo Teruel Governor & Vice-President for
Western Visayas Mr. Mauricio in his column wrote about the same matters and,
in addition, mentioned "talk of personnel of the Department of
Atty. Gladys Tiongco Governor & Vice-President for Labor, especially conciliators and employers, notably Chinese
Eastern Mindanao Filipinos, giving aid and comfort to her (Atty. Drilon's)
candidacy," the billeting of out-of-town delegates in plush
hotels where they were reportedly "wined and dined
Atty. Simeon Datumanong Governor & Vice-President for continuously, womened and subjected to endless haggling
Western Mindanao over the price of their votes x x x" which allegedly "ranged from
Pl5,000 to P20,000, and, on the day of the election, some
The newly-elected officers were set to take the their oath of twelve to twenty votes which were believed crucial,
office on July 4,1989, before the Supreme Court en banc. appreciated to P50,000."
However,disturbed by the widespread reports received by
some members of the Court from lawyers who had witnessed In his second column, Mr. Mauricio mentioned "how a top
or participated in the proceedings and the adverse comments official of the judiciary allegedly involved himself in IBP politics
published in the columns of some newspapers about the on election day by closeting himself with campaigners as they
intensive electioneering and overspending by the candidates, plotted their election strategy in a room of the PICC (the
led by the main protagonists for the office of president of the
FINALS CONSTITUTIONAL LAW I ACJUCO NOV 11, 2017 207

Philippine International Convention Center where the PNB planes to ferry them to distant places in their campaign
convention/election were held) during a recess x x x." to win the votes of delegates. The Philippine Airlines officials
were called to testify on the charge that some candidates gave
Mr. Locsin in his column and editorial substantially re-echoed free air fares to delegates to the convention. Officials of the
Mauricio's reports with some embellishments. Labor Department were also called to enable the Court to
ascertain the truth of the reports that labor officials openly
campaigned or worked for the election of Atty. Drilon.
II. THE COURT'S DECISION TO INVESTIGATE.
The newspaper columnists, Messrs. Luis Mauricio, Jesus
Responding to the critical reports, the Court, in its en banc Bigornia and Emil Jurado were subpoenaed to determine the
resolution dated June 15, 1989, directed the outgoing and nature of their sources of information relative to the IBP
incoming members of the IBP Board of Governors, the elections. Their stories were based, they said, on letters,
principal officers and Chairman of the House of Delegates to phone calls and personal interviews with persons who claimed
appear before it on Tuesday, June 20, 1989, at 2:00 o'clock to have knowledge of the facts, but whom they, invoking the
p.m., and there to inform the Court on the veracity of the Press Freedom Law, refused to identify.
aforementioned reports and to recommend, for the
consideration of the Court, appropriate approaches to the
problem of confirming and strengthening adherence to the The Committee has since submitted its Report after receiving,
fundamental principles of the IBP. and analyzing and assessing evidence given by such persons
as were perceived to have direct and personal knowledge of
the relevant facts; and the Court, after deliberating thereon,
In that resolution the Court "call[ed] to mind that a basic has Resolved to accept and adopt the same.
postulate of the Integrated Bar of the Philippines (IBP), heavily
stressed at the time of its organization and commencement of
existence, is that the IBP shall be non-political in character III. PROHIBITED ACTS AND PRACTICES UNDER IBP BY-
and that there shall be no lobbying nor campaigning in the LAWS.
choice of members of the Board of Governors and of the
House of Delegates, and of the IBP officers, national, or Article I, Section 4 of the IBP By-Laws emphasizes the "strictly
regional, or chapter. The fundamental assumption was that non-political" character of the Integrated Bar of the
officers, delegates and governors would be chosen on the Philippines, thus:
basis of professional merit and willingness and ability to
serve." "SEC. 4. Non-political Bar. — The Integrated Bar is strictly
non-political, and every activity tending to impair this basic
The resolution went on to say that the "Court is deeply feature is strictly prohibited and shall be penalized
disturbed to note that in connection with the election of accordingly. No lawyer holding an elective, judicial, quasi-
members of the Board of Governors and of the House of judicial, or prosecutory office in the Government or any
Delegates, there is a widespread belief, based on reports political subdivision or instrumentality thereof shall be eligible
carried by media and transmitted as well by word of mouth, for election or appointment to any position in the Integrated
that there was extensive and intensive campaigning by Bar or any Chapter thereof. A Delegate, Governor, officer or
candidates for IBP positions as well as expenditure of employee of the Integrated Bar, or an officer or employee of
considerable sums of money by candidates, including vote- any Chapter thereof shall be considered ipso facto resigned
buying, direct or indirect." from his position as of the moment he files his certificate of
candidacy for any elective public office or accepts
The venerable retired Supreme Court Justice and IBP appointment to any judicial, quasi-judicial, or prosecutory
President Emeritus, Jose B.L. Reyes, attended the dialogue, office in the Government or any political subdivision or
upon invitation of the Court, to give counsel and advice. The instrumentality thereof. "'
meeting between the Court en banc on the one hand, and the
outgoing and in coming IBP officers on the other, was an Section 14 of the same By-Laws enumerates the prohibited
informal one. Thereafter, the Court resolved to conduct a acts relative to IBP elections:
formal inquiry to determine whether the prohibited acts and
activities enumerated in the IBP By-Laws were committed SEC. 14. Prohibited acts and practices relative to elections. —
before and during the 1989 elections of IBP's national officers. The following acts and practices relative to election are
prohibited, whether committed by a candidate for any elective
The Court en banc formed a committee and designated Senior office in the Integrated Bar or by any other member, directly
Associate Justice Andres R. Narvasa, as Chairman, and or indirectly, in any form or manner, by himself or through
Associate Justices Teodoro R. Padilla, Emilio A. Gancayco, another person:
Abraham F. Sarmiento, and Carolina C. Griño-Aquino, as
members, to conduct the inquiry. The Clerk of Court, Atty. (a) Distribution, except on election day, of election campaign
Daniel Martinez, acted as the committee's Recording material;
Secretary.
(b) Distribution, on election day, of election campaign material
A total of forty-nine (49) witnesses appeared and testified in other than a statement of the biodata of a candidate on not
response to subpoenas issued by the Court to shed light on more than one page of a legal-size sheet of paper; or causing
the conduct of the elections. The managers of three five-star distribution of such statement to be done by persons other
hotels the Philippine Plaza, the Hyatt, and the Holiday Inn than those authorized by the officer presiding at the elections;
where the three protagonists (Drilon, Nisce and Paculdo)
allegedly set up their respective headquarters and where they
billeted their supporters were summoned. The officer of the (c) Campaigning for or against any candidate, while holding
Philippine National Bank and the Air Transport Office were an elective, judicial, quasi-judicial or prosecutory office in the
called to enlighten the Court on the charge that an IBP Government or any political subdivision, agency or
presidential candidate and the members of her slate used instrumentality thereof;
FINALS CONSTITUTIONAL LAW I ACJUCO NOV 11, 2017 208

(d) Formation of tickets, single slates, or combinations of Among those who signed the nomination forms were: Onofre
candidates, as well as the advertisement thereof; P. Tejada, Candido P. Balbin, Jr., Conizado V. Posadas,
Quirico L. Quirico Ernesto S. Salun-at, Gloria C. Agunos,
(e) For the purpose of inducing or influencing a member to Oscar B. Bernardo, Feliciano F. Wycoco, Amor L. Ibarra, Jose
withhold his vote, or to vote for or against a candidate, (1) M. Atienza, Jose N. Contreras, Romeo T. Mendoza, Leo C.
payment of the dues or other indebtedness of any member; Medialdea, Jr., Paulino G. Clarin, Julius Z. Neil, Roem J.
(2) giving of food, drink, entertainment, transportation or any Arbolado Democrito M. Perez, Abelardo Fermin, Diosdado B.
article of value, or any similar consideration to any person; or Villarin, Jr., Daniel C. Macaraeg, Confesor R. Sansano
(3) making a promise or causing an expenditure to be made, Dionisio E. Bala, Jr., Emesto A. Amores, Romeo V. Pefianco,
offered or promised to any person." Augurio C. Pamintuan, Atlee T. Viray, Ceferino C. Cabanas,
Jose S. Buban, Diosdado Z. Reloj, Jr., Cesar C. Viola, Oscar
C. Fernandez, Ricardo B. Teruel Rodrigo R. Flores, Sixto
Section 12(d) of the By-Laws prescribes sanctions for Marella, Jr., Arsenio C. Villalon, Renato F. Ronquillo, Antonio
violations of the above rules: G. Nalapo Romualdo A. Din Jr., Jose P. Icaonapo Jr., and
Manuel S. Person.
(d) Any violation of the rules governing elections or
commission of any of the prohibited acts and practices defined Atty. Nisce admitted that he reserved rooms at the Hyatt Hotel
in Section 14 prohibited Acts and Practices relative to based on the commitments he had obtained (t.s.n., June 29,
elections) of the by-laws of the Integrated Bar shall be a 1989, pp. 82-85). Unfortunately, despite those formal
ground for the disqualification of a candidate or his removal commitments, he obtained only 14 votes in the election (t.s.n.,
from office if elected, without prejudice to the imposition of June 29, 1 989, p. 86). The reason, he said, is that. some of
sanctions upon any erring member pursuant to the By-laws of those who had committed their votes to him were
the Integrated Bar. "manipulated, intimidated, pressured, or remunerated" (t.s.n.,
June 29,1989, pp. 8695; Exhibit "M-4-Nisce," t.s.n., July 4,
At the formal investigation which was conducted by the 1989, pp. 100-1 04).
investigating committee, the following violations were
established: (2) Use of PNB plane in the campaign.

(1) Prohibited campaigning and solicitation of votes by the The records of the Philippine National Bank (Exhibit C-1-
candidates for president, executive vice-president, the officers Crudo and Exhibit C-2-Crudo) show that Secretary Fulgencio
of candidate the House of Delegates and Board of Governors. S. Factoran, Jr. of the Department of Environment & Natural
Resources (DENR) borrowed a plane from the Philippine
The three candidates for IBP President Drilon, Nisce and National Bank for his Bicol CORD (Cabinet Officers for
Paculdo began travelling around the country to solicit the Regional Development) Assistant, Undersecretary Antonio
votes of delegates as early as April 1989. Upon the invitation Tria. The plane manifest (Exh. C-2-Crudo) listed Atty. Violeta
of IBP President, Leon Garcia, Jr. (t.s.n., July 13,1989, p. 4), Drilon, Arturo Tusi (Tiu), Assistant Secretary for Environment
they attended the Bench and Bar dialogues held in Cotabato and Natural Resources (DENR) Tony Tria, Atty. Gladys
in April 1989 (t.s.n., June 29, 1989, p. 123), in Tagaytay City, Tiongco, and Amy Wong. Except for Tony Tria, the rest of the
Pampanga, and in Baguio City (during the conference of passengers were IBP candidates.
chapter presidents of Northern Luzon (t.s.n., July 3,1989, p.
113; t.s.n., July 10, p. 41; t.s.n., July 13, p. 47) where they Atty. Drilon admitted that she "hitched" a ride on a PNB plane.
announced their candidacies and met the chapter presidents. She said that she was informed by Atty. Tiu about the
availability of a PNB plane (t.s.n., July 3,1989, pp. 116-118).
Atty. Nisce admitted that he went around the country seeking
the help of IBP chapter officers, soliciting their votes, and Atty. Tiu, who ran for the position of IBP executive vice-
securing their written endorsements. He personally hand- president in the Drilon ticket, testified that sometime in May
carried nomination forms and requested the chapter 1989 he failed to obtain booking from the Philippine Airlines
presidents and delegates to fill up and sign the forms to for the projected trip of his group to Bicol. He went to the
formalize their commitment to his nomination for IBP DENR allegedly to follow up some papers for a client. While
President. He started campaigning and distributing the at the DENR, he learned that Assistant Secretary Tria was
nomination forms in March 1989 after the chapter elections going on an official business in Bicol for Secretary Fulgencio
which determined the membership of the House of Delegates Factoran and that he would be taking a PNB plane. As
composed of the 120 chapter presidents (t.s.n., June 29, Assistant Secretary Tria is his fraternity brother, he asked if
1989, pp. 82-86). He obtained forty (40) commitments. He he, together with the Drilon group, could hitch a ride on the
submitted photocopies of his nomination forms which read: plane to Bicol. His request was granted. Their purpose in
going to Bicol was to assess their chances in the IBP
"Nomination Form elections. The Drilon company talked with the IBP chapter
presidents in Daet, Naga, and Legaspi, and asked for their
I Join in Nominating RAMON M. NISCE as support (t.s.n., July 10, 1989, pp. 549).

National President of the Integrated Bar of the Philippines Assistant Secretary Antonio S. Tria confirmed the use of a
PNB plane by Atty. Drilon and her group. He recalled that on
May 23,1989, DENR Secretary Factoran instructed him to go
to Bicol to monitor certain regional development projects there
and to survey the effect of the typhoon that hit the region in
______________ _______________ the middle of May. On the same day, Atty. Tiu, a fraternity
brother (meaning that Tiu belongs to the Sigma Rho fraternity)
went to the DENR office and requested the Secretary
Chapter Signature"
(Factoran) if he (Tiu) could be allowed to hitch a ride on the
plane. Assistant Secretary Tria, together with the Drilon group
FINALS CONSTITUTIONAL LAW I ACJUCO NOV 11, 2017 209

which included Attorneys Drilon, Grapilon, Amy Wong, Gladys (5) Giving free hotel accommodations, food, drinks,
Tiongco, and Tiu, took off at the Domestic Airport bound for entertainment to delegates.
Naga, Daet and Legaspi. In Legaspi the Drilon group had
lunch with Atty. Vicente Real, Jr., an IBP chapter president (a) ATTY. NEREO PACULDO
(t.s.n., July 10, 1989, pp. 54-69).
Atty. Paculdo alleged that he booked 24 regular rooms and
(3) Formation of tickets and single slates. three suites at the Holiday Inn, which served as his
headquarters. The 24 rooms were to be occupied by his staff
The three candidates, Paculdo, Nisce and Drilon, admitted (mostly ladies) and the IBP delegates. The three suites were
having formed their own slates for the election of IBP national to be occupied by himself, the officers of the Capitol Bar
officers on June 3, 1989. Association, and Atty. Mario Jalandoni. He paid P150,000 for
the hotel bills of his delegates at the Holiday Inn, where a room
Atty. Paculdo's slate consisted of — himself for President; cost P990 per day with breakfast.
Bella D. Tiro, for Executive Vice-President; and for Governors:
Justiniano P. Cortez (Northern Luzon), Oscar C. Fernandez Those listed as guests of Atty. Paculdo at the Holiday Inn
(Central Luzon), Mario C.V. Jalandoni (Greater Manila), were: Emesto C. Perez, Tolomeo Ligutan Judge Alfonso
Petronilo A. de la Cruz (Southern Luzon), Teodorico C. Combong, Ricardo Caliwag, Antonio Bisnar, Benedicto
Almine, Jr. (Bicolandia), Ricardo B. Teruel (Western Visayas), Balajadia, Jesus Castro, Restituto Villanueva, Serapio Cribe
Porfirio P. Siyangco (Eastern Visayas), Jesus S. Anonat Juanita Subia, Teodorico J. Almine, Rudy Gumban, Roem
(Western Mindanao), Guerrero A. Adaza, Jr. (Eastern Arbolado, Ricardo Teruel, Shirley Moises, Ramon Roco,
Mindanao) (Exhibit M-Nisce). Alberto Trinidad, Teodoro Quicoy Manito Lucero, Fred
Cledera Vicente Tordilla, Julian Ocampo, Francisco
The Drilon ticket consisted of. Violeta C. Drilon for President, Felizmenio Marvel Clavecilla, Amador Capiral, Eufronio
Arturo Tiu for Executive Vice President, Salvador Lao for Maristela, Porfirio Siyangco, William Llanes, Jr., Marciano
Chairman of the House of Delegates, and, for Governors: Neri, Guerrero Adaza, Diosdado Peralta, Luis C. Formilleza,
Basil Rupisan (Northern 'Luzon), Acong Atienza (Central Jr., Democrito Perez, Bruno Flores, Dennis Rendon, Judge
Luzon), Amy Wong (Metro Manila), Jose Grapilon (Southern Ceferino Chan, Mario Jalandoni, Kenneth Siruelo Bella Tiro,
Tagalog), Teodoro Almine (Bicolandia), Baldomero Estenzo Antonio Santos, Tiburcio Edano James Tan, Cesilo A. Adaza,
(Eastern Visayas), Joelito Barrera (Western Visayas), Gladys Francisco Roxas, Angelita Gacutan, Jesse Pimentel, Judge
Tiongco (Eastern Mindanao), Simeon Datumanong (Western Jaime Hamoy, Jesus Anonat, Carlos Egay, Judge Carlito
Mindanao) (Exhibit M-1-Nisce). Eisma, Judge Jesus Carbon, Joven Zach, and Benjamin
Padon.
Atty. Ramon N. Nisce's line-up listed himself and Confessor
B. Sansano Benjamin B. Bernardino, Antonio L. Nalapo Noel de Guzman, Holiday Inn's credit manager, testified that
Renato F. Ronquillo, Gloria C. Agunos, Mario Valderrama, Atty. Paculdo booked 52 (not 24) rooms, including the
Candido P. Balbin Jr., Oscar C. Fernandez, Cesar G. Viola, presidential suite, which was used as the Secretariat. The
Leo C. Medialdea, Jr., Vicente P. Tordilla, Jr., Jose S. Buban, group bookings were made by Atty. Gloria Paculdo, the wife
Joel A. Llosa, Jesus T. Albacite and Oscar V. Badelles. of Nereo Paculdo (t.s.n. June 28, 1989, pp. 63-68). The total
sum of P227,114.89 was paid to Holiday Inn for the use of the
rooms.
(4) Giving free transportation to out-of-town delegates and
alternates.
(b) ATTY. VIOLETA C. DRILON
Atty. Nisce admitted having bought plane tickets for some
delegates to the convention. He mentioned Oscar Badelles to The delegates and supporters of Atty. Drilon were billeted at
whom he gave four round-trip tickets (worth about P10,000) the Philippine Plaza Hotel where her campaign manager, Atty.
from Iligan City to Manila and back. Badelles was a voting Renato Callanta, booked 40 rooms, 5 of which were suites.
delegate. Nisce, however, failed to get a written commitment According to Ms. Villanueva, Philippine Plaza banquet and
from him because Atty. Medialdea assured him (Nisce) conventions manager, the contract that Atty. Callanta signed
"sigurado na 'yan, h'wag mo nang papirmahin." Badelles won with the Philippine Plaza was made in the name of the "IBP
as sergeant-at-arms, not in Nisce's ticket, but in that of Drilon. c/o Atty. Callanta."

Badelles admitted that Nisce sent him three airplane tickets, Mrs. Lourdes Juco, a sales manager of the Philippine Plaza,
but he Badelles said that he did not use them, because if he recalled that it was Mr. Mariano Benedicto who first came to
did, he would be committed to Nisce, and he Badelles did not book rooms for the IBP delegates. She suggested that he
want to be committed (t.s.n., July 4,1989, pp. 77-79, 95-96). obtain a group (or discounted) rate. He gave her the name of
Atty. Callanta who would make the arrangements with her. Mr.
Benedicto turned out to be the Assistant Secretary of the
Nisce also sent a plane ticket to Atty. Atilano, who was his Department of Labor and Employment (DOLE).
candidate, and another ticket to Mrs. Linda Lim of
Zamboanga. Records of the Philippine Airlines showed that
Atty. Nisce paid for the plane tickets of Vicente Real, Jr. (Exh. The total sum of P316,411.53 was paid by Atty. Callanta for
D-1-Calica), Romeo Fortes (Exh. D-1-Calica), Cesar Batica the rooms, food, and beverages consumed by the Drilon
(Exh. D-2-Calica), Jose Buban of Leyte (Exh. D-2-Calica), group, with an unpaid balance of P302,197.30. Per Attorney
Delsanto Resuello (Exh. D-3- Calica), and Ceferino Cabanas Daniel Martinez's last telephone conversation with Ms.
(Exh. D-3-Calica). Villanueva, Atty. Callanta still has an outstanding account of
P232,782.65 at Philippine Plaza.
In spite of his efforts and expense, only one of Nisce's
candidates won: Renato Ronquillo of Manila 4, as Secretary Atty. Callanta admitted that he signed the contract for 40
of the House of Delegates (t.s.n. July 3, p. 161). rooms at the Philippine Plaza. He made a downpayment of
FINALS CONSTITUTIONAL LAW I ACJUCO NOV 11, 2017 210

P123,000. His "working sheet' showed that the following Atty. Callanta to whom he paid P20,000 (t.s.n. July 6,1989,
persons contributed for that down payment: pp. 30-34).

(a) Nilo Pena (Quasha Law Office) P 25,000 Atty. Carpio assisted Atty. Drilon in her campaign during the
convention, by soliciting the votes of delegates he knew, like
(b) Antonio Carpio 20,000 Atty. Albacite his former teacher (but the latter was already
committed to Nisce), and Atty. Romy Fortes, a classmate of
his in the U.P. College of Law (t. t.s.n. July 6, 1989, pp. 22,
(c) Toto Ferrer (Carpio Law Office) 10,000 29, 39).

(d) Jay Castro 10,000 (c) ATTY. RAMON NISCE.

(e) Danny Deen 20,000 Atty. Nisce, through his brother-in-law, Ricardo Paras, entered
into a contract with the Hyatt Hotel for a total of 29 rooms plus
(f) Angangco Tan (Angara Law Office) 10,000 one (1) seventh-floor room. He made a downpayment of
P20,000 (t.s.n. June 28, 1989, p. 58) on April 20, 1989, and
(g) Alfonso Reyno 20,000 P37,632.45 on May 10, or a total of P57,632.45.

(h) Cosme Rossel 15,300 Ms. Cecile Flores, Ms. Milagros Ocampo, and Mr. Ramon
Jacinto, the sales department manager, credit manager, and
reservation manager, respectively of the Hyatt, testified that
(t.s.n. July 4, 1 989, pp. 3-4) Atty. Nisce's bill amounted to P216,127.74 (t.s.n. June 28,
1989, pp. 57-58; Exhibits E-Flores, F-Jacinto G-Ocampo).
Atty. Callanta explained that the above listed persons have
been contributing money every time the IBP embarks on a As earlier mentioned, Atty. Nisce admitted that he reserved
project. This time, they contributed so that their partners or rooms for those who committed themselves to his candidacy.
associates could attend the legal aid seminar and the IBP
convention too.
The hotel guests of Atty. Nisce were: Gloria Agunos Dennis
Habanel B. Batula, John E. Asuncion, Reynaldo Cortes,
Atty. Drilon alleged that she did not know that Atty. Callanta Lourdes Santos, Elmer Datuin, Romualdo Din, Antonio
had billeted her delegates at the Philippine Plaza. She Nalapo, Israel Damasco, Candido Balbin, Serrano Balot,
allegedly did not also know in whose name the room she Ibarra, Joel Llosa, Eltanal, Ruperto, Asuncion, Q. Pilotin
occupied was registered. But she did ask for a room where Reymundo P. Guzman, Zoilo Aguinaldo, Clarin, R. Ronquillo,
she could rest during the convention. She admitted, however, Dominador Carillo, Filomeno Balinas, Ernesto Sabulan,
that she paid for her hotel room and meals to Atty. Callanta, Yusop Pangadapun, A. Viray, Icampo, Abelardo Fermin, C.
through Atty. Loanzon (t.s.n. July 3,1989). Quiaoit, Augurio Pamintuan, Daniel Macaraeg, Onofre
Tejada.
The following were listed as having occupied the rooms
reserved by Atty. Callanta at the Philippine Plaza: Violeta (6) Campaigning by labor officials for Atty. Violeta Drilon
Drilon, Victoria A. Verciles, Victoria C. Loanzon, Leopoldo A.
Consulto Ador Lao, Victoria Borra, Aimee Wong, Callanta,
Pena, Tiu, Gallardo, Acong Atienza, D. Bernardo, Amores, In violation of the prohibition against "campaigning for or
Silao Caingat, Manuel Yuson, Simeon Datumanong, Manuel against a candidate while holding an elective, judicial, quasi-
Pecson, Sixto Marella, Joselito Barrera, Radon Macalalag, judicial, or prosecutory office in the Government' (Sec. 14[c],
Oscar Badelles, Antonio Acyatan, Ildefonso C. Puerto, Nestor Art. I, IBP By-Laws), Mariano E. Benedicto II, Assistant
Atienza, Gil Batula Array Corot, Dimakuta Corot Romeo Secretary, Department of Labor and Employment, testified
Fortes Irving Petilla, Teodoro Palma, Gil Palma, Danilo Deen, that he took a leave of absence from his office to attend the
Delsanto, Resuello, Araneta, Vicente Real, Sylvio Casuncad IBP convention. He stayed at the Philippine Plaza with the
Espina, Guerrero, Julius Neri, Linda Lim, Ben Lim, C. Batica, Drilon group admittedly to give "some moral assistance" to
Luis Formilleza, Felix Macalag Mariano Benedicto, Atilano, Atty. Violeta Drilon. He did so because he is a member of the
Araneta, Renato Callanta. Sigma Rho Fraternity. When asked about the significance of
Sigma Rho, Secretary Benedicto explained: "More than the
husband of Mrs. Drilon being my boss, the significance there
Atty. Nilo Pena admitted that the Quasha Law Office of which is that the husband is my brother in the Sigma Rho."
he is a senior partner, gave P25,000 to Callanta for rooms at
the Philippine Plaza so that some members of his law firm
could campaign for the Drilon group (t.s.n. July 5,1989, pp. He cheered up Mrs., Drilon when her spirits were low. He
7678) during the legal aid seminar and the IBP convention. talked to her immediate circle which included Art Tiu, Tony
Most of the members of his law firm are fraternity brothers of Carpio, Nilo Pena, Amy Wong, Atty. Grapilon, Victor Lazatin,
Secretary Drilon (meaning, members of the Sigma Rho and Boy Reyno. They assessed the progress of the campaign,
Fraternity). He admitted being sympathetic to the candidacy and measured the strengths and weaknesses of the other
of Atty. Drilon and the members of her slate, two of whom Jose groups The group had sessions as early as the later part of
Grapilon and Simeon Datumanong — are Sigma Rhoans. May.
They consider Atty. Drilon as a "sigma rho sister," her
husband being a sigma rhoan. Room 114, the suite listed in the name of Assistant Secretary
Benedicto toted up a bill of P23,110 during the 2-day IBP
Atty. Antonio Carpio, also a Sigma Rhoan, reserved a room convention/election. A total of 113 phone calls (amounting to
for the members of his own firm who attended the legal aid Pl,356) were recorded as emanating from his room.
seminar and the convention. He made the reservation through
FINALS CONSTITUTIONAL LAW I ACJUCO NOV 11, 2017 211

Opposite Room 114, was Room 112, also a suite, listed in the Atty. Gloria Agunos personnel director of the Hyatt Terraces
names of Mrs. Drilon, Gladys Tiongco (candidate for Hotel in Baguio and president of the Baguio-Benguet IBP
Governor, Eastern Mindanao) and Amy Wong (candidate for Chapter, recalled that in the third week of May 1989, after the
Governor, Metro Manila). These two rooms served as the Tripartite meet of the Department of Labor & Employment at
"action center' or "war room" where campaign strategies were the Green Valley Country Club in Baguio City, she met Atty.
discussed before and during the convention. It was in these Drilon, together with two labor officers of Region 1, Attys.
rooms where the supporters of the Drilon group, like Attys. Filomeno Balbin and Atty. Mansala Atty. Drilon solicited her
Carpio, Callanta, Benedicto, the Quasha and the ACCRA (Atty. Agunos') vote and invited her to stay at the Philippine
lawyers met to plot their moves. Plaza where a room would be available for her. Atty. Paculdo
also tried to enlist her support during the chapter presidents'
(7) Paying the dues or other indebtedness of any number meeting to choose their nominee for governor for the Northern
(Sec. 14[e], IBP BY-Laws). Luzon region (t.s.n. July 13,1989, pp. 43-54).

Atty. Teresita C. Sison, IBP Treasurer, testified that she has Atty. Nisce testified that a Manila Chapter 4 delegate, Marcial
heard of candidates paying the IBP dues of lawyers who Magsino, who had earlier committed his vote to Nisce
promised to vote for or support them, but she has no way of changed his mind when he was offered a judgeship (This
ascertaining whether it was a candidate who paid the statement, however, is admittedly hearsay). When Nisce
delinquent dues of another, because the receipts are issued confronted Magsino about the alleged offer, the latter denied
in the name of the member for whom payment is made (t.s.n. that there was such an offer. Nisce's informant was Antonio
June 28, 1989, pp. 24-28). G. Nalapo an IBP candidate who also withdrew.

She has noticed, though, that there is an upsurge of payments Another Nisce candidate, Cesar Viola, withdrew from the race
in March, April, May during any election year. This year, the and refused to be nominated (t.s.n. June 29, 1989, p. 104).
collections increased by P100,000 over that of last year (a Vicente P. Tordilla who was Nisce's candidate for Governor
non-election year from Pl,413,425 to Pl,524,875 (t.s.n. June became Paculdo's candidate instead (t.s.n. June 29, 1989, p.
28, 1989, p. 25). 104).

(8) Distribution of materials other than bio-data of not more Nisce recalled that during the Bench and Bar Dialogue in
than one page of legal size sheet of paper (Sec. 14[a], IBP By- Cotabato City, Court Administrator Tiro went around saying, "I
Laws). am not campaigning, but my wife is a candidate." Nisce said
that the presidents of several IBP chapters informed him that
labor officials were campaigning for Mrs. Drilon (t.s.n. June
On the convention floor on the day of the election, Atty. 29,1989, pp. 109-110). He mentioned Ciony de la Cerna, who
Paculdo caused to be distributed his bio-data and copies of a allegedly campaigned in La Union (t.s.n. June 29,1989,p.111)
leaflet entitled "My Quest," as wen as, the lists of his slate.
Attys. Drilon and Nisce similarly distributed their tickets and
bio-data. Atty. Joel A. Llosa, Nisce's supporter and candidate for
governor of the Western Visayas, expressed his
disappointment over the IBP elections because some
The campaign materials of Atty. Paculdo cost from P15,000 to delegates flip-flopped from one camp to another. He testified
P20,000. They were printed by his own printing shop. that when he arrived at the Manila Domestic Airport he was
met by an assistant regional director of the DOLE who offered
(9) Causing distribution of such statement to be done by to bring him to the Philippine Plaza, but he declined the offer.
persons other than those authorized by the officer presiding at During the legal aid seminar, Atty. Drilon invited him to transfer
the election (Sec. 14[b], IBP By-Laws). to the Philippine Plaza where a room had been reserved for
him. He declined the invitation (t.s.n. July 4,1989, pp. 102-
Atty. Paculdo employed uniformed girls to distribute his 106).
campaign materials on the convention floor. Atty. Carpio noted
that there were more campaign materials distributed at the Atty. Llosa said that while he was still in Dumaguete City, he
convention site this year than in previous years. The election already knew that the three candidates had their headquarters
was more heated and expensive (t.s.n. July 6,1989, p. 39). in separate hotels: Paculdo, at the Holiday Inn; Drilon, at the
Philippine Plaza; and Nisce, at the Hyatt. He knew about this
Atty. Benjamin Bernardino, the incumbent President of the IBP because a week before the elections, representatives of Atty.
Rizal Chapter, and a candidate for chairman of the House of Drilon went to Dumaguete City to campaign. He mentioned
Delegates on Nisce's ticket, testified that campaign materials Atty. Rodil Montebon of the ACCRA Law Office, accompanied
were distributed during the convention by girls and by lawyers. by Atty. Julve the Assistant Regional Director of the
He saw members of the ACCRA law firm campaigning for Atty. Department of Labor in Dumaguete City. These two, he said,
Drilon (t.s.n. July 3,1989, pp. 142-145). offered to give him two PAL tickets and accommodations at
the Philippine Plaza (t.s.n. July 4,1989, pp. 101-104). But he
declined the offer because he was already committed to Atty.
(10) Inducing or influencing a member to withhold his vote, or Nisce.
to vote for or against a candidate (Sec. 14[e], IBP BY-Laws).
Atty. Llosa also revealed that before he left for Manila on May
Atty. Bernardino disclosed that his cousin, Atty. Romeo 31, 1989, a businessman, Henry Dy, approached him to
Capulong, urged him to withdraw his candidacy for chairman convince him to vote for Atty. Paculdo. But Llosa told Dy that
of the House of Delegates and to run as vice-chairman in Violy he was already committed to Nisce.
Drilon's slate, but he declined (t.s.n. July 3,1989, pp. 137,
149).
He did not receive any plane tickets from Atty. Nisce because
he and his two companions (Atty. Eltanal and Atty. Ruperto)
FINALS CONSTITUTIONAL LAW I ACJUCO NOV 11, 2017 212

had earlier bought their own tickets for Manila (t.s.n. July 4, families who accompanied them) in exchange for their
1989, p. 101). support; the pirating of some candidates by inducing
them to "hop" or "flipflop" from one ticket to another for
SUMMARY OF CAMPAIGN EXPENSES INCURRED BY some rumored consideration; all these practices made a
THE CANDIDATES political circus of the proceedings and tainted the whole
election process.
Atty. Paculdo admitted having spent some P250,000
during his three weeks of campaigning. Of this amount, the The candidates and many of the participants in that
Capitol Bar Association (of which he was the chapter election not only violated the By-Laws of the IBP but also
president) contributed about P150,000. The Capitol Bar the ethics of the legal profession which imposes on all
Association is a voluntary bar association composed of lawyers, as a corollary of their obligation to obey and
Quezon City lawyers. uphold the constitution and the laws, the duty to
"promote respect for law and legal processes" and to
abstain from 'activities aimed at defiance of the law or at
He spent about P100,000 to defray the expenses of his lessening confidence in the legal system" (Rule 1.02,
trips to the provinces (Bicol provinces, Pampanga, Abra, Canon 1, Code of Professional Responsibility). Respect for
Mountain Province and Bulacan) (t.s.n. June 29,1989, pp. law is gravely eroded when lawyers themselves, who are
9-14). supposed to be millions of the law, engage in unlawful
practices and cavalierly brush aside the very rules that the IBP
Atty. Nisce's hotel bills at the Hyatt amounted to formulated for their observance.
P216,127.74. This does not include the expenses for his
campaign which began several months before the June The unseemly ardor with which the candidates pursued the
3rd election, and his purchases of airplane tickets for presidency of the association detracted from the dignity of the
some delegates. legal profession. The spectacle of lawyers bribing or being
bribed to vote one way or another, certainly did not uphold the
The records of the Philippine Plaza Hotel, headquarters of honor of the profession nor elevate it in the public's esteem.
Atty. Drilon's camp, showed that her campaign rang up
over P600,000 in hotel bills. Atty. Callanta paid The Court notes with grave concern what appear to be the
P316,411.53 for the rooms, food, and beverage consumed evasions, denials and outright prevarications that tainted the
by Atty. Drilon's supporters, but still left an unpaid bill of statements of the witnesses, including tome of the candidates,
P302,197.30 at convention's end. during the initial hearing conducted by it before its fact-finding
committee was created. The subsequent investigation
FINDINGS. conducted by this Committee has revealed that those parties
had been less than candid with the Court and seem to have
From all the foregoing, it is evident that the manner in conspired among themselves to deceive it or at least withhold
which the principal candidates for the national positions vital information from it to conceal the irregularities committed
in the Integrated Bar conducted their campaign during the campaign.
preparatory to the elections on June 3, 1989, violated
Section 14 of the IBP By-Laws and made a travesty of the CONCLUSIONS.
idea of a "strictly non-political" Integrated Bar enshrined
in Section 4 of the By-Laws. It has been mentioned with no little insistence that the
provision in the 1987 Constitution (See. 8, Art. VIII) providing
The setting up of campaign headquarters by the three for a Judicial and Bar Council composed of seven (7)
principal candidates (Drilon, Nisce and Paculdo) in five- members among whom is "a representative of the Integrated
star hotels: The Philippine Plaza, the Holiday Inn and The Bar," tasked to participate in the selection of nominees for
Hyatt the better for them to corral and entertain the appointment to vacant positions in the judiciary, may be the
delegates billeted therein; the island hopping to solicit reason why the position of IBP president has attracted so
the votes of the chapter presidents who comprise the 120- much interest among the lawyers. The much coveted "power"
member House of Delegates that elects the national erroneously perceived to be inherent in that office might have
officers and regional governors; the formation of tickets, caused the corruption of the IBP elections. To impress upon
slates, or line-ups of candidates for the other elective the participants in that electoral exercise the seriousness of
positions aligned with, or supporting, either Drilon, the misconduct which attended it and the stern disapproval
Paculdo or Nisce; the procurement of written with which it is viewed by this Court, and to restore the non-
commitments and the distribution of nomination forms to political character of the IBP and reduce, if not entirely
be filled up by the delegates; the reservation of rooms for eliminate, expensive electioneering for the top positions in the
delegates in three big hotels, at the expense of the organization which, as the recently concluded elections
presidential candidates; the use of a PNB plane by Drilon revealed, spawned unethical practices which seriously
and some members of her ticket to enable them to diminished the stature of the IBP as an association of the
"assess their chances" among the chapter presidents in practitioners of a noble and honored profession, the Court
the Bicol provinces; the printing and distribution of hereby ORDERS:
tickets and bio-data of the candidates which in the case
of Paculdo admittedly cost him some P15,000 to P20,000; 1. The IBP elections held on June3,1989 should be as they
the employment of uniformed girls (by Paculdo) and are hereby annulled.
lawyers (by Drilon) to distribute their campaign materials
on the convention floor on the day of the election; the
giving of assistance by the Undersecretary of Labor to 2. The provisions of the IBP By-Laws for the direct election by
Mrs. Drilon and her group; the use of labor arbiters to the House of Delegates (approved by this Court in its
meet delegates at the airport and escort them to the resolution of July 9, 1985 in Bar Matter No. 287) of the
Philippine Plaza Hotel; the giving of pre-paid plane tickets following national officers:
and hotel accommodations to delegates (and some
FINALS CONSTITUTIONAL LAW I ACJUCO NOV 11, 2017 213

(a) the officers of the House of Delegates; 9. Section 39, Article V is hereby amended as follows:

(b) the IBP president; and Section 39. Nomination and election of the Governors at least
one (1) month before the national convention the delegates
(c) the executive vice-president, from each region shall elect the governor for their region, the
choice of which shall as much as possible be rotated among
the chapters in the region.
be repealed, this Court being empowered to amend, modify or
repeal the By-Laws of the IBP under Section 77, Art. XI of said
By-Laws. 10. Section33(a), Article V hereby is amended by addingthe
following provision as part of the first paragraph:
3. The former system of having the IBP President and
Executive Vice-President elected by the Board of Governors No convention of the House of Delegates nor of the general
(composed of the governors of the nine [91 IBP regions) from membership shall be held prior to any election in an election
among themselves (as provided in Sec. 47, Art. VII, Original year.
IBP By-Laws) should be restored. The right of automatic
succession by the Executive Vice-President to the presidency 11. Section 39, (a), (b), (1), (2), (3), (4), (5), (6), and (7) of
upon the expiration of their two-year term (which was Article VI should be as they are hereby deleted.
abolished by this Court's resolution dated July 9,1985 in Bar
Matter No. 287) should be as it is hereby restored. All other provisions of the By-Laws including its amendment
by the Resolution en banc of this Court of July 9, 1985 (Bar
4. At the end of the President's two-year term, the Executive Matter No. 287) that are inconsistent herewith are hereby
Vice-President shall automatically succeed to the office of repealed or modified.
president. The incoming board of governors shall then elect
an Executive Vice-President from among themselves. The 12. Special elections for the Board of Governors shall be held
position of Executive Vice-President shall be rotated among in the nine (9) IBP regions within three (3) months, after the
the nine (9) IBP regions. One who has served as president promulgation of the Court's resolution in this case. Within thirty
may not run for election as Executive Vice-President in a (30) days thereafter, the Board of Governors shall meet at the
succeeding election until after the rotation of the presidency IBP Central Office in Manila to elect from among themselves
among the nine (9) regions shall have been completed; the IBP national president and executive vice-president. In
whereupon, the rotation shall begin anew. these special elections, the candidates in the election of the
national officers held on June 3,1989, particularly identified in
5. Section 47 of Article VII is hereby amended to read as Sub-Head 3 of this Resolution entitled "Formation of Tickets
follows: and Single Slates," as well as those identified in this
Resolution as connected with any of the irregularities
Section 47. National Officers. — The Integrated Bar of the attendant upon that election, are ineligible and may not
Philippines shall have a President and Executive Vice- present themselves as candidate for any position.
President to be chosen by the Board of Governors from
among nine (9) regional governors, as much as practicable, 13. Pending such special elections, a caretaker board shall be
on a rotation basis. The governors shall be ex oficio Vice- appointed by the Court to administer the affairs of the IBP. The
President for their respective regions. There shall also be a Court makes clear that the dispositions here made are without
Secretary and Treasurer of the Board of Governors to be prejudice to its adoption in due time of such further and other
appointed by the President with the consent of the Board. measures as are warranted in the premises.

6. Section 33(b), Art. V, IBP By-Laws, is hereby amended as SO ORDERED.


follows:

(b) The President and Executive Vice President of the IBP


shall be the Chairman and Vice-Chairman, respectively, of the
House of Delegates. The Secretary, Treasurer, and Sergeant-
at-Arms shall be appointed by the President with the consent
of the House of Delegates.'

7. Section 33(g) of Article V providing for the positions of


Chairman, Vice-Chairman, Secretary-Treasurer and
Sergeant-at- Arms of the House of Delegates is hereby
repealed

8. Section 37, Article VI is hereby amended to read as follows:

Section 37. Composition of the Board. — The Integrated Bar


of the Philippines shall be governed by a Board of Governors
consisting of nine (9) Governors from the nine (9) regions as
delineated in Section 3 of the Integration Rule, on the
representation basis of one (1) Governor for each region to be
elected by the members of the House of Delegates from that
region only. The position of Governor should be rotated
among the different Chapters in the region.
FINALS CONSTITUTIONAL LAW I ACJUCO NOV 11, 2017 214

G.R. No. L-57883 March 12, 1982 proceeding on the debate on Batas Pambansa Blg. 129, this
petition was deemed submitted for decision.
GUALBERTO J. DE LA LLANA Presiding Judge, Branch II
of the City Court of Olongapo, ESTANISLAO L. CESA, JR., The importance of the crucial question raised called for
FIDELA Y. VARGAS, BENJAMIN C. ESCOLANGO, intensive and rigorous study of all the legal aspects of the
JUANITO C. ATIENZA, MANUEL REYES ROSAPAPAN, case. After such exhaustive deliberation in several sessions,
JR., VIRGILIO E. ACIERTO, and PORFIRIO AGUILLON the exchange of views being supplemented by memoranda
AGUILA, petitioners, from the members of the Court, it is our opinion and so hold
that Batas Pambansa Blg. 129 is not unconstitutional.
vs.
1. The argument as to the lack of standing of petitioners is
MANUEL ALBA, Minister of Budget, FRANCISCO easily resolved. As far as Judge de la Llana is concerned, he
TANTUICO, Chairman, Commission on Audit, and certainly falls within the principle set forth in Justice Laurel's
RICARDO PUNO, Minister of Justice, Respondents. opinion in People v. Vera. 8 Thus: "The unchallenged rule is
that the person who impugns the validity of a statute must
have a personal and substantial interest in the case such that
FERNANDO, C.J.: he has sustained, or will sustain, direct injury as a result of its
enforcement." 9 The other petitioners as members of the bar
This Court, pursuant to its grave responsibility of passing upon and officers of the court cannot be considered as devoid of
the validity of any executive or legislative act in an appropriate "any personal and substantial interest" on the matter. There is
cases, has to resolve the crucial issue of the constitutionality relevance to this excerpt from a separate opinion in Aquino,
of Batas Pambansa Blg. 129, entitled "An act reorganizing the Jr. v. Commission on Elections: 10 "Then there is the attack
Judiciary, Appropriating Funds Therefor and for Other on the standing of petitioners, as vindicating at most what they
Purposes." The task of judicial review, aptly characterized as consider a public right and not protecting their rights as
exacting and delicate, is never more so than when a conceded individuals. This is to conjure the specter of the public right
legislative power, that of judicial reorganization, 1 may dogma as an inhibition to parties intent on keeping public
possibly collide with the time-honored principle of the officials staying on the path of constitutionalism. As was so
independence of the judiciary 2 as protected and safeguarded well put by Jaffe: 'The protection of private rights is an
by this constitutional provision: "The Members of the Supreme essential constituent of public interest and, conversely,
Court and judges of inferior courts shall hold office during without a well-ordered state there could be no enforcement of
good behavior until they reach the age of seventy years or private rights. Private and public interests are, both in
become incapacitated to discharge the duties of their office. substantive and procedural sense, aspects of the totality of
The Supreme Court shall have the power to discipline judges the legal order.' Moreover, petitioners have convincingly
of inferior courts and, by a vote of at least eight Members, shown that in their capacity as taxpayers, their standing to sue
order their dismissal." 3 For the assailed legislation mandates has been amply demonstrated. There would be a retreat from
that Justices and judges of inferior courts from the Court of the liberal approach followed in Pascual v. Secretary of Public
Appeals to municipal circuit courts, except the occupants of Works, foreshadowed by the very decision of People v. Vera
the Sandiganbayan and the Court of Tax Appeals, unless where the doctrine was first fully discussed, if we act
appointed to the inferior courts established by such Act, would differently now. I do not think we are prepared to take that
be considered separated from the judiciary. It is the step. Respondents, however, would hark back to the
termination of their incumbency that for petitioners justifies a American Supreme Court doctrine in Mellon v. Frothingham
suit of this character, it being alleged that thereby the security with their claim that what petitioners possess 'is an interest
of tenure provision of the Constitution has been ignored and which is shared in common by other people and is
disregarded, comparatively so minute and indeterminate as to afford any
basis and assurance that the judicial process can act on it.'
That is the fundamental issue raised in this proceeding, That is to speak in the language of a bygone era even in the
erroneously entitled Petition for Declaratory Relief and/or for United States. For as Chief Justice Warren clearly pointed out
Prohibition 4 considered by this Court as an action for in the later case of Flast v. Cohen, the barrier thus set up if not
prohibited petition, seeking to enjoin respondent Minister of breached has definitely been lowered." 11
the Budget, respondent Chairman of the Commission on
Audit, and respondent Minister of Justice from taking any 2. The imputation of arbitrariness to the legislative body in the
action implementing Batas Pambansa Blg. 129. Petitioners 5 enactment of Batas Pambansa Blg. 129 to demonstrate lack
sought to bolster their claim by imputing lack of good faith in of good faith does manifest violence to the facts. Petitioners
its enactment and characterizing as an undue delegation of should have exercised greater care in informing themselves
legislative power to the President his authority to fix the as to its antecedents. They had laid themselves open to the
compensation and allowances of the Justices and judges accusation of reckless disregard for the truth, On August 7,
thereafter appointed and the determination of the date when 1980, a Presidential Committee on Judicial Reorganization
the reorganization shall be deemed completed. In the very was organized. 12 This Executive Order was later amended
comprehensive and scholarly Answer of Solicitor General by Executive Order No. 619-A., dated September 5 of that
Estelito P. Mendoza, 6 it was pointed out that there is no valid year. It clearly specified the task assigned to it: "1. The
justification for the attack on the constitutionality of this statute, Committee shall formulate plans on the reorganization of the
it being a legitimate exercise of the power vested in the Judiciary which shall be submitted within seventy (70) days
Batasang Pambansa to reorganize the judiciary, the from August 7, 1980 to provide the President sufficient options
allegations of absence of good faith as well as the attack on for the reorganization of the entire Judiciary which shall
the independence of the judiciary being unwarranted and embrace all lower courts, including the Court of Appeals, the
devoid of any support in law. A Supplemental Answer was Courts of First Instance, the City and Municipal Courts, and all
likewise filed on October 8, 1981, followed by a Reply of Special Courts, but excluding the Sandigan Bayan." 13 On
petitioners on October 13. After the hearing in the morning and October 17, 1980, a Report was submitted by such Committee
afternoon of October 15, in which not only petitioners and on Judicial Reorganization. It began with this paragraph: "The
respondents were heard through counsel but also the amici Committee on Judicial Reorganization has the honor to submit
curiae, 7 and thereafter submission of the minutes of the the following Report. It expresses at the outset its appreciation
FINALS CONSTITUTIONAL LAW I ACJUCO NOV 11, 2017 215

for the opportunity accorded it to study ways and means for the courts was vested in it under the 1973 Constitution, the
what today is a basic and urgent need, nothing less than the trend towards more and more cases has continued." 20 It is
restructuring of the judicial system. There are problems, both understandable why. With the accelerated economic
grave and pressing, that call for remedial measures. The felt development, the growth of population, the increasing
necessities of the time, to borrow a phrase from Holmes, admit urbanization, and other similar factors, the judiciary is called
of no delay, for if no step be taken and at the earliest upon much oftener to resolve controversies. Thus confronted
opportunity, it is not too much to say that the people's faith in with what appears to be a crisis situation that calls for a
the administration of justice could be shaken. It is imperative remedy, the Batasang Pambansa had no choice. It had to act,
that there be a greater efficiency in the disposition of cases before the ailment became even worse. Time was of the
and that litigants, especially those of modest means — much essence, and yet it did not hesitate to be duly mindful, as it
more so, the poorest and the humblest — can vindicate their ought to be, of the extent of its coverage before enacting
rights in an expeditious and inexpensive manner. The Batas Pambansa Blg. 129.
rectitude and the fairness in the way the courts operate must
be manifest to all members of the community and particularly 3. There is no denying, therefore, the need for "institutional
to those whose interests are affected by the exercise of their reforms," characterized in the Report as "both pressing and
functions. It is to that task that the Committee addresses itself urgent." 21 It is worth noting, likewise, as therein pointed out,
and hopes that the plans submitted could be a starting point that a major reorganization of such scope, if it were to take
for an institutional reform in the Philippine judiciary. The place, would be the most thorough after four generations. 22
experience of the Supreme Court, which since 1973 has been The reference was to the basic Judiciary Act generations .
empowered to supervise inferior courts, from the Court of enacted in June of 1901, 23 amended in a significant way,
Appeals to the municipal courts, has proven that reliance on only twice previous to the Commonwealth. There was, of
improved court management as well as training of judges for course, the creation of the Court of Appeals in 1935, originally
more efficient administration does not suffice. I hence, to composed "of a Presiding Judge and ten appellate Judges,
repeat, there is need for a major reform in the judicial so stem who shall be appointed by the President of the Philippines,
it is worth noting that it will be the first of its kind since the with the consent of the Commission on Appointments of the
Judiciary Act became effective on June 16, 1901." 14 I t went National Assembly, 24 It could "sit en banc, but it may sit in
to say: "I t does not admit of doubt that the last two decades two divisions, one of six and another of five Judges, to transact
of this century are likely to be attended with problems of even business, and the two divisions may sit at the same time." 25
greater complexity and delicacy. New social interests are Two years after the establishment of independence of the
pressing for recognition in the courts. Groups long inarticulate, Republic of the Philippines, the Judiciary Act of 1948 26 was
primarily those economically underprivileged, have found passed. It continued the existing system of regular inferior
legal spokesmen and are asserting grievances previously courts, namely, the Court of Appeals, Courts of First Instance,
ignored. Fortunately, the judicially has not proved inattentive. 27 the Municipal Courts, at present the City Courts, and the
Its task has thus become even more formidable. For so much Justice of the Peace Courts, now the Municipal Circuit Courts
grist is added to the mills of justice. Moreover, they are and Municipal Courts. The membership of the Court of
likewise to be quite novel. The need for an innovative Appeals has been continuously increased. 28 Under a 1978
approach is thus apparent. The national leadership, as is well- Presidential Decree, there would be forty-five members, a
known, has been constantly on the search for solutions that Presiding Justice and forty-four Associate Justices, with
will prove to be both acceptable and satisfactory. Only thus fifteen divisions. 29 Special courts were likewise created. The
may there be continued national progress." 15 After which first was the Court of Tax Appeals in 1954, 30 next came the
comes: "To be less abstract, the thrust is on development. Court of Agrarian Relations in 1955, 31 and then in the same
That has been repeatedly stressed — and rightly so. All efforts year a Court of the Juvenile and Domestic Relations for Manila
are geared to its realization. Nor, unlike in the past, was it to b in 1955, 32 subsequently followed by the creation of two other
"considered as simply the movement towards economic such courts for Iloilo and Quezon City in 1966. 33 In 1967,
progress and growth measured in terms of sustained Circuit Criminal Courts were established, with the Judges
increases in per capita income and Gross National Product having the same qualifications, rank, compensation, and
(GNP). 16 For the New Society, its implication goes further privileges as judges of Courts of First Instance. 34
than economic advance, extending to "the sharing, or more
appropriately, the democratization of social and economic
opportunities, the substantiation of the true meaning of social 4. After the submission of such Report, Cabinet Bill No. 42,
justice." 17 This process of modernization and change which later became the basis of Batas Pambansa Blg. 129,
compels the government to extend its field of activity and its was introduced. After setting forth the background as above
scope of operations. The efforts towards reducing the gap narrated, its Explanatory Note continues: "Pursuant to the
between the wealthy and the poor elements in the nation call President's instructions, this proposed legislation has been
for more regulatory legislation. That way the social justice and drafted in accordance with the guidelines of that report with
protection to labor mandates of the Constitution could be particular attention to certain objectives of the reorganization,
effectively implemented." 18 There is likelihood then "that to wit, the attainment of more efficiency in disposal of cases,
some measures deemed inimical by interests adversely a reallocation of jurisdiction, and a revision of procedures
affected would be challenged in court on grounds of validity. which do not tend to the proper meeting out of justice. In
Even if the question does not go that far, suits may be filed consultation with, and upon a consensus of, the governmental
concerning their interpretation and application. ... There could and parliamentary leadership, however, it was felt that some
be pleas for injunction or restraining orders. Lack of success options set forth in the Report be not availed of. Instead of the
of such moves would not, even so, result in their prompt final proposal to confine the jurisdiction of the intermediate
disposition. Thus delay in the execution of the policies appellate court merely to appellate adjudication, the
embodied in law could thus be reasonably expected. That is preference has been opted to increase rather than diminish its
not conducive to progress in development." 19 For, as jurisdiction in order to enable it to effectively assist the
mentioned in such Report, equally of vital concern is the Supreme Court. This preference has been translated into one
problem of clogged dockets, which "as is well known, is one of the innovations in the proposed Bill." 35 In accordance with
of the utmost gravity. Notwithstanding the most determined the parliamentary procedure, the Bill was sponsored by the
efforts exerted by the Supreme Court, through the leadership Chairman of the Committee on Justice, Human Rights and
of both retired Chief Justice Querube Makalintal and the late Good Government to which it was referred. Thereafter,
Chief Justice Fred Ruiz Castro, from the time supervision of Committee Report No. 225 was submitted by such Committee
FINALS CONSTITUTIONAL LAW I ACJUCO NOV 11, 2017 216

to the Batasang Pambansa recommending the approval with Administrative Code to organize courts of original jurisdiction
some amendments. In the sponsorship speech of Minister known as the Courts of First Instance Prior to such statute,
Ricardo C. Puno, there was reference to the Presidential petitioner was the incumbent of such branch. Thereafter, he
Committee on Judicial Reorganization. Thus: "On October 17, received an ad interim appointment, this time to the Fourth
1980, the Presidential Committee on Judicial Reorganization Judicial District, under the new legislation. Unfortunately for
submitted its report to the President which contained the him, the Commission on Appointments of then National
'Proposed Guidelines for Judicial Reorganization.' Cabinet Bill Assembly disapproved the same, with respondent being
No. 42 was drafted substantially in accordance with the appointed in his place. He contested the validity of the Act
options presented by these guidelines. Some options set forth insofar as it resulted in his being forced to vacate his position
in the aforesaid report were not availed of upon consultation This Court did not rule squarely on the matter. His petition was
with and upon consensus of the government and dismissed on the ground of estoppel. Nonetheless, the
parliamentary leadership. Moreover, some amendments to separate concurrence of Justice Laurel in the result reached,
the bill were adopted by the Committee on Justice, Human to repeat, reaffirms in no uncertain terms the standard of good
Rights and Good Government, to which The bill was referred, faith to preclude any doubt as to the abolition of an inferior
following the public hearings on the bill held in December of court, with due recognition of the security of tenure guarantee.
1980. The hearings consisted of dialogues with the Thus: " I am of the opinion that Commonwealth Act No. 145 in
distinguished members of the bench and the bar who had so far as it reorganizes, among other judicial districts, the
submitted written proposals, suggestions, and position papers Ninth Judicial District, and establishes an entirely new district
on the bill upon the invitation of the Committee on Justice, comprising Manila and the provinces of Rizal and Palawan, is
Human Rights and Good Government." 36 Stress was laid by valid and constitutional. This conclusion flows from the
the sponsor that the enactment of such Cabinet Bill would, fundamental proposition that the legislature may abolish
firstly, result in the attainment of more efficiency in the courts inferior to the Supreme Court and therefore may
disposal of cases. Secondly, the improvement in the quality of reorganize them territorially or otherwise thereby
justice dispensed by the courts is expected as a necessary necessitating new appointments and commissions. Section 2,
consequence of the easing of the court's dockets. Thirdly, the Article VIII of the Constitution vests in the National Assembly
structural changes introduced in the bill, together with the the power to define, prescribe and apportion the jurisdiction of
reallocation of jurisdiction and the revision of the rules of the various courts, subject to certain limitations in the case of
procedure, are designated to suit the court system to the the Supreme Court. It is admitted that section 9 of the same
exigencies of the present day Philippine society, and article of the Constitution provides for the security of tenure of
hopefully, of the foreseeable future." 37 it may be observed all the judges. The principles embodied in these two sections
that the volume containing the minutes of the proceedings of of the same article of the Constitution must be coordinated
the Batasang Pambansa show that 590 pages were devoted and harmonized. A mere enunciation of a principle will not
to its discussion. It is quite obvious that it took considerable decide actual cases and controversies of every sort. (Justice
time and effort as well as exhaustive study before the act was Holmes in Lochner vs. New York, 198 U.S., 45; 49 Law. ed;
signed by the President on August 14, 1981. With such a 937)" 44 justice Laurel continued: "I am not insensible to the
background, it becomes quite manifest how lacking in factual argument that the National Assembly may abuse its power
basis is the allegation that its enactment is tainted by the vice and move deliberately to defeat the constitutional provision
of arbitrariness. What appears undoubted and undeniable is guaranteeing security of tenure to all judges, But, is this the
the good faith that characterized its enactment from its case? One need not share the view of Story, Miller and Tucker
inception to the affixing of the Presidential signature. on the one hand, or the opinion of Cooley, Watson and
Baldwin on the other, to realize that the application of a legal
5. Nothing is better settled in our law than that the abolition of or constitutional principle is necessarily factual and
an office within the competence of a legitimate body if done in circumstantial and that fixity of principle is the rigidity of the
good faith suffers from no infirmity. The ponencia of Justice dead and the unprogressive. I do say, and emphatically,
J.B.L. Reyes in Cruz v. Primicias, Jr. 38reiterated such a however, that cases may arise where the violation of the
doctrine: "We find this point urged by respondents, to be constitutional provision regarding security of tenure is
without merit. No removal or separation of petitioners from the palpable and plain, and that legislative power of
service is here involved, but the validity of the abolition of their reorganization may be sought to cloak an unconstitutional and
offices. This is a legal issue that is for the Courts to decide. It evil purpose. When a case of that kind arises, it will be the
is well-known rule also that valid abolition of offices is neither time to make the hammer fall and heavily. But not until then. I
removal nor separation of the incumbents. ... And, of course, am satisfied that, as to the particular point here discussed, the
if the abolition is void, the incumbent is deemed never to have purpose was the fulfillment of what was considered a great
ceased to hold office. The preliminary question laid at rest, we public need by the legislative department and that
pass to the merits of the case. As well-settled as the rule that Commonwealth Act No. 145 was not enacted purposely to
the abolition of an office does not amount to an illegal removal affect adversely the tenure of judges or of any particular judge.
of its incumbent is the principle that, in order to be valid, the Under these circumstances, I am for sustaining the power of
abolition must be made in good faith." 39 The above excerpt the legislative department under the Constitution. To be sure,
was quoted with approval in Bendanillo, Sr. v. Provincial there was greater necessity for reorganization consequent
Governor, 40 two earlier cases enunciating a similar doctrine upon the establishment of the new government than at the
having preceded it. 41 As with the offices in the other time Acts Nos. 2347 and 4007 were approved by the defunct
branches of the government, so it is with the judiciary. The test Philippine Legislature, and although in the case of these two
remains whether the abolition is in good faith. As that element Acts there was an express provision providing for the vacation
is conspicuously present in the enactment of Batas Pambansa by the judges of their offices whereas in the case of
Blg. 129, then the lack of merit of this petition becomes even Commonwealth Act No. 145 doubt is engendered by its
more apparent. The concurring opinion of Justice Laurel in silence, this doubt should be resolved in favor of the valid
Zandueta v. De la Costa 42 cannot be any clearer. This is a exercise of the legislative power." 45
quo warranto proceeding filed by petitioner, claiming that he,
and not respondent, was entitled to he office of judge of the 6. A few more words on the question of abolition. In the above-
Fifth Branch of the Court of First Instance of Manila. There cited opinion of Justice Laurel in Zandueta, reference was
was a Judicial Reorganization Act in 1936, 43 a year after the made to Act No. 2347 46 on the reorganization of the Courts
inauguration of the Commonwealth, amending the of First Instance and to Act No. 4007 47 on the reorganization
FINALS CONSTITUTIONAL LAW I ACJUCO NOV 11, 2017 217

of all branches of the government, including the courts of first influences, is thereby reduced to a barren form of words. The
instance. In both of them, the then Courts of First Instance amended Constitution adheres even more clearly to the long-
were replaced by new courts with the same appellation. As established tradition of a strong executive that antedated the
Justice Laurel pointed out, there was no question as to the fact 1935 Charter. As noted in the work of former Vice-Governor
of abolition. He was equally categorical as to Commonwealth Hayden, a noted political scientist, President Claro M. Recto
Act No. 145, where also the system of the courts of first of the 1934 Convention, in his closing address, in stressing
instance was provided for expressly. It was pointed out by such a concept, categorically spoke of providing "an executive
Justice Laurel that the mere creation of an entirely new district power which, subject to the fiscalization of the Assembly, and
of the same court is valid and constitutional. such conclusion of public opinion, will not only know how to govern, but will
flowing "from the fundamental proposition that the legislature actually govern, with a firm and steady hand, unembarrassed
may abolish courts inferior to the Supreme Court and by vexatious interferences by other departments, or by unholy
therefore may reorganize them territorially or otherwise alliances with this and that social group." 61 The above
thereby necessitating new appointments and commissions." excerpt was cited with approval by Justice Laurel in Planas v.
48 The challenged statute creates an intermediate appellate Gil. 62 Moreover, under the 1981 Amendments, it may be
court, 49 regional trial courts, 50 metropolitan trial courts of affirmed that once again the principle of separation of powers,
the national capital region, 51 and other metropolitan trial to quote from the same jurist as ponente in Angara v. Electoral
courts, 52 municipal trial courts in cities, 53 as well as in Commission, 63 "obtains not through express provision but by
municipalities, 54 and municipal circuit trial courts. 55 There actual division." 64 The president, under Article VII, shall be
is even less reason then to doubt the fact that existing inferior the head of state and chief executive of the Republic of the
courts were abolished. For the Batasang Pambansa, the Philippines." 65Moreover, it is equally therein expressly
establishment of such new inferior courts was the appropriate provided that all the powers he possessed under the 1935
response to the grave and urgent problems that pressed for Constitution are once again vested in him unless the Batasang
solution. Certainly, there could be differences of opinion as to Pambansa provides otherwise." 66 Article VII of the 1935
the appropriate remedy. The choice, however, was for the Constitution speaks categorically: "The Executive power shall
Batasan to make, not for this Court, which deals only with the be vested in a President of the Philippines." 67 As originally
question of power. It bears mentioning that in Brillo v. Eñage framed, the 1973 Constitution created the position of
56 this Court, in an unanimous opinion penned by the late President as the "symbolic head of state." 68 In addition, there
Justice Diokno, citing Zandueta v. De la Costa, ruled: "La was a provision for a Prime Minister as the head of
segunda question que el recurrrido plantea es que la Carta de government exercising the executive power with the
Tacloban ha abolido el puesto. Si efectivamente ha sido assistance of the Cabinet 69 Clearly, a modified parliamentary
abolido el cargo, entonces ha quedado extinguido el derecho system was established. In the light of the 1981 amendments
de recurente a ocuparlo y a cobrar el salario correspodiente. though, this Court in Free Telephone Workers Union v.
Mc Culley vs. State, 46 LRA, 567. El derecho de un juez de Minister of Labor 70 could state: "The adoption of certain
desempenarlo hasta los 70 años de edad o se incapacite no aspects of a parliamentary system in the amended
priva al Congreso de su facultad de abolir, fusionar o Constitution does not alter its essentially presidential
reorganizar juzgados no constitucionales." 57 Nonetheless, character." 71 The retention, however, of the position of the
such well-established principle was not held applicable to the Prime Minister with the Cabinet, a majority of the members of
situation there obtaining, the Charter of Tacloban City creating which shall come from the regional representatives of the
a city court in place of the former justice of the peace court. Batasang Pambansa and the creation of an Executive
Thus: "Pero en el caso de autos el Juzgado de Tacloban no Committee composed of the Prime Minister as Chairman and
ha sido abolido. Solo se le ha cambiado el nombre con el not more than fourteen other members at least half of whom
cambio de forma del gobierno local." 58 The present case is shall be members of the Batasang Pambansa, clearly indicate
anything but that. Petitioners did not and could not prove that the evolving nature of the system of government that is now
the challenged statute was not within the bounds of legislative operative. 72 What is equally apparent is that the strongest
authority. ties bind the executive and legislative departments. It is
likewise undeniable that the Batasang Pambansa retains its
7. This opinion then could very well stop at this point. The full authority to enact whatever legislation may be necessary
implementation of Batas Pambansa Blg. 129, concededly a to carry out national policy as usually formulated in a caucus
task incumbent on the Executive, may give rise, however, to of the majority party. It is understandable then why in Fortun
questions affecting a judiciary that should be kept v. Labang 73 it was stressed that with the provision
independent. The all-embracing scope of the assailed transferring to the Supreme Court administrative supervision
legislation as far as all inferior courts from the Courts of over the Judiciary, there is a greater need "to preserve
Appeals to municipal courts are concerned, with the exception unimpaired the independence of the judiciary, especially so at
solely of the Sandiganbayan and the Court of Tax Appeals 59 present, where to all intents and purposes, there is a fusion
gave rise, and understandably so, to misgivings as to its effect between the executive and the legislative branches." 74
on such cherished Ideal. The first paragraph of the section on
the transitory provision reads: "The provisions of this Act shall 8. To be more specific, petitioners contend that the abolition
be immediately carried out in accordance with an Executive of the existing inferior courts collides with the security of
Order to be issued by the President. The Court of Appeals, tenure enjoyed by incumbent Justices and judges under
the Courts of First Instance, the Circuit Criminal Courts, the Article X, Section 7 of the Constitution. There was a similar
Juvenile and Domestic Relations Courts, the Courts of provision in the 1935 Constitution. It did not, however, go as
Agrarian Relations, the City Courts, the Municipal Courts, and far as conferring on this Tribunal the power to supervise
the Municipal Circuit Courts shall continue to function as administratively inferior courts. 75 Moreover, this Court is em
presently constituted and organized, until the completion of powered "to discipline judges of inferior courts and, by a vote
the reorganization provided in this Act as declared by the of at least eight members, order their dismissal." 76 Thus it
President. Upon such declaration, the said courts shall be possesses the competence to remove judges. Under the
deemed automatically abolished and the incumbents thereof Judiciary Act, it was the President who was vested with such
shall cease to hold the office." 60 There is all the more reason power. 77 Removal is, of course, to be distinguished from
then why this Court has no choice but to inquire further into termination by virtue of the abolition of the office. There can
the allegation by petitioners that the security of tenure be no tenure to a non-existent office. After the abolition, there
provision, an assurance of a judiciary free from extraneous is in law no occupant. In case of removal, there is an office
FINALS CONSTITUTIONAL LAW I ACJUCO NOV 11, 2017 218

with an occupant who would thereby lose his position. It is in cannot, whenever appropriate, avoid the task of reconciliation.
that sense that from the standpoint of strict law, the question As Justice Laurel put it so well in the previously cited Angara
of any impairment of security of tenure does not arise. decision, while in the main, "the Constitution has blocked out
Nonetheless, for the incumbents of inferior courts abolished, with deft strokes and in bold lines, allotment of power to the
the effect is one of separation. As to its effect, no distinction executive, the legislative and the judicial departments of the
exists between removal and the abolition of the office. government, the overlapping and interlacing of functions and
Realistically, it is devoid of significance. He ceases to be a duties between the several departments, however, sometimes
member of the judiciary. In the implementation of the assailed makes it hard to say just where the one leaves off and the
legislation, therefore, it would be in accordance with accepted other begins." 84 It is well to recall another classic utterance
principles of constitutional construction that as far as from the same jurist, even more emphatic in its affirmation of
incumbent justices and judges are concerned, this Court be such a view, moreover buttressed by one of those insights for
consulted and that its view be accorded the fullest which Holmes was so famous "The classical separation of
consideration. No fear need be entertained that there is a government powers, whether viewed in the light of the political
failure to accord respect to the basic principle that this Court philosophy of Aristotle, Locke, or Motesquieu or of the
does not render advisory opinions. No question of law is postulations of Mabini, Madison, or Jefferson, is a relative
involved. If such were the case, certainly this Court could not theory of government. There is more truism and actuality in
have its say prior to the action taken by either of the two interdependence than in independence and separation of
departments. Even then, it could do so but only by way of powers, for as observed by Justice Holmes in a case of
deciding a case where the matter has been put in issue. Philippine origin, we cannot lay down 'with mathematical
Neither is there any intrusion into who shall be appointed to precision and divide the branches into water-tight
the vacant positions created by the reorganization. That compartments' not only because 'the great ordinances of the
remains in the hands of the Executive to whom it properly Constitution do not establish and divide fields of black and
belongs. There is no departure therefore from the tried and white but also because 'even the more specific of them are
tested ways of judicial power, Rather what is sought to be found to terminate in a penumbra shading gradually from one
achieved by this liberal interpretation is to preclude any extreme to the other.'" 85 This too from Justice Tuazon,
plausibility to the charge that in the exercise of the conceded likewise expressing with force and clarity why the need for
power of reorganizing tulle inferior courts, the power of reconciliation or balancing is well-nigh unavodiable under the
removal of the present incumbents vested in this Tribunal is fundamental principle of separation of powers: "The
ignored or disregarded. The challenged Act would thus be free constitutional structure is a complicated system, and
from any unconstitutional taint, even one not readily overlappings of governmental functions are recognized,
discernidble except to those predisposed to view it with unavoidable, and inherent necessities of governmental
distrust. Moreover, such a construction would be in coordination." 86 In the same way that the academe has noted
accordance with the basic principle that in the choice of the existence in constitutional litigation of right versus right,
alternatives between one which would save and another there are instances, and this is one of them, where, without
which would invalidate a statute, the former is to be preferred. this attempt at harmonizing the provisions in question, there
78 There is an obvious way to do so. The principle that the could be a case of power against power. That we should
Constitution enters into and forms part of every act to avoid avoid.
any constitutional taint must be applied Nuñez v.
Sandiganbayan, 79 promulgated last January, has this 10. There are other objections raised but they pose no
relevant excerpt: "It is true that other Sections of the Decree difficulty. Petitioners would characterize as an undue
could have been so worded as to avoid any constitutional delegation of legislative power to the President the grant of
objection. As of now, however, no ruling is called for. The view authority to fix the compensation and the allowances of the
is given expression in the concurring and dissenting opinion Justices and judges thereafter appointed. A more careful
of Justice Makasiar that in such a case to save the Decree reading of the challenged Batas Pambansa Blg. 129 ought to
from the direct fate of invalidity, they must be construed in have cautioned them against raising such an issue. The
such a way as to preclude any possible erosion on the powers language of the statute is quite clear. The questioned
vested in this Court by the Constitution. That is a proposition provisions reads as follows: "Intermediate Appellate Justices,
too plain to be committed. It commends itself for approval." Regional Trial Judges, Metropolitan Trial Judges, municipal
80Nor would such a step be unprecedented. The Presidential Trial Judges, and Municipal Circuit Trial Judges shall receive
Decree constituting Municipal Courts into Municipal Circuit such receive such compensation and allowances as may be
Courts, specifically provides: "The Supreme Court shall carry authorized by the President along the guidelines set forth in
out the provisions of this Decree through implementing orders, Letter of Implementation No. 93 pursuant to Presidential
on a province-to-province basis." 81 It is true there is no such Decree No. 985, as amended by Presidential Decree No.
provision in this Act, but the spirit that informs it should not be 1597." 87 The existence of a standard is thus clear. The basic
ignored in the Executive Order contemplated under its Section postulate that underlies the doctrine of non-delegation is that
44. 82 Thus Batas Pambansa Blg. 129 could stand the most it is the legislative body which is entrusted with the
rigorous test of constitutionality. 83 competence to make laws and to alter and repeal them, the
test being the completeness of the statue in all its terms and
9. Nor is there anything novel in the concept that this Court is provisions when enacted. As pointed out in Edu v. Ericta: 88
called upon to reconcile or harmonize constitutional "To avoid the taint of unlawful delegation, there must be a
provisions. To be specific, the Batasang Pambansa is standard, which implies at the very least that the legislature
expressly vested with the authority to reorganize inferior itself determines matters of principle and lays down
courts and in the process to abolish existing ones. As noted in fundamental policy. Otherwise, the charge of complete
the preceding paragraph, the termination of office of their abdication may be hard to repel. A standard thus defines
occupants, as a necessary consequence of such abolition, is legislative policy, marks its limits, maps out its boundaries and
hardly distinguishable from the practical standpoint from specifies the public agency to apply it. It indicates the
removal, a power that is now vested in this Tribunal. It is of the circumstances under which the legislative command is to be
essence of constitutionalism to assure that neither agency is effected. It is the criterion by which legislative purpose may be
precluded from acting within the boundaries of its conceded carried out. Thereafter, the executive or administrative office
competence. That is why it has long been well-settled under designated may in pursuance of the above guidelines
the constitutional system we have adopted that this Court promulgate supplemental rules and regulations. The standard
FINALS CONSTITUTIONAL LAW I ACJUCO NOV 11, 2017 219

may be either express or implied. If the former, the non- the discussion of Batas Pambansa Blg. 129. They were not
delegation objection is easily met. The standard though does consulted. They did not testify. The challenged legislation is
not have to be spelled out specifically. It could be implied from entirely the product of the efforts of the legislative body. 100
the policy and purpose of the act considered as a whole." 89 Their work was limited, as set forth in the Executive Order, to
The undeniably strong links that bind the executive and submitting alternative plan for reorganization. That is more in
legislative departments under the amended Constitution the nature of scholarly studies. That the undertook. There
assure that the framing of policies as well as their could be no possible objection to such activity. Ever since
implementation can be accomplished with unity, promptitude, 1973, this Tribunal has had administrative supervision over
and efficiency. There is accuracy, therefore, to this interior courts. It has had the opportunity to inform itself as to
observation in the Free Telephone Workers Union decision: the way judicial business is conducted and how it may be
"There is accordingly more receptivity to laws leaving to improved. Even prior to the 1973 Constitution, it is the
administrative and executive agencies the adoption of such recollection of the writer of this opinion that either the then
means as may be necessary to effectuate a valid legislative Chairman or members of the Committee on Justice of the then
purpose. It is worth noting that a highly-respected legal Senate of the Philippines 101 consulted members of the Court
scholar, Professor Jaffe, as early as 1947, could speak of in drafting proposed legislation affecting the judiciary. It is not
delegation as the 'dynamo of modern government.'" 90 He inappropriate to cite this excerpt from an article in the 1975
warned against a "restrictive approach" which could be "a Supreme Court Review: "In the twentieth century the Chief
deterrent factor to much-needed legislation." 91 Further on Justice of the United States has played a leading part in
this point from the same opinion" "The spectre of the non- judicial reform. A variety of conditions have been responsible
delegation concept need not haunt, therefore, party caucuses, for the development of this role, and foremost among them
cabinet sessions or legislative chambers." 92 Another has been the creation of explicit institutional structures
objection based on the absence in the statue of what designed to facilitate reform." 102 Also: "Thus the Chief
petitioners refer to as a "definite time frame limitation" is Justice cannot avoid exposure to and direct involvement in
equally bereft of merit. They ignore the categorical language judicial reform at the federal level and, to the extent issues of
of this provision: "The Supreme Court shall submit to the judicial federalism arise, at the state level as well." 103
President, within thirty (30) days from the date of the effectivity
of this act, a staffing pattern for all courts constituted pursuant 12. It is a cardinal article of faith of our constitutional regime
to this Act which shall be the basis of the implementing order that it is the people who are endowed with rights, to secure
to be issued by the President in accordance with the which a government is instituted. Acting as it does through
immediately succeeding section." 93 The first sentence of the public officials, it has to grant them either expressly or
next section is even more categorical: "The provisions of this impliedly certain powers. Those they exercise not for their own
Act shall be immediately carried out in accordance with an benefit but for the body politic. The Constitution does not
Executive Order to be issued by the President." 94 Certainly speak in the language of ambiguity: "A public office is a public
petitioners cannot be heard to argue that the President is trust." 104 That is more than a moral adjuration It is a legal
insensible to his constitutional duty to take care that the laws imperative. The law may vest in a public official certain rights.
be faithfully executed. 95 In the meanwhile, the existing It does so to enable them to perform his functions and fulfill
inferior courts affected continue functioning as before, "until his responsibilities more efficiently. It is from that standpoint
the completion of the reorganization provided in this Act as that the security of tenure provision to assure judicial
declared by the President. Upon such declaration, the said independence is to be viewed. It is an added guarantee that
courts shall be deemed automatically abolished and the justices and judges can administer justice undeterred by any
incumbents thereof shall cease to hold office." 96 There is no fear of reprisal or untoward consequence. Their judgments
ambiguity. The incumbents of the courts thus automatically then are even more likely to be inspired solely by their
abolished "shall cease to hold office." No fear need be knowledge of the law and the dictates of their conscience, free
entertained by incumbents whose length of service, quality of from the corrupting influence of base or unworthy motives.
performance, and clean record justify their being named The independence of which they are assured is impressed
anew, 97 in legal contemplation without any interruption in the with a significance transcending that of a purely personal right.
continuity of their service. 98 It is equally reasonable to As thus viewed, it is not solely for their welfare. The
assume that from the ranks of lawyers, either in the challenged legislation Thus subject d to the most rigorous
government service, private practice, or law professors will scrutiny by this Tribunal, lest by lack of due care and
come the new appointees. In the event that in certain cases a circumspection, it allow the erosion of that Ideal so firmly
little more time is necessary in the appraisal of whether or not embedded in the national consciousness There is this farther
certain incumbents deserve reappointment, it is not from their thought to consider. independence in thought and action
standpoint undesirable. Rather, it would be a reaffirmation of necessarily is rooted in one's mind and heart. As emphasized
the good faith that will characterize its implementation by the by former Chief Justice Paras in Ocampo v. Secretary of
Executive. There is pertinence to this observation of Justice Justice, 105 there is no surer guarantee of judicial
Holmes that even acceptance of the generalization that courts independence than the God-given character and fitness of
ordinarily should not supply omissions in a law, a those appointed to the Bench. The judges may be guaranteed
generalization qualified as earlier shown by the principle that a fixed tenure of office during good behavior, but if they are of
to save a statute that could be done, "there is no canon such stuff as allows them to be subservient to one
against using common sense in construing laws as saying administration after another, or to cater to the wishes of one
what they obviously mean." 99 Where then is the litigant after another, the independence of the judiciary will be
unconstitutional flaw nothing more than a myth or an empty Ideal. Our judges, we
are confident, can be of the type of Lord Coke, regardless or
11. On the morning of the hearing of this petition on in spite of the power of Congress — we do not say unlimited
September 8, 1981, petitioners sought to have the writer of but as herein exercised — to reorganize inferior courts." 106
this opinion and Justices Ramon C. Aquino and Ameurfina That is to recall one of the greatest Common Law jurists, who
Melencio-Herrera disqualified because the first-named was at the cost of his office made clear that he would not just
the chairman and the other two, members of the Committee blindly obey the King's order but "will do what becomes [him]
on Judicial Reorganization. At the hearing, the motion was as a judge." So it was pointed out in the first leading case
denied. It was made clear then and there that not one of the stressing the independence of the judiciary, Borromeo v.
three members of the Court had any hand in the framing or in Mariano, 107 The ponencia of Justice Malcolm Identified good
FINALS CONSTITUTIONAL LAW I ACJUCO NOV 11, 2017 220

judges with "men who have a mastery of the principles of law,


who discharge their duties in accordance with law, who are
permitted to perform the duties of the office undeterred by
outside influence, and who are independent and self-
respecting human units in a judicial system equal and
coordinate to the other two departments of government." 108
There is no reason to assume that the failure of this suit to
annul Batas Pambansa Blg. 129 would be attended with
deleterious consequences to the administration of justice. It
does not follow that the abolition in good faith of the existing
inferior courts except the Sandiganbayan and the Court of Tax
Appeals and the creation of new ones will result in a judiciary
unable or unwilling to discharge with independence its solemn
duty or one recreant to the trust reposed in it. Nor should there
be any fear that less than good faith will attend the exercise
be of the appointing power vested in the Executive. It cannot
be denied that an independent and efficient judiciary is
something to the credit of any administration. Well and truly
has it been said that the fundamental principle of separation
of powers assumes, and justifiably so, that the three
departments are as one in their determination to pursue the
Ideals and aspirations and to fulfilling the hopes of the
sovereign people as expressed in the Constitution. There is
wisdom as well as validity to this pronouncement of Justice
Malcolm in Manila Electric Co. v. Pasay Transportation
Company, 109 a decision promulgated almost half a century
ago: "Just as the Supreme Court, as the guardian of
constitutional rights, should not sanction usurpations by any
other department or the government, so should it as strictly
confine its own sphere of influence to the powers expressly or
by implication conferred on it by the Organic Act." 110 To that
basic postulate underlying our constitutional system, this
Court remains committed.

WHEREFORE, the unconstitutionality of Batas Pambansa


Blg. 129 not having been shown, this petition is dismissed. No
costs.

Makasiar and Escolin, JJ., concur.

Concepcion, Jr., concur in the result.


FINALS CONSTITUTIONAL LAW I ACJUCO NOV 11, 2017 221

G.R. No. 202242 April 16, 2013 Brief Statement of the Antecedents

FRANCISCO I. CHAVEZ, Petitioner, In this disposition, it bears reiterating that from the birth of the
Philippine Republic, the exercise of appointing members of
vs. the Judiciary has always been the exclusive prerogative of the
executive and legislative branches of the government. Like
their progenitor of American origins, both the Malolos
JUDICIALAND BAR COUNCIL, SEN. FRANCIS JOSEPH G. Constitution11 and the 1935 Constitution12vested the power
ESCUDERO and REP. NIEL C. TUPAS, JR.,Respondents. to appoint the members of the Judiciary in the President,
subject to confirmation by the Commission on Appointments.
RESOLUTION It was during these times that the country became witness to
the deplorable practice of aspirants seeking confirmation of
MENDOZA, J.: their appointment in the Judiciary to ingratiate themselves with
the members of the legislative body.13

This resolves the Motion for Reconsideration1 filed by the


Office of the Solicitor General (OSG) on behalf of the Then, under the 1973 Constitution,14 with the fusion of the
respondents, Senator Francis Joseph G. Escudero and executive and legislative powers in one body, the appointment
Congressman Niel C. Tupas, Jr. (respondents), duly of judges and justices ceased to be subject of scrutiny by
opposed2 by the petitioner, former Solicitor General Francisco another body. The power became exclusive and absolute to
I. Chavez (petitioner). the Executive, subject only to the condition that the appointees
must have all the qualifications and none of the
disqualifications.
By way of recapitulation, the present action stemmed from the
unexpected departure of former Chief Justice Renato C.
Corona on May 29, 2012, and the nomination of petitioner, Prompted by the clamor to rid the process of appointments to
as his potential successor. In his initiatory pleading, the Judiciary of the evils of political pressure and partisan
petitioner asked the Court to determine 1] whether the first activities,15 the members of the Constitutional Commission
paragraph of Section 8, Article VIII of the 1987 Constitution saw it wise to create a separate, competent and independent
allows more than one (1) member of Congress to sit in the body to recommend nominees to the President.
JBC; and 2] if the practice of having two (2) representatives
from each House of Congress with one (1) vote each is Thus, it conceived of a body, representative of all the
sanctioned by the Constitution. stakeholders in the judicial appointment process, and called it
the Judicial and Bar Council (JBC). The Framers carefully
On July 17, 2012, the Court handed down the assailed subject worded Section 8, Article VIII of the 1987 Constitution in this
decision, disposing the same in the following manner: wise:

WHEREFORE, the petition is GRANTED. The current Section 8. (1) A Judicial and Bar Council is hereby created
numerical composition of the Judicial and Bar Council is under the supervision of the Supreme Court composed of the
declared UNCONSTITUTIONAL. The Judicial and Bar Chief Justice as ex officio Chairman, the Secretary of Justice,
Council is hereby enjoined to reconstitute itself so that only and a representative of the Congress as ex officio Members,
one (1) member of Congress will sit as a representative in its a representative of the Integrated Bar, a professor of law, a
proceedings, in accordance with Section 8(1), Article VIII of retired Member of the Supreme Court, and a representative of
the 1987 Constitution. the private sector.

This disposition is immediately executory. From the moment of the creation of the JBC, Congress
designated one (1) representative to sit in the JBC to act as
one of the ex-officio members.16 Pursuant to the
SO ORDERED. constitutional provision that Congress is entitled to one (1)
representative, each House sent a representative to the JBC,
On July 31, 2012, following respondents’ motion for not together, but alternately or by rotation.
reconsideration and with due regard to Senate Resolution
Nos. 111,3 112,4 113,5 and 114,6 the Court set the subject In 1994, the seven-member composition of the JBC was
motion for oral arguments on August 2, 2012.7 On August 3, substantially altered.1âwphi1 An eighth member was added
2012, the Court discussed the merits of the arguments and to the JBC as the two (2) representatives from Congress
agreed, in the meantime, to suspend the effects of the second began sitting simultaneously in the JBC, with each having
paragraph of the dispositive portion of the July 17, 2012 one-half (1/2) of a vote.17
Decision which decreed that it was immediately executory.
The decretal portion of the August 3, 2012 Resolution8 reads:
In 2001, the JBC En Banc decided to allow the representatives
from the Senate and the House of Representatives one full
WHEREFORE, the parties are hereby directed to submit their vote each.18 It has been the situation since then.
respective MEMORANDA within ten (10) days from notice.
Until further orders, the Court hereby SUSPENDS the effect
of the second paragraph of the dispositive portion of the Grounds relied upon by Respondents
Court’s July 17, 2012 Decision, which reads: "This disposition
is immediately executory."9 Through the subject motion, respondents pray that the Court
reconsider its decision and dismiss the petition on the
Pursuant to the same resolution, petitioner and respondents following grounds: 1] that allowing only one representative
filed their respective memoranda.10 from Congress in the JBC would lead to absurdity considering
its bicameral nature; 2] that the failure of the Framers to make
the proper adjustment when there was a shift from
FINALS CONSTITUTIONAL LAW I ACJUCO NOV 11, 2017 222

unilateralism to bicameralism was a plain oversight; 3] that two to a bicameral form of the legislature, is not persuasive
representatives from Congress would not subvert the intention enough. Respondents cannot just lean on plain oversight to
of the Framers to insulate the JBC from political partisanship; justify a conclusion favorable to them. It is very clear that the
and 4] that the rationale of the Court in declaring a seven- Framers were not keen on adjusting the provision on
member composition would provide a solution should there be congressional representation in the JBC because it was not in
a stalemate is not exactly correct. the exercise of its primary function – to legislate. JBC was
created to support the executive power to appoint, and
While the Court may find some sense in the reasoning in Congress, as one whole body, was merely assigned a
amplification of the third and fourth grounds listed by contributory non-legislative function.
respondents, still, it finds itself unable to reverse the assailed
decision on the principal issues covered by the first and The underlying reason for such a limited participation can
second grounds for lack of merit. Significantly, the conclusion easily be discerned. Congress has two (2) Houses. The need
arrived at, with respect to the first and second grounds, carries to recognize the existence and the role of each House is
greater bearing in the final resolution of this case. essential considering that the Constitution employs precise
language in laying down the functions which particular House
As these two issues are interrelated, the Court shall discuss plays, regardless of whether the two Houses consummate an
them jointly. official act by voting jointly or separately. Whether in the
exercise of its legislative23 or its non-legislative functions
such as inter alia, the power of appropriation,24 the
Ruling of the Court declaration of an existence of a state of war,25 canvassing of
electoral returns for the President and Vice-President,26 and
The Constitution evinces the direct action of the Filipino impeachment,27 the dichotomy of each House must be
people by which the fundamental powers of government acknowledged and recognized considering the interplay
are established, limited and defined and by which those between these two Houses. In all these instances, each
powers are distributed among the several departments House is constitutionally granted with powers and functions
for their safe and useful exercise for the benefit of the peculiar to its nature and with keen consideration to 1) its
body politic.19 The Framers reposed their wisdom and vision relationship with the other chamber; and 2) in consonance
on one suprema lex to be the ultimate expression of the with the principle of checks and balances, as to the other
principles and the framework upon which government and branches of government.
society were to operate. Thus, in the interpretation of the
constitutional provisions, the Court firmly relies on the basic In checkered contrast, there is essentially no interaction
postulate that the Framers mean what they say. The language between the two Houses in their participation in the JBC. No
used in the Constitution must be taken to have been mechanism is required between the Senate and the House of
deliberately chosen for a definite purpose. Every word Representatives in the screening and nomination of judicial
employed in the Constitution must be interpreted to exude its officers. Rather, in the creation of the JBC, the Framers
deliberate intent which must be maintained inviolate against arrived at a unique system by adding to the four (4) regular
disobedience and defiance. What the Constitution clearly members, three (3) representatives from the major branches
says, according to its text, compels acceptance and bars of government - the Chief Justice as ex-officio Chairman
modification even by the branch tasked to interpret it. (representing the Judicial Department), the Secretary of
Justice (representing the Executive Department), and a
For this reason, the Court cannot accede to the argument of representative of the Congress (representing the Legislative
plain oversight in order to justify constitutional construction. As Department). The total is seven (7), not eight. In so providing,
stated in the July 17, 2012 Decision, in opting to use the the Framers simply gave recognition to the Legislature, not
singular letter "a" to describe "representative of Congress," because it was in the interest of a certain constituency, but in
the Filipino people through the Framers intended that reverence to it as a major branch of government.
Congress be entitled to only one (1) seat in the JBC. Had the
intention been otherwise, the Constitution could have, in no On this score, a Member of Congress, Hon. Simeon A.
uncertain terms, so provided, as can be read in its other Datumanong, from the Second District of Maguindanao,
provisions. submitted his well-considered position28 to then Chief Justice
Reynato S. Puno:
A reading of the 1987 Constitution would reveal that several
provisions were indeed adjusted as to be in tune with the shift I humbly reiterate my position that there should be only one
to bicameralism. One example is Section 4, Article VII, which representative of Congress in the JBC in accordance with
provides that a tie in the presidential election shall be broken Article VIII, Section 8 (1) of the 1987 Constitution x x x.
"by a majority of all the Members of both Houses of the
Congress, voting separately."20 Another is Section 8 thereof
which requires the nominee to replace the Vice-President to The aforesaid provision is clear and unambiguous and does
be confirmed "by a majority of all the Members of both Houses not need any further interpretation. Perhaps, it is apt to
of the Congress, voting separately."21 Similarly, under mention that the oft-repeated doctrine that "construction and
Section 18, the proclamation of martial law or the suspension interpretation come only after it has been demonstrated that
of the privilege of the writ of habeas corpus may be revoked application is impossible or inadequate without them."
or continued by the Congress, voting separately, by a vote of
at least a majority of all its Members."22 In all these Further, to allow Congress to have two representatives in the
provisions, the bicameral nature of Congress was recognized Council, with one vote each, is to negate the principle of
and, clearly, the corresponding adjustments were made as to equality among the three branches of government which is
how a matter would be handled and voted upon by its two enshrined in the Constitution.
Houses.
In view of the foregoing, I vote for the proposition that the
Thus, to say that the Framers simply failed to adjust Section Council should adopt the rule of single representation of
8, Article VIII, by sheer inadvertence, to their decision to shift Congress in the JBC in order to respect and give the right
FINALS CONSTITUTIONAL LAW I ACJUCO NOV 11, 2017 223

meaning to the above-quoted provision of the Constitution. Congress interacts with the other two co-equal branches of
(Emphases and underscoring supplied) government.

On March 14, 2007, then Associate Justice Leonardo A. It is more in keeping with the co-equal nature of the three
Quisumbing, also a JBC Consultant, submitted to the Chief governmental branches to assign the same weight to
Justice and ex-officio JBC Chairman his opinion,29 which considerations that any of its representatives may have
reads: regarding aspiring nominees to the judiciary. The
representatives of the Senate and the House of
8. Two things can be gleaned from the excerpts and citations Representatives act as such for one branch and should not
above: the creation of the JBC is intended to curtail the have any more quantitative influence as the other branches in
influence of politics in Congress in the appointment of judges, the exercise of prerogatives evenly bestowed upon the three.
and the understanding is that seven (7) persons will compose Sound reason and principle of equality among the three
the JBC. As such, the interpretation of two votes for Congress branches support this conclusion. [Emphases and
runs counter to the intendment of the framers. Such underscoring supplied]
interpretation actually gives Congress more influence in the
appointment of judges. Also, two votes for Congress would The argument that a senator cannot represent a member of
increase the number of JBC members to eight, which could the House of Representatives in the JBC and vice-versa is,
lead to voting deadlock by reason of even-numbered thus, misplaced. In the JBC, any member of Congress,
membership, and a clear violation of 7 enumerated members whether from the Senate or the House of Representatives, is
in the Constitution. (Emphases and underscoring supplied) constitutionally empowered to represent the entire Congress.
It may be a constricted constitutional authority, but it is not an
In an undated position paper,30 then Secretary of Justice absurdity.
Agnes VST Devanadera opined:
From this score stems the conclusion that the lone
As can be gleaned from the above constitutional provision, the representative of Congress is entitled to one full vote. This
JBC is composed of seven (7) representatives coming from pronouncement effectively disallows the scheme of splitting
different sectors. From the enumeration it is patent that each the said vote into half (1/2), between two representatives of
category of members pertained to a single individual only. Congress. Not only can this unsanctioned practice cause
Thus, while we do not lose sight of the bicameral nature of our disorder in the voting process, it is clearly against the essence
legislative department, it is beyond dispute that Art. VIII, of what the Constitution authorized. After all, basic and
Section 8 (1) of the 1987 Constitution is explicit and specific reasonable is the rule that what cannot be legally done directly
that "Congress" shall have only "xxx a representative." Thus, cannot be done indirectly. To permit or tolerate the splitting of
two (2) representatives from Congress would increase the one vote into two or more is clearly a constitutional
number of JBC members to eight (8), a number beyond what circumvention that cannot be countenanced by the Court.
the Constitution has contemplated. (Emphases and Succinctly put, when the Constitution envisioned one member
underscoring supplied) of Congress sitting in the JBC, it is sensible to presume that
this representation carries with him one full vote.
In this regard, the scholarly dissection on the matter by retired
Justice Consuelo Ynares-Santiago, a former JBC consultant, It is also an error for respondents to argue that the President,
is worth reiterating.31 Thus: in effect, has more influence over the JBC simply because all
of the regular members of the JBC are his appointees. The
principle of checks and balances is still safeguarded because
A perusal of the records of the Constitutional Commission the appointment of all the regular members of the JBC is
reveals that the composition of the JBC reflects the subject to a stringent process of confirmation by the
Commission’s desire "to have in the Council a representation Commission on Appointments, which is composed of
for the major elements of the community." xxx The ex-officio members of Congress.
members of the Council consist of representatives from the
three main branches of government while the regular
members are composed of various stakeholders in the Respondents’ contention that the current irregular
judiciary. The unmistakeable tenor of Article VIII, Section 8(1) composition of the JBC should be accepted, simply because
was to treat each ex-officio member as representing one co- it was only questioned for the first time through the present
equal branch of government. xxx Thus, the JBC was designed action, deserves scant consideration. Well-settled is the rule
to have seven voting members with the three ex-officio that acts done in violation of the Constitution no matter how
members having equal say in the choice of judicial nominees. frequent, usual or notorious cannot develop or gain
acceptance under the doctrine of estoppel or laches, because
once an act is considered as an infringement of the
xxx Constitution it is void from the very beginning and cannot be
the source of any power or authority.
No parallelism can be drawn between the representative of
Congress in the JBC and the exercise by Congress of its It would not be amiss to point out, however, that as a general
legislative powers under Article VI and constituent powers rule, an unconstitutional act is not a law; it confers no rights; it
under Article XVII of the Constitution. Congress, in relation to imposes no duties; it affords no protection; it creates no office;
the executive and judicial branches of government, is it is inoperative as if it has not been passed at all. This rule,
constitutionally treated as another co-equal branch in the however, is not absolute. Under the doctrine of operative
matter of its representative in the JBC. On the other hand, the facts, actions previous to the declaration of unconstitutionality
exercise of legislative and constituent powers requires the are legally recognized. They are not nullified. This is essential
Senate and the House of Representatives to coordinate and in the interest of fair play. To reiterate the doctrine enunciated
act as distinct bodies in furtherance of Congress’ role under in Planters Products, Inc. v. Fertiphil Corporation:32
our constitutional scheme. While the latter justifies and, in fact,
necessitates the separateness of the two Houses of Congress
as they relate inter se, no such dichotomy need be made when
FINALS CONSTITUTIONAL LAW I ACJUCO NOV 11, 2017 224

The doctrine of operative fact, as an exception to the general which reads, "This disposition is immediately executory," is
rule, only applies as a matter of equity and fair play. It nullifies hereby LIFTED.
the effects of an unconstitutional law by recognizing that the
existence of a statute prior to a determination of SO ORDERED.
unconstitutionality is an operative fact and may have
consequences which cannot always be ignored. The past
cannot always be erased by a new judicial declaration. The
doctrine is applicable when a declaration of unconstitutionality
will impose an undue burden on those who have relied on the
invalid law. Thus, it was applied to a criminal case when a
declaration of unconstitutionality would put the accused in
double jeopardy or would put in limbo the acts done by a
municipality in reliance upon a law creating it.33

Under the circumstances, the Court finds the exception


applicable in this case and holds that notwithstanding its
finding of unconstitutionality in the current composition of the
JBC, all its prior official actions are nonetheless valid.

Considering that the Court is duty bound to protect the


Constitution which was ratified by the direct action of the
Filipino people, it cannot correct what respondents perceive
as a mistake in its mandate. Neither can the Court, in the
exercise of its power to interpret the spirit of the Constitution,
read into the law something that is contrary to its express
provisions and justify the same as correcting a perceived
inadvertence. To do so would otherwise sanction the Court
action of making amendment to the Constitution through a
judicial pronouncement.

In other words, the Court cannot supply the legislative


omission. According to the rule of casus omissus "a case
omitted is to be held as intentionally omitted."34 "The principle
proceeds from a reasonable certainty that a particular person,
object or thing has been omitted from a legislative
enumeration."35 Pursuant to this, "the Court cannot under its
power of interpretation supply the omission even though the
omission may have resulted from inadvertence or because the
case in question was not foreseen or contemplated."36 "The
Court cannot supply what it thinks the legislature would have
supplied had its attention been called to the omission, as that
would be judicial legislation."37

Stated differently, the Court has no power to add another


member by judicial construction.

The call for judicial activism fails to stir the sensibilities of the
Court tasked to guard the Constitution against usurpation. The
Court remains steadfast in confining its powers in the sphere
granted by the Constitution itself. Judicial activism should
never be allowed to become judicial exuberance.38 In cases
like this, no amount of practical logic or convenience can
convince the Court to perform either an excision or an
insertion that will change the manifest intent of the Framers.
To broaden the scope of congressional representation in the
JBC is tantamount to the inclusion of a subject matter which
was not included in the provision as enacted. True to its
constitutional mandate, the Court cannot craft and tailor
constitutional provisions in order to accommodate all of
situations no matter how ideal or reasonable the proposed
solution may sound. To the exercise of this intrusion, the Court
declines.

WHEREFORE, the Motion for Reconsideration filed by


respondents is hereby DENIED.

The suspension of the effects of the second paragraph of the


dispositive portion of the July 17, 2012 Decision of the Court,
FINALS CONSTITUTIONAL LAW I ACJUCO NOV 11, 2017 225

G.R. No. 213181 August 19, 2014 Lourdes P.A. Sereno (Chief Justice Sereno),manifested that
she would be invoking Section 2, Rule 10 of JBC-0094 against
FRANCIS H. JARDELEZA Petitioner, him. Jardeleza was then directed to "make himself available"
before the JBC on June 30, 2014, during which he would be
informed of the objections to his integrity.
vs.
Consequently, Jardeleza filed a letter-petition (letter-petition)5
CHIEF JUSTICE MARIA LOURDES P. A. SERENO, THE praying that the Court, in the exercise of itsconstitutional
JUDICIAL AND BAR COUNCIL AND EXECUTIVE power of supervision over the JBC, issue an order: 1) directing
SECRETARY PAQUITO N. OCHOA, JR., Respondents. the JBC to give him at least five (5) working days written notice
of any hearing of the JBC to which he would be summoned;
DECISION and the said notice to contain the sworn specifications of the
charges against him by his oppositors, the sworn statements
MENDOZA, J.: of supporting witnesses, if any, and copies of documents in
support of the charges; and notice and sworn statements shall
be made part of the public record of the JBC; 2) allowing him
Once again, the Couii is faced with a controversy involving the to cross-examine his oppositors and supporting witnesses, if
acts of an independent body, which is considered as a any, and the cross-examination to be conducted in public,
constitutional innovation the Judicial and Bar Council (JBC). It under the same conditions that attend the publicinterviews
is not the first time that the Court is called upon to settle legal held for all applicants; 3) directing the JBC to reset the hearing
questions surrounding the JBC's exercise of its constitutional scheduled on June 30, 2014 to another date; and 4) directing
mandate. In De Castro v. JBC,1 the Court laid to rest issues the JBC to disallow Chief Justice Sereno from participating in
such as the duty of the JBC to recommend prospective the voting on June 30,2014 or at any adjournment thereof
nominees for the position of Chief Justice vis-à-vis the where such vote would be taken for the nominees for the
appointing power of the President, the period within which the position vacated by Associate Justice Abad.
same may be exercised, and the ban on midnight
appointments as set forth in the Constitution. In Chavez v.
JBC,2 the Court provided an extensive discourse on During the June 30, 2014 meeting of the JBC, sansJardeleza,
constitutional intent as to the JBC’s composition and incumbent Associate Justice Antonio T. Carpio (Associate
membership. Justice Carpio) appeared as a resource person to shed light
on a classified legal memorandum (legal memorandum) that
would clarify the objection to Jardeleza’s integrity as posed by
This time, however, the selection and nomination process Chief Justice Sereno. According to the JBC, Chief Justice
actually undertaken by the JBC is being challenged for being Sereno questioned Jardeleza’s ability to discharge the duties
constitutionally infirm. The heart of the debate lies not only on of his office as shown in a confidential legal memorandum
the very soundness and validity of the application of JBC rules over his handling of an international arbitration case for the
but also the extent of its discretionary power. More government.
significantly, this case of first impression impugns the end-
result of its acts - the shortlistfrom which the President
appoints a deserving addition to the Highest Tribunal of the Later, Jardeleza was directed to one of the Court’s ante-rooms
land. where Department of Justice Secretary Leila M. De Lima
(Secretary De Lima) informed him that Associate Justice
Carpio appeared before the JBC and disclosed confidential
To add yet another feature of noveltyto this case, a member information which, to Chief Justice Sereno, characterized his
of the Court, no less than the Chief Justice herself, was being integrity as dubious. After the briefing, Jardeleza was
impleaded as party respondent. summoned by the JBC at around 2:00o’clock in the afternoon.

The Facts Jardeleza alleged that he was asked by Chief Justice Sereno
if he wanted to defend himself against the integrity issues
The present case finds its genesis from the compulsory raised against him. He answered that he would defend himself
retirement of Associate Justice Roberto Abad (Associate provided that due process would be observed. Jardeleza
Justice Abad) last May 22, 2014. Before his retirement, on specifically demanded that Chief Justice Sereno execute a
March 6, 2014, in accordance with its rules,3 the JBC sworn statement specifying her objectionsand that he be
announced the opening for application or recommendation for afforded the right to cross-examine her in a public hearing. He
the said vacated position. requested that the same directive should also be imposed on
Associate Justice Carpio. As claimed by the JBC,
Representative Niel G. Tupas Jr. also manifested that he
On March 14, 2014, the JBC received a letter from Dean
wanted to hear for himself Jardeleza’s explanation on the
Danilo Concepcion of the University of the Philippines
matter. Jardeleza, however, refused as he would not be lulled
nominating petitioner Francis H. Jardeleza (Jardeleza),
intowaiving his rights. Jardeleza then put into record a written
incumbent Solicitor General of the Republic, for the said
statement6 expressing his views on the situation and
position. Upon acceptance of the nomination, Jardeleza was
requested the JBC to defer its meeting considering that the
included in the names of candidates, as well as in the
Court en banc would meet the next day to act on his pending
schedule of public interviews. On May 29, 2014, Jardeleza
letter-petition. At this juncture, Jardeleza was excused.
was interviewed by the JBC.

Later in the afternoon of the sameday, and apparently denying


It appears from the averments in the petition that on June 16
Jardeleza’s request for deferment of the proceedings, the JBC
and 17, 2014, Jardeleza received telephone callsfrom former
continued its deliberations and proceeded to vote for the
Court of Appeals Associate Justice and incumbent JBC
nominees to be included in the shortlist. Thereafter, the JBC
member, Aurora Santiago Lagman (Justice Lagman), who
releasedthe subject shortlist of four (4) nominees which
informed him that during the meetings held on June 5 and 16,
included: Apolinario D. Bruselas, Jr. with six (6) votes, Jose C.
2014, Chief Justice and JBC ex-officioChairperson, Maria
FINALS CONSTITUTIONAL LAW I ACJUCO NOV 11, 2017 226

Reyes, Jr. with six (6) votes, Maria Gracia M. Pulido Tan with Jardeleza was deprived of his right to due process. In turn, the
five (5) votes, and Reynaldo B. Daway with four (4) votes.7 JBC violated his right to due process when he was simply
ordered to make himself available on the June 30, 2014
As mentioned in the petition, a newspaper article was later meeting and was told that the objections to his integrity would
published in the online portal of the Philippine Daily Inquirer, be made known to him on the same day. Apart from mere
stating that the Court’s Spokesman, Atty. Theodore Te, verbal notice (by way of a telephone call) of the invocation of
revealed that there were actually five (5) nominees who made Section 2, Rule 10 of JBC-009 against his application and not
it to the JBC shortlist, but one (1) nominee could not be on the accusations against him per se, he was deprived of an
included because of the invocation of Rule 10, Section 2 of opportunity to mount a proper defense against it. Not only did
the JBC rules. the JBC fail to ventilate questions on his integrity during his
public interview, he was also divested of his rights as an
applicant under Sections 3 and 4, Rule 4, JBC-009, to wit:
In its July 8, 2014 Resolution, the Court noted Jardeleza’s
letterpetition in view of the transmittal of the JBC list of
nominees to the Office of the President, "without prejudice to Section 3. Testimony of parties. – The Council may receive
any remedy available in law and the rules that petitioner may written opposition to an applicant on the ground of his moral
still wish to pursue."8 The said resolution was accompanied fitness and, at its discretion, the Council may receive the
by an extensive Dissenting Opinion penned by Associate testimony of the oppositor at a hearing conducted for the
Justice Arturo D. Brion,9 expressing his respectful purpose, with due notice to the applicant who shall be allowed
disagreement as to the position taken by the majority. to cross-examine the oppositor and to offer countervailing
evidence.
The Petition
Section 4. Anonymous Complaints. – Anonymous complaints
against an applicant shall not be given due course, unless
Perceptibly based on the aforementioned resolution’s there appears on its face a probable cause sufficient to
declaration as to his availment of a remedy in law, Jardeleza engender belief that the allegations may be true. In the latter
filed the present petition for certiorari and mandamus under case, the Council may direct a discreet investigation or require
Rule 65 of the Rules of Court with prayer for the issuance of a the applicant to comment thereon in writing or during the
Temporary Restraining Order (TRO), seeking to compel the interview.
JBC to include him in the list ofnominees for Supreme Court
Associate Justice viceAssociate Justice Abad, on the grounds
that the JBC and Chief Justice Sereno acted in grave abuse His lack of knowledge as to the identity of his accusers (except
of discretion amounting to lack or excess of jurisdiction in for yet again, the verbalinformation conveyed to him that
excluding him, despite having garnered a sufficient number of Associate Justice Carpio testified against him) and as to the
votes to qualify for the position. nature of the very accusations against him caused him to
suffer from the arbitrary action by the JBC and Chief Justice
Sereno. The latter gravely abused her discretion when she
Notably, Jardeleza’s petition decries that despite the obvious acted as prosecutor, witness and judge,thereby violating the
urgency of his earlier letter-petition and its concomitant filing very essence of fair play and the Constitution itself. In his
on June 25, 2014, the same was raffled only on July 1, 2014 words: "the sui generis nature of JBC proceedings does not
or a day after the controversial JBC meeting. By the time that authorize the Chief Justice to assume these roles, nor does it
his letter-petition was scheduled for deliberation by the Court dispense with the need to honor petitioner’s right to due
en bancon July 8, 2014, the disputedshortlist had already process."10
been transmitted to the Office of the President. He
attributedthis belated action on his letter-petition to Chief
Justice Sereno, whose action on such matters, especially B. The JBC committed grave abuse of discretion in excluding
those impressed withurgency, was discretionary. Jardeleza from the shortlist of nominees, in violation of its own
rules. The "unanimity requirement" provided under Section 2,
Rule10 of JBC-009 does not find application when a member
An in-depth perusal of Jardeleza’s petition would reveal that of the JBC raises an objection to an applicant’s integrity. Here,
his resort to judicial intervention hinges on the alleged illegality the lone objector constituted a part of the membership of the
of his exclusion from the shortlist due to: 1) the deprivation of body set to vote. The lone objector could be completely
his constitutional right to due process; and 2) the JBC’s capable oftaking hostage the entire voting process by the
erroneous application, if not direct violation, of its own rules. mere expediency of raising an objection. Chief Justice
Suffice it to say, Jardelezadirectly ascribes the supposed Sereno’s interpretation of the rule would allow a situation
violation of his constitutional rights tothe acts of Chief Justice where all thata member has to do to veto other votes,
Sereno in raising objections against his integrity and the including majority votes, would be to object to the qualification
manner by which the JBC addressed this challenge to his of a candidate, without need for factual basis.
application, resulting in his arbitrary exclusion from the list of
nominees.
C. Having secured the sufficient number of votes, it was
ministerial on the part of the JBC to include Jardeleza in the
Jardeleza’s Position subject shortlist.Section 1, Rule 10 of JBC-009 provides that
a nomination for appointment to a judicial position requires the
For a better understanding of the above postulates proffered affirmative vote of at least a majority of all members of the
in the petition, the Court hereunder succinctlysummarizes JBC. The JBC cannot disregard its own rules. Considering
Jardeleza’s arguments, as follows: that Jardeleza was able to secure four (4) out of six (6) votes,
the only conclusion is that a majority of the members of the
A. Chief Justice Sereno and the JBC violated Jardeleza’s right JBC found him to be qualified for the position of Associate
to due process in the events leading up to and during the vote Justice.
on the shortlist last June 30, 2014. When accusations against
his integrity were made twice, ex parte, by Chief Justice D. The unlawful exclusion ofthe petitioner from the subject
Sereno, without informing him of the nature and cause thereof shortlist impairs the President’s constitutional power to
and without affording him an opportunity to be heard, appoint.Jardeleza’s exclusion from the shortlist has unlawfully
FINALS CONSTITUTIONAL LAW I ACJUCO NOV 11, 2017 227

narrowed the President’s choices. Simply put, the President of the selection process. Each member of the JBC relies on
would be constrained to choose from among four (4) his or her own appreciation of the circumstances and
nominees, when five (5) applicants rightfully qualified for the qualifications of applicants.
position. This limits the President to appoint a member of the
Court from a list generated through a process tainted with The JBC then proceeded to defend adherence to its standing
patent constitutional violations and disregard for rules of rules. As a general rule, an applicant is included in the shortlist
justice and fair play. Until these constitutional infirmities are when he or she obtains an affirmative vote of at least a
remedied, the petitioner has the right to prevent the majority of all the members of the JBC. When Section 2, Rule
appointment of an Associate Justice viceAssociate Justice 10 of JBC-009,however, is invoked because an applicant’s
Abad. integrity is challenged, a unanimous vote is required. Thus,
when Chief Justice Sereno invoked the saidprovision,
Comment of the JBC Jardeleza needed the affirmative vote of all the JBC members
tobe included in the shortlist. In the process, Chief Justice
On August 11, 2014, the JBC filed its comment contending Sereno’s vote against Jardeleza was not counted. Even then,
that Jardeleza’s petition lacked proceduraland substantive he needed the votes of the five(5) remaining members. He
bases that would warrant favorable action by the Court. For only got four (4) affirmative votes. As a result,he was not
the JBC, certiorariis only available against a tribunal, a board included in the shortlist. Applicant Reynaldo B. Daway, who
or an officer exercising judicial or quasijudicial functions.11 gotfour (4) affirmative votes, was included in the shortlist
The JBC, in its exercise of its mandate to recommend because his integrity was not challenged. As to him, the
appointees to the Judiciary, does not exercise any of these "majority rule" was considered applicable.
functions. In a pending case,12 Jardeleza himself, as one of
the lawyers for the government, argued in this wise: Lastly, the JBC rued that Jardeleza sued the respondents in
Certioraricannot issue against the JBC in the implementation his capacity as Solicitor General. Despiteclaiming a prefatory
of its policies. appearance in propria persona, all pleadings filed with the
Court were signed in his official capacity. In effect, he sued
In the same vein, the remedy of mandamusis incorrect. the respondents to pursue a purely private interest while
Mandamus does not lie to compel a discretionary act. For it to retaining the office of the Solicitor General. By suing the very
prosper, a petition for mandamus must, among other things, parties he was tasked by law to defend, Jardeleza knowingly
show that the petitioner has a clear legal right to the act placed himself in a situation where his personal interests
demanded. In Jardeleza’s case, there is no legal right to be collided against his public duties, in clear violation of the Code
included in the list of nominees for judicial vacancies. of Professional Responsibility and Code of Professional
Possession of the constitutional and statutory qualifications Ethics. Moreover, the respondents are all public officials being
for appointment to the Judiciary may not be used to legally sued in their official capacity. By retaining his title as Solicitor
demand that one’s name be included in the list of candidates General, and suing in the said capacity, Jardeleza filed a suit
for a judicial vacancy. One’s inclusion in the shortlist is strictly against his own clients, being the legal defender of the
within the discretion of the JBC. government and its officers. This runs contrary to the fiduciary
relationship sharedby a lawyer and his client.
Anent the substantive issues, the JBC mainly denied that
Jardeleza was deprived of due process. The JBC reiterated In opposition to Jardeleza’s prayer for the issuance of a TRO,
that Justice Lagman, on behalf of the JBC en banc, called the JBC called to mind the constitutional period within which a
Jardeleza and informed him that Chief Justice Sereno would vacancy in the Court must be filled. As things now stand, the
be invoking Section 2, Rule 10 of JBC-009 due to a question President has until August 20, 2014 to exercise his
on his integrity based on the way he handled a very important appointment power which cannot be restrained by a TRO or
case for the government. Jardeleza and Justice Lagman an injunctive suit.
spoke briefly about the case and his general explanation on
how he handled the same. Secretary De Lima likewise Comment of the Executive Secretary
informed him about the content of the impending objection
against his application. On these occasions, Jardeleza agreed In his Comment, Executive Secretary Paquito N. Ochoa Jr.
to explain himself. Come the June 30, 2014 meeting, however, (Executive Secretary)raised the possible unconstitutionality of
Jardeleza refused to shed light on the allegations against Section 2, Rule 10 of JBC-009, particularly the imposition ofa
him,as he chose to deliver a statement, which, in essence, higher voting threshold in cases where the integrity of an
requested that his accuser and her witnesses file sworn applicant is challenged. It is his position that the subject JBC
statements so that he would know of the allegations against rule impairs the body’s collegial character, which essentially
him, that he be allowed to cross-examine the witnesses;and operates on the basis of majority rule. The application of
that the procedure be done on record and in public. Section 2, Rule 10 of JBC-009 gives rise to a situation where
all that a member needs to do, in order to disqualify an
In other words, Jardeleza was given ample opportunity to be applicant who may well have already obtained a majority vote,
heard and to enlighten each member of the JBC on the issues is to object to his integrity. In effect, a member who invokes
raised against him prior to the voting process. His request for the said provision is given a veto powerthat undermines the
a sworn statement and opportunity to cross-examine is not equal and full participation of the other members in the
supported by a demandable right. The JBC is not a fact-finding nomination process. A lone objector may then override the will
body. Neitheris it a court nor a quasi-judicial agency. The ofthe majority, rendering illusory, the collegial nature of the
members are notconcerned with the determination of his guilt JBC and the very purpose for which it was created— to shield
or innocence of the accusations against him. Besides, the appointment process from political maneuvering. Further,
Sections 3 and 4, Rule 10,JBC-009 are merely directory as Section 2, Rule 10 of JBC-009 may beviolative of due process
shown by the use of the word "may." Even the conduct of a for it does not allow an applicant any meaningful opportunity
hearing to determine the veracity of an opposition is to refute the challenges to his integrity. While other provisions
discretionary on the JBC. Ordinarily, if there are other ways of of the JBC rules provide mechanisms enabling an applicant to
ascertaining the truth or falsity of an allegation or opposition, comment on an opposition filed against him, the subject rule
the JBC would not call a hearing in order to avoid undue delay does not afford the same opportunity. In this case, Jardeleza’s
FINALS CONSTITUTIONAL LAW I ACJUCO NOV 11, 2017 228

allegations as to the events which transpired on June 30, 2014 the bleeding from which the gaping wound presented to the
obviously show that he was neither informed ofthe Court suffers.
accusations against him nor given the chance to muster a
defense thereto. The issues for resolution are:

The Executive Secretary then offered a supposition: granting I.


that the subject provision is held to be constitutional, the
"unanimity rule" would only be operative when the objector is
not a member of the JBC. It is only in this scenario where the WHETHER OR NOT THE COURT CAN ASSUME
voting ofthe body would not be rendered inconsequential. In JURISDICTION AND GIVE DUECOURSE TO THE
the event that a JBC member raised the objection, what SUBJECT PETITION FOR CERTIORARI AND MANDAMUS
should have been applied is the general rule of a majority vote, (WITH APPLICATION FOR A TEMPORARY RESTRAINING
where any JBC member retains their respective reservations ORDER).
to an application with a negative vote. Corollary thereto, the
unconstitutionality of the said rule would necessitate the II
inclusion of Jardeleza in the shortlist submitted to the
President. WHETHER OR NOT THE ISSUES RAISED AGAINST
JARDELEZA BEFIT "QUESTIONS OR CHALLENGES ON
Other pleadings INTEGRITY" AS CONTEMPLATED UNDER SECTION 2,
RULE 10 OF JBC-009.
On August 12, 2014, Jardeleza was given the chance to refute
the allegations of the JBC in its Comment. He submitted his II.
Reply thereto on August 15, 2014. A few hours thereafter,
orbarely ten minutes prior to the closing of business, the Court WHETHER OR NOT THE RIGHT TO DUE PROCESS IS
received the Supplemental Comment-Reply of the JBC, this AVAILABLE IN THE COURSE OF JBC PROCEEDINGS IN
time with the attached minutes of the proceedings that led to CASES WHERE AN OBJECTION OR OPPOSITION TO AN
the filing of the petition,and a detailed "Statementof the Chief APPLICATION IS RAISED.
Justice on the Integrity Objection."13 Obviously, Jardeleza’s
Reply consisted only of his arguments against the JBC’s
original Comment, as it was filed prior to the filing of the III.
Supplemental Comment-Reply.
WHETHER OR NOT PETITIONER JARDELEZA MAY BE
At the late stage of the case, two motions to admit comments- INCLUDED IN THE SHORTLIST OF NOMINEES
inintervention/oppositions-in-intervention were filed. One was SUBMITTED TO THE PRESIDENT.
by Atty. Purificacion S. Bartolome-Bernabe, purportedly the
President of the Integrated Bar of the Philippines-Bulacan The Court’s Ruling
Chapter. This pleading echoed the position of the JBC.14
I – Procedural Issue: The Court has constitutional bases to
The other one was filed by Atty. Reynaldo A. Cortes, assume jurisdiction over the case
purportedly a former President of the IBP Baguio-Benguet
Chapter and former Governor of the IBP-Northern Luzon. It
A - The Court’s Power of Supervision over the JBC
was coupled with a complaint for disbarment against
Jardeleza primarily for violations of the Code of Professional
Responsibility for representing conflicting interests.15 Section 8, Article VIII of the 1987 Constitution provides for the
creation of the JBC. The Court was given supervisory
authority over it. Section 8 reads:
Both motions for intervention weredenied considering that
time was of the essence and their motions were merely
reiterative of the positions of the JBC and were perceived to Section 8.
be dilatory. The complaint for disbarment, however, was re-
docketed as a separate administrative case. A Judicial and Bar Council is hereby created under the
supervision of the Supreme Courtcomposed of the Chief
The Issues Justice as ex officio Chairman, the Secretary of Justice, and a
representative of the Congress as ex officio Members, a
representative of the Integrated Bar, a professor of law, a
Amidst a myriad of issues submitted by the parties, most of
retired Member of the Supreme Court, and a representative of
which are interrelated such that the resolution of one issue
the private sector. [Emphasis supplied]
would necessarily affect the conclusion as to the others, the
Court opts to narrow down the questions to the very source of
the discord - the correct application of Section 2, Rule 10 JBC- As a meaningful guidepost, jurisprudence provides the
009 and its effects, if any, on the substantive rights of definition and scope of supervision. It is the power of
applicants. oversight, or the authority to see that subordinate officers
perform their duties.It ensures that the laws and the rules
governing the conduct of a government entity are observed
The Court is not unmindful of the fact that a facial scrutiny of
and complied with. Supervising officials see to it that rules are
the petition does not directly raise the unconstitutionality of the
followed, but they themselves do not lay down such rules, nor
subject JBC rule. Instead, it bewails the unconstitutional
do they have the discretion to modify or replace them. If the
effects of its application. It is only from the comment of the
rules are not observed, they may order the work done or
Executive Secretary where the possible unconstitutionality of
redone, but only to conform to such rules. They may not
the rulewas brought to the fore. Despite this milieu, a practical
prescribe their own manner of execution of the act. They have
approach dictatesthat the Court must confront the source of
FINALS CONSTITUTIONAL LAW I ACJUCO NOV 11, 2017 229

no discretion on this matter except to see to it that the rules latter does not exercise judicial, quasi-judicial or ministerial
are followed.16 functions.19

Based on this, the supervisory authority of the Court over the In a case like this, where constitutional bearings are too
JBC covers the overseeing of compliance with its rules. In this blatant to ignore, the Court does not find passivity as an
case, Jardeleza’s principal allegations in his petition merit the alternative. The impassemust be overcome.
exercise of this supervisory authority.
II – Substantial Issues
B- Availability of the Remedy of Mandamus
Examining the Unanimity Rule of the JBC in cases where an
The Court agrees with the JBC that a writ of mandamus is not applicant’s integrity is challenged
available. "Mandamuslies to compel the performance, when
refused, of a ministerial duty, but not to compel the The purpose of the JBC’s existence is indubitably rooted in
performance of a discretionary duty. Mandamuswill not issue the categorical constitutional declaration that"[a] member of
to control or review the exercise of discretion of a public officer the judiciary must be a person of proven competence,
where the law imposes upon said public officer the right and integrity, probity, and independence." To ensure the fulfillment
duty to exercise his judgment in reference to any matter in of these standards in every member of the Judiciary, the JBC
which he is required to act. It is his judgment that is to be has been tasked toscreen aspiring judges and justices,
exercised and not that of the court.17 There is no question among others, making certain that the nominees submitted to
that the JBC’s duty to nominate is discretionary and it may not the President are all qualified and suitably best for
becompelled to do something. appointment. In this way, the appointing process itself is
shieldedfrom the possibility of extending judicial appointment
C- Availability of the Remedy of Certiorari to the undeserving and mediocre and, more importantly, to the
ineligible or disqualified.
Respondent JBC opposed the petition for certiorarion the
ground that it does not exercise judicial or quasi-judicial In the performance of this sacred duty, the JBC itself admits,
functions. Under Section 1 of Rule 65, a writ of certiorariis as stated in the "whereas clauses" of JBC-009, that
directed against a tribunal exercising judicial or quasi-judicial qualifications such as "competence, integrity, probity and
function. "Judicial functions are exercised by a body or officer independence are not easily determinable as they are
clothed with authority to determine what the law is and what developed and nurtured through the years." Additionally, "it is
the legal rights of the parties are with respect to the matter in not possible or advisable to lay down iron-clad rules to
controversy. Quasijudicial function is a term that applies to the determine the fitness of those who aspire to become a Justice,
action or discretion of public administrative officers or bodies Judge, Ombudsman or Deputy Ombudsman." Given this
given the authority to investigate facts or ascertain the realistic situation, there is a need "to promote stability and
existence of facts, hold hearings, and draw conclusions from uniformity in JBC’s guiding precepts and principles." A set of
them as a basis for their official action using discretion of a uniform criteria had to be established in the ascertainment of
judicial nature."18 It asserts that in the performance of its "whether one meets the minimum constitutional qualifications
function of recommending appointees for the judiciary, the and possesses qualities of mind and heart expected of him"
JBC does not exercise judicial or quasijudicial functions. and his office. Likewise for the sake oftransparency of its
Hence, the resort tosuch remedy to question its actions is proceedings, the JBC had put these criteria in writing, now in
improper. the form of JBC-009. True enough, guidelines have been set
inthe determination of competence,"20 "probity and
In this case, Jardeleza cries that although he earned a independence,"21"soundness of physical and mental
qualifying number of votes in the JBC, it was negated by the condition,22 and "integrity."23
invocation of the "unanimity rule" on integrity in violation of his
right to due process guaranteed not only by the Constitution As disclosed by the guidelines and lists of recognized
but by the Council’s own rules. For said reason, the Court is evidence of qualification laid down in JBC-009, "integrity" is
of the position that it can exercise the expanded judicial power closely related to, or if not, approximately equated to an
of review vestedupon it by the 1987 Constitution. Thus: applicant’s good reputation for honesty, incorruptibility,
irreproachableconduct, and fidelity to sound moral and ethical
Article VIII. standards. That is why proof of an applicant’s reputation may
be shown in certifications or testimonials from reputable
government officials and non-governmental organizations and
Section 1. The judicial power is vested in one Supreme Court clearances from the courts, National Bureau of Investigation,
and in such lower courts as may be established by law. and the police, among others. In fact, the JBC may even
conduct a discreet background check and receive feedback
Judicial power includes the duty of the courts of justice to from the public on the integrity, reputation and character of the
settle actual controversies involving rights which are legally applicant, the merits of which shall be verifiedand checked. As
demandable and enforceable, and to determine whether or a qualification, the term is taken to refer to a virtue, such that,
not there has been a grave abuse of discretion amounting to "integrity is the quality of person’s character."24
lack or excess of jurisdiction on the part of any branch or
instrumentality of the Government. The foregoing premise then begets the question: Does Rule
2, Section 10 of JBC-009, in imposing the "unanimity rule,"
It has been judicially settled that a petition for certiorari is a contemplate a doubt on the moral character of an applicant?
proper remedy to question the act of any branch or Section 2, Rule 10 of JBC-009 provides:
instrumentality of the government on the ground of grave
abuse of discretion amounting to lack or excess of jurisdiction SEC. 2. Votes required when integrity of a qualified applicant
by any branch orinstrumentality of the government, even if the is challenged. - In every case where the integrity of an
applicant who is not otherwise disqualified for nomination is
FINALS CONSTITUTIONAL LAW I ACJUCO NOV 11, 2017 230

raised or challenged, the affirmative vote of all the Members Doeshis adoption of a specific legal strategy in the handling of
of the Council must be obtained for the favorable a case bring forth a relevant and logical challenge against his
consideration of his nomination. moral character? Does the "unanimity rule" apply in cases
where the main point of contention is the professional
A simple reading of the above provision undoubtedly elicits judgment sans charges or implications of immoral or corrupt
the rule that a higher voting requirement is absolute in cases behavior?
where the integrity of an applicant is questioned. Simply put,
when an integrity question arises, the voting requirement for The Court answers these questions in the negative.
his or her inclusion as a nominee to a judicial post becomes
"unanimous" instead of the "majority vote" required in the While Chief Justice Sereno claims that the invocation of
preceding section.25 Considering that JBC-009 employs the Section 2, Rule 10 of JBC-009 was not borne out ofa mere
term "integrity" as an essential qualification for appointment, variance of legal opinion but by an "act of disloyalty"
and its doubtful existence in a person merits a higher hurdle committed by Jardeleza in the handling of a case, the fact
to surpass, that is, the unanimous vote of all the members of remains that the basis for her invocation of the rule was the
the JBC, the Court is of the safe conclusion that "integrity" as "disagreement" in legal strategy as expressed by a group of
used in the rules must be interpreted uniformly. Hence, international lawyers. The approach taken by Jardeleza in that
Section 2, Rule 10 of JBC-009 envisions only a situation case was opposed to that preferred by the legal team. For said
where an applicant’s moral fitness is challenged. It follows reason, criticism was hurled against his "integrity." The
then that the "unanimity rule" only comes into operation when invocation of the "unanimity rule" on integrity traces its roots
the moral character of a person is put in issue. It finds no to the exercise ofhis discretion as a lawyer and nothing else.
application where the question is essentially unrelated to an No connection was established linking his choice of a legal
applicant’s moral uprightness. strategy to a treacherous intent to trounce upon the country’s
interests or to betray the Constitution.
Examining the "questions of integrity" made against Jardeleza
Verily, disagreement in legal opinion is but a normal, if not an
The Court will now examine the propriety of applying Section essential form of, interaction among members of the legal
2, Rule 10 of JBC-009 to Jardeleza’s case. community. A lawyer has complete discretion on whatlegal
strategy to employ in a case entrusted to him28provided that
The minutes of the JBC meetings, attached to the he lives up tohis duty to serve his client with competence and
Supplemental Comment-Reply, reveal that during the June diligence, and that he exert his best efforts to protect the
30, 2014 meeting, not only the question on his actuations in interests of his client within the bounds of the law.
the handling of a case was called for explanation by the Chief Consonantly, a lawyer is not an insurer of victory for clients he
Justice, but two other grounds as well tending to show his lack represents. An infallible grasp of legal principles and
of integrity: a supposed extra-marital affair in the past and technique by a lawyer is a utopian ideal. Stripped of a clear
alleged acts of insider trading.26 showing of gross neglect, iniquity, or immoral purpose, a
strategy of a legal mind remains a legal tactic acceptable to
some and deplorable to others. It has no direct bearing on his
Against this factual backdrop, the Court notes that the initial moral choices.
or original invocation of Section 2, Rule 10 of JBC-009 was
grounded on Jardeleza’s "inability to discharge the duties of
his office" as shown in a legal memorandum related to As shown in the minutes, the other JBC members expressed
Jardeleza’s manner of representing the government in a legal their reservations on whether the ground invoked by Chief
dispute. The records bear that the "unanimity rule" was initially Justice Sereno could be classified as a "question of integrity"
invoked by Chief Justice Sereno during the JBC meeting held under Section 2, Rule 10 of JBC-009.29 These reservations
on June 5, 2014, where she expressed her position that were evidently sourced from the factthat there was no clear
Jardeleza did not possess the integrity required tobe a indication that the tactic was a "brainchild" of Jardeleza, as it
member of the Court.27 In the same meeting, the Chief might have been a collective idea by the legal team which
Justice shared withthe other JBC members the details of initially sought a different manner of presenting the country’s
Jardeleza’s chosen manner of framing the government’s arguments, and there was no showing either of a corrupt
position in a case and how this could have been detrimental purpose on his part.30 Even Chief Justice Sereno was not
to the national interest. certain that Jardeleza’s acts were urged by politicking or lured
by extraneous promises.31Besides, the President, who has
the final say on the conduct of the country’s advocacy in the
In the JBC’s original comment, the details of the Chief case, has given no signs that Jardeleza’s action constituted
Justice’s claim against Jardeleza’s integrity were couched in disloyalty or a betrayal of the country’s trust and interest.
general terms. The particulars thereof were only supplied to While this point does notentail that only the President may
the Court in the JBC’s Supplemental Comment-Reply. challenge Jardeleza’s doubtful integrity, itis commonsensical
Apparently, the JBC acceded to Jardeleza’s demand to make to assume that he is in the best position to suspect a
the accusations against him public. At the outset, the JBC treacherous agenda. The records are bereft of any information
declined to raise the fine points of the integrity question in its that indicatesthis suspicion. In fact, the Comment of the
original Comment due to its significant bearing on the Executive Secretary expressly prayed for Jardeleza’s
country’s foreign relations and national security. At any rate, inclusion in the disputed shortlist.
the Court restrains itself from delving into the details thereof
in this disposition. The confidential nature of the document
cited therein, which requires the observance of utmost The Court notes the zeal shown by the Chief Justice regarding
prudence, preclude a discussion that may possibly affect the international cases, given her participation in the PIATCO
country’s position in a pending dispute. case and the Belgian Dredging case. Her efforts inthe
determination of Jardeleza’s professional background, while
commendable, have not produced a patent demonstration of
Be that as it may, the Court has to resolve the standing a connection betweenthe act complained of and his integrity
questions: Does the original invocation of Section 2, Rule 10 as a person. Nonetheless, the Court cannot consider her
of JBC-009 involve a question on Jardeleza’s integrity? invocation of Section 2, Rule 10 of JBC-009 as conformably
FINALS CONSTITUTIONAL LAW I ACJUCO NOV 11, 2017 231

within the contemplation of the rule. To fall under Section 2, devices and deceptive practices, including insider trading,
Rule 10 of JBC-009, there must be a showing that the act throw a monkey wrench right into the heart of the securities
complained of is, at the least, linked to the moral character of industry. Whensomeone trades inthe market with unfair
the person and not to his judgment as a professional. What advantage in the form of highly valuable secret inside
this disposition perceives, therefore, is the inapplicability of information, all other participants are defrauded. All of the
Section 2, Rule 10 of JBC-009 to the original ground of its mechanisms become worthless. Given enough of stock
invocation. marketscandals coupled with the related loss of faith in the
market, such abuses could presage a severe drain of capital.
As previously mentioned, Chief Justice Sereno raised the And investors would eventuallyfeel more secure with their
issues of Jardeleza’s alleged extra-marital affair and acts of money invested elsewhere.41 In its barest essence, insider
insider-trading for the first time onlyduring the June 30, 2014 trading involves the trading of securities based on knowledge
meeting of the JBC. As can be gleaned from the minutes of of material information not disclosed to the public at the time.
the June 30, 2014 meeting, the inclusion of these issues had Clearly, an allegation of insider trading involves the propensity
its origin from newspaper reports that the Chief Justice might of a person toengage in fraudulent activities that may speak
raise issues of "immorality" against Jardeleza.32 The Chief of his moral character.
Justice then deduced that the "immorality" issue referred to by
the media might have been the incidents that could have These two issues can be properly categorized as "questions
transpired when Jardeleza was still the General Counsel of on integrity" under Section 2, Rule 10 of JBC-009. They fall
San Miguel Corporation. She stated that inasmuch as the JBC within the ambit of "questions on integrity." Hence, the
had the duty to "take every possible step to verify the "unanimity rule" may come into operation as the subject
qualification of the applicants," it might as well be clarified.33 provision is worded.

Do these issues fall within the purview of "questions on The Availability of Due Process in the
integrity" under Section 2, Rule 10 of JBC-009? The Court
nods in assent. These are valid issues. Proceedings of the JBC

This acquiescence is consistent with the Court’s discussion In advocacy of his position, Jardeleza argues that: 1] he
supra. Unlike the first ground which centered onJardeleza’s should have been informed of the accusations against him in
stance on the tactical approach in pursuing the case for the writing; 2] he was not furnished the basis of the accusations,
government, the claims of an illicit relationship and acts of that is, "a very confidential legal memorandum that clarifies
insider trading bear a candid relation to his moral character. the integrityobjection"; 3] instead of heeding his request for an
Jurisprudence34 is replete with cases where a lawyer’s opportunity to defend himself, the JBC considered his refusal
deliberate participation in extra-marital affairs was considered to explain, during the June 30, 2014 meeting, as a waiver of
as a disgraceful stain on one’s ethical and moral principles. his right to answer the unspecified allegations; 4] the voting of
The bottom line is that a lawyer who engages in extra-marital the JBC was railroaded; and 5] the alleged "discretionary"
affairs is deemed to have failed to adhere to the exacting nature of Sections 3 and 4 of JBC-009 is negated by the
standards of morality and decency which every member of the subsequent effectivity of JBC-010, Section 1(2) of which
Judiciary is expected to observe. In fact, even relationships provides for a 10-day period from the publication of the list of
which have never gone physical or intimate could still be candidates within which any complaint or opposition against a
subject to charges of immorality, when a lawyer, who is candidate may be filed with the JBC Secretary; 6] Section 2 of
married, admits to having a relationship which was more than JBC-010 requires complaints and oppositions to be in writing
professional, more than acquaintanceship, more than and under oath, copies of which shall be furnished the
friendly.35 As the Court has held: Immorality has not been candidate in order for him to file his comment within five (5)
confined to sexual matters, but includes conduct days from receipt thereof; and 7] Sections 3 to 6 of JBC-010
inconsistentwith rectitude, or indicative of corruption, prescribe a logical, reasonable and sequential series of steps
indecency, depravity and dissoluteness; or is willful, flagrant, in securing a candidate’s right to due process.
or shameless conduct showing moral indifference to opinions
of respectable members of the communityand an
inconsiderate attitude toward good order and public The JBC counters these by insisting that it is not obliged to
welfare.36 Moral character is not a subjective term but one afford Jardeleza the right to a hearing in the fulfillment of its
that corresponds to objective reality.37 To have a good moral duty to recommend. The JBC, as a body, is not required by
character, a person must have the personal characteristic law to hold hearings on the qualifications of the nominees. The
ofbeing good. It is not enough that he or she has a good process by which an objection is made based on Section 2,
reputation, that is, the opinion generally entertained about a Rule 10 of JBC-009 is not judicial, quasi-judicial, or fact-
person or the estimate in which he or she is held by the public finding, for it does not aim to determine guilt or innocence akin
in the place where she is known.38 Hence, lawyers are at all to a criminal or administrative offense but toascertain the
times subject to the watchful public eye and community fitness of an applicant vis-à-vis the requirements for the
approbation.39 position. Being sui generis, the proceedings of the JBC do not
confer the rights insisted upon by Jardeleza. He may not exact
the application of rules of procedure which are, at the most,
The element of "willingness" to linger in indelicate discretionary or optional. Finally, Jardeleza refused to shed
relationships imputes a weakness in one’s values, self-control light on the objections against him. During the June 30, 2014
and on the whole, sense of honor, not only because it is a bold meeting, he did not address the issues, but instead chose
disregard of the sanctity of marriage and of the law, but totread on his view that the Chief Justice had unjustifiably
because it erodes the public’s confidence in the Judiciary. This become his accuser, prosecutor and judge.
is no longer a matter of an honest lapse in judgment but a
dissolute exhibition of disrespect toward sacredvows taken
before God and the law. The crux of the issue is on the availability of the right to due
process in JBC proceedings. After a tedious review of the
parties’ respective arguments, the Court concludes that the
On the other hand, insider trading is an offense that assaults right to due process is available and thereby demandable asa
the integrity of our vital securities market.40Manipulative matter of right.
FINALS CONSTITUTIONAL LAW I ACJUCO NOV 11, 2017 232

The Court does not brush aside the unique and special nature A more pragmatic take on the matter of due process in JBC
of JBC proceedings. Indeed, they are distinct from criminal proceedings also compels the Court to examine its current
proceedings where the finding of guilt or innocence of the rules. The pleadings of the parties mentioned two: 1] JBC-009
accused is sine qua non. The JBC’s constitutional duty to and 2] JBC-010. The former provides the following provisions
recommend qualified nominees to the President cannot be pertinent to this case:
compared to the duty of the courts of law to determine the
commission of an offense and ascribe the same to an SECTION 1. Evidence of integrity. - The Council shall take
accused, consistent with established rules on evidence. Even every possible step to verify the applicant's record of and
the quantum ofevidence required in criminal cases is far from reputation for honesty, integrity, incorruptibility, irreproachable
the discretion accorded to the JBC. conduct, and fidelity to sound moral and ethical standards. For
this purpose, the applicant shall submit to the Council
The Court, however, could not accept, lock, stock and barrel, certifications or testimonials thereof from reputable
the argument that an applicant’s access tothe rights afforded government officials and non-governmental organizations,
under the due process clause is discretionary on the part of and clearances from the courts, National Bureau of
the JBC. While the facets of criminal42 and administrative43 Investigation, police, and from such other agencies as the
due process are not strictly applicable to JBC proceedings, Council may require.
their peculiarity is insufficient to justify the conclusion that due
process is not demandable. SECTION 2. Background check. - The Council mayorder a
discreet background check on the integrity, reputation and
In JBC proceedings, an aspiring judge or justice justifies his character of the applicant, and receive feedback thereon from
qualifications for the office when he presents proof of his the public, which it shall check or verify to validate the merits
scholastic records, work experience and laudable citations. thereof.
His goal is to establish that he is qualified for the office applied
for. The JBC then takes every possible step to verify an SECTION 3. Testimony of parties.- The Council may receive
applicant's trackrecord for the purpose ofdetermining whether written opposition to an applicant on groundof his moral fitness
or not he is qualified for nomination. It ascertains the factors and, at its discretion, the Council mayreceive the testimony of
which entitle an applicant to become a part of the roster from the oppositor at a hearing conducted for the purpose, with due
which the President appoints. notice to the applicant who shall be allowed to cross-examine
the oppositor and to offer countervailing evidence.
The fact that a proceeding is sui generisand is impressed with
discretion, however, does not automatically denigrate an SECTION 4. Anonymous complaints. - Anonymous
applicant’s entitlement to due process. It is well-established in complaints against an applicant shall not begiven due course,
jurisprudence that disciplinary proceedings against lawyers unless there appears on its face a probable cause sufficient to
are sui generisin that they are neither purely civil nor purely engender belief that the allegations may be true. In the latter
criminal; they involve investigations by the Court into the case, the Council may either direct a discreet investigation or
conduct of one of its officers, not the trial of an action or a require the applicant to comment thereon in writing or during
suit.44 Hence, in the exercise of its disciplinary powers, the the interview. [Emphases Supplied]
Court merely calls upon a member of the Bar to accountfor his
actuations as an officer of the Court with the end in view of
preserving the purity of the legal profession and the proper While the "unanimity rule" invoked against him is found in
and honest administration of justice by purging the profession JBC-009, Jardeleza urges the Court to hold that the
of members who, by their misconduct, have proved subsequent rule, JBC-010,46 squarely applies to his case.
themselves no longer worthy to be entrusted with the duties Entitled asa "Rule to Further Promote Public Awareness of
and responsibilities pertaining to the office of an attorney. In and Accessibility to the Proceedings of the Judicial and Bar
such posture, there can be no occasion to speak of a Council," JBC-010 recognizes the needfor transparency and
complainant or a prosecutor.45 On the whole, disciplinary public awareness of JBC proceedings. In pursuance thereof,
proceedings are actually aimed to verifyand finally determine, JBC-010 was crafted in this wise:
if a lawyer charged is still qualifiedto benefit from the rights
and privileges that membership in the legal profession evoke. SECTION 1. The Judicial and Bar Council shall deliberate to
determine who of the candidates meet prima facie the
Notwithstanding being "a class of itsown," the right to be heard qualifications for the positionunder consideration. For this
and to explain one’s self is availing. The Court subscribes to purpose, it shall prepare a long list of candidates who prima
the view that in cases where an objection to an applicant’s facieappear to have all the qualifications.
qualifications is raised, the observance of due process neither
negates nor renders illusory the fulfillment of the duty of JBC The Secretary of the Council shall then cause to be published
torecommend. This holding is not an encroachment on its in two (2) newspapers of general circulation a notice of the
discretion in the nomination process. Actually, its adherence long list of candidates in alphabetical order.
to the precepts of due process supports and enriches the
exercise of its discretion. When an applicant, who vehemently The notice shall inform the public that any complaint or
denies the truth of the objections, is afforded the chance to opposition against a candidate may be filed with the Secretary
protest, the JBC is presented with a clearer understanding of within ten (10) days thereof.
the situation it faces, thereby guarding the body from making
an unsound and capriciousassessment of information brought
before it. The JBC is not expected to strictly apply the rules of SECTION 2.The complaint or opposition shall be in writing,
evidence in its assessment of an objection against an under oath and in ten (10) legible copies, together with its
applicant. Just the same, to hear the side of the person supporting annexes. It shall strictly relate to the qualifications
challenged complies with the dictates of fairness for the only of the candidate or lack thereof, as provided for in the
test that an exercise of discretion must surmount is that of Constitution, statutes, and the Rules of the Judicial and Bar
soundness. Council, as well as resolutions or regulations promulgated by
it.
FINALS CONSTITUTIONAL LAW I ACJUCO NOV 11, 2017 233

The Secretary of the Council shallfurnish the candidate a copy The JBC’s own rules convince the Court to arrive at this
of the complaint or opposition against him. The candidate shall conclusion. The subsequent issuance of JBC-010
have five (5) days from receipt thereof within which to file his unmistakably projects the JBC’s deference to the grave import
comment to the complaint or opposition, if he so desires. of the right of the applicant to be informed and corollary
thereto, the right to be heard. The provisions of JBC-010, per
SECTION 3.The Judicial and Bar Council shall fix a date when se, provide that: any complaint or opposition against a
it shall meet in executive session to consider the qualification candidate may be filed with the Secretary within ten (10) days
of the long list of candidates and the complaint or opposition thereof; the complaint or opposition shall be in writing, under
against them, if any. The Council may, on its own, conduct a oath and in ten (10) legible copies; the Secretary of the
discreet investigation of the background of the candidates. Council shall furnish the candidate a copy of the complaint or
opposition against him; the candidate shall have five (5) days
from receipt thereof within which to file his comment to the
On the basis of its evaluationof the qualification of the complaint or opposition, if he so desires; and the candidate
candidates, the Council shall prepare the shorter list of can be made to explain the complaint or opposition against
candidates whom it desires to interview for its further him.
consideration.
The Court may not close its eyes to the existence of JBC-010
SECTION 4.The Secretary of the Council shall again cause to which, under the rules of statutory construction,bears great
be published the dates of the interview of candidates in the weight in that: 1] it covers "any" complaint or opposition; 2] it
shorter list in two (2) newspapers of general circulation. It shall employs the mandatory term, "shall"; and 3] most importantly,
likewise be posted in the websites of the Supreme Court and it speaks of the very essence of due process. While JBC-010
the Judicial and Bar Council. does not articulate a procedure that entails a trialtype hearing,
it affords an applicant, who faces "any complaint or
The candidates, as well as their oppositors, shall be opposition," the right to answer the accusations against him.
separately notified of the dateand place of the interview. This constitutes the minimum requirements of due process.

SECTION 5.The interviews shall be conducted in public. Application to Jardeleza’s Case


During the interview, only the members ofthe Council can ask
questions to the candidate. Among other things, the candidate Nearing the ultimate conclusion of this case, the Court is
can be made to explain the complaint or opposition against behooved to rule on whether Jardeleza was deprived of his
him. right to due process in the events leading up to, and during,
the vote on the shortlist last June 30, 2014.
SECTION 6. After the interviews, the Judicial and Bar Council
shall again meet in executive session for the final deliberation The JBC gives great weight and substance to the fact that it
on the short list of candidates which shall be sent to the Office gave Jardeleza the opportunity to answer the allegations
of the President as a basis for the exercise of the Presidential against him. It underscores the fact that Jardeleza was asked
power of appointment. [Emphases supplied] to attend the June 30, 2014 meeting so that he could shed
light on the issues thrown at him. During the said meeting,
Anent the interpretation of these existing rules, the JBC Chief Justice Sereno informed him that in connection with his
contends that Sections 3 and 4, Rule 10 of JBC-009 are candidacy for the position of Associate Justice of the Supreme
merely directory in nature as can be gleaned from the use of Court, the Council would like to propound questions on the
the word "may." Thus, the conduct of a hearing under Rule 4 following issues raised against him: 1] his actuations in
of JBC-009 is permissive and/or discretionary on the part of handling an international arbitration case not compatible with
the JBC. Even the conduct of a hearing to determine the public interest;48 2] reports on his extra-marital affair in SMC;
veracity of an opposition is discretionary for there are ways, and 3] alleged insider trading which led to the "show cause"
besides a hearing, to ascertain the truth or falsity of order from the Philippine Stock Exchange.49
allegations. Succinctly, this argument suggests that the JBC
has the discretion to hold or not to hold a hearing when an As Jardeleza himself admitted, he declined to answer or to
objection to an applicant’s integrity is raised and that it may explain his side, as he would not want to be "lulled into waiving
resort to other means to accomplish its objective. his rights." Instead, he manifested that his statement be put
Nevertheless, JBC adds, "what is mandatory, however, is that on record and informed the Council of the then pendency of
if the JBC, in its discretion, receives a testimony of an his letter-petition with the Court en banc. When Chief Justice
oppositor in a hearing, due notice shall be given to the Sereno informed Jardeleza that the Council would want to
applicant and that shall be allowed to cross-examine the hear from him on the three (3) issues against him,Jardeleza
oppositor."47 Again, the Court neither intends to strip the JBC reasoned out that this was precisely the issue. He found it
of its discretion to recommend nominees nor proposes thatthe irregular that he was not being given the opportunity to be
JBC conduct a full-blown trial when objections to an heard per the JBC rules.He asserted that a candidate must be
application are submitted. Still, it is unsound to say that, all given the opportunity to respond to the charges against him.
together, the observance of due process is a part of JBC’s He urged the Chief Justice to step down from her pedestal and
discretion when an opposition to an application is made of translate the objections in writing. Towards the end of the
record. While it may so rely on "other means" such as meeting, the Chief Justice said that both Jardeleza’s written
character clearances, testimonials, and discreet investigation and oral statements would be made part of the record. After
to aid it in forming a judgment of an applicant’s qualifications, Jardeleza was excused from the conference, Justice Lagman
the Court cannot accept a situation where JBC is given a full suggested that the voting be deferred, but the Chief Justice
rein on the application of a fundamental right whenever a ruled that the Council had already completed the process
person’s integrity is put to question. In such cases, an attack required for the voting to proceed.
on the person of the applicant necessitates his right to explain
himself.
After careful calibration of the case, the Court has reached the
determination that the application of the "unanimity rule" on
FINALS CONSTITUTIONAL LAW I ACJUCO NOV 11, 2017 234

integrity resulted in Jardeleza’s deprivation of his right to due against him and his right to answer the same with
process. vigorouscontention and active participation in the proceedings
which would ultimately decide his aspiration to become a
As threshed out beforehand, due process, as a constitutional magistrate of this Court.
precept, does not always and in all situations require a trial-
type proceeding. Due process is satisfied when a person is Consequences
notified of the charge against him and given an opportunity to
explain or defend himself.50 Even as Jardeleza was verbally To write finisto this controversy and in view of the realistic and
informed of the invocation of Section 2, Rule 10 of JBC-009 practical fruition of the Court’s findings, the Court now
against him and was later asked to explain himself during the declares its position on whether or not Jardeleza may be
meeting, these circumstances still cannot expunge an included in the shortlist, just in time when the period to appoint
immense perplexity that lingers in the mind of the Court. What a member of the Court is about to end.
is to become of the procedure laid down in JBC-010 if the
same would be treated with indifference and disregard? To
repeat, as its wording provides, any complaint or opposition The conclusion of the Court is hinged on the following pivotal
against a candidate may be filed with the Secretary withinten points:
(10) days from the publication of the notice and a list of
candidates. Surely, this notice is all the more conspicuous to 1. There was a misapplication of the "unanimity rule" under
JBC members. Granting ex argumenti, that the 10-day Section 2, Rule 10 of JBC-009 as to Jardeleza’s legal strategy
period51 is only applicable to the public, excluding the JBC in handling a case for the government.
members themselves, this does not discount the fact that the
invocation of the first ground in the June 5, 2014 meeting 2. While Jardeleza’s alleged extra-marital affair and acts of
would have raised procedural issues. To be fair, several insider trading fall within the contemplation of a "question on
members of the Council expressed their concern and desire integrity" and would have warranted the application of the
to hear out Jardeleza but the application of JBC-010 did not "unanimity rule," he was notafforded due process in its
form part of the agenda then. It was only during the next application.
meeting on June 16, 2014, that the Council agreed to invite
Jardeleza, by telephone, to a meeting that would be held on
the same day when a resource person would shed light on the 3. The JBC, as the sole body empowered to evaluate
matter. applications for judicial posts, exercises full discretion on its
power to recommend nomineesto the President. The sui
generischaracter of JBC proceedings, however, is not a
Assuming again that the classified nature of the ground blanket authority to disregard the due process under JBC-010.
impelled the Council to resort to oral notice instead of
furnishing Jardeleza a written opposition, why did the JBC not
take into account its authority to summon Jardeleza in 4. Jardeleza was deprived of his right to due process when,
confidence at an earlier time? Is not the Council empowered contrary to the JBC rules, he was neither formally informed of
to "take every possible step to verify the qualification of the the questions on his integrity nor was provided a reasonable
applicants?" It would not be amiss to state, at this point, that opportunity to prepare his defense.
the confidential legal memorandum used in the invocation
ofthe "unanimity rule" was actually addressed to Jardeleza, in With the foregoing, the Court is compelled to rule that
his capacity as Solicitor General. Safe to assume is his Jardeleza should have been included in the shortlist submitted
knowledge of the privileged nature thereof and the to the President for the vacated position of Associate Justice
consequences of its indiscriminate release to the public. Had Abad. This consequence arose not from the
he been privately informed of the allegations against him unconstitutionality of Section 2, Rule 10 of JBC-009, per se,
based on the document and had he been ordered to respond but from the violation by the JBC of its own rules of procedure
thereto in the same manner, Jardeleza’s right to be informed and the basic tenets of due process. By no means does the
and to explain himself would have been satisfied. Court intend to strike down the "unanimity rule" as it reflects
the JBC’s policy and, therefore, wisdom in its selection of
What precisely set off the protest of lack of due process was nominees. Even so, the Court refuses to turn a blind eye on
the circumstance of requiring Jardeleza to appear before the the palpable defects in its implementation and the ensuing
Council and to instantaneously provide those who are willing treatment that Jardeleza received before the Council. True,
to listen an intelligent defense. Was he given the opportunity Jardeleza has no vested right to a nomination, but this does
to do so? The answer is yes, in the context of his physical not prescind from the fact that the JBC failed to observe the
presence during the meeting. Was he given a reasonable minimum requirements of due process.
chance to muster a defense? No, because he was merely
asked to appear in a meeting where he would be, right then In criminal and administrative cases, the violation of a party’s
and there, subjected to an inquiry. It would all be too well to right to due process raises a serious jurisdictional issue which
remember that the allegations of his extra-marital affair and cannot be glossed over or disregarded at will. Where the
acts of insider trading sprung up only during the June 30, 2014 denial of the fundamental right of due process is apparent, a
meeting. While the said issues became the object of the JBC decision rendered in disregard of that right is void for lack of
discussion on June 16, 2014, Jardeleza was not given the jurisdiction.52 This rule may well be applied to the current
idea that he should prepare to affirm or deny his past behavior. situation for an opposing view submits to an undue relaxation
These circumstances preclude the very idea of due process of the Bill of Rights. To this, the Court shall not concede. Asthe
in which the right to explain oneself is given, not to ensnare by branch of government tasked to guarantee that the protection
surprise, but toprovide the person a reasonable opportunity of due process is available to an individual in proper cases,
and sufficient time to intelligently muster his response. the Court finds the subject shortlist as tainted with a vice that
Otherwise, the occasion becomes anidle and futile exercise. it is assigned to guard against. Indeed, the invocation of
Section 2, Rule 10 of JBC-009 must be deemed to have never
Needless to state, Jardeleza’s grievance is not an imagined come into operation in light of its erroneous application on the
slight but a real rebuff of his right to be informed of the charges original ground against Jardeleza’s integrity. At the risk of
being repetitive, the Court upholds the JBC’s discretion in the
FINALS CONSTITUTIONAL LAW I ACJUCO NOV 11, 2017 235

selection of nominees, but its application of the "unanimity The Court further DIRECTS that the Judicial and Bar Council
rule" must be applied in conjunction with Section 2, Rule 10 of REVIEW, and ADOPT, rules relevant to the observance of
JBC-010 being invoked by Jardeleza. Having been able to due process in its proceedings, particularly JBC-009 and JBC-
secure four (4) out of six (6) votes, the only conclusion left to 010, subject to the approval of the Court.
propound is that a majority of the members of the JBC,
nonetheless, found Jardeleza to be qualified for the position This Decision is immediately EXECUTORY. Immediately
of Associate Justice and this grants him a rightful spot in the notify the Office of the President of this Decision.
shortlist submitted to the President. Need to Revisit JBC’s
SO ORDERED.
Internal Rules

In the Court’s study of the petition,the comments and the


applicable rules of the JBC, the Court is of the view that the
rules leave much to be desired and should be reviewed and
revised. It appears that the provision on the "unanimity rule" is
vagueand unfair and, therefore, can be misused or abused
resulting in the deprivation of an applicant’s right to due
process.

Primarily, the invocation of the "unanimity rule" on integrity is


effectively a veto power over the collective will of a majority.
This should be clarified. Any assertion by a member
aftervoting seems to be unfair because it effectively gives him
or her a veto power over the collective votes of the other
members in view of the unanimous requirement. While an
oppositor-member can recuse himself orherself, still the
probability of annulling the majority vote ofthe Council is quite
high.

Second, integrity as a ground has not been defined. While the


initial impression is that it refers to the moral fiber of a
candidate, it can be, as it has been, used to mean other things.
Infact, the minutes of the JBC meetings n this case reflect the
lack of consensus among the members as to its precise
definition. Not having been defined or described, it is vague,
nebulous and confusing. It must be distinctly specified and
delineated.

Third, it should explicitly provide who can invoke it as a ground


against a candidate. Should it be invoked only by an outsider
as construed by the respondent Executive Secretary or also
by a member?

Fourth, while the JBC vetting proceedings is "sui generis" and


need not be formal or trial type, they must meet the minimum
requirements of due process. As always, an applicant should
be given a reasonable opportunity and time to be heard on the
charges against him or her, if there are any.

At any rate, it is up to the JBC to fine-tune the rules


considering the peculiar nature of its function. It need not be
stressed that the rules to be adopted should be fair,
reasonable, unambiguous and consistent with the minimum
requirements of due process.

One final note.

The Court disclaims that Jardeleza's inclusion in the shortlist


is an endorsement of his appointment as a member of the
Court.1âwphi1 In deference to the Constitution and his
wisdom in the exercise of his appointing power, the President
remains the ultimate judge of a candidate's worthiness.

WHEREFORE, the petition is GRANTED. Accordingly, it is


hereby declared that Solicitor General Francis I-I. Jardeleza is
deemed INCLUDED in the shortlist submitted to the President
for consideration as an Associate Justice of the Supreme
Court vice Associate Justice Roberto A. Abad.
FINALS CONSTITUTIONAL LAW I ACJUCO NOV 11, 2017 236

G.R. No. 211833, April 07, 2015 Rights for Equal Opportunity of Employment. The petitioner
also asserted that the requirement of the Prejudicature
FERDINAND R. VILLANUEVA, PRESIDING JUDGE, MCTC, Program mandated by Section 104 of Republic Act (R.A.) No.
COMPOSTELA-NEW BATAAN, COMPOSTELA VALLEY 85575 should not be merely directory and should be fully
PROVINCE, Petitioner, v. JUDICIAL AND BAR COUNCIL, implemented. He further alleged that he has all the
Respondent. qualifications for the position prescribed by the Constitution
and by Congress, since he has already complied with the
requirement of 10 years of practice of law.
DECISION

REYES, J.:
In compliance with the Court's Resolution6 dated April 22,
Presiding Judge Ferdinand R. Villanueva (petitioner) directly 2014, the JBC7 and the Office of the Solicitor General
came to this Court via a Petition for Prohibition, Mandamus, (OSG)8separately submitted their Comments. Summing up
and Certiorari, and Declaratory Relief1 under Rules 65 and 63 the arguments of the JBC and the OSG, they essentially
of the Rules of Court, respectively, with prayer for the stated that the petition is procedurally infirm and that the
issuance of a temporary restraining order and/or writ of assailed policy does not violate the equal protection and due
preliminary injunction, to assail the policy of the Judicial and process clauses. They posited that: (1) the writ of certiorari
Bar Council (JBC), requiring five years of service as judges of and prohibition cannot issue to prevent the JBC from
first-level courts before they can qualify as applicant to performing its principal function under the Constitution to
second-level courts, on the ground that it is unconstitutional, recommend appointees to the Judiciary because the JBC is
and was issued with grave abuse of discretion.awlibrary not a tribunal exercising judicial or quasi-judicial function; (2)
the remedy of mandamus and declaratory relief will not lie
The Facts because the petitioner has no clear legal right that needs to
be protected; (3) the equal protection clause is not violated
because the classification of lower court judges who have
served at least five years and those who have served less
than five years is valid as it is performance and experience
The petitioner was appointed on September 18, 2012 as the based; and (4) there is no violation of due process as the
Presiding Judge of the Municipal Circuit Trial Court, policy is merely internal in nature.awlibrary
Compostela-New Bataan, Poblacion, Compostela Valley
Province, Region XI, which is a first-level court. On September
27, 2013, he applied for the vacant position of Presiding Judge The Issue
in the following Regional Trial Courts (RTCs): Branch 31,
Tagum City; Branch 13, Davao City; and Branch 6,
Prosperidad, Agusan Del Sur.
The crux of this petition is whether or not the policy of JBC
requiring five years of service as judges of first-level courts
before they can qualify as applicant to second-level courts is
In a letter2 dated December 18, 2013, JBC's Office of constitutional.
Recruitment, Selection and Nomination, informed the
petitioner that he was not included in the list of candidates for Ruling of the Court
the said stations. On the same date, the petitioner sent a
letter, through electronic mail, seeking reconsideration of his Procedural Issues:
non-inclusion in the list of considered applicants and
protesting the inclusion of applicants who did not pass the
prejudicature examination.

Before resolving the substantive issues, the Court considers it


necessary to first determine whether or not the action for
certiorari, prohibition and mandamus, and declaratory relief
The petitioner was informed by the JBC Executive Officer, commenced by the petitioner was proper.
through a letter3 dated February 3, 2014, that his protest and
reconsideration was duly noted by the JBC en banc. However,
its decision not to include his name in the list of applicants was
upheld due to the JBC's long-standing policy of opening the
chance for promotion to second-level courts to, among others, One. The remedies of certiorari and prohibition are tenable.
incumbent judges who have served in their current position for "The present Rules of Court uses two special civil actions for
at least five years, and since the petitioner has been a judge determining and correcting grave abuse of discretion
only for more than a year, he was excluded from the list. This amounting to lack or excess of jurisdiction. These are the
caused the petitioner to take recourse to this Court. special civil actions for certiorari and prohibition, and both are
governed by Rule 65."9 As discussed in the case of Maria
Carolina P. Araullo, etc., et al. v. Benigno Simeon C. Aquino
III, etc., et al.,10 this Court explained that:awlibrary
In his petition, he argued that: (1) the Constitution already
prescribed the qualifications of an RTC judge, and the JBC With respect to the Court, however, the remedies of certiorari
could add no more; (2) the JBC's five-year requirement and prohibition are necessarily broader in scope and reach,
violates the equal protection and due process clauses of the and the writ of certiorari or prohibition may be issued to correct
Constitution; and (3) the JBC's five-year requirement violates errors of jurisdiction committed not only by a tribunal,
the constitutional provision on Social Justice and Human corporation, board or officer exercising judicial, quasi-judicial
FINALS CONSTITUTIONAL LAW I ACJUCO NOV 11, 2017 237

or ministerial functions but also to set right, undo and restrain respondent to perform the act required.13The petitioner bears
any act of grave abuse of discretion amounting to lack or the burden to show that there is such a clear legal right to the
excess of jurisdiction by any branch or instrumentality of the performance of the act, and a corresponding compelling duty
Government, even if the latter does not exercise judicial, on the part of the respondent to perform the act. The remedy
quasi-judicial or ministerial functions. This application is of mandamus, as an extraordinary writ, lies only to compel an
expressly authorized by the text of the second paragraph of officer to perform a ministerial duty, not a discretionary one.14
Section 1, supra. Clearly, the use of discretion and the performance of a
ministerial act are mutually exclusive.

Thus, petitions for certiorari and prohibition are appropriate


remedies to raise constitutional issues and to review and/or The writ of mandamus does not issue to control or review the
prohibit or nullify the acts of legislative and executive exercise of discretion or to compel a course of conduct, which,
officials.11 (Citation omitted) it quickly seems to us, was what the petitioner would have the
JBC do in his favor. The function of the JBC to select and
In this case, it is clear that the JBC does not fall within the recommend nominees for vacant judicial positions is
scope of a tribunal, board, or officer exercising judicial or discretionary, not ministerial. Moreso, the petitioner cannot
quasi-judicial functions. In the process of selecting and claim any legal right to be included in the list of nominees for
screening applicants, the JBC neither acted in any judicial or judicial vacancies. Possession of the constitutional and
quasi-judicial capacity nor assumed unto itself any statutory qualifications for appointment to the judiciary may
performance of judicial or quasi-judicial prerogative. However, not be used to legally demand that one's name be included in
since the formulation of guidelines and criteria, including the the list of candidates for a judicial vacancy. One's inclusion in
policy that the petitioner now assails, is necessary and the list of the candidates depends on the discretion of the JBC,
incidental to the exercise of the JBC's constitutional mandate, thus:awlibrary
a determination must be made on whether the JBC has acted
with grave abuse of discretion amounting to lack or excess of The fact that an individual possesses the constitutional and
jurisdiction in issuing and enforcing the said policy. statutory qualifications for appointment to the Judiciary does
not create an entitlement or expectation that his or her name
be included in the list of candidates for a judicial vacancy. By
submitting an application or accepting a recommendation, one
submits to the authority of the JBC to subject the former to the
Besides, the Court can appropriately take cognizance of this search, screening, and selection process, and to use its
case by virtue of the Court's power of supervision over the discretion in deciding whether or not one should be included
JBC. Jurisprudence provides that the power of supervision is in the list. Indeed, assuming that if one has the legal right to
the power of oversight, or the authority to see that subordinate be included in the list of candidates simply because he or she
officers perform their duties. It ensures that the laws and the possesses the constitutional and statutory qualifications, then
rules governing the conduct of a government entity are the application process would then be reduced to a mere
observed and complied with. Supervising officials see to it that mechanical function of the JBC; and the search, screening,
rules are followed, but they themselves do not lay down such and selection process would not only be unnecessary, but
rules, nor do they have the discretion to modify or replace also improper. However, this is clearly not the constitutional
them. If the rules are not observed, they may order the work intent. One's inclusion in the list of candidates is subject to the
done or redone, but only to conform to such rules. They may discretion of the JBC over the selection of nominees for a
not prescribe their own manner of execution of the act. They particular judicial post. Such candidate's inclusion is not,
have no discretion on this matter except to see to it that the therefore, a legally demandable right, but simply a privilege
rules are followed.12 the conferment of which is subject to the JBC's sound
discretion.

Following this definition, the supervisory authority of the Court


over the JBC is to see to it that the JBC complies with its own Moreover, petitioner is essentially seeking a promotional
rules and procedures. Thus, when the policies of the JBC are appointment, that is, a promotion from a first-level court to a
being attacked, then the Court, through its supervisory second level court. There is no law, however, that grants him
authority over the JBC, has the duty to inquire about the the right to a promotion to second-level courts.15 (Emphasis
matter and ensure that the JBC complies with its own rules. in the original)

Clearly, to be included as an applicant to second-level judge


is not properly compellable by mandamus inasmuch as it
Two. The remedy of mandamus cannot be availed of by the involves the exercise of sound discretion by the JBC.
petitioner in assailing JBC's policy. The petitioner insisted that
mandamus is proper because his right was violated when he
was not included in the list of candidates for the RTC courts
he applied for. He said that his non-inclusion in the list of
candidates for these stations has caused him direct injury. Three. The petition for declaratory relief is improper. "An
action for declaratory relief should be filed by a person
interested under a deed, a will, a contract or other written
instrument, and whose rights are affected by a statute, an
executive order, a regulation or an ordinance. The relief
It is essential to the issuance of a writ of mandamus that the sought under this remedy includes the interpretation and
applicant should have a clear legal right to the thing determination of the validity of the written instrument and the
demanded and it must be the imperative duty of the judicial declaration of the parties' rights or duties
FINALS CONSTITUTIONAL LAW I ACJUCO NOV 11, 2017 238

thereunder."16 "[T]he purpose of the action is to secure an vacancy in the judiciary, subject only to the minimum
authoritative statement of the rights and obligations of the qualifications required by the Constitution and law for every
parties under a statute, deed, contract, etc., for their guidance position. The search for these long held qualities necessarily
in its enforcement or compliance and not to settle issues requires a degree of flexibility in order to determine who is
arising from its alleged breach."17 most fit among the applicants. Thus, the JBC has sufficient
but not unbridled license to act in performing its duties.

In this case, the petition for declaratory relief did not involve
an unsound policy. Rather, the petition specifically sought a JBC's ultimate goal is to recommend nominees and not simply
judicial declaration that the petitioner has the right to be to fill up judicial vacancies in order to promote an effective and
included in the list of applicants although he failed to meet efficient administration of justice. Given this pragmatic
JBC's five-year requirement policy. Again, the Court reiterates situation, the JBC had to establish a set of uniform criteria in
that no person possesses a legal right under the Constitution order to ascertain whether an applicant meets the minimum
to be included in the list of nominees for vacant judicial constitutional qualifications and possesses the qualities
positions. The opportunity of appointment to judicial office is a expected of him and his office. Thus, the adoption of the five-
mere privilege, and not a judicially enforceable right that may year requirement policy applied by JBC to the petitioner's case
be properly claimed by any person. The inclusion in the list of is necessary and incidental to the function conferred by the
candidates, which is one of the incidents of such appointment, Constitution to the JBC.
is not a right either. Thus, the petitioner cannot claim any right
that could have been affected by the assailed policy.

Equal Protection

Furthermore, the instant petition must necessarily fail because


this Court does not have original jurisdiction over a petition for
declaratory relief even if only questions of law are involved.18
The special civil action of declaratory relief falls under the There is no question that JBC employs standards to have a
exclusive jurisdiction of the appropriate RTC pursuant to rational basis to screen applicants who cannot be all
Section 1919 of Batas Pambansa Blg. 129, as amended by accommodated and appointed to a vacancy in the judiciary, to
R.A.No. 7691.20 determine who is best qualified among the applicants, and not
to discriminate against any particular individual or class.

Therefore, by virtue of the Court's supervisory duty over the


JBC and in the exercise of its expanded judicial power, the The equal protection clause of the Constitution does not
Court assumes jurisdiction over the present petition. But in require the universal application of the laws to all persons or
any event, even if the Court will set aside procedural things without distinction; what it requires is simply equality
infirmities, the instant petition should still be among equals as determined according to a valid
dismissed.awlibrary classification. Hence, the Court has affirmed that if a law
neither burdens a fundamental right nor targets a suspect
class, the classification stands as long as it bears a rational
Substantive Issues relationship to some legitimate government
end.21ChanRoblesVirtualawlibrary

As an offspring of the 1987 Constitution, the JBC is mandated


to recommend appointees to the judiciary and only those "The equal protection clause, therefore, does not preclude
nominated by the JBC in a list officially transmitted to the classification of individuals who may be accorded different
President may be appointed by the latter as justice or judge in treatment under the law as long as the classification is
the judiciary. Thus, the JBC is burdened with a great reasonable and not arbitrary."22 "The mere fact that the
responsibility that is imbued with public interest as it legislative classification may result in actual inequality is not
determines the men and women who will sit on the judicial violative of the right to equal protection, for every classification
bench. While the 1987 Constitution has provided the of persons or things for regulation by law produces inequality
qualifications of members of the judiciary, this does not in some degree, but the law is not thereby rendered invalid."23
preclude the JBC from having its own set of rules and
procedures and providing policies to effectively ensure its
mandate.

That is the situation here. In issuing the assailed policy, the


JBC merely exercised its discretion in accordance with the
constitutional requirement and its rules that a member of the
The functions of searching, screening, and selecting are Judiciary must be of proven competence, integrity, probity and
necessary and incidental to the JBC's principal function of independence.24"To ensure the fulfillment of these standards
choosing and recommending nominees for vacancies in the in every member of the Judiciary, the JBC has been tasked to
judiciary for appointment by the President. However, the screen aspiring judges and justices, among others, making
Constitution did not lay down in precise terms the process that certain that the nominees submitted to the President are all
the JBC shall follow in determining applicants' qualifications. qualified and suitably best for appointment. In this way, the
In carrying out its main function, the JBC has the authority to appointing process itself is shielded from the possibility of
set the standards/criteria in choosing its nominees for every
FINALS CONSTITUTIONAL LAW I ACJUCO NOV 11, 2017 239

extending judicial appointment to the undeserving and At any rate, five years of service as a lower court judge is not
mediocre and, more importantly, to the ineligible or the only factor that determines the selection of candidates for
disqualified."25 RTC judge to be appointed by the President. Persons with this
qualification are neither automatically selected nor do they
automatically become nominees. The applicants are chosen
based on an array of factors and are evaluated based on their
individual merits. Thus, it cannot be said that the questioned
Consideration of experience by JBC as one factor in choosing policy was arbitrary, capricious, or made without any basis.
recommended appointees does not constitute a violation of
the equal protection clause. The JBC does not discriminate
when it employs number of years of service to screen and
differentiate applicants from the competition. The number of
years of service provides a relevant basis to determine proven Clearly, the classification created by the challenged policy
competence which may be measured by experience, among satisfies the rational basis test. The foregoing shows that
other factors. The difference in treatment between lower court substantial distinctions do exist between lower court judges
judges who have served at least five years and those who with five year experience and those with less than five years
have served less than five years, on the other hand, was of experience, like the petitioner, and the classification
rationalized by JBC as follows:awlibrary enshrined in the assailed policy is reasonable and relevant to
its legitimate purpose. The Court, thus, rules that the
Formulating policies which streamline the selection process questioned policy does not infringe on the equal protection
falls squarely under the purview of the JBC. No other clause as it is based on reasonable classification intended to
constitutional body is bestowed with the mandate and gauge the proven competence of the applicants. Therefore,
competency to set criteria for applicants that refer to the more the said policy is valid and constitutional.
general categories of probity, integrity and independence.

Due Process
The assailed criterion or consideration for promotion to a
second-level court, which is five years experience as judge of
a first-level court, is a direct adherence to the qualities
prescribed by the Constitution. Placing a premium on many The petitioner averred that the assailed policy violates
years of judicial experience, the JBC is merely applying one procedural due process for lack of publication and non-
of the stringent constitutional standards requiring that a submission to the University of the Philippines Law Center
member of the judiciary be of "proven competence." In Office of the National Administrative Register (ONAR). The
determining competence, the JBC considers, among other petitioner said that the assailed policy will affect all applying
qualifications, experience and performance. judges, thus, the said policy should have been published.

Based on the JBC's collective judgment, those who have been Contrary to the petitioner's contention, the assailed JBC policy
judges of first-level courts for five (5) years are better qualified need not be filed in the ONAR because the publication
for promotion to second-level courts. It deems length of requirement in the ONAR is confined to issuances of
experience as a judge as indicative of conversance with the administrative agencies under the Executive branch of the
law and court procedure. Five years is considered as a government.27 Since the JBC is a body under the supervision
sufficient span of time for one to acquire professional skills for of the Supreme Court,28 it is not covered by the publication
the next level court, declog the dockets, put in place improved requirements of the Administrative Code.
procedures and an efficient case management system, adjust
to the work environment, and gain extensive experience in the
judicial process.

Nevertheless, the assailed JBC policy requiring five years of


service as judges of first-level courts before they can qualify
as applicants to second-level courts should have been
A five-year stint in the Judiciary can also provide evidence of published. As a general rule, publication is indispensable in
the integrity, probity, and independence of judges seeking order that all statutes, including administrative rules that are
promotion. To merit JBC's nomination for their promotion, they intended to enforce or implement existing laws, attain binding
must have had a "record of, and reputation for, honesty, force and effect. There are, however, several exceptions to
integrity, incorruptibility, irreproachable conduct, and fidelity to the requirement of publication, such as interpretative
sound moral and ethical standards." Likewise, their decisions regulations and those merely internal in nature, which regulate
must be reflective of the soundness of their judgment, only the personnel of the administrative agency and not the
courage, rectitude, cold neutrality and strength of character. public. Neither is publication required of the so-called letters
of instructions issued by administrative superiors concerning
the rules or guidelines to be followed by their subordinates in
the performance of their duties.29
Hence, for the purpose of determining whether judges are
worthy of promotion to the next level court, it would be
premature or difficult to assess their merit if they have had less
than one year of service on the bench.26 (Citations omitted Here, the assailed JBC policy does not fall within the
and emphasis in the original) administrative rules and regulations exempted from the
FINALS CONSTITUTIONAL LAW I ACJUCO NOV 11, 2017 240

publication requirement. The assailed policy involves a also required for the five-year requirement because it seeks to
qualification standard by which the JBC shall determine implement a constitutional provision requiring proven
proven competence of an applicant. It is not an internal competence from members of the judiciary.
regulation, because if it were, it would regulate and affect only
the members of the JBC and their staff. Notably, the selection
process involves a call to lawyers who meet the qualifications
in the Constitution and are willing to serve in the Judiciary to
apply to these vacant positions. Thus, it is but a natural Nonetheless, the JBC's failure to publish the assailed policy
consequence thereof that potential applicants be informed of has not prejudiced the petitioner's private interest. At the risk
the requirements to the judicial positions, so that they would of being repetitive, the petitioner has no legal right to be
be able to prepare for and comply with them. included in the list of nominees for judicial vacancies since the
possession of the constitutional and statutory qualifications for
appointment to the Judiciary may not be used to legally
demand that one's name be included in the list of candidates
for a judicial vacancy. One's inclusion in the shortlist is strictly
The Court also noted the fact that in JBC-009, otherwise within the discretion of the JBC.30
known as the Rules of the Judicial and Bar Council, the JBC
had put its criteria in writing and listed the guidelines in
determining competence, independence, integrity and probity.
Section 1, Paragraph 1 of Rule 9 expressly provides that
applicants for the Court of Appeals and the Sandiganbayan, As to the issue that the JBC failed or refused to implement the
should, as a general rule, have at least five years of completion of the prejudicature program as a requirement for
experience as an RTC judge, thus:awlibrary appointment or promotion in the judiciary under R.A. No.
8557, this ground of the petition, being unsubstantiated, was
unfounded. Clearly, it cannot be said that JBC unlawfully
RULE 9 - SPECIAL GUIDELINES FOR NOMINATION TO A neglects the performance of a duty enjoined by law.
VACANCY IN THE COURT OF APPEALS AND
SANDIGANBAYAN

Finally, the petitioner argued but failed to establish that the


assailed policy violates the constitutional provision under
Section 1. Additional criteria for nomination to the Court of social justice and human rights for equal opportunity of
Appeals and the Sandiganbayan. - In addition to the foregoing employment. The OSG explained:awlibrary
guidelines the Council should consider the following in
evaluating the merits of applicants for a vacancy in the Court
of Appeals and Sandiganbayan: [T]he questioned policy does not violate equality of
employment opportunities. The constitutional provision does
not call for appointment to the Judiciary of all who might, for
any number of reasons, wish to apply. As with all professions,
it is regulated by the State. The office of a judge is no ordinary
1. As a general rule, he must have at least five years of office. It is imbued with public interest and is central in the
experience as a judge of Regional Trial Court, except when administration of justice x x x. Applicants who meet the
he has in his favor outstanding credentials, as evidenced by, constitutional and legal qualifications must vie and withstand
inter alia, impressive scholastic or educational record and the competition and rigorous screening and selection process.
performance in the Bar examinations, excellent reputation for They must submit themselves to the selection criteria,
honesty, integrity, probity and independence of mind; at least processes and discretion of respondent JBC, which has the
very satisfactory performance rating for three (3) years constitutional mandate of screening and selecting candidates
preceding the filing of his application for nomination; and whose names will be in the list to be submitted to the
excellent potentials for appellate judgeship. President. So long as a fair opportunity is available for all
applicants who are evaluated on the basis of their individual
merits and abilities, the questioned policy cannot be struck
down as unconstitutional.31 (Citations omitted)
x x x x (Emphasis ours)
From the foregoing, it is apparent that the petitioner has not
established a clear legal right to justify the issuance of a
The express declaration of these guidelines in JBC-009, which preliminary injunction. The petitioner has merely filed an
have been duly published on the website of the JBC and in a application with the JBC for the position of RTC judge, and he
newspaper of general circulation suggests that the JBC is has no clear legal right to be nominated for that office nor to
aware that these are not mere internal rules, but are rules be selected and included in the list to be submitted to the
implementing the Constitution that should be published. Thus, President which is subject to the discretion of the JBC. The
if the JBC were so-minded to add special guidelines for JBC has the power to determine who shall be recommended
determining competence of applicants for RTC judges, then it to the judicial post. To be included in the list of applicants is a
could and should have amended its rules and published the privilege as one can only be chosen under existing criteria
same. This, the JBC did not do as JBC-009 and its imposed by the JBC itself. As such, prospective applicants,
amendatory rule do not have special guidelines for applicants including the petitioner, cannot claim any demandable right to
to the RTC. take part in it if they fail to meet these criteria. Hence, in the
absence of a clear legal right, the issuance of an injunctive
writ is not justified.

Moreover, jurisprudence has held that rules implementing a


statute should be published. Thus, by analogy, publication is
FINALS CONSTITUTIONAL LAW I ACJUCO NOV 11, 2017 241

As the constitutional body granted with the power of searching


for, screening, and selecting applicants relative to
recommending appointees to the Judiciary, the JBC has the
authority to determine how best to perform such constitutional
mandate. Pursuant to this authority, the JBC issues various
policies setting forth the guidelines to be observed in the
evaluation of applicants, and formulates rules and guidelines
in order to ensure that the rules are updated to respond to
existing circumstances. Its discretion is freed from legislative,
executive or judicial intervention to ensure that the JBC is
shielded from any outside pressure and improper influence.
Limiting qualified applicants in this case to those judges with
five years of experience was an exercise of discretion by the
JBC. The potential applicants, however, should have been
informed of the requirements to the judicial positions, so that
they could properly prepare for and comply with them. Hence,
unless there are good and compelling reasons to do so, the
Court will refrain from interfering with the exercise of JBC's
powers, and will respect the initiative and independence
inherent in the latter.cralawred

WHEREFORE, premises considered, the petition is


DISMISSED. The Court, however, DIRECTS that the Judicial
and Bar Council comply with the publication requirement of (1)
the assailed policy requiring five years of experience as
judges of first-level courts before they can qualify as applicant
to the Regional Trial Court, and (2) other special guidelines
that the Judicial and Bar Council is or will be implementing.

SO ORDERED.
FINALS CONSTITUTIONAL LAW I ACJUCO NOV 11, 2017 242

February 21, 2017 Sandiganbayan Associate Justice. The Court quotes the
relevant portions from the Motion, as follows:
G.R. No. 224302
The immediate concern of the JBC is this Court's
HON. PHILIP A. AGUINALDO, HON. REYNALDO A. pronouncement that the former's act of submitting six lists for
ALHAMBRA, HON. DANILO S. CRUZ, HON. BENJAMIN T. six vacancies was unconstitutional. Whether the President
POZON, HON. SALVADOR V. TIMBANG, JR., and the can cross-reach into the lists is not the primary concern of the
INTEGRATED BAR OF THE PHILIPPINES JBC in this particular case. At another time, perhaps, it may
(IBP),Petitioners take a position. But not in this particular situation involving the
newly created positions in the Sandiganbayan in view of the
lack of agreement by the JBC Members on that issue.
vs.
What the President did with the lists, for the purpose of this
HIS EXCELLENCY PRESIDENT BENIGNO SIMEON C. particular dispute alone as far as the JBC is concerned, was
AQUINO III, HON. EXECUTIVE SECRETARY PAQUITO N. the President's exclusive domain.2
OCHOA, HON. MICHAEL FREDERICK L. MUSNGI, HON.
MA. GERALDINE FAITH A. ECONG, HON. DANILO S.
SANDOVAL, HON. WILHELMINA B. JORGE-WAGAN, Nonetheless, the JBC did not categorically withdraw the
HON. ROSANA FE ROMERO-MAGLAYA, HON. arguments raised in its previous Motions, and even reiterated
MERIANTHE PACITA M. ZURAEK, HON. ELMO M. and further discussed said arguments, and raised additional
ALAMEDA, and HON. VICTORIA C. FERNANDEZ- points in its Motion for Reconsideration-in-Intervention.
BERNARDO, Respondents Hence, the Court is still constrained to address said
arguments in this Resolution.
RESOLUTION
In its Motion for Reconsideration (with Motion for Inhibition of
the Ponente) the JBC argues as follows: (a) Its Motion for
LEONARDO-DE CASTRO, J.: Intervention was timely filed on November 26, 2016, three
days before the promulgation of the Decision in the instant
In its Decision dated November 29, 2016, the Court En Banc case; (b) The JBC has a legal interest in this case, and its
held: intervention would not have unduly delayed or prejudiced the
adjudication of the rights of the original parties; (c) Even
WHEREFORE, premises considered, the Court DISMISSES assuming that the Motion for Intervention suffers procedural
the instant Petition for Quo Warranto and Certiorari and infirmities, said Motion should have been granted for a
Prohibition for lack of merit. The Court DECLARES the complete resolution of the case and to afford the JBC due
clustering of nominees by the Judicial and Bar Council process; and (d) Unless its Motion for Intervention is granted
UNCONSTITUTIONAL, and the appointments of respondents by the Court, the JBC is not bound by the questioned Decision
Associate Justices Michael Frederick L. Musngi and because the JBC was neither a party litigant nor impleaded as
Geraldine Faith A. Econg, together with the four other newly- a party in the case, the JBC was deprived of due process, the
appointed Associate Justices of the Sandiganbayan, as assailed Decision is a judgment in personam and not a
VALID. The Court further DENIES the Motion for Intervention judgment in rem, and a decision rendered in violation of a
of the Judicial and Bar Council in the present Petition, but party's right to due process is void for lack of jurisdiction.
ORDERS the Clerk of Court En Banc to docket as a separate
administrative matter the new rules and practices of the On the merits of the case, the JBC asserts that in submitting
Judicial and Bar Council which the Court took cognizance of six short lists for six vacancies, it was only acting in
in the preceding discussion as Item No. 2: the deletion or non- accordance with the clear and unambiguous mandate of
inclusion in JBC No. 2016-1, or the Revised Rules of the Article VIII, Section 93 of the 1987 Constitution for the JBC to
Judicial and Bar Council, of Rule 8, Section 1 of JBC-009; and submit a list for every vacancy. Considering its independence
Item No. 3: the removal of incumbent Senior Associate as a constitutional body, the JBC has the discretion and
Justices of the Supreme Court as consultants of the Judicial wisdom to perform its mandate in any manner as long as it is
and Bar Council, referred to in pages 3 5 to 40 of this Decision. consistent with the Constitution. According to the JBC, its new
The Court finally DIRECTS the Judicial and Bar Council to file practice of "clustering," in fact, is more in accord with the
its comment on said Item Nos. 2 and 3 within thirty (30) days purpose of the JBC to rid the appointment process to the
from notice.1 Judiciary from political pressure as the President has to
choose only from the nominees for one particular vacancy.
I Otherwise, the President can choose whom he pleases, and
thereby completely disregard the purpose for the creation of
the JBC. The JBC clarifies that it numbered the vacancies, not
THE JBC MOTIONS to influence the order of precedence, but for practical reasons,
i.e., to distinguish one list from the others and to avoid
The Judicial and Bar Council (JBC) successively filed a Motion confusion. The JBC also points out that the acts invoked
for Reconsideration (with Motion for the Inhibition of the against the JBC are based on practice or custom, but
Ponente) on December 27, 2016 and a Motion for "practice, no matter how long continued, cannot give rise to
Reconsideration-in-Intervention (Of the Decision dated 29 any vested right." The JBC, as a constitutional body, enjoys
November 2016) on February 6, 2017. independence, and as such, it may change its practice from
time to time in accordance with its wisdom.
At the outset, the Court notes the revelation of the JBC in its
Motion for Reconsideration-in-Intervention that it is not taking Lastly, the JBC moves for the inhibition of the ponente of the
any position in this particular case on President Aquino's assailed Decision based on Canon 3, Section 5 of the New
appointments to the six newly-created positions of Code of Judicial Conduct for Philippine Judiciary.4 The JBC
alleges that the ponente, as consultant of the JBC from 2014
to 2016, had personal knowledge of the voting procedures and
FINALS CONSTITUTIONAL LAW I ACJUCO NOV 11, 2017 243

format of the short lists, which are the subject matters of this should not be taken against the JBC because, the JBC
case. The ponente was even present as consultant during the reasons, Chief Justice Sereno only chairs the JBC, but she is
meeting on October 26, 2015 when the JBC voted upon the not the JBC, and the administration of the oath of office was a
candidates for the six new positions of Associate Justice of purely ministerial act.
the Sandiganbayan created under Republic Act No. 10660.
The JBC then expresses its puzzlement over the ponente 's The JBC likewise disputes the ponente 's observation that
participation in the present proceedings, espousing a position clustering is a totally new practice of the JBC. The JBC avers
contrary to that of the JBC. The JBC questions why it was only that even before Chief Justice Sereno's Chairmanship, the
in her Decision in the instant case did the ponente raise her JBC has generally followed the rule of one short list for every
disagreement with the JBC as to the clustering of nominees vacancy in all first and second level trial courts. The JBC has
for each of the six simultaneous vacancies for Sandiganbayan followed the "one list for every vacancy" rule even for
Associate Justice. The JBC further quoted portions of the appellate courts since 2013. The JBC even recalls that it
assailed Decision that it claims bespoke of the ponente 's submitted on August 17, 2015 to then President Benigno
"already-arrived-at" conclusion as to the alleged ill acts and Simeon C. Aquino III (Aquino) four separate short lists for four
intentions of the JBC. Hence, the JBC submits that such vacancies in the Court of Appeals; and present during the JBC
formed inference will not lend to an even-handed deliberations were the ponente and Supreme Court Associate
consideration by the ponente should she continue to Justice Presbitero J. Velasco, Jr. (Velasco) as consultants,
participate in the case. who neither made any comment on the preparation of the
short lists.
Ultimately, the JBC prays:
On the merits of the Petition, the JBC maintains that it did not
IN VIEW OF THE FOREGOING, it is respectfully prayed that exceed its authority and, in fact, it only faithfully complied with
the DECISION dated 29 November 2016 be reconsidered and the literal language of Article VIII, Section 9 of the 1987
set aside and a new one be issued granting the Motion for Constitution, when it prepared six short lists for the six
Intervention of the JBC. vacancies in the Sandiganbayan. It cites the cases of Atong
Paglaum, Inc. v. Commission on Elections6and Ocampo v.
It is likewise prayed that the ponente inhibit herself from Enriquez,7 wherein the Court allegedly adopted the textualist
further participating in this case and that the JBC be granted approach of constitutional interpretation.
such other reliefs as are just and equitable under the
premises.5 The JBC renounces any duty to increase the chances of
appointment of every candidate it adjudged to have met the
The JBC subsequently filed a Motion for Reconsideration-in- minimum qualifications. It asserts that while there might have
Intervention (Of the Decision dated 29 November 2016), been favorable experiences with the past practice of
praying at the very beginning that it be deemed as sufficient submitting long consolidated short lists, past practices cannot
remedy for the technical deficiency of its Motion for be used as a source of rights and obligations to override the
Intervention (i.e., failure to attach the pleading-in-intervention) duty of the JBC to observe a straightforward application of the
and as Supplemental Motion for Reconsideration of the denial Constitution.
of its Motion for Intervention.
The JBC posits that clustering is a matter of legal and
The JBC, in its latest Motion, insists on its legal interest, injury, operational necessity for the JBC and the only safe standard
and standing to intervene in the present case, as well as on operating procedure for making short lists. It presents different
the timeliness of its Motion for Intervention. scenarios which demonstrate the need for clustering, viz., (a)
There are two different sets of applicants for the vacancies;
(b) There is a change in the JBC composition during the
The JBC proffers several reasons for not immediately seeking interval in the deliberations on the vacancies as the House of
to intervene in the instant case despite admitting that it Representatives and the Senate alternately occupy the ex
received copies of the appointments of the six Sandiganbayan officio seat for the Legislature; (c) The applicant informs the
Associate Justices from the Office of the President (OP) on JBC of his/her preference for assignment in the Cebu Station
January 25, 2016, to wit: (a) Even as its individual Members or Cagayan de Oro Station of the Court of Appeals because
harbored doubts as to the validity of the appointments of of the location or the desire to avoid mingling with certain
respondents Michael Frederick L. Musngi (Musngi) and personalities; (d) The multiple vacancies in newly-opened first
Geraldine Faith A. Econg (Econg) as Sandiganbayan and second level trial courts; and (e) The dockets to be
Associate Justices, the JBC agreed as a body in an executive inherited in the appellate court are overwhelming so the JBC
session that it would stay neutral and not take any legal chooses nominees for those particular posts with more years
position on the constitutionality of said appointments since it of service as against those near retirement.
"did not have any legal interest in the offices of Associate
Justices of the Sandiganbayan"; (b) None of the parties
prayed that the act of clustering by the JBC be declared To the JBC, it seems that the Court was in a hurry to
unconstitutional; and (c) The JBC believed that the Court promulgate its Decision on November 29, 2016, which struck
would apply the doctrine of presumption of regularity in the down the practice of clustering by the JBC. The JBC supposes
discharge by the JBC of its official functions and if the Court that it was in anticipation of the vacancies in the Court as a
would have been inclined to delve into the validity of the act of result of the retirements of Supreme Court Associate Justices
clustering by the JBC, it would order the JBC to comment on Jose P. Perez (Perez) and Arturo D. Brion (Brion) on
the matter. December 14, 2016 and December 29, 2016, respectively.
The JBC then claims that it had no choice but to submit two
separate short lists for said vacancies in the Court because
The JBC impugns the significance accorded by the ponente there were two sets of applicants for the same, i.e., there were
to the fact that Chief Justice Maria Lourdes P. A. Sereno 14 applicants for the seat vacated by Justice Perez and 17
(Sereno), Chairperson of the JBC, administered the oath of applicants for the seat vacated by Justice Brion.
office of respondent Econg as Sandiganbayan Associate
Justice on January 25, 2016. Chief Justice Serena's act
FINALS CONSTITUTIONAL LAW I ACJUCO NOV 11, 2017 244

The JBC further contends that since each vacancy creates case.
discrete and possibly unique situations, there can be no
general rule against clustering. Submitting separate, The Motion for Inhibition of the Ponente filed by the JBC is
independent short lists for each vacancy is the only way for denied.
the JBC to observe the constitutional standards of (a) one list
for every vacancy, and (b) choosing candidates of
competence, independence, probity, and integrity for every The present Motion for Inhibition has failed to comply with
such vacancy. Rule 8, Section 2 of the Internal Rules of the Supreme
Court,10 which requires that "[a] motion for inhibition must be
in writing and under oath and shall state the grounds therefor."
It is also the asseveration of the JBC that it did not encroach Yet, even if technical rules are relaxed herein, there is still no
on the President's power to appoint members of the Judiciary. valid ground for the inhibition of the ponente.
The JBC alleges that its individual Members gave several
reasons why there was an apparent indication of seniority
assignments in the six short lists for the six vacancies for There is no ground11 for the mandatory inhibition of the
Sandiganbayan Associate Justice, particularly: (a) The JBC ponente from the case at bar.
can best perform its job by indicating who are stronger
candidates by giving higher priority to those in the lower- The ponente has absolutely no personal interest in this case.
numbered list; (b) The indication could head off the confusion The ponente is not a counsel, partner, or member of a law firm
encountered in Re: Seniority Among the Four Most Recent that is or was the counsel in the case; the ponente or her
Appointments to the Position of Associate Justices of the spouse, parent, or child has no pecuniary interest in the case;
Court of Appeals;8 and (c) The numbering of the lists from and the ponente is not related to any of the parties in the case
16th to 21st had nothing to do with seniority in the within the sixth degree of consanguinity or affinity, or to an
Sandiganbayan, but was only an ordinal designation of the attorney or any member of a law firm who is counsel of record
cluster to which the candidates were included. in the case within the fourth degree of consanguinity or affinity.

The JBC ends with a reiteration of the need for the ponente to The ponente is also not privy to any proceeding in which the
inhibit herself from the instant case as she appears to harbor JBC discussed and decided to adopt the unprecedented
hostility possibly arising from the termination of her JBC method of clustering the nominees for the six simultaneous
consultancy. vacancies for Sandiganbayan Associate Justice into six
separate short lists, one for every vacancy. The ponente does
The prayer of the JBC in its Motion for Reconsideration-in- not know when, how, and why the JBC adopted the clustering
Intervention reads: method of nomination for appellate courts and even the
Supreme Court.
IN VIEW OF THE FOREGOING, it is respectfully prayed that
JBC's Motion for Reconsideration-in-Intervention, Motion for With due respect to Chief Justice Sereno, it appears that when
Intervention and Motion for Reconsideration with Motion for the JBC would deliberate on highly contentious, sensitive, and
Inhibition of Justice Teresita J. Leonardo-De Castro of the important issues, it was her policy as Chairperson of the JBC
JBC be granted and/or given due course and that: to hold executive sessions, which excluded the Supreme
Court consultants. At the JBC meeting held on October 26,
2015, Chief Justice Sereno immediately mentioned at the
1. the Court's pronouncements in the Decision dated 29 beginning of the deliberations "that, as the Council had always
November 2016 with respect to the JBC's submission of six done in the past when there are multiple vacancies, the voting
shortlists of nominees to the Sandiganbayan be modified to would be on a per vacancy basis."12 Chief Justice Sereno
reflect that the JBC is deemed to have followed Section 9, went on to state that the manner of voting had already been
Article VIII of the Constitution in its practice of submitting one explained to the two ex officio members of the JBC who were
shortlist of nominees for every vacancy, including in not present during the meeting, namely, Senator Aquilino L.
submitting on 28 October 2015 six lists to former President Pimentel III (Pimentel) and then Department of Justice (DOJ)
Benigno Simeon C. Aquino III for the six vacancies of the Secretary ALFREDO BENJAMIN S. Caguioa (Caguioa).13
Sandiganbayan, or for the Court to be completely silent on the Then the JBC immediately proceeded with the voting of
matter; and nominees. This ponente was not consulted before the JBC
decision to cluster nominees was arrived at and, therefore,
2. the Court delete the treatment as a separate administrative she did not have the opportunity to study and submit her
matter of the alleged new rules and practices of the JBC, recommendation to the JBC on the clustering of nominees.
particularly the following: (1) the deletion or non-inclusion of
Rule 8, Section 1 of JBC-009 in JBC No. 2016-1, or the It is evident that prior to the meeting on October 26, 2015, the
Revised Rules of the Judicial and Bar Council; and (2) the JBC had already reached an agreement on the procedure it
removal of incumbent Senior Associate Justices of the would follow in voting for nominees, i.e., the clustering of the
Supreme Court as consultants of the JBC, referred to in pages nominees into six separate short lists, with one short list for
35 to 40 of the Decision. And as a consequence, the Court each of the six newly-created positions of Sandiganbayan
excuse the JBC from filing the required comment on the said Associate Justice. That Senator Pimentel and DOJ Secretary
matters.9 Caguioa, who were not present at the meeting on October 26,
2015, were informed beforehand of the clustering of nominees
II only proves that the JBC had already agreed upon the
clustering of nominees prior to the said meeting.
THE RULING OF THE COURT
Notably, Chief Justice Sereno inaccurately claimed at the very
There is no legal or factual basis for the start of the deliberations that the JBC had been voting on a
per vacancy basis "as the Council had always done," giving
the impression that the JBC was merely following established
ponente to inhibit herself from the instant procedure, when in truth, the clustering of nominees for
FINALS CONSTITUTIONAL LAW I ACJUCO NOV 11, 2017 245

simultaneous or closely successive vacancies in a collegiate evidence. Bare allegations of their partiality will not suffice. It
court was a new practice only adopted by the JBC under her cannot be presumed, especially if weighed against the sacred
Chairmanship. In the Decision dated November 29, 2016, oaths of office of magistrates, requiring them to administer
examples were already cited how, in previous years, the JBC justice fairly and equitably - both to the poor and the rich, the
submitted just one short list for simultaneous or closely weak and the strong, the lonely and the well-connected.
successive vacancies in collegiate courts, including the (Emphasis supplied.)
Supreme Court, which will again be presented hereunder.
Furthermore, it appears from the admitted lack of consensus
As previously mentioned, it is the practice of the JBC to hold on the part of the JBC Members as to the validity of the
executive sessions when taking up sensitive matters. The clustering shows that the conclusion reached by the ponente
ponente and Associate Justice Velasco, incumbent Justices did not arise from personal hostility but from her objective
of the Supreme Court and then JBC consultants, as well as evaluation of the adverse constitutional implications of the
other JBC consultants, were excluded from such executive clustering of the nominees for the vacant posts of
sessions. Consequently, the ponente and Associate Justice Sandiganbayan Associate Justice. It is unfortunate that the
Velasco were unable to participate in and were kept in the JBC stooped so low in casting aspersion on the person of this
dark on JBC proceedings/decisions, particularly, on matters ponente instead of focusing on sound legal arguments to
involving the nomination of candidates for vacancies in the support its position. There is absolutely no factual basis for the
appellate courts and the Supreme Court. The matter of the uncalled for and unfair imputation of the JBC that the ponente
nomination to the Supreme Court of now Supreme Court harbors personal hostility against the JBC presumably due to
Associate Justice FRANCIS H. Jardeleza (Jardeleza), which her removal as consultant. The ponente 's removal as
became the subject matter of Jardeleza v. Sereno,14 was consultant was the decision of Chief Justice Sereno, not the
taken up by the JBC in such an executive session. This JBC. The ponente does not bear any personal grudge or
ponente also does not know when and why the JBC deleted resentment against the JBC for her removal as consultant.
from JBC No. 2016-1, "The Revised Rules of the Judicial and The ponente does not view Chief Justice Sereno's move as
Bar Council," what was Rule 8, Section 1 of JBC-009, the particularly directed against her as Associate Justice Velasco
former JBC Rules, which gave due weight and regard to the had been similarly removed as JBC consultant. The ponente
recommendees of the Supreme Court for vacancies in the has never been influenced by personal motive in deciding
Court. The amendment of the JBC Rules could have been cases. The ponente, instead, perceives the removal of
decided upon by the JBC when the ponente and Associate incumbent Supreme Court Justices as consultants of the JBC
Justice Velasco were already relieved by Chief Justice Sereno as an affront against the Supreme Court itself as an institution,
of their duties as consultants of the JBC. The JBC could have since the evident intention of such move was to keep the
similarly taken up and decided upon the clustering of Supreme Court in the dark on the changes in rules and
nominees for the six vacant posts of Sandiganbayan practices subsequently adopted by the JBC, which, to the
Associate Justice during one of its executive sessions prior to mind of this ponente, may adversely affect the exercise of the
October 26, 2015. supervisory authority over the JBC vested upon the Supreme
Court by the Constitution.
Hence, even though the ponente and the other JBC
consultants were admittedly present during the meeting on All the basic issues raised in the Petition
October 26, 2015, the clustering of the nominees· for the six
simultaneous vacancies for Sandiganbayan Associate Justice had been thoroughly passed upon by the
was already fait accompli. Questions as to why and how the
JBC came to agree on the clustering of nominees were no
longer on the table for discussion during the said meeting. As Court in its Decision dated November 29,
the minutes of the meeting on October 26, 2015 bear out, the
JBC proceedings focused on the voting of nominees. It is 2016 and the JBC already expressed its
stressed that the crucial issue in the present case pertains to
the clustering of nominees and not the nomination and disinterest to question President Aquino's
qualifications of any of the nominees. This ponente only had
the opportunity to express her opinion on the issue of the
clustering of nominees for simultaneous and closely "cross-reaching" in his appointment of the
successive vacancies in collegiate courts in her ponencia in
the instant case. As a Member of the Supreme Court, the six new Sandiganbayan Associate Justices.
ponente is duty-bound to render an opinion on a matter that
has grave constitutional implications.
Even if the Motion for Reconsideration and Motion for
Reconsideration-in-Intervention of the JBC, praying for the
Neither is there any basis for the ponente 's voluntary grant of its Motion for Intervention and the reversal of the
inhibition from the case at bar. Other than the bare allegations Decision dated November 29, 2016, are admitted into the
of the JBC, there is no clear and convincing evidence of the records of this case and the issues raised and arguments
ponente 's purported bias and prejudice, sufficient to adduced in the said two Motions are considered, there is no
overcome the presumption that she had rendered her assailed cogent reason to reverse the Decision dated November 29,
ponencia in the regular performance of her official and sacred 2016, particularly, in view of the admission of the JBC of the
duty of dispensing justice according to law and evidence and lack of unanimity among the JBC members on the issue
without fear or favor. Significant herein is the following involving the clustering of nominees for the six simultaneous
disquisition of the Court on voluntary inhibition of judges in vacancies for Sandiganbayan Associate Justice and their
Gochan v. Gochan,15 which is just as applicable to Supreme disinterest to question the "cross-reaching" or non-
Court Justices: observance by President Aquino of such clustering.

In a string of cases, the Supreme Court has said that bias and Hence, the Court will no longer belabor the issue that only
prejudice, to be considered valid reasons for the voluntary three JBC Members signed the Motion for Intervention and
inhibition of judges, must be proved with clear and convincing Motion for Reconsideration and only four JBC Members
FINALS CONSTITUTIONAL LAW I ACJUCO NOV 11, 2017 246

signed the Motion for Reconsideration-in-Intervention, as well merits of the case fail to persuade the Court to reconsider its
as the fact that Chief Justice Sereno, as Chairperson of the Decision dated November 29, 2016.
JBC, did not sign the three Motions.
a. The clustering of nominees for the
To determine the legal personality of the signatories to file the
JBC Motions, the Court has accorded particular significance six vacancies in the Sandiganbayan
to who among the JBC Members signed the Motions and to
Chief Justice Sereno's act of administering the oath of office
to three of the newly-appointed Sandiganbayan Associate by the JBC impaired the President's
Justices, including respondent Econg, in resolving the
pending Motions of the JBC. However, in its Motion for power to appoint members of the
Reconsideration-in-Intervention, the JBC now reveals that not
all of its Members agree on the official position to take in the Judiciary and to determine the
case of President Aquino's appointment of the six new
Sandiganbayan Associate Justices. Thus, the position of the
JBC on the clustering of the nominees for the six simultaneous seniority of the newly-appointed
vacancies for Sandiganbayan Associate Justice rests on
shaky legal ground. Sandiganbayan Associate Justices.

The JBC takes exception as to why the Court allowed the Noteworthy is the fact that the Court unanimously voted that
Petition at bar even when it did not strictly comply with the in this case of six simultaneous vacancies for Sandiganbayan
rules, as it was filed beyond the 60-day period for filing a Associate Justice, the JBC acted beyond its constitutional
petition for certiorari. The Court, in its Decision dated mandate in clustering the nominees into six separate short
November 29, 2016, gave consideration to petitioners' lists and President Aquino did not commit grave abuse of
assertion that they had to secure first official copies of the six discretion in disregarding the said clustering.
short lists before they were able to confirm that President
Aquino, in appointing the six new Sandiganbayan Associate
The JBC invokes its independence, discretion, and wisdom,
Justices, actually disregarded the clustering of nominees into
and maintains that it deemed it wiser and more in accord with
six separate short lists. While the Court is hard-pressed to
Article VIII, Section 9 of the 1987 Constitution to cluster the
extend the same consideration to the JBC which made no
nominees for the six simultaneous vacancies for
immediate effort to explain its failure to timely question or
Sandiganbayan Associate Justice into six separate short lists.
challenge the appointments of respondents Econg and
The independence and discretion of the JBC, however, is not
Musngi as Sandiganbayan Associate Justices whether before
without limits. It cannot impair the President's power to appoint
the OP or the courts, the Court will nevertheless now allow the
members of the Judiciary and his statutory power to determine
JBC intervention by considering the issues raised and
the seniority of the newly-appointed Sandiganbayan
arguments adduced in the Motion for Reconsideration and
Associate Justices. The Court cannot sustain the strained
Motion for Reconsideration-in-Intervention of the JBC in the
interpretation of Article VIII, Section 9 of the 1987 Constitution
interest of substantial justice.
espoused by the JBC, which ultimately curtailed the
President's appointing power.
Incidentally, it should be mentioned that the JBC reproaches
the Court for supposedly hurrying the promulgation of its
In its Decision dated November 29, 2016, the Court ruled that
Decision on November 29, 2016 in anticipation of the
the clustering impinged upon the President's appointing power
impending vacancies in the Supreme Court due to the
in the following ways: The President's option for every
retirements of Associate Justices Perez and Brion in
vacancy was limited to the five to seven nominees in each
December 2016. On the contrary, it appears that it was the
cluster. Once the President had appointed a nominee from
JBC which hurriedly proceeded with the two separate
one cluster, then he was proscribed from considering the other
publications on August 4, 2016 and August 18, 2016 of the
nominees in the same cluster for the other vacancies. All the
opening of the application for the aforesaid vacancies,
nominees applied for and were found to be qualified for
respectively, which was contrary to previous practice, even
appointment to any of the vacant Associate Justice positions
while the issue of clustering was set to be decided by the
in the Sandiganbayan, but the JBC failed to explain why one
Court. Moreover, a scrutiny of the process the Petition went
nominee should be considered for appointment to the position
through before its promulgation negates any haste on the part
assigned to one specific cluster only. Correspondingly, the
of the Court. Bear in mind that the Petition at bar was filed on
nominees' chance for appointment was restricted to the
May 1 7, 2016 and petitioners' Reply, the last pleading allowed
consideration of the one cluster in which they were included,
by the Court in this case, was filed on August 3, 2016. The
even though they applied and were found to be qualified for
draftponencia was calendared in the agenda of the Supreme
all the vacancies. Moreover, by designating the numerical
Court en bane, called again, and deliberated upon several
order of the vacancies, the JBC established the seniority or
times before it was actually voted upon on November 29,
order of preference of the new Sandiganbayan Associate
2016. Indeed, it appears that it was the JBC which rushed to
Justices, a power which the law (Section 1, paragraph 3 of
release the separate short lists of nominees for the said
Presidential Decree No. 160616), rules (Rule II, Section 1 (b)
Supreme Court vacancies despite knowing the pendency of
of the Revised Internal Rules of the Sandiganbayan17), and
the instant Petition and its own filing of a Motion for
jurisprudence (Re: Seniority Among the Four Most Recent
Intervention herein on November 28, 2016. The JBC went
Appointments to the Position of Associate Justices of the
ahead with the release of separate short lists of nominees for
Court of Appeals18), vest exclusively upon the President.
the posts of Supreme Court Associate Justice vice retired
Associate Justices Perez and Brion on December 2, 2016 and
December 9, 2016, respectively. b. Clustering can be used as a device to

Even if the Court allows the intervention of the JBC, as it will favor or prejudice a qualified
now do in the case at bar, the arguments of the JBC on the
FINALS CONSTITUTIONAL LAW I ACJUCO NOV 11, 2017 247

nominee. then he would have lesser chance of being appointed as he


would have to vie for a single vacancy with two other strong
The JBC avers that it has no duty to increase the chances of contenders; and only one of the three would have been
appointment of every candidate it has adjudged to have met appointed. Evidently, the appointments to the six
the minimum qualifications for a judicial post. The Court does simultaneous vacancies for Sandiganbayan Associate Justice
not impose upon the JBC such duty, it only requires that the would have been different by simply jumbling the clusters of
JBC gives all qualified nominees fair and equal opportunity to nominees. Even if we go back in history, had the JBC
be appointed. The clustering by the JBC of nominees for clustered the nominees for the posts vacated by Supreme
simultaneous or closely successive vacancies in collegiate Court Associate Justices Leonardo A. Quisumbing
courts can actually be a device to favor or prejudice a (Quisumbing) and Minita V. Chico-Nazario (Chico-Nazario),
particular nominee. A favored nominee can be included in a and if Associate Justices Perez and Jose Catral Mendoza
cluster with no other strong contender to ensure his/her (Mendoza) were together in the same cluster, then only one
appointment; or conversely, a nominee can be placed in a of them would have been appointed. Also, had the JBC
cluster with many strong contenders to minimize his/her clustered the nominees for the vacancies resulting from the
chances of appointment. retirements of Supreme Court Associate Justices Antonio
Eduardo B. Nachura (Nachura) and Conchita Carpio Morales
(Carpio Morales), and if Associate Justices Bienvenido L.
Without casting aspersion or insinuating ulterior motive on the Reyes (Reyes) and ESTELA M. Perlas-Bernabe (Perlas-
part of the JBC - which would only be highly speculative on Bernabe) were together in the same cluster, then the
the part of the Court - hereunder are different scenarios, using appointment of one of them would have already excluded the
the very same circumstances and nominees in this case, to other.
illustrate how clustering could be used to favor or prejudice a
particular nominee and subtly influence President Aquino's
appointing power, had President Aquino faithfully observed c. There are no objective criteria,
the clustering.
standards, or guidelines for the
The six nominees actually appointed by President Aquino as
Sandiganbayan Associate Justices were the following: clustering of nominees by the JBC.

VACANCY IN THE SANDIGANBAYAN PERSON The problem is that the JBC has so far failed to present a legal,
APPOINTED SHORT LISTED FOR FORMER objective, and rational basis for determining which nominee
POSITION HELD shall be included in a cluster. Simply saying that it is the result
of the deliberation and voting by the JBC for every vacancy is
16th Associate Justice Michael Frederick L. Musngi unsatisfactory. A review of the voting patterns by the JBC
21st Associate Justice Undersecretary for Members for the six simultaneous vacancies for
Special Concerns/ Chief of Staff of the Executive Secretary, Sandiganbayan Associate Justice only raises more questions
OP, for 5 years and doubts than answers. It would seem, to the casual
observer, that the Chief Justice and the four regular JBC
Members exercised block voting most of the time. Out of the
17th Associate Justice Reynaldo P. Cruz 19th Associate 89 candidates for the six vacancies, there were a total of 3 7
Justice Undersecretary, Office of the Executive Secretary, qualified nominees spread across six separate short lists. Out
OP, for 4-1/2 years of the 37 qualified nominees, the Chief Justice and the four
regular JBC Members coincidentally voted for the same 28
18th Associate Justice Geraldine Faith A. Econg 21st nominees in precisely the same clusters, only varying by just
Associate Justice Former Judge, Regional Trial Court (RTC), one vote for the other nine nominees.
Cebu, for 6 years Chief of Office, Philippine Mediation Center
(PMC) Philippine Judicial Academy (PHILJA) It is also interesting to note that all the nominees were listed
only once in just one cluster, and all the nominees
19th Associate Justice Maria Theresa V. Mendoza- subsequently appointed as Sandiganbayan Associate Justice
Arcega 17th Associate Justice Judge, RTC, Malolos were distributed among the different clusters, except only for
Bulacan, for 10 years respondents Econg and Musngi. Was this by chance or was
there already an agreement among the Chief Justice and the
20th Associate Justice Karl B. Miranda 20th Associate regular JBC Members to limit the nomination of a candidate to
Justice Assistant Solicitor General, Office of the Solicitor a specific cluster for one specific vacancy, thus, excluding the
General (OSG), for 15 years same candidate from again being nominated in a different
cluster for another vacancy? It is understandable that the
Chief Justice and the four regular JBC Members would agree
21st Associate Justice Zaldy V. Trespeses 18th on whom to nominate because their nominations were based
Associate Justice Judicial Staff Head, Office of the Chief on the qualifications of the candidates. What is difficult to
Justice (OCJ), Supreme Court, for 2 years comprehend is how they determined the distribution of the
nominees to the different clusters in the absence of any
It would be safe to say that all the aforementioned six criteria or standard to be observed in the clustering of
nominees were strong contenders. If all six nominees were nominees. This was never explained by the JBC in any of its
placed in the same cluster, then only one of them would have Motions even when the issue of clustering is vital to this case.
been actually appointed as Sandiganbayan Associate Justice Resultantly, the Court also asks why were respondents Econg
and the other five could no longer be considered for the still and Musngi nominated in a single cluster? And why was then
unfilled vacancies. If then Atty. Zaldy V. Trespeses Atty. Trespeses not included in the same cluster as
(Trespeses), Judicial Staff Head, OCJ, was included in the respondents Econg and Musngi, or the clusters of then
cluster with respondent Econg, PHILJA Chief of Office for Undersecretary Reynaldo P. Cruz, RTC Judge Maria Theresa
PMC, and respondent Musngi, Undersecretary for Special V. Mendoza-Arcega, or Assistant Solicitor General Karl B.
Concerns and Chief of Staff of the Executive Secretary, OP, Miranda? Furthermore, what criteria was used when Chief
FINALS CONSTITUTIONAL LAW I ACJUCO NOV 11, 2017 248

Justice Sereno and the other four regular JBC Members voted deliberations on said nominations, the ponente and Supreme
for then Atty. Trespeses for only one particular cluster, i.e., for Court Associate Justice Velasco were both present as JBC
the 18th Sandiganbayan Associate Justice, and nowhere consultants but did not raise any objection.
else? Atty. Trespeses did not receive any vote in the other
clusters except for the lone vote for him of an ex officio JBC While it may be true that the JBC already observed clustering
Member for the vacancy for the 21st Sandiganbayan in 2015, it is still considered a relatively new practice, adopted
Associate Justice. only under Chief Justice Sereno's Chairmanship of the JBC.
The clustering then escaped scrutiny as no party questioned
The Court emphasizes that the requirements and the appointments to the said vacancies. The view of the
qualifications, as well as the powers, duties, and consultants was also not solicited or requested by the JBC.
responsibilities are the same for all vacant posts in a collegiate The Court now observes that the vacancies for Court of
court, such as the Sandiganbayan; and if an individual is found Appeals Associate Justice in 2015 were not all simultaneous
to be qualified for one vacancy, then he/she is found to be or closely successive, most of which occurring months apart,
qualified for all the other vacancies - there are no distinctions specifically, vice the late Associate Justice Michael P. Elbinias
among the vacant posts. It is improbable that the nominees who passed away on November 20, 2014; vice retired
expressed their desire to be appointed to only a specific Associate Justice Rebecca De Guia-Salvador, who opted for
vacant position and not the other vacant positions in the same early retirement effective on January 31, 2015; vice Associate
collegiate court, when neither the Constitution nor the law Justice Hakim S. Abdulwahid, who compulsorily retired on
provides a specific designation or distinctive description for June 12, 2015; and vice Associate Justice Isaias P. Dicdican
each vacant position in the collegiate court. The JBC did not who compulsorily retired on July 4, 2015. Even so, the JBC
cite any cogent reason in its Motion for Reconsideration-in- published a single announcement for all four vacancies on
Intervention for assigning a nominee to a particular March 15, 2015, with the same deadlines for submission of
cluster/vacancy. The Court highlights that without objective applications and supporting documents. This is in stark
criteria, standards, or guidelines in determining which contrast to the two-week interval between the compulsory
nominees are to be included in which cluster, the clustering of retirements of Supreme Court Associate Justices Perez and
nominees for specific vacant posts seems to be at the very Brion on December 14, 2016 and December 29, 2016,
least, totally arbitrary. The lack of such criteria, standards, or respectively, for which the JBC still made separate
guidelines may open the clustering to manipulation to favor or publications, required submission of separate applications,
prejudice a qualified nominee. separately processed the applications, and submitted
separate short lists. Additionally, it is noteworthy that the
d. There is technically no clustering of nominations for the four vacant posts of Court of Appeals
Associate Justice were contained in a single letter dated
August 1 7, 2015, addressed to President Aquino, through
nominees for first and second level then Executive Secretary Paquito N. Ochoa, Jr., whereas in
the case of the Sandiganbayan, the JBC submitted six
trial courts. separate letters, all dated October 26, 2015, transmitting one
short list for each of the six vacancies. The separate letters of
The Court further points out that its Decision dated November transmittal further reinforce the intention of the JBC to prevent
29, 2016 only discussed vacancies in collegiate courts. The the President from "cross-reaching" or disregarding the
constant referral by the JBC to separate short lists of clustering of nominees for the six vacancies for
nominees for vacant judgeship posts in first and second level Sandiganbayan Associate Justice and, thus, unduly limit the
trial courts as proof of previous clustering is inapt. The President's exercise of his power to appoint members of the
separate short lists in such situations are technically not Judiciary.
clustering as the vacancies happened and were announced at
different times and candidates applied for specific vacancies, f. The separate short lists for the
based on the inherent differences in the location and
jurisdiction of the trial courts, as well as the qualifications of current vacancies in the Supreme
nominees to the same, hence, justifying a separate short list
for each vacant post.
Court are not in issue in this case,
e. While clustering of nominees was
but has been brought up by the JBC
observed in the nominations for
in its Motion for Reconsideration-in-
vacancies in the Court of Appeals in
Intervention.
2015, it escaped scrutiny as the
The Court takes the occasion herein to clarify that the
application of its ruling in the Decision dated November 29,
appointments to said vacancies were 2017 to the situation involving closely successive vacancies
in a collegiate court may be properly addressed in an actual
not challenged before the Court. case which squarely raises the issue. It also bears to stress
that the current vacancies in the Supreme Court as a result of
As an example of previous clustering in a collegiate court, the the compulsory retirements of Associate Justices Perez and
JBC attached to its Motion for Reconsideration-in-Intervention Brion are not in issue in this case, but has been brought to the
a transmittal letter dated August 1 7, 2015 of the JBC fore by the JBC itself in its Motion for Reconsideration-in-
addressed to President Aquino, which divided the nominees Intervention. Therefore, the Court will refrain from making any
into four clusters for the four vacancies for Court of Appeals pronouncements on the separate short lists of nominees
Associate Justice. The JBC contends that during the submitted by the JBC to President Rodrigo Roa Duterte
(Duterte) on December 2, 2016 and December 9, 2016 so as
FINALS CONSTITUTIONAL LAW I ACJUCO NOV 11, 2017 249

not to preempt the President's decision on how to treat the The JBC again announced the two vacancies for Supreme
separate short lists of nominees for the two current vacancies Court Associate Justice due to the retirements of Associate
in the Supreme Court. The Court will only address the Justices Nachura and Carpio Morales, thus:
statements made by the JBC in relation to said short lists by
reciting some relevant historical facts relating to the filling-up The Judicial and Bar Council (JBC) announces the opening,
of previous vacancies in the Supreme Court. for application or recommendation, of the following positions:

The JBC avers that it had no choice but to submit separate 1. ASSOCIATE JUSTICE OF THE SUPREME COURT (vice
short lists of nominees to President Duterte for the vacancies Hon. Antonio Eduardo B. Nachura and Hon. Conchita Carpio
for Supreme Court Associate Justice vice Associate Justices Morales, who will compulsorily retire on 13 and 19 June 2011,
Perez and Brion, who retired on December 14, 2016 and respectively);
December 29, 2016, respectively, because there were
different sets of applicants for each, with 14 applicants for the
seat vacated by Associate Justice Perez and 17 applicants for x x xx
the seat vacated by Associate Justice Brion. The situation is
the own doing of the JBC, as the JBC announced the Applications or recommendations for vacancies in nos. 1-3
expected vacancies left by the compulsory retirements of must be filed on or before 28 March 2011 (Monday) x x x to
Associate Justices Perez and Brion, which were merely two the JBC Secretariat, 2nd Flr. Centennial Bldg., Supreme
weeks apart, through two separately paid publications on Court, Padre Faura St., Manila (Tel. No. 552-9512; Fax No.
August 4, 2016 and August 18, 2016, respectively, in 552-9598; email address jbcsupremecourt@yahoo.com.ph.
newspapers of general circulation; invited the filing of separate Those who applied before these vacancies were declared
applications for the vacancies with different deadlines; and open must manifest in writing their interest on or before the
separately processed the applications of candidates to the said deadline. In case of recommendations, the
said vacancies. The JBC would inevitably end up with two recommendees must signify their acceptance either in the
different sets of nominees, one set for the position vacated by recommendation letter itself or in a separate document.
Justice Perez and another set for that vacated by Justice
Brion, notwithstanding that the JBC undeniably found all New applicants or recommendees for positions in the
nominees in both sets to be qualified to be appointed as appellate courts must submit the following on or before 4 April
Associate Justice of the Supreme Court, as they all garnered 2011 (Monday) x x x:
at least four votes.

x x xx
There had been no similar problems in the past because the
JBC jointly announced simultaneous or closely successive
vacancies in the Supreme Court in a single publication, invited The single short list dated June 21, 2011, submitted by the
the filing by a candidate of a single application for all the JBC, under the Chairmanship of Supreme Court Chief Justice
vacancies on the same deadline, jointly processed all Renato C. Corona, presented, for President Aquino's
applications, and submitted a single list of qualified nominees consideration, six nominees for the two vacant posts of
to the President, thus, resulting in a simple, inexpensive, and Supreme Court Associate Justice, with President Aquino
efficient process of nomination. Such was the case when the subsequently appointing Associate Justices Reyes and
JBC announced the two vacancies for Supreme Court Perlas-Bernabe.
Associate Justice following the retirements of Associate
Justices Quisumbing and Chico-Nazario in 2009. Pertinent How the new procedure adopted by the JBC of submitting two
portions of the JBC publication are reproduced below: separate lists of nominees will also affect the seniority of the
two Supreme Court Associate Justices to be appointed to the
The Judicial and Bar Council (JBC) announces the opening, current vacancies is another issue that may arise because of
for application or recommendation, of the: two (2) forthcoming the new JBC procedure. Unlike the present two separate lists
vacant positions of ASSOCIATE JUSTICE OF THE of nominees specifying the vacant post to which they are
SUPREME COURT vice Hon. Leonardo A. Quisumbing and short-listed for appointment, the short list of nominees
Hon. Minita V. Chico-Nazario, who will compulsorily retire on submitted by the JBC before did not identify to which of the
6 November and 5 December 2009, respectively, x x x vacant positions, when there are more than one existing
vacancies, a qualified candidate is nominated to as there was
only one list of nominees for all vacancies submitted to the
Applications or recommendation for the two (2) positions in President. Correspondingly, the appointment papers issued
the Supreme Court must be submitted not later than 28 by the President, as in the cases of Supreme Court Associate
September 2009 (Monday) x x x to the JBC Secretariat, 2nd Justices Perez, Mendoza, Reyes, and Perlas-Bernabe, did
Flr. Centennial Bldg., Supreme Court, Padre Faura St., Manila not specify the particular vacant post to which each of them
(Tel. No. 552-9512; Fax No. 552-9607; email address jbc was appointed. The appointment papers of the afore-named
_supreme court@yahoo.com.ph or jbc@sc.judiciary.gov.ph). Supreme Court Associate Justices were all similarly worded
Applicants or recommendees must submit six (6) copies of the as follows:
following:

Pursuant to the provisions of existing laws, you are hereby


x x xx appointed ASSOCIATE JUSTICE OF THE SUPREME
COURT.
The JBC, then headed by Supreme Court Chief Justice
Reynato S. Puno, submitted to President Gloria Macapagal- By virtue hereof, you may qualify and enter upon the
Arroyo (Macapagal-Arroyo) a single short list dated November performance of the duties and functions of the office,
29, 2009 with a total of six nominees for the two vacancies for furnishing this Office and the Civil Service Commission with
Supreme Court Associate Justice, from which, President copies of your Oath of Office.
Macapagal-Arroyo appointed Associate Justices Perez and
Mendoza.
FINALS CONSTITUTIONAL LAW I ACJUCO NOV 11, 2017 250

As earlier stated, the Court makes no ruling on the above- short lists can rid the appointment process to the Judiciary of
mentioned divergence between the procedures in the political pressure; or conversely, how the previous practice of
nomination for existing vacancies in the Supreme Court submitting a single list of nominees to the President for
followed by the JBC before and by the present JBC as it may simultaneous vacancies in collegiate courts, requiring the
be premature to do so and may prejudge whatever action same qualifications, made the appointment process more
President Duterte may take on the two separate short lists of susceptible to political pressure. The 1987 Constitution itself,
nominees for the current Supreme Court vacancies which by creating the JBC and requiring that the President can only
were submitted by the JBC. appoint judges and Justices from the nominees submitted by
the JBC, already sets in place the mechanism to protect the
g. The designation by the JBC of appointment process from political pressure. By arbitrarily
clustering the nominees for appointment to the six
simultaneous vacancies for Sandiganbayan Associate Justice
numbers to the vacant into separate short lists, the JBC influenced the appointment
process and encroached on the President's power to appoint
Sandiganbayan Associate Justice members of the Judiciary and determine seniority in the said
court, beyond its mandate under the 1987 Constitution. As the
posts encroached on the President's Court pronounced in its Decision dated November 29, 2016,
the power to recommend of the JBC cannot be used to restrict
or limit the President's power to appoint as the latter's
power to determine the seniority of prerogative to choose someone whom he/she considers worth
appointing to the vacancy in the Judiciary is still paramount.
the justices appointed to the said court. As long as in the end, the President appoints someone
nominated by the JBC, the appointment is valid, and he, not
the JBC, determines the seniority of appointees to a collegiate
The JBC contends in its Motion for Reconsideration-in-
court.
Intervention that its individual members have different reasons
for designating numbers to the vacant Sandiganbayan
Associate Justice posts. The varying reason/s of each Finally, the JBC maintains that it is not bound by the Decision
individual JBC Members raises the concern whether they dated November 29, 2016 of the Court in this case on the
each fully appreciated the constitutional and legal ground that it is not a party herein. The JBC prays in its Motion
consequences of their act, i.e., that it encroached on the for Reconsideration and Motion for Reconsideration-in-
power, solely vested in the President, to determine the Intervention, among other reliefs and remedies, for the Court
seniority of the justices appointed to a collegiate court. Each to reverse its ruling in the Decision dated November 29, 2016
of the six short lists submitted by the JBC to President Aquino denying the Motion for Intervention of the JBC in the present
explicitly stated that the nominees were for the Sixteenth case. However, the Court has now practically allowed the
(16th), Seventeenth (17th), Eighteenth (18th), Nineteenth intervention of the JBC in this case, by taking into
(19th), Twentieth (20th), and Twenty-First (2 Pt) consideration the issues raised and arguments adduced in its
Sandiganbayan Associate Justice, respectively; and on the Motion for Reconsideration and Motion for Reconsideration-
faces of said short lists, it could only mean that President in-Intervention, but which the Court found to be unmeritorious.
Aquino was to make the appointments in the order of seniority
pre-determined by the JBC, and that nominees who applied To recapitulate, the Petition at bar challenged President
for any of the vacant positions, requiring the same Aquino's appointment of respondents Econg and Musngi as
qualifications, were deemed to be qualified to be considered Sandiganbayan Associate Justices, which disregarded the
for appointment only to the one vacant position to which clustering by the JBC of the nominees for the six simultaneous
his/her cluster was specifically assigned. Whatever the vacancies in said collegiate court into six separate short lists.
intentions of the individual JBC Members were, they cannot The Court ultimately decreed in its Decision dated November
go against what has been clearly established by law,19 29, 2016 that:
rules,20 and jurisprudence.21 In its Decision dated November
29, 2016, the Court already adjudged that:
President Aquino validly exercised his discretionary power to
appoint members of the Judiciary when he disregarded the
Evidently, based on law, rules, and jurisprudence, the clustering of nominees into six separate shortlists for the
numerical order of the Sandiganbayan Associate Justices vacancies for the 16th, 17th, 18th, 19th, 20th, and 21st
cannot be determined until their actual appointment by the Sandiganbayan Associate Justices. President Aquino merely
President. maintained the well-established practice, consistent with the
paramount Presidential constitutional prerogative, to appoint
It also bears to point out that part of the President's power to the six new Sandiganbayan Associate Justices from the 37
appoint members of a collegiate court, such as the qualified nominees, as if embodied in one JBC list. This does
Sandiganbayan, is the power to determine the seniority or not violate Article VIII, Section 9 of the 1987 Constitution
order of preference of such newly appointed members by which requires the President to appoint from a list of at least
controlling the date and order of issuance of said members' three nominees submitted by the JBC for every vacancy. To
appointment or commission papers. By already designating meet the minimum requirement under said constitutional
the numerical order of the vacancies, the JBC would be provision of three nominees per vacancy, there should at least
establishing the seniority or order of preference of the new be 18 nominees from the JBC for the six vacancies for
Sandiganbayan Associate Justices even before their Sandiganbayan Associate Justice; but the minimum
appointment by the President and, thus, unduly arrogating requirement was even exceeded herein because the JBC
unto itself a vital part of the President's power of submitted for the President's consideration a total of 37
appointment.22 qualified nominees. All the six newly appointed
Sandiganbayan Associate Justices met the requirement of
nomination by the JBC under Article VIII, Section 9 of the 1987
It is also not clear to the Court how, as the JBC avowed in its
Constitution. Hence, the appointments of respondents Musngi
Motion for Reconsideration, the clustering of nominees for
and Econg, as well as the other four new Sandiganbayan
simultaneous vacancies in collegiate courts into separate
FINALS CONSTITUTIONAL LAW I ACJUCO NOV 11, 2017 251

Associate Justices, are valid and do not suffer from any


constitutional infirmity.23

The declaration of the Court that the clustering of nominees


by the JBC for the simultaneous vacancies that occurred by
the creation of six new positions of Associate Justice of the
Sandiganbayan is unconstitutional was only incidental to its
ruling that President Aquino is not bound by such clustering in
making his appointments to the vacant Sandiganbayan
Associate Justice posts. Other than said declaration, the Court
did not require the JBC to do or to refrain from doing
something insofar as the issue of clustering of the nominees
to the then six vacant posts of Sandiganbayan Associate
Justice was concerned.

As for the other new rules and practices adopted by the JBC
which the Court has taken cognizance of and docketed as a
separate administrative matter (viz., Item No. 2: the deletion
or non-inclusion in JBC No. 2016-1, or the Revised Rules of
the Judicial and Bar Council, of Rule 8, Section 1 of JBC-009;
and Item No. 3: the removal of incumbent Senior Associate
Justices of the Supreme Court as consultants of the Judicial
and Bar Council, referred to in pages 45 to 51 of the Decision
dated November 29, 2016), the JBC is actually being given
the opportunity to submit its comment and be heard on the
same. The administrative matter was already raffled to
another ponente, thus, any incident concerning the same
should be consolidated in the said administrative matter.

Regarding the Separate Opinion of Associate Justice


Caguioa, it must be pointed out that he has conceded that the
President did not commit an unconstitutional act in
"disregarding the clustering done by the JBC" when he chose
Associate Justices of the Sandiganbayan "outside" of the
"clustered" lists provided by the JBC.

WHEREFORE, premises considered, except for its


motion/prayer for intervention, which the Court has now
granted, the Motion for Reconsideration (with Motion for the
Inhibition of the Ponente) and the Motion for Reconsideration-
in-Intervention (Of the Decision dated 29 November 2016) of
the Judicial and Bar Council are DENIED for lack of merit.

Nota bene: The Court has agreed not to issue a ruling herein
on the separate short lists of nominees submitted by the
Judicial and Bar Council to President Rodrigo Roa Duterte for
the present vacancies in the Supreme Court resulting from the
compulsory retirements of Associate Justices Jose P. Perez
and Arturo D. Brion because these were not in issue nor
deliberated upon in this case, and in order not to preempt the
decision the President may take on the said separate short
lists in the exercise of his power to appoint members of the
Judiciary under the Constitution.

SO ORDERED.

You might also like